SOLUTIONS MANUAL FOR Introduction to Management Accounting 17th Edition. (Global Edition)

Page 1


Introduction to Management Accounting 17e (Global Edition) By Horngren Sundem Stratton Burgstahler Schatzberg (Solutions Manual All Chapters, 100% Original Verified, A+ Grade) CHAPTER 1 COVERAGE OF LEARNING OBJECTIVES

LEARNING OBJECTIVE LO1: Explain why accounting is essential for decision makers and managers. LO2: Describe the major users and uses of accounting information. LO3: Explain the role of budgets and performance reports in planning and control. LO4: Describe the costbenefit and behavioral issues involved in designing an accounting system. LO5: Discuss the role accountants play in the company’s value chain functions. LO6: Identify current trends in management accounting. LO7: Explain why ethics and standards of ethical conduct are important to accountants.

..

FUNDAMENTAL ASSIGNMENT MATERIAL B3

ADDITIONAL ASSIGNMENT MATERIAL 30, 30, 31, 31, 39

A1, B1, B3

28, 29, 33, 39, 39, 40, 40, 42, 42 5, 5, 6, 6, 32, 32, 45, 45

A2, B2, B3

CASES, EXCEL, COLLAB., & INTERNET EXERCISES 52, 55

55 53

41, 43

A1, B1, B3

B3 A3, B3

17, 30, 30, 31, 31, 34, 34, 35, 36, 39, 42, 42, 44, 45, 46, 46 24 37, 38, 38, 47, 48, 48, 49

54 51, 52, 55

1


CHAPTER 1 Managerial Accounting, the Business Organization, and Professional Ethics 1-A1 (10-15 min.) Information is often useful for more than one function, so the following classifications for each activity are not definitive but serve as a starting point for discussion: 1. Scorekeeping. A depreciation schedule is used in preparing financial statements to report the results of activities. 2. Problem solving. Helps a manager assess the impact of a purchase decision. 3. Scorekeeping. Reports on the results of an operation. Could also be attention directing if scrap is an area that might require management attention. 4. Attention directing. Focuses attention on areas that need attention. 5. Attention directing. Helps managers learn about the information contained in a performance report. 6. Scorekeeping. The statement reports what has happened. Could also be attention directing if the report highlights a problem or issue. 7. Problem solving. Assuming the cost comparison is to help the manager decide between two alternatives, this is problem solving. 8. Attention directing. Variances point out areas where results differ from expectations. Interpreting them directs attention to possible causes of the differences. 9. Problem solving. Aids a decision about where to make parts. 10. Attention directing and problem solving. Budgeting involves making decisions about planned activities -- hence, aiding problem solving. Budgets also direct attention to areas of opportunity or concern -- hence, directing attention. Reporting against the budget also has a scorekeeping dimension.

..

2


1-A2 (15-20 min.) 1. Room rental Food Entertainment Decorations Total 2.

Budgeted Amounts $ 170 660 570 210 $1,610

Actual Amounts $ 170 875 570 270 $1,885

Deviations or Variances $ 0 215U 0 60U $275U

Because of the management by exception rule, room rental and entertainment require no explanation. The actual expenditure for food exceeded the budget by $215. Of this $215, $132 is explained by attendance of 16 persons more than budgeted (the budget of $660/80 = $8.25 per person for food and 16 x $8.25 = $132) and the remaining $83 ($215 – $132) is explained by expenditures above $8.25 per person. Actual expenditures for decorations were $60 more than the budget. The decorations committee should be asked for an explanation of the excess expenditures.

1-A3 (10 min.) All of the situations raise possibilities for violation of the integrity standard. In addition, the manager in each situation must address an additional ethical standard: 1. 2.

3.

..

The General Mills manager must respect the confidentiality standard. He or she should not disclose any information about the new cereal. Felix must address his level of competence for the assignment. If his supervisor knows his level of expertise and wants an analysis from a “layperson” point of view, he should do it. However, if the supervisor expects an expert analysis, Felix must disclose his lack of competence. The credibility standard should cause Mary Sue to decline to omit the information from the budget. It is relevant information, and its omission may mislead readers of the budget.

3


1-B1

(15-20 min.)

Information is often useful for more than one function, so the following classifications for each activity are not definitive but serve as a starting point for discussion: 1. Problem solving. Provides information for deciding between two alternative courses of action. 2. Scorekeeping. Recording what has happened. If amounts are compared with expectations, this could also serve an attention-directing function. 3. Problem solving. Helps a manager decide among alternatives. 4. Attention directing. Directs attention to the use of overtime labor. Also scorekeeping. 5. Problem solving. Provides information to managers for deciding whether to move corporate headquarters. 6. Attention directing. Directs attention to why nursing costs increased. 7. Attention directing. Directs attention to areas where actual results differed from the budget. 8. Problem solving. Helps the vice-president decide which course of action is best. 9. Problem solving. Produces information to help the marketing department make a decision about a marketing campaign. 10. Scorekeeping. Records actual overtime costs. If results are compared with expectations, also attention directing. 11. Attention directing. Directs attention to stores with either high or low ratios of advertising expenses to sales. 12. Attention directing. Directs attention to causes of returns of the drug. 13. Attention directing or problem solving, depending on the use of the schedule. If it is to identify areas of high fuel usage it is attention directing. If it is to plan for purchases of fuel, it is problem solving. 14. Scorekeeping. Records items needed for financial statements.

..

4


1-B2

(10-15 min.)

1 & 2. Sales Costs: Fireworks Labor Other Total cost Profit

Budget $80,000

Actual $79,440

Variance $ 560U

$40,000 10,000 7,000 57,000 $23,000

$39,400 13,100 6,900 59,400 $20,040

$600F 3,100U 100F 2,400U $2,960U

3.

The cost of fireworks was $600 ÷ $40,000 = 1.5% under budget while sales were just $560 ÷ $80,000 = .7% under budget. Did fireworks suppliers lower their prices? Were selling prices set higher than expected? There should be some explanation for the lower cost of fireworks. The labor cost was $3,100 ÷ $10,000 =31% over budget. Sales and other costs were close to budget in percentage terms. Why was labor cost so much higher than expected?

1-B3

(15 - 20 min.)

1.

A code of conduct is a document specifying the ethical standards of an organization .

2.

Different companies include different elements in their codes of conduct. Some of the items included in companies’ codes of conduct include maintaining a dress code, avoiding illegal drugs, following instructions of superiors, being reliable and prompt, maintaining confidentiality, not accepting personal gifts from stakeholders as a result of company role, avoiding racial or sexual discrimination, avoiding conflict of interest, complying with laws and regulations, not using organization’s property for personal use, and reporting illegal or questionable activity. Some companies have a simple code with little detail, and others have long lists of rules and regulations regarding appropriate conduct. The key is that the code of conduct must fit with the corporate culture.

3.

Simply having a code of conduct does not guarantee ethical behavior by employees. Most important is top management’s ethical example and its support of the code of conduct. A company’s performance evaluation and reward system must be consistent with its code of conduct. If unethical actions are rewarded, they will be encouraged even if they violate the code of conduct.

..

5


1-1 a. b.

Internal managers and external parties use accounting information: Internal reporting is used by managers for planning and controlling operations, special decision-making, and long-range planning. External reporting is used by stockholders, investors, taxing authorities, government regulators, and other interested parties.

1-2 The emphasis of financial accounting has traditionally been on the historical data presented in the external reports. Management accounting is more future-oriented and emphasizes planning, control, and decision-making. 1-3

The branch of accounting described in the quotation is management accounting.

1-4 Scorekeeping is the recording (including accumulation and classification) of data for a later evaluation of performance. Attention directing is the reporting and interpretation of information for the purpose of focusing on inefficiencies of operation, opportunities for improvement, and imperfections and operating problems. Problem solving is analysis of alternative courses of action to evaluate the best course of action. 1-5

The three categories are: a. The financial reports prepared by public companies for external users must adhere to a set of standards known as generally accepted accounting principles (GAAP). b. Every company is subject to various reporting requirements as specified by the tax rules and regulations of the host country. c. Many companies are subject to various other government regulations relating to corporate governance (e.g. Sarbanes-Oxley Act in the United States), prevention of fraud, etc.

1-6 Internal auditors review and evaluate a company’s accounting system as well as the adequacy of the internal accounting control system. They also conduct the management audit, which is a review to determine whether the policies and procedures specified by the top management are actually implemented or not. 1-7 Many managers believe that the costs of applying the provisions of the SarbanesOxley act are greater then the benefits. This is especially true about the mandated auditing of companies’ internal control systems. 1-8 Users cannot easily observe the quality of accounting information. Thus, they rely on the integrity of accountants to be sure the information is accurate. If accountants do not have a reputation for integrity, the information they produce will not have value to users. 1-9 No. The ethics developed as a student carry over into one’s professional life. Integrity is important at all stages of development. Students who use unethical means to achieve success are likely to try similar methods when in business. ..

6


1-10 Public accounting firms, law firms, management consultants, real estate firms, transportation companies, banks, insurance companies, and hotels are examples of service organizations. Service organizations tend to be labor intensive, have outputs that are difficult to define and measure, and have both inputs and outputs that are difficult or impossible to store. 1-11 Two considerations are cost-benefit balance and behavioral effects. Cost-benefit balance refers to how well an accounting system helps achieve management's goals in relation to the cost of the system. The behavioral-effects consideration specifies that an accounting system should be judged by how it will affect the behavior (that is, decisions) of managers. 1-12 Yes. Measurement and recording is an integral part of management. For example, cash receipts and disbursements must be traced, and receivables and payables must be recorded in order to manage operating activities such as sales and purchases. 1-13 A budget is a quantitative expression of a plan of action; a performance report compares actual results with the budget; and a variance measures the differences between budget and actual. 1-14 No. Management by exception means that management directs more attention to those areas that seem to be out of control and less to areas that are functioning as planned. This method is an efficient way for managers to decide where to put their time and effort. 1-15 Information that is relevant for decisions about a product depends on the product's life-cycle stage. Therefore, to prepare and interpret information, accountants should be aware of the current stage of a product's life cycle. 1-16 The six functions are: (1) research and development – generation and experimentation with new ideas for products, services, or processes; (2) product, service, and process design – detailed design and engineering of products, services, or processes; (3) production – use of resources to produce a product or service; (4) marketing informing customers of the value and features of products or services; (5) distribution – delivering products or services to customers; and (6) customer service – support provided to customers after a sale. 1-17 Accountants play a vital role in a company’s post production value chain functions. They perform the cost-benefit analysis of a proposed marketing program and help managers in choosing a distribution channel, such as selling directly through own retail chain or through wholesalers and retailers. They also assist the managers in selecting the most economical mode of transportation by providing the cost data of the available choices. Accountants also provide relevant cost data associated with customer warranty, cost of repair, and cost of goods returned, thereby helping the managers to

..

7


compare the same against the benefits generated by customer services to evaluate the effectiveness of such service programs. 1-18 Line managers are directly responsible for the production and sale of goods or services. Staff managers have an advisory function – they support line managers. 1-19 Management accountants are the information specialists. In non-hierarchical companies, they are more directly involved with managers and are often parts of crossfunctional teams. 1- 20 A treasurer is concerned mainly with the company's financial matters, the controller with operating matters. In large organizations, there are sufficient activities associated with both financial and operating matters to justify two separate positions. In a small organization the same person might be both treasurer and controller. 1-21 The two parts of the CMA examination are: (1) financial planning, performance, and control, and (2) financial decision making. 1-22 This is not true. About one-third of CEOs in companies with revenues greater than $500 million come from finance or accounting backgrounds. Accounting is excellent preparation for top management positions because accountants are often exposed to many parts of the company early in their careers. 1-23 Changes in technology are affecting how accountants operate. Increasing computing capabilities and decreasing computing costs have changed how accountants gather, store, manipulate, and report data. Today accountants must be able to account for transactions efficiently and safely, integrate their accounting systems into ERP systems, and use XBRL to communicate information electronically. 1-24 Lean manufacturing is a management approach focused on efficiency. Lean manufacturing applies continuous process improvements to eliminate waste from an enterprise. For example, some companies can significantly reduce the production time by replacing conveyor belts with clusters of robots. Successful lean manufacturing can make the manufacturing process significantly more effective and subsequently enhance the company’s profitability. 1-25 Moving tools and products that are in process from one location to another in a plant is an activity that does not add value to the product. So changing the plant layout to eliminate wasted movement and time improves production efficiency. 1-26 The four major responsibilities are: (1) competence - develop knowledge; know and obey laws, regulations, and technical standards; and perform appropriate analyses, (2) confidentiality - refrain from disclosing or using confidential information, (3) integrity avoid conflicts of interest, refuse gifts that might influence actions, recognize limitations,

..

8


and avoid activities that might discredit the profession, and (4) credibility - communicate information fairly, objectively, and completely, within confidentiality constraints. 1-27 Standards do not always provide the needed guidance. Sometimes an action borders on being unethical, but it is not clearly a violation of an ethical standard. Other times two ethical standards conflict. In situations such as these, accountants must make ethical judgments. 1-28

(5-10 min.)

Typical activities associated with the treasurer function include:  Provision of capital  Investor relations  Short-term financing  Banking and custody  Credit management and collections of cash  Investments  Risk management Typical activities associated with the controller function include:  Planning for control  Reporting and interpreting  Evaluating and consulting  Tax administration  Government reporting  Protection of assets  Economic appraisal 1-29

(10 - 15 min.)

1.

Controller. Financial statements are generally produced by the controller's department. Controller. Advising managers aids operating decisions. Controller. Advice on cost analysis aids managers' operating decisions. Treasurer. Analysts affect the company's ability to raise capital, which is the responsibility of the treasurer. Treasurer. Financing the business is the responsibility of the treasurer. Controller. Tax returns are part of the accounting process overseen by the controller. Treasurer. Insurance, as with other risk management activities, is usually the responsibility of the treasurer. Treasurer. Allowing credit is a financial decision.

2. 3. 4. 5. 6. 7. 8.

..

9


1-30 (5-10 min.) Activities 2, 3, 5, and possibly 6, are primarily associated with marketing decisions. The management accountant would assist in these decisions as follows: Blue Star’s pricing decision requires the cost data relevant to the new method of distributing spare parts. Walmart needs to calculate the cost of the advertising program as well as the additional costs of other value chain functions resulting from increased sales. McDonald’s must determine the incremental cost and revenue associated with the customization order. Symphony needs to study the impact of closing one of its stores on both its revenues and costs. 1-31 (5-10 min.) Activities 1, 7, and 8 are primarily associated with production decisions. The management accountant would assist in these decisions as follows: Hyundai Motors needs an analysis of the costs associated with purchasing said part compared to the costs of producing the part in-house. ASUS must draw up the cost of the training program and project the savings resulting from increased efficiency in the setup and changeover activities. Octopus Energy needs to know the costs and salvage values of the replacement equipment, the proceeds of the sale of the old equipment, and the operating savings associated with the use of the new equipment. 1-32

(5 min.) 1. Financial 2. Management 3. Management

..

4. Management 5. Financial 6. Management

7. Management

10


1-33

(10 min.)

1. Performance Report Revenues Advertising cost Net

Budget $356,400

Actual $351,400

Variance $5,000 U

33,000

35,640

2,640 U

Explanation Additional sales below budget* New advertising campaign

$7,640 U

* From the New Products Report, seven new products were added which exceeded the plan to add six. However, the increase in sales was $5,000 less than budgeted 2. Factors that may not have been considered include: a. Raw material costs for new products may have been higher than budgeted. b. Customer satisfaction with new products may have been low, resulting in unanticipated costs of replacement products given to dissatisfied customers. c. External uncontrollable factors such as increases in operating costs, adverse weather, changes in the overall economy, new competitors entering the market, or key employee turnover may have decreased efficiency. 1-34

(5 min.)

1. Line, support 2. Line, marketing

..

3. Line, Product Design 4. Staff, support

5. Staff, support 6. Line, production

11


1-35

(30 min.)

Microsoft is a company that most students will know and have some understanding of what functions its managers perform. Nevertheless, this may not be an easy exercise for those who have little knowledge of how companies operate. Research & development – Because software companies must continually come out with new products and upgrades to their current products, this is a critical function for Microsoft. More than one-fourth of Microsoft’s operating expenses are devoted to R&D. Product and process design – For Microsoft the design and R&D process overlap considerably. Product design is critical; process design is probably less critical. One essential part of design is beta testing – that is, field testing of new software. This quality-control step is essential to prevent customer dissatisfaction with new products. Production – Microsoft produces disks and CD-ROMs and the manuals and packaging to go with them. However, software is increasingly delivered and updated over the Internet, which takes an initial process design and then few resources. Thus, production of physical items is becoming a less important focus for Microsoft. Marketing – Microsoft spends more on sales and marketing than on any other operating expense. Increasing competition in software sales makes marketing essential to the company’s future. This function includes advertising and direct marketing activities, but it also includes activities of the company’s sales force. Distribution – This function is becoming simpler for Microsoft as it delivers more and more software over the Internet. As long as the company does not fall behind competitors in delivery methods, this is not likely to create a major competitive advantage or disadvantage for Microsoft. Customer service – Customer service is important, but Microsoft tries to minimize costs in this area by good product design – making things work right for customers without deep computer expertise. Poor customer service can severely impact a company, so Microsoft must attend to it. Support functions – Most of the time these are not a major focus. One major exception for Microsoft is legal support – the future of the company depended on some recent court decisions.

..

12


1-36

(15-20 min.)

The management accountant's major purpose is to provide information that helps line managers plan and control operations and make decisions. The accountant supplies information for scorekeeping, attention directing, and problem solving. In turn, managers use this and other information for routine and non-routine decisions and for evaluating subordinates and the performance of sub-parts of the organization. Management accountants must walk a delicate line between (1) making sure that managers use information properly and (2) making sure that the managers, not the accountants, are doing the actual managing. Management accountants are increasingly a part of management teams that make important decisions. 1-37

(5 min.)

Costs of a poor ethical environment include legal costs and costs due to absenteeism and high employee turnover. Benefits of a good ethical environment include improved morale, lower absenteeism and employee turnover, lower loss from internal theft, and improved customer satisfaction resulting from better quality and service (that result from a more productive work environment). 1-38

(5 min.)

There are numerous examples. “You understand how important it is to record this sale before year end, don’t you?” “Doing it this way is common for all companies in our business, so don’t worry!” “Trust me, the inventory is at the warehouse.” “I don’t know how you will do it. Just get it done.” “Don’t worry. It won’t be disclosed to anyone else.” “You must do something. We can’t afford to lose this opportunity.”

..

13


1-39

(15-20 min.)

This problem can form the basis of an introductory discussion on the entire field of management accounting. Text Exhibit 1-1 provides more details. 1.

The focus of management accounting is on helping internal users make better decisions, whereas the focus of financial accounting is on helping external users make better decisions. Management accounting helps in making a host of decisions, including pricing, product choices, investments in equipment, making or buying goods and services, and manager rewards.

2.

Generally accepted accounting principles constrain financial accounting but not management accounting. For example, if an organization wants to account for assets on the basis of replacement costs for internal purposes, no outside agency can prohibit such accounting. Of course, this means that the organizations may set up an additional system to keep track of this cost. There is nothing immoral or unethical about having multiple accounting systems, but they come at a cost. Accounting datasets are commodities, just like butter or eggs, and an additional internal accounting system must meet the same cost–benefit test as other commodities. That is, the perceived increases in benefits from the commodity must exceed the perceived increases in costs. Ultimately, benefits are measured by whether managers make better decisions that result in increased net profits.

3.

Budgets, the formal expressions of the management’s plans, are a major feature of management accounting, whereas they are not as prominent in financial accounting. Budgets are major devices for compelling and disciplined management planning.

4.

An important use of management accounting information is the evaluation of performance and provision of incentives and feedback to improve future decisions.

5.

Accounting systems have an enormous influence on the behavior of individuals affected by them. Management accounting is more concerned with the likely behavioral impacts of various accounting alternatives as compared to financial accounting.

..

14


1-40

(10 min.)

When flat fees are being received for different products or services, it is essential for the hospital to have detailed knowledge of the cost of each product or service. Accurate cost information is crucial for decisions regarding which products and services should be emphasized or de-emphasized. Hospitals will increasingly identify costs by product (type of case), not just by departments. In contrast, when all costs are reimbursed, it is less important to the hospital to know which products or services cause costs, because all costs are reimbursed by someone. Even though reimbursors are concerned about how costs are assigned to products and services, the hospital has less reason to be concerned about assigning costs accurately. When somebody’s money is at stake, accounting systems get more attention. Accountability is important for many reasons, including justification of prices, cost control, and response to criticisms by stakeholders (whether they be investors, donors, taxpayers, or others). 1-41

(10 min.)

Paperwork and systems often seem to become ends in themselves. However, the rationale that should underlie systems design is the cost-benefit philosophy that is implied in the quotation. The aim is for the improvement in revenue and/or reduction in costs due to better decisions to outweigh the costs of the accounting system. Marks & Spencer should look at each of the management accounting reports it produces with an eye toward how it helps managers make better decisions. Does it provide needed scorekeeping? Does it direct attention to aspects of operations that might need altering? Does it provide information for specific management decisions? These types of questions will help identify the benefit of the information in the report. Then the company must consider the cost – not just the cost of collecting the data and preparing the reports, but the cost of educating managers to use the information and the cost of the time to read, digest, and act on the information. Too much information may be costly because it makes it time-consuming (and thus costly) to sift through the reams of information to find the few items that are important. An additional cost may be the loss of important information because the total volume of information makes it too difficult to ferret out the important items.

..

15


1-42 (10 min.) Financial information is important in all companies. But how managers get and use financial information can differ depending on the culture and philosophies of the company. Top executives of a company often represent a functional area critical to the competitive economic advantage of the company. When technology is crucial, engineers generally hold important executive positions. If marketing differentiates the company from others, marketing executives usually dominate. But regardless of the source of a company’s competitive advantage, its success will eventually be measured in economic terms. The company must attend to financial aspects to thrive and often even to survive. Management accountants must work with the dominant managers in any organization. The modern trend toward use of cross-functional teams places management accountants at the center of the action regardless of what type of managers and executives dominate. Most companies realize that there is a financial dimension to almost every major decision, so they want the financial experts, management accountants, involved in the decisions. But to be accepted as an important part of these teams, the management accountants must know how to help managers in various functional areas. In Hitachi, if they do not understand the information needs of engineers, they will not provide value. 1-43

(10-15 min.)

1.

Boeing's competitive environment and manufacturing processes changed greatly in recent years. An accounting system that served them well in their old environment would not necessarily be optimal in today. Boeing's management probably thought that changes in the accounting system were necessary to produce the kind of information necessary to remain competitive.

2.

A cost-benefit criterion was probably used. Boeing's management may not have quantified all the costs and benefits, but they certainly assessed whether the new system would help decisions enough to warrant the cost of the system. Many of the benefits of a better accounting system are hard to measure. They affect many strategic decisions of an organization. Without accurate product costs, management will find it difficult to accurately assess the consequences of their decisions.

3.

..

More accurate product costs will usually result in better management decisions. But if the cost of the accounting system that produces the more accurate costs is too high, it may be best to forego increased accuracy. The benefit of better decisions must exceed the added cost of the system for a change to be desirable.

16


1-44

(10 min.)

1.

There are many possible activities for each function of Nike's value chain. Some possibilities are: Research and development -- Determining changes in customers' tastes and preferences for shoes and sportswear to come up with new products (maybe the next "Air Jordans"). Product, service, and process design -- Design a shoe to meet the increasing demands of competitive athletes. Production -- Determine where to produce products and negotiate contracts with the companies producing them. Marketing -- Signing prominent athletes to endorse Nike's products. Distribution -- Select the best locations for warehouses for distribution to retail outlets. Customer service -- Formulate return policies for products that customers perceive to be defective.

2.

Accounting information that aids managers' decisions includes: Research and development -- Trends in sales for various products, to determine which are becoming more and less popular. Product, service, and process design -- Production costs of various shoe designs. Production -- Measure total costs, including both production cost and transportation costs, for production in various parts of the world. Marketing -- The added profits generated by the added sales due to product endorsements. Distribution -- Storage and shipping costs for alternative warehouse locations. Customer service -- The net cost of returned merchandise, to be compared with the benefits of better customer relations.

..

17


1-45 (10-15 min.) This problem can lead to a long discussion. Pointing out the problems can be done reasonably quickly but formulating solutions can take much longer. 1.

The appropriate accounting information presented correctly should be helpful to managers. It is clear that Smith does not regard the accounting performance reports as helpful. Some key problems are: • Smith refers to “their” budget, meaning that the budget belongs to the controller’s department, not to him and his department. Managers should be involved in formulating the budget so that they accept it as a reasonable target. • The controller’s office shows up only when costs are over budget. Controllers should not be “policemen.” They should be business advisors who provide continual assistance not occasional reprimands. • Smith clearly does not understand the performance reports. An important role for the controller is education of managers on how to use accounting information. • Smith believes the performance report has nothing to do with what happens in reality. He may be right. Accounting reports often arrive too late and are not specific enough to be useful to front-line managers. If so, the reports should be changed or the results used differently. • Paperwork takes time away from other activities. This is especially a problem when the numbers have little value to those putting in the time. • Budgeting is not taken seriously, so the numbers reported by Smith and his subordinates are not reliable. • Things have gotten so bad that Smith has an attitude problem toward the controller’s office. Daniel is meeting him for the first time, and he is already disrespectful of him.

2.

Daniel has major problems. His first task is to get the cooperation of Smith and his subordinates. This will probably involve changing the accounting reports received by the line managers, and it will certainly involve changes in how these reports are presented and used. If the reports are not useful, he needs to find out why. Then he can change the reports so that the managers find them helpful. He needs to show managers how they can use information to make better decisions. Foremost, Daniel has to change the attitudes of the line managers toward the controller’s department. This will take time, and it will require some specific instances where the controller or his staff provides information that the managers perceive as useful. To do this, he may need to change the accounting system to produce better information, and he needs to teach his controller’s department staff how to present information in a nonthreatening way. There is no one solution to Daniel’s problems. Different managers would handle it in different ways. If students have had experience, there will be many suggestions about how to proceed. For students with little experience, it may be sufficient to point out the variety of possible approaches.

..

18


1-46

(10-15 min.)

Accountants become the information experts in many companies. In a multinational company, with its varied subsidiaries, the accounting system provides a link between the various operating companies. The accountants provide information about the operations of an individual unit, and they also show how the units fit together as parts of the group. Management accountants should work together with managers to determine what information the managers would find useful. Then the accountants should help devise systems to produce that information, provided that its value is greater than its cost. As such, management accountants are information consultants to managers. Decisions are still the domain of managers, but the accountants provide advice to help managers make better decisions. Accountants have sometimes been viewed as “corporate cops,” staff members who reported on the failings of managers. They were primarily scorekeepers, but when the score showed something awry, they became informants—carriers of bad news to corporate headquarters. Managers resented them. But today, good management accountants are allies of managers. They provide information that helps managers make better decisions, which makes the managers look good. Everyone is better off when management accountants focus on providing the information that aids management decisions. To be effective internal consultants, accountants must have a background in accounting and information systems. In addition, they must have knowledge of all the functions of business and all the areas of the value chain.

..

19


1-47

(10-15 min.)

1.

Brigham’s decisions violate standards of competence and integrity. Competence is violated because the most competent persons apparently are not being hired, jeopardizing the competence of the accounting department. Further, Brigham may be violating equal opportunity employment laws and regulations. Integrity requires an accountant to avoid conflicts of interest, and hiring the sons of personal friends certainly appears to be a conflict of interest. Such hiring was possibly for the personal gain of Brigham at the expense of the company. Further, this practice subverts the company’s equal employment opportunity policy.

2.

Merton’s first step normally would be to discuss this situation with his boss. However, because the alleged unethical behavior is by his boss and Merton has already confronted him and been rebuffed, the next step seems warranted. This would involve going to Brigham’s superior. (Alternately, some organizations have an individual, possibly called an ombudsperson, to whom Merton could report such concerns.) If the matter could not be resolved at that level, he should continue up the line until reaching Creighton, the president. If equal employment opportunity is genuinely a company priority, Creighton should be very concerned about Brigham’s actions. What if the situation is not resolved to Merton’s satisfaction after following the steps in the preceding paragraph? The final step is to go directly to the Board of Directors. If that is unsatisfactory, there may be no recourse but to resign, sending an explanatory memo to an appropriate high-level official of the company. Should Merton go to the press so that they will put on pressure to change the hiring practices? Such a step is generally not appropriate. It would put Merton in the position of violating the ethical standard of confidentiality. The only person external to the firm with whom it is appropriate to discuss this issue is a confidential objective advisor.

..

20


1-48

(15-20 min.)

1.

Because of the standard of confidentiality, the information in the geologist's report should not be revealed.

2.

The standard of integrity would require one to reject the Europe tour offer.

3.

This is a difficult ethical problem, one that deserves discussion. Two ethical standards apparently conflict. Confidentiality would lead to nondisclosure, provided there was no legal requirement to do so. But credibility would indicate that the information about the additional losses should be used in making the earnings prediction. The authors think that credibility should take precedence here, but others might disagree.

4.

The standard of competence (and the standard of integrity) would lead one to research the tax law before deciding whether to exclude the dividend income from the taxable income.

..

21


1-49

(15-25 min.)

There are various possible answers. These are just some of the items that might be mentioned. 1) Environment - Evaluation of environmental disclosure, environmental policies (including management systems), and environmental performance (including toxic emissions, waste management, evidence of chemical and oil spills and environmental fines). Top companies were IBM and Johnson Controls. 2) Climate Change - Climate change disclosure (including the Carbon Disclosure Project, as well as company websites and reports) and climate change policies (including offsets and reduction goals). Top companies were IBM and Johnson Controls. 3) Human rights – Evaluation of disclosure, policy (including codes of conduct and performance goals), and exposure to 45 countries of concern. Companies with higher exposure need to earn higher scores in disclosure and policy to do well. Top among the top ten companies were Johnson Controls, Campbell Soup, Mattel, and Accenture. 4) Employee relations – Evaluates unionization rates, publicly disclosed employee benefits, and Equal Employment Opportunity Commission complaints. Top among the top ten companies: Hewlett-Packard and Campbell Soup. 5) Corporate governance - A majority of a board of directors and key committees of the board must be independent of management. In addition, ratings include general board accountability and demographics (board tenure, age of directors, over-commitment of directors to multiple boards, and annual election of all directors), and the percentage of CEO pay that is incentive based. Of the top ten companies, six were tied for the highest score on governance – Campbell Soup, Bristol-Myers Squibb, Mattel, 3M, HewlettPackard, and Nike. 6) Philanthropy – Includes corporate giving that has a substantial and positive impact on society and evaluates giving levels and policies (including employee match programs). Top among the top ten companies were Johnson Controls and Accenture. 7) Financial – Evaluates the three-year return on investment in the company stock, based on Morningstar rankings. Companies without a three-year return to shareholders were not considered for the ranking. Top among the top ten companies were Nike and Mattel.

..

22


1-50

(10-15 min.)

1.

Line authority is held by those managers directly responsible for the production and sales of goods or services. Staff authority is held by persons who have an indirect responsibility for the production and sale of goods and services. Staff members provide expertise, advice, and support for line positions; line managers are directly responsible for achieving the basic objectives of the organization. Conflicts between line and staff can arise for many reasons, ranging from the types of people that are generally attracted to each type of position to their responsibilities in the organization. Among the reasons are:  Staff personnel tend to be younger, better educated, more professionally established.  Line managers see staff managers as threats to their authority.  Line managers are uncomfortable when they must rely on the knowledge and expertise of staff.  Line managers often think staff managers overstep their authority and have a narrow view of the world.  Staff managers often think line managers ignore their advice and resist their ideas.

2.

Chen has a staff position, providing advice to the controller. His main conflicts will probably arise with the chief accountant and the managers under him. He reports to the chief accountant’s superior, but he prepares reports that affect operations in the chief accountant’s area of responsibility. Paperman is in a staff position because accounting is not directly involved with sales or delivery of leasing services. He provides counsel and advice to all the line managers and most of the staff managers in the company. Conflicts may arise if he tries to exert authority instead of just giving advice or if the other managers ignore his advice. Hodge is in a line position because she is an integral part of the company’s main line of business, leasing equipment. Her main conflicts are likely to arise in areas such as requisitioning of equipment and billing of customers where she must rely on other departments over which she has no authority. Shevlin is in a staff position and offers advice to most other managers in the company. Conflicts might arise if managers perceive her advertising of positions or screening of candidates as not fulfilling their needs, or if she tries to insert her preferences instead of the hiring department’s preferences into the advertising and screening activities. Conflicts can also arise in the performance evaluation functions, where she may be enforcing an unpopular policy.

..

23


1-51

(20-30 min.)

1.

In accordance with Exhibit 1-7, IMA Statement of Ethical Professional Practice, management accountants should not condone the commission of acts by their organization that violate the standards of ethical conduct. The specific standards that apply are:

2.

competence. Management accountants have a responsibility to perform their professional duties in accordance with relevant laws and regulations.

confidentiality. Management accountants must refrain from disclosing confidential information unless legally obligated to do so. Rachel O’Casey may have a legal responsibility to take some action.

integrity. Management accountants have a responsibility to - refrain from engaging in any conduct that would prejudice carrying out duties ethically. - refrain from engaging in or supporting any activity that would discredit the profession.

credibility. Management accountants have a responsibility to communicate information fairly and objectively. They also should disclose all relevant information that could reasonably be expected to influence a user’s understanding of reports, analyses, and recommendations.

In accordance with Exhibit 1-7, the first alternative being considered by Rachel O’Casey, seeking the advice of her boss, is appropriate. To resolve an ethical conflict, the first step recommended is to discuss the problem with the immediate superior, unless it appears that this individual is involved in the conflict. In this case, it does not appear that O’Casey’s boss is involved. Releasing the information to the local newspaper would be an inappropriate course of action. Communication of confidential information to anyone outside of the company is inappropriate unless there is a legal obligation to do so, in which case O’Casey should contact the proper authorities. Contacting a member of the board of directors would be an inappropriate action at this time. In accordance with Exhibit 1-7, O’Casey should report the conflict to successively higher levels within the organization. Thus, the problem should be reported to the board of directors only if the problem is not resolved at lower levels.

3.

..

Assuming there is no established company policy in place to resolve the conflict, O’Casey should report the problem to successively higher levels of management until it is satisfactorily resolved. There is no requirement for O’Casey to inform

24


her immediate supervisor of this action, because he is involved in the conflict. O’Casey could also clarify the situation by confidential discussion with an objective advisor to obtain an understanding of possible courses of action. If the conflict is not resolved after exhausting all courses of internal review, O’Casey may have no other recourse than to resign from the organization and submit an informative memorandum to an appropriate representative of the organization. 1-52 (15-25 min.) These answers are based on information in the Nike 10K for the year ended May 31, 2011. 1.

Nike’s principal business activity is the design, development and worldwide marketing of high quality footwear, apparel, equipment, and accessory products. Nike is the largest seller of athletic footwear and athletic apparel in the world, selling in over 170 countries.

2.

About 43% of Nike revenue comes from sales in the U.S. and 57% from sales abroad. There are 363 retail stores in the U.S.

3.

Nike’s CFO is Donald W. Blair. He came to Nike from Pepsico in 1999. Before that he was a certified public accountant (CPA) with Deloitte, Haskins, and Sells (now Deloitte & Touche).

4.

Nike manufactures 39 percent, 33 percent, 24 percent and 2 percent of total NIKE brand footwear in Vietnam, China, Indonesia, and India, respectively. Almost all of the brand apparel manufacturing is also outside the United States, by independent contract manufacturers in 33 countries. Although Nike imposes on these contractors a code of conduct and other environmental, health, and safety standards, it is difficult to ensure that all of the contractors are always in compliance. This could harm Nike’s reputation for high ethical standards.

1-53 (20-30 min.) For the solution to this Excel Application Exercise, follow the stepby-step instructions provided in the textbook chapter.

..

25


1-54

(90 min. or more)

The purpose of this exercise is to learn about the practice of management accounting. Students often have the mistaken impression that accountants sit in the back room and prepare reports. These articles illustrate the varied skills and abilities that are necessary to be a successful management accountant. The exercise also focuses on critical reading -- identifying the most important points made in an article. It also shows how different students will focus on different aspects of each article. What one student considers important, others might think unimportant. Prioritizing the lessons will bring out differences in opinion and create a need to form consensus from possibly conflicting views. Finally, students should come away with a better understanding of why they are studying management accounting, whether they plan to be an accountant or simply a user of accounting information and services. 1-55 (30-45 min.) NOTE TO INSTRUCTOR: This solution is based on the web site as it existed in early 2012. Be sure to examine the current web site before assigning this problem, as the information there may have changed. 1.

The Institute of Management Accountants is "to provide a forum for research, practice development, education, knowledge sharing, and the advocacy of the highest ethical and best business practices in management accounting and finance."

2.

The IMA offers more than 300 NASBA-approved courses on topics from general finance and accounting standards to strategic management.

3.

The IMA has a detailed code of ethics. It specifies accountants’ obligations to the public, their profession, their organization, and to themselves. It also addresses how to resolve ethical conflicts. It makes it clear that for accountants to fulfill their function in an organization, they must both be ethical and be perceived as being ethical.

..

26


CHAPTER 2 COVERAGE OF LEARNING OBJECTIVES

FUNDAMENTAL ASSIGN-MENT MATERIAL A1, B1, B4

CASES, NIKE 10K, EXCEL, COLLAB., & INTERNET EXERCISES 68

ADDITIONAL ASSIGNMENT LEARNING OBJECTIVE MATERIAL LO1: Explain how cost 25, 26, 27, 28, 30, drivers affect cost behavior. 31, 47, 47, 49, 51, 53, 56, 68, LO2: Show how changes in A1, B1, A2, A3, B2, 25, 26, 29, 30, 31, 68, 69, 73 activity cost-driver levels B3, B4 32, 32, 41, 48, 47, affect variable and fixed costs. 47, 48, 49, 51, 52, 53, 54, 56, 59, 59, 60, 60, 63, 68 LO3: Explain step- and A4,B4 24,36,37, 37, 38, 75 mixed-cost behavior. 38, 39 LO4: Create a cost-volumeB4 33, 33, 34, 34, profit graph and understand 35,40, 49 the assumptions behind it. LO5: Calculate break-even A2, A3, B2, B3 14, 40, 41,42, 42, 68, 69, 73, 74 sales volume in total dollars 43, 47, 47, 50, 52, and total units. 54, 55, 57, 57, 59, 59, 61, 68 LO6: Calculate sales volume A2 , B3 B4 15, 33, 33, 34, 34, 69 in total dollars and total units 42, 43, 47, 47,50, to reach a target profit. 52, 54, 55, 57, 57, 59, 62, 62 LO7: Differentiate between 61 contribution margin and gross margin. LO8: Explain the effects of B4 44, 47, 64, 65 70 sales mix on profits (Appendix 2A). LO9: Compute cost-volume45, 46, 66, 67 71 profit relationships on an aftertax basis (Appendix 2B). ..

27


CHAPTER 2 Introduction to Cost Behavior and Cost-Volume Relationships 2-A1 (20-25 Min.) 1. The cost driver for both resources is number of times the plant is cleaned. Labor cost is a fixed-cost resource, and cleaning supplies is a variable cost. Costs for cleaning between 4 and 8 times a month are: Number of times plant Square Feet Cleaning Supplies is cleaned Cleaned Labor Cost Cost** Total cost * *** 4 200,000 $21,000 $ 8,000 $29,000 5 250,000 21,000 10,000 31,000 6 300,000 21,000 12,000 33,000 7 350,000 21,000 14,000 35,000 8 400,000 21,000 16,000 37,000 * 4 × 50,000 square feet ** Cleaning supplies cost per time the plant is cleaned = $8,000 ÷ 4 = $2,000 *** $2,000 per cleaning × number of times plant is cleaned

Cost per Cleaning $7,250 $6,200 $5,500 $5,000 $4,625

The predicted total cost to clean the plant during the next quarter is the sum of the total costs for monthly cleanings of 5, 6, and 8 times. This is $31,000 + $33,000 + $37,000 = $101,000 2. If Napco hires the outside cleaning company, all its cleaning costs will be variable at a rate of $5,700 per cleaning. The cost driver will be “number of times cleaned.” The predicted cost to clean a total of 5 + 6 + 8 = 19 times is 19 × $5,700 = $108,300. Thus, Napco will save by not hiring the outside cleaning company. The table below shows the total costs for the two alternatives. If Napco expects average “times cleaned” to be 6 or more, it would save by cleaning with its own employees. If Napco expects to average 5 or fewer cleanings per month, it would save by outsourcing. Napco Cleans Plant Times Cleaned Napco 4 $ 29,000 5 31,000 6 33,000 7 35,000 8 37,000

..

Outsource Cleaning Plant Times Cleaned Outside 4 $22,800 5 28,500 6 34,200 7 39,900 8 45,600

28


2-A2 1.

(20-25 min.) Let N Sales $1.00 N $.32 N N Let S S .32 S S

= number of units = Fixed expenses + Variable expenses + Net income = $4,000 + $.68 N + 0 = $4,000 = 12,500 units = sales in dollars = $4,000 + .68 S + 0 = $4,000 = $12,500

Alternatively, the 12,500 units may be multiplied by the $1.00 to obtain $12,500. In formula form: In units

Fixed costs + Net income ($4,000 + 0) = = 12,500 units Contribution margin per unit $.32 In dollars

Fixed costs + Net income ($4,000 + 0) = = $12,500 Contribution margin percentage .32 2.

The quick way: (45,000 – 12,500) × $.32 = $10,400 Compare income statements:

Volume in units Sales Deduct expenses: Variable Fixed Total expenses Effect on net income

3. ..

Break-even Point 12,500 $12,500

Increment 32,500 $32,500

Total 45,000 $45,000

8,500 4,000 12,500 $ 0

22,100 --22,100 $ 10,400

30,600 4,000 34,600 $ 10,400

Total fixed expenses would be $4,000 + $1,600 = $5,600 29


$5,600 $5,600 = 17,500 units; = $17,500 sales $.32/unit .32 or 17,500 units × $1.00 / unit 4.

= $17,500 sales

New contribution margin is $1.00 – $.68 – $.07 = $.25 per unit Breakeven = Fixed cost ÷ contribution margin = $4,000 ÷ $.25 = 16,000 units 16,000 units × $1.00 = $16,000 in sales

5.

The quick way: (45,000 – 12,500) × $.21 = $6,825. On a graph, the slope of the total cost line would have a kink upward, beginning at the break-even point.

2-A3

(20-30 min.)

The following format is only one of many ways to present a solution. This situation is really a demonstration of "sensitivity analysis," whereby a basic solution is tested to see how much it is affected by changes in critical factors. Much discussion can ensue, particularly about the final three changes. The basic contribution margin per revenue mile is $2.00 - $1.60 = $.40

1. 2.

(a) (b) (c) (d) (e) (f) (g)

(1)

(2)

(4)

(5) (3)-(4)

Contribution Margin Per Revenue Mile

(3) (1)×(2) Total Contribution Margin

Revenue Miles Sold

Fixed Expenses

Net Income

500,000

$.40

$200,000

$50,000

$ 150,000

500,000 650,000 500,000 500,000 575,000 350,000 575,000

1.00 .40 (.08) .40 .35 .41 .40

500,000 260,000 (40,000) 200,000 201,250 143,500 230,000

50,000 50,000 50,000 65,000 50,000 50,000 65,000

450,000 210,000 (90,000) 135,000 151,250 93,500 165,000

2-A4 (20-25 min.) Some of these answers are controversial, and reasonable cases can be built for alternative classifications. Class discussion of these answers should lead to worthwhile disagreements about anticipated cost behavior with regard to alternative cost drivers. 1. ..

(b) Fixed cost. 30


2. 3. 4. 5. 6. 7. 8. 9. 10. 11.

..

(d) Step cost. (a) Variable cost with respect to revenue. (a) Variable cost with respect to miles flown. (c) Mixed cost with respect to miles driven. (b) Fixed cost. (b) Fixed cost. (b) Fixed cost. (a) Variable cost with respect to cases of 7-Up. (b) Fixed cost. (b) Fixed cost.

31


2-B1 (20-25 Min.) 1. The cost driver for both resources is number of times the restaurant is cleaned. Labor cost is a fixed-cost resource, and cleaning supplies is a variable cost. Costs for cleaning between 35 and 50 times are: Square Feet Cleaned 210,000* 240,000 270,000 300,000

Cleaning Supplies Cost** $ 16,800 19,200 21,600 24,000

Times Labor Total Cost per Cleaned Cost Cost Cleaning 35 $21,000 $37,800 $1,080 40 21,000 40,200 $1,005 45 21,000 42,600 $ 947 50 21,000 45,000 $ 900 * 35 × 6,000 ** The cost of cleaning supplies per cleaning = $16,800 ÷ 35 = $480 per cleaning. The cost per square foot is $480 ÷ $6,000 = $.08 or $16,800 ÷ 210,000 = $.08. The total cleaning supplies cost is either $480 × number of cleanings or $.08 × square feet cleaned. The predicted total cost to clean during the November and December is the sum of the total costs for monthly cleanings of 45 and 50 times. This is $42,600 + $45,000 = $87,600 2. If Applejack hires the outside cleaning company, all its cleaning costs will be variable at a rate of $0.25 per square foot cleaned. The predicted cost to clean a total of 45 + 50 = 95 times is 95 × 6,000 × $0.25 = $142,500. Thus, Applejack will not save by hiring the outside cleaning company. To determine whether outsourcing is a good decision on a permanent basis, Applejack needs to know the expected demand for the cost driver over an extended time frame. As the following table shows, outsourcing becomes less attractive when cost driver levels are high. If average demand for cleaning is expected to be more than the number of cleanings at which the cost of outsourcing equals the internal cost, Applejack should continue to do its own cleaning. This point is C cleanings, where: $.25 × C × 6,000 = $21,000 + ( $.08 × C × 6,000) C = $21,000 ÷ ($.17× 6,000) = 20.588 cleanings Applejack should also consider such factors as quality and cost control when an outside cleaning company is used.

(1) Times (2) Square Feet (3) Applejack Outside Cleaning Cost * Cleaned Cleaned Total Cleaning Cost $.25 × (2) 35 210,000 $37,800 $52,500 40 240,000 40,200 60,000 45 270,000 42,600 67,500 50 300,000 45,000 75,000 * From requirement 1, total cost is $21,000 + $.08 x square feet cleaned ..

32


2-B2

(15-25 min.)

1.

$2,340 ÷ ($30 - $12) = 130 child-days or 130 × $30 = $3,900 revenue.

2.

176 × ($30 - $12) - $2,340 = $3,168 - $2,340 = $828

3.

a.

198 × ($30 - $12) - $2,340 = $3,564 - $2,340 = $1,224 or (22 × $18) + $828 = $396 + $828 = $1,224

b.

176 × ($30 - $14) - $2,340 = $2,816 - $2,340 = $476 or $828 - ($2 × 176) = $476

c.

$828 - $220 = $608

d.

[(9.5 × 22) × ($30 - $12)] - ($2,340 + $300) = $3,762 - $2,640 = $1,122

e.

[(7 × 22) × ($33 - $12)] - $2,340 = $3,234 - $2,340 = $894

2-B3

(15-20 min.)

1.

$9,100 $9,100 = = 1,300 units $7 ($25 - $18)

2.

Contribution margin ratio:

($43,000 - $30,100) = 30% $43,000

$8,400 ÷ 30% = $28,000 3.

$38,400 ($30,400 + $8,000) = = 2,400 units $16 ($29 - $13)

4.

($51,000 - $18,000) × (120%) = $39,600 contribution margin; $39,600 - $18,000 = $21,600

5.

New contribution margin:

$48 - ($36 - 25% of $36) = $48 - ($36 - $9) = $21;

New fixed expenses: $106,000 × 115% = $121,900;

$144,900 ($121,900 + $23,000) = = 6,900 units $21 $21

..

33


2-B4

(20-25 min.)

The following classifications are open to debate. With appropriate assumptions, other answers could be equally supportable. For example, in #2, the health insurance would be a fixed cost if the number of employees will not change. This problem provides an opportunity to discuss various aspects of cost behavior. Students should make an assumption regarding the time period involved. For example, if the time period is short, say one month, more costs tend to be fixed. Over longer periods, more costs are variable. They also must assume something about the nature of the cost. For example, consider #4. Repairs and maintenance are often thought of as a single cost. However, repairs are more likely to vary with the amount of usage, making them variable, while maintenance is often on a fixed schedule regardless of activity, making them fixed. Another important point to make is the cost/benefit criterion applied to determining “true” cost behavior. A manager may accept a cost driver that is plausible but may have less reliability than an alternative due to the cost associated with maintaining data for the more reliable cost driver. Cost

Cost Behavior

Likely Cost Driver(s)

1.X-ray operating cost

Mixed

Number of x-rays

2.Insurance

Step (or variable)

Number of employees

3. Cancer research

Fixed

4. Repairs

Variable

5.Training cost

Fixed

6.Depreciation

Fixed

7. Consulting

Fixed

8. Nursing supervisors

Step

..

Number of patients

Number of nurses, patient-days

34


2-1 This is a good characterization of cost behavior. Identifying cost drivers will identify activities that affect costs, and the relationship between a cost driver and costs specifies how the cost driver influences costs. 2-2

Two rules of thumb to use are: a. Total fixed costs remain unchanged regardless of changes in cost-driver activity level. b. The per-unit variable cost remains unchanged regardless of changes in costdriver activity level.

2-3 Examples of variable costs are the costs of merchandise, materials, parts, supplies, sales commissions, and many types of labor. Examples of fixed costs are real estate taxes, real estate insurance, many executive and supervisor salaries, and space rentals. 2-4 Fixed costs, by definition, do not vary in total as volume changes within the relevant range and during the time period specified (a month, year, etc.). However, when the cost-driver level is outside the relevant range (either less than or greater than the limits) management must decide whether to decrease or increase the capacity of the resource, expressed in cost-driver units. In the long run, all costs are subject to change. For example, the costs of occupancy such as a long-term non-cancellable lease cannot be changed for the term of the lease, but at the end of the lease management can change this cost. In a few cases, fixed costs may be changed by entities outside the company rather than by internal management – an example is the fixed, base charge for some utilities that is set by utility commissions. 2-5 Yes. Fixed costs per unit change as the volume of activity changes. Therefore, for fixed cost per unit to be meaningful, you must identify an appropriate volume level. In contrast, total fixed costs are independent of volume level. 2-6 No. Cost behavior is much more complex than a simple dichotomy into fixed or variable. For example, some costs are not linear, and some have more than one cost driver. Division of costs into fixed and variable categories is a useful simplification, but it is not a complete description of cost behavior in most situations. 2-7 No. The relevant range pertains to both variable and fixed costs. Outside a relevant range, some variable costs, such as fuel consumed, may behave differently per unit of activity volume. 2-8 The major simplifying assumption is that we can classify costs as either variable or fixed with respect to a single measure of the volume of output activity. 2-9 The same cost may be regarded as variable in one decision situation and fixed in a second decision situation. For example, fuel costs are fixed with respect to the addition of one more passenger on a bus because the added passenger has almost no effect on total fuel costs. In contrast, total fuel costs are variable in relation to the decision of whether to add one more mile to a city bus route. ..

35


2-10 No. Contribution margin is the excess of sales over all variable costs, not fixed costs. It may be expressed as a total, as a ratio, as a percentage, or per unit. 2-11 A "break-even analysis" does not describe the real value of a CVP analysis, which shows profit at any volume of activity within the relevant range. The break-even point is often only incidental in studies of cost-volume relationships. CVP analysis predicts how managers’ decisions will affect sales, costs, and net income. It can be an important part of a company’s planning process. 2-12 No. break-even points can vary greatly within an industry. For example, Rolls Royce has a much lower break-even volume than does Honda (or Ford, Toyota, and other high-volume auto producers). 2-13 No. The CVP technique you choose is a matter of personal preference or convenience. The equation technique is the most general, but it may not be the easiest to apply. All three techniques yield the same results. 2-14 For a given contribution margin per unit, an increase in fixed costs will increase the break-even point, whereas a decrease in fixed costs will reduce the break-even point. Again, given a particular level of fixed costs, any reduction in the unit contribution margin will result in an increase in the break-even point and an increase in the contribution margin per unit will lead to a fall in the break-even point. 2-15 Spreadsheet-based CVP modeling simplifies the analysis of several changes in the key parameters that determine the break-even point. Thus, managers can evaluate simultaneous changes in selling price, unit variable costs, fixed costs, and target profit. Spreadsheets can calculate the results for alternative assumptions extremely quickly and can display the results both numerically and graphically. Moreover, complex real-life situations involving multiple cost drivers, non-linear relationships between costs and cost drivers, and varying sales mixes can easily be evaluated using spreadsheet-based CVP models. 2-16 Operating leverage is a firm's ratio of fixed to variable costs. A highly leveraged company has relatively high fixed costs and low variable costs. Such a firm is risky because small changes in volume lead to large changes in net income. This is good when volume increases but can be disastrous when volumes fall. 2-17 An increase in demand for a company’s products will drive almost all other costdriver levels higher. This will cause cost drivers to exceed capacity or the upper end of the relevant range for its fixed-cost resources. Since fixed-cost resources must be purchased in “chunks” of capacity, the proportional increase in cost may exceed the proportional increase in the use of the related cost-driver. Thus cost per cost-driver unit may increase.

..

36


2-18 The margin of safety shows how far sales can fall before losses occur – that is before the company reaches the break-even sales level. 2-19 No. In retailing, the contribution margin is likely to be smaller than the gross margin. For instance, sales commissions are deducted in computing the contribution margin but not the gross margin. In manufacturing companies the opposite is likely to be true because there are many fixed manufacturing costs deducted in computing gross margin. 2-20 No. CVP relationships pertain to both profit-seeking and nonprofit organizations. In particular, managers of nonprofit organizations must deal with tradeoffs between variable and fixed costs. To many government department managers, lump-sum budget appropriations are regarded as the available revenues. 2-21 Contribution margin could be lower because the proportion of sales of the product bearing the higher unit contribution margin is lower than the proportion budgeted. 2-22 Target income before income taxes

=

Target after-tax net income 1 - tax rate

2-23

Change in Change in volume Contribution margin = × × (1 - tax rate) per unit net income in units 2-24 The fixed salary portion of the compensation is a fixed cost. It is independent of how much is sold. In contrast, the 5% commission is a variable cost. It varies directly with the amount of sales. Because the compensation is part fixed cost and part variable cost, it is considered a mixed cost. 2-25 The key to determining cost behavior is to ask, “If there is a change in the level of the cost driver, will the total cost of the resource change immediately?” If the answer is yes, the resource cost is variable. If the answer is no, the resource cost is fixed. Using this question as a guide, the cost of advertisements is normally variable as a function of the number of advertisements. Note that because the number of advertisements may not vary with the level of sales, advertising cost may be fixed with respect to the cost driver “level of sales.” Salaries of marketing personnel are a fixed cost. Travel costs and entertainment costs can be either variable or fixed depending on the policy of management. The key question is whether it is necessary to incur additional travel and entertainment costs to generate added sales. 2-26 The key to determining cost behavior is to ask, “If there is a change in the level of the cost driver, will the total cost of the resource change immediately?” If the answer is yes, the resource cost is variable. If the answer is no, the resource cost is fixed. Using this question as a guide, the cost of labor can be fixed or variable as a function of the ..

37


number of hours worked. Regular wages may be fixed if there is a commitment to the laborers that they will be paid for normal hours regardless of the workload. However, overtime and temporary labor wages are variable. The depreciation on plant and machinery is not a function of the number of machine hours used and so this cost is fixed. 2-27

Suggested value chain functions are listed below.

New Products   

Marketing R&D Design

2-28

New Technology  

R&D Design

New Pricing 

Marketing

(10-15 min.)

Situation 1.

Best Cost Driver Number of Setups

2.

Setup Time

3.

Cubic Feet

4.

Cubic Feet Weeks

5.

Number of Orders

6.

Number of Orders

..

New Positioning Strategies  Marketing  Support functions

Justification Because each setup takes the same amount of time, the best cost driver is number of setups. Data is both plausible, reliable, and easy to maintain. Longer setup times result in more consumption of mechanics’ time. Simply using number of setups as in situation 1 will not capture the diversity associated with this activity. Assuming that all products are stored in the warehouse for about the same time (that is inventory turnover is about the same for all products), and that products are stacked, the volume occupied by products is the best cost driver. If some types of product are stored for more time than others, the volume occupied must be multiplied by a time dimension. For example, if product A occupies 100 cubic feet for an average of 2 weeks and product B occupies only 40 cubic feet but for an average of 10 weeks, product B should receive twice as much allocation of warehouse occupancy costs. Because each order takes the same amount of time, the best cost driver is number of orders. Data is both plausible, reliable, and easy to maintain. Each order is for different types of products but there is not diversity between them in terms of the time it takes to process the order. (If there was variability in the number of product types ordered, the best driver would be number of order line items.) 38


2-29

(5-10 min.)

1.

Contribution margin Net income

= $960,000 - $533,000 = $427,000 - $310,000

= $427,000 = $117,000

2.

Variable expenses Fixed expenses

= $550,000 - $300,000 = $300,000 - $ 46,000

= $250,000 = $254,000

3.

Sales Net income

= $500,000 + $520,000 = $520,000 - $200,000

= $1,020,000 = $320,000

2-30

(5-10 min.) The $278,000 annual advertising fee is a fixed cost. The $6,100 cost for each advertisement is a variable cost. If the total number of ads is 46 the total cost of advertising is $278,000 + 46 × $6,100 = $558,600 If the total number of ads is 92 the total cost of advertising is $278,000 + 92 × $6,100 = $839,200. The total cost of advertising does not double in response to a doubling of the number of ads because the fixed costs do not change.

2-31 (5-10 min.) With respect to the cost driver sales dollars, the $3,200,000 annual salaries of sales personnel is a fixed cost. The sales commissions, travel costs, and entertainment costs are variable costs. If the total sales dollars is $24 million, the total cost of the selling activity is $3,200,000 + .20 × $24,000,000 = $8,000,000 If the total sales dollars is only $12 million, the total cost of the selling activity is $3,200,000 + .20 × $12,000,000 = $5,600,000. The total cost of the selling activity does not decrease by 50% when the cost driver decreases by 50% because the fixed costs do not change.

..

39


2-32

(10-20 min.)

1.

d ₤720,000 b f

2.

d = c × (a – b) = 100,000 × (₤11 – ₤7) = ₤400,000 f =d–e = ₤400,000 – ₤300,000 = ₤100,000

3.

c = d ÷ (a – b) = ₤60,000 ÷ ₤3 = 20,000 units e =d–f = ₤63,000 – ₤15,000 = ₤48,000

4.

d = c × (a – b) = 60,000 × (₤25 – ₤15) = ₤600,000 e =d–f = ₤600,000 – ₤50,000 = ₤550,000

5.

d ₤160,000 a f

..

= c × (a – b) = 120,000 × (₤25 – b) = ₤19 =d–e = ₤720,000 – ₤650,000 = ₤70,000

= c × (a – b) = 80,000 × (a – ₤13) = ₤15 =d–e = ₤160,000 – ₤120,000 = $40,000

40


2-33 (10 min.)

Using the graph above, the estimated breakeven point in total units sold is about 75,000 (where revenue = ₤825,000 and total costs = ₤300,000 + ₤525,000 = ₤825,000). The estimated net income for 100,000 units sold is ₤100,000 (revenue of $1,100,000 – total costs of $300,000 + ₤700,000 = ₤1,000,000).

..

41


2-34 (10 min.)

Using the graph above, the estimated breakeven point in total units sold is about 55,000. The estimated net loss for 50,000 units sold is ₤50,000 (revenue of ₤1,250,000 – total cost of ₤1,300,000 or contribution margin of ₤500,000 – less fixed cost of ₤550,000).

..

42


2-35 (20–25 min.) Square Feet 100,000 125,000 150,000 175,000 200,000

Labor Cost per Square Foot $ 0.240 $ 0.192 $ 0.160 $ 0.137 $ 0.120

Labor Cost $24,000 24,000 24,000 24,000 24,000

Supplies Cost $ 5,000 6,250 7,500 8,750 10,000

Supplies Cost per Square Foot $0.050 0.050 0.050 0.050 0.050

Labor Cost per Square Foot L abo r Co st p er S qu are F oo t

$0.30 $0.25 $0.20 $0.15 $0.10 $0.05 $-

Fixed-Cost per Unit Behavior 100,000

125,000

150,000

175,000

200,000

Square Feet

S u p p lie s C o s t p e r S q u a re Foot

Supplies Cost per Square Foot $0.06 $0.05 $0.04 $0.03

Variable-Cost per Unit Behavior

$0.02 $0.01 $100,000

125,000

150,000

175,000

200,000

Square Feet ..

43


2-36 (20-25 min.) Square Feet

Labor Cost Per Square Foot (Estimated)

100,000 125,000 150,000

$0.12 0.096 0.08

Total Labor Cost

Supplies Cost per Square Foot

Supplies Cost

$12,000 12,000 12,000

$0.06 0.06 0.06

$ 6,000 7,500 9,000

Total Labor Cost $14,000

Labor Costs

$12,000 $10,000 $8,000 $6,000 $4,000 $2,000 $0 100,000

125,000

150,000

Square Feet

Supplies Cost

Total Supplies Cost $10,000 $9,000 $8,000 $7,000 $6,000 $5,000 $4,000 $3,000 $2,000 $1,000 $0 100,000

125,000

150,000

Square Feet

Labor cost shows a fixed-cost behavior, while supplies cost shows a variable-cost behavior.

..

44


2-37

(5 min.) Only (b) is a step cost. (a) This is a fixed cost. The same cost applies to all volumes in the relevant range. (b) This is a true step cost. Each time 200 new spectators come to buy tickets, the cost increases by the price of one additional counter. (c) This is a variable cost that remains fixed per unit of output. It is not a step cost. Why? Because each unit of the product requires a particular amount of cloth, regardless of the form in which the cotton cloth is purchased.

2-38

(5 min.) Here, the ₩12,000 is a fixed cost irrespective of the units consumed and the ₩103.60 per unit is the variable cost that varies directly with the number of units consumed. By definition, adding a fixed cost and a variable cost together produces a mixed cost.

2-39

(10-15 min.)

1.

Machining labor: G, number of units completed or labor hours

2.

Raw material: B, units produced; could also be D if the company’s purchases do not affect the price of the raw material.

3.

Annual wage: C or E (depending on work levels), labor hours

4.

Water bill: H, gallons of water used

5.

Quantity discounts: A, amount purchased

6.

Depreciation: E, capacity

7.

Sheet steel: D, number of farm implements of various types

8.

Salaries: F, number of solicitors

9.

Natural gas bill: C, cubic feet of usage

2-40

(10 min.)

1.

Let TR TR - .25(TR) -$45,000,000 .75(TR) TR

= total revenue = 0 = $45,000,000 = $60,000,000

2.

Daily revenue per patient = $60,000,000 ÷ 37,500 = $1,600. This may appear high, but it includes the room charge plus additional charges for drugs, x-rays, and so forth.

2-41

(10 min.)

1.

The break-even point in total revenue is fixed cost divided by the contributionmargin ratio (CMR). CMR equals 1 – Variable-Cost Ratio. Break-even Point = Fixed Cost ÷ CMR = $42,000,000 ÷ (1 – 0.7) = $140,000,000.

..

45


2. a.

b.

..

Total revenue Variable cost (.7 × 150,000,000) Contribution margin Fixed costs Net profit

$150,000,000 105,000,000 45,000,000 42,000,000 $ 3,000,000

Total revenue Variable cost (1.1 × .7 × 150,000,000) Contribution margin Fixed costs Net loss

$150,000,000 115,500,000 34,500,000 42,000,000 $ (7,500,000)

46


2-42

(15 min.)

1. Room revenue @ ₹5,000 Variable costs @ ₹4,200 Contribution margin Fixed costs Net income (loss)

a 100% Full

b 50% Full

₹219,000,000a 183,960,000 35,040,000 29,000,000 ₹ 6,040,000

₹ 109,500,000 b 91,980,000 17,520,000 29,000,000 ₹ (11,480,000)

a 120 × 365 = 43,800 beds per year 43,800 × ₹5000 = ₹219,000,000 b ₹219,000,000 ÷ = ₹109,500,000 2. Let N = Number of beds ₹5,000N – ₹4,200N – ₹29,000,000 = 0 N = ₹29,000,000 ÷ ₹800 = 36,250 beds Percentage occupancy = 36,250 ÷ 43,800 = 82.76% 2-43

(15 min.)

1.

$23. To compute this, let X be the variable cost that generates $1 million in profits: ($48 - X ) × 800,000 - $19,000,000 = $1,000,000 ($48 - X) = ($1,000,000 + $19,000,000) ÷ 800,000 $48 - X = $200 ÷ 8 = $25 X = $48 - $25 = $23

2.

Loss of $600,000: ($48 - $25) × 800,000 - $19,000,000 = ($23 × 800,000) - $19,000,000 = $18,400,000 - $19,000,000 = ($600,000)

..

47


2-44 1.

(15-20 min.) Let 2R = pints of raspberries and 5R = pints of strawberries sales - variable expenses - fixed expenses = zero net income ($1.05×5R) + ($1.30×2R) – ($.85×5R) – ($.90×2R) - $15,300 = 0 ($5.25 × R) + ($2.60 × R) – ($4.25 × R) – ($1.80 × R) -$15,300 = 0 $1.8 × R - $15,300 = 0 R = 8,500 2R = 17,000 pints of raspberries 5R = 42,500 pints of strawberries

2.

Let S = pints of strawberries ($1.05 - $.85) × S - $15,300 = 0 .20S - $15,300 = 0 S = 76,500 pints of strawberries

3.

Let R = pints of raspberries ($1.30 - $.90) × R - $15,300 = 0 ($.40 × R) - $15,300 = 0 R = 38,250 pints of raspberries

2-45 1.

(10 min.) ($1.50 × N) – ($1.20 × N) – $18,000 = $864 ÷ (1 - .25) ($.30 × N) = $18,000 + ($864 ÷ .75) ($.30 × N) = $18,000 + $1,152 N = $19,152 ÷ $.30 = 63,840 units

2.

($1.50 × N) – ($1.20 × N) - $18,000 = $1,440 ÷ (1 - .25) ($.30 × N) = $18,000 +( $1,440 ÷ .75) ($.30 × N) = $18,000 + $1,920 N = $19,920 ÷ $.30 = 66,400 units

..

48


2-46

(15 min.) Several variations of the following general approach are possible: Target after-tax net income Sales - Variable expenses - Fixed expenses = 1 - tax rate $136,000 S - .55S - $664,000 = (1 - .32) .45S = $664,000 + $200,000 S = $864,000 ÷ .45 = $1,920,000

Check:

..

Sales Variable expenses (55%) Contribution margin Fixed expenses Income before taxes Income taxes @ 32% Net income

$1,920,000 1,056,000 864,000 664,000 $ 200,000 64,000 $ 136,000

49


2-47 (40-50 min.) 1.

Several variations of the following general approach are possible:

Let N = Unit sales Sales – Variable expenses – Fixed expenses = Profit $4N – $2.80N – ($2,000 x 2 + $2,600) = $600 $1.20N – $6,600 = $600 N = $7,200 ÷ $1.20 = 6,000 boxes of doughnuts Check: Sales (6,000 × $4) Variable expenses (6,000 × $2.80) Contribution margin Fixed expenses Profit

$24,000 16,800 $7,200 6,600 $ 600

2. Let N = Unit Sales Sales – Variable expenses – Fixed expenses = Profit $4N – $2.80N – $6,600 = .10 × ($4N) $1.20N - $6,600 = $.40N N = $6,600 ÷ ($1.20 – $.40) = 8,250 boxes of doughnuts 3. Fixed cost ÷ (Sales price – Cost of meat – Cost of buns – Cost of other ingredients) = # of hamburgers $1,900 ÷ ($1.50 – $.40 – $.13 – $.21) = 2,500 hamburgers 4. (2,000 × $.76) + (4,000 × $1.20) – $1,900 = $1,520 + $4,800 – $1,900 = $4,420 added profit 5. $1,900 ÷ ($.76 + $1.20) = 969 new customers are needed to break-even on the new business. A sensitivity analysis would help provide Andrew with an assessment of the financial risks associated with the new hamburger business. Suppose that Terry is confident that the demand for hamburgers would range between break-even ± 500 new customers and that expected fixed costs will not change within this range. The contribution margin generated by each new customer is $1.96 so Andrew will realize a maximum loss or profit from the new business in the range ± $1.96 × 500 = ± $980. Another way to assess the financial risk that Andrew should be aware of is the company’s operating leverage (the ratio of fixed to variable costs). A highly leveraged company has relatively high fixed costs and low variable costs. Such a firm is risky because small ..

50


changes in volume lead to large changes in net income. This is good when volume increases but can be disastrous when volumes fall. 6. The additional cost of higher quality hamburger ingredients is .5 × $.74 = $.37. Any price for the higher quality hamburgers above the current price of $1.50 plus $.37, or $1.87, will improve profits, assuming the same number of customers purchase hamburgers.

..

51


2-48 (30-40 min.) 1. The cost of labor and equipment rent is fixed at ₹42,000 per month. The cost of cleaning supplies varies in proportion to the number of times the store is cleaned. The cost per cleaning is ₹24,000 ÷ 60 = ₹400. Number of Times the Store Is Cleaned 35 40 45 50 55 60

Labor & Rent Cost ₹42,000 42,000 42,000 42,000 42,000 42,000

Cleaning Supplies Cost at ₹400 per Cleaning ₹14,000 16,000 18,000 20,000 22,000 24,000

Total Cost ₹56,000 58,000 60,000 62,000 64,000 66,000

Cost per Cleaning ₹1,600.00 1,450.00 1,333.33 1,240.00 1,163.64 1,100.00

The total cost of cleaning for the next quarter is: Total cost = = = =

Total fixed cost + Total variable cost 3 × ₹42,000 + ((50 + 46 + 35) × ₹400 per cleaning) ₹126,000 + ₹52,400 ₹178,400

2. The graph is as follows:

..

52


3. Number of Times the Store Month Is Cleaned June 35 May 46 April 50

Costs of Suraksha Polyclinic Labor & Cleaning Rent Supplies Total Cost Cost Cost ₹42,000 ₹14,000 ₹56,000 42,000 18,400 60,400 42,000 20,000 62,000 ₹178,400

Outside Cleaning Cost ₹50,400 66,240 72,000 ₹188,640

Suraksha Polyclinic will save ₹188,640 – ₹178,400 = ₹10,240 by continuing to do its own cleaning rather than using the outside cleaning company, as shown in the above schedule. 2-49

(10-15 min.)

The budget for professional salaries for the coming year is $1,100,000. Refined analysis: Key professional salaries $1,200,000 1,100,000 1,000,000

$2,000,000

$2,400,000

Billings

$2,000,000

$2,400,000

Billings

Simplified analysis: Key professional salaries $1,200,000 1,100,000 1,000,000

Relevant Range ..

53


2-50

(15-20 min.)

1. Microsoft: ($60,420 - $11,598) ÷ $60,420 = .81 or 81% Procter & Gamble: ($83,503 - $40,695) ÷ $83,503) = .51 or 51% There is very little variable cost for each unit of software sold by Microsoft, as the variable cost percentage is only 19%. The variable cost percentage for the soap, cosmetics, foods, and other products of Procter & Gamble is much higher at 49%. 2. Microsoft: $10,000,000 × .81 = $8,100,000 Procter & Gamble: $10,000,000 × .51 = $5,100,000 3. We know that the total contribution margin generated by any added sales will be added to the operating income. Thus, we can simply multiply the contribution margin percentage by the changes in sales to get the change in operating income. The main assumption we make is that the sales volume remains in the relevant range so that total fixed costs do not change and unit variable cost remain unchanged. This generally means that such predictions will apply only to small changes in volume – changes that do not cause either the addition or reduction of capacity.

..

54


2-51

(15-20 min.) Film

1.

Revenue from admissions Variable costs Contribution margin Fixed costs: Auditorium rental Labor Operating income a b c

Refreshments

¥225,000 112,500 ¥112,500

¥31,500 a

17,325 ¥14,175

b c

¥33,000 43,500

Total ¥256,500 129,825 ¥126,675

76,500 ¥50,175

.50 × ¥225,000 = ¥112,500 .14 × ¥225,000 = ¥31,500 .55 × ¥31,500 = ¥17,325

Some labor might be exclusively devoted to refreshments. Labor might be allocated, but such a discussion is not the major point of this chapter. Film 2.

Revenue from admissions Variable costs Contribution margin Fixed costs: Auditorium rental Labor Operating income (loss)

Refreshments

¥140,000 75,000 ¥65,000 ¥33,000 43,500

¥19,600 a

10,780 ¥8,820

b c

Total ¥159,600 85,780 ¥73,820

76,500 ¥(2,680)

a Guarantee is ¥75,000 b .14 × ¥14,000 = ¥19,600 c .55 × ¥19,600= ¥10,780 3.

The offer would shift the risk completely to the movie producer, whereas ordinarily, the theatre owner bears a great deal of the risk. The owner is assured of a specified income; the producer then reaps the reward or bears the cost of the actual attendance level.

..

55


2-52

(15 min.)

1.

Let X = amount of additional fixed costs for advertising (1,300,000 × £15) +£270,000 -.20(1,300,000 × £15) - (£7,300,000 + X) = 0 £19,500,000 + £270,000 - £3,900,000 - £7,300,000 - X = 0 X = £19,770,000 - £11,200,000 X = £8,570,000

2.

Let Y = number of seats sold £15Y + £270,000 - .20 × £15Y - £11,000,000 = £490,000 £12Y = £11,220,000 Y = 935,000 seats

..

56


2-53 (45-55 min.) 1.

Exhibit A shows the relationships between the receiving activity and the resources used. This information can now be used for cost control purposes. Knowing the two rates, gallons per part received and machine hours operated per part received, will help operating managers predict costs. These rates are measures of productivity in the receiving department. Exhibit A

EQUIPMENT RESOURCE $45,000

1,500 Hours/30,000 Parts = 0.05 Hours/Part Received

FUEL RESOURCE $24,000 ÷ 6,000 Gal. = $4 Per Gallon Used

6,000 Gal./30,000 Parts = 0.2 Gal./Part Received

RECEIVING ACTIVITY Cost Driver Number of Parts Received, 30,000 2.

When the activity level increases, the use of resources will increase. Thus, the output measures or cost driver levels will increase – that is, total hours and total gallons. Normally, productivity rates such as gallons per part received and hours operated per part received will not change significantly unless a) there is action taken to improve efficiency, or b) factors act to decrease efficiency. An equation can be derived to predict total cost using the above concept. Total Cost = Variable Cost of Fuel + Fixed Cost of Equipment = (Number of Parts Received × Gallon/Part × Price/Gallon) + $45,000 The total cost of receiving 40,000 parts is (40,000 parts × 0.2 Gallon/Part × $4/Gallon) + $45,000 = $77,000

..

57


3.

The new fuel consumption rate will be .80 × 0.2 gallons/part received = 0.16 gallons per part received. The predicted cost of receiving 30,000 parts is (30,000 Parts × 0.16 Gallons/Part × $4.00 per Gallon) + $45,000 = $19,200 + $45,000 = $64,200. The receiving department will not achieve the 10% cost reduction goal of $62,100 even though productivity in fuel usage improved by 20%. Although the variable cost of fuel declines by 20%, the fixed cost of equipment does not decline at all, and the fixed equipment costs are a large portion of the total cost of receiving. Perhaps management should consider setting cost reduction goals in the light of knowledge of cost behavior.

4.

The new model contains productivity measures that are controllable by operating managers who are responsible for costs incurred. As a result, management can expect higher levels exerted effort by managers as well as improvement in cost control.

5.

One refinement is to note that total fuel usage is a function of both the efficiency in machine use as well as efficiency in fuel consumption. In terms of productivity metrics this can be expressed as follows:

Current model: Total Fuel Cost = $/Gallon × Gallons/Part Received × Total Number of Parts Received. Refined model: Total Fuel Cost = $/Gallon × Gallons Used/Operating Hour × Operating Hours/Part Received × Total Number of Parts Received. The refined model has two productivity measures instead of only one. Both these measures are controllable by operating managers in the receiving department. As a result, management can focus effort in two areas of potential improvement. For example, if there was a 20% improvement in both these productivity measures, the total fuel cost would be Total fuel cost = $4/Gal. × .8 × 4 Gal./Hour Operated × .8 × .05* Hours Operated/Part Received × 30,000 Parts Received = $15,360. * 1,500 operating hours/30,000 parts received The predicted total cost of receiving would then be $15,360 + $45,000 = $60,360 and the target goal would be achieved.

..

58


2-54

(20-30 min.) Many shortcuts are available, but this solution uses the equation technique.

1.

Let N = Sales $18N N= N=

meals sold Variable expenses - Fixed expenses = Profit before taxes $9.50N - $17,000 = $8,500 $25,500 ÷ $8.50 3,000 meals

2.

$18N - $9.50N - $17,000 = $0 N = $17,000 ÷ $8.50 N = 2,000 meals

3.

$22N - $11.40N - $25,420 = $8,500 N = $33,920 ÷ $10.60 N = 3,200 meals

4.

Profit = ($22 × 2,550) – ($11.40 × 2,550) - $25,420 Profit = $1,610

5.

Profit = ($22 × 2,800) –($11.40 × 2,800) - ($25,420 + $2,300) Profit = $29,680 - $27,720 Profit = $1,960, an increase of $350. A shortcut, incremental approach follows: Increase in contribution margin, 250 × $10.60 Increase in fixed costs Increase in profit

..

$2,650 2,300 $ 350

59


2-55

(10-15 min.)

1. Contribution margin per unit = $40 – $20 = $20 Annual fixed cost = $10,000 Break-even point = $10,000 ÷ $20 = 500 units 2. Target profit (before tax) = Target profit after tax ÷ (1 – Tax rate) = $24,000 ÷ 0.60 = $40,000 Target sales =

=

= 2,500 units

During the first five months, only 350 units of the target have been sold. So, the company needs to sell further (2,500 – 350) = 2,150 units in the remaining seven months. 3. Calculation of total profit under different alternatives:

a. Contribution margin during first five months ($20 x 350)

A

B

C

$7,000

$7,000

$7,000

$36.0

$37.0

$38.0

($40 – $4)

($40 – $3)

($40 – 5% of $40)

20.0

17.5

20.0

b. Contribution from remaining months Selling price per unit

Variable cost per unit

($20 – $2.5)

Contribution margin per unit

$16.0

$19.5

$18.0

Expected sales unit

2,700

2,200

2,100

Total contribution

$43,200

$42,900

$37,800

c. Total Contribution for the entire year (a + b)

$50,200

$49,900

$44,800

d. Fixed costs

$10,000

$10,000

$9,000

$39,900

($10000 – $1000) $35,800

e. Expected profit (c – d)

$40,200

The company should choose alternative A.

..

60


2-56

(10-15 min.) Amounts are in millions (rounded with slight rounding errors). Net sales (.9 × $82,559) Variable costs: Cost of goods sold (.9 × $40,768) Contribution margin Fixed costs: Selling, administrative, and general expenses Operating income

$74,303 36,691 37,612 25,973 $11,639

The percentage decrease in operating income would be 1 – ($11,639  $15,818) = 1 – .736 or 26.4%, compared with a 10% decrease in sales. The contribution margin would decrease by 10% or .10 × ($82,559 - $40,768) = $4,179 million. Because fixed costs would not change (assuming the new volume is within the relevant range), operating income would also decrease by $4,179 million, from $15,818 million to $11,639 million. Because of the existence of fixed costs, the percentage decrease in operating income will exceed the percentage decrease in sales. If all costs had been variable, fixed costs would have decreased by an additional .10 × $25,973 = $2,597 million, making operating income $11,639 + $2,597 = $14,236 million, a 10% decrease from the 2011 operating income of $15,818 million.

..

61


2-57

(15-25 min.)

1.

Average revenue per person Total revenue, 200 @ €20.00 = Rent Total available for prizes and operating income

€16.00 + 4(€1.00) = €20.00 €4,000 1,900 €2,100

The church could award cash prizes of €2,100 and break-even. 2.

Number of persons Total revenue @ €20.00 Fixed costs: Rent Prizes Operating income (loss)

€1,900 2,100

60 €1,200

200 €4,000

400 €8,000

4,000 €(2,800)

4,000 €0

4,000 €4,000

Note how “leverage” works. Being highly leveraged means having relatively high fixed costs. In this case, there are no variable costs. Therefore, the revenue is the same as the contribution margin. As volume departs from the break-even point, operating income is affected at a significant rate of €20 per person. 3.

Number of persons Revenue Variable costs Contribution margin Fixed costs: Rent Prizes Operating income (loss)

€1,400 2,100

60 €1,200 120 € 1,080

200 €4,000 400 €3,600

400 €8,000 800 €7,200

3,500 €(2,420)

3,500 €100

3,500 €3,700

The risk is lower with the revised cost structure because of lower operating leverage—fixed costs are lower and variable costs are higher. Some of the risk has been shifted to the hotel. As a result, when attendance is low, the club will not lose as much money, and when attendance is high, the club will not make as much money. For example, the income at 400 persons is €3,700 versus €4,000 and the loss at 60 persons is €(2,420) instead of €(2,800).

..

62


2-58 (10-20 min.) 1. To compute eBay’s operating income, we need to know fixed and variable costs. We are given that its fixed costs are $37 million. Variable costs in the first quarter of 2001 were: Operating expenses - Fixed costs $123 million - $37 million

= Variable costs = $86 million

Variable costs were $86 million ÷ $154 million = 55.84% of sales. If this percentage also applied to 2002, variable costs in 2002 should have been 55.84% × $245 million = 136.8 million. Since sales increased by 1 – ($245 million ÷ $154 million) = 59.09% in 2002, variable costs should also have increased by 59.09%: 2002 variable costs = 1.5909 × $86 million = $136.8 million Therefore, we calculate 2002 operating income as follows: operating income = revenues – variable cost – fixed cost =$245 million - $136.8 million - $37 million = $71.2 million This is a 130% increase in operating income: ($71.2 million ÷ $31 million) – 1 = 130% 2. When sales increased 59%, operating income increased by 130%. This is an example of the effect of operating leverage. The variable cost percentage is approximately $86 ÷ $154 = 56%. Thus, the contribution margin percentage is 100% - 56% = 44%. Every dollar of sales generates $.44 of operating income. The sales increase of $245 million $154 million = $91 million generated $91 million × 44% = $40 million of operating income, while the original $154 million of sales had generated only $31 million of operating income. This is because the same $37 million of fixed cost applied at both level of sales. The additional $91 million of sales caused no additional fixed costs, so its total contribution margin all becomes operating income.

..

63


2-59

(15-25 min.)

1.

Let N = Number of plates of fish and chips per month AUD 4.50N = AUD 3.70N + AUD 1,800 AUD .80N = AUD 1,800 N = 2,250 per month, or 2,250 ÷ 30 = 75 per day

2.

Multiply the answers in (1) by AUD 4.50 2,250 × AUD 4.50 = AUD 10,125 per month 75 × AUD 4.50 = AUD 337.50 per day

3.

Fish and chips plates per month, 3,200 ÷ 2 Revenue per month, 1,600 × AUD 4.50 Variable expenses, 1,600 × AUD 3.70 Contribution margin, 1,600 × AUD .80 Fixed expenses Operating income (loss)

4.

Contribution margin on extra meat pies:

1,600 AUD 7,200 5,920 AUD 1,280 1,800 AUD (520)

Per day: 80 × AUD .60 = AUD 48 Per month, 30 × AUD 48 = AUD 1,440 Income from additional sales of meat pies would increase by AUD 1,440, which more than offsets the AUD 520 loss on the fish and chips sales, making the net effect on operating income of selling fish and chips and selling additional meat pie because of the meat pie sales equal to AUD – 520 + AUD 1,440 = AUD 920. 5.

Operating loss on fish and chips Desired contribution margin on extra meat pies Overall effect on operating income

AUD (520) 520 AUD 0

Desired number of extra meat pies to provide an overall effect on operating income of zero: Per month = AUD 520 ÷ .60 = 867 meat pies Per day = 867 ÷ 30 = 29 meat pies Or, desired contribution margin per day is AUD 520 ÷ 30 = AUD 17.3 Daily number of meat pies = AUD 17.3 ÷ AUD .60 = 29 ..

64


Therefore, if Ethan believes that the extra meat pies sold amount to 29 daily instead of 80, the fish and chips operation would provide an overall effect on operating income of zero.

..

65


2-60 (15-20 min.) Note how, in requirements 2 and 3, the percentage declines exceed the 15% budget reduction. 1.

Let N = Number of orphanages Revenue – Variable expenses – Fixed expenses = 0 ₤1,800,000 – ₤10,000N – ₤560,000 = 0 ₤10,000N = ₤1,800,000 – ₤560,000 N = ₤1,240,000 ÷ ₤10,000 N = 124 orphanages

2.

Revenue is now .85(₤1,800,000) = ₤1,530,000 Let N = Number of orphanages Revenue – Variable expenses – Fixed expenses = 0 ₤1,530,000 – ₤10,000N – ₤560,000 = 0 ₤10,000N = ₤1,530,000 – ₤560,000 N = ₤970,000 ÷ ₤10,000 N = 97 orphanages Percentage drop: (124 – 97) ÷ 124 = 21.8%

3.

Let y = Monetary support per orphanage ₤1,530,000 – 124y – ₤560,000 = 0 124y = ₤1530,000 – ₤560,000 y = ₤970,000 ÷ 124 y = ₤7,822 Percentage drop: (₤10,000 – ₤7,822) ÷ ₤10,000 = 21.8% Regarding requirements 2 and 3, note that the cut in service can be measured by a formula: % cut in service = % budget change ÷ % variable cost The variable cost ratio is ₤1,240,000 ÷ ₤1,800,000 = 68.9% % cut in service = 15% ÷ 68.9% = 21.8%

..

66


2-61

(15-20 min.) Answers are in millions.

1.

Sales Variable costs: Variable costs of goods sold Variable other operating expenses Contribution margin

$6,022 $3,735 487

4,222 $ 1,800

Contribution margin percentage = $1,800  $6,022 = 29.9% The contribution margin equals sales less all variable costs, while gross margin equals sales less cost of goods sold. The variable costs include part of the costs of goods sold and also part of the other operating costs. Note that contribution margin can be either larger than or smaller than the gross margin. If most of the cost of goods sold and a good portion of the other operating costs are variable, then variable costs may exceed the cost of goods sold, and the contribution margin will be smaller than the gross margin. However, if a large portion of both the cost of goods sold and the other expenses are fixed, cost of goods sold may exceed the variable cost, resulting in the contribution margin exceeding gross margin. 2.

Predicted sales increase = $6,022 × .10 = $602.20 Additional contribution margin = $602.20 × .299 = $180 Fixed costs do not change Predicted operating loss = $(600) + $180 = $(420) Percentage decrease in operating loss = $180  $(600) = 30% Note that a 10% increase in sales would decrease the operating loss by 30%.

3.

Assumptions include:  Expenses can be classified into variable and fixed categories that completely describe their behavior within the relevant range.  Costs and revenues are linear within the relevant range.  Predicted sales volume is within the relevant range.  Efficiency and productivity are unchanged.  Sales mix is unchanged.  Changes in inventory levels are insignificant.

..

67


2-62

(20-30 min.)

Variable costs per box are ($.15 + $.10 + $.20), ($.15 + $.10 + $.12), and ($.15 + $.10 + $.03), or $.45, $.37, and $.28, respectively 1.

Let N = Volume level in packets that would earn same profit $15,680 + $.45N = $22,400 + $.37N $.08N = $6,720 N = 84,000 packets

2.

As volume increases, the more expensive models would generate more profits. Compare the super and supreme models: Let N = Volume level in packets that would earn same profit $40,400 + $.28N = $22,400 + .37N $.09N = $18,000 N = 200,000 packets Therefore, the decision rule is as shown below. Anticipated Annual Sales Between 0 – 84,000 84,000 – 200,000 200,000 and above

Use Model Standard Super Supreme

The decision rule places volume well within the capacity of each model.

..

68


2-63

(10-15 min.)

1. Nestle has the higher fixed cost, while Sicca has the higher variable cost. Thus, the contribution margin for Nestle will be higher. Nestle will have more risk. Its profits increase faster as sales increase, but its profits decrease faster (or losses increase faster) as sales decrease. 2. Sicca provides more incentive to its sales force to increase sales. For each $1 of increased sales, Sicca pays more of that increase to the sales force, while Nestle retains more of the increase for the company’s profit. 3. A possible negative of the increased incentive for the Sicca sales force to increase sales is a motivation to increase those short-term sales at any cost. That is, the Sicca sales force might be motivated to sell customers product they don’t need or to record sales that are not yet final. Many companies have found that too much emphasis on sales volumes can cause managers to take unethical actions to increase their sales levels.

..

69


2-64

(20-25 min.)

1.

Net income (loss)

2.

Let B 2B

= (200,000 × $1) + (100,000 × $2) - $680,000 = $200,000 + $200,000 - $680,000 = $(280,000)

= number of units of beef enchiladas to break even = number of units of chicken tacos to break even (C)

Total contribution margin - fixed expenses = zero net income ($2 × B) + ($1 × 2B) - $680,000 $4 ×B B 2B

=0 = $680,000 = 170,000 = 340,000 = C

The break-even point is 170,000 units of beef enchiladas plus 340,000 units of chicken tacos, a grand total of 510,000 units. 3.

If tacos, break-even would be $680,000 ÷ $1 = 680,000 units. If enchiladas, break-even would be $680,000 ÷ $2 = 340,000 units.

4.

Net income (loss)

= (225,000 × $1) + (75,000 × $2) - $680,000 = $225,000 + $150,000 - $680,000 = $(305,000)

Let B = number of units of beef enchiladas to break even 3B = number of units of chicken tacos to break even (C) Total contribution margin - fixed expenses = zero net income ($2 × B) + ($1 × 3B) - $680,000 $5 × B B 3B

=0 = $680,000 = 136,000 = 408,000 = C

The major lesson of this problem is that changes in sales mix affect break-even points and net incomes.

..

70


2-65 1.

(20-25 min.)

Let S = number of self-pay patients (S) 3S = number of other patients (G) ($1,250 × S) + ($950 × 3S) – ($750 × S) – ($750 × 3S) - $52,800,000 ($1,250 × S) + ($2,850 × S) – ($750 × S) – ($2,250 × S) $1,100 × S S 3S

=0 = $52,800,000 = $52,800,000 = 48,000 = 144,000 = G

The break-even point is 48,000 self-pay patient days plus 48,000 × 3 = 144,000 other patient days, a grand total of 192,000 patient days. 2.

Contribution margins: S = $1,250 - $750 = $500 per patient day G = $950 - $750 = $200 per patient day Patient days: S = .40 × 172,000 = 68,800 G = .60 × 172,000 = 103,200 Net income = (68,800 × $500) + (103,200 × $200) - $52,800,000 = $34,400,000 + $20,640,000 - $52,800,000 = $2,240,000 Let S = number of self-pay patients (S) 1.5S = number of other patients (G)

($1,250 × S) + ($950 ×1.5S) – ($750 × S) – ($750 ×1.5S) - $52,800,000 = 0 ($1,250 × S) + ($1,425 × S) – ($750 × S) – ($1,125 × S) = $52,800,000 $800 × S = $52,800,000 S = 66,000 1.5S = 99,000 = G The break-even point is now lower (66,000 + 99,000 = 165,000 patient days instead of 48,000 + 144,000 = 192,000 patient days).

..

71


2-66

(15-25 min.)

1.

Let N = number of rooms $801,000 (1 - .25) ($48 × N) - $8,700,000 = $1,068,000 $48 ×N = $9,768,000 N = 203,500 rooms

($90 × N) – ($42 × N) - $8,700,000 =

$400,500 (1 - .25) ($48 × N) - $8,700,000 = $534,000 $48 × N = $9,234,000 N = 192,375 rooms

($48 × N) - $8,700,000 =

2.

($90 × N) – ($42 × N) - $8,700,000 = 0 $48 × N = $8,700,000 N = 181,250 rooms Number of rooms at 100% capacity = 570 × 365 = 208,050 Percentage occupancy to break even = 181,250 ÷ 208,050 = 87.1%

3.

Using the shortcut approach described in the chapter appendix: Change in net income = Change in vol. in units × Cont. margin/unit × (1 - tax rate) = 6,000 × $48 × (1 - .25) = 6,000 × $36 = $216,000. Note that a 3% increase in rooms rented increased net income by $216,000 ÷ $675,000 or 32%. Rooms rented Contribution margin @ $48 Fixed expenses Income before taxes Income taxes @ 25% Net income Increase in net income Percentage increase

..

200,000 $9,600,000 8,700,000 900,000 225,000 $ 675,000

206,000 $9,888,000 8,700,000 1,188,000 297,000 $ 891,000

$216,000 32%

72


2-67

(15-25 min.)

Current contribution margin = $15 - $8 - $4 = $3. New variable costs per disk will be (125% x $8 ) + $4 = $10 + $4 = $14. $714,000 $15- ($8 + $4)

1.

Break-even point =

= 238,000 CDs

2.

Contribution margin: $15 - ($8 + $4) = $3 Increased after-tax income after 15% increase in volume: Change in net income = Increase in vol. in units × Cont. margin/unit × (1 - tax rate) = 25,500 × $3 × (1 - .40) = $45,900 increase in income

3.

Let N = target sales in units

25% increase in unit purchase price will increase purchase price to $10 (from previous value of $8) so variable costs per unit will be $10 + $4 = $14. target after-tax net income Target sales – Variable costs – Fixed costs = 1 - tax rate ($15 × N) – ($14 × N) - $714,000 = $90,000 ÷ (1 - .4) ($15 × N) – ($14 × N) - $714,000 = $150,000 $1 × N = $864,000 N = 864,000 units $15 × N = $12,960,000 4.

..

Let P = new selling price Current contribution ratio is $3 ÷ $15 New contribution ratio is (P - $14) ÷ P .20P .80P P P

= .20 = .20 = P - $14 = $14 = $14 ÷ .80 = $17.5

73


2-68

(25-35 min.)

1. Calculation for profit for the year 2020 Charges collected from patients

BDT6,991,250

Less: Variable cost of the center

2,714,250

Contribution

4,277,000

Less: Fixed costs Fixed costs of the center

1,245,000

Apportionment of general administration charges

2,000,000

Salary of the Nurses (6 x 60,000)

360,000

Profit

3,605,000 672,000

Note: Annual bed days for 2020 = BDT6,991,250 ÷ 425 = 16,450. So, six nurses had been employed in 2020. Calculation for profit for the year 2020 Charges collected from patients (occupancy being unchanged)

BDT6,991,250

Less: Variable cost of the center (2,714,250 x 110%)

2,985,675

Contribution

4,005,575

Less: Fixed costs

Profit

Fixed costs of the center (1,245,000 x 110%)

1,369,500

Apportionment of general administration charges

2,500,000

Salary of the Nurses (6 x 72000)

432,000

4,301,500 (295,925)

Thus, at the same level of occupancy with similar charges per bed day, the center will incur a loss of BDT295,925. 2. Contribution margin per bed day for 2021 = BDT4,005,575 ÷ 16,450 = BDT243.50 Break-even bed capacity = Fixed cost ÷ contribution margin per bed day = BDT4,301,500 ÷ BDT243.50 = 17,665 bed days

..

74


3. The management will be ready to run the center only if they can fully recover the fixed costs of the center by the contribution available at the current occupancy level. Current occupancy level = 16,450 bed days. Target contribution per bed day = BDT4,301,500 ÷ 16,450 = BDT261.49 Variable cost per bed day = 2,985,675 ÷ 16,450 = BDT181.50 Target charges per bed day = BDT181.50 + BDT261.49 = BDT442.99 So, the increase in charges required = BDT442.99 – BDT425.00 = BDT17.99

..

75


2-69

(15-20 min.)

1.

Old: (Contribution margin × 600,000) - $580,000 = Budgeted profit [($3.10 - $2.10) × 600,000] - $580,000 = $20,000 New: (Contribution margin × 600,000) - $1,140,000 = Budgeted profit [($3.10 - 1.10) × 600,000] - $1,140,000 = $60,000

2.

Old: $580,000 ÷ $1.00 = 580,000 units New: $1,140,000 ÷ $2.00 = 570,000 units

3.

A fall in volume will be more devastating under the new system because the high fixed costs will not be affected by the fall in volume: Old: ($1.00 × 500,000) - $580,000 = –$80,000 (an $80,000 loss) New: ($2.00 × 500,000) - $1,140,000 = –$140,000 (a $140,000 loss) The 100,000 unit fall in volume caused a $20,000 - (- $80,000) = $100,000 decrease in profits in the old environment and a $60,000 - ( - $140,000) = $200,000 decrease in the new environment.

4.

Increases in volume create larger increases in profit in the new environment: Old: ($1.00 × 700,000) - $580,000 = $120,000 New: ($2.00 × 700,000) - $1,140,000 = $260,000 The 100,000 unit increase in volume caused a $120,000 - $20,000 = $100,000 increase in profit under the old environment and a $260,000 - $60,000 = $200,000 increase under the new environment.

5.

Changes in volume affect profits in the new environment (a high fixed cost, low variable cost environment) more than they affect profits in the old environment. Therefore, profits in the old environment are more stable and less risky. The higher risk new environment promises greater rewards when conditions are favorable, but also leads to greater losses when conditions are unfavorable, a more risky situation.

..

76


2-70 (25-30 min.) This case is based on real data that has been simplified so that the numbers are easier to handle. 1.

Daily break-even volume is 85 dinners and 170 lunches: First compute contribution margins on lunches and dinners: Variable cost percentage = ($1,246,500 + $222,380) ÷ $2,098,400 = 70% Contribution margin percentage = 1 - variable cost percentage = 1 - 70% = 30% Lunch contribution margin = .30 × $20 = $6 Dinner contribution margin = .30 × $40 = $12 Annual fixed cost is $170,940 + $451,500 = $622,440 Let X = number of dinners and 2X = number of lunches ($12×X) + ($6×2X) - $622,440 = 0 $24(X) = $622,440 X = 25,935 dinners annually to break even 2X = 51,870 lunches annually to break even On a daily basis: Dinners to break even = 25,935 ÷ 305 = 85 dinners daily Lunches to break even = 85 × 2 = 170 lunches daily or 51,870 ÷ 305 = 170 lunches daily. To determine the actual volume, let Y be a combination of 1 dinner and 2 lunches. The price of Y is $40 + (2 × $20) = $80, and total volume in units of Y is $2,098,400 ÷ $80 = 26,230 and daily volume is 26,230 ÷ 305 = 86. Therefore, 86 dinners and 2 × 86 = 172 lunches were served on an average day. This is 1 dinner and 2 lunches above the break-even volume.

2.

The extra annual contribution margin from the 3 dinners and 6 lunches is: 3 × $40 × .30 × 305 = $10,980 + 6 × $20 × .30 × 305 = 10,980 Total $21,960 The added contribution margin is greater than the $15,000 advertising expenditure. Therefore, the advertising expenditure would be warranted. It would increase operating income by $21,960 - $15,000 = $6,960.

..

77


3.

Let Y again be a combination of 1 dinner and 2 lunches, priced at $80. Variable costs are .70 × $80 = $56, of which $56 × .25 = $14 is food cost. Cutting food costs by 20% reduces variable costs by .20 × $14 = $2.80, making the variable cost of Y $56 - $2.80 = $53.20 and the contribution margin $80 - $53.20 = $26.80. (This could also be determined by adding the $2.80 saving in food cost directly to the old contribution margin of $24.) The required annual volume in Y needed to keep operating income at $7,080 is: $26.80 (Y) - $622,440 = $7,080 $26.80 (Y) = $629,520 Y = 23,490 Therefore, daily volume = 23,490 ÷ 305 = 77 (rounded) If volume drops no more than 86 - 77 = 9 dinners and 172 - 154 = 18 lunches, using the less costly food is more profitable. However, there are many subjective factors to be considered. Volume may not fall in the short run, but the decline in quality may eventually affect repeat business and cause a long-run decline. Much may depend on the skill of the chef. If the quality difference is not readily noticeable, so that volume falls less than, say, 10%, saving money on the purchases of food may be desirable.

..

78


2-71

(25-30 min.)

1.

Break-even in pounds = Annual fixed costs ÷ Contribution margin/pound $566,250 = = 283,125 pounds (5.00 - $3.00)

2.

Contribution margin ratio = $2.00 ÷ $5.00 = 40% Old variable cost = $3.00 Only the cost of salmon is affected: New variable cost = $3.00 + (.15 ×$2.50) = $3.375 Let S Selling price - Variable costs (S - $3.375) .60S S

= Selling price = Contribution margin = .40S = $3.375 = $5.625

Check: ($5.625 - $3.375) ÷ $5.625 = 40% 3.

Current income before taxes: = 390,000 × ($5.00 - $3.00) - $566,250 = $780,000 - $566,250 = $213,750 Current income after taxes: = $213,750 × .60 = $128,250 The problem can be solved by using units and then converting to dollar sales. Let N = sales in pounds Net income 1 - tax rate =$128,250 ÷ (1 - .4) = $213,750 = $780,000 = 480,000 pounds = $2,400,000 sales

Sales - Variable expenses - Fixed expenses = ($5.00 × N) – {[($3.00 + .15×$2.50)] × N} - $566,250 ($5.00 × N) – ($3.375 × N) - $566,250 $1.625 × N N $5.00 × N

An alternative way to get the solution is: New contribution margin ratio = ($5.00- $3.375) ÷ $5.00 = .325 New variable-cost ratio = 1.000 - .325 = .675 Let S = Sales S = (.675 × S) + $566,250 + [$128,250 ÷ (1 - .4)] .325 × S = $780,000 S = $2,400,000 ..

79


4.

Strategies might include: (a) Increase selling price by the $.375 cost increase. (b) Decrease other variable costs by $.375 per pound. (c) Decrease fixed costs by $.375 × 390,000 = $146,250. (d) Increase unit sales by 480,000 - 390,000 = 90,000 pounds. (e) Some combination of the above.

2-72 (15-20 min.) 1.

The following table shows the comparison between percentage changes in total revenue and income before taxes for the six major regions of Nike.

Percent Change Region in Revenue North America 13% Western Europe –2% Central & Eastern Europe 4% Greater China 18% Japan –13% Emerging Markets 24%

Percent Change in Pre-tax Income 14% –16% –8% 22% –37% 32%

The term operating leverage means that a substantial portion of the resources used to generate income were fixed-cost resources and did not increase in response to increased revenue-generating activities. As a result, income changes more than proportional to the change in revenue. 2.

There are many possible explanations. One possibility is that while revenues increased, variable costs may have increased so that the overall contribution margin fell, resulting in a decrease in income. Another possibility is that even if variable costs did not increase, fixed costs may have increased by an amount that more than offset the increased contribution from increased sales.

3.

Nike’s operating leverage is the ratio of its fixed costs to variable costs. A large percentage of Nike’s costs is cost of goods sold, which is primarily a variable cost, making operating leverage low. However, Nike also has many fixed costs. Many of Nike’s fixed costs are related to its distribution function. The costs of the distribution center, equipment, and salaries of regular employees and management all contribute to a substantial fixed-cost component of total cost. Another significant component of fixed costs is the Nike World Campus in Beaverton, Oregon with 16 buildings and almost 6,000 management staff.

2-73

(30-40 min.) For the solution to this Excel Application Exercise, follow the stepby-step instructions provided in the textbook chapter. Answers to the questions follow: Proposal A: Break even in units: $110,000 ÷ ($99 - $55) = 2,500 units Break even in dollars: 2,500 × $99 = $247,500

1.

Proposal B: ..

80


Break even in units: $110,000 ÷ ($129 - $55) = 1,486 units Break even in dollars: 1,486 × $129 = $191,694 Proposal C: Break even in units: $110,000 ÷ ($99 - $49) = 2,200 units Break even in dollars: 2,200 × $99 = $217,800 2.

The break-even points are much smaller because the contribution margin is larger while the fixed costs are unchanged.

3.

The increase in contribution margin was not nearly as large, $6 in proposal C compared to $30 in proposal B.

2-74

(30 min. or more) The purpose of this problem is to develop an intuitive feel for the costs involved in a simple production process and to assess whether various costs are fixed or variable. Then students must assess the market to determine a price so that they can compute a break-even point. Completing this problem can be done quickly or it can take much time. It might even be done in class, with students suggesting the various costs and predicting their levels. A complete analysis might involve finding the actual prices of the resources needed to make the product or service. This could lead to time-consuming research. Whatever approach is taken, students are led to see the real-world application of what they are learning.

..

81


2-75 (30-40 min.) NOTE TO INSTRUCTOR: This solution is based on the web site as it was in early 2012. Be sure to examine the current web site before assigning this problem, as the information there may have changed. 1.

Answers to the questions depend on the student's location and choices of dates. Fares available include business select, anytime, and “wanna get away”. Different fares are offered because of the different costs incurred by SWA to serve customers who have different flying needs. Another factor causing different fares is the need to match products offered by competing airlines. Restrictions such as the requirement to make reservations at least 7 days in advance of travel are necessary to give SWA planning information in advance. Limiting the number of reduced-price wanna get away fares on each flight is necessary in order to keep open seats for customers who must travel on short notice.

2.

It is likely that the fares one week in advance are higher than the fares one month in advance. Customers who need to travel with short notice are willing to pay more. Many business travelers fly with very short notice.

3.

On a particular flight, price paid for a seat (assuming the same class seat) is not a cost driver. The various costs incurred by SWA will change only slightly – possibly the type of food served will vary as a function of the price paid for a seat on a particular trip, but almost all the other costs are independent of the price paid for the seat.

4.

Operating revenues and operating expenses are reported for the current and prior year along with the percentage change. The operating revenues increased from $12.104 billion in 2010 to $15.658 billion in 2011, an increase of 29.4%. Operating expenses increased from $11.116 billion in 2011 to $14.965 billion in 2011, an increase of 34.6%. With expenses rising faster than revenues, profits will fall – as shown by the 29.8% decrease in operating income.

5.

To describe a particular cost as fixed or variable, we must identify the cost driver, the time period involved, and the relevant range. In this case, assume that the period is one year and the relevant range is the number of ASMs that can be available without adding to or subtracting from the current fleet of airplanes. Thus, adding ASMs means flying the existing airplanes for more hours. Costs that would probably vary with ASMs are salaries, wages, and benefits, employee retirement plans, fuel and oil, maintenance materials and repairs, landing fees and other rentals. Aircraft rentals and depreciation would probably be fixed costs. Some of these costs might be more directly caused by other cost drivers. For example, revenue passenger miles (RPM), that is number of passengers times the miles each flies, might drive agency commissions and possibly some salaries (for example, flight attendants whose number depends on how many passengers are on a particular flight).

..

82


CHAPTER 3 COVERAGE OF LEARNING OBJECTIVES

LEARNING OBJECTIVE LO1: Explain management influences on cost behavior. LO2: Measure and mathematically express cost functions and use them to predict costs. LO3: Describe the importance of activity analysis for measuring cost functions. LO4: Measure cost behavior using the engineering analysis, account analysis, highlow, visual-fit, and least-squares regression methods.

..

CASES, NIKE 10K, EXCEL, FUNDAMENTAL ADDITIONAL COLLAB., & ASSIGNMENT ASSIGNMENT INTERNET MATERIAL MATERIAL EXERCISES B2

10, 26, 33, 33, 39, 52

52, 56

B2

27, 29, 30, 30, 31, 31, 34, 35, 36, 37, 37, 38, 39, 40, 40, 41, 44, 44, 46, 46, 48, 50, 51

57, 56, 58

A1, B1, B2

42, 42, 45

53, 55

A2, B2, B2

20, 28, 29, 30, 30, 34, 35, 36, 37, 37, 38, 43, 46, 46, 47, 48, 49, 51

54, 58

83


CHAPTER 3 Measurement of Cost Behavior

3-A1

(25-30 min.)

1.

Support costs based on 75% of the cost of materials: Direct materials cost Support cost (75% of materials cost)

Sign A $300 $225

Sign B $800 $600

Sign A 10 $700

Sign B 2 $140

Support costs based on $70 per power tool operation: Power tool operations Support cost 2.

If the activity analysis is reliable, by using the current method, Dogwood Signs is predicting too much cost for signs that use few power tool operations and is predicting too little cost for signs that use many power tool operations. As a result the company could be losing jobs that require few power tool operations because its bids are too high -- it could afford to bid less on these jobs. Conversely, the company could be getting too many jobs that require many power tool operations, because its bids are too low -- given what the "true" costs will be, the company cannot afford these jobs at those prices. Either way, the sign business could be more profitable if the owner better understood and used activity analysis. Dogwood Signs would be advised to adopt the activityanalysis recommendation, but also to closely monitor costs to see if the activityanalysis predictions of support costs are accurate.

..

84


3-A2 (25-30 min.) 1.

High-Low Method: High month = May Low month = September Difference

Support Cost $22,000 18,000 $ 4,000

Machine Hours 1,700 1,300 400

Variable cost per machine hour = Change in cost ÷ Change in cost driver = $4,000 ÷ 400 = $10.00 Fixed support cost per month = Total support cost - Variable support cost At the high point: = $22,000 - $10.00 × 1,700 = $22,000 - $17,000 = $ 5,000 or at the low point:

= $ 18,000 - $10.00 × 1,300 = $ 18,000 - $13,000 = $ 5,000

2.

The high-low method uses the high and low activity levels to determine the cost function. Since the new October data for machine hours does not change either the high or low level there would be no change in the analysis.

3.

The regression analysis results differ from the results of the high-low method. As a result, estimates of total support cost may differ considerably depending on the expected machine hour usage. For example, consider the following support cost estimates at three levels of machine hour usage (all within the relevant range): Machine Hour Usage High-Low: Fixed Variable:

1,500 Hours

1,600 Hours

$5,000 14,000

$ 5,000

$ 5,000

15,000

Total

$19,000

$20,000

16,000 $21,000

Regression: Fixed Variable:

$4,050 14,700

$ 4,050

$ 4,050

Total ..

$10.00 × 1,400 $10.00 × 1,500 $10.00 × 1,600

1,400 Hours

$10.50 × 1,400 $10.50 × 1,500 $10.50 × 1,600

15,750 $18,750

$ 19,800

16,800 $20,850 85


Because the high-low method has a lower variable cost estimate and a higher fixed cost estimate than the regression-based predictions, the estimates of total support cost differ depending on the expected machine hour usage. The highlow method used only two data points, so the results may not be reliable. Molly would be advised to use the regression results, which are based on all relevant data.

..

86


3-B1 (25-30 min.) Mark-up method: Material cost Support costs (100%) Activity analysis method: Manual operations Support costs (@$6)

Board Z15

Board Q52

$46 $46

$65 $65

19 $114

8 $48

The support costs are different because different cost behavior is assumed by the two methods. If the activity analyses are reliable, then boards with few manual operations are overcosted with the markup method, and boards with many manual operations are undercosted with the markup method. 3-B2

(25-30 min.)

Variable cost per machine hour = Change in Repair Cost ÷ Change in Machine Hours = (P272,000,000 – P202,000,000) ÷ (11,900 – 7,900) = P17,500 per machine hour Fixed cost per month

= total cost - variable cost = P272,000,000 – P17,500 × 11,900 = P272,000,000 – P208,250,000 = P 63,750,000 per month

or

= P202,000,000 – P17,500 × 7,900 = P202,000,000 – P138,250,000 = P 63,750,000 per month

3-1

A cost driver is any output measure that is believed to cause costs to fluctuate in a predictable manner. For example, direct labor costs are probably driven by direct labor hours; materials costs are probably driven by levels of product output; and support costs may be driven by a variety of drivers, such as output levels, product complexity, number of different products and/or parts, and so on.

3-2

Linear cost behavior assumes that costs behave as a straight line. This line is anchored by an intercept, or fixed cost estimate, and total costs increase proportionately as cost driver activity increases. The slope of the line is the estimate of variable cost per unit of cost driver activity.

..

87


3-3

Whether to categorize a step cost either as a fixed cost or as a variable cost depends on the "size" of the steps (height and width) and on the desired accuracy of the description of step cost behavior. If the steps are wide, covering a wide range of cost driver activity, then within each range the cost may be regarded as fixed. If the steps are narrow and not too high, with small changes in cost, then the cost may be regarded as variable over a wide range of activity level, with little error. If the steps are narrow and high, covering big changes in cost, then the cost probably should not be regarded as variable, since small changes in activity level can result in large changes in cost.

3-4

Mixed costs are costs that contain both fixed and variable elements. A mixed cost has a fixed portion that is usually a cost per time period. This is the minimum mixed cost per period. A mixed cost also has a variable portion that is a cost per unit of cost driver activity. The variable portion of a mixed cost increases proportionately with increases in the cost driver. In order to achieve the goals set for the organization, management makes critical choices -- choices that guide the future activities of the organization. These choices include decisions about locations, products, services, organization structure, and so on. Choices about product or service attributes (mix, quality, features, performance, etc.), capacity (committed and discretionary fixed costs), technology (capital/labor considerations, alternative technologies), and incentives (standard-based performance evaluation) can greatly affect cost behavior.

3-5

3-6

Some fixed costs are called capacity costs because the levels of these fixed costs are determined by management's strategic decisions about the organization's expected levels of activities, or capacity.

3-7

Committed fixed costs are costs that are often driven by the planned scale of operations. These costs typically cannot be changed easily or quickly without drastically changing the operations of the organization. Typical committed fixed costs include lease or mortgage payments, property taxes, and long-term management compensation. Discretionary fixed costs are costs that may be necessary to achieve certain operational goals, but there are no contractual obligations to continue these payments. Typical discretionary fixed costs include advertising, research and development, and employee training programs. The distinction between committed and discretionary fixed costs is that discretionary fixed costs are flexible and could be increased, decreased, or eliminated entirely on short notice if necessary, but committed fixed costs usually must be incurred for some time -- greater effort is needed to change or eliminate them.

..

88


3-8

Committed fixed costs are the most difficult to change because long-term commitments generally have been made. These long-term commitments may involve legal contracts that would be costly to renegotiate or dissolve. Committed fixed costs also are difficult to change because doing so may mean greatly changing the way the organization conducts its activities. Changing these committed fixed costs may also mean changing organization structure, location, employment levels, and products or services.

3-9

An organization’s capacity generally determines its committed fixed costs. Management’s choice is the main influence on discretionary fixed costs. Both committed and discretionary fixed costs depend on the organization's strategy relating to capacity, product attributes, and technology. These elements will determine long-term cost commitments (committed costs) and flexible spending responses to changes in the environment (discretionary costs).

3-10

There is no apparent relationship between discretionary fixed costs and the levels of capacity or output activity. They are determined as a part of an organization’s periodic planning process; the management determines how much to spend on discretionary fixed costs in each period. As a result, the level of such costs can certainly vary from period to period or in other words the costs can be considered fixed only for the relevant period.

3-11

High technology production systems often mean higher fixed costs and lower variable costs.

3-12

Incentives to control costs are means of making cost control in the best interests of the people responsible for making cost expenditures. A simple example will illustrate the use of incentives to control costs. Assume that you are an executive who travels for business, purchases professional literature, and keeps current with personal computer technology. Under one incentive system, you simply bill the organization for all your travel and professional expenses. Under another system, you are given an annual budget for travel and professional needs. Which system do you think would cause you to be more careful about how you spend money for travel and professional needs? Most likely, the latter system would be more effective in controlling costs. Usually these incentives are economic, but other non-financial incentives may also be effective.

3-13

Use of cost functions, or algebraic representations of cost behavior, allows cost analysts or management to build models of the organization's cost behavior. These models can be used to aid planning and control activities. One common use of cost functions is in financial planning models, which are algebraic models of the cost and revenue behavior of the firm, essentially extended C-V-P models similar to those discussed in Chapter 2. Understanding relationships between costs and cost drivers allows managers to make better decisions.

..

89


3-14

A "plausible" cost function is one that is intuitively sound. A cost function is plausible if a knowledgeable analyst can make sound economic justifications why a particular cost driver could cause the cost in question. A "reliable" cost function is one that accurately and consistently describes actual cost behavior, past and future. Both plausibility and reliability are essential to useful cost functions. It is difficult to say that one is more important than the other, but one would not have much confidence in the future use of a cost function that is not plausible, even if past reliability (e.g., based on statistical measures) has been high. Likewise, one would not be confident using a cost function that is highly plausible, but that has not been shown to be reliable. The cost analyst should strive for plausible and reliable cost functions.

3-15

Activity analysis identifies underlying causes of cost behavior (appropriate cost drivers) and measures the relationships of costs to their cost drivers. A variety of methods may be used to measure cost functions, including engineering analysis and account analysis.

3-16

Engineering analysis is a method of identifying and measuring cost and cost driver relationships that does not require the use of historical data. Engineering analysis proceeds by the use of interviews, experimentation, and observation of current cost generating activities. Engineering analysis will be more reliable if the organization has had past experience with the activities. Account analysis is a method of identifying and measuring costs and cost driver relationships that depend explicitly on historical cost data. An analyst selects a single cost driver and classifies each cost account as fixed or variable with respect to that cost driver. Account analysis will be reliable if the analyst is skilled and if the data are relevant to future uses of the derived cost function.

3-17

..

There are four general methods covered in this text to measure mixed costs using historical data: (1) account analysis, (2) high-low, (3) visual fit, and (4) regression. • Account analysis looks to the organization's cost accounts and classifies each cost as either fixed, variable, or mixed with regard to an appropriate cost driver. • High-low analysis algebraically measures mixed cost behavior by constructing a straight line between the cost at the highest activity level and that at the lowest activity level. • Visual-fit analysis seeks to place a straight line among data points on a plot of each cost and its appropriate cost driver. • Regression analysis fits a straight line to cost and activity data according to statistical criteria.

90


3-18

Engineering analysis and account analysis often are combined. One of the problems of account analysis is that historical data may contain past inefficiencies. Therefore, account analysis measures what costs were, not necessarily what they should be. Differences in future costs may be desired and/or anticipated, and account analysis alone usually will not account for these differences. Engineering analysis may be combined with account analysis to revise account-based measures for desired improvements in efficiency and/or planned changes in inputs or processes.

3-19

The strengths of the high-low method are also its weaknesses -- the method is simple to apply since it does not require extensive data or statistical sophistication. This simplicity also means that the method may not be reliable because it may not use all the relevant data that are available, and choice of the two points to measure the linear cost relationship is subjective. The method itself also does not give any measures of reliability. The visual-fit method is an improvement over the high-low method because it uses all the available (relevant) data. However, this method, too, may not be reliable since it relies on the analyst's judgment on where to place the line.

3-20

High-low method of cost allocation identifies the high and low points within a given range and then calculates the variable cost first by dividing the change in total costs by the change in activity. Fixed cost is then determined by subtracting the variable cost at any of the two levels from the total cost of that level. Since the method considers only two points among many activity levels within a given range, while determining the costs, it ignores all other information available. As a result of this inefficient use of information, the cost allocation may not be accurate.

3-21

Regression analysis is usually preferred to the high-low method (and the visual-fit method) because regression analysis uses all the relevant data and because easyto-use computer software does the analysis and provides useful measures of cost function reliability. The major disadvantage of regression analysis is that it requires statistical sophistication to use properly. Because the software is easy to use, many users of regression analysis may not be able to critically evaluate the output and may be misled to believe that they have developed a reliable cost function when they have not.

3-22

This is a deceptive statement, because it is true on the face of it, but regression also has many pitfalls for the unwary. Yes, regression software provides useful output that can be used to evaluate the reliability of the measured cost function. If one understands the assumptions of least-squares regression, this output can be used to critically evaluate the measured function. However, the regression software cannot evaluate the relevance or accuracy of the data that are used. Even though regression analysis is statistically objective, irrelevant or inaccurate data used as input will lead to unreliable cost functions, regardless of the strength of the statistical indicators of reliability.

..

91


3-23

Plotting data helps to identify outliers, that is, observations that are unusual and may indicate a situation that is not representative of the environment for which cost predictions are being made. It can also show nonlinear cost behavior that can lead to transformations of the data before applying linear regression methods.

3-24

R2 is a goodness-of-fit statistic that describes the percentage of variation in cost explained by changes in the cost driver.

3-25

Control of costs does require measurement of cost behavior, either what costs have been or what costs should be. Problems of work rules and the like may make changing cost behavior difficult. There are tradeoffs, of course, and the instructor should expect that students could get into an impassioned debate over where the balance lies -- union job protection versus improved efficiency. This debate gets to one of the major roles of accounting in organizations, and it is important that students realize that accounting does matter greatly to individuals, and, ultimately, to society.

3-26

Both depreciation and research and development costs are fixed costs because they are independent of the volume of operations. Depreciation is generally a committed fixed cost. Managers have little discretion over the amount of the cost. In contrast, research and development costs are discretionary fixed costs because their size is often the result of management’s judgment.

3-27

Decision makers should know a product’s cost function if their decisions affect the amount of product produced. To know the cost impact of their decisions, decision makers apply the cost function to each possible volume of production. This is important in many decisions, such as pricing decisions, promotion and advertising decisions, sales staff deployment decisions, and many more decisions that affect the volume of product that the company produces.

3-28

Regression analysis is a statistical method of fitting a cost-function line to observed costs. It is objective; that is, each cost analyst would come up with the same regression line, whereas different analysts might have different cost functions when using a visual fit method. In addition, regression analysis provides measures of how well the cost-function line fits the data, so that managers know how much reliance they can put on cost predictions that use the cost function.

..

92


3-29 (15 min.) The analysis is faulty because of the following errors. 1.

The scales used for both axes are incorrect. The space between equal intervals in number of orders and order-department costs should be the same.

2.

The visual-fit line is too high, and the slope is too steep. It appears that the line has been purposely drawn to pass through the (100,450) data point and the $200 point on the y-axis to simplify the analysis. A visual-fit line most often will not pass through any one data point. Choosing one point (any point) or a data point and the Y-intercept makes this similar to the high-low method, ignoring much of the information contained in the rest of the data.

3.

The total cost for 90 orders is wrong. Either the fixed costs should be expressed in thousands of dollars or the unit variable costs should be $2,000 per order. Even if the derived total cost function was accurate, the resulting cost prediction is incorrect. The formula should be expressed as:

..

93


Total cost (thousands of dollars) = $200 + $2.50 × Number of orders processed, or Total cost = $200,000 + $2,500 × Number of orders processed This would result in a predicted total cost for 90 orders of: Total cost (thousands of dollars) = $200 + $2.50 × 90 = $425, or Total cost = $200,000 + $2,500 × 90 = $425,000. Correct Analysis The following graph has correctly constructed scales. The visual fit line shown indicates that fixed costs are about $200,000 and variable cost is about $2,250 per order – a lower slope than that shown in the text. Order Department Costs 80, 420

$450

100, 450

$400 (Thousands)

Order Department Costs

$500

$350

20, 280 10, 240

$300 $250

70, 320 40, 240

$200 $150 $100

$180

$50 $0

20

40

60

80

100

120

Orders Processed

The total cost function is: Total cost (thousands of dollars) = $200 + $2.25 × Number of orders, or Total cost = $200,000 + $2,250 × Number of orders Variable cost (thousands of dollars)  $180 ÷ 80 orders = $2.25 The predicted total cost for 90 orders is: Total cost = $200,000 + $2,250 × 90 = $200,000 + $202,500 = $402,500.

..

94


3-30 (15-20 min.) Amounts are in millions. 1. Sales revenues Less: Operating income (loss) Operating expenses 2.

2020 ₹148 (22) ₹170

2021 ₹308 122 ₹186

Change in operating expenses ÷ Change in revenues = Variable cost percentage (₹186 – ₹170) ÷ (₹308 – ₹148) = ₹16 ÷ ₹160 = 0.10 or 10% Fixed cost = Total cost – Variable cost = ₹170 – 0.10 × ₹148 = ₹155.2 or = ₹186 – 0.10 × ₹308 = ₹155.2 Cost function = ₹155.2 + 0.10 × Revenue

3.

Because fixed costs do not change, the entire additional total contribution margin is added to operating income. The ₹148 million revenue in 2020 generated a total contribution margin of ₹148 × (1 – 0.10) = ₹133.2 million, which was ₹22 million short of covering the ₹155.2 million of fixed cost. But the additional ₹160 million of revenue in 2021 generated a total contribution margin of ₹160 × (1 – 0.10) = ₹144 million that could go directly to operating income because there was no increase in fixed costs. It wiped out the ₹22 million operating loss and left ₹122 million of operating income.

3-31

(10-15 min.)

1. Lease rental: €5,000 x 2 x 4 = €40,000 for two cranes for four months. 2. Sales department cost: €8,000 + €4.5 x 5,000 = €30,500 3. Telephone charges: €500 + €0.20 x 10,000 = €2,500 4. Private security personnel cost: €1,250 x (20,000 ÷ 500) = €50,000

..

95


3-32

(10-15 min.) There may be some disagreement about these classifications, but

reasons for alternative classifications should be explored. Cost Advertising Depreciation

Discretionary $21,000

Health insurance for the company’s employees Management salaries 87,500 Payment of long-term debt Property tax Grounds maintenance 7,000 Office remodeling 24,000 Research and development 45,500 Totals $97,500 3-33

Committed $ 48,000 24,000 48,500 30,000

$238,000

(15-20 min.)

This problem extends the chapter analysis to preview short-run decision making and capital budgeting. This problem ignores taxes, investment costs, and the time value of money, which are covered in Chapter 11. Solution A Solution B Variable cost per order SGD 8.00 SGD 6.00 Expected number of orders 40,000 40,000 Annual variable costs SGD 320,000 SGD 240,000 Annual fixed cost 150,000 310,000 Annual total costs SGD 470,000 SGD 550,000 Therefore, Solution A is less costly than Solution B by SGD 80,000. Let X = the break-even number of orders, the level at which expected costs are equal. Costs for Solution A = Costs for Solution B SGD 150,000 + SGD 8X = SGD 310,000 + SGD 6X SGD 2X = SGD 160,000 X = 80,000 orders At 80,000 orders, the alternatives are equivalent. If order volumes are expected to be below 80,000 orders, then Solution A would have lower costs because fixed costs are lower. If orders volumes are expected to be greater than 80,000, then Solution B would have lower costs because variable costs are lower.

..

96


3-34 (20-25 min.) A master of the scatter-diagrams with least-square regression lines and high-low lines appears in Exhibit 3-38 on the following page. This exercise enables a comparison of the high-low and visual-fit methods of decomposing mixed-costs into fixed and variable parts. Students find it interesting to compare their best guesses to the least-squares regression results. They find it interesting that a fairly complete and accurate analysis is possible based on a scatter-diagram and a little common sense. We normally have the class determine a “class best guess” before showing the transparency of the regression results. The exercise also introduces students to the concept of a hierarchy of activity levels, although this topic is not covered in the text. The literature contains discussions of four general levels of activities. Recognizing each of these levels can be an aid in choosing appropriate cost drivers. These levels and example cost drivers are: a.

Unit-level activities -- performed each time a unit is produced (units of product, machine hours, labor hours).

b.

Batch-level activities -- performed each time a batch of goods is processed or handled (number of orders processed, number of setups, number of material moves).

c.

Product-level activities -- performed as needed to support the production of each different type of product (number of tests, number of parts, number of engineering change notices, hours of design time, number of inspections).

d.

Facility-level activities -- sustain a facility’s general manufacturing process (square footage, number of employees, hours of training). In this exercise, a batch-level activity is involved -- setups.

Exhibit 3-38 – Maintenance Costs (Thousands)

..

97


$30 $25

Regression

$20 $15 $10

High-Low

$5 $0

1

2

3

4

5

Units Produced (Thousands)

..

98


1. Student answers will vary somewhat. Least-squares regression lines are given as a standard for comparison. Based on regression, the cost functions are: Maintenance costs = $13,108 + $2.17 × Units produced (000s) Maintenance costs = $5,162 + $751 × Number of setups The April observation should be ignored since it does not represent a typical month -- it is an example of an outlier. Other examples would be strikes, abnormal downtime, or scheduled plant closings. 2.

The high-low method uses only the highest and lowest activity levels. Note that, if one is using a scatter diagram, the high-low method can be used without knowing the exact figures. Fixed cost can be easily estimated using a straight edge and should be about $11,500 based on Units Produced and $7,500 based on Number of Setups. Variable costs are estimated using the following computations: Variable maintenance costs Variable maintenance costs

3.

= ($21,000 - $15,000)/(3,900 - 1,200) = $2.22 per unit = ($25,500 - $15,000)/(27 - 11) = $656 per setup

Both cost drivers appear, on the surface, to be plausible. However, if maintenance activity is primarily associated with a “batch-level” activity such as setups, the setup driver is preferred. Of the three costs associated with maintenance activity, supplies and energy are probably variable, so salaries are the primary fixed costs. The monthly salary of two mechanics is $4,167 [(2 × $25,000)/12]. The cost function based on setups estimates fixed costs of about $5,200 (visual-fit method). This is much more plausible than the $15,200 estimate based on units of production. Students may inquire as to the use of “setup time” as an alternative to number of setups. Setup time is an acceptable alternative that is often used when setup times differ among different products. Another consideration is data availability. Setup times by product may not be easily obtained or maintained. Just looking at the two graphs, a linear cost function seems to fit the second graph much better than the first. Reliability of cost drivers is measured by the coefficient of determination, R-Squared. In the regressions used in requirement 1, only 21% of the past year’s variability in maintenance costs can be explained by changes in the volume of units produced, whereas 85% of past fluctuations in maintenance costs can be explained by the number of setups performed. This confirms the visual observation.

..

99


3-35

(15-20 min.) The total cost for the month is $1,730 + (5 × $1,250) = $7,980, based on the following cost function information: Cost Phone Utilities Advertising Insurance Materials Labor Totals

Fixed per month $ 60 75 85 110

Variable per computer

1,400 $1,730 per month

$5,500 750 $6,250 per computer

Algebraically, y = $1,730 + $1,250x ; where y = total cost per month x = number of computers 3-36

(5 min.) All of the functions except (e) are linear cost functions. Functions (c) and (d) and (f) are mixed costs. Note that (e) is not linear because X 1 and X2 are multiplied.

3-37

(5-10 min.) Variable cost per purifier = (£115,000 − £95,000)  (5,500 – 3,500) = (£20,000 ÷ 2,000) = £10/purifier Fixed cost = £115,000 – (5,500 × £10) = £60,000 or = £95,000 – (3,500 × £10) = £60,000 Cost function = £60,000 + £10 × Number of purifiers

3-38

(10-15 min.) The regression analysis results show that more was spent on building maintenance in months of low production volume than in months of high volume. The assistant controller erred in not thinking about the economic logic of this result. The result does not imply that intensive use of the building decreases maintenance costs. When production volume is low, workers do maintenance rather than work on production. When volume is high, little maintenance is done because workers are busy on production. This is a case where the regression analysis does not correctly separate costs into fixed and variable components. Considering the economic plausibility of a negative variable maintenance cost should make this readily apparent. A more correct analysis would probably show that maintenance costs are not related to direct labor, or, if there is a relationship, more labor should cause more maintenance because it implies more intensive use of the production facilities.

..

100


3-39 (50-60 min.) (Masters of the line graph and pie charts appear on the next three pages. Two versions of the pie charts are shown.) 1.

The line graph shows the plot of the total cost for each of the two options at various levels of capacity utilization. The outsource/ overtime option has a steeper slope due to the larger proportion of variable costs, especially beyond the 100% level of production when overtime premiums and outsourcing are required (note the kink in the line). At production (sales) levels below 100% of capacity, total costs are lower with the outsource/overtime option. At production levels above 100%, the build option is the low-cost option.

2.

Controlling risk usually means reducing the financial exposure of a company when business conditions turn unfavorable. Companies attempt to control this risk through various means -- diversifying their product lines and markets and reducing fixed (committed) costs or converting fixed costs into variable costs. In this case, the outsource/overtime option avoids converting variable production costs into committed fixed (capacity) costs in order to retain cost control and hence reduce financial exposure. As can be readily seen from the graph or the table, the benefit of the outsource/overtime option is the decreased financial exposure when production is low. Total costs of the outsource/overtime option at the 60% level of production are $8 million less than those of the build option. The cost of the outsource/overtime option is the lost profit when demand is high -- total costs are $12 million higher at the 120% level. In essence, by choosing the outsource/overtime option, HP is willing to forego $12 million of profit in the near term in order to reduce its financial exposure to an $8 million loss in the future. Why? Perhaps HP’s assessment of the probability of continued high demand is less than the probability of a future downturn, or perhaps HP’s key decision makers prefer to avoid risk.

3.

..

Students’ answers to this question will vary depending on their attitudes toward risk. This part of the problem helps students realize the value of different forms of analysis. We use pie charts to demonstrate one form of analysis -- tables can also be used. The pie charts bring out the importance of fixed costs more readily than the line graph. The four pie charts can be used to point out the value of proportional pie charts. First, focus attention on the two build-option pies. Point out that fixed-cost percentages range from 45% to 63% of total costs if HP builds automated facilities. This range of fixed costs is reduced to 20-36% of total costs if HP continues to use overtime and outsourcing. However, comparing the size of the two 120% pies, it can be easily seen that HP will sacrifice profits by not building if volume approaches the 120% level.

101


BUILD VERSUS OVERTIME/OUTSOURCE OPTIONS

TOTAL COST (MILLIONS)

120

100

80

60

40

20

0 60%

80%

100%

120%

PERCENT OF CURRENT CAPACITY

Build

..

Overtime/Outsource

102


COST BEHAVIOR OF CAPACITY COSTS (Millions)

Build Option @ 120% of Capacity

Build Option @ 60% of Capacity

Materials, $18, 28% Materials, $36, 41%

Other Costs, $40, 45%

Other Costs, $40, 63%

Labor, $6, 9%

Labor, $12, 14%

Outsource/Overtime Option @ 120% of Capacity

Other Costs, $20, 20%

Outsource/Overtime Option @ 60% of Capacity

Other Costs, Materials, $20, 36%

Materials, $18, 32%

$36, 36%

Labor, $44, 44%

..

Labor, $18, 32%

103


COST BEHAVIOR OF CAPACITY COSTS (Millions)

Build Option @ 120% of Capacity

Build Option @ 60% of Capacity

Build @ 120%

Other Costs $40 45%

Build @ 60%

Materials $36 41%

Labor $12 14%

Outsource/Overtime Option @ 120% of Capacity

Materials $18 28%

Outsource/Overtime Option @ 60% of Capacity Outsource @ 60%

Outsource @120%

Materials $36 36%

Other Costs $20 20%

Labor $44 44%

..

Labor $6 9%

Other Costs $40 63%

Other Costs $20 36%

Materials $18 32%

Labor $18 32%

104


3-40

(30-35 min.)

1. The graph of weekly planned cost of guards versus number of visitors shows that this cost is a step cost beyond 80 visitors.

€5,200 4,800 4,400 4,000 3,600 3,200 Guard

2,800

Cost

2,400

per

2,000*

Week : ... 0

80

100

120 140 160 180 Number of Visitors per Week

200

220

240

*(€24,000/48) × 4 2. In January, up to 80 visitors are expected; therefore, €2,000 per week should be planned for guard costs. This represents the minimum staffing level and would represent a fixed cost. The monthly cost for January is €2,000 × 4 = €8,000. 3. Week 1 2 3 4 Total Actual Variance ..

Actual Visitors 125 190 130 215

Minimum Guard Cost € 3,200 4,400 3,200 4,800 €15,600 €13,200 € 2,400 under budget 105


This report indicates that St. Lawrence National Park spent less than planned for guards during July. It might appear that the park authority was extremely efficient because it spent less than planned. However, another interpretation of this report is that the park was understaffed during the critical “busy” summer season. This could mean that visitors were not adequately monitored and served during this time. The €2,400 savings might be small compared to the loss of reputation caused due to understaffing and visitors remaining unserved. 4. The €2,000 fixed amount is the total weekly salary paid to the permanent guard staff. Every twenty visitors (or portion thereof) above 80 require an additional guard at a cost of €400 per week. On average, each additional visitor costs €400 ÷ 20 or €20. 5.

Week 1 2 3 4 Total Actual Variance

Actual Visitors 125 190 130 215

Minimum Guard Cost €2,000 + €20(125 – 80) = €2,900 2,000 + €20(190 – 80) = 4,200 2,000 + €20(130 – 80) = 3,000 2,000 + €20(215 – 80) = 4,700 €14,800 €13,200 €1600

Even though the variance of this report is much less than that of the previous one, this report is not necessarily more accurate. The average cost function predicted actual costs more closely, but if the cost behavior described in 3 above represents committed step costs due to state or federal regulations, then the amounts calculated in 3 are costs that the park should have incurred. On the other hand, the administrator of the park may try to hire guards according to the simplification of the minimum staffing cost behavior presented in 4 and expect that, on average, the park will not be understaffed.

..

106


3-41

(25-30 min.)

1.

Actual Costs Salaries

$56,880

Planned Costs 8,300 × $5 = $41,500

Variance $15,380 Unfavorable

2.

If the cost measurements are reliable, then the audit office is overstaffed by approximately five auditors. Each auditor should be able to process 4 weeks × 5 days per week × 8 hours per day × 4 returns per hour or 640 returns during the 4week period. The 8,300 returns should have been processed by 8,300  640 or less than 13 auditors, five less than were employed. Alternatively, the $15,380 variance represents $15,380 ($790 × 4 weeks) or approximately 5 excess auditors.

3.

The variance may be due to inefficiency of the auditors, improperly trained or inexperienced auditors, inaccuracy of the cost measures, a batch of unusually complex returns, or a combination of all these factors. The role of the cost variance is to identify where something is different than planned. The variance itself usually does not identify the cause of the variance, due to which the management attention may be required.

4.

Besides number of returns, alternative cost drivers might include number of individual forms included in filed returns, number of pages of returns processed, and amount of taxes shown on returns filed.

3-42 (10-15 min.) In the solution, only one possible cost driver is shown, with cost behavior with respect to the cost driver in parentheses. Other cost drivers are also possible. a. Yearly maintenance contract fees (AMC) for all equipment – Number of equipment (fixed) b. Advertising costs – This is a discretionary fixed cost for which identifying a cost driver is difficult c. Payroll – Number of employees (fixed) d. Food, beverages, and house-keeping cleaning supplies – Room days (variable) e. Laundry operations – Room days (variable) f. Guest supplies and amenities – Room days (variable) g. Electricity and water supply – Room days (mixed)

..

107


3-43

(25-30 min.)

The first temptation may be to measure the cost behavior with the high-low method, using the cost and activity levels from 20X7 and 20X9 since they are the lowest and highest cost and activity levels given. However, Dr. Hyde has indicated that drug test procedures are both more numerous and more complex than they were in the past. Accordingly, if additional equipment and more expensive equipment have been acquired and more complex testing is commonplace to meet this new demand, then the past data may not be a relevant base for cost-behavior measurement. A simple graph of cost and activity on the next page illustrates the possible problems. A line drawn through the points for the first two years will have a much different slope and intercept than that implied by the most recent year's experience. It is likely that cost behavior has changed significantly, and this cost behavior may not be revealed by the data currently available. New cost behavior probably has a higher intercept (greater fixed costs per month) and a steeper slope (greater variable cost per procedure) than in the past. More current data are required. Since the data given are monthly averages, the raw monthly data are probably available. One recommendation would be to disregard data from the first two years and to use monthly data from the third year to measure current cost behavior. It could well be that this analysis will indicate a need for the price increase demanded by Dr. Hyde. Cost of Test Procedures 160,000

140,000

120,000

100,000

Cost in Dollars

80,000

60,000 Cost

40,000

20,000

0 0

100

200

300

400

500

600

700

Num ber of Procedures

..

108


3-44

(30-35 min.) The data should be used to first determine variable expenses as a function (percentage) of course fees. Then fixed expenses can be calculated. Since only two data points are available, the high-low method is the appropriate approach. Variable expenses = Change in expenses ÷ Change in revenues = (BDT 425,000 – BDT 411,000) ÷ (BDT 460,000 – BDT 420,000) = BDT 14,000 ÷ BDT 40,000 = 35% of course fee Fixed expenses = Total expenses – Variable expenses = BDT 425,000 – (.35 × BDT 460,000) = BDT 425,000 – BDT 161,000 = BDT 264,000 per year or

= BDT 411,000 – (.35 × BDT 420,000) = BDT 411,000 – BDT 147,000 = BDT 264,000 per year

Income for 2021 may be predicted as follows: IMPERIAL LAW COLLEGE Projected Income For the Year Ending July 31, 2021 Course fees Less: Variable expenses (.35 × BDT 480,000)

BDT 480,000 BDT 168,000

Fixed expenses

264000 432,000

Net Income

BDT 48,000

Or, Net Income = Course fees – Variable expenses – Fixed expenses ..

109


= BDT 480,000 – (.35× BDT 480,000) – BDT 264,000 = (.65 × BDT 480,000) – BDT 264,000 = BDT 48,000 Break-even course fee revenue may be found by setting Net Income = 0 and solving for the unknown course fee, CF, as below: 0

= CF –.35CF – BDT 264,000

0

= .65CF – BDT 264,000

BDT 264,000 = .65CF CF

..

= BDT 406154 at break-even point (rounded)

110


3-45 1.

(20-25 min.) Support cost measurement: Customer Mystical Plants

Fixed charge method: Basic cost of FertiMix Support cost @ 65% Activity Analysis method: Lines of customized code Estimated cost per line of customized code Activity support cost 2.

Todal Blooms

$8,000 $5,200

$8,000 $5,200

520

160

× $20 $10,400

× $20 $ 3,200

The activity analysis approach indicates that products requiring large amounts of customizing incur much more support cost than those that require relatively little customizing. The old approach leads to distorted costs that might lead to poor planning and control and either lost sales or unprofitable sales. The benefits of adopting the activity analysis approach are (1) more accurate measures of support costs, (2) more competitive cost-based prices, (3) better planning of support costs, and (4) better control of support costs. The disadvantages are that the activity analysis will be more costly to implement and monitor (and may not be necessary for pricing if Washta Software’s industry is not a competitive one).

3-46

(10-15 min.)

1.

Variable cost/unit = (AUD 5,620 – AUD 3,200)  (665 – 340) = AUD 2420  325 = AUD 7.4462 Fixed cost = AUD 5,620 – (665 × AUD 7.4462) = AUD 5,620 – AUD 4,951.72 = AUD 668.28 Predicted cost for 530 units = AUD 668.28 + (530 × AUD 7.4462) = AUD 4,614.77

2.

Predicted cost for 530 units = AUD 1733 + (530 × AUD 5.76) = AUD 4,785.80

3.

The regression analysis gives better cost estimates because it uses all the data to form a cost function. The two points used by the high-low method may not be representative of the general relation between costs and volume.

..

111


3-47

(35-40 min.) If supplies cost is at least partly fixed with regard to production volume, then treating supplies cost as if it were purely variable (e.g., using the average supplies cost per unit of production as the variable cost rate) will result in predicting too little supplies cost at low levels of production and too much at high levels of production. See the graph below:

Supplies Cost

"True" cost

Predicted cost assuming purely variable cost Below Average

Average

Above Average

Production Level 1.

The preferred cost function uses "number of tents" as the cost driver for supplies cost. Although many other statistical criteria could be (should be) used to make this determination, this choice is based on the relative R-squared values. The Rsquared measures the amount (percentage) of fluctuation (variation) in historical supplies cost that is associated with either number of tents or with square feet of material used. The cost function using number of tents has a much higher Rsquared value and, therefore, is more closely associated with historical variations in supplies cost. The interpretation of the preferred cost function is that, based on past data, supplies cost has a fixed component, $2,500 per month, and a variable component, $0.071 per number of tents in a month. The total supplies cost function can be written as: Total supplies cost = $2,500 per month + $0.071 × number of tents

2.

..

Approximately 48.5% of the variation in historical supplies cost is associated with variations in square feet of materials. The remaining 51.5% of variation in supplies cost (100% - 48.5%) depends on other factors, not included in the cost function. Square feet of materials used does not explain this 51.5% of the variation in supplies cost.

112


3-48

(40-45 min. unless data supplied by instructor, then 25-30 min.)

1.

The accompanying graphs can be used to discuss requirements 2-4. Cost $350 $300

Support Costs

$250 $200 Cost $150 $100 $50 $0 0

5

10

15

20

25

30

35

Batch Size

Cost $350 $300

Support Costs

$250 $200 Cost $150 $100 $50 $0 0

5

10

15

20

25

30

35

Batch Size

..

113


2.

3.

Regression Output: Constant R Squared X Coefficient Support costs

4. High Level Low Level Difference

24.42553 0.955692 10.53191

= fixed cost + variable cost = $24.43 + $10.53 × 25 = $287.68 Support cost $320 110 $210

Batch size 30 9 21

Variable cost = Change in cost ÷ Change in activity = $210 ÷ 21 = $10 per unit in batch Fixed Cost

= total cost - variable cost = $320 - $10×30 = $20 per batch

Support costs of a batch of size 25 = $20 + $10 × 25 = $270 Although the cost functions and cost estimates are fairly close, the manager would probably be better off using the regression result. The regression results appear to be very reliable and plausible. Since regression uses all the data and no data appear to be unusual (per the graph), there is little reason not to use the regression.

..

114


3-49

(35-50 min.)

1.

The three graphs are as follows: $80

Support Costs ($000)

$70

$60

$50

$40

$30 7,500

7,600

7,700

7, 800

7,900

8, 000

Direct Labor Hours

$80

Support Costs ($000)

$70

$60

$50

$40

$30 2,000

2,200

2,400

2,600

2,800

3,000

3,200

Boards Com pleted

..

115


$80

Support Costs ($000)

$70

$60

$50

$40

$30 100

120

140

160

180

200

Average Cycl e Tim e

2.

This output is generated by a spreadsheet. Regressions of circuit board support costs using the following as cost drivers: Regression Output: Direct Labor Hours Constant 9,466.871 R Squared 0.0059439 X Coefficient(s) 5.960404 Regression Output: Number of Boards Completed Constant 21,810.742 R Squared 0.2219548 X Coefficient(s) 13.945440 Regression Output: Average Cycle Time Constant 6,572.774 R Squared 0.8540626 X Coefficient(s) 330.482863

3.

..

The most plausible and reliable regression function, identifying the best single cost driver for American Micro Devices’ circuit board support costs appears to be the one that uses cycle time as the cost driver. (A multiple regression using both cycle time and number of boards is even better.) All of the functions are plausible -- they indicate increasing support cost as the cost driver increases. The measure for choosing among the functions are the relative R 2 values of each function. Using cycle time as the cost driver generates the highest R 2 value of all the models using a single cost driver. 116


4.

The economic meaning of the function using cycle time as the cost driver is that circuit board support costs have a fixed component of $6,573 per week and a variable component of $330.48 per hour of average cycle time. Reducing cycle time by an average of one hour should reduce support costs by about $330.

3-50 (30-35 min.) This problem anticipates the use of cost functions for pricing purposes. Alternatively, the instructor may wish to use this problem in conjunction with Chapter 5. 1.

One would expect that the third cost function, using average cycle time as the cost driver, would be the most reliable for explaining and predicting support costs. Although all the functions are plausible, average cycle time as a cost driver generates the highest R2, which means that it best explains past support costs. If the process remains unchanged, then this function should be reliable for predicting future support costs.

2.

The first part of this solution uses the cost function developed in problem 3-49. The second part uses the cost function given in the problem. (a)

(b)

Cost function from 3-49: Cost using Direct Labor Hours as the cost driver: Fixed cost: $9,467/wk × 3 weeks = Variable cost: $5.96 × 20,000 hours = Total Cost using Number of Boards as the cost driver: Fixed cost: $21,811/wk × 3 weeks = Variable cost: $13.95 × 6,000 boards = Total Cost using Average Cycle Time as the cost driver: Fixed cost: $6,573/wk × 3 weeks = Variable cost: $330.48 × 180 hrs × 3 weeks = Total Cost function from the problem: Cost using Direct Labor Hours as the cost driver: Fixed cost: $9,000/wk × 3 weeks = Variable cost: $6 x 20,000 hours = Total Cost using Number of Boards as the cost driver: Fixed cost: $20,000/wk × 3 weeks = Variable cost: $14 × 6,000 boards = Total Cost using Average Cycle Time as the cost driver: Fixed cost: $5,000/wk × 3 weeks = Variable cost: $350 × 180 hrs × 3 weeks = Total

..

$ 28,401 119,200 $147,601 $ 65,433 83,700 $149,133 $ 19,719 178,459 $198,178

$ 27,000 120,000 $147,000 $ 60,000 84,000 $144,000 $ 15,000 189,000 $204,000 117


3.

For this three-week period and the particular boards manufactured, the average cycle-time cost function yields materially different cost predictions. We know from the regression analyses that the direct labor function and, to a lesser degree, the number of boards function are not reliable functions. Unless there was something unusual about the production activity of those three weeks, American Micro Devices should use the cost estimates from the average cycle-time regression.

4.

In a highly competitive environment, the market influences prices more than does cost. Therefore, setting prices by marking up costs, even if costs are accurate, is not a sufficient pricing policy. At a minimum, American Micro Devices should examine its prices compared to those of its competition in addition to comparing them to its costs.

3-51 (25-30 min.) This is not a difficult problem, but it forces students to think through cost and revenue behavior in a situation that differs from those illustrated in the text. 1. Fixed cost = $64,000 + $51,000 = $115,000 Variable cost per student month = ($100,000 + $54,000)  2,200 = $70 per student month 2.

Revenues and fixed costs would not change. Variable costs would decrease by $70 for each of the students to whom lessons are not provided: (1,200-810) × $70 = $27,300 Therefore, profits would increase by $27,300: 2010-11 profit = $1,000 + $27,300 = $28,300

3.

To make a $1,000 profit, the contribution margin from the students served must equal the fixed cost plus the profit: $115000 + $1,000 = $116,000. The contribution margin per student-month is $120 - $70 = $50. Therefore, $116,000  $50 = 2,320 student months are needed. However, this is outside the relevant range for fixed costs. An additional facilities charge of $2,800 will be incurred, making the new required contribution margin $116,000 + $2,800 = $118,800. Students needed are: $118,800  $50 = 2,376 student months, or 2,376  9 = 264 students.

..

118


3-52

(30-35 min.)

1. This is only a first pass; obviously Dr. Mukherjee would be able to specify more accurately which of these are committed costs and which discretionary costs. Students will have different ideas about what constitutes committed and discretionary. The important thing for them is to be able to explain and justify their classifications. Program Area Administrative salaries: Administrator Assistant Two secretaries Supplies Advertising and Promotion Professional meetings, dues, and literature Purchased Services Accounting and billing Custodial and maintenance Security Consulting Community health services Salaries (two social workers) Travelling * Outpatient health treatment Salaries Two general physicians Two social workers Totals

Committed

Discretionary

₹960,000 180,000 420,000

₹480,000 180,000 210,000 250,000

120,000 80,000 240,000 140,000 360,000 75,000

75,000

1560,000 360,000 ₹4355,000

₹1335,000

* We assume that half of the travelling cost is discretionary. 2. If all discretionary costs were eliminated, about ₹1335,000 could be saved. However, some of these “discretionary” cuts may seriously affect the ability of the health center to deliver its services. There does not seem to be much “fat” in this budget to begin with, and eliminating such items as travelling for social workers would mean that the community would have to come to the clinic rather than vice-versa. Cutting down on professional development opportunities of the staff could mean losing quality staff or reducing their quality over time. Dropping advertising and promotion may be the least painful since the center is apparently at capacity now. However, this could mean that individuals who really need the services will not find out about them. Eliminating the consulting may mean that the center cannot refer individuals with unique problems to specialists. Finally, eliminating the administrative assistant and one secretary will mean a greater ..

119


burden for Dr. Mukherjee and the remaining secretarial staff. Cutting these “discretionary” expenses may be necessary, but it will be painful. 3.

Dr. Mukherjee should prepare for the worst but begin now to build her case for even higher resources given the past budget cuts and increasing demand for services at the center. Documentation of community needs, benefits provided by the center, and needs not being met is necessary. A good-faith effort to first eliminate any possible waste may convince budgetary authorities that no further budget cuts are necessary and even that some budget enhancement is desirable.

3-53 (45-50 min.) This problem extends the use of activity analysis for control and transfer-pricing purposes. The instructor may wish to use this problem as a preview of later applications or in conjunction with Chapter 10. 1.

The number of employees may be an indicator of service department costs in general. If all users of service departments have roughly the same per capita usage of services, then using number of employees may be a simple and reasonably accurate and equitable means of charging for these costs. However, more specialized services may have more specific cost drivers that are not distributed according to number of employees, as is apparently the case of SS department costs at Southeast Pulp and Paper. Whether activity analysis is justified to identify and measure this cost behavior more accurately depends, of course, on the costs and benefits of the effort. This case is similar to the experience of Weyerhaeuser. Weyerhaeuser felt the effort was worthwhile, and while we do not have post-audit type information on the continued viability of activity analysis of service costs at Weyerhaeuser, we will assume that the benefits continue.

2.

2008 SS Cost per Employee = 2008 SS Costs ÷ Number of Employees =$300,000 ÷ 1,721 = $174.32 2008 SS Cost per Report

= 2008 SS Costs ÷ Number of Reports = $300,000 ÷ 1,232 = $243.51

2009 SS Cost per Employee = 2009 SS Costs ÷ Number of Employees = $385,000 ÷ 1,295 = $297.30 2009 SS Cost per Report

..

= 2009 SS Costs ÷ Number of Reports = $385,000 ÷ 1,556 = $247.43

120


2008 Number of Employees

Total 1,721

2008 SS Costs Charged to Divisions via Employees ($174.32 × 762, etc.) 2008 Number of Reports

1,232

2008 SS Costs Charged to Divisions via Reports ($243.51 × 410, etc.) 2009 Number of Employees

1,295

2009 SS Costs Charged to Divisions via Employees ($297.30 × 751, etc.) 2009 Number of Reports 2009 SS Costs Charged to Divisions via Reports ($247.43 × 412, etc.)

1,556

Forest Management 762

Lumber Products 457

Paper Products 502

$132,832

$ 79,664

$ 87,509

410

445

377

$99,839

$108,362

$91,803

751

413

131

$223,272

$122,785

$38,946

412

432

712

$101,941

$106,890

$176,170

It is clear that the other divisions have what they see are legitimate complaints. Each of the other divisions, Forest Management and Lumber Products, has reduced the number of employees, but not as drastically as the Paper Products division. The result has been that more of the SS department costs have been shifted to Forest Management and Lumber Products, even as Paper Products has increased its demands for SS services. It would appear that Paper Products is not paying its fair share of SS costs. 3.

Charging for SS department costs on the basis of number of employees creates an incentive to reduce the number of employees or to add employees only if the added benefits exceed the wage/salary cost plus SS (and other service) department costs. Charging for SS costs based on the number of reports creates the incentive to demand additional reports only if their value to the division exceeds the cost charged. This latter form of charge, based on the department's cost driver(s), probably will permit planning and control of service department costs more effectively than using generic charges.

4.

It appears that activity analysis should be extended to all of Southeast’s service departments. Using number of employees as the basis for charging for service costs probably distorts incentives for divisions to control costs.

..

121


3-54

(35-50 min.)

1.

See the accompanying graphs. One can see two different cost behaviors that appear to mirror changes in the cost time series. Matching the cost table and the graph shows that both the intercept and the slope of the cost function have changed after week 13 -- fixed costs have increased at the same time that variable costs per order have decreased. In fact, logistics costs seem to be an almost purely fixed cost after week 13. 30

Logistics Costs ($000)

25 20 15 10 5 0 0

200

400

600

800

1,000

1,200

1,400

1,600

1,000

1,200

1,400

1,600

Number of Orders

30

Logistics Costs ($000)

25 20 15 10 5 0 0

200

400

600

800 Num ber of Orders

..

122


2.

The first 13 weeks of data appear to be irrelevant to current cost behavior. Any measures of cost behavior that are to be used for current and future use should be based on the most recent data (weeks 14-25) only.

3.

See the accompanying graph. The data do support Hudson’s expectations: Fixed costs have increased, and variable costs have decreased. Regression analyses (though on limited numbers of observations) confirm this.

Regression Output: Weeks 1-13 Constant R Squared X Coefficient(s)

5,497.172 0.8401874 12.9031643

Regression Output: Weeks 14-25 Constant R Squared X Coefficient(s)

20,337.159 0.1282178 0.5556408

However, average total logistics costs do not appear to have decreased. On logistics cost behavior alone, the switch does not seem to be justified. However, the new ordering system may be more flexible and may contribute to cost and quality savings in other departments. A full analysis of the benefits of the new ordering system should try to capture those benefits as well. If the ordering activity should increase greatly, the new system may be able to operate at lower total cost than the old system, but this level of activity would be well outside the relevant range of experience. In the last 12 weeks, the average number of orders is 528 per month. Under the old system, these would have cost $5,497 + (528 × $12.90) = $12,308. Under the current system the cost is $20,337 + (528 × $0.56) = $20,633.

..

123


3-55 (20-25 min.)  

Step Fixed Costs Depreciation of forklift trucks Supervisor salaries per shift: within a single 8-hour shift the cost of supervision increases in “chunks” as sales volume increases All regular labor and supervision salaries when additional 8-hour shifts are added

 

  

Mixed Costs Facilities maintenance Lease of equipment based on a base charge plus a usage charge Total salaries of labor that includes regular (fixed), overtime (variable), and temporary (variable) Total salaries of sales managers that include a flat amount plus a variable commission amount Telecommunications costs consisting of a fixed charge per month plus an additional charge depending on the number of minutes used

Activity Receiving Unpacking incoming cases of footwear Picking and packing cases of footwear for shipment to retail accounts (customers) Processing orders from retail accounts Providing customer service to retail accounts Processing order changes from retail accounts

         

Plausible Cost Driver Pallets Cases Pallets Cases Pallets Line items Orders Orders Calls from customers Number of changes

3-56 (20-30 min.) For the solution to this Excel Application Exercise, follow the stepby-step instructions provided in the textbook chapter. 3-57

(10-30 min.)

The purpose of this exercise is to develop an understanding of different types of costs and their behavior. Assigning the problem ahead of time allows students to prepare lists of each type of cost, forcing them to think about types of cost and their behavior. However, the game could be played without advance preparation, especially if many students have some business experience.

..

124


3-58 (40-60 min.) NOTE TO INSTRUCTOR: This solution is based on the web site as it was in late 2012. Be sure to examine the current web site before assigning this problem, as the information there may have changed. 1.

For the 2011 One Report, the ten-year summary statistics section is located under past performance. The information found on the summary report includes operating revenues separated according to those earned from passenger, freight, and other sources, operating expenses, operating income, other income/expenses, tax expense and net income. The information also provides data on income per share, basic and diluted, dividends per share, total assets, long-term debt, and stockholders’ equity. The report also includes two subsections, consolidated financial ratios and consolidated operating statistics.

2.

The information about revenues is divided into categories – passenger, freight, and other. The information about operating expenses is listed as one lump sum. It does not differentiate between costs associated with the differing types of revenues that are earned. This is likely due to the fact that the majority of the costs are incurred for providing passenger services and the freight costs are insubstantial in comparison. The firm also is not likely to want to provide too much detail that the competitors for freight could use against SWA.

3.

RPM is revenue passenger miles and ASM is available seat miles. The available seat miles is larger. The RPM is found by taking the number of passengers on each plane and multiplying it by the number of miles that the plane flies for that trip. This is done for all trips taken during the year. The ASM is determined by taking the number of seats on the plane and multiplying it by the miles that the plane flies for a trip. If all seats on the plane are filled with paying customers for all flights during the year, then the RPM and the ASM could be the same. The ASM is essentially a measure of capacity and RPM is a measure of how much of that capacity was used during the year. The actual RPMs and ASMs in miles are given for the year. The passenger revenue yield per RPM and operating revenue yield per ASM are provided, as is the operating expense per ASM.

4.

Using the high-low method for years 2002 and 2005, variable operating costs are (millions) 2005 2002 Difference

Op. Exp. $6,859 5,181 $1,678

RPM 60,223 45,392 14,831

Variable operating cost per RPM = Change in cost ÷ Change in cost driver =$1,678 ÷ 14,831 = $0.1131

..

125


Fixed op. cost per year = Total operating cost - Variable operating cost Using 2005:

$6,859 – ($0.1131 × 60,223) = $6,859 - $6,811 = $48

Total operating cost per RPM = 2005 Operating Cost ÷ 2005 RPM = $6,859 ÷ 60,223 = $0.1139 The total operating cost is almost the same as the variable operating cost, and the fixed cost is very small. This is not what would generally be expected. Airlines usually have large fixed costs and small variable costs. 5.

As just explained, airlines usually have large fixed costs. In this case the highlow method is not accurate. The increase in costs between 2002 and 2005 may have included much investment in airplanes. If this investment in airplanes was driven by the demand for more RPMs, then it is part of long-run variable costs but not necessarily part of short-run variable costs. That is, it does not cost $0.1131 per passenger mile to add a passenger to a flight that is already scheduled. However, if attracting more passengers requires additional flights, and therefore additional airplanes, additional RPMs do generate $0.1131 of cost per RPM. The relationship between short- and long-run cost behavior may look like the graph below.

Short-run variable cost per RPM< long-run variable cost per RPM

Operating Costs

Purchase of a plane increases fixed costs

Long-run variable cost = $.1131 per RPM

Revenue Passenger Miles, RPM

..

126


CHAPTER 4 COVERAGE OF LEARNING OBJECTIVES

LEARNING OBJECTIVE

CASES, EXCEL, FUNDAMENTAL ADDITIONAL COLLAB. & ASSIGNMENT ASSIGNMENT INTERNET MATERIAL MATERIAL EXERCISES

LO1: Describe the purposes of cost management systems. LO2: Explain the relationship between cost, cost object, cost accumulation, and cost assignment. LO3: Distinguish between direct and indirect costs.

B4

31, 34, 50

B4

45, 45, 51, 52, 53

56

A1, B1, B4

56

LO4: Explain the major reasons for allocating costs. LO5: Identify the main types of manufacturing costs: direct materials, direct labor, and indirect production costs. LO6: Explain how the financial statements of merchandisers and manufacturers differ because of the types of goods they sell. LO7: Understand the main differences between traditional and activity-based costing systems and why ABC systems provide value to managers. LO8: Use activity-based cost information to make strategic and operational control decisions.

B2, B4

3, 18, 36, 36, 37, 38, 38, 39, 44, 45, 45, 49, 51, 52, 53 33, 41, 41, 46, 51, 52, 53 36, 36, 37, 38, 38, 39

..

B4

A2

54

57, 60, 61, 61

A3, A4, B3, B4

23, 33, 40, 41, 41, 42, 47, 48, 49, 50

55, 56, 57, 58, 59, 60, 61, 61

A4, B4

32, 35, 37, 43, 47, 48, 49, 50

55, 56, 60

127


CHAPTER 4 Cost Management Systems and Activity-Based Costing 4-A1

(20-30 min.)

See Table 4-A1 on the following page. 4-A2 (25-30 min.) 1.

Merchandise Inventories, 2,500 devices @ $75

$187,500

Direct materials inventory Work-in-process inventory Finished goods inventory, 2,500 units @ $75* Total inventories * Production costs = $900,000 / 12,000 units produced = $75/unit

$ 40,000 0 187,500 $227,500

2.

3. DANUBE ELECTRONICS PRODUCTS Statement of Operating Income For the Year Ended December 31, 20X9 Sales (9,500 units at $180) Cost of goods sold: Beginning inventory Purchases (12,000 units @ $75) Cost of goods available for sale Less ending inventory Cost of goods sold (an expense) Gross margin or gross profit Less other expenses: selling & administrative costs Operating income (also income before taxes in this example)

.

$1,710,000 $

0 900,000 $ 900,000 187,500 712,500 $ 997,500 170,000 $ 827,500

128


TABLE 4-A1 STATEMENT OF OPERATING INCOME EXTERNAL REPORTING PURPOSE Sales Cost of goods sold: Direct material Indirect manufacturing costs Total cost of goods sold Gross profit Selling and administrative expenses: Commissions Distribution to warehouses Total selling and admin. expenses Contribution to corporate expenses and profit Unallocated expenses: Administrative salaries Other administrative expenses Total unallocated expenses Operating income before tax

$307,900

OPERATING INCOME BY PRODUCT LINE INTERNAL STRATEGIC DECISION MAKING PURPOSE Custom Large Small Detailed Std. Std. Cost Type, Assignment Method $57,600 $58,300 $192,000

47,800 50,000 97,800 210,100

5,800 32,5001 38,300 19,300

19,000 5,000 24,000 34,300

23,000 Direct, Direct Trace 12,500 Indirect, Alloc. – Mach. Hours 35,500 156,500

14,800 10,900 25,700

1,200 1,0902 2,290

3,000 2,180 5,180

10,600 Direct, Direct Trace 7,630 Indirect, Allocation - Weight 18,230

184,400

$17,010

$29,120 $138,270

8,200 4,500 12,700 $ 171,700

1

Total machine hours is 1,365 + 210 + 525 = 2,100. Indirect manufacturing cost per machine hour is then $50,000 ÷ 2,100 = $23.80952. The allocation to custom detailed is $23.80952 × 1,365 machine hours = $32,500. 2 Total weight shipped is 28,500 kg + 57,000 kg + 199,500 kg = 285,000 kg. Indirect distribution costs per kilogram is then $10,900 ÷ 285,000 kg = $0.0382456. The allocation to custom detailed is $0.0382456 × 28,500 kg = $1,090.

..

129


4.

ANNANDALE, INC. Statement of Operating Income For the Year Ended December 31, 20X9 Sales (9,500 units at $180) Cost of goods manufactured and sold: Beginning finished goods inventory Cost of goods manufactured: Beginning WIP inventory $0 Direct materials used 490,000 Direct labor 200,000 Indirect manufacturing 210,000 Total mfg. costs to account for $900,000 Less ending work-inprocess inventory 0 Cost of goods available for sale Less ending finished goods inventory Cost of goods sold (an expense) Gross margin or gross profit Less other expenses: selling and administrative costs Operating income (also income before taxes in this example)

5.

$1,710,000 $0

900,000 $900,000 187,500 712,500 $ 997,500 170,000 $ 827,500

The balance sheet for the merchandiser (Danube) has just one line for inventories, the ending inventory of the items purchased for resale. The balance sheet for the manufacturer (Annandale) has three items: direct materials inventory, work-in-process inventory, and finished goods inventory. The income statements are similar except for the computation of cost of goods available for sale. The merchandiser (Danube) simply shows purchases for the year plus beginning inventory. In contrast, the manufacturer (Annandale) shows beginning work-in-process inventory plus the three categories of cost that comprise manufacturing cost (direct materials used, direct labor, and factory (or manufacturing) overhead) and then deducts the ending work-in-process inventory. The manufacturer then adds the beginning finished goods inventory to this cost of goods manufactured to get the cost of goods available for sale.

6.

..

The purpose is providing aggregate measures of inventory value and cost of goods manufactured for external reporting to investors, creditors, and other external stakeholders.

130


4-A3 (10-15 min.) There can be many justifiable answers for each item other than the listed cost driver and behavior. The purpose of this exercise is to generate an active discussion regarding those chosen by Silver Springs Bank’s managers. One point that should be emphasized is that many times managers choose cost drivers that are not the most plausible or reliable because of lack of data availability. Cost drivers are also used as a basis to allocate activity and resource costs and so the availability of data is often an important consideration.

a. * b.** c. d. e. f. g.*** h. i. j. k. l.

Activity Or Resource R R R A R R A A R R R A

Cost Driver Number of square feet Number of person hours Number of computer transactions Number of schedules Number of person hours Number of loan inquiries Number of investments Number of applications Number of person hours Number of minutes Number of person hours Number of loans

Cost Behavior F F V F V V V F

* An argument can be made that maintenance of the building is an activity. If this was the case, resources such as supplies and labor would be resources consumed, and several resource cost drivers would be needed. In addition, a separate resource and associated cost driver would be needed for insurance costs. However, the company had a contract for maintenance (fixed price), so this was a fixed-cost resource that was added to other occupancy costs such as insurance. The cost driver chosen for all these occupancy costs was square feet occupied by the various departments. ** Normally, the cost driver used for any labor resource is person hours. It is assumed that the staff person hours used are regular hours rather than overtime or temporary labor hours. Thus, the cost is fixed with respect to changes in hours used. As the hours used increases (decreases) the utilization of the resources increases (decreases) and eventually, management will need to make a decision whether to expand capacity (or whether to cut back on labor). This is an example of a step cost that is fixed over wide ranges of cost-driver level. *** Students may try to determine the cost behavior of activities even though the problem requirements do not ask for it. Point out that activities almost always have mixed cost behavior because they consume various resources. Some of these are fixed-cost and others variable-cost resources. For example, the activity “research to evaluate a loan application” consumes such fixed-cost resources as manager labor time and computers (assumed owned by the bank). This

..

131


activity also consumes variable-cost resources such as telecommunications time and external computing services.

..

132


4-A4

(20-30 min.)

1.

The first step is to determine the cost per cost-driver unit for each activity: Monthly Manufacturing Overhead $15,400 99,000 11,000 $125,400

Activity [Cost driver] Material Handling [Direct materials cost] Engineering [Engineering change notices] Power [Kilowatt hours] Total Manufacturing Overhead

CostDriver Activity $220,000 110 550,000

Cost per Driver Unit $ 0.07 900.00 0.02

Next, the costs of each activity can be allocated to each of the three products: PHYSICAL FLOW / ALLOCATED COST Cost Senior Basic Deluxe Material Handling $.07 × 25,000 = $1,750 $.07 × 60,000 = $ 4,200 $.07 × 135,000 = $ 9,450 Engineering $900 × 21 = 18,900 $900 ×20 = 18,000 $900 × 69 = 62,100 Power $.02 × 230,000 = 4,600 $.02 × 220,000 = 4,400 $.02 × 100,000 = 2,000 Total $25,250 $26,600 $73,550 2.

Overhead rate based on direct labor costs: Rate = Total manufacturing overhead ÷ Total direct labor cost = $125,400 ÷ $25,080 = $5.00/DL$ Overhead allocated to each product is: Senior: $5.00 × 14,546 Basic: $5.00 × 3,762 Deluxe: $5.00 × 6,772 Total

= = =

$ 72,730 18,810 33,860 $125,400

Notice that much less manufacturing overhead cost is allocated to Basic using direct labor as a cost driver. Why? Because Basic uses only a small amount of direct labor but large amounts of other resources, especially power. 3.

The product costs in requirement 1 are more accurate if the cost drivers are good indicators of the causes of the costs -- they are both plausible and reliable. For example, kilowatt hours is certainly a better measure of the cost of power costs than is direct labor hours. Therefore, the allocation of power costs in requirement 1 is certainly better than in requirement 2. Materials handling and engineering are likewise more plausible. A manager would be much more confident in the manufacturing overhead allocated to products in requirement 1. Remember, however, that there are incremental costs of data collection associated with the more accurate ABC system. The benefit/cost criteria must be applied in deciding which costing system is “best.”

..

133


4-B1

(20-30 min.)

See Table 4-B1 on the following page. 4-B2

(25-30 min.)

1.

$1,034,000 ÷ 50,000 hours = $20.68 per direct-labor hour

2.

(a) $540,000 ÷ 12,000 hours = $45 per direct-labor hour (b) $494,000 ÷ 38,000 hours = $13 per direct-labor hour

3.

(a) $540,000 ÷ 90,000 hours = $6 per machine hour (b) $494,000 ÷ 38,000 hours = $13 per direct-labor hour

4.

(a) Product A: $20.68 × (2.5 + 13.0) = $320.54 Product B: $20.68 × (3.0 + 3.3) = $130.28 Product C: $20.68× (2.7 + 8.5) = $231.62 (b) Product A: ($45 × 2.5) + ($13 × 13.0) = $112.50 + $169.00 = $281.50 Product B: ($45 × 3.0) + ($13 × 3.3) = $135.00 + $ 42.90 = $177.90 Product C: ($45 × 2.7) + ($13 × 8.5) = $121.50 + $110.50 = $232.00 (c) Product A: ($6 × 10.0) + ($13 × 13.0) = $ 60.00 + $169.00 = $229.00 Product B: ($6 × 18.0) + ($13 × 3.3) = $108.00 + $ 42.90 = $150.90 Product C: ($6 × 11.0) + ($13 × 8.5) = $ 66.00 + $110.50 = $176.50 (d) First consider using departmental instead of firm-wide rates (part b vs. part a). Departmental rates in Machining are much higher, so products that use relatively more time in machining (such as product B which uses almost half the direct labor hours in machining) will have higher costs under departmental rates while products that use relatively more time in assembly (such as product A) will have lower costs under departmental rates. Now examine changing to a base of machine hours in machining (part c vs. part a). Product B is the only one with an increase in cost in (c) compared to (a). Why? Because B uses less direct labor hours than A or C, so it is is allocated less of the costs on the basis of direct labor hours. But it uses more machine hours than either A or C. Therefore, it receives relatively more costs with a base of machine hours than with a base of direct-labor hours.

..

134


TABLE 4-B1 STATEMENT OF OPERATING INCOME

Sales Cost of goods sold: Direct material Indirect manufacturing Total Cost of Goods Sold Gross profit Selling and administrative expenses: Commissions Distribution to warehouses Total selling and administrative expenses Contribution to corporate expenses and profit Unallocated expenses: Corporate salaries Other general expenses Total unallocated expenses Operating income before tax

$1,221,500

OPERATING INCOME BY PRODUCT LINE Lawn Hand Scooter Mower Tool Cost Type, Parts Parts Parts Assignment Method $560,000 $342,300 $319,200

410,000 60,000 470,000 751,500

145,000 44,750 1 189,750 370,250

175,000 8,250 183,250 159,050

90,000 Direct, Direct Trace 7,000 Indirect – Mach.Hours 97,000 222,200

73,900 135,000 208,900 542,600

30,100 12,000 2 42,100 $328,150

21,000 101,000 122,000 $ 37,050

22,800 Direct, Direct Trace 22,000 Indirect - Weight 44,800 $177,400

10,000 17,000 27,000 $ 515,600

1

Total machine hours is 8,950 + 1,650 + 1,400 = 12,000. Indirect manufacturing cost per machine hour is then $60,000 ÷ 12,000 = $5. The allocation to scooter parts is $5 × 8,950 machine hours = $44,750. 2 Total weight shipped is 60,000 kg + 505,000 kg + 110,000 kg = 675,000 kg. Indirect distribution costs per kilogram is then $135,000 ÷ 675,000 kg = $0.20. The allocation to scooter parts is $0.20 × 60,000 kg = $12,000.

..

135


4-B3

(30-35 min.)

1. The existing system allocates all costs based on direct labor cost. The rate for allocating indirect production costs is: Estimated indirect production cost ÷ Estimated direct labor cost = ¥21,276,000 ÷ ¥28,368,000 = 75% That is, each time ¥1 is spent on direct labor, Kondo adds ¥0.75 of indirect production cost to the cost of the product. 2. Under an ABC system, Kondo would allocate indirect production costs separately for each activity. This would result in the following four allocation rates: Receiving: Receiving costs ÷ Direct material cost =¥6,600,000 ÷ 55,000,000 = ¥0.12 per ¥1of direct materials Assembly: Assembly costs ÷ Number of control units =¥9,900,000 ÷ 99,000 = ¥100 per control unit Qual. Control: Quality control cost ÷ QC hours =¥1,456,000 ÷ 560 = ¥2,600 per QC hour Shipping: Shipping cost ÷ # of boxes shipped =¥3,320,000 ÷ 8,300 = ¥400 per box shipped 3. (a) The cost will contain 3 components (in thousands of yen): Direct material ¥ 4,500 Direct labor 1,800 Indirect production cost (¥1,800 × .75) 1,350 Total cost ¥ 7,650 Price (¥7,650 × 1.4) ¥10,710 (b) The cost will have 7 components, 4 of them allocating indirect production costs (totals in thousands of yen): Direct materials Direct labor Receiving (¥0.12 × 4,500) Assembly (¥100 × 9,000) Quality control (¥2,600 × 60) Shipping (¥400 × 670) Total indirect production cost Total cost Price (8,164 × 1.4)

..

¥ 4,500 1,800 ¥540 900 156 268 1,864 ¥ 8,164 ¥11,429.60

136


4. The order from Mazda requires a relatively large amount of receiving, quality control, and shipping resources compared to the relative amount of labor required. This makes its allocated indirect production costs based on ABC higher than the allocated cost in the traditional system where the allocation is based on the labor required. The following illustrates why an allocation on the basis of labor cost results in less costs than allocations based on the four activities: Budgeted CostMazda CostMazda Activity Allocation Base Allocation Base Percentage Direct materials 55,000,000 4,500,000 8.2% Direct labor 28,368,000 1,800,000 6.3 Receiving 55,000,000 4,500,000 8.2 Assembly 99,000 9,000 9.1 Quality control 560 60 10.7 Shipping 8,300 670 8.1 Using the single direct-labor cost-allocation base, this order would receive 6.3% of all indirect production costs. The main reason that indirect production costs for this order under the ABC system are relatively high is the large relative cost of materials that drives a larger allocation (8.2%) of receiving costs to this order. The allocations of assembly, quality control, and shipping costs are also larger that they would be using a direct-labor cost-allocation base. 4-B4

(50-60 min.)

1. A summary of the analyses follows. Base Gross Profit Percentage* Plan Gross Profit Percentage** Support of Product Manager? Support of President?

Pen Casings 1.25% 10.80% Strong

Cell-Phone Casings 38.75% 37.20% None

Company 8.07% 17.40% Moderate

* See Exhibit 4-6 in the text. ** See Panel B of Exhibit 4-B4 that follows. Exhibit 4-B4, Panels A and B on the following pages can be used to explain the impact of the controller’s idea using the process map of the traditional costing system and the related financial reports. Notice that the $80,000 annual decrease in the cost of engineers needs to be converted to a $20,000 quarterly savings. The controller’s idea will result in an increase of 9.55 percentage points in the gross profit of the pen-casings line but a decrease of 1.55 percentage points in the cell-phone line. The product manager of pen casings would probably give strong support to the idea but the cell-phone casings manager would most likely not support the idea. Although the company-level gross profit margin improves, the president’s support may not be strong. Why? There is not a strong consensus among product-line managers. Top management is normally hesitant to support actions that do not have the unanimous support among product-line managers unless there is solid evidence of material improvement in profitability. While the current loss would be reversed, the return on sales is still nominal at 3,500 ÷ $480,000 = .73%. ..

137


Exhibit 4-B4: Panel A Process Map of Traditional Cost System

All Indirect Resources $200,000

All Unallocated Value Chain Costs $80,000

Cost Driver [Direct Labor Hours = 4,500 + 1,000 = 5,500]

Direct Materials for Pen Casings $22,500

Direct Labor for Pen Casings $135,000

Pen Casings Sales $360,000

..

Direct Materials for Cell Phone Casings $24,000

Direct Labor for Cell Phone Casings $15,000

Cell Phone Casings Sales $120,000

Unallocated $80,000

138


Exhibit 4-B4: Panel B PRO-FORMA FINANCIAL REPORTS: TRADITIONAL COST ALLOCATION SYSTEM STATEMENT OF OPERTING INCOME

CONTRIBUTION TO CORPORATE COSTS AND PROFIT [INTERNAL STRATEGIC DECISION MAKING AND OPERATIONAL-CONTROL PURPOSE]

[EXTERNAL REPORTING PURPOSE]

Pen Casings

Cell Phone Casings $120,000 1

Sales Cost of goods sold:

$480,000

$360,000

Direct material Direct labor

46,500 150,000

22,500 135,000

Indirect manufacturing Cost of goods sold Gross profit Corporate expenses (unallocated) Operating income

200,000 396,500 83,500 80,000 $ 3,500 17.40% 5

163,636 3 321,136 $ 38,864

75,364 $ 44,636

10.80%

37.20%

Gross profit margin

24,000 2 15,000 36,364 4

1. $80,000 × .75 × 2 2. $12,000 × 2 3. $200,000 × [4,500/(4,500 + 1,000)] 4. $200,000 × [1,000/(4,500 + 1,000)] 5. $83,500/$480,000

..

139


Perhaps the most important factor bearing on the president’s support is lack of confidence in the accuracy of the cost and hence gross margin figures. She probably will inquire whether the shift in the consumption percentages by the two activities is captured by the traditional costing system. Does the change in allocation rates from 90:10 to 82:18 based on direct labor hour changes accurately capture the impact of the operational changes? An informal analysis of the controller’s idea might look like the following table.

Operational Change Less purchasing work to supply parts for cell-phone casings Less engineering design work on cell phone casings Less equipment used to support cell-phone production Increase in cell-phone production

Likely Impact on the Consumption of Resources that Support: Cell-Phone Pen Casings Casings    

Based on the informal analysis, the President probably would expect the profitability of cellphone casings to improve and the profitability of pen casings to be unaffected. This disagrees with the numerical analysis. Given the propensity of managers to embrace numerical results, less weight will likely be given this analysis compared to the “objective” numbers. As a result, she may question the validity of the numerical analysis as well as the value of the traditional costing system! Finally, the focus of improvement efforts should be directly on the pen-casing product line. This initiative deals mostly with the cell-phone line. What can be done to improve profitability of the pen casings? Can prices be raised without losing too much volume? Can operational improvements be made to lower the indirect manufacturing costs? The controller’s idea is worthy of some support but it does not address the profitability issue head on. 2. Exhibit 4-B4, Panel C on the following page is a process map that can be used to explain the impact of the controller’s idea. Panel D at the end of the solution provides a detailed evaluation of the controller’s idea. Pen Cell-Phone Casings Casings Company Base Gross Profit Percentage* 16.22% (28.63%) 8.07% Plan Gross Profit Percentage** 17.26% 17.81% 17.40% Support of Product Manager? Neutral Strong Support of President? Strong * See the table on p. 141 of the text. ** See panel D of Exhibit 4-B4 that follows.

..

140


The controller’s idea will result in a slight increase in the gross profit of the pen-casings line but a dramatic turnaround in the profitability of the cell-phone line. The product manager of pen casings would probably be neutral or slightly positive about the idea because the idea does not focus on operational improvements that directly affect the pen-casings line. The cell-phone casings manager would give strong support to the idea – this may save his/her job! The president would strongly support this idea while encouraging all managers involved to keep up the good work. Also, note that the numbers agree with the informal analysis – generating confidence in the integrity of the cost accounting system. Exhibit 4-B4, Panel C Process Map for ABC System

Plant & Machinery $180,000

50% 80% Processing Activity $154,000 [Direct Labor Hours = 4,500 + 1,000] Direct Materials for Pen Casings $22,500

Direct Labor for Pen Casings $135,000

SALES $360,000

..

All Unallocated Value Chain Costs $80,000

Engineers & CAD Equip. $20,000

50% 20% Production Support Activity $46,000 Cost Driver [Distinct Parts = 5 + 11] Direct Materials Direct Labor for Cell Phone for Cell Phone Casings $24,000 Casings $15,000

SALES $120,000

UNALLOCATED $80,000

141


Exhibit 4-B4: Panel D PRO-FORMA FINANCIAL REPORTS FOR LOPEZ PLASTICS COMPANY: ACTIVITY-BASED COST ALLOCATION SYSTEM STATEMENT OF OPERTING INCOME

CONTRIBUTION TO CORPORATE COSTS AND PROFIT [INTERNAL STRATEGIC DECISION MAKING AND OPERATIONAL-CONTROL PURPOSE]

[EXTERNAL REPORTING PURPOSE]

Sales Cost of goods sold: Direct material Direct labor

$480,000

Pen Casings $360,000

Cell Phone Casings $120,000

46,500 150,000

22,500 135,000

24,000 15,000

Processing activity

154,000

Production support activity Cost of goods sold Gross profit Corporate expenses (unallocated) Operating loss Gross profit margin

46,000 396,500 83,500 80,000 $ 3,500 17.40%

126,000 1 14,375 3

28,000 2 31,625 4

1. 2. 3. 4.

..

297,875 $ 62,125

98,625 $ 21,375

17.26%

17.81%

$154,000 × [4,500 labor hours/(4,500 labor hours + 1,000 labor hours)] $154,000 × [1,000 labor hours/(4,500 labor hours + 1,000 labor hours)] $46,000 × [5 distinct parts/(5 distinct parts + 11 distinct parts)] $46,000 × [11 distinct parts/(5 distinct parts + 11 distinct parts)]

142


3. As vice president, you probably are pleased with the new ABC system. The cost drivers that are used to allocate activity costs appear to be plausible and reliable and thus probably represent a sound cause-effect model of operations. This will improve both the accuracy of product costing and operating managers’ control over costs. Operating managers will be pleased with the ABC system because it helps them understand how their day-to-day work impacts costs and profits. From a behavioral perspective, this should be highly motivational. This problem emphasizes the importance of the cost-accounting system to managers. Different systems can result in significantly different management decisions. In this case, the product-line managers’ support for the controller’s idea changes when an ABC system is used to evaluate the idea. Although the company-level gross margins do not change, it is possible that the president would strongly support the idea based on ABC data. Why? Neither of the product-line managers is against the idea, and one strongly supports it. In addition, the president may have more confidence in the accuracy of the ABC analysis. The substantial losses of the current quarter have been completely eliminated and the serious profitability problem of the cell-phone casing product line has been reversed. 4-1

A cost management system is a collection of tools and techniques that identifies how management’s decisions affect costs. The three purposes of a CMS are to provide 1. cost information for operational control, 2. cost information for strategic decisions, and 3. measures of inventory value and cost of goods manufactured (or purchased) for external reporting to investors, creditors, and other external stakeholders.

4-2

a. The production manager needs operational control information. b. Setting the product mix is a strategic decision. c. The cost of inventory that appears on the balance sheet is information that is used by external investors, creditors, and other stakeholders.

4-3 Cost objects are any items for which decision makers desire a separate measurement of costs. Examples include departments, products, services, territories, customers, and activities such as processing orders or moving materials. 4-4 No. Products are one of the main cost objects for most companies, but departments are also important cost objects because they represent a logical grouping of activities for which managers desire a separate determination of costs. 4-5 The major purpose of a detailed cost-accounting system is to measure costs for decision making and financial reporting. Cost accounting systems become more detailed as management seeks more accurate data for decision making.

..

143


4-6 The two major processes performed by a cost accounting system are cost accumulation and cost assignment. Cost accumulation is collecting costs by some “natural” classification, such as materials or labor, or by activities performed such as order processing or machine processing. Cost assignment is attaching costs to one or more cost objects, such as activities, processes, departments, customers, or products. 4-7 Managers make important decisions on a daily basis. They base these decisions in large part on financial data provided by the cost accounting system. So it is critically important that the cost accounting system provide accurate and reliable financial information. 4-8 Managers can specifically and exclusively identify direct costs with a given cost object (that is, directly trace them) in an economically feasible way. Indirect costs cannot be so identified. However, managers can usually identify a plausible and reliable cost driver to use to allocate resource costs to cost objects that consume the resources. When direct tracing is not economically feasible and a plausible and reliable cost driver cannot be found, costs should remain unallocated. 4-9 Yes, the same cost (for example, the department supervisor's salary) can be direct with respect to a department but indirect with respect to the variety of products flowing through a department (e.g., tables, chairs, and cabinets). 4-10 Some costs can be physically linked with a department (or a product), but not in an economically feasible way. An example is the use of departmental meters for measuring power usage. Such devices could measure power costs as direct costs of a department. The alternative is to regard factory power costs as indirect costs of individual departments. Managers often decide whether the resulting increased accuracy provided by individual power meters is worth their additional cost; thus, the test of economic feasibility will decide whether a particular cost is regarded as direct or indirect. 4-11 The four purposes of cost allocation are (1) to predict the economic effects of strategic and operational control decisions, (2) to provide desired motivation and give feedback for performance evaluations , (3) to compute income and asset valuations, and (4) to justify costs or obtain reimbursement. 4-12 Generally Accepted Accounting Principles (GAAP) require publically-held companies to allocate all production-related costs and only production-related costs to its products for financial reporting to the public. 4-13 No. The costs in a cost pool are not physically traced to cost objects. Only direct costs are traced to cost objects. A cost pool contains indirect costs that are allocated to cost objects using a single cost-allocation base. 4-14 Some possible terms are reallocate, assign, distribute, redistribute, load, apportion, reapportion, and burden.

..

144


4-15 For financial statement purposes, the typical accounting system allocates only production costs to the physical units produced. For guiding decisions regarding product-pricing and product-mix decisions, many companies allocate all costs, including R&D, design, marketing, distribution, and customer service costs. However, the allocations of these costs may not be embedded in the system that generates financial statements. 4-16 Yes. The two criteria that should be met before using any measure as a cost-allocation base are economic plausibility and reliability. A measure should be plausible – make common sense. If managers cannot easily understand the logical relationship between a cost allocationbase and the costs of an activity or resource, managers will perceive the resulting allocations as arbitrary. 4-17 Production maintenance costs are normally indirect. Sales commissions normally can be directly traced to specific products. The costs associated with process design are normally unallocated because it is too difficult to identify plausible and reliable cost-allocation bases, although some companies elect to allocate them. 4-18 “Tracing of costs” refers to the process of identifying the amount of direct cost that relates exclusively to a particular cost object. This is possible for direct costs. “Cost allocation” is the process of assigning indirect costs to cost objects in proportion to the cost object’s use of a particular cost allocation base. 4-19 Depreciation related to production activities is a product cost, not a period cost. Hence, it will become an expense as a part of manufacturing cost of goods sold. Thus, depreciation is not always an immediate expense. Depreciation not related to production activities is a period cost. 4-20 "Expenses" denote all costs deducted from (matched against) revenues in a given period. "Costs" is a much broader term; for example, "cost" is used to describe an asset (the cost of inventory or the cost of plant and equipment) or an expense (the cost of goods sold). 4-21 Manufacturing is the transformation of materials into other goods through the use of labor and factory facilities. In contrast, merchandising companies (retailers or wholesalers) sell goods without changing their basic forms. Manufacturing companies differ from merchandising companies in the way they accumulate and report the cost of inventories. Merchandising companies show inventories at the cost they pay to acquire them. In contrast, manufacturing companies classify production costs as either (1) direct material, (2) direct labor, or (3) indirect production costs, and they use the accounting system to determine how much cost of each type should be assigned to product inventories. 4-22 Direct-labor costs are incurred at the same time the direct labor is used in production. Unlike direct materials, it is not purchased and stored for future use. Therefore, there is no direct-labor inventory account.

..

145


4-23 1. The organization manufactures hundreds or thousands of products. 2. Indirect production costs constitute a large percentage of total costs. 3. Different products consume resources at widely varying rates. 4. The cause-and-effect relationship between the cost and the cost drivers can be identified precisely. 4-24 Activity-based management is using activity-based cost information to improve the operations of an organization. Managers use ABC information for decision making, planning, and control purposes. Cost information is vital for each of these purposes. The accuracy level of the cost information is a critical factor in determining the effectiveness of decision making, planning, and control. 4-25 One of the most significant non-value-added activities in any manufacturing company is moving inventory, materials, and parts from one point to another during the production process. The time it takes for material-handling labor to move material can be reduced by changing the layout of production equipment. For example, one company changed its layout so several machines that were used to produce about 50% of its products were placed next to one another. This layout change reduced the distance and time required to move partially completed products from one machine to another. The cost savings were significant. 4-26 Managers seek to eliminate, or at least reduce as much as possible, non-value-added activities. Separating these from value-added activities helps focus attention on the costs to be examined for potential reductions. 4-27 Benchmarking is the continuous process of comparing products, services, and activities against the best industry standards. Companies use benchmarking as a tool to help measure their competitive posture. Benchmarks can come from within the organization, from competing organizations, or from other organizations having similar processes. 4-28 No two businesses operate in the same manner, and often their competitive environments are significantly different. As a result, comparing either financial or operational measures to benchmarks under the assumption that “all things are equal” should be done with caution. Another important difference between businesses is the degree of accuracy of their cost accounting system. A financial measure such as “cost to serve a commercial customer” in a bank that allocates almost all of its operating costs should not be compared to a benchmark measure from another bank that allocates only a small portion of its operating costs.

..

146


4-29 Six factors that explain why more and more organizations are adopting activity-based costing systems are: 1.

Fierce competitive pressure has resulted in shrinking margins, making accurate cost determinations essential. While companies may know that their overall margin is shrinking, they often do not have faith in the accuracy of the margins for individual products or services. Some are winners and some are losers -- but which ones?

2.

Business complexity has increased, resulting in greater diversity in the types of products and services as well as customer classes. This means that the consumption of a company’s shared resources also varies substantially across products and customers.

3.

New production techniques have increased the indirect proportion of total costs -- that is, indirect costs are far more important in today’s world-class manufacturing environment. In many industries direct labor is being replaced by automated equipment. It is not unusual for indirect cost to be more than 50% of total cost.

4.

The rapid pace of technology change has shortened product life cycles. This means that companies do not have time to make price or cost adjustments once costing errors are discovered.

5.

The costs associated with bad decisions that result from inaccurate cost determinations are substantial (bids lost due to over-costed products, hidden losses from under-costed products, failure to detect activities that are not cost effective, etc.).

6.

Computer hardware and software technology has reduced the costs of developing and operating ABC systems that track many activities.

4-30

The four steps are:

Step 1. Determine the key components of the ABC system (cost objects, key activities, resources, and related cost drivers). Step 2. Determine the relationships among cost object, activities, and resources. Step 3. Collect relevant data concerning costs and physical flow of cost-driver units among resources and activities. Step 4. Calculate and interpret the new activity-based cost information. 4-31 Whenever a resource is constraining the capacity to meet demand, a company can take one or more of the following actions: a. Reduce demand for the resource. In this case, this means either saying “no” to the increased business or deferring business (this may not be feasible). b. Increase capacity. The company can hire additional staff, outsource part of its order processing function, or permit overtime. c. Institute process improvements that reduce the consumption of the capacityconstraining resource. The company can investigate ways to reduce the resource consumption rates.

..

147


4-32 Activity-based management is using the output of an activity-based cost accounting system to aid strategic decision making and to improve operational control of an organization. Therefore, ABC produces the information used in ABM. If managers in companies with ABC systems do not use that information to aid decision making, they are sacrificing many (if not most) of the benefits of the investment in an ABC system. 4-33 No. The main reasons that activity-based costing is becoming so popular relate to planning and control, not product costing, including (1) decisions about product mix, prices, and other product-related decisions, and (2) control of costs focused on managing activities instead of products. The term activity-based management refers to the use of activity-based costs for planning and control purposes. 4-34 In such a company the ABC system would not produce more accurate product costs. However, cost planning and control (the strategic and operational purposes of a cost management system) require information about processes, activities, and resources regardless of how many or few products are made. Cost reduction programs are most effective when managers understand the interrelationships between activities and resources. Thus, an ABC system often contributes significantly to an effective planning and control environment, even in a single-product firm. 4-35 Benchmarking financial measures should be done with care. Many factors outside the influence of responsible managers can reduce the comparability of performance measures. For example, labor costs can vary substantially across regions. If the local labor rates in Youngstown are low compared to rates in Los Angeles, Youngstown may have lower cost per driver unit for those activities that are labor intensive such as processing deposits and withdrawals. Another factor is the scope of the ABC system. The Youngstown area billing center may have chosen not to allocate many indirect costs that the Los Angeles center allocates. This would lower the costs accumulated in the Youngstown activity-cost pools and therefore lower the resulting cost per driver unit.

..

148


4-36

(10 min.)

This exercise demonstrates how a given cost item may be seen from different viewpoints. This will compel students to think instead of memorizing. Exceptions can be cited for nearly every answer so class discussion may be prolonged. To accomplish the purpose of these problems, it is not necessary to reach agreement on every item. Cost Items Manufacturing Costs Direct (D) or Variable (V) Indirect (I) or Fixed (F) 1. Factory lighting excluding meter rental I V 2. Salary of storeroom security personnel I F 3. Brick for a road constructor D V 4. Factory supervisor’s salary I F 5. Annual maintenance contract fee for factory equipment I F 6. Food supplies for factory canteen I V 7. Group insurance premium for factory workers I F 8. Lubricant for factory machines I V 9. Aluminium tubing in bicycles D V 10. Power to operate factory machine I V 4-37

(10 min.)

1. The cost of components used in products is almost always directly traceable and is a variable cost. 2. As volume changes over a wide range, the amount of supplies consumed for maintenance will also change so this cost is variable. However, there usually would not be an economically feasible way to trace these costs to individual products, so these costs would be indirect and allocated (often the cost driver would be machine hours). 3. The wages of machine operators who work on only one product can be easily traced to the product. If these wages would not vary over wide ranges of volume, they would be a fixed cost. If increases in production require paying for additional wages for machine operators, they would be variable. We should note that if volume increases rapidly in a short time frame, it is often necessary to work overtime. In this case, the overtime portion of operator wages would be a variable cost even if the base wage is fixed. 4. Training costs for mechanics would not vary as a function of volume of production assuming that no new products would be made and that no new mechanics would be hired. Training costs are indirect assuming that the training cannot be associated with only one product.

..

149


4-38

(20 min.)

1. 2. 3. 4. 5. 6.

d, f b, f d, f b, e d, f d, e

4-39

(10 min.)

7. 8. 9. 10. 11. 12.

c, e b, e c, e b, f a, f b, f

13. 14. 15. 16. 17. 18.

c, f c, e b, e b, e c, e a, f

Using Exhibit 4-7 and the chapter discussion of the Lopez Plastics production and cost accounting system, the classification of these costs and resources is: 1. 2. 3. 4. 5. 6. 7. 8.

..

I D I I I or U I D U

150


4-40

(20-25 min.)

The first step is determining the revised activity-cost pool amounts (stage 1). Processing Activity Cost = .90 × $180,000 + .30 × $40,000 = $162,000 + $12,000 = $174,000 Production Support Cost = .10 × $180,000 + .70 × $40,000 = $18,000 + $28,000 = $46,000

Sales Cost of goods sold: Direct material Direct labor Processing Production support Cost of goods sold Gross margin

Contribution to Corporate Costs and Profit Pen Casings Cell Phone Casings $360,000 $80,000 22,500 135,000 156,600 1 9,200 2 323,300 $ 36,700

12,000 15,000 17,400 3 36,800 4 81,200 $ (1,200)

1 $174,000 × .9 2 $46,000 × .2 3 $174,000 × .1 4 $46,000 × .8

..

151


4-41

(20-30 min.)

Calculation of total production overhead: Magnetic Chess: Magnetic Ludo: Magnetic Monopoly:

1,500 x 1.5 x 28 = CNY 63,000 2,500 x 1 x 28 = CNY 70,000 14,000 x 3 x 28 = CNY 1,176,000 CNY 1,309,000 Overhead to be apportioned in different activities: Setup costs 1,309,000 x 0.35 = CNY 4,58,150 Machine-related costs 1,309,000 x 0.20 = CNY 261,800 Material handling cost 1,309,000 x 0.15 = CNY 196,350 Testing and inspection costs 1,309,000 x 0.30 = CNY 392,700 Calculation of overhead absorption rates: Overhead Costs Cost Cost Driver Units No. of Cost Cost per Driver (CNY) Driver Units Unit (CNY) Setup costs Machine-related costs Material handling costs Testing and Inspection costs Total * Machine hours: Magnetic Chess: Magnetic Ludo: Magnetic Monopoly:

458,150 261,800 196,350 392,700 1,309,000

Setups Machine hours* Material moves Tests and inspection

1,340 46,750 240 2,000

341.90 5.60 818.13 196.35

1,500 x 1.5 = 2,250 2,500 x 1.0 = 2,500 14,000 x 3.0 = 42,000 46,750

Cost allocation under ABC (in CNY): Particulars

Magnetic Magnetic Magnetic Chess Ludo Monopoly Setup costs @ CNY 341.90 per set up 51,286 78,638 328,226 Machine-related costs @ CNY 5.60 per m.hr. 12,600 14,000 235,200 Material handling costs @ CNY 818.13 per move 19,635 34,361 142,354 Testing and Inspection costs @ CNY 196.35 per test 58,905 70,686 263,109 and inspection Total 142,426 197,685 968,889 Volume (units) 1,500 2,500 14,000 Overhead cost per unit (CNY) 94.95 79.07 69.20 Calculation of cost per unit under ABC system (in CNY) Particulars Magnetic Chess Magnetic Ludo Magnetic Monopoly Direct materials 40 24 50 Direct labor @ CNY 12 per 6 18 12 labor hour Production overhead: 94.95 79.07 69.20 140.95 121.07 131.20 ..

152


4-42

(15-20 min.)

Activity Open New Accounts

Process Deposits and Withdrawals

Process Other Transactions

Cost $370,000

Consumption Rate 20%

Allocated Cost $74,000

Total Cost

Resource Tellers Retail Sales Managers Tellers

$220,000

15%

$33,000

$107,000

$370,000

55%

$203,500

$220,000

15%

$33,000

$120,000

10%

$12,000

$370,000

15%

$55,500

$220,000

30%

$66,000

$120,000

35%

$42,000

Retail Sales Managers Managing Officer Tellers Retail Sales Managers Managing Officer

..

$248,500

$163,500

153


4-43 1.

(20-25 min.)

Activity Open new accounts Process deposits and withdrawals Process other transactions

Total Cost $ 107,000

Annual Flow of Driver Units 580

Cost Per Driver Unit $184.4827

248,500

166,000

1.4970

163,500

46,000

3.5543

Using these rounded costs per driver units, we get the following rounded costs per account:

Open new accounts Process deposits and withdrawals Process other transactions Total cost Number of accounts Cost per account

Retail Customers

Commercial Customers

Cost Per Driver Unit

Driver Units

$184.4827

560

$ 103,310

20

$ 3,690

1.4970

77,000

115,269

89,000

133,233

3.5543

24,000

85,303 $303,882 2,700 $ 112.55

22,000

78,195 $215,118 400 $537.79

Cost

Driver Units

Cost

2. Benchmarking is not appropriate when different costing systems are used, when benchmarks represent costs from branches that allocated significantly different resources or when benchmark are from branches with different defined activities. 3. The comparison below indicates that the Maple Lake branch performed extremely well, much better than the lowest cost across other branches. This assumes that the benchmarks used are appropriate. Open Process Process Retail Commercial New Dep. And Other Cost Per Cost Per Accounts With. Trans. Account Account Benchmark $204.73 $2.25 $3.83 $ 126.85 $593.83 Maple Lake $184.48 $1.497 $3.55 $112.55 $537.79

..

154


4-44

(20-30 min.)

Answers can vary depending on the assumptions. For example, some companies would consider the order processing activity part of the distribution function in the value chain and would include this activity in the new ABC system. Others would consider this a support activity and leave it unallocated. Similarly, production scheduling can be considered part of the production function and thus be allocated under both traditional and ABC systems. Also, note that the costallocation bases for the resource costs are not the same as those for the activity. For example, the cost-allocation base for the activity order processing of customer orders might be number of orders, whereas the cost-allocation base for the salaries of order processing staff is staff labor hours used. Activity Setting up for a production run Purchasing materials and parts to be used in products Shipping sold products to customers (distributors)

Related Cost Mechanic wages

Traditional Indirect (direct labor hours) Materials and parts Direct trace cost Fuel used on Unallocated company’s fleet of (period cost) trucks Market research study Salaries of market Unallocated research staff (period cost) Production scheduling Salaries of Indirect (direct production labor hours) scheduling managers Purchasing materials and Salary of Indirect (direct parts to be used in products purchasing agents labor hours) Order processing of customer Salaries of order Unallocated orders processing staff Preparing cost analyses

Salary of the cost accountant

Designing a new product

Salaries of design engineers who are fully dedicated to this new product Salary of the executive

Managing overall operations of the company

..

Unallocated (administrative cost) Unallocated (R&D cost, which is a period cost) Unallocated (administrative cost)

ABC Indirect (mechanic labor time) Direct trace Indirect (miles) Unallocated (R & D function not allocated) Indirect (hours of prod. scheduling managers) Indirect (material cost) Indirect (assuming this is part of the distribution function) (number of hours) Unallocated Direct trace

Unallocated

155


4-45 1.

(20-30 min.) Calculation of department-wise costs:

Particulars Direct materials Direct labor Indirect production costs

Component Manufacturing (¥) 480,000 (800,000 x 60%) 36,000 (180,000 x 20%) 20,000 536,000

Assembling (¥)

Total (¥)

320,000 (800,000 x 40%) 144,000 (180,000 x 80%) 60,000 524,000

800,000 180,000 80,000 1,060,000

Calculation of total costs of refrigerators: Particulars

Direct Materials (¥)

Direct Labor (¥)

Component manufacturing

192,000 14,400 (480,000 x 40%) (36,000 x 40%) Assembling 160,000 57,600 (320,000 x 50%) (144,000 x 40%) Total 352,000 72,000 Total cost of refrigerator = ¥(352,000 + 72,000 + 32,000) = ¥456,000

Indirect Production Costs (¥) 8,000 (20,000 x 40%) 24,000 (60,000 x 40%) 32,000

Calculation of total costs of air conditioners Particulars Component manufacturing Assembling Total

Direct Materials (¥)

Direct Labor (¥)

192,000 (480,000 x 40%) 128,000 (320,000 x 40%) 320,000

7,200 (36,000 x 20%) 57,600 (144,000 x 40%) 64,800

Indirect Production Costs (¥) 4,000 (20,000 x 20%) 24,000 (60,000 x 40%) 28,000

Total cost of air conditioner = ¥ (320,000 + 64,800 + 28,000) = ¥ 412,800 Calculation of total costs of washing machines: Particulars Component manufacturing

Direct Materials (¥)

Direct Labor (¥)

96,000 14,400 (480,000 x 20%) (36,000 x 40%) Assembling 32,000 28,800 (320,000 x 10%) (144,000 x 20%) Total 128,000 43,200 Total cost of air conditioner = ¥ (128,000 + 43,200 + 20,000) = ¥ 191,200 ..

Indirect Production Costs (¥) 8,000 (20,000 x 40%) 12,000 (60,000 x 20%) 20,000

156


4-46

(20 min.)

This problem is based on a description in Cost Finding and Rate Setting for Hospitals (Chicago: American Hospital Association), p. 6 and p. 74. It illustrates the idea of using a weighted average, which is really a version of equivalent units (explained in Chapter 14 on process costing). If process costing is covered in this course, the applicability of equivalent units in a hospital context is important to see. The cost driver to be used as a cost-allocation base would be "weighted number of pounds processed" instead of "number of pounds processed." The new computations are: Laundry department costs Weighted number of pounds processed: Radiology, (7,000 × 5) and (16,000 × 1) Other centers, 784,000 - 23,000 Divided by total weighted pounds Cost per weighted pound ($227,360 / 812,000 lbs.)

$227,360 51,000 761,000 812,000 $ .28

Total costs to radiology: Using unweighted base, 23,000 × $.29 Using weighted base, 51,000 × $.28

$ 6,670 $14,280

The practical problems of cost allocation are described in the cost-finding publication: One basis for the allocation of laundry costs is pounds of soiled laundry. This is a good basis provided it is possible for the hospital to identify the source of soiled linens by department. However, because of the use of linen chutes, or for other reasons, many hospitals cannot identify the source of laundry by department. In smaller hospitals the use of an outside commercial laundry service also is common, and some hospitals do not have weighing facilities. In every hospital there is a linen distribution function, and a study of the amount and type of linen distributed to the various departments will produce a reliable basis for allocation. As a minimum, the number of pieces, adjusted for weight differentials, should be tallied for a period of perhaps two weeks, two or three times a year. The weight factor, however, needs to be established only once. Reliable weight factors can be developed by weighing the various pieces and establishing standard piece weights for each type of laundry. The average of the several two-week tests can be used as the basis for distributing laundry costs for cost-finding purposes. Another method is to use laundry prices as a weighting factor for the different types of pieces. This method has the advantage of allowing for the relative difficulty of different processing functions, such as pressed versus mangle finishing.

..

157


Hospitals can conduct very refined studies if they so desire. It may be appropriate, especially in larger institutions, to give special consideration to refinements. For example, where the laundry processes a large volume of uniforms for student nurses and doctors, and for dietary, housekeeping, and other service personnel, it may be desirable to separate washing and pressing costs and to develop separate allocation bases for each of these functions based on the production statistics. Furthermore, it may be desirable to consider the effect of different processing cycles, such as the special rinse cycles for operating room linens or the special cold wash cycles for woolen blankets. Through discussions with the laundry manager, it is possible to determine whether further analysis is justified. Another example of possible refinements is the personal laundry done for medical interns, residents, student nurses, and others; if the volume is large, it might have to be recognized in the departmental operations. 4-47

(50-60 min.)

1. A summary of results follows. Base Gross Profit Percentage* Plan Gross Profit Percentage** Support of Product Manager? Support of President?

Pen Casings 1.25% 6.25% Moderate

Cell-Phone Casings 38.75% 36.17% None

Company 8.07% 13.73% Moderate

* See panel A of Exhibit 4-6 on p. 138 of the text. ** See Panel B of Exhibit 4-47 that follows. Panels A and B of Exhibit 4-47 on the following page can be used to explain the impact of the controller’s idea using the process map of the traditional costing system and the related financial reports. The controller’s idea will result in an increase of 5 percentage points in the gross profit of the pen-casings line but a decrease of about 2.6 percentage points in the cell-phone line. The product manager of pen casings would probably give moderate support to the idea but the cellphone casings manager would likely not support the idea. Although the company-level gross profit margin improves, the president’s support may not be strong. Why? Top management is normally hesitant to support actions that do not have total support among product-line managers unless there is solid evidence of material improvement in profitability. While the overall gross margin percentage improves, the bottom line still is in red ink! Perhaps the most important factor bearing on the president’s support is lack of confidence in the accuracy of the cost and, hence, the gross margin figures. Based on her own informal analysis, the president asked, “Why does an initiative that is solely focused on the cell-phone casing product line have little impact on this product’s profitability while improving the profitability of the pen-casing line? Further, if overall company profitability increases, shouldn’t there be improved profitability in cell phone casings?”

..

158


Exhibit 4-47: Panel A Traditional Cost System Process Map

All Indirect Resources $220,000

All Unallocated Value Chain Costs $75,000

Cost Driver [Direct Labor Hours = 4,500 + 1,000 = 5,500]

Direct Materials for Pen Casings $22,500

Direct Labor for Pen Casings $135,000

Pen Casings Sales $360,000

..

Direct Materials for Cell Phone Casings $21,600

Direct Labor for Cell Phone Casings $15,000

Cell Phone Casings Sales $120,000

Unallocated $75,000

159


Exhibit 4-47: Panel B PRO-FORMA FINANCIAL REPORTS FOR LOPEZ PLASTICS COMPANY: TRADITIONAL COST ALLOCATION SYSTEM STATEMENT OF OPERTING INCOME

CONTRIBUTION TO CORPORATE COSTS AND PROFIT [INTERNAL STRATEGIC DECISION MAKING AND OPERATIONAL-CONTROL PURPOSE]

[EXTERNAL REPORTING PURPOSE]

Pen Casings

Cell Phone Casings $120,000 1

Sales Cost of goods sold:

$480,000

$360,000

Direct material Direct labor

44,100 150,000

22,500 135,000

Indirect Production Cost of goods sold Gross profit Corporate expenses (unallocated) Operating loss

220,000 414,100 65,900 75,000 ($ 9,100) 13.73% 5

180,000 3 337,500 $ 22,500

76,600 $ 43,400

6.25%

36.17%

Gross profit margin

21,600 2 15,000 40,000 4

1. $80,000 × .75 × 2 2. $12,000 × 2 × .90 3. $220,000 × [4,500/(4,500 + 1,000)] 4. $220,000 × [1,000/(4,500 + 1,000)] 5. $65,900/$480,000

..

160


Finally, the focus of improvement efforts should be directly on the pen-casing product line. This initiative deals mostly with the cell-phone line. What can be done to improve profitability of the pen casings? Can prices be raised without losing too much volume? Can operational improvements be made to lower the indirect manufacturing costs? The controller’s idea is worthy of some support but it does not address the profitability issue head on. 2. Panel C of Exhibit 4-47 that follows is a process map that can be used to explain the impact of the controller’s idea. Panel D provides a detailed evaluation of the controller’s idea. Pen Cell-Phone Casings Casings Company Base Gross Profit Percentage* 16.22% (28.63%) 8.07% Plan Gross Profit Percentage** 16.11% 6.58% 13.73% Support of Product Manager? Neutral Strong Support of President? Strong * Gross Profit Percentages from the table in the Summary Problem for Review on p. 141 of text. ** See panel D of Exhibit 4-47 on the next page. The controller’s idea will result in a slight decrease in the gross profit of the pen-casings line but a dramatic turnaround in the profitability of the cell-phone line. The product manager of pen casings would probably be neutral or even negative about the idea because the idea does not focus on operational improvements that directly affect the pen-casings line. The cell-phone casings manager would give strong support to the idea – this may save his/her job! The president might note that the numbers agree with her informal analysis – generating confidence in the integrity of the cost accounting system. For reasons previously stated, the president would strongly support this idea while encouraging all managers involved to keep up the good work, reminding them that even if all these predictions are realized, the company still is operating at a loss.

..

161


Exhibit 4-47: Panel C Process Map of ABC System

Plant & Machinery $180,000

20% 75% Processing Activity $143,000 [Direct Labor Hours = 4,500 + 1,000] Direct Materials for Pen Casings $22,500

Direct Labor for Pen Casings $135,000

SALES $360,000

..

All Unallocated Value Chain Costs $75,000

Engineers & CAD Equipment $40,000 80% 25% Production Support Activity $77,000

Cost Driver [Distinct Parts = 5 + 9] Direct Materials Direct Labor for Cell Phone for Cell Phone Casings $21,600 Casings $15,000

SALES $120,000

UNALLOCATED $75,000

162


Solution 4-47: Panel D PRO-FORMA FINANCIAL REPORTS FOR LOPEZ PLASTICS COMPANY: ACTIVITY-BASED COST ALLOCATION SYSTEM STATEMENT OF OPERTING INCOME

CONTRIBUTION TO CORPORATE COSTS AND PROFIT [INTERNAL STRATEGIC DECISION MAKING AND OPERATIONAL-CONTROL PURPOSE]

[EXTERNAL REPORTING PURPOSE]

Sales Cost of goods sold: Direct material Direct labor

$480,000

Pen Casings $360,000

Cell Phone Casings $120,000

44,100 150,000

22,500 135,000

21,600 15,000

Processing activity

143,000

Production support activity Cost of goods sold Gross profit Corporate expenses (unallocated) Operating loss Gross profit margin

77,000 414,100 65,900 75,000 ($ 9,100) 13.73%

117,000 1 27,500 3

26,000 2 49,500 4

1. 2. 3. 4.

..

302,000 $ 58,000

112,100 $ 7,900

16.11%

6.58%

$143,000 × [4,500 labor hours/(4,500 labor hours + 1,000 labor hours)] $143,000 × [1,000 labor hours/(4,500 labor hours + 1,000 labor hours)] $77,000 × [5 distinct parts/(5 distinct parts + 9 distinct parts)] $77,000 × [9 distinct parts/(5 distinct parts + 9 distinct parts)]

163


3. As vice president, you probably are pleased with the new ABC system. The cost drivers that are used to allocate activity costs appear to be plausible and reliable and thus probably represent a sound cause-effect model of operations. This will improve both the accuracy of product costing and operating managers’ control over costs. Operating managers will be pleased with the ABC system because it helps them understand how their day-to-day work impacts costs and profits. From a behavioral perspective, this should be highly motivational. This problem emphasizes the importance of the cost-accounting system to managers. Different systems can result in significantly different management decisions. In this case, the product-line managers’ support for the controller’s idea changes when an ABC system is used to evaluate the idea. Although the company-level gross margins do not change, it is possible that the president would strongly support the idea based on ABC data. Why? Neither of the product-line managers is against the idea, and one strongly supports it. In addition, the president may have more confidence in the accuracy of the ABC analysis. While the substantial losses of the current quarter will not be completely eliminated, the serious profitability problem of the cell-phone casing product line has been reversed.

..

164


4-48

(30-35 min.)

1. The Produce department has the highest operating income per euro of sales: Packaged Produce Food Meat Total Gross margin €177,700 €500,000 €282,400 €960,100 Allocation of support costs* 80,000 400,000 320,000 800,000 Operating income € 97,700 €100,000 € (37,600) €160,100 Op. Inc. ÷ Sales 19.63% 4.76% (2.41)% 3.85% *(€320,000 ÷ 3,200,000) × €800,000 = €80,000; etc. 2. Activity-based rates per unit of cost driver for each activity are: Ordering Delivery Shelf-stocking Customer support Produce monitoring

€67,870 ÷ 6,170 = €11 per order €179,200 ÷ 11,200 = €16 per delivery €128,858 ÷ 3,391 = €38 per hour €245,440 ÷ 613,600 = €0.40 per item sold Direct trace

Operating income for each department is:

Gross margin Support costs: Ordering Delivery Shelf-stocking Customer support Produce monitoring Operating income Op. Inc. ÷ Sales

Produce €177,700

Packaged Food €500,000

Meat €282,400

Total €960,100

15,400 19,840 8,284 20,040 178,632 € (64,496) (12.96)%

36,300 140,320 80,560 176,320 0 €66,500 3.17%

16,170 19,040 40,014 49,080 0 € 158,096 10.12%

67,870 179,200 128,858 245,440 178,632 €160,100 3.85%

This activity-based analysis shows that Produce has the lowest (rather than the highest) operating income as a percent of sales. The Meat Department has the highest ratio.

..

165


3. The activity-based costing system is generally more accurate because it better allocates costs on a cause/effect basis. The Produce Department uses all of the produce monitoring activity. The activity-based costing system recognizes this and allocates these costs to the department. However, even knowing an accurate operating income for each product is not sufficient for making a good decision. Operating income as a percent of sales is not a good criterion for making a decision about the allocation of scarce capacity. Several other pieces of information would help make a better decision: a) Capacity utilization by each department. For expansion decisions, management should expand first the department that gives the highest contribution margin per square foot (or per linear shelf foot or other capacity measure). b) Fixed and variable costs. The activity-based costing system allocates all costs on a per unit basis. Fixed costs are not separated from variable costs, so a contribution margin cannot be determined. This may be ok for long-term strategic decisions (as this one probably is). However, it will not provide a short-term prediction of the effect of the expansion. c) Market demand – What managers really want to know is how much sales will increase with an increase in space allocated to a department. The activity-based costs can be used to estimate the impact of different sales levels, but we need to know how much sales will increase as a result of the expansion of capacity in each department. d) Product interactions. Will increased (or decreased) space for one department affect the sales of another department? e) Opportunities for re-engineering. The activity-based costing numbers provide costs for providing each type of support activity. It will help determine whether these activities are worth their cost or if any of them might be accomplished more efficiently. The high costs of ordering and delivery for the Packaged Food Department or the high cost of produce monitoring for the Produce Department might be reduced by re-engineering the processes, thereby increasing the profitability of the department.

..

166


4-49

(30-35 min.)

1.

Activity: Cost-Allocation Base Quality: Pieces scrapped Production scheduling: Setups Setup: Setups Shipping: Containers shipped Shipping admin.: Shipments Production: Machine hours Total indirect cost Direct materials Direct labor Total Cost of Part 24Z2 Cost per unit ($25,282  2,800) Selling price Gross margin Gross margin percentage 2.

Annual Cost $ 880,000 72,000 880,000 384,000 105,000 1,800,000 $4,121,000

Annual Cost Driver Activity 16,000 800 800 64,000 1,500 12,000

Cost Cost Driver Total Per Consumption Cost Driver Part Part Unit 24Z2 24Z2 $

55 90 1,100 6 70 150

150 5 5 12 7 16

$ 8,250 450 5,500 72 490 2,400 $17,162 6,160 1,960 $25,282 $ 9.03 8.20 $ (0.83) (10.1%)

Assuming that the results of the activity analysis are accurate, product 24Z2 is much more costly than O’Hanlon’s existing costing system estimates. The existing system is under-costing product 24Z2 by $9.03 - $6.40 = $2.63 per unit or $2.63 ÷ $9.03 = 29%! Chrysler’s proposal should be rejected unless Chrysler is willing to increase the price or unless O’Hanlon can significantly reduce its costs. O’Hanlon should be aware, however, that the existing costing system is also over-costing other products since all indirect costs are allocated. The activity-based costing system should be used to cost all product lines in order to identify O’Hanlon’s “winners” as well as “losers.” Benefits of activity-based implementation include:  More accurate costing of activities, products, customers, and other cost objects  A solid foundation for activity-based management -- using ABC information as a management tool for budgeting, planning, and control purposes  An effective communication tool since successful ABC implementation should involve all functional areas of the company

..

167


Costs of implementing activity-based costing include:  The cost of a pilot study includes salaries of managers who are dedicated to the study.  Consultants are often necessary.  Data collection is extensive since operational and financial data are often not available as required to support the new ABC system.  It may be necessary to maintain an ABC system separate from the accounting system used for external reporting. 4-50

(60 min. or more)

The purpose of this exercise is to force students to look beyond the textbook. The library has many examples of applications of management accounting that show that topics presented in textbooks are of interest in the "real world". The expectation in this exercise should not be for a detailed understanding of how a company applied activity-based costing. At this point in the course students should get a general impression of how textbook topics are being applied. Look for some expression of understanding the article, making sure that students are expressing the information in their own words. A brief, intuitive explanation is much better than a detailed description taken nearly verbatim from the article.

..

168


4-51

(20-30 min.)

For (a), (c), and (e), calculation for variable selling and administration overhead, fixed manufacturing overhead, and gross profit (figures in thousand ₩): Sales ₩300 Cost of goods manufactured and sold (i.e., manufacturing cost of goods sold): Direct material ₩87.5 Direct labor 62.5 Variable manufacturing overhead 12.5 Fixed manufacturing overhead 37.5** Total manufacturing cost of goods sold 200 Gross profit 100* Selling and administrative expenses: Variable 32.5 Fixed 30 62.5*** Net profit ₩ 37.5 *Answer (e) is simply ₩300 – ₩200 = ₩100. **Answer (c) is ₩200 – ₩(87.5 + 62.5 + 12.5) = ₩37.5. ***Total selling and administrative expenses = ₩100 – ₩37.5 = ₩62.5 Then answer (a) is ₩62.5 – ₩30 = ₩32.5 b. Calculation for contribution margin (figures in thousand South Korean Won) Sales ₩300 Direct materials (87.5) Direct labor (62.5) Variable manufacturing overhead (12.5) Variable selling and administrative expenses (32.5) Contribution margin ₩105 d. Calculation for break-even point in sales ₩ Contribution margin % = ₩105 ÷ ₩300 = 35% Break-even point in sales= (Fixed mfg. overhead + Fixed sell. and adm.) ÷ Contribution margin % = ₩(37.5 + 30) ÷ .35 = ₩192.857

..

169


4-52 (30-40 min.) (a) Calculation for fixed manufacturing overhead (in thousand pounds): Sales £300 Cost of goods sold: Direct material £87 Direct labor 90 Variable manufacturing overhead 15 Fixed manufacturing overhead 48b Total cost of goods sold £240a Gross profit (20% of £300) £60 a £300 – £60 = £240. b £240 – £(87 + 90 + 15) = £48. (b) Calculation for variable selling and administration overhead (in thousand pound) Sales £300 Variable costs: Direct material £87 Direct labor 90 Variable manufacturing overhead 15 Variable selling and administrative overhead 33b Total variable costs 225a Contribution margin £75 a

£300 – £75 = £225 £225 – £(87 + 90 + 15) = £33. Note that this can be computed without having to know the gross profit, the break-even point, or the fixed manufacturing overhead computed in part (a). (c) Calculation for fixed selling and administration overhead: Note that the contribution margin percentage = £75,000 ÷ £300,000 = 25%. b

Break-even point

= Total fixed costs ÷ Contribution margin percentage

. . . Total fixed costs = Break-even point × Contribution margin percentage = £252,000 × .25 = £63,000 Now, Total fixed costs = Fixed mfg. overhead + Fixed selling and admin overhead £63,000 = £48,000 + X X = £15,000 Therefore, the answer is £15,000. An alternate approach to part (c) is: Let F = Total fixed expenses ..

170


Break-even point sales= Variable expenses + Total fixed expenses £252,000 = (.75 × £252,000) + F F = £252,000 – £189,000 = £63,000 Then the fixed selling and admin overhead = Total fixed expenses – Fixed manufacturing overhead: Fixed selling and admin expenses = £63,000 – £48,000 = £15,000

..

171


4-53 (30-35 min.) 1. and 2. Calculation for variable and fixed selling and administrative overhead (in SGD) Sales SGD 390,000 Variable expenses: Direct materials used SGD 105,000 Direct labor 33,000 Variable manufacturing overhead 54,000 Variable selling and administrative overhead 33,000b Total variable expenses 225,000a Contribution margin 165,000 Fixed expenses: Fixed manufacturing overhead 39,000 Fixed selling and administrative overhead 27,000d Total fixed expenses 66,000c Operating income SGD 99,000 a

Total variable expense = SGD 390,000 – SGD 165,000 = SGD 225,000 Variable selling and administrative overhead = SGD 225,000 – SGD (105,000 + 33,000 + 54,000) = 33,000 c Total fixed expenses are = SGD 165,000 – SGD 99,000 = SGD 66,000 d Fixed selling and administrative expenses = SGD 66,000 – SGD 39,000 = SGD 27,000 b

3. Calculation of cost of goods sold Cost of goods sold is the total production (or manufacturing) cost of the 3,000 handbags: SGD (105,000 + 33,000 + 54,000 + 39,000) = SGD 231,000 4.

..

a.

Contribution Margin Ratio: (SGD 390,000 – SGD 225,000) / SGD 390,000 = 42.3077% Break-even point in sales: SGD 66,000/42.3077% = SGD 156,000 b. Let U = Target sales units (SGD 55 × U) – SGD 57,750 = SGD 52,800 U = SGD 110,550 ÷ SGD 55 = 2,010 units c. Let P = Target selling price Variable costs per unit = SGD 225,000 ÷ 3,000 = SGD 75 Contribution margin per unit = P – SGD 75 2730 × (P – SGD 75) – SGD 57,750 = SGD 187,950 2730 × (P – SGD 75) = SGD 245,700 P – SGD 75 = SGD 90 P = SGD 165

172


4-54 1.

(30-45 min.) Cost pool* 0. Direct-material cost 1. Direct-labor hours 2. Machine hours 3. Pounds of materials 4. Number of production setups 5. Number of production orders 6. Number of orders shipped Total budgeted cost Number of units Unit cost

Alpha** £ 700,000 560,000 560,000 90,000 80,000 13,500 60,000 £2,063,500 ÷ 10,000 £ 206.35

Beta £ 70,400 201,600 96,000 9,600 40,000 9,000 48,000 £474,600 ÷ 800 £ 593.25

Gamma £ 225,000 630,000 280,000 30,000 40,000 3,150 120,000 £1,328,150 ÷ 5,000 £ 265.63

*Identified by the cost driver used **Calculations for Alpha (Beta and Gamma are similar): 0. 10,000 × £70 = £700,000 1. [(4 × 10,000) ÷ (4 × 10,000 + 18 × 800 + 9 × 5,000)] × £1,391,600 = £560,000 2. [(7 × 10,000) ÷ (7 × 10,000 + 15 × 800 + 7 × 5,000)] × £936,000=£560,000 3. [(3 × 10,000) ÷ (3 × 10,000 + 4 × 800 + 2 × 5,000)] × £129,600 = £90,000 4. [100 ÷ (100 + 50 + 50)] × £160,000 = £80,000 5. [300 ÷ (300 + 200 + 70)] × £25,650 = £13,500 6. [1,000 ÷ (1,000 + 800 + 2,000)] × £228,000 = £60,000 2.

Total cost (except direct materials) per direct-labor hour: £2,870,850 ÷ (4 × 10,000 + 18 × 800 + 9 × 5,000) = £2,870,850 ÷ 99,400 = £28.88 per direct-labor hour Direct material cost per unit Other manufacturing cost per unit* Total manufacturing cost per unit Number of units Total budgeted cost**

Alpha £ 70.00 115.52 £185.52 × 10,000 £1,855,200

Beta £ 88.00 519.84 £607.84 × 800 £486,272

Gamma £ 45.00 259.92 £304.92 × 5,000 £1,524,600

* £28.88 × 4; £28.88 × 18; £28.88 × 9 ** Total = £1,855,200 + £486,272 + £1,524,600 = £3,866,072, which differs from £3,866,250 due to rounding error in the £28.88 rate. 3.

The new system is more complex and more costly. The added expense would be justified if the added value of better decisions made using the new system exceeds the added cost of the system.

..

173


4-55 (50-60 min.) This is a difficult problem that forces students to think carefully about the principles in the chapter. It requires an extension of those principles beyond the examples in the chapter. 1. The existing traditional system simply divides all costs by the number of cheesecakes produced, regardless of the market for which the cheesecakes are produced: Ingredients ($900,000 ÷ 500,000) Production overhead ($2,216,000 ÷ 500,000) Total

$1.800 4.432 $6.232

This system doesn’t recognize that retail and restaurant cheesecakes consume different proportions of many of the support activities represented by the production overhead costs. Therefore, it over-costs cheesecakes that use relatively less of the resources and under-costs those that use relatively more. 2. The ABC system allocates the costs of each of the four activities separately, and it directly traces the cost of packaging materials as well as the ingredients. Administration and Facilities Operations and Maintenance costs are first allocated to Mixing and Baking and Decorating and Packaging as follows because they support the activities in those departments: Mixing and Decorating and Baking Packaging Direct costs $540,000 $840,000 Administration* 100,000 40,000 Facilities Op. & Maint.** 256,000 64,000 Total $896,000 $944,000 ÷ Processing hours 6,000 ÷ Decorating/Packaging hours 42,000 Allocation rate to production $149.3333 $22.4762 *$140,000 ÷ 7 = $20,000; $20,000 × 5; $20,000 × 2 **$320,000 ÷ 5,000 = $64; $64 × 4,000; $64 × 1,000 The allocation to retail and restaurant customers follows: Ingredients Packaging materials Mixing/baking* Decorating/packaging** Total Per Cheesecake

Retail $ 720,000 360,000 746,667 899,048 $2,725,715 $6.814

Restaurants $180,000 16,000 149,333 44,953 $390,286 $3.903

Total $ 900,000 376,000 896,000 944,000*** $3,116,000 $6.232

*Processing hours: 400,000 ÷ 80 = 5,000; 100,000 ÷ 100 = 1,000 $896,000 ÷ 6,000 = $149.3333; $149.3333 × 5,000; $149.3333 × 1,000 **Decorating/packaging hours: 400,000 ÷ 10 = 40,000; 100,000 ÷ 50 = 2,000 $944,000 ÷ 42,000 = $22.4762; $22.4762 × 40,000; $22.4762 × 2,000 *** $1 rounding error ..

174


3. In the traditional system, the restaurant cheesecakes received 20% of all production overhead (100,000 ÷ 500,000 = 20%). In the ABC system, the restaurant cheesecakes received much less of the cost because they used much fewer resources. In total they are 37% less costly than previously thought. Thus, they can be more competitively priced and still make a reasonable profit. In addition, the retail cheesecakes are 9% more costly than thought previously. In addition, the ABC costs give better targets for cost control. The costs of the various activities can be compared to benchmarks to see if they can be accomplished less expensively. 4-56

(50-60 min.) Numbers except per unit amounts are in thousands.

This case focuses on activity-based costing in a service organization. It contains both straightforward numerical requirements and more conceptual, discussion-oriented issues. It contrasts a traditional cost accounting system with an ABC system. 1. a) The indirect cost allocation rate is $2,850 / 11,400 = $.25 per check processed. b) # of checks processed Cost / check processed Total cost

Retail Line 2,280 × $.25 $ 570

Business Line 9,120 × $.25 $2,280

Total 11,400 × $.25 $2,850

c) Retail line = $570/$2,850 = 20% Business line = $2,280/$2,850 = 80% d) Retail = $570/150 = $3.80 Business = $2,280/50 = $45.60 e) Revenue per account Cost per account Profit (loss) per account

Retail $10.00 3.80 $ 6.20

Business $40.00 45.60 $(5.60)

f) The original cost system suggests that retail customers are profitable, but business customers are not. This suggests emphasizing retail customers. The bank would also try to make business customers more profitable, perhaps by increasing fees for services, reducing interest paid on business accounts, or requiring businesses to hold higher account balances.

..

175


2. Broken machines simply stop running. In contrast, “broken” or outdated cost systems continue producing potentially misleading costs. Consequently, managers need to recognize clues that the cost system needs refinement. Signs that Columbia City’s original cost system is broken include: ◦ Profits are declining even though the bank is serving more customers ◦ The CEO does not understand the results ◦ The customer mix is changing to more small retail customers and fewer large business customers, and these two different types of customers are likely to make different use of the bank’s resources ◦ The cost system is an old (1988) single-allocation-base system ◦ The cost system has not changed since the new customer service call center was added ◦ The manager (Rose Perez) does not trust the accounting system’s numbers 3. Columbia City has three activities, each with the following expected costs: Check payments: $440 + $700 = $1,140 Teller withdrawals and deposits: $1,200 Customer service call center: $450 + $60 = $510 The cost drivers and the estimated total quantity of each cost driver are: Activity Check payments Teller withdrawals and deposits Customer service call center

Cost Driver Checks processed Teller transactions Customer calls to center

Units of Cost Driver 11,400 400 100

a. The indirect cost allocation rate for each activity is: Check payments: ($440 + $700)/11,400 = $.10 per check processed Teller withdrawals and deposits: $1,200/400 = $3 per teller transaction Customer call center: ($450 + $60)/100 = $5.10 per customer call b. The quantity of cost driver activity in each customer line is (in thousands): Retail Line Business Line Total Checks processed 2,280 9,120 11,400 Teller transactions 320 80 400 Customer calls to call center 95 5 100

Columbia City must allocate costs from each cost pool to each customer line: ..

176


Activity Check payments: $.10 × (2,280; 9,120) Teller withdrawals and deposits: $3 × (320; 80) Customer call center: $5.10 × (95; 5) Total indirect costs

Total Indirect Cost Assigned To Retail Customer Line

Total Indirect Cost Assigned To Business Customer Line

$ 228.00

$ 912.00

960.00

240.00

484.50 $1,672.50

25.50 $1,177.50

You might note that this solution assumes that all calls to the call center are equally costly, as are all teller transactions. That is, business customers do not tend to take more or less time per call or per teller transaction than do retail customers. c. The proportions of each activity’s resources used by each customer line are: Retail Business Total Check payments 2,280 (20%) 9,120 (80%) 11,400 (100%) Teller withdrawals and deposits 320 (80%) 80 (20%) 400 (100%) Customer call center 95 (95%) 5 (5%) 100 (100%) d. The ABC indirect costs per retail and business customer are: Retail: $1,672.50/150 = $11.15 Business: $1,177.5/50 = $23.55 e. The original cost system allocates all indirect costs based on the number of checks processed. Because retail customers write 20% of the checks, the original system allocated 20% of the indirect costs to retail customers and 80% of the indirect costs were allocated to business customers. The ABC analysis shows that retail customers used much more than 20% of the other two activities: teller withdrawals and deposits and customer service call center. Retail customers made 80% of the teller transactions and 95% of the calls to the customer service center. Thus, the new ABC system allocates to retail customers 80% of the teller transaction costs and 95% of the customer service call center costs. Thus, the end result is that ABC allocates 59% (rather than 20%) of the indirect costs to retail customers and 41% (rather than 80%) of the indirect costs to business customers. f. Using the new ABC data, the average profit per account for retail and business customers is: Retail Business Revenue per account $10.00 $40.00 ABC cost per account 11.15 23.55 ..

177


ABC profit (loss) per account Original profit (loss) per account

$(1.15) $6.20

$16.45 $(5.60)

The ABC system suggests that business customers are profitable, but retail customers are not – exactly opposite the conclusion based on the original cost data. This example illustrates how ABC can significantly affect management’s strategy. The ABC data suggest that managers should emphasize business customers. The incentive system should provide more reward (e.g. larger bonuses) for adding business customers than for adding retail customers. In addition, the bank should try to make retail customers more profitable, perhaps by increasing fees for services, reducing interest paid on retail accounts, or requiring retail customers to maintain higher account balances. 4. a) The ABC data suggest that the plan to simply increase the number of checking account customers was not wise. Assuming it is easier to attract a given number of retail customers than business customers, it is not surprising that the incentive plan resulted in an influx of retail customers. Unfortunately, the ABC data suggest that the retail customers are not profitable. Given the existing revenue and cost structure, Columbia City Bank may want to provide a bonus based on attracting and retaining new business customers only. The bank should not encourage more retail customers until it changes the revenue structure (for example, by increasing the required minimum balance for retail checking accounts) or the cost structure (for example, by cutting back service at the customer service call center). b) Two main benefits of ABC are: 1) More accurate cost information a. Can help Columbia City better price its services. b. Can help Columbia City assess the profitability of different customers and/or different services. Such analyses can affect services offered and marketing strategy. 2) Cost control a. Reduce consumption of costly cost drivers. For example, Columbia City may want to encourage on-line banking to reduce the number of teller transactions. b. Reduce the indirect cost per unit of the cost driver. For example, train the customer service representatives to handle more calls per hour. This can reduce the cost per call if the bank can handle the call load with fewer customer service representatives.

..

178


c) The relative advantage of ABC, that is, when its benefits exceed its cost, is greatest: 1) When a company sells in a highly competitive industry, accurate cost information is essential for setting prices that are competitive yet still allow the company to earn a profit. Competitors will capitalize on a company’s mispricing. In addition, ABC can pinpoint opportunities for cost savings, which increase the company’ profit or are passed on to customers in lower sales prices. 2) When a company has high indirect costs. If indirect costs are low, it doesn’t matter much how they are allocated. 3) When different customers/products/services use different amounts of the company’s various resources. 4) When the company has sufficient information technology and accounting expertise to implement the system and to record cost driver data. Columbia City Bank is in a highly competitive environment. Most of its costs are indirect. The retail and business customer lines use different amounts of the 3 resources, check processing, teller transactions, and customer service center calls. Given the magnitude of the data processing requirements, banks typically have advanced information technology and accounting expertise. All these factors suggest that the benefits of ABC are likely to outweigh the costs. d) Non-accounting managers need to understand ABC because: 1) Non-accounting managers often serve on ABC teams. As in the case, ABC teams typically include managers familiar with operations – in addition to the accountants. ABC is not just an accounting exercise, and even managers who are not in the financial function may be involved in developing or updating an ABC system. 2) Managers need to understand ABC data to use it in decisions such as pricing, product and customer profitability analysis, cost control, etc.

..

179


4-57 (30 min.) 1. and 2. The following are the most likely activities and resources that student will identify. There may be disagreements on the cost behavior – those listed below are probably most likely, but different behavior can be supported under different assumptions. Likewise, students might suggest other, equally reasonable, cost drivers, depending on their assumptions. Activities Setups Molding process Resources Maintenance mechanics hours Supervisors Molding machine operators Machine supplies Energy Building Molding machines

..

Cost Behavior

Cost Drivers No. of setups Machine hours

Fixed

Mechanic

Fixed Fixed Variable Variable

No. of people Operator hours Machine hours Kilowatt hours Fixed Square footage Machine hours

Fixed

180


4-58

(15-20 min.)

Nike’s balance sheet shows only one line for Inventories, with a 2011 balance of $2,715 million. Note 2 states that the inventories “were substantially all finished goods.” If Nike had significant manufacturing operations, it would also have raw materials and work-inprocess inventories. Thus, it must be primarily a merchandiser. This is confirmed by the first sentence under the Manufacturing heading: “Virtually all of our footwear is produced by factories we contract with outside of the United States.” 4-59 (50-60 min.) For the solution to this Excel Application Exercise, follow the stepby-step instructions provided in the textbook chapter. Answers for #1 and #2 come directly from the spreadsheet: 1. PCB124 = $81.43; PCB136 = $228.02 2. PCB124 = $97.80; PCB136 = $203.46 3. The cost for PCG124 is greater under the ABC system by ($97.80 - $81.43) ÷ $81.43 = 20.1%, while the cost of PCB136 is less by ($228.02 - $203.46) ÷ $228.02 = 10.7%. These changes are significant enough to affect Sunstar’s assessment of product profitability and decisions relating to product mix. 4-60 (60 min. or more) Answers will vary based on the industry and particular company chosen.

..

181


4-61 (25 – 30 min.) NOTE TO INSTRUCTOR: This solution is based on the website as it was in late 2020. Be sure to examine the current website before assigning this problem, as the information there may have changed. 1. #LoveIsInTheBEAR is a unique program to establish a direct connect with the customers. It asks every customer or recipient of a Vermont Teddy Bear to share their story when they unbox the Bear gift. It requires the recipient to upload their photo on the website or share the same on Twitter or Instagram with #LoveIsInTheBEAR and @VTTeddyBear. The customers may also win a $500 GIFT CARD for the same. 2. A Bear-Gram is a unique idea that is a creative alternative to flowers. The Bear-Gram is a hand-crafted teddy bear that is guaranteed to last for life. The classic teddy bears are made in Vermont, and no two are alike. Customers can choose from different colors and types of fur. Every Bear-Gram gift includes the handcrafted, fully jointed Vermont Teddy Bear and a fun gift box with an air hole to ensure that the teddy bear gets lots of air on its journey. Customers also have the option to make their gift even more special with personalized bear paws, by adding a name, date, or any message. 3. Every Vermont Teddy Bear is handmade in the United States only. The process includes cutting, sewing, stuffing, and stitching the bears, and then dressing them up and shipping them out. Vermont’s skillful employees take utmost care in this process so that every bear made can meet the highest standard of quality. The company sources most of the fur from Wisconsin but for some special edition bears they even import fine quality fur from China and Germany. Strict inspection is done to ensure that the raw materials are of the best quality. Hybrid bears are, however, built initially in China and then completed in Vermont. This helps the company reduce the cost significantly and as a result the product is sold at a cheaper price. Vermont also designs and prepares the outfits for its bears in its own factory with the best quality raw materials. Also, every single outfit worn by the custom-made bears is handmade and exquisitely tailored right in Shelburne, at the Vermont factory. The activities involved in the entire process are: a. Receiving raw materials after due inspection b. Cutting the fur c. Sewing d. Stuffing e. Stitching f. Designing the outfits g. Manufacturing the outfits h. Dressing up the bears i. Shipping Two direct costs are: a. Direct materials (fur for the bears, cloth, paw pads, eye balls) b. Direct labor Two indirect costs are: ..

182


a. b.

Lubricants for sewing machines Factory lighting

Both direct material and direct labor are variable costs. The cost of lubricant is also variable, while factory lighting is mixed. (Meter rental is fixed and charges for units consumed are variable.) 4. Yes, activity-based costing may be recommended for Vermont. Any company that has complexity in one or more functions of its value chain is a good candidate for ABC. In addition, any company can benefit from the increased understanding of its business that results from implementing an ABC system. As the Vermont Teddy Bear factory grows, it will undoubtedly realize increased complexity across its value chain. Having a good cost management system in place, including an ABC system, will be of significant value. Some of the activity cost-pools with relevant cost drivers are: a. Material receiving—quantity of materials b. Material handling—no. of material moves c. Quality testing and inspection—inspection hours d. Packaging and delivery—no. of orders delivered 5. As the company is a manufacturer, it takes raw materials, applies labor and overhead, and turns the materials into finished goods—teddy bears. Thus, its inventory account would include three categories: raw materials, work in process, and finished goods. From Vermont Teddy Bear’s production process, it seems that the largest inventory account might be its raw materials. Its production process is quite short, so there is likely to be only a small work-in-process inventory balance. Since the company builds custom teddy bears only after receiving an order, they probably ship them immediately upon completion, making for a small finished-goods inventory balance.

..

183


CHAPTER 5 COVERAGE OF LEARNING OBJECTIVES

LEARNING OBJECTIVE LO1: Discriminate between relevant and irrelevant information for making decisions. LO2: Apply the decision process to make business decisions. LO3: Construct absorption and contribution-margin income statements and identify their relevance for decision making. LO4: Decide to accept or reject a special order using the contribution margin approach. LO5: Explain why pricing decisions depend on the characteristics of the market. LO6: Identify the factors that influence pricing decisions in practice. LO7: Compute a target sales price by various methods, and compare the advantages and disadvantages of these methods. LO8: Use target costing to decide whether to add a new product.

..

FUNDAMENTAL ASSIGNMENT MATERIAL B3

B3

CASES, EXCEL, ADDITIONAL COLLAB. & ASSIGNMENT INTERNET MATERIAL EXERCISES 23, 30, 30, 37, 38, 66 38, 49, 49, 50, 50, 51, 51, 54, 55, 57, 57 28, 28, 29, 39, 51

A1, B1, B3

8, 24, 31, 31, 32, 32, 33, 33, 34, 34, 35, 35, 48, 49, 50, 57

A2, B2, B3

36, 40, 40, 49, 50, 51, 55, 55, 56, 57

A2, B2, B3

25, 42, 58, 58

B3

15, 26, 41, 41, 47, 52, 52, 53

A3, B3

43, 43, 44, 50

A4, B3

27, 45, 45, 46, 46, 59, 59, 60, 61, 61

62, 63,64

65

184


CHAPTER 5 Relevant Information for Decision Making with a Focus on Pricing Decisions 5-A1 1.

(40-50 min.) LIBERTY COMPANY Contribution Income Statement For the Year Ended December 31, 2012 (in thousands of dollars)

Sales Less variable expenses Direct material Direct labor Variable manufacturing overhead (Schedule 1) Total variable manufacturing cost of goods sold Variable selling expenses Variable administrative expenses Total variable expenses Contribution margin Less fixed expenses: Fixed manufacturing overhead (Schedule 2) Selling expenses Administrative expenses Total fixed expenses Operating income

..

$2,500 $410 330 160 $900 76 21 997 $ 1,503 $360 220 128 708 $ 795

185


LIBERTY COMPANY Absorption Income Statement For the Year Ended December 31, 2012 (in thousands of dollars) Sales Less manufacturing cost of goods sold: Direct material Direct labor Manufacturing overhead (Schedules 1 and 2) Total manufacturing cost of goods sold Gross margin Less: Selling expenses Administrative expenses Operating income

$2,500 $410 330 520 1,260 $ 1,240 $296 149

445 $ 795

LIBERTY COMPANY Schedules of Manufacturing Overhead For the Year Ended December 31, 2012 (in thousands of dollars) Schedule 1: Variable Costs Supplies Utilities, variable portion Indirect labor, variable portion Schedule 2: Fixed Costs Utilities, fixed portion Indirect labor, fixed portion Depreciation Property taxes Supervisory salaries Total manufacturing overhead 2.

Change in revenue Change in total contribution margin: Contribution margin ratio in part 1 is $1,503 ÷ $2,500 = .601 Ratio times decrease in revenue is .6012 × $200,000 Operating income before change New operating income

$ 25 42 93 $ 17 51 215 18 59

$160

360 $520 $200,000

$ 120,240 795,000 $674,760

This analysis is readily done by using data from the contribution income statement. In contrast, the data in the absorption income statement must be analyzed and split into variable and fixed categories before the effect on operating income can be estimated. ..

186


5-A2

(25-30 min.)

1.

A contribution format, which is similar to Exhibit 5-6, clarifies the analysis. Without Special Order 3,000,000

Units Sales Less variable expenses: Manufacturing Selling & administrative Total variable expenses Contribution margin Less fixed expenses: Manufacturing Selling & administrative Total fixed expenses Operating income

$15,900,000 $ 5,850,000 1,050,000 $ 6,900,000 $ 9,000,000

$322,000 26,740 $348,740 $261,660

$2.30 2 $ 6,172,000 .191 3 1,076,740 $2.491 $ 7,248,740 $1.869 $ 9,261,660

$ 3,600,000 3,300,000 $ 6,900,000 $ 2,100,000

0 0 0 $261,660

0.00 0.00 0.00 $1.869

1 $610,400 ÷ 140,000 = $4.36 2 Regular unit cost = $5,850,000 ÷3,000,000 = Logo Variable manufacturing costs 3 Regular unit cost = $1,050,000 ÷ 3,000,000 = Less sales commissions not paid (3% of $5.30) Regular unit cost, excluding sales commission 2.

With Special Order 3,140,000

Effect of Special Order 140,000 Total Per Unit $610,400 $4.36 1

$16,510,400

$ 3,600,000 3,300,000 $ 6,900,000 $ 2,361,660

$1.95 .35 $2.30 $ .35 (.159) $ .191

Operating income from selling 4.67% more units would increase by $261,660 ÷ $2,100,000 = 12.46%. Note also that the average selling price on regular business was $5.30. The full cost, including selling and administrative expenses, was $4.60. The $4.60 plus the 35¢ per logo, less savings in commissions of .159¢ came to $4.791. The president apparently wanted $4.791 + .08($4.791) = $4.791 + .3833 = $5.1743 per pen. Most students will probably criticize the president for being too stubborn. The cost to the company was the forgoing of $261,660 of income in order to protect the company's image and general market position. Whether $261,660 was a wise investment in the future is a judgment that managers are paid for rendering.

..

187


5-A3

(15-20 min.)

The purpose of this problem is to underscore the idea that any of a number of general formulas might be used that, properly employed, would achieve the same target selling prices. Desired sales = $12,000,000 + $1,200,000 = $13,200,000. The target markup percentage would be: 1.

164% of direct materials and direct labor costs of $5,000,000. Computation is: ($13,200,000 - $5,000,000) ÷ $5,000,000 = 164%

2.

76% of the full cost of jobs of $7,500,000. Computation is: ($13,200,000 - $7,500,000) ÷ $7,500,000 = 76%

3.

[$13,200,000 – ($3,000,000 + $2,000,000 + $1,000,000)] ÷ $6,000,000 = 120%

4.

($13,200,000 - $12,000,000) ÷ $12,000,000 = 10%

5.

[$13,200,000 – ($3,000,000 + $2,000,000 + $1,000,000 + $2,250,000)] ÷ $8,250,000 = $4,950,000 ÷ $8,250,000 = 60%

If the contractor is unable to maintain these profit percentages consistently, the desired operating income of $1,200,000 cannot be obtained.

..

188


5-A4

(15-20 minutes)

1.

Revenue ($380 × 65,000) Total cost over product life Estimated contribution to profit Desired (target) contribution to profit 50% × $24,700,000 Excess (deficiency) in profit

$24,700,000 13,320,000 $11,380,000 $

12,350,000 (970,000)

The product should not be released to production. 2.

Previous total estimated cost Cost savings from suppliers .15 × .40 × $5,000,000 Revised total estimated cost Revised total contribution to profit: $24,700,000 - $13,020,000 Desired (target) contribution to profit Excess (deficiency) in profit

$13,320,000 300,000 $13,020,000 $11,680,000 12,350,000 $ (670,000)

The product should not be released to production. 3.

Previous revised total estimated cost from requirement 2. $13,020,000 Process improvement savings: .30 × .60 × $5,000,000 $900,000 Less cost of new technology 220,000 680,000 Revised total estimated cost 12,340,000 Revised total contribution to profit: $24,700,000 - $12,340,000 $12,360,000 Desired (target) contribution to profit 12,350,000 Excess (deficiency) in profit $ 10,000

Now the product should be released to production.

..

189


5-B1 1.

(40-50 min.) ZEALAND MANUFACTURING Contribution Income Statement For the Year Ended December 31, 2012 (In thousands of dollars)

Sales Less variable expenses: Direct material Direct labor Variable indirect manufacturing costs (Schedule 1) Total variable manufacturing cost of goods sold Variable selling expenses: Sales commissions Shipping expenses Variable clerical salaries Total variable expenses Contribution margin Less fixed expenses: Manufacturing (Schedule 2) Selling (advertising) Administrative-executive salaries Total fixed expenses Operating income

$14,000 $3,500 1,700 1,102 $6,302 $470 320

790 370 7,462 $ 6,538 $ 723 430 100 $

1,253 5,285

ZEALAND MANUFACTURING Absorption Income Statement For the Year Ended December 31, 2012 (In thousands of dollars) Sales Less manufacturing cost of goods sold: Direct material Direct labor Indirect manufacturing costs (Schedules 1 and 2) Gross profit Selling expenses: Sales commissions Advertising Shipping expenses Administrative expenses: Executive salaries Clerical salaries Operating income ..

$14,000 $3,500 1,700 1,825 $470 430 320

$1,220

$100 370

470

7,025 6,975

1,690 $ 5,285

190


ZEALAND MANUFACTURING Schedules 1 and 2 Indirect Manufacturing Costs For the Year Ended December 31, 2012 (In thousands of dollars) Schedule 1: Variable Costs Cutting bits Abrasives for machining Indirect labor Schedule 2: Fixed Costs Factory supervisors' salaries Factory methods research Long-term rent, factory Fire insurance on equipment Property taxes on equipment Depreciation on equipment Factory superintendent's salary Total indirect manufacturing costs 2.

$ 53 99 950 $105 42 85 4 26 430 31

$ 1,102

723 $1,825

Operating income would decrease from $5,285,000 to $4,351,000: Decrease in revenue

$2,000,000

Decrease in total contribution margin*: Ratio times revenue is .467 × $2,000,000

$ 934,000

Operating income before decrease

5,285,000

New operating income $4,351,000 *Contribution margin ratio in contribution income statement is $6,538 ÷ $14,000 = .467 The above analysis is readily calculated by using data from the contribution income statement. In contrast, the data in the absorption income statement must be analyzed and divided into variable and fixed categories before the effect on operating income can be estimated.

..

191


5-B2 1.

(30-40 min.) PELLE COMPANY Income Statement For the Year Ended December 31, 20X0 Total Sales Less variable expenses: Manufacturing Selling & administrative Contribution margin Less fixed expenses: Manufacturing Selling & administrative Operating income

2.

$35,200,000 16,500,000 $11,000,000 12,100,000

$77,000,000

Per Unit $35.00

51,700,000 $25,300,000

23.50 $11.50

23,100,000 $ 2,200,000

10.50 $ 1.00

Additional details are either in the statement of the problem or in the solution to requirement 1: Total Per Unit Full manufacturing cost $46,200,000 $21.00 Variable cost: Manufacturing $35,200,000 $16.00 Selling and administrative 16,500,000 7.50 Total variable cost $51,700,000 $23.50 Full cost = fully allocated cost* Full manufacturing cost $46,200,000 $21.00 Selling and administrative expenses 28,600,000 13.00 Full cost $74,800,000 $34.00 Gross margin ($77,000,000 - $46,200,000) $30,800,000 $14.00 Contrib. margin ($77,000,000 - $51,700,000) $25,300,000 $11.50 * Students should be alerted to the loose use of these words. Their meaning may not be exactly the same from company to company. Thus, "fully allocated cost" in some companies may be used to refer to manufacturing costs only.

3.

Chuck’s analysis is incorrect. He was on the right track, but he did not distinguish sufficiently between variable and fixed costs. For example, when multiplying the additional quantity ordered by the $21 full manufacturing cost, he failed to recognize that $5.00 of the $21 full manufacturing cost was a "unitized" fixed cost allocation. The first fallacy is in regarding the total fixed cost as though it fluctuated like a variable cost. A unit fixed cost can be misleading if it is used as a basis for predicting how total costs will behave.

..

192


A second false assumption is that no selling and administrative expenses will be affected except commissions. Shipping expenses and advertising allowances will be affected also -- unless arrangements with Costco on these items differ from the regular arrangements. The following summary, which is similar to Exhibit 5-6 in the textbook, is a correct analysis. The middle columns are all that are really necessary.

Units

Without Special Order 2,200,000

Effect of Special Order 140,000 Total Per Unit $4,760,000 $34.00

With Special Order 2,340,000

Sales $77,000,000 $81,760,000 Less variable expenses: Manufacturing $35,200,000 $2,240,000 $16.00 $37,440,000 Selling and administrative 16,500,000 805,000 5.75* 17,305,000 Total variable expenses $51,700,000 $3,045,000 $21.75 $54,745,000 Contribution margin $25,300,000 $1,715,000 $12.25 $27,015,000 Less fixed expenses: Manufacturing $11,000,000 0 0.00 $11,000,000 Selling and administrative 12,100,000 47,600 0.34** 12,147,600 Total fixed expenses $23,100,000 47,600 0.34 $23,147,600 Operating income $ 2,200,000 $1,667,400 $11.91 $ 3,867,400 * Regular variable selling and administrative expenses, $16,500,000 ÷ 2,200,000 = $ 7.50 Less: Average sales commission at 5% of $35 = (1.75) Regular variable selling and admin. expenses, less commission $ 5.75 **Fixed selling and administrative expenses, special commission, $47,600 ÷ 140,000 $ .34 Some students may wish to enter the $47,600 as an extra variable cost, making the unit variable selling and administrative cost $6.09 and thus adding no fixed cost. The final result would be the same; in any event, the cost is relevant because it would not exist without the special order. Some instructors may wish to point out that a 6.4% increase in volume would cause a 75.8% increase in operating income, which seems like a high investment by Pelle to maintain a rigid pricing policy.

..

193


4.

Chuck is incorrect. Operating income would have declined from $2,200,000 to $1,600,000, a decline of $600,000. Chuck’s faulty analysis follows: Old fixed manufacturing cost per unit, $11,000,000 ÷ 2,200,000 = New fixed manufacturing cost per unit, $11,000,000 ÷ 2,800,000 = "Savings" Loss on variable manufacturing costs per unit, $15.00 - $16.00 Net savings per unit in manufacturing costs

$5.00 3.93 $ 1.07 (1.00) $ .07

The analytical pitfalls of unit-cost analysis can be avoided by using the contribution approach and concentrating on the totals:

Sales Variable manufacturing costs Other variable costs Total variable costs

Without Special Order $77,000,000

Effect of Special Order a $9,000,000

$35,200,000 16,500,000 $51,700,000

$9,600,000 0 $9,600,000

Contribution margin

$25,300,000

$ (600,000)

With Special Order $86,000,000

b

c

$44,800,000 16,500,000 $61,300,000 $24,700,000

a

600,000× $15.00 selling price of special order 600,000 × $16.00 variable manufacturing cost per unit of special order c 600,000 × $1.00 negative contribution margin per unit of special order b

No matter how fixed manufacturing costs are unitized, or spread over the units produced, their total of $11,000,000 remains unchanged by the special order.

..

194


5-B3

(10-15 min.)

1. Cost-plus pricing is adding a specified markup to cost to cover those components of the value chain not included in the cost plus a desired profit. In this case the markup is 30% of production cost. Price charged for piston pin = 1.30 × $50.00 = $65.00. If the estimated selling price is only $46 and this price cannot be influenced by Caterpillar, a manager would be unlikely to favor releasing this product for production. 2. Target costing assumes the market price cannot be influenced by companies except by changing the value of the product to consumers. The price charged would then be the $46 estimated by market research. The highest acceptable manufactured cost or target cost, T, is Dollars $ 46.00 T $ .30T

Target Price Target Cost Target Gross Margin

46 – T = .30T 1.30T = 46 T = 46 ÷ 1.30 = $35.38 3.

The required cost reduction over the product’s life is Existing manufacturing cost Target manufacturing cost Required cost reduction

$50.00 35.38 $14.62

Steps that Caterpillar managers can take to meet the required cost reduction include value engineering during the design phase, Kaizen costing during the production phase, and activity-based management throughout the product’s life. 5-1 The accountant's role in decision-making is primarily that of a technical expert on relevant information analysis, especially relevant costs. The accountant is usually an information provider, not the decision maker, although the accountant may be part of a management team charged with making decisions. 5-2 No. Only future costs that are different under different alternatives are relevant to a decision.

..

195


5-3 Past data are unchangeable regardless of present or future action and thus would not differ under different alternatives. 5-4 Past costs may be bases for formulating predictions. However, past costs are not inputs to the decision model itself because past costs cannot be changed by the decision. 5-5 Precision is a measure of the accuracy of certain data. It is a quantifiable term. Relevance is an indication of the pertinence of certain facts for the problem at hand. Ideally, data should be both precise and relevant, but relevance generally takes priority. 5-6 Decisions may have both quantitative and qualitative aspects corresponding to the nature of the facts being considered before deciding. Quantitative implications of alternative choices can be expressed in monetary or numerical terms, such as variable costs, initial investment, etc. Other relevant features may not be quantifiable, such as the quality of life in a choice between locating in San Francisco or New York. The advantage of quantitative information is that it is more objective and often easier to compare alternatives than with qualitative judgments. 5-7 The contribution approach has several advantages over the absorption approach, including a better analysis of cost-volume-profit relationships, clearer presentation of all variable costs, and more relevant arrangement of data for such decisions as make-or-buy or product expansion. 5-8 The primary classification of costs on the income statement prepared under absorption costing is by three major management functions: manufacturing, selling, and administrative. 5-9 The commonalty of approach is the focus on the differences between future costs and revenues that will differ among the alternatives. 5-10 No, fixed costs are not always irrelevant. Often they are not relevant. However, they can be relevant if they are affected by the decision being considered. 5-11 Customers are one of the factors influencing pricing decisions because they can buy or do without the product, they can make the product themselves, or they can usually purchase a similar product from another supplier. 5-12 The variable costs of a job can be misused as a guide to pricing. However, the adjusted markup percentages based on variable costs can have the same price result as those based on total costs, plus they have the advantage of indicating the minimum price at which any sale may be considered profitable in the short run. 5-13 Three examples of pricing decisions are (1) pricing new products, (2) pricing products sold under private labels, and (3) responding to new prices of a competitor's products. ..

196


5-14 Three popular markup formulas are (1) as a percentage of variable manufacturing costs, (2) as a percentage of total variable costs, and (3) as a percentage of full costs. 5-15

(a) Competitors’ action (b) Customer demands

5-16 Full costs are more popular than variable costs for pricing because price stability is encouraged and in the long run all costs must be recovered to stay in business. 5-17 Target cost per unit is the average total unit cost over the product’s life cycle that will yield the desired profit margin. 5-18 Value engineering is a cost-reduction technique, used primarily during the design function in the value chain, that uses information about all value chain functions to satisfy customer needs while reducing costs. 5-19 Kaizen costing is the Japanese term for continuous improvement during manufacturing. 5-20 In target costing, managers start with a market price. Then they try to design a product with costs low enough to be profitable at that price. Thus, prices essentially determine costs. 5-21 Customer demands and requirements are important in the product development process. Many companies seek customer input on the design of product features. They seek to reduce non-value-added costs without affecting product features that are valuable to customers. Suppliers are also important. Companies purchase many of the materials used in products. They have to work with suppliers to get the lowest cost for these materials. 5-22 Not necessarily. There are other important factors that management must consider before discontinuing a product. The product may be necessary to round out a product line. The product may be the company’s attempt to break into a new market area or new product class. 5-23 No. There is confusion between total fixed costs and unit fixed costs. Increasing sales volume will decrease unit fixed costs, but not total fixed costs. This assumes that the volume increase results in operating levels that are still within the relevant range. 5-24 Managers generally find contribution margin income statements more useful, especially if they are concerned with short-term results. The contribution margin statement provides information on the immediate profit impact of increases or decreases in sales.

..

197


5-25 Marginal cost is the additional cost resulting from producing and selling one additional unit. It changes as production volume changes. With a given fixed capacity, marginal cost generally decreases up to a point and then increases. Variable cost is the accountant's approximation to marginal cost. It remains constant over the relevant range of volume. Because the difference between these two costs often is not material (within the relevant range), in such cases we can use the variable-cost estimate of marginal cost for decision-making purposes. 5-26 Pricing decisions must be made within legal constraints. These laws help protect companies from predatory and discriminatory pricing. Predatory pricing involves setting prices so low that they drive competitors out of the market. Discriminatory pricing is charging different prices to different customers for the same product or service. 5-27 Managers are directly involved in the research and development and the design functions. During the initial product research phase, managers often are involved in surveys, focus groups (with major airlines), and other market research activities to explore the potential for a new airplane. During process and product design, managers help with such tasks as negotiations with suppliers and cost analyses. Production managers provide input regarding cost reduction ideas. Marketing managers provide input regarding customer needs (a super large plane with more than 500 seats versus more medium-sized planes that can serve more markets). Distribution managers provide input regarding the costs of various channels of distribution. Finally, managers involved with customer relations provide input regarding the likely cost-to-serve profile for expected customers for a new product. 5-28

(5 min.)

All the data given comprises historical costs. Most students will identify the $30 and $42 as relevant prices. They will also declare that the $10 cost for snacks is irrelevant. Guide them to see that the relevant admission prices are expected future costs that will differ between the alternatives. The past prices are being used as a basis for predicting the future prices. Similarly, the snacks costed the same at both stadiums in the past. Hence, it is regarded as irrelevant under the assumption that the price will remain the same in the upcoming matches as well.

..

198


5-29 (20 min.) Some students may forget to apply the 10% wage rate increase to both alternatives.

(1)

(2)

(A) Historical Information

(B) Other Information

Prediction Method Predictions as inputs to decision model

(3)

Decision Model

Decisions by managers with aid of decision model (4)

Implementation and Evaluation Feedback

..

(1) Historical direct materials were $5.00 per unit; direct labor was $6.00 per unit. (2) Direct material costs are expected to fall by 10%, or 50¢ per unit. Direct labor costs are affected by a 10% rate increase and a 5% increase in labor time if the new material is used. (3) Cost comparisons per unit: Old Material Direct material $ 5.00 Direct labor $6.00 × 110% 6.60 $6.00×110%×105% Expected future cost $11.60

New Material $ 4.50 6.93 $11.43

(4) The chosen action is implemented, and the evaluation of performance becomes a principal source of feedback. This historical information aids the decision process (prediction, decision, and implementation) of future decisions.

199


5-30

(10 min.)

Relevant costs are future costs that differ between alternatives, while irrelevant costs remain constant. Here, the irrelevant costs are the Broadway show ticket cost, automobile costs, and baby-sitting cost for the first three hours. The relevant costs are: Tickets, 2 @ $70 each Parking Baby-sitting, 1 extra hour @ $20 Total

Broadway show Concert $0 $140 0 10 0 $0

Difference $140 10

20 $170

20 $170

The music concert is more costly by $170 to Benjamin and Emma than the Broadway show. 5-31

(10 min.) This is a basic exercise. Answers are in dollars (in thousands).

1. 2. 3. 4.

360 + 460 + 420 = 1,240 1,560 – 1,240 = 320 320 – 260 = 60 1,240 – 360 = 880; or 460 + 420 = 880

5-32

(10-15 min.) This is a basic exercise. Data are in million ¥. Sales Variable expenses: Direct materials Direct labor Variable factory overhead (a) Variable manufacturing cost of goods sold Variable selling and admin expenses Total variable expenses (b) Contribution margin Fixed expenses: Fixed factory overhead Fixed selling and administrative expenses (c) Operating income

..

¥2,970 ¥750 420 195 ¥1,365 345 1,710 ¥1,260 ¥330 225

555 ¥ 705

200


5-33

(15-20 min.)

This is a straightforward exercise in basic terms and relationships. To fill all the blanks, both absorption and contribution income statements must be prepared. The data is in million €. Required answers are in italics. Absorption Approach Sales Direct materials used Direct labor Variable indirect manufacturing costs Variable manufacturing cost of goods sold Variable selling and administrative expenses Total variable expenses k. Contribution margin Fixed factory overhead g. Manufacturing cost of goods sold j. Gross profit Fixed selling and administrative expenses Variable selling and administrative expenses

Contribution Approach

€2,760

€2,760

€1,050

€1,050

630

630

300

300

f.

Operating income

..

1,980 270 2,250 510 150

150 2,130 630

240 270

240

390

510 € 120

€ 120

201


5-34

(10-20 min.) Answers are in thousand C$. Prime costs = Direct material + Direct labor 2,300 = 1,420 + DL DL = 880

The body of a model income statement follows. The computations are explained for each item that was originally blank. Numbers given in the problem are in bold. Sales, 2,980 + 540 Direct materials Direct labor (as above) Factory overhead, 2,980 (1,420 + 880) Manufacturing cost of goods sold Gross margin Selling and administrative expenses, 540 – 120 Operating income

C$3,520 C$1,420 880 680 2,980 C$540 420 C$120

5-35 (15-20 min.) The data is placed in the format of the income statement and the unknowns are computed as shown. Answers are in thousand ZAR. Sales Variable expenses Direct materials Direct labor Variable indirect manufacturing Variable manufacturing cost of goods sold Variable selling and administrative expenses Total variable expenses (17,100 – 3,700) Contribution margin Fixed expenses Fixed indirect manufacturing Fixed selling and administrative expenses Operating income

ZAR17,100 ZAR3,200 3,300 2,000

8,500 1 4,900 2 13,400 3,700

ZAR700 3 2,100 2,800 ZAR 900

1

3,200 + 3,300 + 2,000 = 8,500 17,100 – 3,700 – 8,500 = 4,900 3 Total fixed expenses = 3,700 – 900 = 2,800 Fixed indirect manufacturing = 2,800 – 2,100 = 700 2

..

202


5-36

(10-15 min.)

1.

Operating income would increase by $300 if the order is accepted.

Units Sales Purchase cost Variable printing cost Total variable cost Contribution margin Fixed cost Operating income

Without Special Order 2,000 $36,000 20,000 4,000 24,000 12,000 8,000 $ 4,000

Effect of Special Order 100 $1,500 1,000 200 1,200 300 0 $ 300

With Special Order 2,100 $37,500 21,000 4,200 25,200 12,300 8,000 $ 4,300

2.

If maximizing operating income in the short run were the only goal, the order should be accepted. However, if qualitative considerations favoring rejection are worth more than the $300 increase in operating income, the manager would reject the offer. For example, accepting the offer from F. C. Kitsap may generate similar offers from other clubs who now willingly pay the $18 normal price. Lost profits on such business might more than offset the $300 gain on this sale. On the other hand, this might be a way of gaining F. C. Kitsap as a regular customer who will then buy other items that generate a profit well in excess of the $300.

..

203


5-37

(20 min.)

1.

Total Variable Costs

Total Fixed Costs

$9

$150

Volume in Number of Lunches

Volume in Number of Lunches

Total Costs Variable Fixed Volume in Number of Lunches 2.

There are correct ways and incorrect ways to analyze the data. A correct way follows: Total cost = Total FC + Total VC = $150 per year + $9 per lunch Let X = The number of lunches Then, Unit cost = ($150 ÷ X) + $9 If 1 lunch, Unit cost = ($150 ÷ 1) + $9= $159.00 per lunch If 12 lunches, Unit cost = ($150 ÷ 12) + $9 = $ 21.50 per lunch If 200 lunches, Unit cost = ($150 ÷ 200) + $9 = $ 9.75 per lunch

..

204


3.

(a) The CPA can compare either total annual costs or unit costs. Let X = the total number of lunches in question.

Elsewhere In general.............................$ 10X For 1 lunch ..........................$ 10 For 12 lunches .....................$ 120 For 200 lunches ...................$2,000

Total Costs At Club $150+$ 9X $150+$ 9 = $159 $150+$ 108 = $258 $150+$1,800 =$1,950

Unit Costs Elsewhere At Club $10.00 ($150÷X)+$9 $10.00 $159.00 $10.00 $ 21.50 $10.00 $ 9.75

Let X = Number of lunches $10X = $150 + $9X X = 150 lunches is point of indifference. (b) Elsewhere, 200 × $10 At Club, $150 + 200($9) Savings

$2,000 1,950 $ 50

The preceding parts concentrated on how total costs behave in relation to chosen volume levels. Generally, the decision maker should take a straightforward, analytical approach by thinking in terms of total costs rather than unit costs. By keeping an eye on the total picture, the manager is less likely to fall into some analytical traps that come from misinterpreting unit costs. In addition, of course, the qualitative aspects should not be ignored. For example, there may be an intangible benefit of dining with actual and potential clients at the luncheon club. 5-38

(15 min.)

1.

Except for the advertising costs, the fixed costs are irrelevant in this situation. The contribution margin per student is: £15,000 – £9,000 = £6,000 Break-even point for the campaign is: £1,530,000 ÷ £6,000 = 255 additional students

2.

320 × £6,000 = £1,920,000

3.

90 × £6,000 = £540,000

..

205


5-39

(10 min.)

Variable manufacturing cost Variable selling and admin. cost (a) Total variable cost Fixed manufacturing cost (b) Full manufacturing cost Fixed selling and administrative cost (c) Full cost

Cost per Unit of Product $12.00 $12.00 $12.00 7.00 7.00 $19.00 5.00* 5.00 $17.00** 8.10* $32.10

* Fixed manufacturing cost, $500,000 ÷ 100,000 = $5.00 Fixed selling and admin. cost, $810,000 ÷ 100,000 = $8.10 ** This amount must be used by U.S. companies for inventory valuation in reports to shareholders. 5-40

(20 min.)

This solution may be obvious to most students. However, the use of this problem in executive programs and regular classes has shown that some students need this exercise before they become convinced that the “unitization” of fixed costs can be misleading. Moreover, in decision-making in general, the use of total rather than unit cost, especially for fixed costs, is nearly always less confusing. This special order increases the company’s revenue by $900,000 and variable costs by $960,000. Total fixed costs remain unchanged at $600,000. This $600,000 is unaffected regardless of how it is allocated to product units. Therefore, net income will be affected only by the changes in revenue and variable costs. Summary of regular operations: Per Unit Total Revenue $4.00 $1,200,000 Variable costs 3.20 960,000 Contribution margin $ .80 $240,000 Fixed costs 2.00 600,000 Net income ($ 1.20) ($360,000) The new business would alter the picture as follows, assuming fixed costs are “spread” on a 50/50 basis: Regular Special Total Revenue $1,200,000 $900,000 $2,100,000 Variable costs 960,000 960,000 1,920,000 Contribution margin $240,000 ($60,000) $ 180,000 ..

206


Fixed costs 300,000 300,000 600,000 Net income ($30,000) ($360,000) ($420,000) No matter how the fixed costs are spread, the total fixed costs will remain $600,000 and the total net loss will be $420,000. This is true despite the fact that fixed costs per unit have reduced from $2.00 to $.100. The moral is: beware of unit fixed costs. Some instructors may want to emphasize how the unitization of fixed costs differs. That is, the unit cost depends on the production volume chosen as the denominator. Fixed costs per unit = Total fixed costs ÷ Production vol. = $600,000 ÷ 300,000 units = $2 or $300,000 ÷ 300,000 units = $1.00 The total fixed cost is unaffected by what volume is chosen as the denominator for computing the cost per unit.

..

207


5-41

(10-15 min.)

Pricing policies always seem to spark much student interest. This “break-even” philosophy is similar to the “base or bulk volume” philosophy favored by many executives. That is, the “normal” pricing applies to the bulk or base of the business, but price-cutting can be applied to incremental business. In the case of the auto business, this normal-incremental pricing is applied by many dealers in the manner described in the problem. Many observers think such pricing does not make sense unless it is a response to changes in demand and in competitor pricing. Why are some observers against such pricing? Because prices should be influenced by customer demand and competition, not by where sales happen to be on a break-even graph. Ordinarily, a pricing strategy should aim to maximize the contribution margin, all other factors being equal. Some critics maintain that it is foolhardy to cut a price to the same potential customer just because he or she appears on, say, May 27 rather than on May 23. As prospective customers, most rational people would shop for a car during the final two or three days of the month. 5-42

(15 min.)

1.

Assuming that total fixed costs are the same at production levels of 6,000 and 10,000 units, the analysis can focus on contribution margins: CM@ $12.50: 6,000 units × ($12.50-$6) = $39,000 CM@ $10: 10,000 units × ($10 - $6) = $40,000 Profits will be $40,000 - $39,000 = $1,000 higher at the $10 price.

2.

Subjective factors include image in the marketplace (higher price may give an image of quality), market penetration (satisfied customers may become repeat customers), and effects on the sales force.

5-43

(10 min.)

1.

(₹1,935,000 – ₹774,000) ÷ ₹774,000 = 150%

2.

(₹1,935,000 – ₹1,075,000) ÷ ₹1,075,000 = 80%

3.

(₹1,935,000 – ₹645,000) ÷ ₹645,000 = 200%

..

208


5-44

(10-15 min.)

1.

(150% × $30,000) + ($75 × 2,000 hours) = $45,000 + $150,000 = $195,000

2 & 3. Materials and supplies, at cost Hourly pay for consultants Fringe benefits for consultants Total variable cost Extra fixed costs incurred 2. Minimum bid Additional allocated fixed costs Total cost Desired mark-up, 20% × $168,000 3. Bid to achieve desired profit 5-45

$ 30,000 70,000 24,000 $124,000 9,000 $133,000 35,000 $168,000 33,600 $201,600

(10 min.)

Unit Target Cost = Target Price – Target Profit = £500 – (.2 × £500) = £400 Total Target Cost (£400 × 160,000) Less cost to design and develop Total target cost to manufacture, sell, distribute and service the home security systems Unit target cost to manufacture, sell, distribute, and service the home security systems (£62,850,000 160,000)

£64,000,000 1,150,000 £62,850,000 £392.81

Note that the unit amounts are averages over the entire product life cycle. Thus, £500 is the expected average selling price and £392.81 is the average cost to manufacture, sell, distribute, and service the home security systems. The initial selling price may be substantially higher than £500 and the initial costs may also be higher than £392.81. Continuous improvements across the value chain (kaizen costing) will bring down the costs just as competitive market forces will very likely bring down the price.

..

209


5-46 1.

(15 – 20 min.)

Unit target cost = Target price – Target profit = $250 – (.25 × $250) = $187.50 Existing unit cost = Total cost over product life  Total demand = $20,000,000  100,000 = $200 The new product should not be released to production.

2.

Total cost savings = (.4 × $10,000,000) – $2,200,000 = $1,800,000

On a per unit basis, the saving amounts to $1,800,000  100,000 = $18 The new expected average unit cost will be reduced to $200 – $18 = $182, which is below the target cost. So, the new product should be released to production. 5-47

(10 min.)

This problem raises issues for which there are no right answers. Determining the types of product promotion activities that are ethically and legally appropriate is not an easy question, and the role of price discrimination is especially difficult. For a company to legally charge different prices to different customers, it usually must show a cost difference in serving the customers. But many companies promote their products by charging a zero price (i.e., giving free samples for a limited amount of the product). Is this case any different than a breakfast cereal company sending free samples through the mail? If so, how? Further, establishing physicians’ confidence in the medication has a potential long-run benefit; does this justify giving the drug free to physicians? In addition, physicians need to know how to administer the drug and how to look for possible side effects, so are the free samples justified as an educational investment? Or are the free drug samples essentially bribes to convince physicians to prescribe the new drug? What about the difference in price between hospital and retail pharmacies? GLPI may think that if a hospital pharmacy starts a patient on the new drug, he or she will stay on it even if further purchases are from a retail pharmacy. Does this justify a price differential? Or it may be that distribution costs are less to hospital pharmacies than to retail pharmacies. Is this difference enough to justify a $15 difference in price? Students are likely to disagree on the appropriateness of the policies, and some may feel passionately about their opinion. At some time the discussion should be turned to the effect of cost on the pricing policies. For example, a lead-in question may be whether the eventual price of $50 is fair for a product whose production cost is only $12. Then it can proceed to considering whether a cost differential can justify the $15 ..

210


difference between the prices to hospital and retail pharmacies. Finally, the issue of price and incentives to physicians can be addressed. This last issue may be the first one students want to focus on, and it may be the one with the most ethical content, but it should not be the sole issue discussed. 5-48

(25-35 min.) All amounts are in millions of Euros.

1.

LAGRANDE CORPORATION Contribution Income Statement For 2012 Sales Less variable expenses: Manufacturing cost of goods sold Selling and administrative expenses Contribution margin Less fixed expenses: Manufacturing costs Selling and administrative expenses Operating income

2.

..

(a)

€900 € 300 140 280 60

440 460 340 €120

Sales: €900 × 90% × 130% Variable expenses: €440 × 130% Contribution margin Fixed expenses Operating income

€1,053 572 481 * 340 € 141

*Alternative computation of contribution margin: Sales after a 10% reduction in prices: € 900 × 90% Variable expenses Contribution margin before volume change Add 30% of € 370 Estimated new contribution margin

€ 810 440 370 111 € 481

(b)

Contribution margin: €460 × 110% Fixed expenses: €340 + 30 Operating income

€506 370 €136

(c)

Sales Variable expenses: Manufacturing: €300 × 85% Selling and administrative Contribution margin Fixed expenses: €340 + € 80 Operating income

€900 €255 140

395 505 420 € 85

211


(d)

Sales: €900 × 120% × 105% Variable expenses: Manufacturing: €300 × 120% Selling and administrative: €140 × 120% × 125% Contribution margin Fixed expenses: Manufacturing Selling and administrative: €60 × 2 Operating income **Alternate computation of contribution margin: Sales after a 5% increase in prices: €900 × 105% Variable expenses: Manufacturing Selling and admin. after a 25% increase in unit costs: €140 × 125% Contribution margin before volume change Add 20% of €470 Estimated new contribution margin

(e)

€1,134 €360 210 €280 120

570 564 ** 400 € 164

€ 945 €300 175

475 470 94 € 564

These computations are good examples of "sensitivity analysis"--testing various inputs to a model to measure the effects on estimated outputs. This is a planning procedure. An important point to make with students is that the contribution form of income statement is much more appropriate for these purposes than the absorption form. The analysis is readily calculated by using data from the contribution income statement. In contrast, the data in the absorption income statement must be analyzed and split into variable and fixed categories before the effect on operating income can be estimated.

3.

Alternative (c) is clearly undesirable because it produces less operating income than the status quo. Alternatives (a), (b) or (d) would be better than the status quo. However, if these alternatives cannot be undertaken simultaneously, and if there is no subjective reason to favor alternatives (a) or (b), alternative (d) seems best. It produces €164 – €141 = €23 (or 23 million Euros) more operating income than the next best alternative (a).

..

212


5-49

(10-15 min.)

1. Sales Fully allocated operating expenses Variable operating expenses (75% × ₩15,000,000) Apparent change in operating income (loss) 2.

Contribution Approach ₩12,000,000 11,250,000 ₩750,000

Fully Allocated Cost Approach ₩12,000,000 15,000,000 ₩(3,000,000)

Yes, Global Logistics should accept this special order. Not accepting the order would mean that the short-run income would be lower by ₩750,000. In effect, by turning down the business, Global Logistics invests ₩750,000 to possibly achieve some long-run benefits. Chin Sun can find the contribution approach helpful because he can weigh decisions of this sort by asking whether the probability of long-run benefits (neither encouraging price-cutting by competitors, not encouraging customers to expect lower prices) is worth a quantifiable present investment equal to the contribution margin (₩750,000 in this case). By itself, the contribution approach does not say “go forth and cut prices.” All it does is quantify a manager’s options more sharply.

5-50

(20-25 min.)

1.

Net income would be increased by £1,175 if the order were taken: Without Effect of With the Order the Order the Order Sales £3,078,500 £40,000 £3,118,500 Direct material £737,500 £13,250 £750,750 Direct labor 850,000 15,500 865,500 Variable overhead 552,500* 10,075 562,575 Fixed overhead 425,000 0 425,000 Total costs £2,565,000 £38825 £2,603,825 Operating income £513,500 £1,175 £514,675 * Variable overhead is Total overhead – Fixed overhead, or £977,500 – £425,000 = £552,500. Variable overhead rate = £552,500 ÷ £850,000 = 65% of Direct labor

2.

A contribution approach to pricing might appear as follows: Selling price Direct materials £13,250 Direct labor 15,500 Variable overhead at 65% of direct labor 10,075 Total variable cost Contribution margin

..

£40,000

38,825 £1,175 213


The contribution approach essentially attempts to provide a measure of the decrease in immediate net income that would result from rejecting an order. This is the contribution margin forgone. Traditional approaches to pricing do not supply such a number. In part (1), the £1,175 tells Amelia that she is investing £1,175 now to uphold her pricing policies. She can then assess whether preserving such policies and the long-run pricing structure is worth an investment of such magnitude. She also may assess whether accepting marginal business will cause this customer to seek such concessions regularly. Alternatively, Amelia may want to make such concessions occasionally to attract new customers. A possible contribution margin formula may be illustrated as follows: Direct material £13,250 Direct labor 15,500 Variable overhead at 65% of direct labor 10,075 Total variable cost £38,825 Markup at 48.1%* of £38,825 18,675 Target selling price £57,500 *Normal markup percentage = (£57,500 – £38,825) ÷ £38,825 = 48.1%. Note that the markup of 48.1% is much higher than the 20% used previously because the markup must provide for the recovery of fixed overhead as well as the making of net income. The key to the contribution approach is its intelligent use with full recognition that total variable cost is not total cost.

..

214


5-51

(15-20 min.)

1. Tuition revenues Costs of courses Contribution margin General administrative expenses Operating income 2.

Year to Date $3,000,000 1,200,000 $1,800,000 600,000 $ 1,200,000

Final Course Enrollment 40 10 More $10,000 $1,500 6,000 600 $4,000 $900

Grand Totals $3,011,500 1,206,600 $1,804,900

0 $4,000

600,000 $1,204,900

0 $900

The same general considerations influence pricing decisions in profit-seeking and nonprofit organizations. The exception is price-setting by many governmentowned entities, which often is heavily affected by legislative bodies. The familiar three Cs—customers, costs, and competition—do influence price setting. Executive education is highly competitive; the rates for top-flight teachers are relatively high; and customers often do without or conduct their own in-house training. The offering of discounts is often risky. It may alienate full-paying customers, may lead to widespread price-cutting, and may encourage the particular customers to bargain hard regarding course after course. The setting of tuition in private universities is similar to setting prices in private companies. Customers may go to the competition, to other private or public universities. Costs must be recovered if the institution is to survive. Of course, tuition is only one part of a university’s revenue. Private institutions are especially dependent on endowment income and on donations from friends and alumni.

..

215


5-52 (15 min.) 1.

Contribution margin from direct sales through website = $13 - $3 = $10 Royalty from online streaming partners = 40% of $5.00 = $2 Total royalty from online streaming partners = 20 million x $2 = $40 million Direct download from website @ $13 to get CM of $40,000,000 = $40,000,000  $10 = 4,000,000. So, more than 4,000,000 direct downloads would be required.

2.

Total contribution from direct downloads = 5.5 million × ($13 – $3) = $55 million. No of views required to earn CM of $55 million = $55 million ÷ $2 = $27.5 million

3.

The cost of producing the movie is irrelevant to the decision because it is already incurred and has no effect on the revenues. However, a correct way of promoting the movie may affect the revenue and so promoting is relevant to this decision.

..

216


5-53

(20 min.)

Number of flights per month Available seats Seats filled Percent filled Revenue Variable expenses Contribution margin

Basic 3,000 300,000 156,000 52% $31,200,000 21,840,000 $ 9,360,000

Marginal Total 120 3,120 12,000 312,000 2,400 158,400 20% 51% $240,000 $31,440,000 120,000 21,960,000 $120,000 $ 9,480,000

Continental's approach was described by Chris F. Whelan, vice president in charge of economic planning, who made the scheduling decisions. He used a marginal (variable cost) approach, which was described as follows: “Whelan considers that the bulk of his scheduled flights have to return at least their fully allocated costs. Overhead, depreciation, and insurance are very real expenses and must be covered. The out-of-pocket approach comes into play, says Whelan, only after the line's basic schedule has been set. "Then you go a step farther," he says, and see if adding more flights will contribute to profits. Similarly, if he's thinking of dropping a flight with a disappointing record, he puts it under the marginal microscope: "If your revenues are going to be more than your out-of-pocket costs, you should keep the flight on." By "out-of-pocket costs" Whelan means just that: The actual dollars that Continental has to pay out to run a flight. He gets the figure not by applying hypothetical equations but by circulating a proposed schedule to every operating department concerned and finding out just what extra expenses it will entail. If a ground crew already on duty can service the plane, the flight isn't charged a penny of their salary expense. There may even be some costs eliminated in running the flight; they won't need employees to roll the plane to a hanger, for instance, if it flies on to another stop. Most of these extra flights, of course, are run at off-peak hours, mainly late at night. At times, though, Continental discovers that the hours aren't so unpopular after all. A pair of night coach flights on the Houston-San Antonio-El Paso-Phoenix-Los Angeles leg, added on a marginal basis, have turned out to be so successful that they are now more than covering fully allocated costs. Alternative. Whelan uses an alternative cost analysis closely allied with the marginal concept in drawing up schedules. For instance, on his 11:11 p.m. flight from Colorado Springs to Denver and a 5:20 a.m. flight the other way, Continental uses Viscounts that carry cargo but often go without a single passenger. But the net cost of these flights is less than the rent for overnight hangar space for the Viscount at Colorado Springs. ..

217


And there are other absolute-loss flight scheduled solely to bring passengers to a connecting Continental long-haul flight; even when the loss on the feeder service is considered a cost on the long-haul service, the line makes a net profit on the trip. Continental's data handling system produces weekly reports on each flight, with revenues measured against both out-of-pocket and fully allocated costs. Whelan uses these to give each flight a careful analysis at least once a quarter. But those added on a marginal basis get the fine-tooth-comb treatment monthly. The business on these flights tends to be useful as a leading indicator, Whelan finds, since the off-peak traffic is more than normally sensitive to economic trends and will fall off sooner than that on the popular-hour flights. When he sees the night coach flights turning in consistently poor showings, it's a clue to lower his projections for the rest of the schedule. 5-54

(15-20 min.)

1.

Total variable costs are $.85 + $.65 = $1.50 per boomerang. Total fixed costs are $109,000 + $23,000 = $132,000

Volume in units Sales @ $3.20 Total variable costs @ $1.50 Contribution margin Fixed costs Operating income Operating income as a percentage of sales 2.

..

170,000 $544,000 255,000 289,000 132,000 $157,000

220,000 $704,000 330,000 374,000 132,000 $242,000

260,000 $832,000 390,000 442,000 132,000 $310,000

28.9%

34.4%

37.3%

Note the significant difference in predictions. For example, the correct analysis indicates $157,000 operating income at a 170,000 volume level; the incorrect analysis indicates $187,000 operating income. The manager's tabulation is incorrect because it assumes that all costs are variable. The presence of a larger proportion of fixed costs causes much wider swings in operating income when volume deviates from the volume used to develop the full costs per boomerang.

218


5-55

(15-20 min.)

1.

Compare option a to option b: Extra revenue from option a: (₹3,840 – ₹1,800) × 30 passengers = ₹61,200 Extra costs for option a: [(₹264 – ₹24) × 88 km] + ₹48,000 = ₹69,120 Therefore, option b (adding a car to an existing train) is more profitable by ₹69,120 - ₹61,200 = ₹7,920. Costs that are the same for both alternatives are irrelevant. These include the cost of the tour guide, cost of moving the car or car and engine to the main track (assuming both options require an additional car to be moved to the track), and depreciation.

2.

This depends on the total additional revenues and costs for option b, the best of the two options: Revenue: ₹1,800 × 30 Costs: Fuel – 88 km × ₹24/km Tour guide Moving car Total additional cost Extra profit

₹54,000.00 ₹ 2,112.00 24,000.00 4,800.00 30,912.00 ₹23,088.00

This option is definitely profitable, generating extra profit of ₹23,088. The cost of the tour guide and the cost of moving the car to the main track are relevant for this decision because they would be incurred only if the agreement with the tour operator is accepted. The depreciation remains irrelevant as long as excess cars are available. 5-56

(15-20 min.)

1.

Net income will be increased by 300 × (€40 - €25 - €10) = €1,500.

2.

The lowest sales price per unit is equal to the variable manufacturing costs per unit: €25.

3.

€180,000, €70,000, €30,000, €10; i.e., all numbers are irrelevant except €25.

4.

Selling price: €180,000 ÷ 2,000 units = €90 Total sales: 2,400 × 2 × €90 = Less expenses: Fixed: €70,000 + €30,000 + €125,000* Variable: 2,400 × 2 × (€25 + €10) Net income

€432,000 €225,000 168,000

393,000 € 39,000

*Depreciation: €500,000 ÷ 4 = €125,000. ..

219


5-57

(15-25 min.)

1.

Budgeted fixed manufacturing overhead per unit: €84,000,000 ÷ 6,000,000 = €14

2.

Relevant items: Additional sales Additional variable manufacturing costs, 150,000 × €12 Additional selling and administrative expenses Total relevant costs Additional operating income

€2,300,000 €1,800,000 10,000 €1,810,000 € 490,000

Fixed manufacturing costs are irrelevant because their total will be the same regardless of the special order being accepted or rejected. 3.

Students may raise many points, including: a.

Whether the CEO is willing to "invest" €490,000 in forgone operating income now to preserve a marketing policy or to prevent a general weakening of prices among competitors.

b.

Whether accepting the order now may lead to more profitable orders from the same customer subsequently.

4.

Budgeted fixed manufacturing overhead rate would be €84,000,000 ÷ 3,000,000 = $28. However, the additional operating income in requirement 2 would be unaffected by how fixed costs are "unitized." (Of course, the original budgeted operating income would have been different, but that is irrelevant in requirements 1 and 2.) The only point to consider would be if they have enough excess capacity to accept the special order.

..

220


5-58 (20-30 min.) When this problem was used in an exam, it was well done by students who used contribution margin analysis in total dollars. A number of students attempted to force a decision by means of analysis of unit costs or by break-even analysis, failing to consider the effect of sales volume on profits. A number of good solutions were marred by failure to draw specific conclusions. Output and pricing: Volume 130,000 150,000 170,000 190,000 210,000

CM per Price Unit $25.00 $13.00 24.00 12.00 23.00 11.00 22.00 10.00 21.00 9.00

Total Contribution Margin $1,690,000 1,800,000 1,870,000 1,900,000 1,890,000

The contribution margin per unit decreases as volume increases. Output of 190,000 at selling price of $22 yields the largest contribution margin. However, this is in excess of present capacity. Maximum contribution at present capacity: 150,000 units output at $24 = Contribution margin of $1,800,000 This is $1,800,000 - $1,690,000 = $110,000 more contribution than is generated by the current price of $25 for 130,000 units. Even with no capacity expansion, the price should be dropped to $24. Now consider increasing capacity: Additional investment Useful life Cost per year ($1,250,000 ÷ 10)

$1,250,000 10 years $125,000

By increasing the capacity from 150,000 units to 190,000 units, which maximizes the total contribution margin, the company gains an additional $100,000 ($1,900,000 – $1,800,000) in contribution margin but incurs an additional fixed cost of $125,000. In simple words, benefits from the expansion are less than the cost incurred for expansion. Conclusions:

..

Do not invest in new capacity. Rather utilize the maximum available capacity of 150,000 units and sell all 150,000 units at $24 per unit.

221


5-59

(10-15 min.)

1.

Manufacturing cost $24.00 Gross margin, (15% × $24.00) 3.60 Price $27.60 GenNext would not produce a smart LED bulb because it would not be able to sell them at $27.60, assuming that market research is right about the market price of $25.00.

2.

Using target costing, GenNext would begin with the market price of $25.00. From this, managers would compute the largest acceptable manufacturing cost, $21.67: Price Less gross margin Manufacturing cost

$25.00 3.26* $21.74

* Price = Cost + (.15 × Cost) $25.00 = 1.15 × Cost Cost = $25.00  1.15 = $21.74 Margin = $25.00 - $21.74 = $3.26 3.

GenNext managers would have to determine if they could design the smart LED bulbs and its production process in a way that manufacturing costs were below $21.74. Both the design specifications for the smart LED bulbs and the production process would need to be looked at. If there is no way to reduce production costs to $21.74 or below, the product should not be produced. However, target costing forces managers to examine ways to lower the production costs through product and process design. Instead of taking the design and process as givens and then examining the market to see if GenNext can sell the product for a high enough price, the company’s managers will try to design a product and process that meets the constraints of the market.

..

222


5-60

(35-45 min.) C-200472

Direct material Setup/maintenance Processing Marketing Customer service Total existing cost Total demand in units Existing cost per unit Target cost per unit Required cost reduction RCR as a percent of market price Decision

Cost per driver unit $1.60/ pound $1,015/ setup $370/ mach. hr. $860/order $162/sales call

Number of units 2,000 10 20 30 55

Cost $3,200 10,150 7,400 25,800 8,910 $55,460 2,000 $ 27.73 $23.40* $ 4.33 11.1%

Redesign product and process using value engineering

C-200473 Numbe r of units Cost 1,000 $ 1,600 4 4,060 12 4,440 10 8,600 35 5,670 $24,370 1,400 $ 17.41 $ 16.80 $ 0.61 2.2% Release to production and set kaizen cost improvement plan

C-200474 Number of units 4,000 12 32 50 20

Cost $ 6,400 12,180 11,840 43,000 3,240 $76,660 4,000 $ 19.17 $ 21.00 $ 0

C-200475 Numbe r of units Cost 800 $1,280 5 5,075 12 4,440 16 13,760 28 4,536 $29,091 600 $ 48.49 $ 30.00 $ 18.49

0%

37.0%

Release to production

Abandon subject to approval

* Target cost = (1 – desired contribution percentage) × market price = (1 - .40) × $39 = $23.40.

..

223


5-61

(20 min.)

1.

Contribution margin = $950 - ($610 + $45) = $295 Total contribution = $295 × 47,700 blowers = $14,071,500 Total fixed costs = 7 years × ($900,000 + $75,000) = $6,825,000 Development costs = $5,300,000 Life-cycle profit = $14,071,500 - $6,825,000 - $5,300,000 = $1,946,500

2.

Desired profit = .10 × ($950 × 47,700) = $4,531,500 The life cycle profit is $4,531,500 - $1,946,500 = $2,585,000 short of what is desired. Therefore, unless some changes can be made, River Line will not enter the snow blower market.

3.

A target costing company does not quit when the first cost estimate comes in too high. Managers establish a target cost and try to adjust design, production and marketing processes to meet the target cost. In this case, the target cost is: Revenue Desired profit Target cost Expected costs are: Variable production costs Fixed production costs Variable selling costs Fixed selling costs Development costs Total costs

$45,315,000 4,531,500 $40,783,500 $29,097,000 6,300,000 2,146,500 525,000 5,300,000 $43,368,500

If total costs can be reduced by at least $2,585,000, to $40,783,500 or less by changes in the product’s design, the production process design, or production or selling methods, this will begin to be a profitable product.

..

224


5-62

(30 – 40 min.)

Fixed overhead allocation rate per machine hour = €2,160,000  90,000 = €24 Variable overhead allocation rate = €40-€24 = €16 per machine hour St. Tropez should not accept either order. The company does not have adequate plant capacity to manufacture the order of 20,000 jewelry cases from Lyon Inc. without subcontracting. The order from Avignon Co. does not yield St. Tropez a positive contribution margin. The calculations showing that St. Tropez does not have the necessary plant capacity in the second quarter to produce 20,000 jewelry cases for Lyon are as follows: Annual plant capacity Monthly plant capacity Estimated monthly capacity use, .8 × 7,500 Excess capacity per month Period involved, second quarter Total excess capacity available

90,000 machine hours 7,500 machine hours 6,000 machine hours 1,500 machine hours × 3 months 4,500 machine hours

Machine hours required to produce 20,000 jewelry cases = Number of cases × machine hours per case = 20,000 × .25 = 5,000 hours. The Lyon Inc. order for 20,000 jewelry cases would require 5,000 machine hours, but only 4,500 machine hours are available in the second quarter. Computations related to the order from Avignon Co. are as follows: Price offered per case Variable production cost per case: Raw materials Direct labor, .5 hours @ €60 Overhead, .5 machine hours @ €16* Contribution margin per case Number of cases Total contribution margin Fixed costs related to the order: Setup costs Special device Loss from taking the order

€ 85 €43 30 8

€15,000 20,000

81 € 4 × 7,500 €30,000 35,000 € (5,000)

*Fixed costs are not relevant in this case and should be omitted. The Avignon Co. order should be rejected because it is unprofitable in the short run with the present price and cost structure.

..

225


5-63 (10 – 15 min.) 1. Capacity is not sufficient to accept both orders, but there is enough capacity to accept either the Nordstrom or the Macy’s order. There is excess capacity for 150,000 shoes, but the two orders together would require production of 90,000 + 75,000 = 165,000 shoes. Nordstrom Macy’s Order Order Revenue Variable Costs: Direct Materials Direct Labor Var. Factory OH Packaging Contribution margin Unit sales Total contribution margin

$136.00

$130.00

49.00 22.00 14.00 3.50 $ 47.50 × 75,000 $3,562,500

49.00 22.00 14.00 2.00 $ 43.00 × 90,000 $3,870,000

Based on the analysis above, accepting the Macy’s order is the optimal decision, generating an additional contribution margin of $3,870,000 over not accepting a special order, and an additional $307,500 in contribution margin relative to the Nordstrom order. 2. Some considerations would involve cannibalization of existing sales by lower prices, and whether variable and fixed cost distinctions remain valid within the relevant range (especially as maximum capacity is approached). 5-64

(20 – 30 min.) For the solution to this Excel Application Exercise, follow the step-by-step instructions provided in the textbook chapter. The answers to the first two questions can be read directly from the spreadsheet.

1 & 2. Contribution margin Contribution margin percentage Total contribution margin

Plain Circular Saw $15 23% $600,000

Professional Circular Saw $25 25% $500,000

3.

The relevant costs are the variable costs, so we can focus on the contribution margin. The special order for the 40,000 plain circular saws provides $100,000 more contribution margin and therefore $100,000 more operating profit.

5-65

(60 min. or more)

Pricing tends to be more of an "art" than a "science" in small firms. In large firms, students will find a wide variety of tools and techniques but will most likely get interesting answers to all the recommended questions. ..

226


Perhaps the most significant factor that influences the process for establishing a pricing policy is company size. For many small companies, the process is simple. For example, one restaurant establishes prices using a formula of three times the cost of food used in each menu item. This markup is designed (hoped) to cover all the operating costs in the restaurant's value chain beyond food cost (direct material). Other important factors commonly mentioned include market conditions and the experience level of management. Small companies tend not to use target costing. Some form of cost plus pricing is most often used. When target costing is used and managers are asked to explain the target-costing process, it is often discovered that only some elements of a fully developed target costing process are used. Students may discover that different pricing policies are used for different product or service families in the same firm. This is particularly true for large companies that compete in many different markets. 5-66

(50 – 60 min.)

NOTE TO INSTRUCTOR. This solution is based on the web site as it was in late 2012. Be sure to examine the current web site before assigning this problem, as the information there may have changed. 1. In its 2011 Annual Report, the major component of Colgate’s strategy is investing in innovative new products with growth potential. The company supports this strategy by focusing on the value chain functions R & D and marketing. The company that places an emphasis on rapid product development needs relevant information regarding expected revenues and costs for proposed new products. In 2011, Colgate’s advertising spending was at a record high at 1.734 billion. This is generating healthy volume and strong market share gains worldwide. According to Ian Cook, CEO: “We are confident that Colgate is well positioned for long-term sustainable growth as we are financially strong, are market leaders in many of our core categories around the world and have the right strategies in place to succeed.” Colgate also needs to have reliable estimates of the impact of its advertising and promotion on sales. If a company uses an ABC system as discussed in chapter 4, the impact of increased sales from new products will be estimated via increased activity levels as well as higher variable costs and capacity utilization. This information is very relevant in Colgate’s planning process. 2. The importance of Colgate’s code of conduct can be measured several ways. Not only does the company provide a link to the actual code of conduct for public viewing, the following excerpt from the section “Living our Values” gives a good feel for the code of conduct priority at Colgate. Since 1987, our Code of Conduct has served as a guide for our daily business interactions, reflecting our standard for proper behavior and our corporate values. The Code clearly conveys to each of us that the manner ..

227


in which we achieve our business results matters just as much as achieving them. The Colgate Code of Conduct applies to all Colgate people, including Directors, Officers, and all employees of the Company and its subsidiaries around the globe. Vendors and Suppliers are also subject to these requirements as adherence to the Code is a condition for conducting business with Colgate. The Code of Conduct is regularly updated and reissued to ensure its comprehensiveness. On June 7, 2012, Colgate’s Board of Directors approved certain amendments to the Code of Conduct, including (i) enhancements to existing provisions regarding conflicts of interest (to address potential conflicts from internal interactions), protection of Colgate’s confidential information and use of information technology resources and (ii) additional provisions regarding data privacy and social media. The updated Code of Conduct also highlights Colgate’s values and leading with respect principles. Yet another excerpt from that section speaks to Colgate’s commitment to ethical values: Most importantly, each employee is responsible for demonstrating integrity and leadership by complying with the provisions of the Code of Conduct, Global Business Practices Guidelines, Company policies and all applicable laws. By fully including ethics and integrity in our ongoing business relationships and decision-making, we demonstrate a commitment to a culture that promotes the highest ethical standards. 3. As of late 2012, one of the new products is a Hill’s® Prescription Diet® y/d™ cat food, which can restore thyroid health safely and effectively in cats. Obviously this is a variation of an existing product. 4. The company displays its major product groups and major market regions using a heading page. In late 2012, eight laundry conditioners were listed. The information provided is sketchy with little differentiation made between the different conditioners. The site gives no guidance as to when to use a product. It basically provides a list and some advertising information on selected items. The site does not indicate which conditioner is best for a particular fabric. There is not enough information to select the best conditioner for a specific laundry situation. 5. The company’s financial strategy is to continuously improve gross margin percentage, reduce overhead (sales, general, and administrative expenses), and increase advertising. The income statement and other disclosures in the annual report provide sufficient data to evaluate the effectiveness of the company’s strategy.

..

228


By emphasizing high-profit-margin products, and implementing numerous cost reduction programs, the gross margin percentage has remained relatively stable from 59.1% in 2010 to 57.3% in 2011 (from the comparative consolidated statements of income). Selling, general, and administrative expenses (a good surrogate for overhead) have decreased slightly from 34.8% of net sales in 2010 to 34.4% of net sales in 2011, even with higher advertising costs. Advertising costs are included in selling, general, and administrative expenses. The amount of advertising costs is not given in the footnotes or the financial statement. Management’s discussion in the annual report graphically displays it at a record level of $1.734 billion in 2011. These results are the primary source supporting increased R & D and advertising for rapid new product development. When all components of Colgate’s strategy are taken together, the company’s overall profitability improved as measured by net profit, which has increased by 10% from $2,203,000,000 in 2010 to $2,431,000,000 in 2011.

..

229


CHAPTER 6 COVERAGE OF LEARNING OBJECTIVES

LEARNING OBJECTIVE LO1: Use a differential analysis to examine income effects across alternatives, and show that an opportunity cost analysis yields identical results. LO2: Decide whether to make or buy certain parts or products. LO3: Choose whether to add or delete a product line using relevant information. LO4: Compute the optimal product mix when production is constrained by a scarce resource. LO5: Decide whether a joint product should be processed beyond the split-off point. LO6: Decide whether to keep or replace equipment. LO7: Identify irrelevant and misspecified costs. LO8: Discuss how performance measures can affect decision making.

..

FUNDAMENTAL ASSIGNMENT MATERIAL B6

ADDITIONAL ASSIGNMENT MATERIAL 24, 27, 28, 29, 30, 31, 31, 42, 42, 44, 45, 46, 47, 47, 48, 49, 50, 56, 56, 61

A1, B1, B6

25, 32, 33, 34, 62, 62, 63, 63

B3, B6

9, 36, 36

A2, B2, B6

35, 51, 53

A3, B4, B6

37, 38, 54, 55

A4, B5, B6

40, 57, 59, 59

B6

18, 26, 39, 39, 41, 41, 52, 58, 64

B6, B6

43, 60

CASES, EXCEL, COLLAB. & INTERNET EXERCISES

65, 66, 67, 68, 70

69

71

230


CHAPTER 6 Relevant Information and Decision Making With a Focus on Operational Decisions 6-A1

(20 min)

1.

The key to this question is what will happen to the fixed overhead costs if production of the boxes is discontinued. Assume that all $46,800 of fixed costs will continue. Then, Vineyard Fruit will lose $16,800 by purchasing the boxes from Weyerhaeuser: Payment to Weyerhaeuser, 60,000 × $2.24 Costs saved, variable costs ($96,000 + $12,000 + $9,600) Additional costs

$134,400 117,600 $ 16,800

2.

Some subjective factors are:  Might Weyerhaeuser raise prices if Vineyard Fruit closed down its box-making facility?  Will sub-contracting the box production affect the quality of the boxes?  Is a timely supply of boxes assured, even if the number needed changes?  Does Vineyard Fruit sacrifice proprietary information when disclosing the box specifications to Weyerhaeuser?

3.

In this case the fixed costs are relevant. However, it is not the depreciation on the old equipment that is relevant. It is the cost of the new equipment. Annual cost savings by not producing the boxes now will be: Variable costs $117,600 Investment avoided (annualized), $375,000 ÷ 5 75,000 Total saved $192,600 The payment to Weyerhaeuser is $192,600 - $134,400 = $58,200 less than the savings, so Vineyard Fruit would be $58,200 better off subcontracting the production of the boxes.

..

231


6-A2 (10 min.) 1.

Contribution margins: Plain = $50 - $35 = $15 Professional = $100 - $60 = $40 Contribution margin ratios: Plain = $15 ÷ $50 = 30% Professional = $40 ÷ $100 = 40%

2.

Plain a. Units per hour b. Contribution margin per unit Contribution margin per hour Total contribution for 40,000 hours

4 $15 $60 $2,400,000

Professional 1 $40 $40 $1,600,000

3.

The plain circular saws are the best use of the scarce machine hours. For a given capacity, the criterion for maximizing profits is to obtain the greatest possible contribution to profit for each unit of the limiting or scarce factor. Moreover, fixed costs are irrelevant unless their total is affected by the choice of products.

6-A3

(15 min.) Table is in thousands of dollars.

1,2.

(a)

(b)

(a)-(b)

Sales Beyond Split-Off

Sales at Split-Off

Incremental Sales

(c) Separable Costs Beyond Split-Off

A 230 54 176 190 B 330 32 298 300 C 175 54 121 100 Increase in overall operating income from further processing of A, B, and C

(a)-(b)-(c) Incremental Gain or (Loss) (14) (2) 21 5

The incremental analysis indicates that Product C should be processed further, but Products A and B should be sold at split-off. The overall operating income would be $44,000, as follows: Sales: $54,000 + $32,000 + $175,000 Joint cost of goods sold Separable cost of goods sold Operating income

$261,000 $117,000 100,000

217,000 $ 44,000

Compare this with the present operating income of $28,000. That is, $230,000 + $330,000 + $175,000 - ($190,000 + $300,000 + $100,000 + $117,000) = $28,000. The extra $16,000 of operating income comes from eliminating the $16,000 loss resulting from processing Products A and B beyond the split-off point.

..

232


6-A4

(30-40 min.)

Problem 6-60 is an extension of this problem. The two problems make a good combination. 1.

Operating inflows for each year, old machine: $850,000 - ($740,000 + $69,000) Operating inflows for each year, new machine: $850,000 - ($740,000 + $17,000*)

$41,000 $93,000

* $69,000 - $52,000 Cash flow statements (in thousands of dollars): Keep Year 1 41

Receipts, inflows from operations Disbursements: Purchase of "old" equipment (108)* Purchase of "new" equipment: Total costs less proceeds from disposal of "old" equipment ($108,000-$35,000) -Net cash inflow (outflow) (67)

Three Years Years 2 & 3 Together 41 123

Year 1 93

Replace Three Years Years 2 & 3 Together 93 279

--

(108)

(108)

--

(108)

-41

-15

(73) (88)

-93

(73) 98

* Assumes that the outlay of $108,000 took place on January 2, 2013, or sometime during 2013. Some students will ignore this item, assuming correctly that it is irrelevant to the decision. However, note that a statement for the entire year was requested. The difference for three years taken together is $98,000 - $15,000 = $83,000. Note particularly that the $108,000 book value can be omitted from the comparison. Merely cross out the entire line; although the column totals will be affected, the net difference will still be $83,000.

..

233


2.

Income statements (in thousands of dollars): Keep

Sales Expenses: Other expenses Operating of machine Depreciation Total expenses Loss on disposal: Proceeds ("revenue") Book value ("expense") Loss Total charges Net income

Replace

Years 1, 2 & 3 850

Three Years Together 2,550

Year 1 850

Years 2&3 850

Three Years Together 2,550

740 69 36 845

2,220 207 108* 2,535

740 17 36 793

740 17 36 793

2,220 51 108 2,379

---845 5

---2,535 15

(35) 108 73 866 (16)

---793 57

(35) 108* 73 2,452 98

* As in part (1), the $108,000 book value can be omitted from the comparison without changing the $83,000 difference. This would mean dropping the depreciation item of $36,000 per year (a cumulative effect of $108,000) under the "keep" alternative, and dropping the book value item of $108,000 in the loss on disposal computation under the "buy" alternative. Difference for three years together, $98,000 - $15,000 = $83,000. Note the motivational factors here. A manager may be reluctant to replace simply because the large loss on disposal will severely harm the profit performance in Year 1. 3.

..

The net difference for the three years taken together would be unaffected because the item is a past cost. You can substitute any number for the original $108,000 figure for the old equipment without changing this answer.

234


For example, examine how the results would change in part (1) by inserting $800,000 where the $108,000 now appears for the old equipment (in thousands of dollars):

Receipts, inflows from operations Disbursements: Purchase of old equipment Purchase of new equipment: Gross price Disposal proceeds of "old" Net cash outflow

Keep: Three Years Together 123

Replace: Three Years Together 279

Difference 156

(800)

(800)

0

-( 677)

(108) 35 ( 73) ( 594)

(73) 83

In sum, this may be a horrible situation. The manager really blundered. But keeping the old equipment will compound the blunder to the cumulative tune of $83,000 over the next three years. 4.

Diplomatically, Slater should try to convey the following. All of us tend to indulge in the erroneous idea that we can soothe the wounded pride of a bad purchase decision by using the item instead of replacing it. The fallacy is believing that a current or future action can influence the long-run impact of a past outlay. All past costs are down the drain. Nothing can change what has already happened. The $108,000 has been spent. Subsequent accounting for the item is irrelevant. The schedules in parts (1) and (2) clearly show that we may completely ignore the $108,000 original outlay and still have a correct analysis. The important point is that the $108,000 is not an element of difference between alternatives and, therefore, may be safely ignored. The only relevant items are those expected future items that will differ between alternatives.

5.

The $108,000 purchase of the original equipment, the sales, and the other expenses are irrelevant because they are common to both alternatives. The relevant items are the following (in thousands of dollars): Three Years Together Keep Replace

Operating of machine (3 × $69; 3 × $17) Incremental cost of new machine: Total cost Less proceeds of old machine Incremental cost Total relevant costs Difference in favor of buying

..

$207

$ 51 $108 35

-$207

73 $124 $ 83

235


6-B1

(15-20 min.)

1.

Make Total

Purchase cost Direct material Direct labor Factory overhead, variable Factory overhead, fixed avoided Total relevant costs Difference in favor of making

Per Unit

€5,500,000 1,900,000 1,100,000

€27.50 9.50 5.50

900,000 €9,400,000 € 600,000

4.50 €47.00 € 3.00

Buy Total €10,000,000

Per Unit €50

€10,000,000

€50

The numerical difference in favor of making is €600,000 or €3.00 per unit. The relevant fixed costs are €900,000, not €3,000,000. 2. Make Rent revenue Obtaining of components Net relevant costs

-€(9,400,000) €(9,400,000)

Buy and Leave Capacity Idle -€(10,000,000) €(10,000,000)

Buy and Rent € 1,150,000 €(10,000,000) € (8,850,000)

The final column indicates that buying the components and renting the vacated capacity will yield the best results in this case. The favorable difference is €9,400,000 €8,850,000 = €550,000. 6-B2

(15 min.)

1.

If fixed manufacturing cost is applied to products at $1.00 per machine hour, it takes $.70 ÷ $1.00, or .70 of an hour to produce one unit of XY-7. Similarly, it takes $.20 ÷ $1.00 or .20 of an hour to produce BD-4.

2.

If there are 140,000 hours of capacity: XY-7: BD-4:

140,000 hours ÷ .70 = 200,000 units. 140,000 hours ÷ .20 = 700,000 units.

Total contribution margins show that BD-4 should be produced, generating $525,000 of contribution margin, which is $135,000 more than would be earned by XY-7. XY-7 BD-4

..

Per Unit $6.45 - ($2.70 + $1.80) = $1.95 $4.20 - ($1.70 + $1.75) = $ .75

Units 200,000 700,000

Total $390,000 $525,000

236


6-B3

(15-20 min.) All amounts are in thousands of British pounds.

The major lesson is that a product that shows an operating loss based on fully allocated costs may nevertheless be worth keeping. Why? Because it may produce a sufficiently high contribution to profit so that the firm would be better off with it than any other alternative. The emphasis should be on totals:

Sales Variable expenses Contribution margin Fixed expenses Operating income a

Existing Operations 6,000 4,090 1,910 1,100 810

Replace Magic Department With General Merchandise Electronic Products -600 + 250 = 5,650 -600 + 200 = 5,600 -390 + 175a = 3,875 -390 + 100 b = 3,800 -210 + 75 = 1,775 -210 + 100 = 1,800 -120 + 0 = 980 -120 + 30 = 1,010 - 90 + 75 = 795 - 90 + 70 = 790

(100% - 30%) × 250 (100% - 50%) × 200

b

The facts as stated indicate that the magic department should not be closed. First, the total operating income would drop. Second, fewer customers would come to the store, so sales in other departments may be affected adversely.

..

237


6-B4

(15 min.)

1.

Sales ($400 + $600 + $100) Costs: Raw materials Processing Total Profit

2.

Sales ($840 + $850 + $170) Costs: Joint costs Frozen dinner costs Salisbury steak costs Tanning costs Total costs Profit

$1,100 $700 100 800 $ 300 $1,860 $800 440 200 80 1,520 $ 340

Although it is more profitable to process all three products further than it is to sell them all at the split-off point, it is important to look at the economic benefit from further processing of each individual product. 3.

Steaks to frozen dinners: Additional revenue from processing further ($840 - $400) Additional cost for processing further Increase (decrease) in profit from processing further

$440 440 $ 0

Hamburger to Salisbury steaks: Additional revenue from processing further ($850 - $600) Additional cost for processing further Increase (decrease) in profit from processing further

$250 200 $ 50

Untanned hide to tanned hide: Additional revenue from processing further ($170 - $100) Additional cost for processing further Increase (decrease) in profit from processing further

$ 70 80 $ (10)

Only the hamburger dictates that it should be processed further, because it is the only product whose additional revenue for processing further exceeds the additional cost. You are indifferent about processing further steak to frozen dinners, as the incremental profit is 0.

..

238


4.

The resulting profit would be $350: Sales ($400 + $850 + $100) Costs: Joint costs Further processing of hamburger Total cost Profit

6-B5

$1,350 $800 200 1,000 $ 350

(15-20 min.)

1. Cash operating costs Old equipment, book value: Periodic write-off as depreciation or lump-sum write-off Disposal value New equipment, acquisition cost Total costs

Keep $51,000 20,400 $71,400

Three Years Together Replace Difference $33,000 $18,000 20,400* -3,600* 14,700** $64,500

3,600 - 14,700 $ 6,900

*In a formal income statement, these two items would be combined as "loss on disposal" of $20,400 - $3,600 = $16,800. **In a formal income statement, written off as straight-line depreciation of $14,700 ÷ 3 = $4,900 for each of three years. 2. Cash operating costs Disposal value of old equipment New equipment, acquisition cost Total relevant costs

Three Years Together Keep Replace Difference $51,000 $33,000 $18,000 -3,600 3,600 14,700 - 14,700 $51,000 $44,100 $ 6,900

This tabulation is clearer because it focuses on only those items that affect the decision. 3.

Benefits of the replacement alternative* Deduct initial net cash outlay required** Difference in favor of replacement * 3 × ($17,000 - $11,000) ** $14,700 - $3,600

$18,000 11,100 $ 6,900

Also, the new equipment is likely to be faster, thus saving operator time. The latter is important, but it is not quantified in this problem.

..

239


6-B6

(10 min.)

1.

The replacement alternative would be chosen because the county would have $6,900 more cash accumulated in three years.

2.

The keep alternative would be chosen because the higher overall costs of photocopying for the first year would be shown for the replacement alternative (under accrual accounting): First Year Keep Replace Cash operating costs $17,000 $ 11,000 Depreciation expense 6,800 4,900 Loss on disposal 16,800 Total costs $23,800 $32,700 Thus, the performance evaluation model might motivate the manager to make a decision that would be undesirable in the long run.

6-1 An opportunity cost does not entail a disbursement of cash at any future time, whereas an outlay cost does entail an additional disbursement sooner or later. 6-2 The $800 represents an opportunity cost. It is the amount forgone by rejecting an opportunity. It signifies that the value to the owner of keeping those strangers out of the summer house for that two-week period is at least $800. 6-3 Accountants do not ordinarily record opportunity costs in accounting records because those records are traditionally concerned with real transactions rather than possible transactions. It is impossible to record data on all lost opportunities. 6-4 A differential cost is any difference in total cost or revenue between two alternatives. A differential cost is an incremental cost when one of the alternatives contains all the costs of the other plus some additional costs. The additional costs are the incremental costs – which are also differential. 6-5 No. Incremental cost has a broader meaning. It is the addition to total costs by the adoption of some course of action. Another term, marginal cost, is used by economists to indicate the addition to costs from the manufacture of one additional unit. Of course, marginal cost is indeed the incremental cost of one unit. 6-6

The decline in costs would be called differential or incremental savings.

6-7 Not necessarily. Qualitative factors can favor either making or buying. Often factors such as product quality and assurance of delivery schedules favor making. However, sometimes establishing long-term relationships with suppliers is an important qualitative factor favoring the purchase of components. ..

240


6-8 The choice in many cases is not really whether to make or buy. Instead, the choice is how best to use available capacity. 6-9 Nonfinancial information may play an important role while deciding to add or delete products or departments. When deciding to delete a product or close a facility, there may be ethical considerations. For example, will the employees be shifted to other departments or will they be terminated? How will the customers, who have bought the product in the past, get the support services? What will be the impact on the community in which a discontinued operation is located? Though difficult, the nonfinancial impact of the above considerations must be assessed. A committed workforce and loyal customers are important assets for an organization. Negative impact on the employees, customers, or community may lead to serious financial problems for the organization in the long term. 6-10 Four examples of scarce factors are: (a) labor hours, (b) money (investment capital), (c) supervisory hours, and (d) computer hours. 6-11 Joint products are two or more manufactured products that (1) have relatively significant sales values and (2) are not separately identifiable as individual products until their split-off point. Examples of joint products include chemicals, lumber, flour, and meat. 6-12 The split-off point is where the individual products produced in a joint process become separately identifiable. Costs before the split-off point are irrelevant for decisions about the individual products. They affect the decision about whether to undertake the entire production process, but they do not influence decisions about what to do with the individual products. 6-13 Yes. Techniques for assigning joint-product costs to individual products are useful only for product costing, not for deciding on further processing after the split-off point. The product must be considered separately at that point apart from its joint cost. The proper basis of the decision on further processing is a comparison of incremental revenue versus incremental expense between the alternatives of selling at the split-off point and processing further. 6-14 No. Once inventory has been purchased, the price paid is a sunk cost. It is true that selling at a price less than $5,000 would produce a reported loss. However, a sale at any price above $0 is economically beneficial provided that the only alternative is to scrap the inventory. 6-15

No. Sunk costs are irrelevant to the replacement decision.

6-16 No. Past costs are not relevant because they cannot be affected by a decision. Although past costs are often indispensable for formulating predictions, past costs themselves are not the predictions that are the inputs to decision models. Clear thinking is enhanced by these distinctions.

..

241


6-17

Only b and c are relevant.

a.

Book value of old equipment is irrelevant to a replacement decision because it does not change under any alternative and cannot be realized. Disposal value of old equipment is relevant to a replacement decision because it can either be realized (by replacement) or forgone (by continued use). Cost of new equipment is relevant to a replacement decision because it can be incurred (by replacement) or avoided (by continued use).

b. c.

6-18 Costs can be mis-specified in two ways: (1) by including irrelevant costs such as unavoidable fixed costs, thereby increasing the unit cost, and (2) by comparing unit costs not computed on the same-volume basis. 6-19

Yes. The statement is correct in terms of total variable costs.

6-20

Two reasons why unit costs should be analyzed with care in decision making are:

1.

Most unit costs are stable only over a certain range of output, and care must be taken to see that allowances are made when alternatives are considered outside that range. Some unit costs are an allocation of fixed costs; thus when a higher volume of output is being considered, unit cost will decrease proportionately, and vice versa.

2.

Two other reasons are mentioned in the text: 1. 2.

Some unit costs are based on both relevant and irrelevant factors and should be broken down further before being considered. Unit costs must be reduced to the same base (denominator) before comparing or combining them.

6-21 Sales personnel sometimes neglect to point out that the unit costs are based on outputs far in excess of the volume of their prospective customer. 6-22 An inconsistency between a decision model and a performance evaluation model occurs when a decision about whether to replace a piece of equipment is based on the cash flow effects over the life of the equipment but a manager's performance evaluation is based on the first year's reported income. The loss on disposal of the equipment is irrelevant for decision purposes, but it affects the first year income, hence the performance evaluation. 6-23 The wide use of income statements to evaluate performance may overly influence managers to maximize short-run performance that may hurt long-run performance. They may pass up profitable opportunities to replace equipment because of the large loss on disposal shown on the first year’s income statement.

..

242


6-24 Yes, this statement is generally correct. Accountants record transactions. But opportunity cost is the cost of transactions that do not occur (or have not occurred yet). It is the cost of opportunities forgone. Managers usually have much better information about forgone opportunities than do accountants. 6-25 Deciding whether to outsource payroll functions requires estimates of the cost of designing, maintaining, and using a payroll system internally compared to the cost of a contract with an outside supplier. To operate an internal payroll system requires hiring personnel with the needed expertise in both legal/governmental issues affecting payroll and information processing to implement a system. Small companies often find it less costly to outsource payroll to a company that has broad expertise in these areas. 6-26 Whenever total costs are unitized by dividing by total units and the resulting unit costs are then used to predict new total costs based on a different level of production, errors are being made if any of the costs are fixed. If the new production level is higher, predicted total costs are overestimated. If the new production level is lower, predicted total costs are underestimated. Never unitize fixed costs if the resulting unit cost will be used for planning purposes! Consider the following simple example: Total Units Unit Cost

Fixed Cost $100 ÷10 $ 10

Variable Cost $100 ÷10 $ 10

Total $200 ÷10 $ 20

If a new planned number of units is 20, what will be the new, predicted total cost? The correct cost function and cost prediction is Total Cost = $100 + $10 × Number of units = $100 + $10 × 20 =$300 The correct cost function is based on the two amounts that are constant within the relevant range – the total fixed cost and the unit variable cost. The incorrect unitized cost function and incorrect and overestimated prediction is Total Cost = $20 × Number of units = $20 × 20 = $400 It is easy to see that the error comes from treating fixed costs as if they were variable.

..

243


6-27 The amount paid for inventory is a sunk cost. Once a company has the inventory, it cannot change what it paid for it. Thus, the only relevant issue is what can be done with the inventory. If there is a choice of selling the inventory for less than what the company paid for it or not selling it at all, it is certainly better to get something rather than nothing for it. 6-28

(10-15 min.)

1. Operating revenues Operating expenses Income effects per year

Independent Practice

Employee

Difference

$410,000 290,000 $120,000

$85,000 -$85,000

$325,000 290,000 $ 35,000

Revenues Expenses: Outlay costs Opportunity cost of employee compensation Income effects per year

Choose Independent Practice $410,000 $290,000 85,000

375,000 $ 35,000

Each tabulation produces the key difference of $35,000. As a general rule, we favor using the first tabulation when feasible. It offers a straightforward presentation of inflows and outflows under sharply stated alternatives. 2. Revenue Expenses: Outlay costs Opportunity cost of accounting practice Income effects per year

Choice as Employee $ 85,000 $

0 120,000

120,000 $ (35,000)

If the employee alternative is selected, the key difference in favor of becoming a sole practitioner is again $35,000. Adessa is sacrificing $35,000 to avoid the risks of an independent practice.

..

244


6-29

(10-15 min.) Alternatives Under Consideration (1) (2) Sell, Rent, and Hold Invest in Bonds Present Home

Revenue Less: Outlay cost Income effects per year *3% × $490,000 ** $750 × 12

$14,700* 18,000 $ (3,300)

$

9,000** $(9,000)

(1) - (2) Difference $14,700 9,000 $ 5,700

Advantage of selling the home is $9,000 – 3,300 = $5,700. Obviously, if rent is higher, the advantage decreases. The above analysis does not contain explicit opportunity costs. If opportunity costs were a part of the analysis, the following presentation applies (whereby the interest on investment in bonds is not listed as a separate alternative but is regarded as a forgone alternative):

Opportunity cost Outlay cost Income effects per year

Alternative Chosen: Hold Present Home $(14,700) 9,000 $ (5,700)

As before, the advantage of selling the home and renting is $5,700. The opportunity cost of home ownership is 3% × 490,000 = $14,700. 6-30 (15-20 min.) Opportunity cost is the maximum available contribution to profit forgone by using limited resources for a particular purpose. In this case, the opportunity cost of the machine when analyzing the alternative to produce 12-oz. bottles of Juice Cocktails is $90,000, the larger of the $90,000 contribution margin from additional sales of the 100% Juices or the $75,000 proceeds from the sale of the machine. The $160,000 historical cost of the machine is a past cost and thus irrelevant.

..

245


6-31 (15-20 min.) The first tabulation is probably easier to understand, but the choice of tabulation is a matter of taste:

Revenues Expenses Income effects per year

(a) Expand MRI Unit BDT2,250,000 1,850,000 BDT 400,000

(b) Expand Dental Dept. BDT1,150,000 950,000 BDT 200,000

(c) Rent to Pharmacy Chain BDT110,000 0 BDT110,000

Treating the pharmacy chain as the forgone (rejected) alternative, the tabulation is: (a) (b) Expand Expand MRI Unit Dental Dept. Revenue BDT2,250,000 BDT1,150,000 Expenses: Outlay costs BDT1,850,000 BDT950,000 Opportunity cost, rent forgone 110,000 1,960,000 110,000 1060,000 Income effects per year BDT 290,000 BDT 90,000 The numbers favor the expansion of the MRI unit, which will generate a contribution to the hospital income that is BDT200,000 greater than that of the dental department. The numbers have been analyzed correctly under both tabulations. Both answer the key query: What difference does it make? As a general rule, we prefer using the first tabulation. It is a straightforward presentation. 6-32

(15 min.)

1.

It is easiest to analyze total costs, not unit costs.

Direct materials Avoidable overhead costs: Indirect labor Supplies Allocated occupancy cost Purchase cost Total relevant costs

Make $400,000

Purchase

30,000 20,000 0 $450,000

$420,000 $420,000

The difference in favor of purchasing is $450,000 - $420,000 = $30,000. The manager ignored the avoidable fixed costs in his analysis.

..

246


2.

Because the quantitative difference is small, qualitative factors may dominate the decision. Companies using a just-in-time system need assurance of both quality and timeliness of supplies of materials, parts, and components. A small, local company may not be reliable enough for Bose. In essence, Bose may be willing to "invest" $30,000, the quantitative advantage of purchasing, in order to have more control over the supply of the components. The division manager may have made the right decision for the wrong reason. He incorrectly ignored avoidable fixed costs, leading to a mistaken belief that making the components was less costly by $.20 per unit or $20,000 in total. The $50,000 of avoidable fixed costs makes the purchase option less costly by $30,000. If the manager's decision is to make the component, it should be because forgoing profits of $30,000 has a long-run qualitative benefit of more than $30,000, not because the bid is greater than the variable cost.

6-33

(20-25 min.) Nantucket Nectars should make the bottles. Make

Purchase cost Direct materials Direct labor Variable overhead Avoidable fixed overhead Total relevant costs Difference in favor of making 6-34

Per Bottle

$80,000 30,000 60,000

$.080 .030 .060

60,000 $230,000 $ 20,000

.060 $.230 $.020

Per Total Bottle $250,000 $.250

$250,000

$.250

(15-20 min.)

Make Contribution from other activities Rent revenue Relevant cost of bottles ..

Total

Buy

Buy and Leave Facilities Idle

Buy and Use Facilities for Other Activities

Buy and Rent Out Facilities

$ 75 $(230)

$(250)

(250)

$ 55 (250) 247


Net relevant costs

$(230)

$(250)

$(175)

$(195)

To maximize profits, Nantucket Nectars should buy the bottles and use the facilities for other activities. 6-35

(20 min.)

1.

These warehouse stores attempt to maximize profits by cutting prices and increasing inventory turnover. Since profit is the product of contribution margin and unit sales, it can be affected by changing either. Total profit can be increased if the added inventory turnover brought about by a lowering of price brings in more contribution margin than was lost by the price cut. They also try to minimize fixed costs by limiting their investment in buildings and equipment. Characteristics: (a) choose product lines and sizes that move quickly and avoid stocking slow-moving items and sizes; (b) rely heavily on self service; (c) attempt to cut costs by providing fewer services, and (d) build low-cost buildings in a place where property costs are not too high.

2.

Such a criterion by itself gives no indication what total contribution margin (TCM) can be expected. Inventory turnover or sales volume must be used also. The total contribution margin is determined by TCM =

Unit contribution margin × Total sales in units

If inventory turnover can be assumed to be fairly constant among items, then such a figure as a 20% average target gross profit might be meaningful.

..

248


6-36

(10-15 min.)

1.

The key is to focus on lost revenues and avoidable costs: Revenues, @₹15,000 per student for 60 students Avoidable costs*: Teacher remuneration Stationery Decrease in operating income

₹ 900,000 ₹ 600,000 50,000

650,000 ₹ 250,000

*In addition to the avoidable costs shown, there might be some savings in depreciation (less wear and tear on equipment) and overhead. Unless these savings are more than the ₹250,000 decrease in operating income, the school will be worse off financially without the coaching program. 2.

Among the qualitative factors to consider are that the coaching program might attract students to the regular program, it provides additional compensation to teachers, and there is a social need for such programs.

6-37

(10 min.)

Product M should not have been processed further. The only valid approach is to concentrate on the separable costs and revenues beyond split-off:

Revenues, 2,500,000 gallons @30¢ & 36¢ Separable costs beyond split-off Income effects for April

Sell at Split-off as M

Process Further as Super M

Difference

$750,000 -$750,000

$900,000 165,000 $735,000

$150,000 165,000 $ (15,000)

The joint costs do not differ between alternatives and are irrelevant to the question of whether to sell or process further. The next table (not required) confirms the results (in thousands): Alternative 1 Alternative 2 Super Differential L M Total L M Total Effects Revenues $1,000 $750 $1,750 $1,000 $900 $1,900 $ 150 Joint costs $1,600 $1,600 --Separable costs --165 165 165 Total costs $1,600 $1,765 $ 165 Income effects $ 150 $ 135 $(15)

..

249


6-38

(10 min.)

1.

Answer (a): $6,000 ÷ 6,000 = $1.00

2.

Answer (a): Product C is the only product that produces an incremental profit ($35,000 $28,000) - $6,000 = $1,000.

6-39

(5-10 min.)

1.

The only relevant item is the ₤1,100 to be received for the diaries. No additional costs will be incurred. Therefore, profit will be ₤1,100 higher if the offer is accepted than if it is rejected.

2.

The amount paid for the diaries is irrelevant. Even if ₤1 million had been paid for them, the added profit from selling them for ₤1,100 is ₤1,100. The ₤5,720 paid is a past cost, a sunk cost, one that should not affect the decision.

6-40

(15-20 min.)

1.

The difference in total costs over the five years is $2,000 in favor of replacing, computed as follows:

Cash operating costs Old machine (book value): Periodic write-off as depreciation or Lump-sum write-off Disposal value New machine: Acquisition cost Total costs

Keep $26,500 7,000 ---$33,500

Five Years Together Replace Difference $13,500 $13,000 --7,000 -2,500 13,500 $31,500

2,500 -13,500 $ 2,000

2.

The loss on disposal of the old machine combines the lump-sum write-off (an irrelevant item) with the disposal value (a relevant item), $7,000 - $2,500 = $4,500 loss on disposal. Because of the inclusion of an irrelevant item, this amount does not affect the computation in requirement 1. It is best to keep the lump-sum write-off and the disposal value separate, as is done in the table in requirement 1.

..

250


6-41

(10 min.)

1.

Variable cost Fixed cost Total cost

₤ 135,000 240,000 ₤375,000

Cost per unit, ₤375,000  15,000

₤ 25.00

Variable cost Fixed cost Total cost

₤225,000 240,000 ₤465,000

Cost per unit, ₤465,000  25,000

₤ 18.60

2.

3.

The two unit costs are equally accurate (or, more appropriately, equally inaccurate). Unit costs that include unitized fixed costs are always suspect. A unit cost that includes fixed costs will be accurate at only one volume; using it at any other volume will be misleading.

6-42

(10 min.)

The original investment is the “cash equivalent” cost. “Excess” trade-in allowances, such as the €8,000 in this instance, are really reductions in the “list price.” The €1,450 sales tax is added to the original cost. The problem is silent regarding how the sales tax is computed. The original investment is: List price Less: price allowance, €8,000 - €4,500 Cash equivalent cost before sales tax Sales tax Cash equivalent cost

€45,000 3,500 €41,500 1,450 €42,950

The annual cash operating costs are irrelevant. Another way of computing the €42,950 is: Cash payment (€45,000- €8,000 + €1,450) Opportunity cost of truck traded in Total cost

..

€38,450 €4,500 €42,950

251


6-43

(10 min.)

The $9 million is gone. It is irrelevant for decision purposes. The relevant comparison is whether to invest $5 million in the division or to invest it elsewhere:

Investment required Income generated

Sell Division $5 million ?

Hold Division $5 million $500,000 yearly*

*This assumes that the division has truly "turned around" and will now make a net profit of $500,000 per year for the foreseeable future. The $5 million is relevant because Lake Forest is forgoing the opportunity to invest it elsewhere for some return. If projects or divisions of comparable risk can be expected to generate more than $500,000 yearly, the division should be sold. 6-44

(10-15 min.)

The purpose of this problem is to sharpen the student's concept of "opportunity cost." Daily fees are $225 × 7 hours, or $1,575. 1.

The difference in annual income is $434,700 - $398,475 = $36,225:

Work, $1,575 × 6 days × 46 weeks Don't work on every other Saturday: $1,575 × 5 days × 23 weeks $1,575 × 6 days × 23 weeks Totals

(a) Work $434,700

$434,700

(b) Don't Work $181,125 217,350 $398,475

2.

The calculation in (1) seems awkward and unnecessary. The opportunity cost is the maximum amount forgone by not working on every other Saturday, which is $1,575 × 1 day × 23 weeks, or $36,225. This is really the key number because it answers the crucial question, "What difference does it make?" Opportunity cost is defined as the maximum available contribution to profit forgone by using limited resources for a particular purpose.

3.

If she has already decided to take the day off, her opportunity cost is zero because in any case she would not see patients. Note that opportunity cost is a "situation-specific" concept. If one of the possible alternatives is not even allowed into the feasible set by the decision maker, its financial effects are irrelevant. On the other hand, if she decided to repair her car instead of keeping the appointments with patients on a working Saturday, her opportunity cost for the day would be $1,575; for half a day, $787.50.

..

252


6-45 1.

(15-25 min.) With American Airlines Personnel Contribution margin for October 20: $150 × 50 $ 70 × 50

Without American Airlines Personnel $7,500

$3,500

Opportunity cost is a slippery term, so we are reluctant to be overly rigid about its definition during classroom sessions. The strict definition would be that the opportunity cost is $7,500 --the maximum profit forgone by rejecting the best forsaken alternative. Nevertheless, some students will insist that the $7,500 - $3,500 = $4,000 difference between the alternatives is the opportunity cost. On December 28, the opportunity cost would be 10 × $100 = $1,000. 2.

The simplest approach is: Let X = % of occupancy Then $110 × X = $70 X = $70 ÷ $110 = 63.636% A longer approach follows. To be indifferent, Marriott would have to generate the same rent as the American Airlines contract which is $70 × 50 rooms × 365 days = $1,277,500. Let Y $110 × Y × 365 $40,150 × Y Y

= Number of rooms per day @ $110 = $1,277,500 = $1,277,500 = 31.82 rooms per day

Percentage of occupancy of the 50 rooms = 31.82 ÷ 50 = .63636 = 63.636% To check the answer: $110 × .63636 × 50 × 365 = 1,277,493 ≈ $1,277,500

..

253


6-46

(10-15 min.)

1.

Contribution margin from airlines: ($70 - $10) × 50 × 365 = $1,095,000 General contribution margin: ($110 - $10) × 50 × 365 × .62 = $1,131,500 Marriott should reject the contract.

2.

Let X = occupancy rate ($110 - $10) × 50 × 365 × X = $1,095,000 $1,825,000 × X = $1,095,000 X = .60 or 60% occupancy rate

6-47

(10-20 min.)

1. Total Purchase cost Direct material Direct labor Variable factory overhead Fixed factory overhead that can be avoided by not making Total relevant costs Difference in favor of buying

Make Per Unit

$ 1,170,000 1,430,000 1,300,000

$9 11 10

260,000 $4,160,000 $ 520,000

2 $32 $4

Buy Total Per Unit $3,640,000 $28

$3,640,000

2. Make Rent revenue Contribution from other products Obtaining of parts Net relevant costs

$

-

Buy and Leave Facilities Idle $ -

(4,160,000) $(4,160,000)

(3,640,000) $(3,640,000)

Buy and Use Facilities for Gears $ -

Buy and Lease $ 50,000 -

(3,640,000) $ (3,590,000)

$28

30,000 (3,640,000) $ (3,610,000)

The analysis indicates that buying the parts and leasing out the facilities is the alternative that should yield the best results in this instance. The advantage over just buying the parts is $3,640,000 – $3,590,000 = $50,000.

..

254


6-48

(35-50 min.)

Note: Requirement 2 of this problem usually gives trouble to students; because Requirement 2 takes considerable class time for a clear explanation, you may prefer to assign Requirement 1 only. 1.

There are several ways to approach this problem. The easiest is probably to concentrate on the difference in the total contribution margin. The total fixed costs of $780,000, before considering the increase in advertising, will be unaffected and may be ignored. Production and sales will decline by 10%, from 60,000 to 54,000 units:

Sales at $90 and $98, respectively Variable costs at $70* Contribution margin

60,000 Units $5,400,000 4,200,000 $1,200,000

54,000 Units $5,292,000 3,780,000 $1,512,000

Difference $312,000

*$35 + $12 + $8 + $15 Advertising may be increased by $312,000 without affecting the current operating income level of $420,000 (contribution margin of $1,200,000 minus fixed expenses of $780,000). 2.

If the total fixed costs do not change, the company will need a total contribution margin of $1,200,000 from the two products together. How many units of the new product can be sold? The clue to the production capacity of the plant is in how fixed factory overhead was unitized: $300,000 ÷ $6 per unit = 50,000 units of expected sales. New product budget @ 50,000 Units: Sales at $40 Variable costs at $30* Contribution margin, new product *Direct material Direct labor Variable factory overhead Variable selling expense, 10% × $40 Total variable costs per unit

..

$2,000,000 1,500,000 $ 500,000 $ 6 12 8 4 $30

255


Therefore, the needed contribution margin on the old product is $1,200,000 - $500,000, or $700,000. Sales, 60,000 units at $90 Contribution margin needed Total variable costs that can be sustained Variable selling costs at $9* Maximum that may be paid to the supplier

$5,400,000 700,000 $4,700,000 540,000 $4,160,000

*60% ×$15 = $9/unit or 60% ($15 × 60,000) = $540,000 Maximum unit purchase price, $4,160,000 ÷ 60,000 = $69.33. If students do not accept the above analysis, the following proof may be helpful (in thousands):

Sales Variable costs Contribution margin Fixed manufacturing costs Fixed selling costs Total fixed costs Operating income

Old

Difference

$5,400 4,200 $1,200 300 480 $ 780 $ 420

$2,000 2,000 $ $ $ -

New Product 1 Product 2 $5,400 4,700* $ 700 380** $ 380 $ 320

$2,000 1,500 $ 500 300 100** $ 400 $ 100

*An alternate approach to this whole solution is to use the above format and solve toward the unknown purchases figure. The $4,700,000 is the maximum allowable variable cost. Because $540,000 of the $4,700,000 represents selling expense, the remainder, $4,160,000 must be the maximum amount paid to the supplier. **This allocation uses the $2.00 unit cost figure for the new product and assigns the remaining fixed costs to the old product. Note the allocation of the total fixed selling costs is irrelevant because total fixed costs are unaffected by allocation methods or by how such costs are assigned to products.

..

256


6-49

(15-25 min.)

1.

Alternative Without With Contract Contract Contribution margin: (200 rooms × 365 days) × ($86 - $12) ×.85 (200 - 40) × 365 × ($86-$12) × .95 40× 365 × ($50 - $12) Total contribution margin Difference in favor of contract

2.

$4,591,700 $4,591,700

$4,105,520 554,800 $4,660,320

$68,620

Let X = contribution margin per room (40 × 365 × X) + $4,105,520 = $4,591,700 14,600 × X = $486,180 X = $33.30 Add back variable cost: $33.30 + $12.00 = $45.30 Note how this room rate is the "point of indifference." The manager has $50.00 - $45.30 = $4.70 of excess contribution margin to bargain on contract rates.

..

257


6-50

(10-20 min.)

The basic message here is that airlines can maintain the same revenue per mile even in the face of switching by some passengers to lower fares. 1. Revenue, 75 @ $.12 Revenue 72 @ $.12 6 @ $.072 Total per airplane mile

Without Discount $9.00

$9.00

With Discount $8.64 .43 $9.07

Note that a minor (4%) gain in passengers will be beneficial. Note, too, that airlines have negligible variable costs of adding a few passengers in otherwise empty seats. Some instructors may want to use the language of "opportunity costs" here, but such language is not really necessary and may be confusing. For example, some observers would say that the three passengers who switch cause an opportunity cost of 3 × $.12 or $.36 that is more than offset by the added revenue of 6 × $.072 or $.43. 2.

Let X = number of passengers who switch Revenue with discount = Revenue without discount 50 × .60 × $.12 = X × $.12 50 × $.072 = $.12 × X $3.60 = $.12 × X X = $3.60 ÷ $.12 = 30 passengers Check: Revenue, 75 @ $.12 Revenue: (75 - 30) @ $.12 50 @ $.072 Total per airplane mile

Without Discount $9.00

$9.00

With Discount $5.40 3.60 $9.00

Therefore, if more than 20 of the 50 discount passengers are "new," that is, they would not have flown without the discount, there is more revenue with the discount plan. The indifference point results in 95 passengers, 50 paying the discount fare and 45 paying full fare.

..

258


6-51

(15-20 min.)

1. Items that can be displayed in 8,000 square feet Contribution margin per item Contribution margin per turnover of inventory Relative number of turnovers for a given time period Total contribution margin for a given time period

Designer 300 $120 $36,000 2 $72,000

Moderately Priced 400 $65 $26,000 3 $78,000

Students should recognize that square feet of floor space is the limiting or scarce factor. Note that the contribution margin percentage and the contribution margin per item are greater for the designer items. Nevertheless, the moderately priced items will generate a larger contribution margin in total. Why? Because more moderately priced items are sold in any given period of time. The analysis above implies sales of 300 × 2 = 600 designer items versus 400 × 3 = 1,200 moderately priced items. The designer items should be dropped. 2.

The solution in requirement 1 assumes that moderately priced items can outsell designer items 2 to 1 and that the store will be 100% full of such items. Interdependencies between the items are ignored. If these factors do not hold, some combination of the two items may be preferable. Additional considerations include the investment in inventories, the number of sales personnel, the skills and training of sales personnel, and the degree of substitutability between the types of items. This problem could also be addressed on a unit basis. Suppose one designer item is displayed and sold in a given time period. How many moderately priced items could be sold in the same period? First, compute how many moderately priced items would be displayed: Moderate priced items displayed = 4/3 × designer items displayed = 4/3 × 1 = 1 1/3 For each item displayed, 1 1/2 moderately priced items would be sold in the same time period that 1 designer item is sold. Why? Because turnover of designer items is 2/3 that of moderately priced items, which implies that turnover of moderately priced items is 1 1/2 times that of designer items. Therefore, Moderate priced items sold = 1 1/2 × 1 1/3 × designer items sold = 2 × designer items sold Gulf Coast Fashions can use a given amount of space to sell either 1 designer item or 2 moderately priced items. Contribution margins are: Designer items 1 × $120 = $120

Moderately priced items 2 × $65 = $130

The contribution is greater from selling 2 moderately priced items than from selling 1 designer item. ..

259


6-52

(15 min.)

The standard line should be produced. The major lesson here is that gross profit per unit of product is not necessarily indicative of the relative profitability of products. In this case the limiting factor (scarce resource) is production capacity. The most desirable product is the one that maximizes the contribution to profit for the given production capacity. In this case, the standard product will yield a $39 contribution per hour of machine time, while the premium product will yield $30: Per Unit Standard Premium Selling price $60 $70 Variable costs 21 25 Contribution margin per unit of product $39 $45 Divide by machine time per unit of product ÷1 ÷1.5* Contribution margin per hour of machine time $39 $30 Comparisons of gross profit percentages do not help in these instances, because they are not dependent on the scarce resource, machine time. (Of course, the rate of return on investment may be affected by different required amounts of assets, but that complication is not introduced here.) * $21 ÷ $14 = 1.5 hours of machine time required per unit of premium product. This is the key to the solution because it means that, if the full productive capacity is allocated to one of the products, the company could produce fewer premium products than standard products. Many students are not comfortable with this idea until an example is explained. Assume total fixed overhead of $700,000 and total machine hours of 50,000. The fixed overhead rate would be $14 per hour. (This is also $14 per standard unit.) But because $21 is charged per premium unit, the hours of machine time must be $21 ÷ $14 = 1.5 hours per unit. Therefore, only 33,333 units of the premium product could be produced: 50,000 ÷ 1.5 = 33,333. [Proof: at $21 fixed overhead each, total fixed overhead is $21 × 33,333 = $700,000 (rounded).]

..

260


6-53

(30-50 min.)

This might be assigned at the end of this chapter as a review of chapters 5 and 6. This problem is more challenging than nearly all of the others in this chapter. Accordingly, this solution is more elaborate than is really necessary to answer the question. 1.

The total amount of fixed overhead is common to all alternatives. Therefore, it is irrelevant to this analysis. The scarce resource is hours of capacity. The objective is to maximize the contribution per hour:

Revenue per unit Variable cost per unit Contribution per unit Contribution per hour Hours available Total contribution

Subcomponents $2.20 1.40 $ .80 $ 48.00* × 600,000 $28,800,000

Plug-in Assemblies $5.30 3.30 $2.00 $ 40.00** × 600,000 $24,000,000

Difference $-1.20 $ 8.00 $4,800,000

* $ .80 × 60 units per hour = $48.00 ** $2.00 × 20 units per hour = $40.00 Plug-in assemblies should be dropped because it is diverting the limited resource from a more profitable use. Note that the sales manager is incorrect. These decisions should not be reached by "all-costs" allocations and consequent computations of net profits or losses on units of product. Each plug-in assembly is making $2.00 contribution to profit and to the recovery of fixed costs, but it takes three times as long to produce a plug-in assembly. 2.

The lowest price must yield a contribution of $28,800,000. The contribution per unit would be $28,800,000 divided by the number of units produced in one year, or: $28,800,000 ÷ (600,000 hours × 20 unit per hour) = $28,800,000 ÷ 12,000,000 units = $2.40 per unit Because the contribution is currently $2.00 per unit at a selling price of $5.30, the minimum acceptable price must be $5.70 in order to provide a unit contribution of $2.40.

..

261


To double check, consider the following: 100% of Capacity To Subcomponents To Plug-in Assemblies Sales in units Sales at $2.20 and $5.70 Variable costs at $1.40 and $3.30 Contribution margin Fixed costs* Operating income

36,000,000 $79,200,000 50,400,000 $28,800,000 21,600,000 $ 7,200,000

12,000,000 $68,400,000 39,600,000 $28,800,000 21,600,000 $ 7,200,000

* 36,000,000 × Unit fixed overhead rate of $.60, and 12,000,000 × Unit fixed overhead rate of ($1.20 + the $.60 transferred-in), respectively. 3.

Note that this increase in variable cost per hour is common to both alternatives. That is, the variable processing cost would rise by $14.40 per hour: Variable overhead = 40% of old fixed overhead = .4 × $21,600,000 = $8,640,000 Variable overhead rate per hour = $8,640,000 ÷ 600,000 = $14.40 The contribution per hour is therefore reduced from $48 to $33.60 for subcomponents and from $40 to $25.60 for plug-in assemblies. Note that the crucial difference per hour is still $8.00. The critical question in relevant cost analysis is: what difference does it make? Incidentally, many individuals often jump to the conclusion that relevant cost analysis is simple: variable costs are always relevant, and fixed costs are irrelevant. This is an example where the variable overhead cost is irrelevant. (For that matter, in this case, the labor cost, another variable cost, is also irrelevant.) Irrelevant costs can be included in the analysis. If they are analyzed correctly, they will not make any difference between alternatives. However, if analyzed incorrectly, they will provide misleading information.

..

262


In short, the answer here is the same as the answers to (1) and (2). The lowest acceptable price is still $5.70. To prove this, use the same format as in (2): 100% of Capacity To SubcomTo Plug-in ponents Assemblies Sales in units 36,000,000 12,000,000 Sales at $2.20 and $5.70 $79,200,000 $68,400,000 Variable costs* at $1.64 and $4.02 59,040,000 48,240,000 Contribution margin $20,160,000 $20,160,000 Fixed costs** 12,960,000 12,960,000 Operating income $ 7,200,000 $ 7,200,000 *

$1.40 + ($14.40 ÷ 60 units) = $1.64, and $3.30 + ($14.40 ÷ 20 units) = $4.02 ** .6 × $21,600,000 = $12,960,000 Finally, note that the fixed costs could be ignored completely in all of the above requirements. If you want to include them, you may -- but, if the fixed costs are analyzed correctly, they won't affect the decisions. In part (2), $21.6 million of fixed costs would be deducted no matter what product mix is chosen; in part (3), $12.96 million of fixed costs would be deducted under any alternative.

..

263


6-54 1.

(25-40 min.) Sets result in a 15% sales increase: 1,000 × 1.15 = 1,150 dresses. Total Number of

Complete sets Dress and cape Dress and handbag Dress only Total units if accessories are introduced Unit sales if accessories are not introduced Incremental sales Incremental contribution margin per unit Total incremental contribution margin

Percent of Total 72% 10 12 6

Dresses 828 115 138 69

Capes 828 115

100%

1,150

943

966

1,000 150

--943

--966

500

€75,000

Incremental contribution margin Additional costs Additional material cost (150 × €360) Cutting cost on additional dresses (150 × €105) Additional cutting cost* (1,081 × €34) Lost remnant sales** (931 × €24) Incremental profit

Handbags 828

Total

138

50

€47,150

10 €9,660 € 131,810 € 131,810

€54,000 15,750 36,754 22,344 128,848 € 2,962

* Only 1,150 – 69 = 1,081 dresses need extra cutting costs. ** Remnant sales on 69 dresses will continue, so net loss of remnant sales will be (1,000 – 69) × $24. 2. Nonquantitative factors that could influence management in its decision to manufacture matching capes and handbags include: • accuracy of forecasted increase in dress sales. • accuracy of forecasted product mix. • company image from dress manufacturer only to a more extensive supplier of women's apparel. • competition from other manufacturers of women's apparel. • whether there is adequate capacity (labor, facilities, storage, etc.).

..

264


6-55 1.

(15 min.) Incremental Revenue, $10 × 4,000 Incremental Cost Incremental Profit

$40,000 30,000 $10,000

Therefore, Western should process further. 2. a. The joint costs can increase by any amount, since they are sunk and irrelevant. Western should always choose to process further. b. Total Revenue, $160,000 + $120,000 Total Costs: Separable costs: $80,000 + $50,000 + $30,000 Joint costs Net Profit

$280,000 160,000 60,000 $ 60,000

Therefore, joint costs can increase by $60,000 before it is better to not be in the business of processing tobacco leaves.

..

265


6-56

(15-30 min.)

1.

Sales: (20,000 × 12) × ¥1,995 Less expenses: Direct materials ¥ 108,360,000 Direct labor 23,940,000 Overhead (Variable & Fixed) 54,180,000 Selling expenses 99,540,000 (20,000 × 12) × ¥1,191.75 Operating income [(20,000 × 12) × (¥1,995 - ¥1,191.75)]

2.

Sales: (20,000 × 12 × 112%) × ¥1,680 = 268,800 × ¥1,680 Less variable expenses: [(¥451.5 + ¥99.75 + ¥115.50 + ¥304.5) × (268,800)] Contribution margin Less: fixed expenses, (¥110.25 + ¥110.25) × (240,000) Operating income

¥478,800,000

286,020,000 ¥192,780,000 ¥451,584,000 261,072,000 ¥190,512,000 52,920,000 ¥137,592,000

(A common student error is to use 268,800 units at old fixed costs per unit.) 3.

Cost to obtain order (¥856,800 ÷ 6,800) Direct materials Direct labor Variable overhead Variable selling expenses: 70% of ¥304.50 Minimum price for special order

4.

The variable selling expenses only

..

¥126.00 451.50 99.75 115.50 213.15 ¥1,006.25 ¥304.50

266


6-57

(15-20 min.)

1.

The salesman's analysis is faulty because it includes depreciation on the old equipment, which is irrelevant. Moreover, both the total and unit costs are based on an annual volume of 40,000 units, which may not necessarily be accurate. A correct analysis would compare the old machine cost of ($6.50 variable cost × expected volume) with the new machine cost of ($4 variable cost × expected volume + $60,000 fixed cost).

2.

New Machine 20,000 $ 80,000 60,000 $140,000 $7.00

Units Variable costs Straight-line depreciation Total cost Unit cost 3.

6-58

Old Machine 20,000 $130,000 $130,000 $6.50

Let X = Number of units $60,000 + $4X = $6.5X X = 24,000 units (15 min.)

Management misjudged the life of the old freight cars. This may raise questions about the accuracy of the estimated useful life of the new freight cars. However, the unexpired costs of the old freight cars are not relevant to this decision. The conceptual error being made by the operating manager is the failure to distinguish between two decisions: the original decision and the current decision. Instead, he is mixing the two so that neither is evaluated correctly. The current decision should be influenced solely by expected future revenues and outlays, including the capital investment. The book value of the old equipment is per se irrelevant. The current decision should not carry the burden of past blunders. The past decision should be audited. In this instance, hindsight reveals that marketing management was overly optimistic. The key question is whether unwarranted optimism is being used again to justify additional outlays. Some instructors may wish to point out how decisions such as these might be affected by the long-term relationships with a big customer at this and other locations. Many decisions have such interdependencies.

..

267


6-59

(15-30 min.)

1.

Cost Comparison—Replacement of Truck Relevant Items Only

Cash operating costs Disposal value of old equipment Acquisition cost--new equipment Total relevant costs

Over the life of the New Truck Keep Replace Difference $53,100 $27,900 $ 25,200 (5,250) 5,250 22,950 (22,950) $53,100 $45,600 $ 7,500

The advantage of replacement is $7,500 for the 300,000 km together. 2.

Cost Comparison—Replacement of Old Truck Including Relevant and Irrelevant Items Keep Cash operating costs Old equipment (book value): Periodic write-off as depreciation or Lump-sum write-off Disposal value New equipment, acquisition cost Total costs

Over the life of the New Truck Replace Difference

$53,100

$27,900

$ 25,200

17,100 ------$70,200

17,100* (5,250)* 22,950** $62,700

5,250 (22,950) $ 7,500

* In a formal income statement, these two items would be combined as “loss on disposal” of $17,100 – $5,250 = $11,850. **In a formal income statement, written off as units of production depreciation based on actual kilometers travelled for a specific year. (Since it is based on actual kilometers travelled, it would be incorrect to use straight-line depreciation.)

6-60

(10 min.) This problem extends problem 6-A4. It should not be assigned without also assigning 6-

A4. 1.

..

The "replace" alternative would be chosen because it enhances cumulative wealth by $83,000.

268


2.

The division would show lower income, a loss of $16,000 instead of a gain of $5,000, for the first year under the "replace" alternative. The manager who wants to show better short-run performance will oppose replacement.

3.

The answers to the first two parts probably would be unaffected. The point is that decision models and performance evaluation models may conflict in nonprofit organizations too. Moreover, the money in the budget appropriation may have been spent. In addition, there is a higher likelihood of unfavorable publicity and also a danger of cuts in subsequent budget appropriations.

..

269


6-61

(20 min.)

The numbers in this case are a slight modification of those given in an article in the New York Times, November 21, 1994. 1. Attendance Revenue Expenses Net profit (loss)

On Broadway 400 $192,000 252,000* $ (60,000)

Off Broadway 400 $128,000 102,000 $ 26,000

*$102,000 + $150,000 = $252,000 2. Attendance Revenue Expenses Net profit 3.

a. b.

On Broadway 750 $360,000 252,000 $108,000

Off Broadway 375 $120,000 102,000 $ 18,000

$252,000  $60 = 4,200 weekly attendance 4,200  8 = 525 per show attendance $102,000  $40 = 2,550 weekly attendance 2,550  8 = 319 per show attendance

4. Attendance Revenue Expenses Net profit

On Broadway 600 $288,000 252,000 $ 36,000

Off Broadway 400 $128,000 102,000 $ 26,000

Total profit for a 26-week run: On Broadway: ($36,000 × 26) - $1,295,000 = $(359,000) Off Broadway: ($26,000 × 26) - $440,000 = $236,000 5.

Total profit for a 100-week run: On Broadway: ($36,000 × 100) - $1,295,000 = $2,305,000 Off Broadway: ($26,000 × 100) - $440,000 = $2,160,000

6.

a. b.

7.

Let X be the length of run in weeks at which on-Broadway profit equals off-Broadway profit: $36,000 X - $1,295,000 = $26,000 X - $440,000 $10,000 X = $855,000 X = 85.5 weeks

..

$1,295,000  $36,000 = 36weeks $ 440,000  $26,000 = 17 weeks

270


8.

Mr. Simon’s decision depends on his predictions of attendance on Broadway versus off Broadway and his attitude toward risk. The on-Broadway production has more risk because of its bigger up-front investment. If the attendance figures in requirements 4 and 5 are accurate, the off-Broadway alternative is better for any runs less than 85.5 weeks. If this may not be a long run, it appears that the off-Broadway alternative might be best. However, if attendance on Broadway exceeds 600 per show, especially if it is almost 1,000 per show, the Broadway alternative is better. There is a trend for non-musical plays to be produced off Broadway because of the large investment required on Broadway. Many plays do not last beyond a few weeks, and even filling a theater to capacity would require almost a 5-week run just to recoup the initial investment. Weekly profit would be ($60 × 1,000 × 8) - $252,000 = $228,000, so it would take $1,295,000  $228,000 = 5.7 weeks to break even. There is less risk off Broadway, especially because it takes many fewer theatergoers to reach the break-even point. For example, at capacity operations it takes 5.7 × 8 × 1,000 = 45,600 attendees to break even on Broadway. Off Broadway it requires only two-thirds of that number: ($40 × 500 × 8) - $102,000 = $58,000 weekly profit $440,000  $58,000 = 7.6 weeks to break even 7.6 × 8 × 500 = 30,400 attendees to break even.

6-62 (20-30 minutes) 1. Assume they outsource: Costs: 10,000 × €36.00 = Cost savings: Variable manufacturing costs (€14.00 + €18.00 + €6.00) × 10,000 Fixed overhead saved (€3.40 × 20,000 units) Net cost savings

€(360,000) 380,000 68,000 € 88,000

Therefore, yes, Excel should outsource. 2. If they outsource, their costs are: (10,000 × €36.00) + (€6.60* × 20,000) = €492,000 *This is the part of the fixed costs that are not saved: €10.00 - €3.40 If they accept the special order: They earn revenues of 6,000 × €56.00 Their costs are (16,000 × €38.00) + (€10.00 × 20,000) Net cost of the special order is

€336,000 808,000 €472,000

Therefore, the special order makes them better off by €20,000. ..

271


6-63

(15-20 min.)

1.

The opportunity cost of the land is 10% × ₹36,000,000 =

2.

Costs saved by closure of potato farm: Variable production costs Shipping costs Saved fixed costs Opportunity cost of land Total

₹3,600,000.

₹ 60,000,000

6,000,000 6,000,000 3,600,000 ₹75,600,000

Cost of purchasing potatoes: 8,000,000 kg × ₹8.00 = ₹64,000,000 Net savings to Parle from closing the potato farm and buying potatoes on the market is ₹75,600,000 – ₹64,000,000 = ₹11,600,000. 3.

The main ethical issue involves the impact of the plant closure on employees and on the community.

6-64

(10–15 min.)

1.

Even if reprocessing creates beans of acceptable quality, Starbucks should sell the beans as-is because it generates more profit than reprocessing them. Sell as is: Reprocess:

Revenue, $3.80 × 1,200 Revenue, $4.90 × 1,200 Reprocessing cost Shipping cost $.45 × 1,200 Total

$4,560 $5,880 (900) (540) $4,440

2.

Sell as is Reprocess Advantage to selling as-is

3.

The cost of buying and roasting the original beans is irrelevant because they are past or sunk costs.

..

$4,560 4,440 $ 120

272


6-65

(30-40 min.)

1.

Minnetonka Corporation should make the bindings. Cost saved by purchasing bindings: Material, 20% × $30 Labor, 10% × $35 Overhead, 10% × $2.50* Total

$6.00 3.50 .25 $9.75

Cost to buy per pair

$10.50

*Total overhead is $15 per pair Allocated overhead is $125,000  10,000 = $12.50 per pair Therefore, variable overhead is $15 - $12.50 = $2.50 per pair. 2.

Minnetonka Corporation would not pay more than $9.75 each because that is the cost to make the product internally.

3.

At a volume of 12,500 pair, Minnetonka should buy the bindings. The cost of buying 12,500 pair is $131,250. The cost of making 12,500 pair is: 12,500 × $9.75 Added fixed costs Total cost of making

$121,875 10,000 $131,875

Buying the bindings will save*

$625

* $131,875 – 131,250 Making the bindings saves variable costs of $.75 per pair. If sales exceed $10,000 ÷ $.75/pair = 13,333 pair, it is cheaper to make the bindings. 4.

Minnetonka Corporation needs 12,500 pair of bindings. The cost to buy 12,500 pair is $131,250. The cost to make 10,000 and buy 2,500 is: Cost to make 10,000 pair Cost to buy 2,500 pair Total

$97,500 26,250 $123,750

Therefore, Minnetonka should choose this latter course of action, which saves $131,250 - $123,750 = $7,500.

..

273


5.

There are many non-quantifiable factors that Minnetonka should consider in addition to the economic factors calculated above. Among such factors are: a. b. c. d. e. f.

..

The quality of the purchased bindings as compared to Minnetonka-produced bindings. The reliability of delivery to meet production schedules. The financial stability of the supplier. Development of an alternate source of supply. Alternate uses of binding manufacturing capacity. The long-run character and size of the market.

274


6-66

(30-45 min.)

1.

The $10,000 disposal value of the old equipment is irrelevant because it is the same for either choice. This solution assumes that the direct department fixed overhead is avoidable. You may want to explicitly discuss this assumption. Cost Comparison for Make or Buy Decision

Outside purchase cost at $1.00 Direct material at $.30 Direct labor and variable overhead at $.10 Depreciation ($188,000 - $20,000) ÷ 7 Direct departmental fixed overhead** at $.10 or $6,000 annually Totals

At 60,000 Units Normal Volume Make Buy $60,000 $18,000 -6,000 -24,000 -6,000 $54,000*

-$60,000

*On a unit basis, which is very dangerous to use unless proper provision is made for comparability of volume: Direct material Direct labor and variable overhead Depreciation, $24,000 ÷ 60,000 Other fixed overhead**, $6,000 ÷ 60,000 Total unit cost

$.30 .10 .40 .10 $.90

Note particularly that the machine sales representative was citing a $.24 depreciation rate that was based on 100,000 unit volume. She should have used a 60,000 unit volume for the Rohr Company. **Past records indicate that $.05 of the old unit cost was allocated fixed overhead that probably will be unaffected regardless of the decision. This assumption could be challenged. This total of $3,000 ($.05 × 60,000 units) could be included under both alternatives, causing the total costs to be $57,000 and $63,000, and the unit costs to be $.95 and $1.05, respectively. Note that such an inclusion would have no effect on the difference between alternatives. Also, this analysis assumes that any idle facilities could not be put to alternative profitable use. The data indicate that manufacturing rather than purchasing is the better decision--before considering required investment.

..

275


2. Outside purchase at $1.00 Direct material at $.30 Direct labor and variable overhead at $.10 Depreciation Other direct fixed overhead Totals

At 50,000 Units Make Buy $50,000 $15,000 --

At 70,000 Units Make Buy $70,000 $21,000 --

5,000 24,000 6,000 $50,000

7,000 24,000 6,000 $58,000

---$50,000

---$70,000

At 70,000 units, the decision would not change. At 50,000 units, Rohr would be indifferent. The general approach to calculating the point of indifference is: Let X = Point of indifference in units Total costs of making = Total costs of buying $.30X + $.10X + $24,000 + $6,000 = $1.00 X $30,000 = $.60 X X = 50,000 units 3.

Other factors would include: Dependability of estimates of volume needed, need for quality control, possible alternative uses of the facilities, relative merits of other outside suppliers, ability to renew production if price is unsatisfactory, and the minimum desired rate of return. Factors that are particularly applicable to the evaluation of the outside supplier include: short-run and long-run outlook for price changes, quality of goods, stability of employment, labor relations, and credit standing.

..

276


6-67

(20 - 30 minutes)

1.

Assume they outsource:

Costs: 25,000 × $38 Less cost savings: Variable manufacturing costs ($15 + $8 + $10) × 25,000 Fixed overhead saved ($2 × 40,000 units) Net Cost

($950,000) 825,000 80,000 $ 45,000

Therefore, do not outsource. 2.

If they outsource and make the Scanmeister, their cost savings are $825,000 in variable manufacturing cost. Additionally, they earn a contribution margin of 10,000 × $15 =$150,000 on the Scanmeister. Therefore, they would be willing to pay up to $975,000 ÷ 25,000 = $39 per unit for the outsourced units.

6-68 (30-40 minutes) 1. Assume they outsource: Cost Savings per casing: Variable manufacturing costs ($12 + $8 + $4.50) Fixed overhead saved ($3 + $1.50) Total cost savings

$ 24.50 4.50 $ 29.00

Since the outsourcing price is $29.75 per casing, Nike should not outsource production. They are better off by $.75 × 20,000 = $15,000, if they make it themselves. Note that the rent is irrelevant. 2. Assume they outsource: Cost Savings per casing: Variable manufacturing costs ($12 + $8 + $4.50) Fixed overhead saved* Fixed supervision saved ($30,000 ÷ 15,000) Total cost savings

$ 24.50 4.00 2.00 $ 30.50

* Total depreciation = $3 × 20,000 = $60,000; Depreciation/casing @ 15,000 casings = $60,000 ÷ 15,000 = $4.00/casing Since the outsourcing price is $29.75 per casing, Nike should outsource production. They are better off by $.75 × 15,000 = $11,250 if they buy casings. Again, note that the rent is irrelevant. ..

277


6-69 (25 - 30 min.) For the solution to this Excel Application Exercise, follow the step-bystep instructions provided in the textbook chapter. From this analysis we can answer the 3 questions. 1. It does not make any difference how the $117,000 is allocated. It is irrelevant to the decision about whether to sell at the split-off point or to process further. 2. Only product C should be processed further. Products A and B should be sold at the split-off point. 3. The combined operating income would be $44,000. 6-70

(60 min. or more)

This exercise provides experience searching the literature of a particular subject as well as developing a better understanding of outsourcing decisions. Students will research the literature individually and then share their findings with their group. Requirements 2 and 3 help develop critical thinking. The articles are not likely to answer these questions directly, but students will probably be able to infer answers from the information given. The short report in requirement 4 will help develop an ability to select the most important points from the literature and report them in a way that is helpful to others.

..

278


6-71 (30-45 min.) NOTE TO INSTRUCTOR. This solution is based on the web site as it was in late 2012. Be sure to examine the current web site before assigning this problem, as the information there may have changed. 1.

Some of the topics are “Our Story,” “Bagged Coffee”, “About the Company”, “Investor Services”, “Social and Environmental Responsibility”, “Career Opportunities”, “Contact Us”, “Our Catalog”, and “Email newsletter.” The information that would be provided for each link would be different. That is the reason that the different links are highlighted.

2.

To find out information concerning the financial statements, the place to look would be in the “Investor Services,” which is under the category “About the Company.” In 2011, the firm had a profit of $199,501,000. The biggest cost the firm encountered was cost of sales (product costs). The firm did not pay any dividends. The firm has never paid any dividends and does not plan to start paying them in the foreseeable future. Instead, the profits are being reinvested in the firm. Thus, if I wanted an income producing stock, this would not be a logical investment choice.

3.

To learn more about coffee, you would click on the link “Our Story,” followed by the additional link “Behind the Bean.” The links that likely would provide information about coffee differences in general (but not their own specific coffees), based solely on the titles given to such links, would be “Selecting Your Coffee,” and “Brewing and Tasting.” The solution to the information provided will depend on which link the student selects. Links that provides information about differences in their coffees and their prices can be found on the home page, such as “Bagged Coffee,” and “K-Cup Packs.” Under these links, students will find information about various coffee groupings (and their prices). The pricing information is likely to be a major factor of difference between differing types of coffee.

4.

Green Mountain Coffee provides information regarding the environment, environmental actions the company has taken, the organizations, including those in the coffee community and those in the local community, that the company supports. Additionally, it displays its 2011 Corporate Social Responsibility Report. The information, while interesting, would not help in determining how the product tasted, nor would it tell about the quality of the product. However, such information might be relevant to an investor who wants to invest only in socially conscious companies or one who believes that socially conscious companies will have an advantage if the government imposes costs (taxes) based on the company’s impact on the environment.

..

279


CHAPTER 7 Introduction to Budgets and Preparing the Master Budget COVERAGE OF LEARNING OBJECTIVES

LEARNING OBJECTIVE LO1: Explain how budgets facilitate planning and coordination. LO2: Anticipate possible human relations problems caused by budgets. LO3: Explain potentially dysfunctional incentives in the budget process. LO4: Explain the difficulties of sales forecasting. LO5: Explain the major features and advantages of a master budget. LO6: Follow the principal steps in preparing a master budget. LO7: Prepare the operating budget and the supporting schedules. LO8: Prepare the financial budget. LO9: Use a spreadsheet to develop a budget (Appendix 7).

..

CASES, EXCEL, FUNDAMENTAL ADDITIONAL COLLAB. & ASSIGNMENT ASSIGNMENT INTERNET MATERIAL MATERIAL EXERCISES A1, B1

B1

17, 25, 40

B1

22, 39, 39, 40

B1

23, 42

A1, B1

4, 24, 26, 39

A1, B1

29, 40

43,45

A1, B1

28, 29, 30, 30, 31, 31, 40

43, 45, 46, 48

A1, B1

19, 27, 27, 29, 32, 33, 33, 34, 34, 35, 36, 36, 37, 37, 38, 38 41, 42

43, 44, 47, 48

B1

49

280


CHAPTER 7 Introduction to Budgets and Preparing the Master Budget 7-A1 (60-90 min.) 1.

Exhibit I GREATBUY ELECTRONICS STORE Budgeted Income Statement For the Three Months Ending August 31, 20X8

Sales Cost of goods sold (.63 × $580,000) Gross profit Operating expenses: Salaries, wages, commissions Other expenses Depreciation Rent, taxes and other fixed expenses Income from operations Interest expense* Net income

$580,000 365,400 $214,600 $139,200 17,400 3,000 27,000

186,600 $ 28,000 1,219 $ 26,781

* See schedule g for calculation of interest.

..

281


Exhibit II GREATBUY ELECTRONICS STORE Cash Budget For the Three Months Ending August 31, 20X8 Beginning cash balance Minimum cash balance desired (a) Available cash balance Cash receipts & disbursements: Collections from customers (schedule b) Payments for merchandise (schedule d) Fixtures (purchased in May) Payments for operating expenses (schedule f) Payments for interest (b) Net cash receipts & disbursements Excess (deficiency) of cash before financing (a + b) (c) Financing: Borrowing, at beginning of period Repayment, at end of period (d) Ending cash balance (beginning balance + b + c)

June 6,600 4,000 2,600

July $ 4,600 4,000 $ 600

August $ 4,521 4,000 $ 521

$ 163,000

$195,900

$ 184,700

(151,200) (5,000)

(107,100) -

(107,100) -

(73,800) $ (67,000)

(54,900) (979)* $ 32,921

(54,900) (240)* $ 22,460

(64,400)

33,521

22,981

$ $

$ 65,000 -

$

(33,000)

$

(22,000)

$ 4,600

$ 4,521

$

4,981

* See schedule g for calculation of interest

..

282


Exhibit III GREATBUY ELECTRONICS STORE Budgeted Balance Sheet August 31, 20X8 Assets Cash (Exhibit II) Accounts receivable* Merchandise inventory Total current assets Net fixed assets: $52,000 less depreciation of $3,000 Total assets

Liabilities and Stockholders’ Equity $ 4,981 154,700 75,600 $234,281 49,000 $284,281

Accounts payable Notes payable Total current liabilities Stockholders' equity: $171,900 plus net income of $26,781 Total liabilities and stockholders’ equities

*July sales, 30% × 70% × $170,000 August sales, 100% × 70% × $170,000 Accounts receivable ** See schedule g Schedule a: Sales Budget Credit sales (70%) Cash sales (30%) Total sales (to Exhibit I)

$ 75,600 10,000** $ 85,600

198,681 $284,281

$ 35,700 119,000 $154,700

June

July

August

Total

$168,000 72,000 $240,000

$119,000 51,000 $170,000

$119,000 51,000 $170,000

$406,000 174,000 $580,000

Schedule b: Cash Collections Cash sales On accounts receivable from: April sales May sales June sales July sales Total collections (to Exhibit II)

June $ 72,000

July $ 51,000

August $ 51,000

27,300 63,700 $163,000

27,300 117,600 $195,900

50,400 83,300 $184,700

Schedule c: Purchases Budget May Desired purchases: 63% × next month's sales $151,200 Schedule d: Disbursements for Purchases Last month's purchases (to Exhibit II) ..

June

July

August

$107,100 June $151,200

$107,100 July $107,100

$75,600 August $107,100 283


Other required items related to purchases Accounts payable, August 31, 20X8 (63% × September sales - to Exhibit III) Cost of goods sold (to Exhibit I)

$151,200

Schedules e & f: Cash & Total Operating Expense Budgets June July Salaries, wages, commissions $57,600 $40,800 Other Variable expenses 7,200 5,100 Fixed expenses 9,000 9,000 Cash expenditures 73,800 54,900 Depreciation 1,000 1,000 Total operating expenses $74,800 $55,900

$107,100 August $40,800 5,100 9,000 54,900 1,000 $55,900

$75,600 $107,100 Total $139,200 17,400 27,000 183,600 3,000 $186,600

Schedule g: Borrowing, repayment, and interest calculations for bank loan Beginning balance Monthly interest expense @ 9% p.a. Ending balance before repayment Principal repayment (from statement of receipts and disbursements) Interest payment Ending balance

June $65,000 488 $65,488

July $ 65,488 491 65,979

August $ 32,000 240 32,240

(33,000) (979) $ 32,000

(22,000) (240) $ 10,000

2. This is an example of the classic short-term, self-liquidating loan. The need for such a loan often arises because of the seasonal nature of a business. The basic source of cash is proceeds from sales to customers. In times of peak sales, there is a lag between the sale and the collection of the cash, yet the payroll and suppliers must be paid in cash right away. When the cash is collected, it in turn may be used to repay the loan. The amount of the loan and the timing of the repayment are heavily dependent on the credit terms that pertain to both the purchasing and selling functions of the business.

..

284


7-B1

(60-120 min.) $ refers to Australian dollars.

1.

See Exhibits I, II, and III and supporting schedules a, b, c, d.

2.

The cash budget and balance sheet clearly show the benefits of moving to just-intime purchasing (though the transition would rarely be accomplished as easily as this example suggests). However, the company would be no better off if it left much of its capital tied up in cash -- it has merely substituted one asset for another. At a minimum, the excess cash should be in an interest bearing account -- the interest earned or forgone is one of the costs of inventory.

Schedule a: Sales Budget Total sales (100% on credit)

January $378,000

February $413,000

March $273,000

Schedule b: Cash Collections 40% of current month's sales 10% of previous month's sales 50% of second previous month's sales Total collections

$151,200 16,100 84,000 $251,300

$165,200 37,800 80,500 $283,500

$109,200 41,300 189,000 $339,500

Schedule c: Purchases Budget Desired ending inventory Cost of goods sold Total needed Beginning inventory Purchases Schedule d: Disbursements for Purchases 100% of previous month's purchases March 31 accounts payable

..

December

January

February

March

$153,000 57,500 $210,500 59,000 $151,500

$ 22,000 135,000 $157,000 153,000 $ 4,000

$ 22,000 147,500 $169,500 22,000 $147,500

$ 22,000 97,500 $119,500 22,000 $ 97,500

$151,500

$ 4,000

$147,500 $ 97,500

285


Exhibit I FLYING FISH KITE Cash Budget For the Three Months Ending March 31, 20X2

Cash balance, beginning Minimum cash balance desired (a) Available cash balance Collections from customers (Schedule b) Payments for merchandise (Schedule d) Rent Wages and salaries Miscellaneous expenses Dividends Payments for interest Purchase of fixtures (b) Net cash receipts & disbursements Excess (deficiency) of cash before financing (a + b) Financing: Borrowing, at beginning of period Repayment, at end of period (c) Total cash increase (decrease) from financing (d) Cash balance, end (beginning balance + b + c)

..

January

February

March

$ 30,000 15,000 15,000

$ 16,950 15,000 1,950

$ 209,920 15,000 194,920

251,300

283,500

339,500

(151,500) (28,450) (80,000) (4,000) (2,400) $ (15,050)

(4,000) (500) (80,000) (4,000) (30) $194,970

(147,500) (500) (80,000) (4,000) (4,000) $ 103,500

$ (50)

$196,920

$298,420

$

2,000 -

$

(2,000)

$

-

$

2,000

$ (2,000)

$

-

$ 16,950

$ 209,920

$313,420

286


Exhibit II FLYING FISH KITE Budgeted Income Statement For the Three Months Ending March 31, 20X2 Sales (Schedule a) Cost of goods sold (Schedule c) Gross margin Operating expenses: Rent* Wages and salaries Depreciation Insurance Miscellaneous Net income from operations Interest expense Net income

$1,064,000 380,000 $ 684,000 $ 52,200 240,000 2,700 1,350 12,000

308,250 $ 375,750 30 $ 375,720

*[(January-March sales less $50,000) × .05] + (3 × $500)

..

287


Exhibit III FLYING FISH KITE Budgeted Balance Sheet March 31, 20X2 Assets Current assets: Cash (Exhibit I) Accounts receivable* Merchandise inventory (Schedule c) Unexpired insurance Fixed assets, net: $62,000 + $4,000 - $2,700 Total assets Liabilities and Stockholders' Equity Liabilities: Accounts payable (Schedule d) Rent payable. Dividends payable Stockholders' equity** Total liabilities and stockholders' equity.

$313,420 370,300 22,000 4,050

$ 97,500 50,700 2,400

$709,770 63,300 $773,070

$150,600 622,470 $773,070

*February sales (.50 × $413,000) plus March sales (.60 × $273,000) = $370,300 **Balance, December 31, 20X1 Add: Net income Total Less: Dividends paid Balance, March 31, 20X2

..

$249,150 375,720 $624,870 2,400 $622,470

288


7-1 Budgeting 1) provides an opportunity for managers to reevaluate existing activities and evaluate possible new activities, 2) compels managers to think ahead by formalizing their responsibilities for planning, 3) aids managers in communicating objectives to units and coordinating actions across the organization, and 4) provides benchmarks to evaluate subsequent performance. 7-2 Budgeting is primarily attention directing because it helps managers to focus on operating or financial problems early enough for effective planning or action. 7-3 Strategic planning covers no specific time period, is quite general, and often is not built around financial statements. Long-range planning usually has a 5- or 10-year horizon and consists of financial statements without much detail. Budgeting usually has a horizon of one year or less, and consists of financial statements with much detail. 7-4 Continuous budgets add a month (or quarter) in the future as the month (or quarter) just ended is dropped. Therefore, the continuous budget provides a continually updated budget looking twelve months ahead. When the new month (or quarter) is added, the budget for the remainder of the current year may also be revised. When companies revise the budgets for the remainder of the current year, they usually compare subsequent results to the original budget (a fixed target) in addition to comparing them to the latest revised budget. Thus, the comparison always reflects the reality. 7-5 If the measures used to reward employees in the performance evaluation system are not aligned with the goals of the company, the incentives from the evaluation system may lead employees to take actions that conflict with the interests of the company. 7-6 Lower-level managers bias their forecasts to create budgetary slack or padding. Upper-level managers adjust for this bias in creating a revised budget. Therefore, lowerlevel managers introduce additional bias to compensate for the adjustment that will be made by upper-level managers, and upper-level managers introduce additional adjustments for the additional bias. This cycle can quickly destroy the potential benefits of budgets. 7-7 A manager may make short-run decisions to increase profits that are not in the company’s best long-run interests, such as offering customers excessively favorable credit terms or cutting discretionary expenditures such as R&D and advertising, trading future sales for current profits. In the extreme, the manager might choose to falsely report inflated profits. 7-8 First, by moving this year's sales into next year or moving next year's expenses into this year, the manager ensures a higher level of reported profit (and probably a higher bonus) next year. Second, by decreasing this year's income, the manager avoids ratcheting up of performance expectations in setting the bonus target for the next year. 7-9 Budgeted performance is better than past performance as a basis for judging current performance because the budget contains no hidden inefficiencies and can be founded on current rather than past economic conditions. ..

289


7-10 Budgets are especially important in environments that are rapidly changing. They force managers to look forward and plan for change. Budgets force analysis of the factors that are bringing about the changes. 7-11 No. When budgeting in done correctly, it is an important aid to managers. Managers need time to plan and coordinate their various activities. Budgeting forces them to take time from the day-to-day problems and focus on longer-term issues. 7-12 The sales forecast is the starting point for budgeting because all other operating activities of the company are affected by the volume of sales. 7-13 The sales forecast is influenced by past patterns of sales, estimates made by the sales force, general economic conditions, competitors' actions, changes in prices, market research studies, and advertising and sales promotion plans. 7-14 An operating budget is used as a guide for production and sales and it focuses on the income statement. A financial budget is used to control the receipt and disbursement of funds and it focuses on the statement of cash receipts and disbursements. 7-15 Operating expenses are costs charged to the income statement in a particular period. Some operating expenses may be associated with the sales of the period, and others may be costs of being in business for the period. Cash disbursements for these operating expenses may fall in a previous period (assets purchased in one period and depreciated over future periods) or in a future period (wages accrued in a period but paid in the next period), as well as during the period. 7-16 A cash budget is an attempt to monitor and regulate the flow of cash in optimum fashion. 7-17 Lower level managers often have negative attitudes toward budgets as they believe that budgets primarily aim to cut costs. Their beliefs find justification when their performance is evaluated by simply comparing actual expenditures against the budgeted figures without substantive input from the managers. This makes the budgeting ineffective. In a participative budgeting process, managers from all levels participate in setting the budgets and hence the targets are realistic. This gains trust from the employees who are responsible for controlling costs. As a result, the budgeting seems to be more effective.

..

290


7-18 Both functional and activity-based master budgets begin with the forecasted demand for products or services. However, whereas functional budgets then determine the inventory, materials, labor, and overhead budgets, the activity-based budget focuses on determining the demand for key activities. This demand is measured by the cost-driver unit for each activity. Then the budgeted resource consumption rates are used to set the budgets for resources such as materials, labor, and overhead. The focus on activities and consumption rates in activitybased budgeting is what managers believe offers value from an operational control perspective. 7-19 A financial planning model can provide operation and financial budgets under alternative assumptions relating to product mix, sales expectations, production constraints, scheduling, etc. As a result, financial planning models can considerably shorten a manager’s response time as revised plans can be prepared within a very short span of time. Spreadsheet software has brought financial planning within the reach of all businesses irrespective of their size or volume of operations. However, the results of these models are reliable so far as the assumptions are realistic. Faulty assumptions may lead to devastating consequences. Hence, a blind dependence on financial planning model without catering to the feasibility of the assumptions must always be avoided. 7-20 Setting up the master budget on a spreadsheet is time-consuming -- the first time. However, if it is done properly, with maximum flexibility, then the ease of subsequent use probably will more than offset that initial cost. Ultimately, though, the master budget system must meet the cost-benefit test. Improved budgeting systems are only worthwhile if they offer net benefits. Preparing and revising the master budget of a large company just would not be feasible without the aid of a computer. 7-21 Spreadsheets can be used to make a mathematical model of an organization. It may take much effort to create the model, but once it is in place it can be used over and over again with minimal effort. Such a model is especially useful for sensitivity analysis, which is the asking of "what if" questions. 7-22 Budgets that are used primarily for limiting spending provide incentives for “game playing.” Accurate forecasts and estimates give way to strategies designed to avoid budget cuts or to justify increased budgets. Budgets should have a much larger role in the effective and efficient management of an organization. A budget should be a decision tool. It helps managers project the results of their decisions, thereby aiding them in making the right decisions. It also provides a base for adapting to change. Anything that results in loss of budget accuracy will limit the decision usefulness of the budget.

..

291


7-23 Accurate sales forecasts are essential to budgeting. Sales personnel are often “closest to the action” and therefore in the best position to make accurate forecasts. They are in direct contact with customers, and often they are the first to notice trends. A central staff function, such as market research, can set parameters for forecasting and give some common ground rules. But usually it is important to get sales personnel heavily involved because they have information that no one else has. Most importantly, the more involved sales personnel are, the more committed they will be to achieving budgeted sales goals. 7-24 The planning that comes through a good budget process is important to all segments of an organization. Segments with both revenues and expenses can show a budgeted profit. Other segments that have only expenses, such as a research and development department, still have to plan their operations. It is important to predict the resources needed to meet the segment’s objectives so that required resources can be obtained. Budgeting provides a formal channel for communication between the segment and top management about what activities the segment is to undertake. 7-25 A key to employee acceptance of a budget is participation. Budgets created with the active participation of all affected employees are generally more effective than budgets imposed on subordinates. If a budget is to help direct future activities, employees must accept the budget. Acceptance means believing that the budget reflects a desired future path for the organization. If a manager has been a participant in determining the future path – that is, helped develop the budget – he or she is more likely to accept it as a desirable objective. 7-26

(5 min.)

1. a. Capital budget b. Cash budget c. Budgeted balance sheet 7-27

2. Sales budget (or operating budget) 3. Continuous (rolling) 4. Overall goals of the organization

(10-15 min.)

Connect2Win will be using cash until the beginning of 2023, when cash receipts will begin to exceed cash disbursements. Therefore, the following amount of venture capital is needed to carry the firm to the beginning of 2023: Initial capital investment in 2021 £760,000 Cash outflow during 2021 (12 × £70,000) 840,000 Cash outflow during 2022 [12 × (£70,000 - £60,000)] 120,000 Total £1,720,000

..

292


7-28 1.

2.

(10-15 min.) Cost + (.20 × Cost) = Sales 1.20 × Cost = $1,980,000 Cost = $1,650,000 Use the familiar identity, Beginning Inventory plus Purchases equals Cost of Goods Sold plus Ending Inventory. To compute required purchases, compute the inventory needed (Cost of Goods Sold plus Ending Inventory) and then subtract the amount that will come from Beginning Inventory: July Merchandise Purchases Cost of goods sold ($2,076,000  1.20) Add: Target ending inventory .25 × ($2,184,000  1.20) Cost of goods needed Less: Beginning inventory .25 × ($2,076,000  1.20) Required Purchases

7-29

$1,730,000 455,000 $2,185,000 432,500 $1,752,500

(25-30 min.)

1. July collections include: May sales billed June 5, .28 × .5 × $790,000 June sales billed June 20, .28 × .5 × $810,000 June sales billed July 5, .70 × .5 × $810,000 × .98 July sales billed July 20, .70 × .5 × $990,000 × .98 Total

$110,600 113,400 277,830 339,570 $841,400

2. .60 × .25 × $810,000 = $121,500 3. Ending inventory, .60 × .25 × $990,000 Merchandise needed for current month's sales, .60 × $810,000 Total needs Beginning inventory, .60 × .25 × $810,000 Required Purchases 4. Ending inventory, .60 × .25 × next month’s sales Merchandise needed for current month's sales, .60 × sales Total needs Beginning inventory, .60 × .25 × current month's sales Required Purchases Payments, 1/2 of current purchases, 1/2 of preceding month's purchases, .5 × $586,500 + .5 × $522,000

..

$148,500 486,000 634,500 121,500 $513,000 July $141,000 594,000 735,000 148,500 $586,500

August $ 99,000 564,000 663,000 141,000 $522,000 $554,250

293


7-30 (15 min.) This exercise is straightforward and follows the chapter example closely. All amounts are in dollars. September October November Sales budget Credit sales, 30% $153,000 $160,500 $186,000 Cash sales, 70% 357,000 374,500 434,000 Total sales, 100% $510,000 $535,000 $620,000 Cash collections budget Cash sales this month 100% of last month's credit sales Total collections

$357,000 120,000 $477,000

$374,500 153,000 $527,500

$434,000 160,500 $594,500

7-31 (15-25 min.) This problem is slightly more complex than 7-30. All amounts are in thousands of Japanese yen. July August September Sales budget Credit sales, 70% ¥284,200 ¥317,800 ¥347,200 Cash sales, 30% 121,800 136,200 148,800 Total sales ¥406,000 ¥454,000 ¥496,000 Cash collections budget Cash sales this month ¥121,800 ¥136,200 ¥148,800 60% of this month's credit sales 170,520 190,680 208,320 30% of last month's credit sales 90,600 85,260 95,340 10% of next-to-last month's credit sales 30,000 30,200 28,420 Total collections ¥412,920 ¥442,340 ¥480,880 7-32

(10-15 min.) Collections from: January sales: February sales: February sales: March sales: Total cash collections

..

$370,000 × 15% $420,000 × 15% × 98% $420,000 × 10% $460,000 × 55% × 97%

$ 55,500 61,740 42,000 245,410 $404,650

294


7-33

(15-20 min.) GREEN STAR INC. Purchases and Disbursements Budgets

Purchases budget Ending inventory Cost of goods sold, 60% of sales Total needed Beginning inventory Purchases Disbursements for purchases 20% of this month's purchases 60% of last month's purchases 20% of second-last month's purchases

April

May

June

$361,600 431,040 792,640 433,600 $359,040

$334,400 344,640 679,040 361,600 $317,440

$385,600 293,760 679,360 334,400 $344,960

$71,808 180,480*

$63,488 215,424

$68992 190,464

82,560** $334,848

60,160 $339,072

71,808 $331,264

*.60 × 300,800 = 180,480 **.20 × 412,800 = 52,560

..

295


7-34 1.

2.

(20-25 min.) €460,000 – € [30,000 + .8 × (.7 × 600,000)]

= €460,000 – € [30,000 + .8 x (420,000)] = €460,000 – €366,000 = €94,000

Purchases and Disbursements Budgets July

August

September

Purchases budget Ending inventory target* Cost of goods sold, 70% of sales Total inventory needed Beginning inventory Purchases

€354,800 420,000 774,800 460,000 €314,800

€410,800 406,000 816,800 354,800 €462,000

€478,000 476,000 954,000 410,800 €543,200

Disbursements for purchases 70% of last month's purchases 30% of this month's purchases Disbursements for purchases

€210,000 94,440 €304,440

€220,360 138,600 €358,960

€323,400 162,960 €486,360

*Inventory targets, end of month: July: 30,000 + .8 × (0.7 × 580,000) = 30,000 + .8 × (406,000) = €354,800 August: 30,000 + .8 × (0.7 × 680,000) = 30,000 + .8 × (476,000) = €410,800 September: 30,000 + .8 × (0.7 × 800,000) = 30,000 + .8 × (560,000) = €478,000

..

296


7-35

(20 min.) This is a straightforward exercise. CARLSON COMPANY Cash Budget For the Month Ended June 30, 20X4 (in thousands) Cash balance, May 31, 20X4 Cash Receipts: Collections from customers from: June sales (.80 × $290) May sales (.5 × 24)* April sales Total cash available during June Cash Disbursements: On accounts payable of May 31 On June purchases, .25 × $192 Wages Utilities Advertising Office expenses Ending Cash, June 30, 20X4

$ 15 $232 12 20 $145 48 36 5 10 4

264 $279

248 $ 31

*$24,000 = 20% of May sales, 10% of which or half the remainder will be collected in June. All of April's remaining sales will be collected in June.

..

297


7-36 (20-25 min.) The collections from August sales are a bit tricky. Note that the receivable balance from August sales at August 31 is €1,350,000; therefore, four fifths of this balance will be received in September (because 40/50 will be collected in September and 10/50 will be collected in October). WONDER TOYS Budgeted Statement of Cash Receipts and Disbursements For the Month Ending September 30, 2020 Cash balance, August 31, 2020 Add receipts, collections from customers: From September sales, 1/2 × €3,000,000 From August sales, 4/5 × €1,350,000 From July sales Total cash available Less disbursements: Merchandise purchases, €1,350,000 × 40% Payment on accounts payable Payrolls Insurance premium Other expenses Repayment of loan and interest Cash balance, September 30, 2020

..

€300,000 €1,500,000 1,080,000 240,000 € 540,000 1,380,000 270,000 4,500 135,000 291,600

2,820,000 €3,120,000

2,621,100 €498,900

298


7-37

(40-60 min.) EXCLUSIVE CANDLES Statement of Estimated Cash Receipts and Disbursements For the Month Ended October 31, 2020

Cash balance, October 1, 2020 Receipts, collections of receivables (Schedule 1) Total cash available Less disbursements: Merchandise purchases (Schedule 2) Variable expenses (Schedule 3) Fixed expenses (Schedule 3) Cash balance, October 31, 2020

$ 10,000 71,262 $81,262 $35,160 8,160 2,350

45,670 $35,592

Schedule 1, Collections of Accounts Receivable: Collected in October

From August sales From September sales From October sales Total October collections * Rounded to the nearest whole dollar.

Sales $17,600 $88,000 $74,800

Percent 5% 30% 60% × 98%

Amount $ 880 26,400 43,982* $71,262

Schedule 2, Payments for Merchandise: Target ending inventory Cost of goods sold Total needs Beginning inventory Purchases Payments, 1/2 × $27,360 October purchases Accounts payable, end of September, 1/2× $42,960 purchases Total payments in October

September $ 28,560* 48,000 $76,560 33,600** $42,960

October $ 15,120* 40,800 $55,920 28,560 $27,360 $13,680 21,480 $35,160

* (24/44) × .7 × 74,800 = $28,560; (24/44) × .7 × 39,600 = $15,120 ** (24/44) × .7 × 88,000 = $33,600 Schedule 3, Payments for Selling and General Administrative Expenses: Total selling and general administrative expenses Less fixed expenses ..

$169,200 54,000 299


7-38

Total variable expenses for year (vary with sales)

$115,200

October variable expenses: $115,200 × (October sales  Year's sales) = $115,200 × ($74,800  $1,056,000)

$ 8,160

Total fixed expenses Less depreciation (no current cash outlay) Total cash required for fixed expenses for year

$54,000 25,800 $28,200

October cash required for fixed expenses: ($28,200  12)

$ 2,350

(30 - 40 min.) This problem is solved most easily on a spreadsheet.

1. The IHC’s monthly cash budget is shown on Exhibit 7-38 on the two following pages. 2. Increase in annual profit: (In ₹ ’000): 6 months × .05 × 300 rooms × ₹20,000 × 30 days × .98 collected Increase in costs: Variable cost 6 months × .05 × 300 rooms × ₹2,400 × 30 days Increase in profit

..

₹52,920 6,480 ₹46,440

300


EXHIBIT 7-38 INDIAN HOTEL COMPANY Monthly Cash Budget (in ₹ ‘000) January Revenues ₹144,000 Collections: Previous Month Sales ₹ 37,260 This Month Sales 86,400 Next Month Sales 21,600 Total collections 145,260 Disbursements: Variable costs (₹2,400/room) 17,280 Fixed salaries 32,000 Fixed operating costs 9,600 Interest payments _________ Total disbursements 58,880 Net cash inflow (outflow) ₹86,380

..

February ₹144,000

March ₹126,000

April ₹126,000

May ₹126,000

June ₹ 126,000

₹ 33,120 86,400 18,900 138,420

₹ 33,120 75,600 18,900 127,620

₹ 28,980 75,600 18,900 123,480

₹ 28,980 75,600 18,900 123,480

17,280 32,000

15,120 32,000

15,120 32,000

15,120 32,000

15,120 32,000

9,600 _________ 58,880 ₹79,540

9,600 _________ 56,720 ₹70,900

9,600 _________ 56,720 ₹66,760

9,600 _________ 56,720 ₹ 66,760

9,600 288,000 344,720 ₹(221,240)

28,980 75,600 18,900 123,480

301


EXHIBIT 7-38 (Continued) INDIAN HOTEL COMPANY Monthly Cash Budget (in ₹ ‘000) July ₹126,000

August ₹126,000

September ₹144,000

October ₹162,000

November ₹162,000

December ₹162,000

Total ₹1,674,000

₹ 28,980 75,600 18,900 123,480

₹ 28,980 75,600 21,600 126,180

₹ 28,980 86,400 24,300 139,680

₹ 33,120 97,200 24,300 154,620

₹ 37,260 97,200 24,300 158,760

₹ 37,260 97,200 21,600 156,060

₹ 385,020 1,004,400 251,100 1,640,520

15,120 32,000

15,120 32,000

17,280 32,000

19,440 32,000

19,440 32,000

19,440 32,000

200,880 384,000

9,600

9,600 _________ 56,720 ₹ 69,460

9,600 _________ 58,880 ₹ 80,800

9,600 _________ 61,040 ₹ 93,580

9,600 _________ 61,040 ₹97,720

9,600 288,000 349,040 ₹(192,980)

115,200 576,000 1,276,080 ₹ 364,440

56,720 ₹ 66,760

..

302


7-39

( 15 min.)

1.

Cost-saving actions would probably focus on one or more of the activities of the Packaging and Delivery Department. Isabella might start with the non-value added activities, handling and recordkeeping. For example, the activity-based budget data suggest that the cost per move is $10 ($112,000 ÷ 11,200 moves). Assuming that handling costs are variable with respect to the number of moves, reducing the number of moves by about 6,500 moves (about 58%) would provide the required savings. Reorganizing the warehouse might help reduce the number of moves. Similarly, the recordkeeping cost per transaction is about $.40 ($26,350 ÷ 65,000 transactions). Reduction or elimination of record-keeping transactions could achieve some savings, but not the entire amount needed. Isabella might also focus on activities that cost the most (and have the most potential for cost savings). For example, the activity-based budget shows that the highest total-cost activity is shipping, so that might be the best place to look for potential cost saving from changing processes.

2.

Regardless of what methods are selected to achieve cost saving, the activity-based budget seems to be a better starting point. The traditional budget does not show how changes in activities might affect costs, whereas the activity-based budget does.

7-40

(25-30 min.)

1.

An optimistic preliminary budget might be as follows, assuming level sales volume, a $.94 per pound price, and a 2% decrease in variable costs. Sales, 1.6 million pounds @ $.94/pound Variable costs Fixed costs, primarily depreciation Pretax profit

$1,504,000 (862,400) (450,000) $ 191,600

This budget does not meet the $209,000 profit goal. Stark has a dilemma of submitting a realistic budget that does not meet Philp's goal or preparing an unrealistic budget. The following budget, which assumes that prices will not fall, sales levels will be maintained, and some fixed costs will be saved, would meet the profit target. Although Stark does not believe the assumptions, she might feel pressure to submit it (or something similar) to headquarters: Sales, 1.6 million pounds @ $.95/pound Variable costs, .98 × $880,000 Fixed costs, primarily depreciation Pretax profit *$1,520,000 - $862,400 - $209,000

..

$1,520,000 (862,400) (448,600)* $ 209,000

303


2.

Two major problems are the arbitrary setting of budget targets by top management without regard to whether the targets can be achieved and the draconian measures used when a budget is not met, even if the shortfall is small or reasonable explanations for the shortfall are given.

3.

Apparently the preliminary financial results are as follows: Sales, 1.6 million pounds @ $.945/pound Variable costs, .98 × $880,000 Fixed costs, primarily depreciation Pretax profit

$1,512,000 (862,400) (450,000) $ 199,600

Extending the depreciable lives of fixed assets by 2 years could increase this profit by $15,000 to $214,600, well above the target. But doing so would be manipulating the accounting system to achieve desirable results. When the estimates of depreciable lives were first made, there may have been much uncertainty in the estimates. However, changing the accounting method to make the financial results look better is an ethical violation. Managers should not change accounting methods just to make their performance look better (or in this case, to save their job). Although changing the depreciation schedule is not ethical, it is easy to see how the budgeting process creates an incentive for such unethical behavior. If the budget and reporting process makes excellent performance appear deficient, there may be great temptation for managers to cheat the system. 7-41

(50-90 min.) Amounts are in dollars.

1 and 2. See Exhibit 7-41A on the following two pages. This spreadsheet is constructed so that only formulas are entered in the disbursements and operating income schedules. You can compare the total operating income figures at the bottom of each spreadsheet to assess the effects of each scenario. 3.

..

See Exhibit 7-41 B on the following pages.

304


EXHIBIT 7-41A SPEEDY-MART STORE, NORTHCENTER MALL Spreadsheet for Profit Planning, Parts 1 & 2 Table of Budget Data June July August Sales forecasts 375,000 330,000 420,000 Sales increase (decrease) 0% 0% 0% 375,000 330,000 420,000 Cost of goods sold percentage 70% Misc. expense percentage 6% Sales commissions 10% Employee salaries per month 22,000 Rent per month 6,000 Insurance expense per month 450 Depreciation per month 2,850 Disbursements for Operating Expenses (2a) Cost of goods sold 262,500 Commissions 37,500 Salaries 22,000 Miscellaneous 22,500 Rent 6,000 Total 350,500

..

231,000 33,000 22,000 19,800 6,000 311,800

294,000 42,000 22,000 25,200 6,000 389,200

787,500 112,500 66,000 67,500 18,000 1,051,500

305


EXHIBIT 7-41A (Continued) Operating Income (2b) Sales Cost of goods sold Gross margin Operating expenses Commissions Salaries Miscellaneous Rent Insurance Depreciation Total Operating income

..

June 375,000 262,500 112,500

July 330,000 231,000 99,000

August 420,000 294,000 126,000

Total 1,125,000 787,500 337,500

37,500 22,000 22,500 6,000 450 2,850 91,300

33,000 22,000 19,800 6,000 450 2,850 84,100

42,000 22,000 25,200 6,000 450 2,850 98,500

112,500 66,000 67,500 18,000 1,350 8,550 273,900

21,200

14,900

27,500

63,600

306


Table of Budget Data Sales forecasts Sales increase (decrease)

EXHIBIT 7-41B SPEEDY-MART STORE, NORTHCENTER MALL Spreadsheet for Sensitivity Analysis, Part 3a June July August 375,000 330,000 420,000 5% 5% 5% 393,750 346,500 441,000

Cost of goods sold percentage Misc. expense percentage Sales commissions Employee salaries per month Rent per month Insurance expense per month Depreciation per month Disbursements for Operating Expenses Cost of goods sold Commissions Salaries Miscellaneous Rent Total

..

70% 6% 10% 22,000 6,000 450 2,850

275,625 39,375 22,000 23,625 6,000 366,625

242,550 34,650 22,000 20,790 6,000 325,990

308,700 44,100 22,000 26,460 6,000 407,260

Total 826,875 118,125 66,000 70,875 18,000 1,099,875

307


EXHIBIT 7-41B Part 3a (Continued) Operating Income Sales Cost of goods sold Gross margin Operating expenses Commissions Salaries Miscellaneous Rent Insurance Depreciation Total Operating income

..

June 393,750 275,625 118,125

July 346,500 242,550 103,950

August 441,000 308,700 132,300

Total 1,181,250 826,875 354,375

39,375 22,000 23,625 6,000 450 2,850 94,300 23,825

34,650 22,000 20,790 6,000 450 2,850 86,740 17,210

44,100 22,000 26,460 6,000 450 2,850 101,860 30,440

118,125 66,000 70,875 18,000 1,350 8,550 282,900 71,475

308


EXHIBIT 7-41B (Continued) Part 3b Table of Budget Data Sales forecasts Sales increase (decrease)

June 375,000 -2% 367,500

July 330,000 -2% 323,400

Cost of goods sold percentage Misc. expense percentage Sales commissions Employee salaries per month Rent per month Insurance expense per month Depreciation per month Disbursements for Operating Expenses Cost of goods sold Commissions Salaries Miscellaneous Rent Total

..

August 420,000 -2% 411,600 70% 6% 0% 52,500 6,000 450 2,850

257,250 0 52,500 22,050 6,000 337,800

226,380 0 52,500 19,404 6,000 304,284

288,120 0 52,500 24,696 6,000 371,316

Total 771,750 0 157,500 66,150 18,000 1,013,400

309


EXHIBIT 7-41B Part 3b (Continued) Operating Income Sales Cost of goods sold Gross margin Operating expenses Commissions Salaries Miscellaneous Rent Insurance Depreciation Total Operating income

..

June 367,500 257,250 110,250

July 323,400 226,380 97,020

August 411,600 288,120 123,480

Total 1,102,500 771,750 330,750

0 52,500 22,050 6,000 450 2,850 83,850 26,400

0 52,500 19,404 6,000 450 2,850 81,204 15,816

0 52,500 24,696 6,000 450 2,850 86,496 36,984

0 157,500 66,150 18,000 1,350 8,550 251,550 79,200

310


7-42

(50-90 min.)

1. See Exhibit 7-42A on the following two pages. The spreadsheet below contains data from the problem in the top of the spreadsheet space. Computations of operating expenses are accomplished with formulas that reference the table. Comparing the summary calculations of operating expenses (labeled TOTAL OPERATING EXPENSE) allows the user to assess the effects of alternate scenarios. 2. See Exhibit 7-42B on the following pages. 3. See Exhibit 7-74C on the following pages. Spreadsheet for Operating Expense Budget Cost behavior

..

Fixed

50” Displays

Variable per display $240

42” Displays

$165

Assembly labor

$40,000

48

Packaging

$8,000

4

Shipping

$5,000

2

311


EXHIBIT 7-42A HIGH DEFINITION LCD DIVISION Sales forecasts

50” Displays

42” Displays

Sales mix

1

1.25

Sales change

1

1

1-Oct

3,200

3,200

4,000

2-Nov

2,400

2,400

3,000

3-Dec

5,600

5,600

7,000

4-Jan

3,200

3,200

4,000

5-Feb

3,200

3,200

4,000

6-Mar

2,400

2,400

3,000

7-Apr

2,400

2,400

3,000

8-May

2,800

2,800

3,500

Month

October November December January February March Totals

..

Components $1,428,000 1,071,000 2,499,000 1,428,000 1,428,000 1,071,000 $8,925,000

OPERATING EXPENSES Assembly Packaging Shipping $ 385,600 $ 36,800 $ 19,400 299,200 29,600 15,800 644,800 58,400 30,200 385,600 36,800 19,400 385,600 36,800 19,400 299,200 29,600 15,800 $2,400,000 $228,000 $120,000

Total $ 1,869,800 1,415,600 3,232,400 1,869,800 1,869,800 1,415,600 $11,673,000

312


Sales forecasts

EXHIBIT 7-42B Revised Budget for 10% Sales Decrease 50” Displays Sales mix

1

1.25

Sales change

0.9

0.9

2-Nov

2,400

2,160

2,700

3-Dec

5,600

5,040

6,300

4-Jan

3,200

2,880

3,600

5-Feb

3,200

2,880

3,600

6-Mar

2,400

2,160

2,700

7-Apr

2,400

2,160

2,700

8-May

2,800

2,520

3,150

Month

November December January February March April Totals

..

42” Displays

Components $ 963,900 2,249,100 1,285,200 1,285,200 963,900 963,900 $7,711,200

Assembly $ 273,280 584,320 351,040 351,040 273,280 273,280 $2,106,240

OPERATING EXPENSES Packaging Shipping Total $ 27,440 $ 14,720 $ 1,279,340 53,360 27,680 2,914,460 33,920 17,960 1,688,120 33,920 17,960 1,688,120 27,440 14,720 1,279,340 27,440 14,720 1,279,340 $203,520 $107,760 $10,128,720

313


Sales forecasts

EXHIBIT 7-42C Revised Budget for Increased Sales of 42” Displays 50” Displays Sales mix

1

1.5

Sales growth

0.9

0.9

3-Dec

5,600

5,040

7,560

4-Jan

3,200

2,880

4,320

5-Feb

3,200

2,880

4,320

6-Mar

2,400

2,160

3,240

7-Apr

2,400

2,160

3,240

8-May

2,800

2,520

3,780

Month

December January February March April May Totals

..

42” Displays

Components $2,457,000 1,404,000 1,404,000 1,053,000 1,053,000 1,228,500 $8,599,500

Assembly $ 644,800 385,600 385,600 299,200 299,200 342,400 $2,356,800

OPERATING EXPENSES Packaging Shipping Total $ 58,400 $ 30,200 $ 3,190,400 36,800 19,400 1,845,800 36,800 19,400 1,845,800 29,600 15,800 1,397,600 29,600 15,800 1,397,600 33,200 17,600 1,621,700 $224,400 $118,200 $11,298,900

314


7-43

(80-100 min.)

1. On January 1, Salt Lake Light Opera needs to borrow $2,057,000, on April 1 it needs an additional $562,000, on September 31 it can repay $2,014,000, but on October 1 it must again borrow $726,000. This can be seen from the following analysis (in thousands of dollars): Qtr. 1 Qtr. 2 Qtr. 3 Qtr. 4 Beginning cash balance 208 200 200 200 Minimum cash balance desired 200 200 200 200 Available cash balance 8 0 0 0 Cash receipts & disbursement: Collections from customers (1) 883 1,893 4,504 2,024 Payments for supplies (2) (780) (200) Other expenses (3) (30) (30) (30) (30) Payments for payroll (4) (2,046) (2,100) (2,100) (2,100) Major equipment (5) (100) (300) Small equipment (6) (60) (60) (60) (60) Mortgage principal (7) (125) (125) Mortgage interest (8) (140) (135) Interest on working capital (9) (32) Net cash receipts & disbursements (2,065) (562) 2,014 (726) Excess (deficiency) of cash before financing (2,057) (562) 2,014 (726) Financing: Borrowing (at beginning of quarter) 2,057 562 726 Repayment (at end of quarter) (2,014) Total cash increase (decrease) from financing 2,057 562 (2,014) 726 Ending cash balance 200 200 200 200 Explanations: (1) Collections are revenues for the quarter less the increase (or plus the decrease) in accounts receivable. (2) Payments for supplies in the first quarter are the accounts payable carried over from 20X4 and in the third quarter (July) are the purchases in June. December’s purchases will be paid for in 20X6. (3) Other expenses are $10,000 per month, paid as incurred. (4) Payroll payments in the first quarter are those of December 20X4 ($646,000) plus the $700,000 from each of January and February. Each other quarter they are three $700,000 payments. (5) $100,000 of major equipment payments are made in September, October, November, and December. (6) Small equipment payments are $20,000 each month. (7) The mortgage payments semi-annually are $4,000,000 ÷ 32 = $125,000. (8) $3,500,000 × .04 = $140,000; $3,375,000 × .04 = $135,000. (9) The $32,000 payment is the interest that was payable at the end of 20X4. ..

315


The result of 20X5 operations will be an increase in the working capital loan from $1,588,000 (without the accrued interest) to $3,308,000, an increase of $1,720,000: Qtr. 1 $1,588 91 2,057 $3,736

Beginning loan Accrued interest (rounded*) Additional borrowing Ending loan

Qtr. 2 $3,736 108 562 $4,406

*The unrounded amounts of quarterly interest expense @8% are: Qtr. 1 Qtr. 2 Qtr. 3 $91,125 $107,453 $110,139

Qtr. 3 $4,406 110 (2,014) $2,502

Qtr. 4 $2,502 80 726 $3,308

Qtr. 4 $80,693

Total $389,410

2. Salt Lake Light Opera’s projected income statement and balance sheet for 20X5 are (in thousands): SALT LAKE LIGHT OPERA Budgeted Income Statement For the Year Ended December 31, 20X5 Revenues Expenses: Salary & wages Supplies Depreciation Other Total expenses Operating margin Interest: Mortgage Loan Net income

$11,059 $8,400 800 500 120 9,820 1,239 275 389

664 $ 575

SALT LAKE LIGHT OPERA Budgeted Balance Sheet December 31, 20X5 Assets Cash Receivables Supplies inventory Total current assets Fixed assets Total assets

..

$

200 6,195 600 6,995 5,949 $12,944

Liabilities & Equities Loan payable Accrued interest payable Accounts payable Payroll payable Current mortgage Total current liabilities Mortgage payable Total liabilities Net assets Total liabilities & equities

$ 2,919 389 700 700 250 4,958 3,000 7,958 4,986 $12,944

316


3. Salt Lake Light Opera has a net income of $575,000 but a shortfall in cash requiring borrowing of $1,720,000 ($1,331,000 borrowed plus accrued interest of $389,000). This is not uncommon for a growing organization. However, it is borrowing on a short-term basis via a working capital loan, while the need seems to be a long-term need. The $640,000 of investment is clearly long-term, but the $449,000 needed for operations also appears to be a long-term need unless receivables can be collected more quickly. Therefore, SLLO should consider additional long-term borrowing, possibly a second mortgage. The organization is in danger of defaulting on its loan because it cannot meet the condition that the loan must be paid off at least once a year, so it needs a loan without such a stipulation.

..

317


7-44

(40-60 min.)

1.

HIGHLINE HOSPITAL Budgeted Cash Receipts For the Quarter Ending September 30, 20X7 (in thousands)

May: 3rd-party billings May: patient billings June: 3rd-party billings June: patient billings June: 3rd-party billings June: patient billings July: 3rd-party billings July: patient billings July: 3rd-party billings July: patient billings July: 3rd-party billings July: patient billings August: 3rd-party billings August: patient billings August: 3rd-party billings August: patient billings Sept: 3rd-party billings Sept: patient billings Total receipts from billings Endowment fund income Total cash receipts 2.

Salaries:

Calculation .9 × 5700 × .2 .1 × 5700 × .4 .9 × 6000 × .2 .1 × 6000 × .4 .9 × 6000 × .5 .1 × 6000 × .4 .9 × 5800 × .2 .1 × 5800 × .4 .9 × 5800 × .5 .1 × 5800 × .4 .9 × 5800 × .2 .1 × 5800 × .1 .9 × 6200 × .5 .1 × 6200 × .4 .9 × 6200 × .2 .1 × 6200 × .1 .9 × 6600 × .2 .1 × 6600 × .1

August

September

$1,080 240 2,700 240 $ 1,044 232 2,610 232 1,044 58 2,790 248 1,116 62 $5,296 210 $5,506

$5,340 210 $5,550

1,188 66 $5,568 210 $5,778

Budgeted Cash Disbursements For the Quarter Ending September 30, 20X7 (in thousands)

$1,800 + (.2 × $5,800) $1,800 + (.2 × $6,200) $1,800 + (.2 × $6,600) Purchases, previous month Interest expenses Total cash disbursements

..

July $1,026 228

July $2,960

August

September

$3,040 1,450

1,500

$3,120 1,800 540

$4,410

$4,540

$5,460

318


3.

Budgeted Cash Receipts and Disbursements For the Third Quarter, 20X7 (in thousands)

Beginning cash balance Budgeted cash receipts ($5,506 + $5,550 + $5,778) Less budgeted cash disbursements ($4,410 + $4,540 + $5,460) Budgeted cash balance, September 30, 20X7 Minimum cash balance (.1 × $2,200) Cash available for capital expenditures Budgeted capital expenditures Borrowing needed on October 1, 20X7 7-45

$

350 16,834 (14,410) $ 2,774 (220) $ 2,554 (4,000) $ (1,446)

(50-60 min.)

1.

NEBRASKA STATE UNIVERSITY Projected Enrollment, Credits, and Faculty Academic Year 20X7-20X8 Expected enrollmenta Average credit hours Total credit hoursb Full-time-equivalent enrollmentc Credit hours per faculty memberd Total faculty needede

Undergraduate 3,528 25 88,200 2,940 720 122.5

Graduate 1,890 20 37,800 1,575 360 105

Total 5,418 126,000 4,515 1,080 227.5

a

98% × 3,600 = 3,528; 105% × 1,800 = 1,890 25 × 3,528 = 88,200; 20 × 1,890 = 37,800 c 88,200 ÷ 30 = 2,940; 37,800 ÷ 24 = 1,575 d 24 × 30 = 720; 18 × 20 = 360 e 88,200 ÷ 720 = 122.5; 37,800 ÷ 360 = 105 b

2.

NEBRASKA STATE UNIVERSITY Faculty Salaries Budget Academic Year 20X7-20X8

Undergraduate Graduate Total

..

Faculty Needed 122.5 105.0 227.5

Average Salary $61,480 61,480

Total Faculty Salaries $ 7,531,300 6,455,400 $13,986,700

319


3.

NEBRASKA STATE UNIVERSITY Tuition and Legislative Revenue Budget Academic Year 20X7-20X8

Total credit hours Less: Scholarship credit hours* Tuition paying credit hours Tuition per credit hour Total tuition budget Full time equivalent students Legislative apportionment per full-time equivalent student Total legislative apportionment

Undergrad Division 88,200 900 87,300 × $92 $8,031,600 2,940

Graduate Division 37,800 1,200 36,600 × $92 $3,367,200 1,575

Total 126,000 2,100 123,900 × $92 $11,398,800 4,515

× $780 $2,293,200

× $780 $1,228,500

× $780 $3,521,700

*30 × 30 = 900; 50 × 24 = 1,200 4.

NEBRASKA STATE UNIVERSITY Annual Budget Shortfall Academic Year 20X7-20X8

Budgeted operating expenditures: Faculty salaries Operation and maintenance of facilities: Salaries and wages (1.06 × $240,000) Other ($260,000 + $12,000) General Administrative Library: Acquisitions Operations Health Services Intramural athletics Intercollegiate athletics Insurance and retirement Interest Total budgeted operating expenditures Budgeted revenues: Tuition Legislative apportionment Endowment income Auxiliary services Intercollegiate athletics Total budgeted operating revenues Deficit from operations Budgeted capital expenditures Total cash needed from fund-raising ..

$13,986,700 254,400 272,000 525,000 155,000 200,000 50,000 60,000 245,000 560,000 75,000 $16,383,100 $11,398,800 3,521,700 210,000 335,000 300,000 $15,765,500 $ 617,600 575,000 $ 1,192,600

320


7-46 (30 min.) Amounts are in millions. Revenue in Fiscal 2011 was $20,862 S&A Expense in 2011 was $6,693 1. (a) 10% revenue increase Revenue $22,948 Cost of Sales 12,621 Gross Margin $10,327 S&A Expense 7,343 Income before income taxes $ 2,984 Income tax expense 746 Net income $ 2,238 2. Revenue Cost of Sales Gross Margin S&A Expense Income before income taxes Income tax expense Net income

(a) 10% revenue increase $22,948 12,621 $10,327 6,693 $ 3,634 909 $ 2,725

(b) 10% revenue decrease $18,776 10,327 $ 8,449 6,008 $ 2,441 610 $ 1,831 (b) 10% revenue decrease $18,776 10,327 $ 8,449 6,693 $ 1,756 439 $ 1,317

In part 1, where all costs are variable, net income increases or decreases in proportion to the change in revenue. In part 2, where S&A costs are fixed, net income increases or decreases by more than the proportional change in revenue. 3. Revenue Cost of Sales Gross Margin S&A Expense Income before income taxes Income tax expense Net income

(a) Gross Margin 46% $20,862 11,265 $ 9,597 6,693 $ 2,904 726 $ 2,178

(b) Gross Margin 44% $20,862 11,683 $ 9,179 6,693 $ 2,486 622 $ 1,865

Note how a small percentage change in gross margin translates into a large percentage change in net income.

..

321


7-47 For the solution to this Excel Application Exercise, follow the step-by-step instructions provided in the textbook chapter. 1.

The first month that cash receipts exceed cash expenditures in January 20X9.

2 & 3. Expenditures before January 20X9 total $1,220,000. Venture capital of only $860,000 is required because there will be cash receipts of $360,000 in 20X8. 7-48

(30-60 min.) The purpose of this exercise is to prepare a budget for an organization (an individual student) that is familiar to all students and to see the effect of assumptions on the budget. Each student will have some ideas about both the revenue and expense budgets of a typical student, but these ideas will likely vary across students. They will experience the process of negotiation needed to get a budget that the group can agree on. When the groups get together and compare budgets, it will be instructive to see how different groups make different assumptions that lead to different budgets. This should reinforce the importance of assumptions to the budget process and show how decisions made during the budget process affect the resulting budget. 7-49 (30-45 min.) NOTE TO INSTRUCTOR. This solution is based on the 2011 10K, which was the most recent set of annual results available on the web site in late 2012 when the book went to press. Be sure to examine the current web site before assigning this problem, as the information there may have changed. 1.

There are 10 brand lines under the corporation shell. They are Carnival Cruise Lines, Holland America Line, Princess Cruises and Seabourn in North America; P&O Cruises and Cunard Line in the United Kingdom; AIDA in Germany; Costa Cruises in Southern Europe; Ibero cruceros in Spain; and P&O Cruises in Australia. Each of the lines focuses on a different part of the world or offers a different class of cruise. Different names allow for the association or branding of a particular line with a particular type of cruise. For instance, Cunard focuses on the traditional ocean liner experience while Carnival focuses on more of a festive atmosphere on board ship – that is, fun times. The corporation also operates several tour companies.

2.

Total revenues in fiscal 2011 were $15.793 billion. The occupancy percentage was 106.2%, which is given in the Management’s Discussion and Analysis section of the report. How can one explain occupancy greater than 100%? For Carnival, it means that capacity is defined as two persons per room, yet many rooms have capacity of three or more persons, so when more than two persons occupy a room capacity utilization is greater than 100%.

..

322


3.

According to the Executive Overview, passenger capacity will increase by 3.4% during 2012, the net effect of adding new ships and replacing older ships. Assuming that revenue increases in proportion to capacity, budgeted revenue for 2012 would be: 1.034 × $15,793 million = $16,330 million With added revenue from additional capacity there would be added costs. Variable costs would probably increase proportionately with the volume but fixed costs are more complicated. Fixed costs of added capacity would increase, and the increase might be at least roughly proportional to the increase in revenue. Other fixed costs, for example corporate headquarters costs, might not increase much if at all, which would tend to make the increase in total costs less than proportional to the increase in capacity.

4.

The prices for cruises of the same length to the same location are not all the same. They differ according to when the cruise dates are – high season or low season – and also according to the level of capacity utilization that the particular cruise has achieved. The firm’s goal is to have the maximum capacity utilization possible for each cruise. If demand is high for a particular cruise, then the firm will be able to command a higher price and still fill the cabins. Since much of the cost of the cruise is likely to be fixed in nature, the firm will incur the cost even if they don’t fill the cabins. Thus, a price that covers variable cost and contributes to fixed costs will be preferred to an empty cabin. Destinations or dates that are less popular are cheaper because once the company schedules and commits to a cruise, it is in its best interests to fill as many cabins as possible as long as the price is above the variable costs. Last-minute deals can be especially cheap if a particular cruise is likely to have excess capacity.

..

323


CHAPTER 8 COVERAGE OF LEARNING OBJECTIVES

LEARNING OBJECTIVE LO1: Identify variances and label them favorable or unfavorable. LO2: Distinguish between flexible budgets and static budgets. LO3: Use flexible-budget formulas to construct a flexible budget based on the volume of sales. LO4: Compute and interpret static-budget variances, flexible-budget variances, and sales activity variances LO5: Understand how the setting of standards affects the computation and interpretation of variances LO6: Compute and interpret price and quantity variances for materials and labor.

LO7: Compute variable overhead spending and efficiency variances. LO8: Compute the fixed overhead spending variance.

..

CASES, EXCEL, FUNDAMENTAL ADDITIONAL COLLAB., & ASSIGNMENT ASSIGNMENT INTERNET MATERIAL MATERIAL EXERCISES

A1 A1, B3

A1, A2, B1, B3

2, 24, 24, 25, 25, 52, 53, 55, 57, 26, 27, 39, 39, 58, 59 41, 42, 43, 43, 49, 50 34, 34, 35, 35, 52, 54, 55, 57 36, 36, 37, 37, 38, 39, 39, 43, 43, 50, 50

B3

11, 18, 49

56, 58

A3, B2, B3

29, 30, 30, 31, 31, 32, 32, 33, 33, 39, 39, 40, 41, 43, 43, 44, 44, 45, 45, 46, 47, 48, 48, 50, 50, 51 17, 40, 41, 44, 44, 45, 45, 46, 47, 48, 48

55, 57, 58

B3

51

B3

324


CHAPTER 8 Flexible Budgets and Variance Analysis 8-A1

(30-45 min.) Amounts are in thousands.

1. Revenue

Flexible Budget Amounts $12,600 $13,000 $13,400

Fuel Repairs and maintenance Supplies and miscellaneous Variable payroll Total variable costs

$

504 378 2,016 7,812 $10,710

520 390 2,080 8,060 $11,050

536 402 2,144 8,308 $11,390

Supervision Rent Depreciation Other fixed costs Total fixed costs

$

$

$

$

Total costs Operating income

$11,700 $ 900

160 200 460 170 990

$

$

160 200 460 170 990

$12,040 $ 960

$

$

160 200 460 170 990

$12,380 $ 1,020

2.

Cost = $990,000 per quarter plus .85 of revenue = $990,000 + .85 × Revenue

3.

Variances are defined as deviations of actual results from plans. The total variances in the problem can be subdivided to provide answers to two broad questions: (a) What portion is attributable to not attaining a predetermined level of volume or activity? When volume is measured in terms of sales, this variance is called the sales-activity variance. (b) What portion is attributable to non-volume effects? This variance is often called the flexible-budget variance. The existing performance report, which is based solely on a static budget, cannot answer these questions clearly. It answers (a) partially, because it compares the revenue achieved with the original targeted revenue. But the report fails to answer (b). A more complete analysis follows:

..

325


Summary of Performance (in thousands) (1) (2) Actual =(1)-(3) Results at Actual FlexibleActivity Budget Level Variances Net revenue $12,700 $ Total variable costs 10,924 129U Contribution margin $ 1,776 $129U Fixed costs 1,000 10U Operating income $ 776 $139U U = Unfavorable

(3) Flexible Budget for Actual Sales Activity $12,700 10,795 $ 1,905 990 $ 915

(4) =(3)-(5) SalesActivity Variances $300 U 255 F $ 45 U $ 45 U

(5)

Static Budget $13,000 11,050 $ 1,950 990 $ 960

Column (4) focuses on the effects of sales volume. It shows that a $300,000 drop in sales activity is expected to cause a $45,000 decrease in contribution margin and hence a $45,000 decrease in operating income. Column (2) generally focuses on efficiency. Without a flexible budget, operating inefficiencies cannot be isolated from the effects of changes in sales activity. Cost control performance may be reported in more detail, where the focus is on efficiency (in thousands):

..

326


8-A2 (20-30 min.) This analysis of flexible budget and static budget variances follows Exhibit 8-6. Actual Results at Actual Activity Level (1) Systems consulting, variable

$46,000

Flexible Budget for Actual Sales Activity (2)

Static Budget (3)

90 requests × $500 = 75 requests × $500 = $45,000 $37,500 Flexible-budget Sales-Activity variance variance (2)-(3) (1) – (2) $45,000 - $37,500 = $46,000 - $45,000 = $7,500 U $1,000 U Static budget variance (1)-(3) $46,000 -$37,500 = $8,500 U

Systems consulting, fixed

$78,000 (given)

$65,000

$65,000

Flexible-budget Sales-Activity variance variance (2)-(3) (1) – (2) $65,000 - $65,000 = $78,000 - $65,000 = -0$13,000 U Static budget variance (1)-(3) $78,000 - $65,000 = $13,000 U Note that the activity-level variance for fixed costs is always zero (as long as the activity stays within the relevant range) because flexible- and static-budget fixed costs are always the same.

..

327


8-A3

(20 - 30 min.)

1.

Direct materials: Direct labor: Total

2.

The flexible budget is based on actual output achieved, not scheduled or budgeted output.

Direct Materials

$ 50.00 250.00 $300.00

C Flexible Budget: Standard Input Actual Cost Incurred: Quantities Allowed for Actual Input Quantities Actual Input Quantities Outputs Achieved × Standard Prices × Actual Prices × Standard Prices $xxx $yyy $zzz Price variance Quantity variance (A - B) (B - C) Flexible-budget variance (A - C) A

In general:

5 lb. × $10.00 = 10 hrs. × $25.00 =

B

3,100 lbs × $9.00 = 3,100 lbs × $10.00 = 525 units × 5 × $10.00 $27,900 $31,000 = $26,250 Price variance Quantity variance (A - B) = (B - C) $27,900 - $31,000 = $31,000 - $26,250 = $3,100 F $4,750 U Flexible-budget variance (A - C) $27,900 - $26,250 = $1,650 U

A B C Direct Labor 5,500 hrs × $26.00 5,500 hrs × $25.00 525 units × 10 hrs × = $143,000 = $137,500 $25.00 = $131,250 Price variance Quantity variance (A - B) = (B - C) = $143,000 - $137,500 = $137,500 - $131,250 = $5,500 U $6,250 U Flexible-budget variance (A - C) $143,000 - $131,250 = $11,750 U

..

328


3.

..

Among the possible explanations for the performance are: (a)

Were substandard materials used because they were cheaper, resulting in higher waste than usual? (Note that the unfavorable quantity variance more than offset the favorable price variance and resulted in a net unfavorable materials variance.)

(b)

Net savings in material costs may be undesirable if they cause inefficient use of direct labor. It is possible that use of substandard materials led to increased use of direct labor and the unfavorable direct labor quantity variance.

(c)

Direct labor is expensive. A wage rate that is about 4% above the standard rate creates a significant dollar amount of direct-labor price variance.

329


8-B1

(15-20 min.)

1.

ROBERT CAMPBELL TAX SERVICES Summary Performance Report

Physical units (clients) Sales Variable costs Contribution margin Fixed costs Operating income 2.

..

Actual Results at Actual Activity Level

Flexible Budget Variances

Flexible Budget for Actual Activity Level

Sales Activity Variances

3,000 $1,080,000 920,000

$30,000F 20,000U

3,000 $1,050,000 900,000

500F 3,500 $175,000U $1,225,000 150,000F 1,050,000

$ 160,000 159,500 $ 500

$10,000 F 9,500U $ 500 F

$ 150,000 150,000 $ 0

$ 25,000U $ 175,000 150,000 $ 25,000U $ 25,000

Static budget operating income Variances: Sales activity variance Flexible-budget variance Static-budget variance Actual operating income

Static Budget

$25,000 $ 25,000U 500F $

24,500U 500

330


8-B2 (20-30 min.) 1. A

Direct Materials

Direct Labor

B

C Flexible Budget: Actual Cost Incurred: Standard Input Actual Input Quantities Allowed Quantities Actual Input Quantities for Outputs Achieved × Actual Prices × Standard Prices × Standard Prices 116,000 lb × $7.50 = 116,000 lb × $7.00 = 14,400 units × 10 × $870,000 $812,000 $7.00 = $1,008,000 Price variance Usage variance (A - B) = (B - C) $870,000 - $812,000 = $812,000 - $1,008,000 = $58,000 U $196,000 F Flexible-budget variance (A - C) = $870,000 - $1,008,000 = $138,000F 29,000 hr × $12 = 29,000 hr × $14.00 = 14,400 units × 2 hr × $348,000 $406,000 $14.00 = $403,200 Price variance Usage variance (A - B) = (B - C) $348,000 - $406,000 = $406,000 - $403,200 = $58,000 F $2,800 U Flexible-budget variance (A - C) $348,000 - $403,200 = $55,200 F

2.

Tradeoffs may have been made in each category. Materials more expensive than standard may have been acquired with the hope of achieving less waste. Lessskilled or less-experienced labor may have been used that cost less per hour, but the less-skilled or less-experienced workers may have required more hours to do the job. The overall effects on costs as measured by these variances were favorable. However, management should also consider the effects of these tradeoffs on quality, on-time delivery, customer satisfaction (and so on) that are not measured in the variances.

..

331


8-B3 (20-30 min.) If the total overhead incurred is $204,000, of which $138,000 is fixed, then variable overhead was $204,000 - $138,000 = $66,000. The following analysis should be helpful. All given items are designated by an asterisk (*). Computations for the derived items are explained in items 1-4 below.

Orderprocessing departmentvariable overhead

A

B

Actual Overhead Costs Incurred

Predicted Overhead Based on Actual Driver Use × Standard Prices

$66,000

$66,000 + $3,600* = $69,600;

C Flexible Budget: Standard Driver Use Allowed for Output Achieved × Standard Prices $70,000 - $9,600 = $60,000

116,000 hr × 10* × 100,000 hr × 10* × $.06* = $69,600 $.06* = 60,000 Spending variance Efficiency variance $3,600* F $9,600 U Flexible-budget variance (A - C) $6,000* U

1.

$9,600U. The efficiency variance is computed by subtracting the spending variance from the flexible-budget variance, $6,000 U – ($3,600 F).

2.

116,000 hours. The actual hours can be computed by adding the variable overhead spending variance to the actual variable overhead and then dividing the result by $.60: ($66,000 + $3,600) ÷ $.60 = 116,000 hours. Alternatively, this answer could be obtained by taking the answer in part (3) and adding 16,000 hours because the unfavorable efficiency variance represents 16,000 hours of work ($9,600 ÷ $.60).

3.

100,000 hours. The standard hours allowed for output achieved can be computed in one of two ways:

4.

..

(a)

Take the answer in part (2) and deduct 16,000 hours: 116,000 – 16,000 = 100,000 hours.

(b)

Deduct the efficiency variance from the $69,600 and then divide the result, $60,000, by $.60: ($69,600 - $9,600) ÷ $.60 = 100,000 hours.

Budgeted fixed overhead is equal to actual fixed overhead ($138,000) less the unfavorable fixed overhead spending variance ($2,500 U), or $135,500.

332


8-1 Favorable variances arise when actual costs are less than budgeted costs (or actual revenue exceeds budgeted revenue). Unfavorable variances mean that actual costs are greater than budgeted costs (or actual revenue falls short of budgeted revenue). 8-2 Master budgets are comprehensive, encompassing all the components of costs spread through the entire organization. However, they are not necessarily static, i.e., prepared for a single level of activity. They can also be flexible, i.e., prepared for a number of alternative levels of activity. 8-3 No. A flexible budget adjusts costs as the level of activity changes, not as prices change. 8-4 The use of flexible budgeting requires cost formulas or functions to predict what costs should be at different levels of cost driver activity. It is essential to understand cost behavior to develop these flexible-budget cost formulas. 8-5 No. A "flex" in a flexible budget generally refers to adjustments made because of changes in volume. Activities that drive variable costs will therefore generate "flexes" in the budget. Activities that do not drive changes in costs will not have a "flex." 8-6 No. Performance can be either effective or efficient, or both, or neither. For example, the targeted sales level may be achieved or not (effectiveness), independent of whether the actual level of operations minimized the amount of resources used (efficiency). 8-7 A static budget variance is the difference between the originally planned (static budget) amount and the actual amount. A flexible-budget variance is the difference between the actual amount and the amount that is expected for the actual level of output achieved. 8-8 Favorable and unfavorable variances do not necessarily mean good and bad performance, respectively, and therefore rewards and punishments should not necessarily follow favorable and unfavorable variances. Variances mean simply that actual results differed from the standards. These differences may arise from inaccurate standards, or they may be the result of factors that are beyond the control of management. Variances should be a signal to ask the question "Why did the difference arise?" but they do not automatically give the answer. 8-9 No. The primary function of a control system is explanation and understanding, not placing blame. 8-10 Sales-activity variances are often the responsibility of sales or marketing managers. However, if factors such as quality of product and meeting of delivery schedule impact the volume of sales, production managers, who affect quality and delivery, may also be responsible for the sales-activity variance. ..

333


8-11 Perfection standards are ideal standards that are most difficult to attain. There are no provisions for waste, spoilage, machine breakdowns, or other inefficiencies inherent in a normal system. As a result, adverse variances are hardly avoidable even if the employees put in their earnest efforts. Consequently, they have adverse effects on employee motivation and hence are not often advocated widely. 8-12 One approach sets standards just tight enough so that employees regard their fulfillment as probable if they exert normal effort and diligence. The second approach sets standards so tight that employees regard their fulfillment as possible though unlikely. 8-13 There is much room for measurement error when a standard is set. Consequently, random fluctuations around the standard can really be conceived of as defining the band of acceptable outcomes rather than as variances from a precise standard. The standard is often the midpoint of the band of acceptable outcomes. 8-14 Price variances separate out the effects of deviations of actual price from the standard price. Therefore, price variances should be computed even if prices are outside of company control. This helps managers to better understand and measure production performance by separating price effects from quantity effects. Following the usual approach to computing price and quantity variances, the quantity variances are not affected by deviations of price from the standard. 8-15 Some common causes of unfavorable quantity (or usage or efficiency) variances are improper handling, poor quality of material, poor workmanship, changes in methods, new workers, slow machines, breakdowns, and faulty designs. 8-16 Failure to meet price standards is often the responsibility of the purchasing officer, but responsibility may be shared with the production manager when he or she has frequent rush orders for materials that result in higher prices paid. Of course, market conditions may be such that it is beyond the control of anyone in the company to attain the price standard. 8-17 The variable-overhead variance depends on the deviation of actual quantity of cost drivers used from the budgeted quantity of cost drivers for actual production. It shows how control of cost drivers actively affects the variable overhead cost. 8-18 Though for critical items variances of any kind must always be investigated, it is not warranted for all items. This is because, even if everything goes well, for most of the items, variances are unlikely to be exactly zero. Therefore, for these items, managers specify acceptable ranges for variances based on economic analysis and clarify how big the variances must be before investigation would be justified. 8-19 The narrow interpretation of the unfavorable label is that, holding everything else equal, revenue being $2,000 lower than planned has an unfavorable effect on profit. ..

334


However, the unfavorable label for the revenue variance does not necessarily indicate that the decision to reduce revenue was incorrect. In this situation, the decision to lower revenue by $2,000 results in higher profit because the $2,000 loss in revenue is more than offset by the corresponding $2,500 difference in costs ($6,500 of costs to achieve $8,000 of revenue versus $4,000 of costs to achieve $6,000 of revenue). 8-20 The impact of changes in sales volume on profit depends on the amount of variable versus fixed costs. If sales (revenues) drop 10%, the contribution margin drops by 10% also, but fixed costs will not change (assuming the new sales volume remains in the relevant range). With operating profit of $100 on sales of $1,000, total costs must have been $900. Suppose half of those costs were fixed. Then, the contribution margin would be $1,000 - $450 = $550. A 10% drop in sales would reduce contribution margin (and profit) by 10% × $550 = $55, corresponding to a 55% reduction in profit. 8-21 Changes in production volume will affect variable costs but not fixed costs, provided that the new production level remains within the relevant range. If production volume increases by 10%, costs will increase by less than 10% if there are any fixed costs. Suppose that half of the production costs for 100 units are fixed and half are variable. That means that per unit variable costs are ($1,000 × .5) ÷ 100 = $5. Producing an extra 10 units should cause an extra cost of 10 × $5 = $50, giving a total cost of $1,050 for 110 units. The production manager should have a cost target of $1,050, not $1,100. 8-22 If a purchasing manager saves money by paying less per pound than planned, we want to make sure this savings did not come at the expense of quality. By examining the material usage variance, we can see whether more than planned of the cheaper material had to be used. Perhaps there was more scrap or waste because of using inferior materials. One might also examine the labor usage variance. Inferior materials may also be harder to handle, thus requiring additional labor time. Or, partially completed products might have to be scrapped when defects are found, wasting not only the materials put into the product but also the labor used up to the point it is scrapped. 8-23 Unfavorable variable-overhead efficiency variances arise when there is excess usage of the cost driver used to apply the variable overhead. To know who is responsible for the unfavorable variable-overhead efficiency variance we need to know the cost driver for the variable overhead and who is responsible for controlling that cost driver. If direct labor hours drives variable overhead, then the plant manager is responsible for the variable-overhead efficiency variance. Why? Because he or she is responsible for controlling use of the variable overhead cost-driver, direct labor hours.

..

335


8-24

(5 min.)

Variable cost is AUD 1,760,000 ÷ 88,000 units = AUD 20.0 per unit Budgeted cost = (AUD 20.0 × 110,000) + AUD 88,000 = AUD 2,288,000 8-25

(10 min.) Mileage Fuel @ £.25 Depreciation Total

8-26

30,000 £ 7,500 7,200 £14,700

65,000 £16,250 7,200 £23,450

(10 min.) Answers are in italics. Budget Formula per Unit Units Sales Variable costs: Direct material Hand labor Fixed costs: Depreciation Salaries

8-27

45,000 £ 11,250 7,200 £18,450

Various Levels of Output 10,000

11,000

12,000

$19

$190,000

$209,000

$228,000

$6.50 4.40

$65,000 $44,000

$71,500 $48,400

$78,000 $52,800

18,000 33,000

18,000 33,000

18,000 33,000

(10-15 min.)

The manager's delight is unjustified. A more informative analysis is obtained when a flexible budget is introduced:

Units of product Direct materials Direct labor Total

Actual Costs 5,300 $ 49,900 39,200 $89,100

FlexibleBudget Variance $ 7,500U 2,100U $9,600U

Flexible Budget 5,300 $ 42,400 37,100 $79,500

Sales Activity Variance 2,100U $16,800F 14,700F $31,500F

Static Budget 7,400* $ 59,200 51,800 $111,000

*$59,200 ÷ $8 = 7,400 or $51,800 ÷ $7 = 7,400 Note that the manager should have expected lower costs when actual volume was 2,100 units lower (about 28% lower) than the static budget. The flexible budget shows these expected costs were $31,500 lower than in the static budget. However, the manager was unable to bring the actual costs below the amounts in the flexible budget and spent $9,600 more than the flexible budget amounts. ..

336


8-28

(10-15 min.) A

B C Flexible Budget Actual Results for Actual at Actual Pounds of Static Activity Level Activity Budget $177,000 650,000 lb × $.25 = 750,000 lb × $.25 = (given) $162,500 $187,500 Flexible-budget Materials-activity variance (A - B) variance (B - C) $177,000 - $162,500 $162,500 - $187,500 = $14,500 U = $25,000 F Static-budget variance (A - C) $177,000-$187,500 = $10,500 F

Materials support:

8-29

(10-15 min.)

Actual Cost Incurred: Actual Input Quantities × Actual Prices 4,100 sq. yd × B686 = B2,812,600

Actual Input Quantities × Standard Prices

Flexible Budget: Standard Input Quantities Allowed for Outputs Achieved × Standard Prices

4,100 sq. yd. × B680 = B2,788,000

3,700 sq. yds. × B680 = B2,516,000

4,100 × (B686 – B680) = Price variance, B24,600U

..

(4,100 – 3,700) × B680 = Quantity variance, B272,000U

337


8-30

(15-20 min.)

The analytical framework showing only given items is: Actual Hours × Actual Price

Actual Hours × Standard Price

Standard Hours × Standard Price

3,560 hrs. 3,560 hrs. D hrs. × Actual Price × €20.00 × €20.00 = €A = €B = €C 3,560 × (Actual price - €20.00) (3,560 – D) × €20.00 = Price variance, €2,314 U = Quantity variance, €E Flexible-budget variance, €3,286 F 1. Given the price variance of €2,314 U and the actual hours of 3,560, the actual labor rate will be €2,314 ÷ 3,560 hours = €.65 / hour higher than the standard rate. Therefore, the actual labor rate per hour y = €20.00 + .65 = €20.65. 2. Items A, B, C, and D in the framework can now be completed as follows: A = 3,560 hours × €20.65 per hour =73,514. B = 3,560 hours × $20.00 per hour = 71,200. E = Quantity variance = Flexible Budget variance – Price Variance = 3,286 F – 2,314 U = 5,600 F C = 71,200 + quantity variance = 71,200 + 5,600 = 76,800. D = 76,800 ÷ 20.00 = 3,840 standard hours allowed. 8-31

(10 min.)

Material quantity (usage) variance = (Actual usage in kilos – Standard kilos) × Standard price = (28,050 actual sq. ft. – 29,250 standard sq. ft.) × $4.80 = $5,760, favorable Labor quantity (usage) variance = (Actual hours - Standard hours) × Standard price = (100,650 actual hours – 99,450 standard hours) × $12 = $14,400, unfavorable

..

338


8-32

(10-20 min.)

1.

Quantity variance = (Actual hours – Standard hours) × Standard price €18,000 = (Actual hours – 10,000) × €30.00 €18,000 = (Actual hours × €30.00) – €300,000 Actual hours = €318,000 ÷ €30.00 = 10,600 or Excess hours, €18,000 ÷ €30.00 600 Standard hours 10,000 Total actual hours 10,600

2.

Price variance = Actual quantity × (Actual price – Standard price / unit) €-1,000 = 2,500 × (Actual price – €9.20) €-1,000 = 2,500 × (Actual price) – €23,000 Actual price = €22,000 ÷ 2,500 = €8.80 or Standard price €9.20 Variance per unit, €1,000 ÷ 2,500 .40F Actual price €8.80

8-33

(10-15 min.)

Direct material: Price variance: £240,990 – £254,310 = £13,320 F Usage variance: £254,310 – £260,400 = £6,090 F Flexible-budget variance: £240,990 – £260,400 = £19,410 F Direct labor: Price variance: £202,510 – £192,500 = £10,010 U Usage variance: £192,500 – £185,000 = £7,500 U Flexible-budget variance: £202,510 – £185,000 = £17,510 U You may wish to call the students' attention to trade-offs. For example, more efficient use of materials may sometimes be attained by more careful work that takes more time than allowed by the labor standard.

..

339


8-34

(10-15 min.) (In million ₩) U = Unfavorable; F = Favorable (1) (2) (3) (4) =(1)-(3) =(3)-(5) Actual Flexible Budget Results Flexiblefor Actual Salesat Actual Budget Sales Activity Activity Level Variances Activity Variances Revenue ₩ 4,560 ₩ ₩ 4,560 ₩1,440U Variable costs 456 456 144F Contribution margin ₩ 4,104 ₩ ₩ 4,104 ₩1,296U Fixed costs 5,040 360F 5,400 Operating income ₩ (936) ₩300F ₩ (1,296) ₩1,296U

(5)

Static Budget ₩6,000 600 ₩5,400 5,400 ₩ 0

This is an example of a “high fixed cost” or “high operating leverage” organization. This means a high sensitivity of operating income in relation to changes in revenue. In this case, income plummeted when revenue dropped by 24% of the static budget. Note that leverage works both ways—if the change in revenue had been an increase of 24%, income would have soared. 8-35

(15-25 min.)

1. Attendees Revenue Cost of dinner Beverages Hotel rental Music Profit

Actual Results 90 ₹260,400 141,360 37,280 6,480 70,000 ₹ 5,280

FlexibleBudget Variance ₹8,400 F 4,560 U 5,920 F 0 10,000 U ₹ 240 U

Flexible Budget 90 ₹252,000 136,800 43,200 6,480 60,000 ₹ 5,520

Sales Activity Variance 15 F ₹42,000 F 22,800 U 7,200 U 0 0 ₹12,000 F

Static Budget 75 ₹210,000 114,000 36,000 6,480 60,000 ₹ (6,480)

2.

If all costs had behaved as budgeted, the extra 15 attendees would have produced an extra ₹12,000 of profit (₹42,000 more revenue and ₹22,800 + ₹7,200 = ₹30,000 more cost). The sales activity variance summarizes this effect of volume. Revenue was ₹8,400 over budget for the number of attendees—perhaps three tickets were sold to persons who did not attend. Costs ran ₹4,560 – ₹5,920 + ₹10,000 = ₹8,640 more than the flexible budget for 90 attendees. Dinner cost was ₹4,560 over budget; this is the cost of three dinners—again, perhaps three people who purchased the tickets did not attend but the caterer was still paid for their dinners. Beverages were under budget by ₹5,920. The band seems to have played (or at least was paid for) an extra half hour at their rate of ₹20,000 per hour.

..

340


8-36

(20-30 min.)

1.

Physical units Sales Variable costs Contribution margin Fixed costs Operating income

Actual Results at Actual Prices 60,000 $786,600c 378,000 $408,600d 81,600e $327,000f

FlexibleBudget Variances $ 6,600 F 18,000 U $11,400 U 9,600 U $21,000 U

Flexible Budget 60,000 $780,000b 360,000 $420,000 72,000 $348,000

Sales Activity Static Variances Budget a 12,000 F 48,000 $156,000 F $624,000 72,000 U 288,000g $ 84,000 F $336,000 72,000 $ 84,000 F $264,000

a

b 60,000 - 48,000 $624,000 ÷ 48,000 = $13 per unit; $13 × 60,000 = $780,000 c d $780,000 + $6,600 $786,600 - $378,000 e f $72,000 + $9,600 $408,600 – $81,600 g ($360,000 ÷ $780,000) × $624,000 = $288,000

2.

Sales were 12,000 units higher than originally budgeted. The flexible budget shows that this higher sales volume should have produced an operating income of $348,000 (up from $264,000 by the additional contribution margin of $84,000). However, only $327,000 was achieved. Sales prices were higher than the flexible budget amounts by $6,600, and costs exceeded the flexible budget by $18,000 + $9,600 = $27,600: Actual operating income Flexible Budget Variances: Sales price Variable costs Fixed costs Flexible budgeted income Sales-activity variance Static budgeted operating income

..

$327,000 $ 6,600 F 18,000 U 9,600 U

21,000 U $306,000 84,000 F $264,000

341


8-37

(20-30 min.)

Consultancies Sales @ £100 Variable costs Contribution margin Fixed costs Operating income

HORIZON CONSULTANTS Analysis of Income Statement For the Year 2020 (in thousands) (1) (2) (3) (1)-(3) FlexibleBudget Flexible Actual Variances Budget 350 350 £35,000 £ £35,000 10,950 450 U 10,500

(4) (3)-(5) Sales Activity Variances 50 U £5,000 U 1,500 F

Static Budget 400 £40,000 12,000*

£24,050 22,300

£ 450 U 300 U

£24,500 22,000

£3,500 U -

£28,000 22,000

£ 1,750

£ 750 U

£ 2,500

£3,500 U

£6,000

(5)

* Contribution margin = 70% × (400 × £100) = £28,000; £40,000 sales - £28,000 contribution margin = £12,000 variable costs. Note: The spending variance for fixed costs is a flexible-budget variance, not a sales activity variance. The variances in Column (4) are traceable solely to changes in volume: The effects of price and efficiency changes and any other deviations from the flexible budget are presented in Column (2). The £6,000,000 budgeted income was not attained because volume was down by 50,000 consultancies, causing a £3,500,000 shortfall in the contribution margin. In addition, the amount we paid for variable costs was £450,000 higher than the flexiblebudget standards, due to some combination of price and quantity variances that cannot be determined from the information given. Finally, we spent £300,000 in excess of our fixed-cost advertising budget.

..

342


8-38

(15-20 min.)

1.

The sales activity variance is RMB 990,000 favorable, and the flexible-budget variance is RMB 350,000 unfavorable. The following numbers are in millions of Chinese RMBs):

Sales Variable costs Contribution margin Fixed costs Income

Actual Results at Actual Activity Level 10.00 5.85 4.15 2.10 2.05

FlexibleBudget Variances

FlexibleBudget for Actual Sales Activity

SalesActivity Variances

Static Budget

--.35 U .35 U --.35 U

10.00 5.50 4.50 2.10 2.40

2.20 F 1.21 U .99 F --.99 F

7.80 4.29 3.51 2.10 1.41

The increase in sales volume should have increased income by RMB990,000, but it increased by only RMB 640,000 because of a RMB 350,000 unfavorable flexible-budget variance in variable costs. 2.

It is likely that the sales level of RMB 7,800,000 is within the relevant range, so the fixed costs are likely to stay at RMB 2,100,000. If we assume that the variable cost percentage returns to the 2012 budgeted level of 55%, the static budget for sales of RMB 7,800,000 forecasts income of (.45 × RMB 7,800,000) – RMB 2,100,000 = RMB 1,410,000. However, if the variable cost percentage remains at the 2012 actual level of 58.5%, the static budget for sales of RMB 7,800,000 would have income of (.415 × RMB 7,800,000) – RMB 2,100,000 = RMB 1,137,000.

..

343


8-39

(20-25 min.)

1.

Monetary amounts are in thousands of euro.

Millions of passenger miles Revenue Variable expenses Contribution margin Fixed expenses Operating income

Actual Results at Actual Prices

FlexibleBudget Variance

Flexible Budget

Sales Activity Variances

Static Budget

1,584 521,136** 342,960 178,176 114,360 63,816

33,264 U 17,400 F 15,864 U 8,760 U 24,624 U

1,584 264 F 554,400 92,400 F 360,360*** 60,060 U 194,040 32,340 F 105,600 88,440 32,340 F

1,320* 462,000 300,300 161,700 105,600 56,100

*462,000 ÷ €.35 = 1,320,000 **554,400 – (.06 × 554,400) = 521,136 ***300,300 × (1,584,000 ÷ 1,320,000) = 360,360 Actual Results at Actual Prices 2.

Fuel

95,040

FlexibleBudget Variance 15,840 U**

Flexible Budget

Sales Activity Variances

Static Budget

79,200

13,200 U*

66,000

*A 20% increase in passenger miles implies a 20% × €66,000 = €13,200 increase in fuel cost. **Price variance due to 20% increase in fuel prices is 20% × €79,200 = €15,840 U. Thus, the flexible budget variance for fuel expenses is €15,840 unfavorable.

..

344


8-40 (30-45 min.) The computations of variances are straightforward, although the context is different from that in the text. The explanation of variances is potentially complex and difficult. 1. Nursing price variance = Actual cost - (Actual hours × Standard price) = $33,180 - (2,080 × $15) = $33,180 - $31,200 = $1,980U Nursing hours quantity (or usage) variance = (Actual hours - Standard hours allowed) × Standard price = [2,080 - (4,000 × .5)] × $15 = 80 × $15 = $1,200U 2. Supplies and VOH Efficiency variance = (Actual hours - Std. hours allowed) × Variable overhead rate = (2,080 - 2,000) × $10 = $800U Supplies and VOH Spending variance = Actual cost - Actual hours × Variable overhead rate = $20,340 - (2,080 × $10) = $20,340 - $20,800 = $460F 3.

The nursing price and usage variances are unfavorable. This may be due to inefficient scheduling. More nurses are being used than are required according to the flexible budget, and a higher proportion than normal are in the high wage rate categories. But this might be exactly what is expected when volume increases. The nurses who perform the extra work that physicians usually handle are probably highly skilled, and, therefore, are paid more than average. Thus, the average pay rate for nurses increases. Further, as the hospital sees additional patients, nurses may bear nearly the entire added load. Why? Because physicians may already be at capacity. Therefore, although at a volume of 3,800 patients nurses average .5 hours per patient, they may put in more than .5 hours with each additional patient because they pick up some physician tasks as well as their own normal tasks. For example, suppose a physician spends an average of .5 hours on each patient case. Since additional physician time may not be available, nurses might average a full hour for each additional patient, the .5 hours that they normally spend plus the .5 hours a physician usually spends. It appears that Dr. Narr has controlled supplies and other variable costs quite well. Of the $1,340 unfavorable static budget variance, $1,000 is an activity variance due to the added volume. An additional $800 unfavorable variance is due to the use of extra nursing hours; that is, the variable overhead efficiency variance arises solely due to the use of extra nursing hours. The favorable spending variance for supplies and other variable overhead shows that spending is $460 less than expected.

..

345


8-41

(20-30 min.) This is an excellent basic problem in flexible budgeting.

1.

CNN LONDON MOTOR POOL Monthly Budget Report For March 20X1

Petrol (gasoline) Oil, minor repairs, parts and supplies Outside repairs Insurance Salaries and benefits Depreciation Totals

March Actual £ 8,200 2,540 50 416 1,800 1,976 £14,982

Monthly Flexible Budget £ 7,700 2,800 234 416 1,800 1,976 £14,926

Number of automobiles Actual kilometers Cost per kilometer

26 140,000 £ .1070

26 140,000 £ .1066

Under (over) £(500) 260 184 £ (56) £(.0004)

Supporting Calculations for Monthly Budget Amounts: Petrol: (140,000 actual km ÷ 8 km per liter) × £.44 per liter = £7,700 Oil, etc.: 140,000 km × £.02 per km = £2,800 Outside repairs: (£108 per auto × 26 autos) ÷ 12 months = £234 Insurance: Annual cost for one auto = £4,800 ÷ 25 autos = £192 per auto Annual cost for 26 autos = 26 × £192 = £4,992 Monthly cost = £4,992 ÷ 12 = £416 Salaries and benefits: No change, monthly cost = £21,600 annual cost÷12 months = £1,800 Depreciation: Annual depreciation per auto = £22,800 ÷ 25 autos = £912 Annual depreciation for 26 autos = £912 × 26 = £23,712 Monthly depreciation = £23,712 ÷ 12 = £1,976 2. Outside automobile repairs are a function of the use of the automobile over its lifetime. However, these repairs occur irregularly throughout the year and the life of the car. A monthly budget figure based upon a per-mile charge becomes questionable. Therefore, the use of one-twelfth of the estimated annual outside repair costs adjusted for the number of cars in operation during a month would appear to be more reasonable. Further, repairs probably occur more frequently in low-volume months, so the favorable variance in this high-volume month is not surprising.

..

346


8-42

(50-70 min.) The following notation applies to requirements 1-3. TC FI FC FB FV VI VC VB VV HR LR LN AV

1.

= Total cost = Fixed costs of the Account Inquiry activity center = Fixed costs of the Correspondence activity center = Fixed costs of the Account Billing activity center = Fixed costs of the Bill Verification activity center = Variable cost/labor hour in the Account Inquiry activity center = Variable cost/letter in the Correspondence activity center = Variable cost/line in the Account Billing activity center = Variable cost/account in the Bill Verification activity center = Account Inquiry labor hours = Letters of correspondence issued = Lines printed = Accounts verified

Activity Center

Flexible-Budget Formula

Account Inquiry

FI + VI × HR

Correspondence

FC + VC × LR

Account Billing

FB + VB × LN

Bill Verification

FV + VV × AV

(1) $79,910  3,300 (2) $ 9,800  2,800 2.

Variable Costs Fixed Costs Total Flexible Budget

= $156,380 + $24.22(1) × HR = $25,584 + $ 3.50(2) × LR = $81,400 + $ 0.063(3)× LN = $78,050 + $ 0.54(4) × AV (3) $154,377  2,440,000 (4) $10,797  20,000

Flexible Budget -- Account Inquiry Activity Center Budget Formula: $24.22/HR $156,380

Cost Driver: Number of Labor Hrs. (HR) 3,000 4,000 5,000 $ 72,660 $ 96,880 $121,100 156,380 156,380 156,380 $229,040 $253,260 $277,480

Flexible Budget -- Correspondence Activity Center

Variable Costs Fixed Costs Total Flexible Budget

..

Budget Formula: $3.50/LR $25,584

Cost Driver: Number of Letters (LR) 2,500 3,000 3,500 $ 8,750 $10,500 $12,250 25,584 25,584 25,584 $34,334 $36,084 $37,834

347


Flexible Budget -- Account Billing Activity Center

Variable Costs Fixed Costs Total Flex. Bud.

Budget Formula: $0.063/LN $81,400

Cost Driver: Number of Lines (LN) 2,000,000 2,500,000 3,000,000 $126,000 $157,500 $189,000 81,400 81,400 81,400 $207,400 $238,900 $270,400

Flexible Budget -- Bill Verification Activity Center

Variable Costs Fixed Costs Total Flex. Bud. 3.

Budget Formula: $0.54/AV $78,050

Cost Driver: Number of Accounts (AV) 15,000 20,000 25,000 $ 8,100 $10,800 $13,500 78,050 78,050 78,050 $86,150 $88,850 $91,550

TC = FI + FC + FB + FV + VI × HR + VC × LR + VB × LN + VV × AV = $341,414 + $24.22 × HR+ $3.50 × LR + $.063 × LN + $0.54 × AV

4. FlexibleBudget Variances

Activity Center

Actual Costs

Flexible Budget

Account Inquiry

$235,400

$25,126F

Correspondence

38,020

$260,526(1) 36,784(2)

Account Billing

285,000

17,750U

Bill Verification Total Costs

105,320 $663,740

267,250(3) 90,470(4)

(1) (2) (3) (4)

..

$655,030

1,236U 14,850U $ 8,710U

$156,380 + ($24.22 × 4,300) = $260,526 $25,584 + ($3.50 × 3,200) = $36,784 $81,400 + ($0.063 × 2,950,000) = $267,250 $78,050 + ($0.54 × 23,000) = $90,470

348


8-43

Direct materials

(25-30 min.) Flexible Budget Actual Flexible Budget Standard Inputs Cost Incurred: Based on Allowed for Actual Actual Inputs Actual Inputs Outputs Achieved × × Actual Prices × Standard Prices Standard Prices 6,600 lbs. × £0.97 6,600 lbs. × £1.00 6,000 lbs. × £1.00 = £6,402 = £6,600 = £6,000 6,600 × (£0.97 - £1.00) = (6,600 - 6,000) × £1.00 = Price variance £198 F Quantity variance £600 U Flexible-budget variance, £402 U

Direct labor

11,000 hrs. × £7.70 11,000 hrs. × £8.00 10,000 hrs. × £8.00 = £84,700 = £88,000 = £80,000 11,000 × (£7.70 - £8.00) = (11,000 -10,000) × £8.00 = Price variance £3,300 F Quantity variance, £8,000 U Flexible-budget variance, £4,700 U

Variable overhead

11,000 hrs. × £0.80 10,000 hrs. × £0.80 = £9,240 = £8,800 = £8,000 11,000 × (£0.84 - £0.80) (11,000 -10,000) × £0.80 = Spending variance,£440 U = Efficiency variance, £800 U Flexible-budget variance, £1240 U

U = Unfavorable, F = Favorable

..

349


8-44 (30-35 min.) The format of the solution may seem awkward at first, but students find that it provides perspective on the analysis of variances. 1. Actual Cost Incurred: Actual Inputs × Actual Prices

Actual Input Quantities × Standard Prices

Flexible Budget: Standard Input Quantities Allowed for Outputs Achieved × Standard Prices

Direct Materials: 495,000 lbs. × C$0.90 495,000 lbs. × C$0.85 450,000 lbs. × C$0.85 =C$445,500 = C$420,750 = C$382,500 495,000 × C$0.05= Price 45,000 × C$0.85 = Quantity variance, variance, C$24,750 U C$38,250 U Flexible-budget variance C$63,000 U ex Direct Labor: 825,000 hrs. × C$7.70 = C$6,352,500

825,000 hrs. × C$8.00 750,000 hrs. × C$8.00 = C$6,600,000 = C$6,000,000 825,000 × C$0.30 75,000 × C$8.00 = Price variance, Quantity variance, C$247,500 F C$600,000 U Flexible-budget variance, C$352,500 U

Variable Manufacturing Overhead:

Actual Overhead Costs Incurred C$693,000

Predicted Overhead Based on Actual Driver Use × Standard Prices 825,000 hrs. × C$0.80 = C$660,000 Spending variance, C$693,000 – C$660,000 =

..

Flexible Budget: Standard Driver Use Allowed for Outputs Achieved × Standard Prices 750,000 hrs. × C$0.80 = C$600,000 75,000 × C$0.80 = Efficiency variance, 350


C$33,000 U C$60,000 U Flexible-budget variance, C$93,000 U 2.

The flexible-budget allowance for any variable cost is the same as (is equal to) the total standard quantity allowed for the good units produced times the standard price. The budget allowance under standard costing for variable costs always depends on output, the units produced. Therefore, the direct labor budget for 150,000 units is, as shown above, 150,000 units × 5 hours × C$8.00 = C$6,000,000. For 160,000 units, the budgetary allowance would be 160,000 units × 5 hours × C$8.00 = C$6,400,000. Note again that a budget can be established after the fact, i.e., after the number of units produced is known.

..

351


8-45

(30-50 min.)

1.

Department Performance Report Direct Labor and Variable Overhead Actual hours Standard hours allowed, 2/3 hour × 16,200 units Excess hours

Actual Costs Incurred $59,160

Direct labor Variable overhead: Lubricants @$0.60 Other supplies @$0.30 Rework @$0.60 Other indirect labor @ $1.50 Total variable overhead 2.

11,600 10,800 800

Budget Based on Flexible 10,800 Standard Budget Direct Labor Hours Variance Allowed for 16,200 to be Good Units Produced Explained $56,700 $2,460 U

$ 5,820 3,690 7,380 17,100 $33,990

$ 6,480 3,240 6,480 16,200 $32,400

$ 660 F 450 U 900 U 900 U $ 1,590 U

Summary explanation:

Direct labor

Incurred: Actual Hours × Actual Price 11,600 hrs. × $5.10 = $59,160

Actual Hours × Expected Price 11,600 hrs. × $5.25 = $60,900

Flexible Budget: Standard Hours Allowed × Expected Price 10,800 hrs. × $5.25 = $56,700

11,600 × $0.15 = 800 × $5.25 = Price variance, Quantity variance, $1,740 F $4,200 U Flexible-budget variance, $2,460 U

Incurred: Actual Overhead Cost Incurred

..

Actual Driver Units (Hours) × Expected Price

Flexible Budget: Standard Hours Allowed × Expected Price

352


Variable overhead

3.

($32,400 plus flex. budget variance) = $32,400 + 1,590 U = 11,600 hrs. × $3.00 = 10,800 hrs. × $3.00 $33,990 $34,800 = $32,400 Spending var. = flexible- Efficiency variance = budget var. – Effic. var. = $34,800 - $32,400 $1,590 U - $2,400 U = $810 = $2,400 U F Flexible-budget variance, $1,590 U

The subdivision of the budget variance for variable overhead into spending and efficiency variances appears to be similar to the split of the total direct labor variance into a price variance and a quantity (or usage) variance. However, the interpretation is quite different. The efficiency variance for variable overhead measures the extra overhead costs (or savings) incurred solely because direct labor usage exceeded (or was less than) the standard direct labor hours allowed. When variable overhead is closely related to labor time, fluctuations in overhead costs should correspond with variations in labor time. Both the labor usage and overhead efficiency variances are measured by multiplying a standard price by the difference between actual hours and standard hours allowed. The variable overhead spending variance is similar to the labor price variance, but its causal factors encompass more than price changes alone. Other causes include poor budget estimates for one or more individual overhead items, variation in attention and control regarding individual costs, and erratic behavior of specific overhead items that have been squeezed for convenience into a budget formula that assumes strictly variable behavior and relation to one volume base—labor hours. For example, material handling within the factory (getting material to the workbench) is more closely related to goods started during a period than to standard hours allowed for the work done.

..

353


8-46

(30-40 min.) This solution uses the formulas for variances given in the text.

1.

Material price variance = ($5.50 - $5.30) × 27,000 = $5,400 F Material quantity variance = $5.50 × (27,000 - (60 × 430)) = $6,600 U Labor price variance = ($15.90 - $16.00) × 670 = $67 F Labor quantity variance = $16.00 × (670 - (1.5 × 430)) = $400 U Overhead flexible-budget variance = $5,335 - (($5.76 × 430) + $2,808) = $50.20U

2.

The person in charge of purchasing saved $5,400 by purchasing direct materials for $.20 per pound below standard cost. However, more of the material was used, causing an unfavorable quantity variance of $6,600. If the purchase of low-quality materials caused the excess usage, the net effect was an unfavorable variance of $1,200. There was a small savings of $67 because the average wage rate was $.10 per hour below standard. However, more than the standard amount of labor was used, costing $400. If the hiring lower quality (and therefore lower paid) workers caused the entire excess usage, the net effect is $333 unfavorable. The excess labor might also have been related to the excess use of materials. If some processing is completed before the defective materials are identified, some labor would also be wasted in the process. Thus, the $400 unfavorable labor usage variance might be caused by the purchase of low quality materials. The overhead variance is not large, but it too is unfavorable; $50.20 more overhead costs were incurred than would be expected for the production of 430 kayaks.

3.

Suppose variable overhead varies with total labor hours. Then the overhead variance can be broken into a variable-overhead efficiency variance and an overhead spending variance (but the overhead spending variance cannot be broken into fixed and variable components without more information): Var. OH/DLH = $5.76 ÷ 1.5 = $3.84 Var.-overhead efficiency variance = $3.84 × (670 - (1.5 × 430)) = $96 U Overhead spending variance = $5,335 - (($3.84 × 670) + $2,808)= $45.80 F The unfavorable variable overhead efficiency variance indicates that $96 of overhead costs were incurred because of the excess use of labor. Whatever caused the extra labor hours also caused this variance. Control of overhead itself was good, as shown by the favorable $45.80 overhead spending variance. Note: None of the variances indicate why actual costs differed from standard costs. The variances point toward possible causes and lead managers to ask the right questions about possible causes. The explanations above are possible causes, but they cannot be known for sure until managers do further exploration.

..

354


8-47

(15-20 min.)

1.

Direct Materials: Price variance = ($7.80 - $8.00) × 2,800 = $560F Quantity variance = (2,800 - 2,700) × $8 = $800U Total direct materials variance = $21,840 - (900 × $24) = $21,840 - $21,600 = $240U Overhead:

Supplies Power Rent and other bldg. services Factory labor Depreciation Total overhead

Actual Results at Actual Prices $ 2,132 1,612 2,775 1,618 4,500 $12,637

Flexible Budget for Actual Output Achieved $ 2,250 1,572 2,815 1,500 4,500 $12,637

FlexibleBudget Variances $118F 40U 40F 118U 0 $ 0

2.

KPM does not account separately for the price (rate) and quantity of labor. Therefore, labor price and quantity variances cannot be computed. Many highly automated companies account for labor this way. Because labor is a small proportion of cost in some automated manufacturing processes, it is not worth the cost to monitor price and quantity effects. KPM treats labor as a fixed overhead cost. Therefore, the company does not expect to adjust the amount of labor used as volume changes. Therefore, it compares the actual labor cost to a fixed monthly budget of $1,500.

8-48

(10-20 min.) (a)

Raw Initial material mix X-10 12.0 kg V-10 9.6 ltr S-10 5.0 kg Standard material cost -10-liter container

(b) Allowance for reduction 0.8 0.8 1.0

(c) Required quantity a ÷ b* 15 kg 12 ltr 5 kg

(d) Unit cost $4.40 9.20 6.40

Standard material cost (c × d) $ 66.00 110.40 32.00 $208.40

* Instead of being divided by 0.8, the first two items could be multiplied by 1.25 to obtain the same answer.

..

355


8-49 (25-35 min.) This problem is complicated by the need to include an allowance for defective units in the standard costs. Note that some accountants object to standards that "accept defects" by allowing for defects in the standards. Direct Materials Pounds in final product Allowance for normal scrap Total pounds per finished unit Allowance for defective units Total pounds per good unit Standard price per pound Standard direct material cost per good unit

3.2 .4 3.6 .72* 4.32 $12.20** $52.704

*Defective units = 1/6 of total units or 20% of good units; Material allowance for defectives = .20 × 3.6 lb. = .72 lb. **$11.40 price of materials + $.80 shipping and handling Direct Labor Hours of actual machining per unit Allowance for nonproductive time Total hours per finished unit Allowance for defective units Total hours per good unit Standard rate per hour Standard direct labor cost per good unit

4.00 1.00* 5.00 1.00** 6.00 $ 28.00*** $168.00

*For every 4 hours worked there is 1 hour of nonproductive time because 32 of 40 hours per week are productive. **Defective units = 1/6 of total units or 20% of good units; Direct labor allowance for defectives = .20 × 5 hr = 1.00 hr. ***Basic rate $20.00 Fringe benefits 6.00 Payroll taxes 2.00 Total labor rate $28.00

..

356


8-50

(50-75 min.)

1.

Budgeted and Actual Operating Income for 200,000 units Budgeted Cost Actual Cost a.

Direct Material

b.

Direct Labor

40,000 b.

Direct Labor

40,950

c.

Variable overhead

32,000 c.

Variable overhead

33,000

Total Variable Cost

CHF132,000 a.

Direct Material

CHF140,000

CHF204,000 Total Variable Cost

CHF213,950

Fixed Cost

50,000 Fixed Cost

50,000

Total Cost

CHF254,000 Total Cost

CHF263,950

2.

Actual Input Quantities × Actual Prices

Actual Input Quantities × Expected Prices

Flexible Budget: Standard Input Quantities Allowed for Outputs Achieved × Expected Prices

2,240,000 oz. × CHF0.0625 = CHF140,000

2,240,000 oz. × CHF0.06 = CHF134,400

(200,000 units × 11) × CHF0.06 = CHF132,000

Cost Incurred:

Direct Materials

2,240,000 × (CHF0.0625(2,240,000 – 2,200,000) CHF0.06) CHF0.06 = Price variance, = Quantity variance, CHF5,600 U CHF2,400 U Flexible-budget variance, CHF8,000 U

Cost Incurred:

..

×

Flexible Budget: Standard Input Quantities Allowed for 357


Direct Labor

Actual Input Quantities × Actual Prices

Actual Input Quantities × Expected Prices

19,500 hrs. × CHF2.10

19,500 hrs. × CHF2.00

= CHF40,950

= CHF39,000

Output Achieved × Expected Prices (200,000 units × .1 hrs. × CHF2.00) or (20,000 hrs. × CHF2.00) = CHF40,000

19,500 × (CHF2.10(19,500 - 20,000) × CHF2.00 CHF2.00) = Price variance, = Quantity variance, CHF1950 U CHF1,000 F Flexible-budget variance, CHF950 U 3.

8-51

..

The purchasing manager apparently purchased material for CHF0.0025 per oz. more than the standard, paying an extra CHF5,600. Additionally, the company used more of the material, perhaps because the poor quality of the materials caused excessive waste. The cost of the extra material was CHF2,400, taking the total additional cost to CHF8,000 as against the standard. But this is not necessarily the end of the analysis. Price variance of labor was unfavorable while quantity variance was favorable. Saving in quantity of labor may be due to their superior efficiency as a result of which they are paid more than the standard rate. (30 min.) The solution is given in the textbook after the problem itself.

358


8-52

(15-20 min.)

1. Variable cost per visit, 20X8: $114, 750 + $204,000 + $153,000 + 21% × ($204,000 + $153,000) + ($194,250 - $181,500) = $559,470. Therefore, variable cost per visit are $559,470 ÷ 17,000 = $32.91. Fixed costs are $181,500 + [$676,200 - .20 × (204,000 + $153,000)] = $786,300. Cost Function: $32.91 per visit variable and $786,300 per year fixed. 2. Revenues ($76 per visit; 18,000 visits) Variable costs: Supplies Physician salaries Nurse salaries Overhead General and administrative Total variable cost Fixed costs: Overhead General and administrative Total fixed costs Net Loss

$1,368,000 $ 121,500 216,000 162,000 79,380 13,500 592,380 604,800 181,500 786,300 $ (10,680)

3. Variance to explain: Budgeted loss – Actual loss = $10,680 - $15,500 = $4,820 U Sales-activity income variance = 400 visits × ($76 - $32.91) = $17,236 F Flexible Budget Variance = $4,820 U - $17,236 F = $22,056 U a) The extra 400 units generated $17,236 additional profit. b) Actual costs were higher than the flexible budget for 18,400 visits by $22,056. The total effect was a larger loss than budgeted, caused by spending $22,056 more than budgeted for this level of volume but offset partly by the $17,236 benefit of the additional 400 visits. If costs had not exceeded the flexible budget allowance, the program would have had a net profit of $17,236 - $10,680 = $6,556.

..

359


8-53

(35-45 min.)

1.

Printing department costs for the first month: A B

Printing

Actual Cost Incurred: Actual Inputs × Actual Prices $51,000

C

Flexible Budget for Actual Printing Activity Static Budget 40,000 pages × $1.00 35,000 pages × $1.00 = = $40,000 $35,000

Flexible-budget variance (A - B) = $51,000 - $40,000 = $11,000 U

Printing activity variance (B - C) = $40,000 - $35,000 = $5,000 U

Static budget variance (A - C) $51,000 - $35,000 = $16,000 U 2.

Likely explanations are that (1) the $1.00 charge per page is an average printing cost, but costs per page can differ greatly with relative complexity, (2) the printing department has not identified the appropriate cost drivers to enable it to charge for the costs the department incurs. The static budget was inaccurate for the reasons given in the problem: Printing activity was higher, and the mix of types of jobs differed from what was expected. All the users recognize that four-color, graphic printing is a bargain, and single-color printing is too expensive. As a result, everyone wishes to use more of the low-price, high complexity printing.

3. a.

The ABC analysis is an attempt to measure the costs of printing complexity. If the analysis is accurate, then four-color printing jobs will cost at least $1.35 per page ($.35 + $1.00). Simple, black-and-white jobs will cost only $.35 per page. Since the costs of different types of jobs will vary under this new system, it is critical that the ABC estimates are accurate.

b.

These cost estimates are themselves averages per type of cost because some portions of the printing department costs are fixed in the short run. There is some controversy over whether these fixed costs should be spread over normal or expected levels of cost driver activity. Not "unitizing" these costs preserves the distinction between fixed and variable cost behavior. Under this approach, fixed costs would be recovered by an annual or monthly charge based on expected usage. An objection is that this merely passes the fixed cost problem along to the line units who are selling products or services to external customers. An alternative approach is to unitize these costs over expected or normal cost driver

..

360


activity and charge users as if all costs were variable. If cost driver activity expectations are accurate, there should not be significant static budget variances. Large errors in forecasting, however, will lead to large budgeting errors because total fixed costs charged will not equal total fixed costs incurred. c.

Costs of commercial jobs: 27,500 pages

Old System @$1/page = $27,500

27,500 use color Total cost 8-54

ABC System @$.35/page =$9,625

--

@ $1.00/color page = $27,500 $37,125

$27,500

(40-50 min.)

Before proceeding to answer the questions asked, it is helpful to get a good understanding of the cost behavior. The costs of the outpatient clinic can be broken down into budgeted fixed and variable costs as follows: Cost Physicians Nurses and technicians Supplies Overhead Total

Fixed Total $240,000 180,000 $420,000

Variable Total $180,000 60,000 72,000 $312,000

Per Unit $45 15 18 $78

1.

Whether Hopkins Community Hospital would save money by closing its outpatient clinic depends on what fixed costs it might avoid if the clinic were closed. The budgeted contribution margin is $180 - $78 = $102 per patient, giving a total contribution from 4,000 patients of 4,000 × $102 = $408,000. If the hospital can avoid more than $408,000 of the $420,000 of fixed costs, it would be financially better off without the clinic. It is likely that the $240,000 of physician cost would be saved; the hospital would not need to employ any physicians for the clinic. It is hard to tell how much of the $180,000 of fixed overhead would be saved, but it is unlikely that Hopkins would save the $30,000 of the fixed overhead that is an allocation of hospital-wide administrative costs nor the $37,500 that is depreciation on equipment. Thus, a reasonable guess is that by closing the clinic, the hospital would lose $408,000 of contribution margin and reduce fixed costs by about $240,000 + $112,500 = $352,500. If this is correct, the only way the hospital would gain a financial advantage by closing the clinic is if the closure frees up resources that are valuable to the hospital.

..

361


2.

The difference between the static budget loss of $12,000 and the actual loss of $20,200 can first be divided into a sales-activity variance and a flexible-budget variance: A Actual Profit (Loss) $(20,200)

B Flexible Budget Profit (Loss) $102 × 3,800 $420,000 = $(32,400)

Flexible-budget variance (A - B) = $(20,200) - $(32,400) = $12,200 F

C Static Budget Profit (Loss) $102 × 4,000 $420,000 = $(12,000)

Sales activity variance (B - C) = $(32,400) - $(12,000) = $20,400 U

Static budget variance (A - C) $(20,200) - $(12,000) = $8,200 U Therefore, the main explanation of the additional loss is the decrease in volume. In fact, the loss of volume cost Hopkins $20,400, and cost savings of $12,200 reduces the overall shortfall to only $8,200. The $12,200 flexible-budget variance can be further analyzed by cost category. First, consider the physician cost. Since physician costs are fixed, we can only compute a total physician cost variance: $240,000 - $231,000 = $9,000 F. Nurse and technician costs are variable and have a standard rate of $30 per hour and an actual rate of $182,700 ÷ 5,800 = $31.50. A total of 5,800 hours was used; standard hours allowed for 3,800 patients is 1.5 hrs./patient × 3,800 patients = 5,700 hours. Price (rate) and quantity (usage) variances are as follows: Price (rate) variance = ($30 - $31.50) × 5,800 = $8,700 U Quantity variance = (5,800 – 5,700) × $30 = $3,000 U Supplies cost is variable, but we have no measure of amount used. Therefore, we can compute only a total supplies variance: $15 × 3,800 - $58,500 = 1,500 U. From the information given, the overhead variance cannot be computed in any more detail than a total overhead variance: [($18 × 3,800) + $180,000] $232,000 = $248,400 - $232,000 = $16,400 F Therefore, the total flexible-budget variance can be explained as follows: Physician variance $ 9,000 F Nurse & technician price variance 8,700 U Nurse & technician quantity variance 3,000 U Supplies variance 1,500 U Overhead variance 16,400 F ..

362


Total flexible-budget variance

$12,200 F

One possible explanation for this pattern of variances is that nurses have been covering some time that physicians usually cover. The physician may not be spending full time in the clinic. Nurses and technicians are spending more time per patient than is budgeted, and that may be causing overtime premiums, which might explain the unfavorable price variance. The large favorable overhead variance is more difficult to explain from the information given. It might be possible (though this is just speculation) that some expensive equipment was not fully utilized at the clinic and was moved to the hospital. This might explain why the volume was down (i.e., those who needed that equipment went to the hospital rather than the clinic) and why the fixed charges to the clinic were reduced. 8-55

(45 – 60 min.)

1. Revenue (2,000 × $200) Variable costs (2,000 × $139) Contribution margin (2,000 × $61) Fixed costs Operating income

$400,000 278,000 $122,000 60,000* $ 62,000

* Fixed costs = ($18 + $12) × 2,000 = $60,000. 2. Static budget variance = $62,000 - $47,740 = $14,260 U Sales activity (volume) variance = 200 × ($200 - $139) $12,200 F Flexible bud. var. = $392,260 – ($139 × 2,200 + $60,000) $26,460 U Static budget variance

$14,260 U

Material variances: Housing variance = $44,000 – 2,200 × $20 PCB variances: PCB price variance = ($16 - $15) × 4,700 PCB usage variance = (4,700 – 4,400) × $15 Total PCB variance = $75,200 – 2,200 × $30 Reading heads variances: RH price variance = ($11 - $10) × 9,200 RH usage variance = (9,200 – 8,800) × $10 Total RH variance Total material variances

$13,200 U $22,400 U

..

$

0

$ 4,700 U 4,500 U $ 9,200 U $ 9,200 U 4,000 U

363


Labor variances: Assembly variances: Assembly price variance = ($8 - $8) × 3,900 Assembly quantity var. = ($4,400 – 3,900) × $8 Total assembly variances PCB variances: PCB price variance = ($9.90 - $9) × 2,400 PCB quantity variance = (2,400 – 2,200) × $9 Total PCB variances Reading heads variances: RH price variance = ($11 - $10) × 3,500 RH quantity variance = ($3,500 – 3,300) × $10 Total RH variances Total labor variances

0 $4,000 F $4,000 F $2,160 U 1,800 U $3,960 U $3,500 U 2,000 U

Overhead variances: Variable OH efficiency var. = (9,800 – 9,900) × $2 200 F Variable OH spending var. = $18,800 – (9,800 × $2) 800 F Fixed OH spending variance = $37,600 - $36,000 1,600 U Total overhead variances Selling & administrative var. = $22,000 – ($12 × 2,000) Total flexible-budget variances

$5,500 U $5,460 U

$ 600 U

$2,000 F $26,460 U

From these variances we learn that operations were not very efficient. The extra 200 units of sales increased income before taxes by $12,200, but this was more than offset by operating inefficiencies that cost Gates $26,460, leaving the company short of budget by $26,460 - $12,200 = $14,260. The material variances and the RH and PCB labor variances were all unfavorable and quite large, explaining much of the shortfall in income. The bright spots were assembly labor, variable overhead, and selling and administrative costs that had small favorable variances. Still, the performance was much below expectations.

..

364


8-56

(30-40 min.) Monetary amounts in millions of dollars.

The actual results from the Results of Operations summary for 2010 are: Net sales 19,014 Variable cost (cost of sales) 10,214 Contribution margin 8,800 Fixed costs 6,326 Income (before taxes) 2,474 These amounts are used to prepare a static budget assuming 10% growth in revenue and variable cost. (Income before taxes shown in the Nike 10-K is not equal to the actual amounts computed here for 2010 and 2011 due to some minor non-operating omitted items.) The flexible budget is based on the 2011 actual sales and the assumption that growth in variable costs should be proportional to the growth in sales. Flexible Static Actual Budget Budget 2011 2011 2011 Net sales 20,862 20,862 20,915 Variable cost (cost of sales) 11,354 11,207 11,235 Contribution margin 9,508 9,655 9,680 Fixed costs 6,693 6,326 6,326 Income (before taxes) 2,815 3,329 3,354

Sales Variable cost Contribution margin Fixed costs Operating income

Actual Results at Actual Activity Level 20,862 11,354 9,508 6,693 2,815

FlexibleBudget Variances

FlexibleBudget for Actual Sales Activity

Sales Activity Variances

10% Growth Static Budget

(147) (147) (367) (514)

20,862 11,207 9,655 6,326 3,329

(53) 29 (25) (25)

20,915 11,235 9,680 6,326 3,354

*Unfavorable variances denoted by parentheses The variances show that actual income was $539 million less than in the static budget that assumed 10% growth for three main reasons: 1) Actual sales growth was less than 10%, yielding a $25 U sales-activity income variance. 2) Variable costs increased at a higher rate than sales, yielding a $147 U flexible budget variable cost variance.

..

365


3) Spending for fixed costs was $367 more than budgeted, yielding a $367 U fixed overhead spending variance. This may mean that some of the fixed costs were not truly fixed – that they increased somewhat with the increase in volume of sales. Note that the sum of the variances, 25 U + 147 U + 367 U = 539 U. 8-57 (20-30 min.) For the solution to this Excel Application Exercise, follow the step-by-step instructions provided in the textbook chapter. 1. The favorable flexible-budget variance for sales is caused by an increase in selling prices. 2. Actual income fell $24,500 short of the static-budget income. 3. The flexible-budget variance ($500 F) and the sales-activity variance ($25,000 U) completely explain the $24,500 U static-budget variance. 8-58

(60 min. or more)

The purpose of this exercise is to understand the difficulty of setting standard costs for even simple products or services. For many products or services, identifying the direct material and direct labor inputs may not be hard, but identifying the overhead support can be a challenge. Students are likely to make various assumptions, which can lead to very different standard costs in all cost categories. Requirements 2 and 3 also lead to consideration (albeit implicitly) of many of the behavioral issues organizations may have in setting standards. Different managers have different objectives and different levels of knowledge. These must be combined into a single standard cost estimate. If class time allows, it may be useful to have one or more groups present their standard costs to the class and describe how they were determined. The class may have suggestions that the group failed to consider -- possibly some implicit assumptions that would not necessarily hold true. 8-59 (30-50 min.) NOTE TO INSTRUCTOR: This solution is based on the web site in late 2012. Be sure to examine the current web site before assigning this problem, as the site may have changed. In particular, a more recent annual report for use in part 2 will be available beginning in early 2013. 1.

..

Hershey’s Web site seems primarily directed to customers. The most prominent items across the top of the page are products, promotions, ads, recipes, and items available for sale. Hershey uses the site as a form of advertising and as a way to provide detailed information about their products. To find financial information you have to click on “Corporate Information” at the bottom of the home page. 366


2.

The static budget for 2012 income before income taxes, assuming a 4% increase in net sales and variable costs (but no increase in fixed costs) relative to 2011, is: 2012 Static Budget $6,324,020 3,690,852 2,633,168 1,569,047 $1,064,121

2011 Actual $6,080,788 3,548,896 2,531,892 1,569,047 $ 962,845

Net sales Variable cost (cost of sales) Contribution margin Fixed costs Income before income taxes

Note that a 4% increase in sales volume leads to a ($1,064,121-$962,845) ÷ $962,845 = 10.5% increase in operating income due to operating leverage, as discussed in Chapter 2. 3.

The flexible budget for a sales increase of 6% and actual results assuming sales and variable costs increased by 6% and fixed costs increased by 1%:

Net sales Variable cost (cost of sales) Contribution margin Fixed costs Income before income taxes

2012 Flexible Budget for a 6% Sales Increase $6,445,636 3,761,830 2,683,806 1,569,047 $1,114,759

2012 Actual with 1% higher fixed costs $6,445,636 3,761,830 2,683,806 1,584,738 $1,099,068

The analysis of static-budget, flexible-budget, and sales-activity income variances is: A Actual Net Income $1,099,068

B Flexible Budget Net Income $1,114,759

Flexible-budget variance (A - B) = $1,099,068 - $1,114,759 = $15,691 U

C Static Budget Net Income $1,064,121

Sales activity variance (B - C) = $1,114,759- $1,064,121 = $50,638 F

Static budget variance (A - C) $1,099,068 - $1,064,121 = $34,947 F ..

367


Note that actual sales and variable costs are equal to flexible-budget sales and variable costs, and the flexible-budget income variance is due entirely to actual fixed costs ($1,584,738) higher than budgeted fixed costs ($1,569,047) by $15,691.

..

368


CHAPTER 9 COVERAGE OF LEARNING OBJECTIVES

LEARNING OBJECTIVE LO1: Describe the relationship of management control systems to organizational goals. LO2: Use responsibility accounting to define an organizational subunit as a cost center, a profit center, or an investment center. LO3: Develop performance measures and use them to monitor the achievements of an organization. LO4: Explain the importance of evaluating performance and how it impacts motivation, goal congruence, and employee effort. LO5: Prepare segment income statements for evaluating profit and investment centers using the contribution margin and controllable-cost concepts. LO6: Use a balanced scorecard to recognize both financial and nonfinancial measures of performance. LO7: Measure performance against quality, cycle time, and productivity objectives. LO8: Describe the difficulties of management control in service and nonprofit organizations.

..

FUNDAMENTAL ASSIGNMENT MATERIAL B3

CASES, EXCEL, ADDITIONAL COLLAB., & ASSIGNMENT INTERNET MATERIAL EXERCISES 28, 30, 36, 36 50, 52, 53, 54

A1, B3

28, 43, 43

B3

15, 32, 35, 36, 36, 42, 44, 45

50

A1, B1, B3

14, 30, 33, 33, 34, 37, 42, 43, 48, 48

50, 52, 53, 54

A2, B2, B3

38, 38, 43, 43

52, 54

B3, B3

16, 35, 45

56

A3, B1, B3

31, 32, 33, 33, 39, 40, 41, 46, 46, 47, 48, 48, 49, 49 29, 37

50, 51 50, 53, 55

369


CHAPTER 9 Management Control Systems and Responsibility Accounting 9-A1

(20 min.) Excel Electronics Company may have a legitimate claim against the supplier that would offset the penalty. However, the disposition of any claim is a separate issue. The penalty of $30,000 should be charged to the purchasing department. Amy Greer may have done everything in her power to see that the special part was delivered on time, but she is the one who is responsible for purchasing necessary material when it is needed. Greer may not have control over her suppliers and subsequent delivery, but it is her responsibility to have the purchased parts when they are needed. She is the person in the organization who has the most influence over delivery. Everybody makes mistakes. The important point is to minimize the number of mistakes and also to understand fully that the extensive control reflected in responsibility accounting is the necessary balance to the great freedom of action that individual executives are given. Other questions to discuss are: Did the sales department behave responsibly in accepting the order with penalty? Is it conceivable that a careful statistical study of delays by suppliers would permit the development of an "expected amount" of penalty to be incurred in a probabilistic sense, which then could be budgeted as part of the purchasing department's costs? Discussions of this problem have again and again revealed a tendency among students (and among accountants and managers) to "fix the blame" -- as if the variances arising from a responsibility accounting system should pinpoint misbehavior and provide answers. The point is that no accounting system or variances can provide answers ipso facto. However, variances can raise questions. In this case, in deciding where the penalty should be assigned, the student might inquire who should be asked in this situation -- not who should be blamed.

..

370


9-A2

(30-40 min.) See Exhibit 9-A2 on the following page.

9-A3

(15-20 min.)

1.

Without adjusting for inflation, it appears that both companies had large increases in productivity in terms of revenues per employee.

Sorteberg Forsythe

20X1 $7,997,000,000 ÷77,900 = $102,657 $4,720,000,000 ÷53,600 = $ 88,060

20X7 $9,007,000,000 ÷78,200 = $115,179 $6,500,000,000 ÷57,800 = $112,457

However, the 20X1 productivity measures should be expressed in constant 20X7 dollars for comparability: 20X1 20X7 Sorteberg (1.18 × $7,997,000,000) ÷77,900 = $121,136 115,179* Forsythe (1.18 × $4,720,000,000) ÷53,600 = $103,910 112,457* * Calculation is the same as above for 20X7. 2.

Using productivity measures that are correctly adjusted for inflation, we see that Forsythe had an increase in productivity between 20X1 and 20X7. In contrast, Sorteberg decreased its productivity by $121,136 - $115,179 = $5,957 per employee, a decrease of 5,957 ÷ 121,136 = 4.9%. Although both Forsythe and Sorteberg had an increase in number of employees, the larger sales increase for Forsythe led to a higher productivity number.

..

371


EXHIBIT 9-A2 Answers are in thousands of dollars. Company as a Breakdown into Whole Two Divisions Cedar Waterloo Rapids Division Division Net Sales 8,000 3,200 4,800 Variable costs: Cost of merchandise sold 3,500 1,400 2,100 Variable operating expenses 640 280 360 Total variable costs 4,140 1,680 2,460 Contribution margin 3,860 1,520 2,340 Less: Fixed costs controllable by segment managers 960 265 695 Contribution controllable by segment managers 2,900 1,255 1,645 Less: Fixed costs controllable by others 490 140 350 Contribution by segment 2,410 1,115 1,295 Less: Unallocated costs 110 Income before income taxes 2,300

..

Breakdown of Waterloo Division Not Allocated Downtown 2,400

Breakdown of Cedar Rapids Division Sumner 800

Not Allocated

Downtown Solon 2,400 1,200

Airport 1,200

1,050

350

1,050

525

525

240 1,290 1,110

40 390 410

240 1,290 1,110

60 585 615

60 585 615

100

125

40

210

125

180

180

(100)

985

370

(210)

985

435

435

35 (135)

70 915

35 335

70 (280)

70 915

105 330

105 330

372


9-B1

(15-20 min.)

1.

It is not possible to determine the validity of Liz Elder’s claim that the system is disadvantageous to her department. It would be valid only if, on a proportional basis, the number of unidentified rejects caused by the other departments were greater than their proportionate number of identified rejects. Although the claim cannot be substantiated, a legitimate issue has been raised. The rejects charged to all the departments contain amounts not clearly attributable to the respective departments. This violates the concept that performance measures should not contain items outside the control of the manager. Further, a manager's effort to control the variation will be influenced by the result she can get from her actions. The fact that some of the rejects are likely caused by other departments will reduce the amount of the reported rejects within her control.

2.

There are two solutions to this problem. First, remove the apportioned rejects from the reports and charge the managers with only the rejects identified with their department. Second, if the number of unidentified rejects is large and represents a large dollar value (which could be reduced if adequate information as to cause were available), then Kephart Company should consider inspection at the end of production in each department.

..

373


9-B2

(30-35 min.)

1.

See Exhibit 9-B2 on the following page.

2.

The incremental costs of running such sightseeing tours can be identified with much more confidence than in many other instances. Net income will be improved by the excess of tour revenue over incremental costs; routine allocations of other operating costs and indirect costs will not be relevant to the decision to run such tours unless these costs change incrementally with the tours. Those railroads that do not run such tours either: (a) Do not expect incremental revenue to exceed incremental costs; or (b) Have other objectives that outweigh the potential incremental profit from running tours. For example, some railroads may not want to engage in passenger tours that would slightly improve short-run profits because their long-run objective is to reduce passenger business as much as possible.

3.

If the entire $200,000 of separable discretionary fixed costs can be avoided by dropping Division No. 1, net income would decrease by the controllable contribution of $900,000. If only part of the separable discretionary fixed costs can be avoided, net income would decrease by between $900,000 and the contribution margin of $1,100,000. The separable committed costs should also be carefully considered. The assumption in the statement above is that these are truly committed costs that cannot be saved if the division is dropped. However, if some of these can be saved if Division No. 1 is dropped, the analysis is changed. As an example, if all $3 million of separable committed costs could be avoided if Division 1 is dropped, then income would increase by $2,100,000 from dropping the division.

..

374


Exhibit 9-B2 READING RAILROAD Income Statement For the Year Ended December 31, 20X3 (in thousands of dollars)

Revenue........................................ Variable costs ............................... Contribution margin ..................... Separable discretionary fixed costs ....................... Contribution controllable by segment managers ................. Separable committed costs ........... Contribution by segments ............ Unallocated costs ......................... Income before income taxes ...........................................

..

Railroad as a Whole 80,000 40,000 40,000

Breakdown into Two Divisions PasFreight senger Traffic Traffic 72,000 8,000 36,000 4,000 36,000 4,000

Possible Breakdown of Passenger Traffic Only Not Allocable -

No.1 3,200 2,100 1,100

Division No.2 4,000 1,600 2,400

No.3 800 300 500

8,000

7,600

400

80

200

100

20

32,000 25,000 7,000 800

28,400 20,000 8,400

3,600 5,000 (1,400)

(80) 1,000 (1,080)

900 3,000 (2,100)

2,300 700 1,600

480 300 180

6,200

375


9-B3

(25 min)

1. Students will come up with many possible measurements. Among the possibilities are: Financial: Customer:

Internal:

Learning:

a. Growth in profitability Number of new clients Revenues from new clients a. Number of face-to-face meetings with clients Customer survey – satisfaction scores b. Number of cases completed on time Customer survey – how well needs were met a. Number of team-based cases handled Number of staff generated entries to Intranet b. Internal conflicts and number successfully resolved Employee survey – ranking in internal communications c. Number of staff-generated solutions Ratio of partners to legal staff a. Voluntary turnover Employee survey – satisfaction with environment b. Percentage of underrepresented minorities Diversity of undergraduate degrees Variety of skills and interests represented

2.

The firm will want to balance the benefits from the balanced scorecard with the costs of using it. The firm might routinely collect customer satisfaction scores at the completion of each case. It might collect employee satisfaction scores once or twice a year. The key will be to set up a system to 1) carefully define each measure, 2) collect the needed information, and 3) use the information to provide feedback on performance. For measures such as number of new clients or number of face-to-face meetings, collecting the information will be easy. For more subjective measures, such as customer or employee satisfaction, the firm must devise detailed measurement methods. These must be accepted as reasonable bases on which to assess performance. Finally, the firm must set up a system for weekly, monthly, quarterly, or annual reporting of the measures and evaluation of performance based on these reports.

3.

The impact of a balanced scorecard will be greater if the firm bases individual performance evaluations and compensation on the scorecard results. This can have both benefits and drawbacks. Among the benefits are 1) aligns staff priorities with firm priorities, 2) focuses staff attention on reaching the firm’s strategic goals, and 3) provides motivation to increase performance in areas that are important to the firm. Drawbacks include 1) imperfect measures may lead to dysfunctional behaviors and 2) focus on items measured in the balanced scorecard may lead to neglect of non-measured items. Whether to tie compensation to the

..

376


balanced scorecard results is a matter of judgment – whether the benefits outweigh the drawbacks. 9-1

A management control system is a logical integration of techniques to gather and use information to make planning and control decisions, to motivate employee behavior, and to evaluate performance.

9-2

A management control system    

9-3

clearly defines and communicates the organization’s goals ensures that managers and employees understand the specific actions required to achieve organizational goals communicates results and coordinates actions across the organization, and motivates managers and employees to achieve the organization’s goals. The major components of a management control system are:    

Setting goals and targets Developing and executing the plan Measuring, monitoring, and reporting results of actions Evaluating and rewarding performance

9-4

A key success factor is a characteristic or attribute that must be achieved in order to drive the organization towards its goals. Note the difference between a key success factor and an action. Actions require effort and can be observed on a short-term basis. A cause-effect statement can be made that relates specific actions (or activities) to key success factors. “If we ________________ (fill in the action), then we will __________________ (fill in the key success factor).” For example, “If we reduce order lead time, then we will be more responsive to our customers.” Actions are verbs, key success factors are characteristics or attributes.

9-5

Goals without performance measures may not be completely useless, but performance measures greatly enhance the achievement of goals. They provide signals to managers about whether goals are being achieved.

9-6

Some typical corporate goals other than those which immediately improve profit are (a) growth, (b) high quality products, (c) market domination, (d) excellent social service, (e) high prestige, and (f) improved productivity.

9-7

Key success factors are those aspects of performance that are essential to achieve if the organization is to be successful. Management examines an organization’s strategic plan and major goals and decides what factors are most important to achieving its goals - these are the key success factors.

..

377


9-8

Examples of sacrificing long-range goals to short-run performance gain are: a. Wasteful disposal of inventory to improve the turnover rate. b. Maintaining short-run peak personnel efficiency by refusing to rotate assignments in a way that would improve long-range flexibility and individual capabilities. c. Postponing desirable maintenance and repairs.

9-9

Three types of responsibility centers are: • Cost center - responsibility for control of costs • Profit center - responsibility for both costs and revenues • Investment center - responsibility for both profit and investment.

9-10

Investment centers go a step farther than profit centers. Both measure profits, but an investment center also compares that profit to investment using measures such as return on investment, residual income or EVA (which are discussed in Chapter 10).

9-11 Good performance measures will: • Relate to the organization's goals. • Balance long-term and short-term considerations. • Reflect key activities of the organization. • Be affected by managers' actions. • Be easily understood by managers and employees. • Be used in evaluating and rewarding managers and employees. • Be reasonably objective and easily measured. • Be used consistently. 9-12

Examples of nonfinancial measures of performance are percentage of products delivered on time, proportion of defective units produced, setup time for a batch of production, average time from order to delivery, and pounds of output per direct labor hour.

9-13

Goal congruence and motivation are two aspects important to achieving an organization's goals through managers' actions and decisions. Goal congruence is achieved if managers seek the goals sought by top management -- that is, managers aim in the direction that is best for the organization. Managerial effort is exertion toward a goal. A good performance evaluation system provides the managers with appropriate goals and the incentive to achieve the goals.

9-14

Through responsibility accounting, it is possible to identify what parts of the organization have primary responsibility for each action, develop performance measures and targets, and design reports of these measures by responsibility centers. As a result, for a particular adverse result, identification of responsibility become possible and control action can be initiated easily.

..

378


9-15

Uncontrollable costs do not provide any insight into a manager’s decision and action because these are not affected by a manager’s action. On the contrary, controllable costs are affected by a manager’s decision and action. Thus, a manager can be evaluated only based on the controllable costs and not on the basis of uncontrollable costs.

9-16

Improper selection of productivity measures may lead to motivating decisions that improve one dimension of performance but hurt another. For example, measuring and rewarding productivity per machine would incentivize longer production runs. However, longer production runs may result in excess inventory handling and holding costs. Again, measuring productivity per labor hour may motivate the workers to produce more units per hour but this may lead to workers spending less time per unit, resulting in a dip in quality and higher number of reworks.

9-17

Examples of segments are divisions, territories, branches, product lines, and stores.

9-18

Managers should be judged on how well they attain their currently attainable objectives, focusing on deducting controllable costs from revenues, whereas the subunit should be judged on its performance as an economic investment.

9-19

No. The contribution margin format does not ignore items that are not a part of the contribution margin. Rather, it separates costs by their behavior (variable and fixed) and by who can control the cost. The fixed costs are still important, and the contribution margin format analyzes them in terms of their appropriate cost behavior.

9-20

A balanced scorecard is a performance report that contains measures of all the key financial and nonfinancial variables that are important for a company to prosper. Many companies find this a useful tool to help managers focus on the multidimensional factors that make an organization successful.

9-21

Key performance indicators are measures that drive the organization to achieve its goals.

9-22

The four categories are: (a) prevention -- costs incurred to prevent the production of defective products or services, (b) appraisal -- costs incurred to identify defective products or services, (c) internal failure -- costs of defective products that are scrapped or reworked, and (d) external failure -- costs caused by delivery of defective products or services to customers.

..

379


9-23

Many companies are finding that it is less costly to prevent defects than it is to identify and correct defects.

9-24

Control of nonfinancial performance requires setting objectives, measuring results, and evaluation of results by comparing outcomes to expectations (or objectives). This is the same sequence indicated by control of financial performance.

9-25

Three measures of productivity are: (a)

Standard direct labor hours allowed for good output achieved Actual direct labor hours of input

(b)

Sales revenue Direct labor cost

(c)

Sales revenue Number of employees

9-26

Comparing productivity measures over time is complicated by changes in the production process and by inflation. Consider changes in the production process that substitute one input for another. Such changes make productivity with respect to the replaced input increase, while productivity with respect to the input that is increased will decrease. Further, if either input or output (but not both) is measured in monetary terms, inflation can distort productivity measures across time.

9-27

Yes. There are several reasons that developing control systems in nonprofit organizations is more difficult than in profit-seeking organizations, including: (a) There are often multiple goals, and often the goals are not explicit. (b) There is no single, measurable objective such as profit that determines the trade-off between various goals. (c) The types of people in nonprofit organizations, frequently professionals, are often less receptive to the demands imposed by control systems. (d) The relationship of inputs to outputs is hard to specify. (e) A large portion of the costs are discretionary fixed costs, which are the hardest to manage through a control system. Nevertheless, control systems can be valuable to nonprofit organizations.

..

380


9-28

This question cannot be answered directly from the text. It requires students to think about an issue closely related to those in the text. An article in FE: The Magazine for Financial Executives (Vol. 1, No. 8) addresses these questions. After studying several large firms that encourage innovation, the author concluded that such firms had not abandoned sound financial controls or even watered them down. "The companies surveyed had achieved superior financial results and had sound financial systems in place ...The CFO [Chief Financial Officer] in each firm knew the key financial factors needed for the company's success, and had a financial control system to carefully track that success" (p.36). The article made two structure-oriented and five process-oriented suggestions to adapt a financial control system to foster innovation. Regarding structure: 1. "The primary focus should be on setting up profit centers. A decentralized organization allows for expanding profit center accounting. Profit center accountability in turn permits more discretion and enhances innovation. Our study indicated that flexibility and entrepreneurial decision-making can be fostered by a well-structured profit center reporting system." 2. "A second structural factor in a large, decentralized organization committed to innovation calls for divisional financial executives to have a direct, solid-line reporting to the divisional general manager. However, a solid-line reporting of divisional financial executives to a corporate senior financial executive virtually precludes an entrepreneurial spirit at the division level." The process-related suggestions are these: a. Planning -- "The successful entrepreneurial firms...have a well-developed strategy...The strategy is well understood through all levels of management...Highly structured, precisely quantified planning is not done...Planning is directed toward allowing flexibility and changes dictated by the changing business environment." b. Budgeting -- "An annual budget, with interim period breakouts, is well accepted as essential for any successful business. An entrepreneur is not greatly burdened by and accepts the need for stating in numbers his or her program for the coming 12 months." c. Resource allocation -- "Approval systems for capital expenditures frequently require extensive reporting to higher levels of management...The CFO should measure the needs for capital controls against the driving force of an innovation entrepreneur. Achieving a fair balance is not easy."

..

381


d. Reporting -- "A profit center seeking to be independent and innovative can lose its thrust if it perceives that every action is being followed by corporate headquarters through the monthly financial reporting. The challenge is to provide a system that maintains financial strength while allowing the flexibility and independence that produce superior results through innovation and entrepreneurism." e. Analyzing operating results -- "One factor in this area stood out: the frequent reference to comparisons of actual results to budget, giving full weight to noncontrollable factors and to changed conditions." 9-29

New York City provides a good case study in the consequences of not identifying and measuring the financial responsibilities of managers and how adding such measures can lead to organizational success. A responsibility accounting system keeps top management informed about activities and decisions made by middlemanagers and can also motivate managers to act in the best interests of the municipality. Identifying responsibility centers is an important first step in developing a system. Financial results for each responsibility center enable top management to know the city's financial situation. Before developing the IFMS, New York City officials did not know exactly why the desperate financial situation had developed. IFMS allows them to anticipate financial demands. It also allows a check on managers who might tend to be fiscally irresponsible. To motivate managers, the financial results for a responsibility center should affect performance evaluation. Of course, non-financial matters also affect evaluations.

9-30

In an article in the Web magazine Optimize (April 2003) Bruce Guptill discussed customer-centric metrics. The four most popular metrics were: 1) customer satisfaction, 2) customer loyalty, 3) decreased complaints, and 4) increased customer behavior. In addition to these metrics, Volvo might consider results from research by third parties such as J. D. Power & Associates, market share data, and time from order to fulfillment. Students may come up with many more potential metrics.

9-31

Quality, cycle time, and productivity are related because improvements in cycle time and productivity are dependent upon high quality processes and inputs. High quality depends on good product (or service) and process designs, highly trained employees, and commitment to continuous improvement. These factors also lead to improvements in cycle time and productivity.

9-32

There are many possible answers for each company or organization. Examples are:  British Airways: Percent on-time arrivals; capacity utilization  Amazon: Number of standard stocking units (SKUs); sales per square foot of space

..

382


 

Sony Corporation: Number of new products; product development time Regional Passport Offices under the Ministry of External Affairs, India: Cost of services; number of passports issued per employee

9-33

(20 min.) Production assistant cum intra-departmental security personnel cost should be charged the standard labor rate of ₹3,000 p.m. This assumes that the goldsmiths are qualified to do the production assistant cum intra-departmental security personnel’s job. It is up to the production supervisor to get ₹3,000 p.m. worth of work from the goldsmiths. The ₹2000 monthly rate difference (₹5,000 – ₹3,000) should certainly not be charged to the production assistant cum intra-departmental security personnel’s cost, since the regular help need be paid only ₹3,000 p.m. The ₹2000 monthly rate difference could be charged to Loss from Idle Capacity or some similar account. From a control viewpoint, the loss should be the responsibility of the individual who decides to retain the goldsmiths rather than to lay them off. Because the goldsmiths must be retained in order to maintain high quality workmanship and perhaps the reputation and sales position of the company, a conceptual case could even be made for treating the ₹2000 as an asset because the decision to keep high-priced personnel implies a future cost saving, possibly in hiring and training new employees, or a future revenue enhancement. Because the value of this “asset” is highly contingent on future events, this is rarely done in practice. 9-34

(35 min.)

1.

Compensation: If quota is met: If quota is not met:

January February March April 2.

¥50,000 + ¥68,000+.05 x (actual - quota) ¥50,000 A ¥ 50,000 190,750 226,000 50,000

Clerk B ¥118,000 118,000 50,000 118,250

C ¥193,000 50,000 413,500 50,000

Notice the wide variation in month-to-month sales and the even wider variation in compensation. The variation suggests that sales are only partially under the control of the salesclerks. Therefore, it is likely to be almost impossible for the salesclerks to continually increase sales by 3% per month. Given sales quotas of 103% of the previous month's actual sales, the bonus of ¥68,000 plus 5% commission on sales over quota is extremely high and provides incentives to manipulate the quota system similar to the dysfunctional incentives discussed in Chapter 7 for budgets. In any given month, the salesclerks have incentives to either a) meet or surpass their quota (to earn the substantial bonus),

..

383


or b) miss the quota by a substantial amount (to lower the quota as much as possible for the subsequent month). The incentives are also affected by the salesclerk’s sales volume. Salesclerk B, with low sales volume, appears to be manipulating sales to just meet his or her quota several months running and then has a low volume month in March to bring the quota down substantially in preparation for another run of months of just meeting the quota. Salesclerks A and C, with larger sales volumes, have incentives to surpass their sales quotas by substantial amounts and then have a low volume month to bring down their quotas in preparation for a large bonus the following month. Salesclerk A has low volume months to reduce the quota in January and April and Salesclerk C’s low volume months occur in February and April. The bonus system should be modified. If an incentive is to be provided, the quotas should be tied to a more reasonable standard of what constitutes a normal month's sales (and should specifically not be tied directly to the previous month's sales). Most companies believe a bonus payment should be small in relation to basic compensation and should be related to actual sales effort rather than to clever manipulation. Moreover, most companies believe that most compensation should be tied to performance over a longer time span than one month. A yearly span would be less subject to manipulation. 9-35

(10 min.)

Students may classify some of these measures differently than shown here. The point should be made that the important feature in a balance scorecard is to have all perspectives represented. PERFORMANCE MEASURE Return on sales Retention of target customers Net cash flow Training hours Employee turnover rate Material handling cost per unit Market share Product development cycle time Revenue growth in segments Occupational injuries and illness Day’s sales in inventory Average cost per invoice

..

PERSPECTIVE Financial Customer Financial Learning and growth Learning and growth Internal process, financial Customer Internal process Financial Learning and growth Internal process, financial Internal process, financial

384


9-36

(10-15 min.) Students will suggest many different goals and measures in each area. This solution lists one possibility for each of the five areas. 1. Coverage and speed Goal: Reach out to the most distant places Measure: Population/towns covered 2. Usage Goal: Increase customers’ usage of mobile services Measure: Average call duration 3. Operational efficiency Goal: Decrease operating cost Measure: Operating expenditure as a percentage of revenue 4. Customer satisfaction Goal: Reduce number of complaints Measure: Number of unresolved customer complaints 5. Overall financial performance Goal: Annual growth in ARPU (average revenue per user) Measure: ARPU

9-37

(15-20 min.) Increasing sales activity can be related to increased number of new accounts; thus many stock brokerages set objectives for its brokers to make a set number of "cold calls" to solicit investments from potential clients. However, a large number of small accounts probably do not have the same impact on sales as a few large accounts. The brokerage firm must be careful not to divert its employees' energies so much to finding new accounts that research, analysis, and service for existing accounts are neglected. Service firms have found that it is much more profitable to retain existing customers than to find new customers. Therefore, customer retention has become a major objective, and performance is measured on activities that are believed to aid in retaining profitable customers. These measures include how quickly phones are picked up, how quickly inquiries are answered, accessibility of data bases, and so on.

..

385


9-38 1.

(15 - 20 min.)

Revenues Variable costs Contribution margin Fixed costs controllable by restaurant managers Contribution controllable by restaurant managers Fixed costs controllable by others Contribution by restaurant Unallocated costs Operating income 2. a.

Company £1,200,000 810,000 390,000

Chester £600,000 390,000 210,000

Coventry £600,000 420,000 180,000

92,500

55,000

37,500

297,500

155,000

142,500

142,500 155,000 30,000 £ 125,000

50,000 $105,000

92,500 $ 50,000

The new shop in Coventry is not yet as profitable as the Chester shop. The advertising campaign to build up the customer base in Coventry weighs on the profits of the Coventry shop. However, both shops have a positive contribution after subtracting all costs that the company can specifically identify with individual shops.

b. The Chester manager has £55,000 more controllable contribution currently, but there is a potential for improvement in Coventry when the effect of the extra advertising is realized. Coventry has lower fixed costs that the manager controls but higher variable costs. Although the two managers reach different levels of profitability with a different mix of costs, they seem to both be performing well given the stage of their development—Chester is more well-established and shows higher contribution, but Coventry shows potential for higher contributions in the future. 9-39

(10-15 min.) The figures on the next page can be used as both a solution to this exercise and point of further discussion. The left-hand graph in the exercise (and the top graph below) represents the traditional view of quality costs, while the right-hand graph in the exercise (and the bottom graph below) represent the view espoused by “total-quality” guru Deming and accepted by most firms today. In both graphs, the optimal level of quality occurs at the minimum of the total cost curve. In the TQM approach all phases of the company’s operations are incorporated in the quality program. For example, the quality of incoming materials and parts is higher. This reduces (or eliminates) appraisal costs (and associated costs), while failures that result from poor quality are also reduced. Another example of a win-win scenario is training employees to reduce errors resulting in cost savings from reduced inspection

..

386


(appraisal) and internal and external failures. If the cost savings from reduced appraisal activity exceeds the training costs (prevention costs), the prevention and appraisal cost curve will shift downward, as shown in the Total Quality Management panel. As a result, the minimum of the total cost curve shifts to the right. The minimum total cost occurs at a higher level of quality under the TQM view. Also note that the total cost of quality is lower for firms producing higher quality products or services. Deming predicted this by pointing to the close relationship between quality and costs such as waste, rework, returns, lost sales, and inspection.

..

387


9-40

(10-15 min.)

1.

One trend is the overall upward trend in defective units. The overall rate of defective units has almost doubled during the last 8 weeks, from about .75% the first week to 1.4% in the 8th week. A second trend is a trend within each week, with low defects on Monday and increasing each day of the week, with the most defects produced on Friday.

2.

It is essential to arrest and reverse the overall trend toward more defective units. Even by the first week of this eight-week period, the defective rate was well above the target of .5%, and it grew each of the eight weeks. The control chart will not tell what actions are needed to reverse this trend, but it focuses attention on the problem and allows managers to explore potential solutions. The weekly pattern is also disturbing. There is no reason that defect rates must increase as the week goes on. Apparently employees come in refreshed on Monday and are quite attentive to quality. This attentiveness drops steadily until, by Friday, they don't seem to pay as much attention to quality. Incentives for better quality late in the week might be effective, or the company may try changing the work patterns so that employees are not bored, tired, or whatever else besets them by Friday.

..

388


9-41

(20-30 min.) Units Completed 564 544 553 571 547 552

Week 1 2 3 4 5 6

Total Cycle Time 14,108 14,592 15,152 16,598 17,104 16,673

Average Cycle Time 25.0 26.8 27.4 29.1 31.3 30.2

The cycle time objective was met only in the first week. After that, however, cycle time generally has steadily increased. With knowledge of the acceptable control limit, an analyst probably could determine within the third or fourth week that cycle time is tending to be out of control. Corrective action could have been initiated before increased cycle times lead to higher costs and possible difficulty in meeting schedules. A control chart approach shows the increasing cycle time graphically:

Hours

Average Cycle Time 35 30 25 20 15 10 5 0 0

2

4

6

Week

9-42

(20-35 min.) The purpose of this problem is to get students to recognize that measurements affect behavior and that financial measures tend to place too much focus on shortrun results. An exhaustive study of the eight goals is impossible and unwarranted at this stage of the course. The aim is to provide an overview, a perspective on where accounting often fits in management control.

1.

Students may add many alternative measurements in each of the following categories. a.

..

Profitability. Total dollars of profit. Percentage of profit on sales. Rate of return on investment. Residual income. Note that General Electric chose residual income, which is described in Chapter 10. Using residual income, a manager maximizes an absolute amount (residual income) rather than a rate. 389


Regardless of the alternative chosen, another question is whether the measurement should be based on historical costs, replacement costs, net realizable value, or some other alternative. b.

Market position. Share of served markets. How is a market defined? For example, should the market consist of that served by the electric-range industry only? Or should it consist of all ranges, including gas ranges? Note that this area is important because a division could be showing handsome profitability yet simultaneously be losing its share of the market.

c.

Productivity. This measure attempts to gauge efficiency. Productivity focuses on physical and/or financial relationships between inputs and outputs. Through the years, there have been various attempts, both by General Electric and others, to measure productivity for a division (as opposed to an individual worker or small cost center, where the measurement difficulties are less imposing). G.E. has changed its approach through the years, and it still has not found a completely satisfactory measure. As a rule, all measures should be adjusted for changes in unit prices because price changes should not affect measures of productivity. G.E. believes that productivity must be tied to all the factors of production, not just labor alone. For example, the company has used the following measure: _________Sales Billed_________ Employee Compensation + Facilities Charge + Direct materials Costs + Business Services costs

..

d.

Product leadership. This still tends to be a qualitative evaluation, but at least it should be conducted routinely on a standard evaluation form. Among the questions asked are: How does each product compare with its competition and general company standards? Where is the research conducted? Who introduced the basic product (for example, did G.E. or Westinghouse introduce the electric toothbrush)? Trends are important.

e.

Personnel development. The objective of this area is to assure the steady flow of promotable employees. An inventory of various executive positions is taken to see whether orderly succession in the hierarchy is likely. Among the various programs that are evaluated are: recruitment, training, review, and counseling.

f.

Employee attitudes. Among these measurements are employee turnover, absenteeism, and results of attitude surveys.

390


g.

Public responsibility. Measures are routine but less precise, as you might expect. Explicit surveys of executive participation in community affairs and public attitudes are used.

h.

Balance between short-range and long-range goals. This balance is not measured separately. It is included in the eight-point list to reinforce the basic idea of the entire measurements project. Note that areas (b) through (g) essentially counteract the built-in tendency of the accounting profitability measurements to stress short-range results.

2.

By its very reliance on the other goals, G.E. must believe that profitability cannot encompass all the other goals. Nevertheless, profitability is usually dominant and other goals play a secondary (though still important) role.

9-43 1.

(30 min.) Glacial Refrigeration Inc.

Sales Cost of sales Parts and service materials Parts and service labor Sales commissions Subtotal Mark-up on “variable” material and labor* Parts and service overhead Advertising Sales salaries General dealership overhead Net income

Parts and Service €600,000 €120,000 240,000 €360,000

Refrigerator Sales €3,720,000 €2,976,000 186,000 €3,162,000

€240,000 € 48,000 192,000** € 0

€ 558,000 € 120,000 105,600 24,000 € 308,400

*Roughly equivalent to contribution margin. ** €240,000 – €48,000 = 192,000. Following the management’s view that the parts and service operation exists only to recover costs, this amount is allocated to parts and service so that the income (after allocated overhead) is exactly zero. This approach also implies that the remaining €24,000 of general overhead is allocated to refrigerator sales. 2.

The operating statement by departments would be the same as (1) through “markup on variable material and labor.” At that point general overhead should be allocated €36,000 to parts and service and €106,800 to refrigerator sales. The remaining €73,200 of general overhead should not be allocated at all. The bottom of the income statement would appear as follows:

..

391


Markup on “variable” material and labor Parts and service overhead Advertising Sales salaries Direct allocation of general overhead Total expenses directly charged to departments Departmental contribution to net income General overhead not allocable Net income of the company as a whole 3.

Parts and Service

Refrigerator Sales

€240,000 € 48,000 -

€558,000 € 120,000 105,600

36,000

106,800

€ 84,000

€332,400

416,400

€156,000

€225,600

€381,600 73,200

Total €798,000

€308,400

Note that both approaches yield exactly the same evaluation of income for the company as a whole. The first approach regards only one function of the company as a source of net income and therefore allocates just enough general overhead to parts and service to reduce its profit to zero. The remainder is allocated to refrigerator sales, which is viewed as the single source of net income. A more modern approach is to view a company as having two profit centers (segments of a business that have revenue as well as expenses), refrigerator sales, and parts and service. Each segment is regarded as an independent venture. Each contributes its individual segment margin to the overall dealership overhead that cannot be directly assigned to a specific profit center.

..

392


9-44

(35-45 min.)

1.

A = 800,000 F = 800,000 Performance

2.

X = Y =

F = 700,000 = = = =

A = 800,000 Performance

Z = 1.00

= (Y × F) + [X × (A – F)] = (.80 × 800,000) + .50(800,000 - 800,000) = 640,000

A = 800,000 Performance

.50 .80

(Y × F) + [X × (A – F)] (.80 × 700,000) + .50(800,000 - 700,000) 560,000 + 50,000 610,000 F = 900,000

= = = =

(Y × F) - [Z × (F – A)] (.80 × 900,000) - 1.00 × (900,000 - 800,000) 720,000 - 100,000 620,000

Notice that when F < A, increasing F by one TV gains Y and loses X, a net gain of Y - X = .80 - .50 = .30. When Chavez predicts production 100,000 TVs below actual, it costs him a .30 × 100,000 = 30,000 point drop in the performance measure. Therefore, there is an incentive not to predict a volume below the expected actual volume. Likewise, when F > A, decreasing F by one TV gains Z and loses Y, a net gain of Z - Y = 1.00 - .80 = .20. The overly optimistic forecast causes a .20 × 100,000 = 20,000 point drop in the performance measure. There is an incentive not to predict a volume above the expected actual volume. The system motivates a forecast equal to the expected actual volume. 3.

..

When actual volume falls short of target, additional production increases the performance measure by Z = 1.0 per TV. It is worthwhile to achieve as much production as possible. When actual volume exceeds forecast, additional production increases the production measure by .50 per TV, still creating an incentive for continued production.

393


9-45 (15-20 min.) Students may suggest a variety of measures. There is not a single right measure for each objective. Listed below are some possible measures: 1. Maintain strong financial health a. Daily cash balance b. Percentage increase in sales and income c. Return on investment or residual income or EVA (discussed in Chapter 10) 2. Provide excellent service to customers a. Customer satisfaction surveys b. Average time from receipt of order to shipping c. Percent of products returned by customers, or amount of allowances for quality defects d. Number and dollar amount of exclusive supplier agreements 3. Be among the industry leaders in product and process innovations a. Percent of sales from products less than 2 years old b. Dollars (or percent of sales) spent on process improvements 4. Develop and maintain efficient, state-of-the-art production processes a. Cost per unit b. Average delay from projected date of availability to actual delivery 9-46 1.

(20-30 min.) (a) Prevention cost: This includes costs incurred to prevent the production of defective units, such as programs to train personnel, simplified production processes, and improved production planning. These costs have increased between 2018 and 2020 both in absolute amount and as a percentage of total quality cost. Apparently, more attention is now being given to the prevention of defects. (b) Appraisal cost: These costs are incurred to identify defective products. They include testing, inspection, and various other quality control procedures. Although these costs were a larger percentage of total quality costs in 2020 than in 2018, their total amount has increased in almost direct proportion to the increase in product cost. Therefore, appraisal procedures have probably remained much the same as in 2018. (c) Internal failure cost: These are the costs of items scrapped and the costs of rework to correct defects in products. These costs are up slightly as a percentage of total quality costs but down significantly in absolute amount. Despite higher product costs, much less is being spent on defective units. This is most likely to mean that the money spent on prevention has decreased the number of defective units being scrapped or reworked. (d) External failure cost: These are costs caused by delivery of defective units, including warranty expenses and sales returns and allowances. There has been a dramatic decrease in this cost, probably because fewer defective units are being delivered to customers.

2.

The overall costs of quality are much lower in 2020 than in 2018. The decrease comes primarily in the two categories of internal and external failure costs. Green

..

394


Star is following a popular approach to modern quality control: preventing defects is less costly than identifying and correcting them. By increasing spending on prevention of defects, Green Star has reduced overall quality costs. In addition to the costs in the quality cost report, companies should be concerned with potential lost sales if customers receive a large proportion of defective units. Green Star’s decrease in external failure costs probably means that it is delivering fewer defective units, and therefore there will also be smaller opportunity costs due to lost sales. 9-47

(20–30 min.)

From a customer perspective, simply lowering the mean delivery time without any change in the variability does little to address customers’ concerns about certainty of delivery dates. To see this, compute the mean and standard deviation before Six Sigma and compare them to the mean and standard deviation after Six Sigma. Standard Deviation Mean

Before Six Sigma 10.4 22

After Six Sigma 6.3 11

Both the mean delivery time and the standard deviation of delivery times have been reduced. The reduced mean implies that a customer can expect delivery sooner and the reduced standard deviation implies the customer can also be more certain that the delivery will take place within a specified range than before the Six Sigma improvement. This is an important factor for planning purposes. 9-48

(20-30 min.)

This problem is similar to the problem in the chapter, with a slight difference. The purchase of Excel Web and the pooling of its operating statistics may be misleading because of fundamental differences in operations. 1.

Trio only

2019

No. of broadband subscribers No. of employees Subscriptions per employee

7,393,500 34,875 212

2020 7,685,000 36,250 212

Productivity in 2020 remained at the same level as in 2019.

..

395


2.

Trio With Excel Web No. of broadband subs. 10,730,000 No. of employees 58,000 Subs. per employee 185

Excel Web Only 10,730,000 – 7,685,000 = 3,045,000 58,000 – 36,250 = 21,750 140

The low productivity of Excel Web reduces the productivity of the combined company. 3.

The employees of the acquired company probably will not be able to immediately achieve the level of productivity achieved by Trio’s other employees. Attempts to force a rapid increase in their productivity could lead to problems such as labor unrest and political difficulties. A dramatic increase in productivity at Excel Web will probably also require considerable investment in improved technology and in the education and training of employees.

9-49

(20-30 min.)

1.

The best productivity measure based on the physical measures given is: number of wallets produced ÷ direct-labor-hours worked. Comparing 2018 and 2020: 2018 213,000 ÷ 66,750 = 3.19 wallets/hour

2020 225,750 ÷ 67,675= 3.34 wallets/hour

Productivity has increased by 3.34 – 3.19 = 0.15 wallets/hour, an increase of 0.15 ÷ 3.19 = 4.70% 2.

The best productivity measure based on the financial measures given is: sales revenue ÷ direct labor cost. Comparing 2018 and 2020, we get the following sales per dollar of direct-labor cost: 2018 £10,350,000 ÷ £4,770,000 = £2.17

2020 £15,360,000 ÷ £6,000,000 = £2.56

By this measure, productivity has increased by 0.39 ÷ 2.17 = 17.97%. Three factors contribute to this increase: 1) increase in physical productivity, as shown in requirement 1; 2) increase in revenue per wallet at a rate greater than inflation (£10,350,000 ÷ 213,000 = £48.59 in 2018 compared to £15,360,000 ÷ 225,750 = $68.04 in 2020, an increase of (£68.04 - £48.59) ÷ £48.59 = 40.03%); and 3) increase in wage rates per hour more than inflation (£4,770,000 ÷ 66,750 = £71.46 in 2018 compared to £6,000,000 ÷ 67,675 = £88.66 in 2020, an increase of (£88.66 - £71.46) ÷ £71.46 = 24.07%).

..

396


3. This productivity measure mixes financial and physical measures. Therefore, it is essential to adjust the financial portion for inflation. Expressing both 2018 and 2020 productivity measures in 2020 dollars: 2018 (£10,350,000 × 1.13) ÷ 66,750 = £175.21/hour

2020 £15,360,000 ÷ 67,675 = £226.97/hour

The inflation-adjusted measure shows an increase in productivity of (£226.97 £175.21) ÷ £175.21 = 29.54%. It incorporates the increase in physical productivity and the revenue increase at a rate greater than the inflation rate. 9-50 (30 min.) There are numerous approaches to this case. Here is one possible solution. An alternative solution would be to try to increase the productivity of the employees. 1 & 2. Rico Estrada is faced with difficult tradeoffs. His sub goal of retaining a skilled and motivated work force is threatened by new, competitive pressures. As Estrada loses accounts, he is spreading his (discretionary fixed) labor costs over fewer accounts, and the average cost rises. If he tries to maintain his customary 25% markup, Estrada will become less competitive and probably will lose even more accounts. This has been termed the cost "death spiral," and if left unchecked it could lead to bankruptcy. Estrada must find a solution or he eventually will not be able to cover his costs. It is likely that Estrada can maintain quality service and customer satisfaction with a reduced work force. By November it appears that Estrada has 41 - (680 ÷ 20) = 7 excess employees unless this downturn in business is only temporary. This excess employment is costing Estrada 7 × $3,000 = $21,000 per month (ignoring taxes and fringe benefits). If Estrada could save this amount, he could reduce the average cost per account as shown: Number of accounts Average monthly cost per account

680 $191

Total monthly cost (680 × $191) Less: salary savings Revised total monthly cost

$129,880 21,000 $108,880

Revised average cost per account = $108,880 ÷ 680

$ 160.12

CDS’s current price: Average cost Markup @ 25% CDS’s price

$191.00 47.75 $238.75

Competitor's price, 81% × $238.75

$193.39

..

397


Possible CDS Responses: a. Maintain current markup of 25%: Target cost Total cost Required additional cost reduction Additional employee reduction required @ $3,000 each b.

Reduce markup to 15%: Target cost Total cost Required cost reduction

$193.39 ÷ 1.25 = $154.71 680 × $154.71 = $105,203 $108,880 - $105,203 = $3,677 Approximately 1.2 $193.39 ÷ 1.15 = $168.17 680 × $168.17 = $114,356 None

To maintain the current 25% markup, Estrada would have to achieve a target cost of $154.71, but that would entail further cost savings of $3,677. Laying off one employee would save almost enough, and laying off two employees would save more than enough. Estrada could avoid further layoffs by reducing his desired markup, but then the business may not be as attractive to him. It is likely that the business will become even more competitive on the service dimensions, so a skilled, motivated work force will be critical to keeping current customers and regaining lost customers. Can Estrada reduce his work force and maintain the loyalty of the remaining employees? This will be difficult, but it may be necessary, and it may be at least partly accomplished through attrition and/or early retirements. The equipment lease in August was probably in response to business growth, which now appears to be unnecessary. Can Estrada get out of the lease? If so, he may be able to cut costs further and/or retain some employees that otherwise would be laid off. Perhaps the best approach would be for Estrada to present the work force with the magnitude of the problem and enlist their aid in solving it. There are numerous stories in the business press about innovative solutions developed by employees who are able to achieve significant productivity increases. This could even lead to a purchase of the company by the employees.

..

398


9-51

(30-40 min.)

Answers will vary. At one point, the following quotations appeared on the companies’ Web sites. These quotes may have been replaced by the time this problem is assigned. Nevertheless, it is very likely that each Web site will contain references to Six Sigma because it is a central tenet to the operations in each of the four companies. Motorola – “When practiced as a management system, Six Sigma is a high performance system for executing business strategy. Six Sigma is a top-down solution to help organizations: · Align their business strategy to critical improvement efforts · Mobilize teams to attack high impact projects · Accelerate improved business results · Govern efforts to ensure improvements are sustained The Six Sigma Management System drives clarity around the business strategy and the metrics that most reflect success with that strategy. It provides the framework to prioritize resources for projects that will improve the metrics, and it leverages leaders who will manage the efforts for rapid, sustainable, and improved business results.” GE – “Today's competitive environment leaves no room for error. We must delight our customers and relentlessly look for new ways to exceed their expectations. This is why Six Sigma Quality has become a part of our culture. First, What is Six Sigma? First, what it is not. It is not a secret society, a slogan or a cliche. Six Sigma is a highly disciplined process that helps us focus on developing and delivering nearperfect products and services. Why "Sigma"? The word is a statistical term that measures how far a given process deviates from perfection. The central idea behind Six Sigma is that if you can measure how many "defects" you have in a process, you can systematically figure out how to eliminate them and get as close to "zero defects" as possible. To achieve Six Sigma Quality, a process must produce no more than 3.4 defects per million opportunities. An "opportunity" is defined as a chance for nonconformance, or not meeting the required specifications. This means we need to be nearly flawless in executing our key processes. Key Concepts of Six Sigma At its core, Six Sigma revolves around a few key concepts. Critical to Quality: Attributes most important to the customer Defect: Failing to deliver what the customer wants Process Capability: What your process can deliver Variation: What the customer sees and feels Stable Operations: Ensuring consistent, predictable processes to improve what the customer sees and feels ..

399


Design for Six Sigma: Designing to meet customer needs and process capability 3M – “3M employees continuously challenge the upper limits of product reliability and capability through process and product innovation and the application of proven quality principles. Underlying this effort is a strong corporate commitment to the Six Sigma strategy for achieving breakthrough performance in all areas of our business. Six Sigma is a disciplined methodology of continuous improvement, which requires thorough process and product understanding to reduce inherent variability or defects. It is clearly focused on customer-driven expectations and on data-driven decisions.” Dow Chemical – “Dow began its implementation of Six Sigma in 1999. In each subsequent year, Dow has continued its Six Sigma commitment with renewed vigor. Through our implementation of Six Sigma, Dow has gained increasing value while equipping employees with critical problem-solving skills and a mindset for reducing variation and defect.” “We're also applying our Six Sigma mindset to improve our social performance – because we view employee dissatisfaction and shortcomings in community relations as defects in our operations, the same as waste generation or shortfalls in plant productivity. “

..

400


9-52

(60-90 min.)

This problem provides a comprehensive review of many of the techniques and terms that were introduced in previous chapters. It might be used as a final examination. You may wish to skip part (7). Some answers are based on the following detailed master budget: Product Sales, 50,000 at $9.00 and 70,000 at $6.00 Variable manufacturing costs at $7.50 and $3.00 Contribution margin Fixed discretionary manufacturing costs Contribution controllable by product managers Fixed committed manufacturing costs Contribution by products* Unallocable fixed costs: Manufacturing (committed) Selling and administrative (discretionary) Selling and administrative (committed) Total unallocable fixed costs Operating income

A $450,000 375,000 $ 75,000 4,500 $ 70,500 40,500 $ 30,000

B $420,000 210,000 $210,000 8,500 $201,500 76,500 $125,000

Division $870,000 585,000 $285,000 13,000 $272,000 117,000 $155,000 $ 25,000 72,000 48,000 $145,000 $ 10,000

*This is the answer to part (2). Note: Fixed manufacturing costs = $740,000 - $585,000 = $155,000, subdivided into components of $13,000 + $117,000 + $25,000 = $155,000.

..

401


Answers to requirements: 1. Contribution margin ratio: $285,000 ÷ $870,000 = .327586 Break-even point: ($13,000 + 117,000 + $145,000) ÷ .327586= $839,474 Contribution margin per unit, A: $9.00 - $7.50 = $1.50 Contribution margin per unit, B: $ 6.00 - $3.00 = $3.00 2.

See the footnote to the Master Budget above.

3.

Product __ A Selling and administrative expenses: Discretionary, 53/117 and 64/117 Committed, 50/120 and 70/120 Totals

$32,615 20,000 $52,615

B

Total

$39,385 28,000 $67,385

$ 72,000 48,000 $120,000

There is an arbitrary distinction between the allocation bases. The purpose of this part is to ask whether budgeted or actual numbers should be used as bases for allocating these costs. The chapter discusses this issue. Another point worth discussing is whether the actual costs or only the budgeted costs should be allocated. The answer often depends on the extent of controllability by the product managers (if any controllability exists). If the product managers have zero influence over the level of costs, they should not be allocated. 4.

This raises the issue of incentives and goal congruence. Product A has the higher selling price but the lower contribution margin ($9.00 and $1.50 for A versus $6.00 and $3.00 for B). The resulting incentives to push the higher-priced product will likely contribute less to the firm's overall profit performance (all other things equal).

5.

Actual results were: Product __A _ Sales, 53,000 units at $9.00 and 64,000 units at $6.00 Variable manufacturing costs: Material Labor Overhead Total variable manufacturing costs Contribution margin Fixed manufacturing costs* Fixed selling and administrative costs Operating income

..

B

Total

$477,000

$384,000

$861,000

$134,500 156,350 108,650 $399,500

$102,400 50,000 50,000 $202,400

$601,900 $259,100 147,300 116,000 $ (4,200)

402


*The $749,200 total manufacturing costs given in the problem minus $601,900 of variable manufacturing cost, also given, equals $147,300. The "controllable contribution" is the actual contribution margin less the fixed discretionary costs, which would be: Actual contribution margin Total actual fixed costs Less committed fixed costs, which are the same as those budgeted (because there are no variances), $117,000 + $25,000 + $48,000 Contribution controllable by segment managers *Selling & administrative expenses Fixed manufacturing costs, $749,200 - $601,900 Total actual fixed costs 6.

$259,100 $263,300*

190,000

73,300 $185,800

$ 116,000 147,300 $263,300

The analysis rests solely on master budgeted sales and costs versus actual sales and costs at budgeted unit prices:

Product A: Sales Variable costs Contribution margin Product B: Sales Variable costs Contribution margin Contribution margin for both products

Actual Sales at Budgeted Prices

Budgeted Sales at Budgeted Prices

Sales Activity Variance

$477,000 397,500* $ 79,500

$450,000 375,000 $ 75,000

$27,000 F 22,500 U $ 4,500 F

$384,000 192,000** $192,000

$420,000 210,000 $210,000

$36,000 U 18,000 F $18,000 U

$271,500

$285,000

$13,500U

*53,000 units × $7.50 **64,000 units × $3.00

..

403


7.

Product A Direct materials

Cost Incurred: Actual Input Quantities × Actual Prices

Actual Input Quantities × Standard Prices

Flexible Budget: Standard Input Quantities Allowed for Outputs Achieved × Standard Prices

538,000 pieces × $.25 = $134,500

538,000 pieces × $.25 = $134,500

530,000 pieces × $.25 = $132,500

Price variance, 0 Quantity variance, $2,000U Flexible-budget variance, $2,000U Labor

53,000 hours × $2.95 = $156,350

53,000 hours × $3.00 = $159,000

53,000 hours × $3.00 = $159,000

Price variance, $2,650F Quantity variance, 0 Flexible-budget variance, $2,650F Variable overhead

53,000 hours × $2.05 = $108,650

53,000 hours × $2.00 = $106,000

53,000 hours × $2.00 = $106,000

Spending variance, $2,650U Efficiency variance, 0 Flexible-budget variance, $2,650U

..

404


Product B Direct materials

320,000 lbs. × $.32 = $102,400

320,000 lbs. × $.30 = $96,000

320,000 lbs. × $.30 = $96,000

Price variance, $6,400U Quantity variance, 0 Flexible-budget variance, $6,400U Labor

20,000 hours × $2.50 = $50,000

20,000 hours × $2.50 = $50,000

19,200 hours × $2.50 = $48,000

Price variance, 0 Quantity variance, $2,000U Flexible-budget variance, $2,000U Variable overhead

20,000 hours × $2.50 = $50,000

20,000 hours × $2.50 = $50,000

19,200 hours × $2.50 = $48,000

Efficiency variance, Spending variance, 0 $2,000U Flexible-budget variance, $2,000U

..

405


Check: Material Labor Variable overhead Totals

Product A $2,000U 2,650F 2,650U $2,000U

Total actual variable costs [item (6) in problem statement] Standard variable costs: Product A: 53,000 × $7.50 Product B: 64,000 × $3.00 Total variance Summary of all variances: Budgeted operating income Variances: Sales-activity variance Price and quantity/efficiency variances for variable costs Budget variance for fixed costs: Actual* Budgeted** Total variances Actual operating loss

Product B $ 6,400U 2,000U 2,000U $10,400U

Total

$12,400U $601,900

$397,500 192,000

589,500 $ 12,400U $10,000

$13,500U 12,400U $263,300 275,000

11,700F 14,200U ($4,200)

*See actual results in solution to requirement 5. **$13,000 + $117,000 + $145,000.

..

406


9-53 (20 – 30 min.) NOTE TO INSTRUCTOR: This solution is based on the 2012 10-K included in Appendix C. A more recent 10-K could be assigned to get an updated view of strategy. Note, however, that the five key areas and the four financial goals have remained constant for a number of years. 1.

According to the Nike 10-K, the five key areas are: a) Making the supply chain a competitive advantage, through operational discipline b) Reducing product costs through a continued focus on lean manufacturing and product design that strives to eliminate waste c) Improving selling and administrative expense productivity by focusing on investments that drive economic returns in the form of incremental revenue and gross margin, and leveraging existing infrastructure across our portfolio of brands to eliminate duplicative costs d) Improving working capital efficiency e) Deploying capital effectively to create value for our shareholders

2.

The four long-term financial goals are: a) High single digit revenue growth b) Mid-teens EPS growth c) Increased return on invested capital and accelerated cash flows d) Consistent results through effective management of our diversified portfolio of business

3.

Over the past ten years, all of the financial goals have been met. Revenues grew 8%, and EPS grew 15%. Return on invested capital has increased from 14% to 22%, and gross margins grew by more than 5 percentage points.

4.

Students might suggest a variety of non-financial goals from the customer perspective, business process perspective, or innovation and learning perspective.

9-54 (45 min.) For the solution to this Excel Application Exercise, follow the stepby-step instructions provided in the textbook chapter. 1. Salesclerk A’s average salary of ¥129,185.7 is greater than B’s ¥101,062.5. 2. Just meeting the quota has a big payoff. Although B’s total sales of ¥4,575,000 were only 41% of A’s ¥11,250,000, B’s salary was 78% of A’s because B met the quota three times but only exceeded the quota once, and that was by only ¥5,000. 3. Having a big jump in salary at just meeting the quota can cause problems, including possibly manipulation of sales numbers.

..

407


9-55

(60 min. or more) The purpose of this exercise is to develop goals and objectives for a familiar organization. By working in teams, students may see the possibly conflicting objectives of various stakeholder groups. They will also see how difficult it can be to develop measures for some seemingly obvious goals. For example, quality of education is certainly a goal of a university department. But how does one measure this quality? Standardized tests are often suggested, but they may motivate "teaching to the test" rather than generating overall quality. Eventual success in a career might be used, but this measure is available only after a long delay. If the optional interview is obtained, it will be useful to see how the faculty member's goals and objectives differ from those of the student group. Does the faculty member have a different perspective? Would legislators (for a state university) or a board of trustees (for a private college or university) have an even different perspective? What about the staff of the university? The interview might lead to a better understanding of how difficult it is to set goals and objectives for an organization with many diverse stakeholders. 9-56 (20 – 30 min.) NOTE TO INSTRUCTOR: This solution is based on the web site as it existed in late-2012. Be sure to examine the current web site and annual report before assigning this problem, as the information there may have changed. 1.

The factors driving their growth strategy are “strengthening our core business, renewing our focus on discontinuous innovation, and implementing a $10 billion productivity program,” all discussed in more detail in the letter to shareholders. According to the 2012 annual report: “We have the right metrics to incent results that are aligned with shareholder objectives. Our long-term bonus metrics are simple: organic sales growth relative to competition, operating earnings growth, EPS growth and free cash flow productivity — all of which drive total shareholder return. We’ve aligned the entire Company next year on short-term metrics of volume and sales growth, market share, operating profit growth, productivity (which is delivering against the $10 billion plan), cash flow and internal controls. These are the metrics that we will measure ourselves against and that you can measure our progress by.”

2.

P&G lists two major categories under Brands: (1) Beauty and Grooming, and (2) Household Care. The household care category lists 32 brands (including Mr. Clean, Bounty, Swiffer, and Febreze products). Bounty and Mr. Clean are wellestablished brand names with several specific products under each brand name. Swiffer and Febreze are newer, less well-known brands that are increasing in brand awareness. Brand building is one of P&G’s five core strengths. Financial measures of success in building brands might include gross margin, contribution margin, product-line return on investment, residual income or economic value added for major business segments or the company as a whole. Nonfinancial measures might include market share, increase in market share for key brands, brand recognition, brand loyalty, and number of new markets.

..

408


CHAPTER 10 COVERAGE OF LEARNING OBJECTIVES

FUNDAMENTAL ASSIGNMENT MATERIAL A2, A4, B4

LEARNING OBJECTIVE LO1: Define decentralization and identify its expected benefits and costs. LO2: Distinguish between B4 responsibility centers and decentralization. LO3: Explain how the linking B4 of rewards to responsibility center results affects incentives and risk. LO4: Compute ROI, A1, A4, B2, B4 economic profit and economic value added (EVA). LO5: Compare the incentives created by income, ROI, and EVA performance measures. LO6: Define transfer prices and identify their purpose. LO7: State the general rule for transfer pricing and use it to assess transfer prices based on total costs, variable costs, and market prices. LO8: Identify the factors affecting multinational transfer prices. LO9: Explain how controllability and management by objectives (MBO) aid the implementation of management control systems.

..

ADDITIONAL ASSIGNMENT MATERIAL 5, 25, 26

CASES, EXCEL, COLLAB., & INTERNET EXERCISES 59, 60

46, 46

55

6, 23, 37, 43

27, 27, 28, 28, 29, 30, 30, 31, 31, 32, 33, 33, 38, 38, 40, 40, 41, 42, 44 24, 25, 39, 39, 40, 40, 43, 43, 44, 44, 45, 54

57, 58

55

A2, A3, A4, B1, B3, B4

26, 46, 46, 47, 48, 49, 49, 50, 51, 51, 54 18, 34, 34, 35, 46, 46, 47, 48, 49, 49, 52, 54

B4

36, 53, 53

B1, A2, A3, A4, B3, B4

B4

55

56

409


CHAPTER 10 Management Control in Decentralized Organizations 10-A1 (10-15 min.) Dollar amounts are in thousands. Hubert Return on sales: $180 ÷ $3,600 $84 ÷ $1,200 $216 ÷ $9,000 Capital turnover: $3,600 ÷ $2,000 $1,200 ÷ $600 $9,000 ÷ $1,800 Rate of return on invested capital: $180 ÷ $2,000 (or 1.8 × 5%) $84 ÷ $600 (or 2 × 7%) $216 ÷ $1,800 (or 5 × 2.4%) 2.

Division Duane

Louis

5% 7% 2.4% 1.8 2 5 9% 14% 12%

If ROI is used for judging relative performance, Duane is best for this period. Other factors deserving discussion include the risks faced by each division and the short-run versus long-run implications of current performance.

3. Income Cost of capital Economic profit

Hubert $180 200 $ –20

Division Duane $84 60 $ 24

Louis $216 180 $ 36

Louis Division has the highest economic profit, even though it has a slightly lower ROI than does Duane. Hubert Division is earning less than its cost of capital and has a negative economic profit.

..

410


10-A2 (30 min.) 1.

Assume that fixed costs will continue. The company as a whole will be worse off if Lucerne buys from outside supplier: Purchase costs from outside supplier, 3,000 units at CHF300 Less: Savings in variable costs by reducing Geneva's output, 3,000 at CHF280 Disadvantage to company as a whole from purchase

2.

CHF900,000

925,000 CHF 25,000

Company will benefit if Lucerne buys on outside: Purchase costs from outsider, 3,000 units at CHF250 Less: Savings in variable costs by reducing Geneva’s output, 3,000 at CHF 280 (as above) Advantage to company as a whole from purchase

4.

840,000 CHF 60,000

Company will benefit if Lucerne buys outside: Purchase costs from outsider, 3,000 units at CHF300 Less: Savings in variable costs as above CHF840,000 Contribution from other production operations 85,000 Advantage to company as a whole from purchase

3.

CHF900,000

CHF750,000 CHF840,000 CHF 90,000

As president, I would not want to become immersed in these disputes. If arbitration is necessary, it probably should be conducted by some other officer on the corporate staff. One possibility is to have the immediate line boss of the two managers make a decision. If decentralization is to be strictly adhered to, the arbitrator probably should not intervene under any of the conditions described. If no forced transfer were made, Lucerne would probably go outside, resulting in an optimal decision for the overall company under the assumptions in parts (2) and (3) but not under the assumptions in part (1). Of course, in part (1) if the manager of Geneva understood cost-volume-profit relationships and wanted to maximize the division’s short-run net income, he or she would probably accept a price of CHF300. This would bring a contribution to the divisional profit of 3,000 × (CHF300 - CHF280), or CHF60,000.

..

411


Suppose, however, that Geneva refuses to meet the price of CHF300. This would mean that the company will be CHF60,000 poorer in the short run. Should top management interfere and force a transfer at CHF300? This would undercut the philosophy of decentralization. Many managers would not interfere because they would view the CHF60,000 as the price that has to be paid for true decentralization. But how high must this price go before the temptation to interfere would be irresistible? CHF70,000? CHF80,000? How much? On the other hand, the Geneva manager may realize that CHF60,000 is being sacrificed but may have decided that it is worth more than CHF60,000 to achieve some long-term subjective benefits. In sum, the point of this question is that any structure that interferes with lowerlevel decision-making weakens decentralization. Of course, such interference may occasionally be necessary to prevent horrendous blunders. But recurring interference and constraints simply transform a decentralized organization into a centralized organization. 10-A3 (10 min.) The company as a whole would benefit because the CHF60,000 disadvantage of purchasing from an outside supplier calculated in 10-A2 Part 1 would be more than offset by the additional contribution margin on sales by Geneva to other customers: Geneva's sales to other customers, 3,000 units at CHF330 Variable costs, at CHF305 Contribution margin

CHF990,000 915,000 CHF 75,000

The net advantage would be CHF75,000 - CHF60,000, or CHF15,000. 10-A4 (30-35 min.) 1. a. 20% of $960,000 = $192,000 target operating income Let X

= Unit sales price

Dollar sales = Variable expenses + Fixed expenses + Operating income 150,000X = (150,000 × $.72) + $300,000 + $192,000 X = $600,000 ÷ 150,000 = $4.00

..

b.

Expected capital turnover = $600,000 ÷ $960,000 = .625

c.

Return on sales = $192,000 ÷ $600,000 = 32%

412


2.

a, b.

Sales, at $4.00 Variable expense, at $.72 Fixed expenses Total expenses Operating income Rate of return on $960,000 assets

150,000 Units*

Sales Volume 170,000 Units

130,000 Units

$600,000 $108,000 300,000 $408,000 $192,000

$680,000 $122,400 300,000 $422,400 $257,600

$520,000 $ 93,600 300,000 $393,600 $126,400

20.0%

26.83%

13.17%

*Column not required. A summary analysis of these three cases, in equation form, follows:

Volume 150,000 Volume 170,000 Volume 130,000 3.

Return Rate of on Sales

Capital Turn×

32.00% 37.88% 24.31%

× × ×

over = .6250 .7083 .5417

Return

= 20.0% = 26.83% = 13.17%

If the units are not sold to the other division, average available assets would decrease by $150,000, from $960,000 to $810,000. Fixed overhead would be reduced to $300,000 - $22,500 = $277,500. Results would be: Sell Sell Difference 105,000 150,000 45,000 Units Units Units Sales, 105,000×$4.00; 45,000×$2.25 $420,000 $521,250 $101,250 Variable expenses, at $.72 $ 75,600 $108,000 $ 32,400 Fixed expenses 277,500 300,000 22,500 Total expenses $353,100 $408,000 $ 54,900 Operating income $ 66,900 $113,250 $ 46,350 Total assets needed $810,000 $960,000 $150,000 Rate of return on assets 8.26% 11.80% 30.9% Based on the information given, he should sell at the $2.25 price. Both divisions and the company as a whole will benefit from such a decision. Although the original overall target rate of return of 20% is unattainable, the division will nevertheless earn a better rate of return on assets with the intracompany business than without it. The additional units will earn a 30.9% incremental rate of return, which exceeds the 8.26% rate earned on 105,000 units. As a result, the overall rate of return would increase from 8.26% to 11.80%, as shown in the schedule above.

..

413


Despite this economic analysis, the Austin Division manager may still decide against transferring goods at such a low price. For example, he may feel entitled to a higher profit. This would mean that the company would be worse off, assuming the incremental costs of the other division are $2.25. Should top management interfere and force a transfer of $2.25? Such intervention would weaken the decentralization structure. Obviously, authoritarian action sometimes may be needed to prevent costly mistakes. But recurring interference by top management effectively transforms a decentralized organization into a centralized organization. Of course, if managers repeatedly make costly dysfunctional decisions, a more centralized organizational design may be desirable. 10-B1 (30-45 min.) 1.

The percentage return for each project is as follows: Project 1 2 3 4 5 6

Percentage Return $1,200,000 ÷ $4,800,000 = 25% $ 627,000 ÷ $1,900,000 = 33% $ 182,000 ÷ $1,400,000 = 13% $ 152,000 ÷ $ 950,000 = 16% $ 136,500 ÷ $ 650,000 = 21% $ 90,000 ÷ $ 300,000 = 30%

a. Under assumption (a), projects 1, 2, 4, 5, and 6 would be taken. Total investment Total return Return on investment Economic profit *$2,205,500 - ($8,600,000 × .20)

$8,600,000 $2,205,500 25.65% $ 485,500*

The manager taking the above projects would be following the company rule. b. Under assumption (b), the rational manager will take only project 2, since this gives a return on investment of $627,000 ÷ $1,900,000 = 33% (and an economic profit of $627,000 - ($1,900,000 × .20) = $247,000). Adding any further projects at lower returns lowers the overall return on capital invested. Note that if this were not a new division, the manager under this alternative would select those projects that have a higher expected rate of return than the existing rate of return.

..

414


c. Under assumption (c), the manager will take projects 1, 2, 5 and 6, which have returns greater than the cost of capital. Total investment Total income Return on investment (2,053,500 Economic profit *$2,053,500 - ($7,650,000 × .20) 2.

$7,650,000 $2,053,500 26.84% $ 523,500*

7,650,000)

The essence of the concept of economic profit is that it requires the manager to take all projects that promise a return to the company over and above the cost of the capital invested. This will maximize total return to the company for the capital it has available. To maximize ROI means that the company takes only the highest ROI projects. To use a required rate of 15% means that the company is investing in unprofitable opportunities where the return is lower than the 20% cost of capital. For example, by taking project 4 in (a), the division manager lowered the economic profit since its return on investment of 16% is less than the cost of the capital of 20%.

10-B2 (10 min.) Amounts are in millions. 1. Pretax operating income Less: Cash taxes Operating income after tax Less: Capital charge (14% × $16,125 and 14% × $18,110) EVA

20X4 $6,105 (1,686) 4,419

20X5 $6,100 (1,620) 4,480

(2,258) $2,161

(2,535) $1,945

2. Although profit after tax increased slightly in 20X5, EVA decreased by ($1,945 $2,161) ÷ $2,161 = 10%, primarily due to the increase in average invested capital and corresponding increase in the charge for invested capital. Therefore, the company created less value for its shareholders in 20X5 than in 20X4.

..

415


10-B3 (20 min.) The appropriate transfer price is the market price of $3.35 per gallon. As long as the market price is met, the buying divisions must purchase from the internal divisions. At this price, both divisions have incentives to make choices that maximize operating income for the corporation as a whole. At this price, the analysis by the manager of the ice cream machine shows the overall effect on company profit (and the manager of Okemos Drive-In is indifferent between inside versus outside purchase because the cost to Okemos Drive-In is the same under either option):

Sales* Variable costs @ $2.00 Contribution margin Fixed costs Operating margin

Ice Cream Machine Including Excluding Sales to Sales to Okemos Okemos Drive-In Drive-In $61,400 $48,000 32,000 24,000 $29,400 $24,000 13,600 12,700 $15,800 $11,300

*12,000 gallons @ $4 + 4,000 gallons @ $3.35; 12,000 gallons @ $4 This can also be seen by analyzing overall company operating income, which is higher by $4,500 with sales to Okemos Drive-In: Cost of purchase from outside Less: Incremental cost of inside production Incremental fixed costs Variable costs, 4,000 gallons @ $2.00 Net savings from inside purchases

..

$13,400 (900) (8,000) $4,500

416


10-B4 (30-45 min.) 1.

a. Contribution margin per unit = ¥7,350 - ¥4,900 = ¥2,450 Total contribution = ¥2,450 × 3,350 units = ¥8,207,500 Operating income = ¥8,207,500 - ¥5,700,000 = ¥2,507,500 ROI = ¥2,507,500 ÷ ¥16,000,000 = 15.67% b. Revenue = ¥7,350 × 3,350 units = ¥24,622,500 Capital turnover = ¥24,622,500 ÷ ¥16,000,000 = 1.539 c. Return on sales = ¥2,507,500 ÷ ¥24,622,500 = 10.18%

2.

a. Desired operating income = 24% × ¥16,000,000 = ¥3,840,000 Let X = units to be sold to reach desired return ¥2,450 × X units = ¥5,700,000 + ¥3,840,000 X = ¥9,540,000 ÷ ¥2,450 = 3,894 units b. Let Z = required decrease in total assets Operating income (¥2,507,500 + .1Z) .34Z Z Operating income

÷ ÷ = = =

Total assets

=

ROI = 3.

Examine the operating income and rate of return on assets with and without the 1,200-unit transfer (amounts are thousands of Japanese Yen):

Sales, 2,950 units @ ¥7,350 and 1,200 units @ ¥6,150 Variable costs, ¥4,900/unit Fixed costs Total costs Operating income Total assets needed Rate of return on assets

..

total assets = .24 (¥16,000,000 - Z) = .24 ¥1,332,500 ¥3,919,118 ¥2,507,500 + .1 × (¥3,919,118) = ¥2,899,412 ¥16,000,000 - ¥3,919,118 = ¥12,080,882 ¥2,899,412 ÷ ¥12,080,882 = 24%

Sell 2,950 units

Sell 4,150 units

Difference 1,200 units

21,682.5 14,455.0 5,400.0 19,855.0 1,827.5 11,000 16.6%

29,062.5 20,335.0 5,700.0 26,035.0 3,027.5 16,000 18.9%

7,380 5,880 300 6,180 1,200 5,000 24%

417


Based on the information given, the Sendai division should sell 1,200 units to the European Marketing division at the ¥6,150 price. Both divisions and the company as a whole will benefit from such a decision. Although the original overall target rate of return of 24% is unattainable, the division will nevertheless earn a better rate of return with the intracompany business than without it. The additional units earn a 24% incremental rate of return, which exceeds the 16.6% rate earned on 2,950 units. As a result, the overall rate of return will increase from 16.6% to 18.9%, as shown in the schedule above. In addition, the extra sales meet the return target of 24%. Despite this economic analysis, the Sendai manager may still decide against transferring goods at such a low price. For example, he may feel entitled to a higher profit. This would mean that the company would be worse off in the short run if the European Marketing division must pay the equivalent of ¥6,150 to purchase the games elsewhere. Should top management interfere and force a transfer at ¥6,150? Such intervention would weaken the decentralization structure. Obviously, top management intervention sometimes may be needed to prevent costly mistakes. But recurring interference and constraints simply transform a decentralized organization into a centralized organization. Of course, if managers repeatedly make costly dysfunctional decisions, the costs of decentralization may exceed the benefits. Then a more centralized organizational design may be desirable. Further, the Sendai manager may acknowledge the ¥1,200,000 loss by not transferring but believe that some long-run objective is worth the ¥1,200,000 short-term sacrifice. 10-1 Benefits of decentralization include: 1) lower-level managers may make better decisions because they have better knowledge of local conditions; 2) managers develop their management skills so that there are more managers qualified to move up in the organization; and 3) managers have higher status and therefore are more highly motivated. Costs of decentralization include: 1) managers may make decisions that are not in the best interest of the organization because they are not aware of or not interested in facts that don't pertain to their own segment; 2) managers may perform functions at the division level that would be less costly if centralized; and 3) the cost of information to coordinate and control activities may increase with decentralization. 10-2 One of the limitations in decentralization is lack of knowledge in segments of the organization. This is especially true in geographically decentralized operations. Accounting systems give a common language and structure for sharing information throughout an organization. Sophisticated communications systems make this information available without delay. Many companies have “data warehouses” that let managers anywhere in the organization have immediate access to whatever accounting information they want.

..

418


10-3 It is more difficult to hold managers of nonprofit organizations responsible for performance because inputs and outputs are generally more difficult to measure. Without reliable performance measures, granting managerial freedom is more risky. 10-4 No. Profit centers facilitate decentralization, but one can exist without the other. They are different concepts, as the chapter explains. 10-5 The main aim of decentralization is to improve the effectiveness of the decisionmaking process. However, the same can work best only if the mangers of various segments are allowed to make their own decision. In other words, segment autonomy is the key to successful decentralization. 10-6 An ideal performance metric should be one that rewards a manger for all the factors that they can control but neither reward nor punishes them for factors that they cannot control. 10-7 The major advantage of the rate of return analysis of performance is its attention to the required asset investment in relation to operating income. A percentage return is also a common measure that is familiar to managers. 10-8 ROI is affected by a division’s income and the amount of its investment. You can also think of ROI as the product of return on sales and capital turnover. 10-9 The major difference is that economic profit includes a capital charge, that is, a cost of using all capital. In contrast, net income includes a charge for using debt capital (interest) but no charge for using equity capital. Some measures of economic profit rely primarily on financial reporting numbers and others, such as EVA, include adjustments such as deducting cash taxes rather than tax expense or capitalizing rather than expensing R&D. 10-10 Economic value added (EVA) is after-tax operating income minus the after-tax weighted-average cost of capital multiplied by the sum of average long-term liabilities and stockholders’ equity. Companies can improve EVA by     

Investing in products or projects that generate more after-tax operating income than the cost of the capital used, Divesting products or projects that do not generate enough after-tax operating income to cover the cost of the capital used, Reducing the weighted-average cost of capital, and Increasing the after-tax operating income without using more invested capital Decreasing invested capital without decreasing after-tax operating income.

An alternative definition of the capital charge portion of the EVA equation is the product of the after-tax weighted-average cost of capital multiplied by the sum of average working capital and non-current assets. The equivalency of these two definitions can be shown as follows: ..

419


Total assets = Total liabilities + Stockholders' equity Current assets + Non-current assets = Current liabilities + Long-term debt + Stockholders' equity Current assets - Current liabilities + Non-current assets = Long-term debt + Stockholder's equity The right-hand side of the above equation represents the capital structure of an organization - that is, how capital is financed. The left-hand side represents the actual capital -- working capital and non-current assets. Either definition can be used. 10-11 Division A’s manager would reject the proposed project because it would reduce the division’s ROI. Division B’s manager would accept the proposed project because it would increase the division’s ROI. If evaluation is based on economic profit, both managers would be motivated to accept the proposed project because the project’s return is greater than the imputed cost of invested capital. 10-12 Three possible definitions of invested capital are: 1. 2. 3.

Total assets Total assets less current liabilities Stockholders' equity

10-13 There is some truth to this statement. However, using a historical cost accounting system with budgets is not backward looking. Budgets force managers to plan for the future, including predicting future prices. 10-14 The use of gross book value rather than net book value of assets to compute ROI may affect the speed with which managers replace assets. Gross book value leads to more rapid replacement. A manager who suggests using gross book value probably has assets that are relatively new compared to those in other divisions. When net book value is used, the manager's relatively new assets are valued considerably higher than older assets because they have little accumulated depreciation. When gross book value is used, accumulated depreciation is irrelevant and there is less difference between the values of older assets and newer assets. 10-15 Companies need transfer-pricing systems to accurately determine the efficiency of various divisions of a company's operation when using profit or investment centers. If inefficiency exists somewhere in a corporation that does not employ a transfer - pricing system, it is much harder to pin down the area or process that is most directly responsible. In addition, transfer pricing is an attention-directing device that highlights good performance and motivates personnel to maintain efficiency. Transfer pricing systems also preserve segment autonomy.

..

420


10-16 Using full costs can mask the real behavior of a cost. Any transfer price that includes a fixed cost element makes a fixed cost in the producing department look like a variable cost to the buying department. Using actual costs can pass on inefficiencies and make planning difficult for the buying department. The amount of the transfer is not known until actual costs are available, and factors beyond the control of the buying department can affect the transfer price. 10-17 If a producing division has idle capacity, a transfer price at the variable cost will usually be optimal. Why? Because it costs the firm as a whole only the variable costs to produce the item to be transferred. However, if there is not idle capacity, the selling (producing) division, and hence the firm as a whole, gives up the contribution it would obtain from selling to outside customers or using the resource for some other use. A higher transfer price that reflects the opportunity cost would be optimal in order to assure that the value to the buying division is at least as great as the sacrifice made by the selling division to transfer the item internally. 10-18 A market price-based transfer price is ideal when there is a ready market for an item or service that is transferred from one segment to another segment. This is because the market price is equal to the variable cost plus opportunity cost. 10-19 Negotiated transfer prices are likely to lead to better transfer pricing decisions because those with the best knowledge are making the decisions. On the other hand, valuable time and effort can be lost in the negotiating process. Further, with negotiated transfer prices, divisional profit becomes a function of negotiating ability and power. 10-20 Multinational transfer prices are influenced by the relative income-tax rates in the countries in which the producing and purchasing divisions are located. They are also influenced by import duties and restrictions on flows of capital between countries. 10-21 In organizations using management by objectives (MBO), managers and their superiors jointly formulate and agree on the goals and plans for the forthcoming period. Managers are then evaluated against these agreed-upon goals and plans. 10-22 When making performance targets becomes too important, managers may be tempted to manipulate the numbers to meet their targets. Such pressures can result in unethical behavior. 10-23 A decision to decentralize is based on a desire to achieve specific objectives in a particular environment. If a company’s objectives change or its environment change, a different level of decentralization may be appropriate. Further, companies try to achieve a balance between centralization and decentralization, but sometimes a movement in one direction or another over-shoots its target and a retrenchment is needed. 10-24 No. ROI and economic profit create different motivations for managers. Goal congruence and managerial effort would usually be accomplished better by economic profit. ..

421


10-25 Profit measures and other performance measures by themselves do not cause unethical behavior. However, the way managers use them may. Managers charged with meeting performance targets by whatever means possible might resort to unethical means. Thus, it is important that management control systems not only include the right performance measures, but they must also specify how management should use these measures in evaluating managers. 10-26 When top management second-guesses divisional managers frequently, many advantages of decentralization are lost. Segment autonomy disappears. Essentially, top management intervention implies that top managers know more about the local market conditions than do segment managers. If this is indeed the case, a decentralized operation may not be appropriate. 10-27 (10 min.) Pound amounts are in thousands. 1.

Turnover of capital

= Sales ÷ Invested capital = £455,000 ÷ £130,000 = 350%, or 3.5 times

2.

Return on sales

= £18,200 ÷ £455,000 = 4.0%

3.

Return on Investment (ROI) or

= £18,200 ÷ £130,000 = 14% = 3.5 × 4.0% = 14%

10-28

(5 min.)

The basic equation: ROI = Return on Sales × Capital turnover P: 5% × 4 = 20% Q: 16% ÷ 8% = 2.0 R: 18% ÷ 3 = 6%

..

422


10-29

(15-20 min.)

1.

The filled-in blanks are underscored:

Invested capital Income Revenue (sales) Return on sales, Income ÷ Sales Capital turnover, Sales ÷ Invested capital Return on invested capital, Income ÷ Invested capital

X $1,050,000 $ 69,300 $2,310,000

Division Y $1,360,000 $ 142,800 $2,856,000

Z $1,200,000 $ 210,000 $4,200,000

3%

5%

5%

2.2

2.1

3.5

6.6%

10.5%

17.5%

X $ 69,300 105,000 ($ 35,700)

Division Y $142,800 136,000 $ 6,800

Z $210,000 120,000 $ 90,000

2. Income Capital charge Economic profit 3.

If the criterion for judging relative performance is ROI, Division Z is best for this period. If the criterion is economic profit, then both Division Y and Division Z are producing positive economic profit, with Division Z producing more. Other factors deserving attention include the relative risks faced by each division and the short-run versus long-run implications of current performance.

10-30

(10 – 15 min.) Euro amounts are in thousands.

1. 2019:

2020:

2.

..

Adjusted net operating profit after tax (€119,000 – €30,750) Capital charge (.096 × €975,000) EVA

€ 88,250 93,600 € (5,350)

Adjusted net operating profit after tax (€121,000 – €33,900) Capital charge (.096 × €900,000) EVA

€87,100 86,400 € 700

Hexa’s overall performance improved, with EVA increasing from a negative €5,350 to a positive €700. Although the adjusted after-tax operating profit decreased by €1,150, this was more than offset by the decrease of €7,200 in the capital charge, resulting in the net EVA increase of €6,050. 423


10-31

(20 min.) Assets Liabilities Stockholders’ equity Income before interest 14% interest Net income Rate of return on: Assets Stockholders’ equity *$1,032,800 ÷ $3,520,000 = 29.34%

..

Xenon $6,000,000 2,480,000 3,520,000 1,380,000 347,200 $ 1,032,800 17.2% 29.34%*

Neon $6,000,000 0 6,000,000 1,380,000 0 $ 1,380,000 23% 23%

424


10-32 (20-30 min.) This problem presses the student more than those immediately preceding it. Dollar amounts are in thousands. 1. Income: .27 × $5,000; .16 × $20,000 Revenue: $450 ÷ .05; $1,350 ÷ .09; 2 × $20,000 Invested capital: $9,000 ÷ 6 Return on sales, 16% ÷ 2 Capital turnover, 27% ÷ 9% Rate of return on invested capital, $450 ÷ $1,500 Cost of capital Economic profit, $450 - (.07 × $1,500); $1,350 - (.14 × $5,000)

J

Division K

L

$ 450

$ 1,350

$ 3,200

$9,000 $1,500 5% 6

$15,000 $ 5,000 9% 3

$40,000 $20,000 8% 2

30% 7%

27% 14%

$ 345

$

650

16% 15%* $

200

*$3,200- $200= $3,000; $3,000 ÷ $20,000 = 15% 2.

This requirement can generate much discussion or little discussion, as the instructor desires. Using ROI as the criterion, J is the best performer. Using economic profit as the criterion, K is the best performer. Note that this company uses different interest rates for different divisions, probably because of wide variations in risks. Note, too, that economic profit, an absolute amount, is usually easier to generate by large divisions.

..

425


10-33

(15 min.)

1 & 2. (2) (1) Income Net Rate Gross Rate Before Operating Book of Book of Year Depreciation Depreciation Income Value* Return Value Return 1 150,000 60,000 90,000 150,000 60% 180,000 50% 2 150,000 60,000 90,000 90,000 100% 180,000 50% 3 150,000 60,000 90,000 30,000 300% 180,000 50% * ($180,000 + $120,000) ÷ 2; ($120,000 + $60,000) ÷ 2; ($60,000 + $0) ÷ 2 3.

Charlotte might prefer the net book value because it always gives a higher ROI. However, she might prefer to be evaluated based on the gross book value for two interrelated reasons. First, if she is being compared to other divisions, she knows that they will also report higher ROIs using net book value. Second, the benefit to her division of using net book value will occur mainly in the second and third years of the investment, after she transfers out.

10-34 1.

(10 min.) (b)

Selling Price Variable costs: Cutting Division Finishing Division Total variable costs Contribution to net income 2.

Process Further Finishing Division Overall PerforPerformance mance £41.50 £41.50

(a) Sell to Outsiders at Transfer Point Finishing Division Overall PerforPerformance mance £---£32.50

£26.00 13.00 39.00

39.00

----

£26.00 ---26.00

£ 2.50

£ 2.50

£----

£6.50

If the transfer price is based on variable cost of £26, the manager of the Finishing Division would want the product processed further. But this would hurt overall company performance. The incremental revenue from finishing the chair (£41.50 –£32.50 = £9) is less than the incremental cost of £13 incurred to finish the chair. Thus, finishing a chair decreases firm profit by £13 – £9 = £4. If the transfer price is instead based on outlay cost plus opportunity cost (£26 + £6.50 = £32.5), the manager of the finishing division will not want to process the product further, because the cost of the product (£32.5 + £13 = £45.5) will exceed the selling price of £41.50.

..

426


10-35

(10-15 min.)

1.

If wheels were available for $14 each in the market, Dayton would not be willing to pay more than $14 to Toledo. If wheels could not be purchased in the market, the maximum price would be $20, computed as follows: Sales price Variable costs (except for wheels) Contribution available for wheels and to cover fixed costs and profit

$170 130 $ 40

The $40 must pay for two wheels, so the most that can be paid per wheel is $40÷2 = $20. The manager of the Dayton Division would likely not pay the entire amount of $20 per wheel, because that would leave no contribution to fixed costs plus profit. However, any price less that $20 would produce a positive contribution margin and therefore be better than not producing the bicycles. 2.

Because there is excess capacity, any transfer price above the variable cost of $10 would result in a positive contribution margin. No price below $10 would be acceptable. If there were no excess capacity, the minimum transfer price would be the market price of $14. Why? Because the Toledo division would have to forgo $14 of revenue from an external sale in order to transfer a wheel internally.

10-36 (10 min.) 1.

Princeton is better off using a £800 transfer price. A £800 transfer price places an extra £800 - £500 = £300 of income in Ireland instead of Japan, while a £500 transfer price places an extra £300 of income in Japan instead of Ireland. In addition, the £800 transfer price adds to the import taxes paid to Japan. The net effect is as follows: Irish income is £300 higher, .24 × £300 more taxes Japanese income is £300 lower, .45 × £300 less taxes Japanese import tax, .13 × £300 more taxes Net reduction in taxes from £800 transfer price

2.

..

£ (72) 135 (39) £ 24

The total taxes saved by the £800 transfer price is £24 per unit.

427


10-37

(20-25 min.)

1.

The two contracts illustrate the tradeoff between incentive and risk in employment contracts. The bonus contract provides more incentive to generate profits than does the straight salary. This should benefit Tamura International. On the other hand, it may cause the manager to focus too much on short-run profitability. Further, it imposes risk on the manager. A manager demands extra compensation to bear this risk. If the vice-president/personnel is correct, the expected cost to Tamura International of this risk is ¥90,000 (the amount by which the expected compensation with the bonus plan exceeds the straight salary). The choice should be based on whether the extra incentive under the bonus plan is likely to be worth at least ¥90,000 to Tamura International. Another factor to consider is what type of manager will be attracted by each type of contract. Sometimes it is hard to determine a manager's qualifications at the time of hiring. The manager knows his or her abilities better than does the company. Highly qualified managers would seek contracts with a bonus. Why? They would be confident that they would do better than average and therefore receive compensation above what the firm expects an average manager to receive. The opposite is true for less qualified managers. Because Tamura International receives ¥9 of every ¥10 of extra profit generated, attracting a highly qualified manager is likely to be advantageous.

2.

Managers are generally risk averse. This means that they prefer a contract with less risk to one with more risk if the expected compensation does not differ. It does not mean that managers avoid risks, only that they want to be compensated for such risk-seeking. If a risky contract has an expected compensation high enough, it will be preferred to a given risk-free contract. For Tamura International, a quality manager willing to accept a straight-salary contract at ¥400,000 might not accept a risky bonus contract with an expected compensation of ¥400,000. Extra compensation must be paid to offset the added risk. The Tamura International Trading Company should recognize that besides normal operating risk, they have imposed an added risk in the bonus contract. Because the contract is in Japanese currency, possible movement in exchange rates adds to the noncontrollable factors affecting the bonus, hence it increases the risk if the manager is in Mexico City and wants to be paid in pesos.

..

428


10-38

(25 min.)

1.

The two separate components highlight certain features of profitability that are not revealed by the single calculation. a. The importance of capital turnover as a key to profits is stressed. b. The importance of sales volume is explicitly recognized. c. It reduces important elements to ratios instead of monetary figures. This often enhances comparability of different divisions, businesses, and time periods. d. The breakdown stresses the possibility of trading off capital turnover for return on sales so as to increase the average rate of return at a given level of output. e. It gives clues to the companies' strategies, such as their trade-off between profit margins and sales volume.

2. Return on sales Turnover on capital Return on investment

Prime 9% × 2.7 23.9%

Company Quality 12% × .125 1.5%

Rare .6% × 2.4 1.5%

Income and investment alone shed little light on comparative performance because of disparities in size between Prime and the other two companies. The fact that Quality and Rare have identical income and capital suggests that the same conditions underlie the low rate of return, but this conclusion is erroneous. Introducing sales to measure level of operations helps to disclose specific areas for more intensive investigation. Quality does better than Prime in terms of profit margin, 12% versus 9% return on sales. But Quality has a much lower turnover of capital than does Prime. Whereas a euro of investment in Prime supports 2.7 euros in sales each period, a euro investment in Quality supports only 12.5 cents in sales each period. This suggests that the analyst should look carefully at Quality’s investment. Is the company keeping an inventory larger than necessary for its sales volume? Are receivables being collected promptly? Or did Prime acquire its fixed assets at a price level that was much lower than that at which Quality purchased its plant? On the other hand, Rare’s capital turnover is closer to Prime’s, but Rare’s return on sales is much lower. Why? Are its operations inefficient, are its material costs too high, or does its location entail high transportation costs? Analysis of return on capital raises questions such as the foregoing. When answers are obtained, basic reasons for differences between rates of return may be discovered. For example, in Quality’s case, it is apparent that the emphasis will have to be on increasing capital turnover by reducing investment or increasing sales. Most likely, Quality cannot appreciably increase its rate of return on ..

429


investment simply by increasing its return on sales. In contrast, Rare’s management should concentrate on increasing the return on sales. 3.

Quality has a high return on sales and a low capital turnover strategy and probably focuses on high-end merchandise. In contrast, Rare has a low return on sales and a high capital turnover. It might attempt to create high volume by offering low prices.

10-39

(15-20 min.)

1.

(a) Beauty Product Beverages Oral Care

₹630 ÷ 420 ÷ 795 ÷

₹3,150 = 2,100 = 3,180 =

20% 20% 25%

(b) Beauty Product Beverages Oral Care

₹3,150 ÷ 2,100 ÷ 3,180 ÷

₹3,000 = 4,200 = 2,544 =

1.05 .50 1.25

(c) Beauty Product Beverages Oral Care

₹630 ÷ 420 ÷ 795 ÷

₹3,000 = 21% 4,200 = 10% 2,544 = 31.25%

2.

This requirement can lead to a lengthy discussion of what causes differences in the three measures computed in requirement 1. The obvious difference is the low return on investment in the Beverages. It is worth noting that this difference arises primarily because of the low capital turnover in the segment. The Beverages segment is generating less than half as much sales revenue per rupee of invested capital as either of the other two segments.

..

430


10-40

(25 – 30 min.) Amounts in thousands.

Income from operations Net Adjustments to income for EVA Provision for taxes Cash taxes Net Operating Profit After-tax (NOPAT) Average total assets, (3,668,912 + 3,778,642) ÷ 2 Average current liabilities, (680,250 + 943,718) ÷ 2 Average capital for economic profit Added capital from adjustments for EVA Average capital for EVA Cost-of-capital percentage Capital charge Economic profit or EVA * Rounded to whole dollars.

Economic Profit $ 534,800

Economic Value Added $ 534,800 10,796

(114,910) (129,600) 419,890 415,996 3,723,777 3,723,777 811,984 811,984 2,911,793 2,911,793 468,318 33,80,111 ×11.3% ×11.3% 329,033* 381,953* $ 90,857 $ 34,043

10-41

(10-15 min.) Amounts are in millions.

1.

Year 2 EVA = $4,463 - $3,569 - $292 - (11.5% × $2,854) = $274 Year 1 EVA = $4,510 - $3,615 - $255 - (11.5% × $2,689) = $331

2.

EVA decreased from $331 million to $274 million. The decrease was caused by a decline in operating income and, despite the decline in pre-tax income, an increase in income taxes. However, both years showed positive EVA and therefore positive value creation.

10-42

(10-15 min.)

1.

Weighted-average cost of capital: 55% × 10% = 5.50% 45% × 12% = 5.40% 10.90% EVA = $8,210,000,000 - $1,395,000,000 – (10.9% × $27,555,000,000) = $3,811,505,000

2.

Holt’s EVA of $3.8 billion means that Holt generated $3.8 billion of value for its shareholders above the cost of capital, the weighted average of the normal return expected by Holt's investors. Note that this is an accounting measure of change in value, and it is not necessarily equivalent to the change in market value of Holt’s stock.

..

431


10-43

(20-30 min.) Dollar amounts are in thousands. TV

AC

Oven

Historical Cost: Average net assets Operating income Capital charge Economic Profit Rate of return on net assets

€45,000 7,800 4,500 3,300 17.3%

€132,000 20,250 13,200 7,050 15.3%

€81,000 15,000 8,100 6,900 18.5%

Replacement Cost: Average net assets Operating income Capital charge Economic Profit Rate of return on net assets

€48,000 7,500 4,800 2,700 15.6%

€165,000 18,450 16,500 1,950 11.2%

€144,000 11,700 14,400 (2,700) 8.1%

Neither base is full proof regarding the evaluation of an individual manager’s performance. First, the short-run emphasis of such measures includes only a part of the activities that promote profitability in the long run. Second, the environmental conditions facing a particular division plus unfavorable carryover of past mistakes may severely hamper divisional performance even though the manager is clearly superior by any test other than the rate of return or economic profit. That is, the “best” managers are often deliberately given the sickest divisions precisely because they have the most ability to improve a bad situation. Improvement or fulfilling carefully budgeted targets may be the best tests of management performance as distinguished from divisional performance. 2.

The following rankings exist: Rate of Return

First Second Third

On Historical Cost Oven TV AC

On Replacement Cost TV AC Oven

Economic Profit On Historical Cost AC Oven TV

On Replacement Cost TV AC Oven

3.

In this case, if historical cost is the base, the use of rate of return on net assets ranks Oven first, whereas economic profit ranks AC first. Used indiscriminately, each method has its drawbacks, regardless of whether historical cost or replacement cost is used as a base. Rate of return inhibits divisions with high rates from expansion, whereas economic profit tends to favor large divisions as long as they earn in excess of the cost of capital.

..

432


Replacement costs are more helpful than historical costs as indicators of the relative profitability of divisions because they are usually good approximations of the current economic sacrifice being made to conduct such operations. As for managers, their ability to meet budgeted goals, however measured, is paramount. Students, professors, and managers have disagreements regarding which asset base is preferable. 10-44

(50-60 min.)

1.

See Exhibit 10-44 on the following page.

2.

Some major companies, including du Pont and Monsanto Chemical, have used gross assets as an investment base. One reason often cited for using undepreciated cost is that it partially compensates for the impact of the changing price level on historical cost. However, if a company desires to use replacement cost as a base, it should not try to tailor historical costs to the measurement problems of changing prices; the results of such hybrid attachments can be unreliable. The reasoning in support of the gross assets base must be aligned with the purpose for its use: appraisal of company results as a whole (column 11 of the answer to requirement 1) or appraisal of a plant's or division's performance (column 6). A company's performance as a whole is the responsibility of top management. When profits are made, depreciation is recouped out of sales revenue. If dividends are paid in the amount of net income, cash may accumulate in the amount of the annual $65,000 depreciation (column 8). (No cash is kept in the business from earnings, but there is a conversion of fixed assets into cash as measured by depreciation.) To count original cost plus the cash accumulation as a part of the investment base (column 10) is duplication; it does not provide as useful a base as net assets. In contrast, a plant manager’s or division manager’s performance often is best analyzed by using gross assets as the investment base (column 6). The reinvestment of the cash accumulation in the amount of depreciation charges may be beyond the manager’s control. Those who favor gross asset value as a base state that it facilitates comparisons among plants or divisions. If income moves downward as a plant ages, the decrease in earning power will be evident under a gross asset base, while the constantly decreasing net asset base will reflect a possibly misleading higher rate of return in later years (column 7).

..

433


EXHIBIT 10-44 Rate of Return on Assets Using Original Cost of Fixed Assets vs. Using Net Book Value of Fixed Assets

Gross Year Cost (1) 1 $260,000 2 260,000 3 260,000 4 260,000 * ** ***

..

Plant Performance Company Performance Fixed Assets Rate of Return Accumu- Net Average On Total Gross Assets Net Assets lated Value, Book Annual On Average Cash Average*** DepreciEnd Value Net Gross Book Accumu- Cash Rate of Rate of ation of Year for Year* Income Cost Value lation** for Year Base Return Base Return (2) (3) (4) (5) (6) (7) (8) (9) (10) (11) (12) (13) $65,000 $195,000 $227,500 $10,000 3.8% 4.4% $65,000 $ 32,500 $ 292,500 3.4% $260,000 3.8% 130,000 130,000 162,500 10,000 3.8% 6.2% 130,000 97,500 357,500 2.8% 260,000 3.8% 195,000 65,000 97,500 10,000 3.8% 10.3% 195,000 162,500 422,500 2.4% 260,000 3.8% 260,000 0 32,500 10,000 3.8% 30.8% 260,000 227,500 487,500 2.1% 260,000 3.8%

½ × (Beginning balance plus Ending balance), e.g., ½ × ($195,000 + $130,000) = $162,500 for year 2. Assume that sales and expenses except depreciation are on a cash basis, and that dividends equal net income. Thus, cash in the amount of the depreciation charge will accumulate each year. This situation is unrealistic in the sense that idle cash is being accumulated without being reinvested to earn a return.

434


The proponents of using net book value as a base maintain that it is less confusing because (a) it is consistent with the total assets shown on the conventional balance sheet and (b) it is consistent with net income computations, which include deductions for depreciation. Using net book value prevents duplication of the same asset in the base and shows a constantly rising rate of return on plant performance. See column 7. Note that the inclusion of the cash accumulation and gross fixed assets duplicates the same item, so that the total fixed and current gross asset base rises from year to year. The definition of income should be consistent with the definition of the capital base to which it is related. Thus, interest expense is ordinarily excluded in computing incomes that are related to asset bases, while interest expense is deducted in computing income that is related to stockholders' equity bases. Nonrecurring items are ordinarily excluded when current operating performance is to be appraised. 10-45

(30-40 min.) The issues in this problem are covered briefly in a section in the chapter. This problem was originally used on a final examination. In particular, note that the quotation is dealing with how to evaluate performance, as distinguished from decisions to buy, hold, or sell assets. A basic question, then, is why we bother to evaluate performance. Fundamentally, it is to assist future decisions and to provide managers with incentives toward organizational goals. One set of numbers may be appropriate for evaluating the economic performance of a segment, whereas a different set may be appropriate for appraising an individual manager's performance. The last sentence in the problem clearly recognizes this distinction, but students tend to pay insufficient attention to it in their solutions. Of course, the major reason for the distinction is that events uncontrollable by the manager sometimes dominate the economic performance of an entity; simultaneously, the manager may be doing either a superhuman or an abysmal job with respect to the critical factors under his or her control. The issues presented in the statement assume the following logical pattern: 1. 2. 3. 4. 5.

Economic values are the best for performance measurement. Replacement values will probably be less than economic value throughout an asset's life. Market (exit) value is inherently less than or equal to (usually the former) economic value for a given asset. Use of economic value is infeasible; hence, replacement value should be used. Replacement value will facilitate the evaluation of the division's performance more easily than the division manager's performance.

The statement correctly establishes economic value as the "ideal" measure of an asset's value. The statement fails to disclose the characteristics of economic value that make it "infeasible." Infeasibility probably refers to the difficulty of determining (a) cash flows

..

435


in the future and (b) the appropriate discount rate to be applied to those flows in the present value process. The statement presents a reasonable case in favor of replacement value over exit value. Some remarks might be made about the fact that replacement costs of highly specialized assets may be more difficult to obtain than a direct approximation of their economic values via discounted cash flow techniques. The biggest defect of the commentary is its failure to mention the cost and value of information tradeoffs in deciding whether some "current" value basis for evaluation of performance is superior to continuing to use historical cost. 10-46

(20-30 min.) The rule to be used is that goods and services should be transferred at a price equivalent to that prevailing in an outside market at the time of transfer. Where the internal division meets these selling prices, the buying division must purchase internally. Market prices establish the ceiling for transfer-pricing. In many instances, a lower price may easily be justified, particularly where high-volume purchases are made or where selling costs are less. In the two cases cited, the transfer prices should be no higher than those that could be obtained consistently by buying the used computers, parts, or services from outside parties. The whole idea of decentralization is the manager's independence; unless a manager can resort to buying and selling outside the company, his or her profit center is essentially in a centralized company. Nevertheless, profit centers may promote more goal congruence than cost centers.

..

436


10-47

(30 min.)

1.

Both the shocks and struts division and the company as a whole will benefit if the $42.70 price is met. If the shocks and struts division does not sell to the automotive division, 70% of the strut assembly volume will disappear, and gross margin will fall to $2,550,000 as follows: Sales, 300,000 at $61 Variable costs, at $38.50 Fixed costs Total costs Gross margin

$18,300,000 $11,550,000 4,200,000 15,750,000 $ 2,550,000

If the $42.70 price is met, the shocks and struts division will show a gross margin of $5,490,000 as follows: To Automotive Division Sales: 700,000 at $42.70 300,000 at $61 Variable costs Contribution margin Fixed costs Gross margin

To Outsiders

Total

$29,890,000 26,950,000 $ 2,940,000

$18,300,000 11,550,000 $ 6,750,000

$48,190,000 38,500,000 $ 9,690,000 4,200,000 $ 5,490,000

The rejection of intracompany business will reduce margins by $5,490,000 $2,550,000 = $2,940,000. An alternative way to arrive at the same conclusion is to note that the acceptance of intracompany business will add a contribution margin of $4.20 per strut assembly ($42.70 less $38.50 variable costs) or $4.20 × 700,000 = $2,940,000 that will be forgone if intracompany business is rejected. 2.

Yes, the division should reject intracompany sales and concentrate on outside sales since the gross margin would be $8,300,000, whereas the gross margin if automotive division business were accepted would be $5,490,000. The gross margin would increase by $2,810,000 as follows: Sales, 1,000,000 at $58 Variable costs, at $43 Fixed costs Total costs Gross margin (new proposal) Gross margin (accepting intracompany business) Difference

..

$58,000,000 $43,000,000 6,700,000 49,700,000 $ 8,300,000 5,490,000 $ 2,810,000 437


10-48

(30 min.)

1.

The U.S. Division should not supply the Australian Division with the sound system for the $7.00 per unit price. The U.S. Division is operating at capacity and would lose $4.00 ($11.00 - $7.00) for each part sold to the Australian Division. The management performance of the U.S. Division is measured by return on investment and dollar profits; selling to the Australian Division at $7.00 per unit would adversely affect those performance measures.

2.

Samtech Electronics would be $5.40 better off in the short run if the U.S. Division supplied the Australian Division the part for $7.00 and the Game Box was sold for $62.00. Assuming that the $10.00 per unit for fixed overhead and administration is an allocation of costs the Australian Division incurs regardless of the Game Box order, Samtech would lose $4.00 in cash flow for each sound system sold to the Australian Division but gain at least $9.40 from each Game Box the Australian Division sells, a net gain of $9.40 - $4.00 = $5.40.

3.

In the short run there is an advantage to Samtech Electronics of transferring the sound system at the $7.00 price and thus selling the Game Box for at least $62.00. To make this happen, Samtech Electronics could overrule the decision of the U.S. Division management. This action would be counter to the purposes of decentralized decision-making. If such action were necessary on a regular basis the decentralized decision-making inherent in the divisionalized organization would be a sham. Alternatively, the problem could be placed back with the Australian Division. Even if the Australian Division had to pay the market price of $11.00, the contract would increase its profit. The variable cost would be $28.10 + $11.00 + $17.50 = $56.60. As long as the Australian Division has excess capacity, its profit would increase by at least $5.40 (i.e., $62.00 - $56.60) per Game Box. Therefore, both the Australian division and Samtech Electronics as a whole would be better off if the transfer were made at market price. It is up to the Australian Division managers to find a way to price the Game Box at $56.60 or more even when the sound system is transferred at $11.00.

..

438


10-49 (20-25 min.) 1.

The Dhaka Division manager would not buy the cloth for BDT14,000 and would not produce the dresses. The division would lose BDT800 on each dress produced at that price: Revenue per dress Division cost per dress: Cloth Manufacturing Selling Division loss per dress

BDT19,000 BDT14,000 4,600 1,200

19,800 BDT (800)

However, the company as a whole would benefit by BDT3,600 per dress if the dresses were produced and sold: Revenue per dress Additional costs per dress: Cloth Manufacturing Selling Total contribution per dress

BDT19,000 BDT9,600 4,600 1,200

15,400 BDT3,600

Therefore, the policy to transfer at fully allocated costs motivates the manager to make a decision not in the best interests of the company as a whole. 2.

When there is no idle capacity at the Chittagong Mill, transferring cloth to the Dhaka Division causes the Mill to pass up sales to outside customers. Compare the total contribution from selling the cloth to the total contribution from using the cloth to make dresses and selling the dresses: Sell Cloth to Outside Customers Revenue Total additional costs: Variable cost of cloth Total contribution margin Make and Sell Dresses Revenue Total additional costs: Variable cost of cloth Manufacturing Selling Total contribution margin

..

BDT14,400 9,600 BDT4,800 BDT19,000 BDT9,600 4,600 1,200

15,400 BDT3,600

439


The company is BDT1,200 better off with the contribution of BDT4,800 from selling cloth rather than BDT3,600 from selling the dress. Another way to view this problem is that, if the cloth can be sold for BDT14,400, using it to make a dress adds BDT19,000 – BDT14,400 = BDT4,600 of additional revenue. The additional costs are BDT4,600 + BDT1,200 = BDT5,800. The company is BDT1,200 worse off if it spends BDT5,800 to gain BDT4,600 in revenue. 10-50

(25-30 min.)

1.

Cost to San Jose division of using ArnoPrint: ($.20 × 180 pages) + (250 copies per page × $.02 × 180 pages) = $36.00 + $900.00 = $936.00 Thus, Jiffy Press, at a bid price of $918.25, is the least expensive. In addition, the reports would be ready sooner. If the San Jose office is not directed by top management to do otherwise, it would choose Jiffy Press. If ArnoPrint would use otherwise idle capacity, Arno Legal Services would have variable cost of .50 × $936 = $468, which is less than the amount the San Jose division would pay to Jiffy Press. Thus, giving the business to Jiffy Press is not an optimal economic decision from the entire corporation's point of view.

2.

If ArnoPrint has idle capacity, the minimum transfer price is its variable costs, .50 × $936 = $468. If ArnoPrint can get other orders outside at $936, the minimum transfer price should be $936. The best outside bid, $918.25, generally provides an appropriate transfer price.

3.

The optimal decision might be to go with Jiffy Press since one to two days may be saved in getting the reports to the client. Potential future earnings for consulting services could be greater than the contribution forgone. However, it is uncertain whether the delay would affect the client’s decision to utilize Arno Legal Services in the future. The client's goodwill towards Arno Legal Services is also determined by factors such as the competence of the individuals in Arno Legal, the quality of the report, the price of the report, and the time required to prepare the report for printing.

4.

Top management has decreased the sense of autonomy of its San Jose office in suggesting that ArnoPrint be utilized. This could affect morale and cause dysfunctional behavior, particularly since ArnoPrint’s quality is poor.

..

440


10-51

(10 - 15 min.) The minimum transfer price is $25. Any price below $25 would cause the glass and ceramic products division to lose profit. In fact, the minimum transfer price could be slightly above $25 if the glass and ceramic products division, despite its current situation with excess capacity, would limit its future flexibility by agreeing to the production and transfer. The maximum price is $36, the price at which the consumer durable division could buy the turntables on the market. It might be slightly less than $36 if the consumer durable division can save some transportation or handling costs by buying internally, or if it can be more confident of the quality when purchasing internally. 10-52

(15 min.) This simple example provides a good opportunity to discuss the issue of moving profits from one division to another through transfer prices. The setting is different from any presented in the chapter. Michelin certainly has an incentive to transfer tires at as low price as possible. A €1 larger transfer price shifts €1 of profit from Michelin’s parent-company account to its subsidiary’s account. Michelin retains 100% of its parent-company profits, but it gets only 70% of the subsidiary’s profits. Thus, for every €1 addition to the transfer price, Michelin Group loses €1 and gains only €.70, a net loss of €.30. Of course, the minority shareholders of Stomil Olsztyn want as much profit as possible transferred to their company. Thus, they favor a high transfer price. Each extra €1 of transfer price gains them €.30 in profit. Suppose Stomil Olsztyn could sell the tires on the market and receive a contribution of €9 rather than the contribution of €5 they get from Michelin. Then, the minority shareholders would gain 30% × €4 = €1.20 per tire. A key to a fair transfer price is Stomil Olsztyn’s alternative opportunities. If the subsidiary could sell the same tire on the market for a net price (market price less discounts less costs incurred to sell on the market that the subsidiary does not incur on sales to Michelin) of more than €25, the transfer price is too low. Or, if it uses resources that could make alternative products that would have a contribution margin greater than €5, the price is too low. In such cases, Michelin is gaining at the expense of the minority shareholders of Stomil Olsztyn. Arms-length negotiation between managers of Michelin and Stomil Olsztyn may lead to optimal transfer prices, provided that both seek to maximize their own unit’s profits.

..

441


10-53

(15 min.)

1.

The optimal transfer price is $1,400 per unit:

2.

(a)

Tax savings with $1,400 transfer price: [.4 × ($1,400 - $900)] – [.25 × ($1,400 - $900)] = $200 - $125 = $75

(b)

Additional duty with $1,400 transfer price: .03 × ($1,400 - $900) = $15

(c)

Advantage of $1,400 transfer price over $900 transfer price: $75 - $15 = $60

With these changes, the $900 transfer price is optimal: (a)

Tax savings with $1,400 transfer price: [.30 ($1,400 - $900)] – [.25 ($1,400 - $900)] = $150 - $125 = $25

(b)

Additional duty with $1,400 transfer price: .10 ($1,400 - $900) = $50

(c)

A $1,400 transfer price generates $50 extra duty and saves only $25 in taxes. Therefore, the $900 transfer price is best.

Multinational transfer pricing is heavily affected by the constraints of various countries’ laws on taxes and tariffs. Moreover, the resulting transfer prices complicate the evaluation of the performance of the managers and the economic investments in a particular country.

..

442


10-54

(15-20 min.)

1.

1,500 units × ($37 - $21) = $24,000 increase in operating income if units are purchased inside.

2.

Variable manufacturing costs of $21 per unit.

3.

Currently available outside purchase price of $37 per unit.

4.

(a)

Benefit of $24,000 from the Montreal Division’s viewpoint, but disadvantage of 1,500 units × ($40 - $21) = $28,500 from the Toronto Division viewpoint. Therefore, net decrease in Canadian Instruments Company’s operating income of $4,500.

(b)

Benefit of zero to the Montreal Division, but disadvantage of ($40 - $37) (1,500) = $4,500 to the Toronto Division. Net decrease in Canadian Instruments Company’s operating income of $4,500.

(a)

Toronto Division's current ROI = $36,000 ÷ $300,000 = 12%. Proposed investment earns an ROI = $2,000 ÷ $20,000 = 10%. Therefore, the Toronto Division's ROI will decrease if the proposal is accepted.

(b)

$2,000 - .07($20,000) = $600 increase in the Toronto Division economic profit, so the Toronto Division would accept proposal.

5.

..

443


10-55

(25-35 min.) In a short space, this case gets to the heart of the problems of a control system: goal congruence and effort. In particular, it focuses on how the widespread accounting convention of writing off engineering costs as immediate expenses may inhibit wise investments. It is also a good problem on the motivational impact of cost allocations, so it might be assigned in conjunction with Chapter 12. 1.

The strong points of the present plan include the tendency of the PED manager to hire the optimal number of engineers and to use them efficiently. At first glance, the production managers will also tend to behave in similar fashion. In addition, the user receives no surprises because the total cost of each "contract" is known in advance. The weakest point of the present plan is not explicitly pinpointed in the case. (We usually do not raise this point until the proposed plan is discussed.) Why is top management considering a switch to a "no-charge" system? To encourage greater use of PED services! Such services are evidently being under-used. A likely reason for small usage is that the "expense" borne in the first year may exceed the prospective savings for the first year. Therefore, even if the investment is justified on a longer-run basis, the production managers feel too much pressure for short-run performance to look beyond the current year. (Moreover, many managers are transferred or promoted nearly every year.) Under the proposed plan, the PED manager may continue to hire engineers until their marginal cost exceeds the marginal savings. But a tenser atmosphere is likely. PED services would be a "free good." When the selling price is zero, the production managers will increase their demand. The PED manager (or some committee) will have to determine priorities. In contrast, the present plan uses a "market price" system of sorts. Priorities are determined by a negotiated contract at a predetermined price.

2.

Most students will favor the present system, although a minority may like the proposed system. Of course, other systems are possible. For example, an internal accounting system could capitalize the PED costs and amortize them over the "useful life" of the expected cost savings. The latter system would then provide a method of performance evaluation (incentive) that would be consistent with the decision model (long-run net savings) apparently favored by top management. Again, in the final analysis, the choice of a system will depend on top management's prediction of the impact of the particular method on the collective decisions of the affected managers. In this instance, incidentally, top management adopted the proposed plan. A major lesson here is that internal accounting systems are neither inherently good nor inherently bad. The role of timing and the wishes of top management dramatically affect the choice of a system. Thus, a particular system may solve

..

444


the problems of goal congruence and effort for a year or two or more. However, as time passes, the system invariably warrants correction or revamping. For example, after a class discussion of this case in an executive program, a French executive said in effect: "This case is one that I've experienced. A few years ago, our top management adopted the no-charge system to spur heavier use of our central research and development department. Five years later we returned to a charge system, because our central staff had ballooned to an intolerable level." In both instances, the choice of the system could have been correct. Finally, the literature on agency theory emphasizes risk congruence. That is, incentives may be designed to encourage or discourage risk-taking. The existing system discourages risk-taking on the part of individual managers because they have less chance to have a diversified portfolio of projects. The proposed system shifts the risk to the PED manager. Because this manager can attain a diversified portfolio, he may accept more risky projects. Top management may prefer the latter.

..

445


10-56

(20-30 min.)

1.

Management by Objectives (MBO) is a formal system for developing and making measurable the goals for each position in the organization for a given time period. Mutually agreed upon goals are set for each subordinate with his or her superior. Both agree on the objectives to be met and how they will be measured. Advantages most often claimed for the MBO system include: 1. 2. 3. 4. 5.

Increased subordinate motivation to accomplish goals. Channeling of subordinate efforts toward organizationally recognized goals rather than individual goals. Increased development of subordinate abilities through the systematic establishment of goals by subordinates. Improved performance appraisal accuracy over time because substantive measures are used rather than subjective supervisor evaluation. Increased communication between subordinate and superior.

Disadvantages associated with MBO include: 1. 2. 3. 4. 5. 2.

Likely emphasis on short-run rather than long-run consequences. Difficulty in dealing with non-quantifiable factors. Emphasis on organizational rather than personal goals, needs, and wants. The increased emphasis on counseling often requires too much time. Limited effectiveness in turbulent or less-structured environments.

The human value premises of MBO suggest that subordinates will attempt 100% achievement if they accept a clear and tangible set of objectives. Inherent in MBO is the premise that goal formation is a joint process, where individual subordinates are involved in setting goals for their activities and developing programs that lead to attainment of organizational goals. In addition, the MBO system allows for adjustments to be made in goals to account for errors that may have occurred during the formation of them. During the appraisal process of MBO, recognition should be given for partial achievement of goals as well as for reaching the various goals. Roger Ravenhill does not incorporate the human value premises of MBO in his management style for the following reasons: 1. 2. 3. 4.

..

Goal setting at Haida Company is not a joint process. Ravenhill assumes that only he can establish organizational and individual goals. Subordinates apparently are not consulted. Ravenhill has assumed that no errors have been made in assigning objectives. Apparently no analysis was conducted to determine the cause for any lack of achievement. It is likely that Ravenhill failed to use periodic review sessions to help subordinates find ways to meet their goals.

446


10-57 (25-30 min.) All amounts in this solution (except percentages) are in millions of dollars. ROA (based on segment EBIT and assets) Segment 2011 2010 North America 1,750 ÷ 2,433 = 71.9% 1,538 ÷ 1,941 = 79.2% Western Europe 721 ÷ 1,272 = 56.7 856 ÷ 1,031 = 83.0 Central & Eastern Europe 233 ÷ 448 = 52.0 253 ÷ 384 = 65.9 Greater China 777 ÷ 471 = 165.0 637 ÷ 379 = 168.1 Japan 114 ÷ 595 = 19.2 180 ÷ 568 = 31.7 Emerging Markets 688 ÷ 953 = 72.2 521 ÷ 683 = 76.3

Segment North America Western Europe Central & Eastern Europe Greater China Japan Emerging Markets

Economic Profit (based on segment EBIT and assets) 2011 2010 1,750 – (.1 × 2,433) = $1,506.7 1,538 – (.1 × 1,941) = $1,343.9 721 – (.1 × 1,272) = $593.8 856 – (.1×1,031)= $ 752.9 233 – (.1 × 448) = $188.2 253 – (.1 × 384) = $ 214.6 777 – (.1 × 471) = $729.9 637 – (.1 × 379) = $ 599.1 114 – (.1 × 595) = $54.5 180 – (.1 × 568) = $ 123.2 688 – (.1 × 953 ) = $592.7 521 – (.1 × 683) = $ 452.7

The North America segment produced the most economic profit both years, and Greater China had the highest ROI in both years. The Japan segment had the lowest ROI both years and the lowest economic profit both years. In 2011 the ROI decreased for every segment. The largest decrease was for Western Europe. Economic profit increased for three segments and decreased in three, where the increases were in North America, Emerging Markets, and Greater China. To assess a manager’s performance, all of these performance metrics must be examined relative to the economic situation in the segment’s part of the world. If the economy in a segment’s part of the world is not doing well, the manager that just maintains ROI and economic profit levels might be performing very well. Performance metrics should also be examined in light of the actions of competitors. Sometimes it is important to sacrifice short term return to maintain market position relative to a major competitor. It is hard to determine which manager is performing best simply from the data given. It is tempting to say that the manager of the Japan segment is performing worst since the segment had the lowest ROI or and the lowest economic profit. However, the competitive situation may be quite different for these segments. And at least the economic profit is positive for all segments (indicating that the segments are generating more than the cost of capital). 10-58 (25-30 min.) For the solution to this Excel Application Exercise, follow the step-by-step instructions provided in the textbook chapter.

..

447


1. Duane has the best performance using ROI, while Louis has the highest economic profit. 2. Hubert has both the lowest ROI and the lowest economic profit. 3. The method chosen will affect the decisions of the managers. The manager of Duane, the one with the largest ROI, would reject proposed investments returning between 10% and 14% even though such projects would be profitable for the company as a whole. Why? Because they would reduce the Duane division’s ROI. In contrast, under economic profit, every division would accept investment proposals if and only if they returned more than 10%. Thus, economic profit motivates managers to make decisions that are more congruent with management’s objectives.

10-59

(40 min. or more) The purpose of this exercise is to recognize that return on investment, a summary performance measure, is composed of two parts that may differ greatly by company and by industry. It also requires students to find publicly available information about a company, possibly using the Internet to do so. Requirement 1 is an individual exercise in information gathering and analysis. Requirement 2 brings in the group aspect. By comparing results across companies, students should be able to see that some businesses generate returns on their investment through large margins (e.g., computer software companies), while some have high capital turnover (e.g. grocery stores). Strategies to improve ROI can emphasize either increasing margins or turnover. If class time permits, reports from the groups would be worthwhile. In as little as 10 minutes of class time, students can see the variety in margins and turnover. They can also be reminded that the ultimate objective is return of investment, so focus on either margins or turnover without at least maintaining the other is not productive.

10-60 (40-50 min.) NOTE TO INSTRUCTOR: This solution is based on the web site as it was in late-2012. Be sure to examine the current web site before assigning this problem, as the information there may have changed. 1.

The main focus is on being able to locate a Marriott hotel in a particular city, state, or country and make a reservation. The emphasis is on promotion, with less emphasis as a portal to corporate information about Marriott.

2.

Each of Marriott’s brands appears to operate at least somewhat independently. They are good candidates for treatment as investment centers. Why? Each brand probably has some degree of control over its investment decisions. If not investment centers, the segments are likely to be profit centers.

3.

In the 2011 annual report, footnote 16 reported information on four segments, North American Full-service Lodging, North American Limited-service Lodging,

..

448


International Lodging, and Luxury Lodging. In previous years, there had been another segment, Timeshare, which has now been spun off (see footnote 17 of the 2011 annual report). Information reported for each segment includes Revenues, Income, Noncontrolling interest (shown only for 2009 when Marriott still had a timeshare segment with noncontrolling interests), Equity in earnings (losses) of equity method investees, Depreciation and amortization, Assets, Equity method investments, Goodwill, and Capital expenditures. 4.

Using average balances for assets, the ROA calculations are: NA Full-service NA Limited-service International Luxury

$351 ÷ $1,231 = 28.5% $382 ÷ $481 = 79.4% $175 ÷ $924 = 18.9% $74 ÷ $901 = 8.2%

5.

Using average balances for assets and income (which both include amounts not allocated to segments), the ROA for the corporation as a whole was $198 ÷ $7,447 = 2.7%. The ROA for the corporation is substantially lower than the ROA for any of the segments because there are substantial costs not allocated to the segments in determining segment income. In determining whether to use return on assets as a measure of how well a segment performed, you would want to consider several factors. For example: Does each of the segments have autonomy to operate independent of central management? Do segment managers have control over sales and investments in assets? The answers to these questions help determine the appropriate type of performance measures to use.

6.

Marriott primarily provides lodging services. If one segment provides lodging to employees of another segment, transfer prices are necessary. However, this is unlikely to comprise a large percentage of the business of any of the segments. Nevertheless, the company may have a policy such as charging a low transfer price if a hotel has excess capacity but charging the rack rate if a particular hotel is full. Marriott may also transfer services (cleaning, repairing, etc.) between different brands of hotels. For example, Marriott may have full-service hotels and limitedservice hotels in the same area that use services available from one another. The transfer prices for these exchanges would likely be set somewhere between variable cost and market values.

..

449


CHAPTER 11 COVERAGE OF LEARNING OBJECTIVES CASES, EXCEL, COLLAB., & INTERNET EXERCISES 68, 69, 70, 71, 73, 75

LO2: Use sensitivity analysis to evaluate effect of changes in predictions. LO3: Calculate the NPV difference between two projects using both the total project and differential approaches. LO4: Identify relevant cash flows for NPV analyses.

B5

ADDITIONAL ASSIGNMENT MATERIAL 7, 26, 29, 29, 30, 30, 31, 32, 32, 33, 33, 34, 34, 35, 36, 36, 37, 38, 38, 39, 39, 40, 41, 41, 51, 52, 52, 53, 53, 54, 54, 55, 55, 56, 56, 62, 64 40, 41, 41

B5

51, 52, 52

B5

LO5: Compute the after-tax net present values of projects. LO6: Explain the after-tax effect of cash of disposing of assets. LO7: Use the payback model and the accounting rate-ofreturn model and compare them with the NPV model. LO8: Reconcile conflict between using NPV model for making a decision and using accounting income for evaluating the related performance. LO9: Compute the impact of inflation on a capitalbudgeting project.

A3, A4, B3, B5 A5, B5, B5

25, 27, 51, 52, 52, 53, 53, 54, 54, 55, 55, 56, 56, 57, 57, 60, 60, 61 14, 15, 16, 19, 42, 43, 44, 45, 57, 57, 58, 58, 59, 60, 60, 64, 65, 65, 66 28, 58, 58, 59

71

B5

40, 46, 47, 48, 48, 61, 62

73

B5

63, 63

B5

49, 50, 66, 67, 67

LEARNING OBJECTIVE LO1: Describe capital budgeting decisions and use the net present value (NPV) method to make such decisions.

..

FUNDAMENTAL ASSIGNMENT MATERIAL A1, A2, B1, B2, B5

68, 71

68, 69, 70, 74 70, 72

450


CHAPTER 11 Capital Budgeting 11-A1 (15-25 min.) Answers are printed in the text at the end of the assignment material. 11-A2 (20-30 min.) This is a straightforward exercise. 1 & 2. The model indicates that the computers should be acquired because the net present value is positive.

Cash effects of operations, $110,000 Investment Net present value

14% Discount Factor

Total PV @ 14%

2.3216

£ 255,376 (240,000) £ 15,376

Sketch of Cash Flows (in thousands) 0 1 2 3 110

110 110

(240)

11-A3 (20-30 min.) This is a straightforward exercise. 1.

The model indicates that the servers should not be acquired.

Cash effects of operations, $300,000(1-.40) Cash effect of depreciation, savings of income taxes: $220,000 × .40 = $88,000 Total after-tax effect on cash Investment Net present value 2.

Total PV @ 12%

2.4018

$ 432,324

180

180 180

2.4018

211,358 643,682 (660,000) $ (16,318)

88

88

88

(660)

The computers should be acquired. The net present value rises, and now it is positive: After-tax impact of disposal on cash: .60 × ($90,000 - 0) PV is $54,000 × .7118 Net present value as above

..

Sketch of Cash Flows (in thousands) 0 1 2 3

12% Discount Factor

$ 54,000 $ 38,437 (16,318) 451


New net present value 3.

$ 22,119

This requirement demonstrates that the choice of a discount rate often is critical. Applying an 8% discount factor: $300,000 × (1 - .40) × 2.5771 $220,000 × .40 × 2.5771 Investment NPV is positive, so acquire

$463,878 226,785 $690,663 (660,000) $ 30,663

11-A4 (25-30 min.) 1.

Cash effects of operations: Before tax annual cash inflow Taxes @ 45%: 420,000 × .45 After-tax cash inflow Present value @ 14%: $231,000 × 5.2161

$ 420,000 189,000 $ 231,000 $1,204,919

Cash effects of depreciation*: Year Tax Savings** 1 .1429 × $2,000,000 × .45 = $ 128,610 2 .2449 × 2,000,000 × .45 = 220,410 3 .1749 × 2,000,000 × .45 = 157,410 4 .1249 × 2,000,000 × .45 = 112,410 5 .0893 × 2,000,000 × .45 = 80,370 6 .0892 × 2,000,000 × .45 = 80,280 7 .0893 × 2,000,000 × .45 = 80,370 8 .0446 × 2,000,000 × .45 = 40,140 Total present value

PV factor Present Value .8772 $ 112,817 .7695 169,605 .6750 106,252 .5921 66,558 .5194 41,744 .4556 36,576 .3996 32,116 .3506 14,073 $579,741

* Short-cut using Exhibit 11-7: .6441 × .45 × $2,000,000 = $579,690, which differs from the $579,741 computed above only because of rounding error. **Factors .1429, .2449, etc. are from Exhibit 11-6. Summary: Present value of cash effects of operations Present value of cash effects of depreciation Total after-tax effect on cash Investment Net present value is negative, so don't acquire

..

$ 1,204,919 579,741 $ 1,784,660 (2,000,000) $ (215,340)

452


2.

The 7-year MACRS analysis will apply regardless of the economic life of the equipment. The only change from requirement 1 will be the added five years of cash effects from operations: This change can be incorporated by re-computing the value of the $231,000 aftertax inflow but now for 15 years, and then using that in the calculation: Present value @ 14%: $231,000 × 6.1422 $1,418,848 Summary: Present value of cash effects of operations Present value of cash effects of depreciation Total after-tax effect on cash Investment NPV is still negative, so don't acquire

$ 1,418,848 579,741 $ 1,998,589 (2,000,000) $ (1,411)

Alternatively, the change can be incorporated by finding the present value of the additional 5 years of $231,000 after tax savings, and adding it to the previous NPV: PV of $231,000 per year for 5 years at 14% = 3.4331 × $231,000 = $793,046 To account for the delay of 10 years before savings begin: $793,046 × .2697 $213,885* NPV as above (215,340) NPV is still negative, so don’t acquire. $(1,455) * Or, $231,000 × (6.1422 – 5.2161) = $231,000 × .9261 = $213,929, which gives the same NPV as the first approach, $(1,411). The small relative difference in the answers here is attributable to rounding of the present value factors. The two alternative approaches are conceptually identical but because the present value factors are rounded to four decimal places, the computations are numerically slightly different. 11-A5 (5-10 min.) Many students forget to add the cash proceeds to the tax effect. Answers are in dollars. (a) Cash sale price Book value Gain (loss) Effect on cash income taxes at 25%: (b) Cash tax saving (inflow effect) (c) Cash tax paid (outflow effect)

..

305,000 250,000 55,000

230,000 250,000 (20,000) 5,000

(13,750)

453


Total after-tax effect on cash (a) plus (b) (a) minus (c)

..

235,000 291,250

454


11-B1 (15-20 min.) 1.

Using the right table is essential. Factors for this part are from Table 1: (a) PV = $22,000 × .7473 = $16,440.60 (b) PV = $22,000 × .6209 = $13,659.80 (c) PV = $22,000 × .5194 = $11,426.80

2.

Use Table 2: (a) PV = $50,000 × 3.4651 = $173,255 (b) PV = $50,000 × 3.1699 = $158,495 (c) PV = $50,000 × 2.9137 = $145,685

3.

Use Table 2: (a)

PV = annual withdrawal × F $6,000,000 = annual withdrawal × 13.5903 Annual withdrawal = $6,000,000 ÷ 13.5903 = $441,491.36 (b)

PV = annual withdrawal × F $6,000,000 = annual withdrawal × 9.8181 Annual withdrawal = $6,000,000 ÷ 9.8181 = $611,116.20

4.

Contract Y has the higher present value despite the lower total dollars paid: Year 1 2 3 4 Total

Present Value @16% From Table 1 .8621 .7432 .6407 .5523

Present Value of Contract X $215,525 557,400 544,595 552,300 $1,869,820

Present Value of Contract Y $474,155 609,424 416,455 386,610 $1,886,644

11-B2 (20-30 min.) This is a straightforward exercise. 1 & 2. The model indicates that the equipment should be acquired because the net present value is positive. Sketch of Cash Flows 14% Total (in thousands) Discount PV 0 1 2 3 4 5 Factor @ 14% Cash effects of operations, $140,000 3.4331 $ 480,634 140 140 140 140 140 Investment (420,000) (420) Net present value $ 60,634

..

455


11-B3

(20-30 min.) This is a straightforward exercise.

1.

The model indicates that the equipment should not be acquired. 14% Total Discount PV Factor @ 14%

Cash effects of operations, $130,000 × (1-.40) Cash effect of depreciation, savings of income taxes*: Total after-tax effect on cash Investment Net present value

Sketch of Cash Flows (in thousands) 0 1 2 3 4 5

3.4331

$267,782

78

78

78

78

78

3.4331

109,859 $377,641 (400,000) $ (22,359)

32 32

32

32

32

(400)

*Depreciation is $400,000 ÷ 5 = $80,000 per year; annual tax savings is $80,000 × .40 = $32,000.

2.

The equipment should not be acquired. The net present value is negative. After-tax impact of disposal on cash: .60 × ($25,000 - 0) = $15,000 PV is $15,000 × .5194 Net present value as above New net present value

3.

$ 7,791 (22,359) $(14,568)

Applying 10% discount factors: $130,000 × (1 - .40) × 3.7908 $80,000 × .40 × 3.7908

$ 295,682 121,306 $ 416,988 Investment (400,000) NPV is positive, so acquire. $ 16,988 $120,000 × (1 - .30) × 2.6730 $ 224,532 $50,000 × .30 × 2.6730 40,095 $ 264,627 Investment (150,000) NPV is even more positive, so acquire. $ 114,627

..

456


11-B4

(25-30 min.)

1.

See Exhibit 11-B4 on the following page for requirement 1.

2.

The major reason for this requirement is to underscore the fact that the present value of the depreciation tax savings is unchanged regardless of the length of the economic life of the asset. PV @ 10% of 6 years of $41,250 4.3553 × 41,250 Present value of tax savings from depreciation Total Less: Initial Investment NPV

$ 179,656 104,388 284,044 300,000 $(15,956)

Alternative calculation to just add present value of 6th year of operational inflow: PV of the $41,250 to be received in the 6th year, $41,250 × .5645 factor $ 23,286 NPV as above (39,241) NPV is still negative, so don’t acquire. $(15,955) Note that the alternative approaches are conceptually identical but differ numerically due to rounding of the present value factors. 11-B5 (5-10 min.) Many students forget to add the cash proceeds to the tax effect. Answers are in dollars. (a) Cash sale price Book value Gain (loss)

1.

2.

35,000 40,000 (5,000)

85,000 40,000 45,000

Effect on cash income taxes at 30%: (b) Cash tax saving (inflow effect) (c) Cash tax paid (outflow effect)

1,500

Total after-tax effect on cash (a) plus (b) (a) minus (c)

36,500

..

(13,500)

71,500

457


EXHIBIT 11-B4 (Dollar amounts in thousands) 1. Cash effects on operations, $75 × (1 - .45) Cash effects of depreciation: Savings Year Tax Deduction @ 45% 1 .20 × $300 = $60.0 $27.0 2 .32 × 300 = 96 43.2 3 .192 × 300 = 57.6 25.92 4 .1152 × 300 = 34.56 15.552 5 .1152 × 300 = 34.56 15.552 6 .0576 × 300 = 17.28 7.776 PV of tax shield Total after-tax effect on cash Investment NPV is negative, so don't acquire.

10% Discount Factor

Total PV @ 10% ($000)

3.7908

156.371

<––– 41.25 41.25 41.25 41.25 41.25

.9091 .8264 .7513 .6830 .6209 .5645

24.546 35.700 19.474 10.622 9.656 4.390 104.388 260.759 (300.000) (39.241)

<––– 27.0 <–––––––––43.20 <–––––––––––––––25.92 <––––––––––––––––––––15.552 <–––––––––––––––––––––––––––15.552 <–––––––––––––––––––––––––––––––– 7.776

0

Sketch of Cash Flows (in dollars) 1 2 3 4 5

6

(300)

Note: The cash effects of MACRS depreciation can be computed more easily using Exhibit 11-7. Present value of tax savings = Original cost × Tax rate × Factor from 11-7 = $300.000 × .45 × .7733 = $104.396. This differs slightly from the $104.388 calculated above because of rounding.

..

458


11-1 The accountant has a limited role in the project identification phase. In the selection phase, accountants provide information for predicting cash inflows and outflows and often are in charge of summarizing the information using a capital budgeting model. The post-audit phase uses information about the results of investment projects; this information usually comes from the accounting system. 11-2 Discounted cash flow is a superior method for capital budgeting because it measures profitability and takes into account the time value of money. 11-3 No. A higher required rate of return reduces the present value of future cash inflows and outflows, and hence the difference between them. The initial investment (at time zero) is unaffected. Therefore, the net present value is less. Higher discount rates reduce the price a company should be willing to pay. 11-4 No. It is true that the DCF model assumes certainty and perfect capital markets. But all practical capital budgeting models make even more limiting assumptions. The DCF model is not perfect, but in most situations it is the best practical alternative. 11-5 Yes, double counting does occur if depreciation expense is considered separately. The cost of an investment is represented by its cash outflow at year zero. An additional consideration of depreciation would be double counting. Note, however, that the tax savings from depreciation is considered separately. 11-6 No. The IRR and NPV models generally make the same decision. Suppose we compute the NPV of a project using the cost of capital as the discount rate. If its NPV is greater than zero, then its IRR is generally greater than the cost of capital. 11-7 The real option model recognizes the value of contingent investments. Thus, in the case of a project that can either be implemented all at once or in stages at a greater cost, the latter alternative, i.e., staging the project, may be better if the company gains enough information in the early stages to make better decisions in the later stages. A real option model successfully recognizes the value of this staging. 11-8 Sensitivity analysis is especially appropriate for organizations that do not have accurate cash flow predictions. Sensitivity analysis can help a manager decide whether it is worth gathering information to improve cash flow predictions. 11-9 The differential approach should lead to the same choice between alternatives as the total approach because it merely disregards the factors that are constant for each alternative: those that make no difference. 11-10 The NPV model is appropriate for all types of investments. However, with some types of investments, such as those in advanced technology, NPV must be carefully applied. Managers should quantify as many qualitative effects as best as possible and

..

459


include them in the model, or they should consider them as subjective factors in addition to the NPV analysis. 11-11 The marginal tax rate is the rate paid on additional amounts of pretax income. In contrast, the average tax rate is the total taxes paid divided by the total pretax income. 11-12 No. Two sets of books are appropriate. The objectives of tax reporting and shareholder reporting differ; therefore, the rules for reporting to each differ. If companies used tax rules for financial reporting, users of the statements would not receive the information they judge to be most useful. Likewise, if tax authorities accepted financial reporting rules, certain social goals sought by the taxation system would not be met. 11-13 Tax avoidance is the achieving of a reduction in income tax payments through legal means; tax-evasion achieves the same end through illegal means. Tax avoidance is considered moral; tax evasion, immoral. Tax avoidance uses the rules of the system (tax laws) in an optimum way; tax evasion disregards the rules. 11-14 Depreciation spreads the cost of an asset over its useful life. Thus, while computing taxable income, it is often allowed as a deduction in tax law. This results into a lower taxable income and consequently lower burden of tax. Thus, the existence of depreciation leads to tax savings. 11-15 The present value of tax savings from depreciation is called depreciation tax shield. When higher costs of an asset are allocated in the initial years, it increases the tax savings in the initial years and reduces them in the later years. Since present value factors are relatively higher during the initial years, the present value of initial high tax savings yields higher total present value of tax savings on depreciation, and the depreciation tax shield improves. 11-16 When a company sells its asset at termination at a value different from the book value, it attracts tax implications. If at termination the asset is sold at a value higher than the book value, there is a gain and an additional tax payment. At the same time, selling assets at lower than the book value leads to a loss and tax savings. 11-17 Because of the time value of money, the earlier a company takes tax deductions and thereby saves taxes, the larger the present value of the tax savings. 11-18 Yes. MACRS treats assets as if they were purchased at midyear, so they have depreciation effects for one tax year more than the number of years of their depreciable lives. For example, if a company purchases a three-year MACRS asset during 2010 and pays taxes on a calendar year basis, its depreciation begins July 1, 2010, and extends through June 30, 2013, affecting taxes in 4 years (2010, 2011, 2012, and 2013). 11-19 When an asset is sold before the end of the recovery period, it attracts two separate tax effects. While the first is the additional tax payment on the gain on sales or ..

460


tax saving on the loss on sales of an asset, the second is the eliminated future depreciation and corresponding tax savings. Both the tax effects have significant implications in project evaluation. 11-20 It is useful to learn the "payback" and "accounting rate-of-return" methods of capital budgeting because they are widely used today. In addition, the payback method may give a rough indication of riskiness, and the accounting rate of return shows a project's effect on an accrual accounting income statement. The comparative advantage of discounted cash flow can also be seen by contrast. Surveys show that most companies use more than one method to evaluate capital budgeting projects, so it is likely that you will need to understand more than DCF models. 11-21 The basic flaw in the payback method is that it does not compare the total profitability of alternative projects. It simply measures the rate of recoupment of the initial investment. It also ignores the time value of money. 11-22 If a company makes capital-budgeting decisions using DCF and evaluates performance using accrual accounting numbers, a conflict may arise. Often accrual accounting can show low profits in the early years of a project's life because of high depreciation, write-offs of old equipment that is being replaced, or slow growth in revenues as the company penetrates new markets. Such low accrual accounting profits might discourage managers from making investments that have a positive net present value. 11-23 The three components of the market or nominal interest rate are: 1) risk-free element, or pure rate of interest, 2) business-risk element, and 3) inflation element. 11-24 The correct analysis under inflation (a) uses a required rate that includes an element attributable to inflation and (b) explicitly adjusts the predicted operating cash flows for the effects of inflation. 11-25 It is difficult to predict the cash flows to be received from an investment in research and development (R&D) because there is so much uncertainty involved. The manager’s assertion that this is impossible is not uncommon. However, even investments in R&D must, on average, return more than the cost of capital if they are to be desirable investments. Therefore, it is usually worth the effort to predict, as well as possible given the uncertainties, the possible outcomes of R&D activities and their effect on the company’s cash flows. From that, a net present value can be computed. 11-26 The net present value of an investment project represents the increase (or decrease) in the value of the firm from investing in the project, provided the cash flows and cost of capital are estimated correctly. Thus, implementing a positive net present value project will increase the company’s value, while implementing a negative net present value project will decrease it.

..

461


11-27 The direct cash flows are the easiest to predict. These would include the investment in the new machine (less any salvage value of the old machine), the savings in labor and other variable operating costs because of the decreased production time per unit, and the estimated salvage value of the new machine at the end of its economic life. Those more difficult to measure are revenue from increased sales because of higher quality or more timely delivery schedules, cost savings from reworking defective units because of the more accurate standards of the new machine, and decreased storage costs because the faster production process allows quicker adaptation to changes in demand and thus less need for large inventories. 11-28 The first situation is reasonably clear. There is no legal or ethical reason not to take the depreciation allowed by the tax law. The second is much more problematic. Investing offshore is generally not illegal, although its ethics might be questionable. If the offshore investment is really a sham to avoid (or evade) taxes while the company still takes advantage of the business climate provided in the U. S. for most of its business, it is not contributing to society as much as it is taking from it. But even more questionable is the use of transfer prices to move profits into a tax haven. Again, there may be no technical illegality (although there are laws relating to transfer prices that might be violated), artificially transferring profits from the society in which the company really earns them to a tax haven where the company is located only for the tax purposes is certainly of dubious ethics. 11-29 (10-15 min.) 1.

The present value is €320,000 and the annual payments are an annuity, requiring use of Table 2: (a) €320,000 = annual payment × 7.7217 annual payment = €320,000 ÷ 7.7217 = €41,441 (b) €320,000 = annual payment × 6.4177 annual payment = €320,000 ÷ 6.4177 = €49,862 (c) €320,000 = annual payment × 5.4262 annual payment = €320,000 ÷ 5.4262 = €58,973

2.

(a)€320,000 = annual payment × 5.7864 annual payment = €320,000 ÷ 5.7864 = €55,302 (b) €320,000 = annual payment × 5.0330 annual payment = €320,000 ÷ 5.0330 = €63,580 (c)€320,000 = annual payment × 4.4226 annual payment = €320,000 ÷ 4.4226 = €72,356

3.

(a) Total payments= 10 × €49,862= €498,620 Total interest paid= €498,620 - €320,000 = €178,620 (b) Total payments= 7 × €63,580 = €445,060 Total interest paid = €445,060- €320,000 = €125,060

..

462


11-30 (10 min.) The initial step on solving present value problems focuses on a basic question: Which table should I use? No computations should be made until you are convinced that you are using the correct table. 1.

Use Table 1, row 10, 5% column. The Bank of England will lend £552,510,000. The £900 million is a future amount. Its present value is: PV = £900,000,000 × .6139 = £552,510,000

2.

Use Table 2, row 10, 5% column. The Bank of England will lend £694,953,000. The £90 million is a uniform periodic payment at the end of a series of years. Therefore, it is an annuity. Its present value is: PV = £90,000,000 × 7.7217 = £694,953,000 A In particular, note that the Bank of England is willing to lend more than in requirement 1 even though the interest rate is the same. Why? Because the bank will receive the £900,000,000 more quickly if it receives 10 payments of £90,000,000 per year rather than waiting to receive the entire £900,000,000 at the end of 10 years.

11-31

(10-20 min.)

1.

a. PV = $250,000 × .6806 = $170,150 b. PV = $250,000 × .5674 = $141,850

2.

The annual rates would be halved and the periods doubled. Present values decline: a. PV = $250,000 × .6756 = $168,900 b. PV = $250,000 × .5584 = $139,600

3.

Present values rise because the money is repaid more quickly: a. PV = $50,000 × 3.9927 = $199,635 b. PV = $50,000 × 3.6048 = $180,240

11-32

(10-15 min.)

1.

$450,000 = Future amount × 0.5674 Future amount = $450,000 ÷ 0.5674 = $793,091

2.

$450,000 = Annual installments × 3.6048 Annual installments = $450,000 ÷ 3.6048 ..

463


= $124,834 11-33

(10 min.) The deferral cost SBI ₹12,624,000 in present value, computed as follows: Present value of ₹80,000,000 at the end of 2022–2023 ₹60,104,000 Present value of ₹80,000,000 at the end of 2020–2021 72,728,000 Sacrifice in present value ₹12,624,000 A more detailed analysis follows: Present Value Present Value Present Value @ 10% of Original Loan of Revised Loan Year(end) from Table 1 Contract Contract (with moratorium) 2020-21 .9091 ₹145,456,000 ₹ 72,728,000 2021-22 .8264 165,280,000 165,280,000 2022-23 .7513 165,286,000 225,390,000 Total ₹476,022,000 ₹463,398,000 Difference (₹476,022,000 - ₹463,398,000) = ₹12,624,000

11-34

(5-10 min.) This simple exercise requires use of both Table B-1 and Table B-2.

PV of 3 payments of $60,000 each: $60,000 × 2.7232 $ 163,392 PV of $350,000 payment in 3 years: $350,000 × 0.8638 302,330 Total NPV $465,722 11-35 (20-25 min.) This basic exercise develops comfort with the tables and the NPV method. Number of years Amount of annual cash inflow Required initial investment Required rate of return Net present value

7 $8,000 $37,967a 10% $ 980

18 $13,749 b $70,000 18% ($10,009)

18 $ 30,000 $50,000 8%c $231,157

28 $ 16,000 $29,000 20% $50,515 d

a

(4.8684 × $8,000) - $980 = $38,947 - $980 = $37,967 (5.2732 × CF) - $70,000 = ($10,009); CF = ($70,000 - $10,009)  5.2732 = $11,377 c(F × $30,000) - $50,000 = 231,157; F = $281,157  $30,000 = 9.3719 On the 18 year row, the factor 9.3719 is a 8% rate b

d

PV Factor for 20% on 28-year row is 4.9697; $16,000 × 4.9697 = $79,515 NPV= $79,515 - $29,000 = $50,515

11-36

..

(10 min.)

464


Yes. The net present value is positive. Initial outlay * € (88,000) Present value of cash operating savings, from 8-year, 14% column of Table 2, 4.6389 × €44,000 204,112 Net present value € 116,112 * The trade-in allowance really consists of a €2,000 adjustment of the selling price and a bona fide €40,000 cash allowance for the old equipment. The relevant amount is the incremental cash outlay, €88,000. The book value is irrelevant. 11-37

(10-15 min.)

1.

The quickest solution is to "net" the flows for each year: 1. $200,000 - $150,000 = $ 50,000 ┐ 2. 250,000 - 200,000 = 50,000 ├ an annuity of 3 payments (a) 3. 300,000 - 250,000 = 50,000 ┘ 4. 450,000 - 300,000 = 150,000 ┐ an annuity of 2 payments 5. 500,000 - 350,000 = 150,000 ┘ deferred three years (b) (a) $50,000 × 2.3216 (b)$150,000 × 1.6467 × .6750 Total Less initial investment Net Present Value (NPV)

$116,080 166,728 $282,808 235,000 $ 47,808

Various other approaches would reach the same answer, but they would involve more computations. 2.

The NPV is positive because at a 12% rate, the present value of the net inflows will be higher than at 14%, so NPV will increase.

11-38

(10 min.)

1.

NPV @ 10% = $4,600 × 6.8137 = $31,343.02 - $30,000 = $1,343.02. With a required rate of 10%, the NPV is positive, so this investment is desirable.

2.

NPV @ 12% = $4,600 × 6.1944 = $28,494.24 - $30,000 = $(1,505.76). With a required rate of 12%, the NPV is negative so the investment is undesirable.

3.

The higher the required rate of return, the lower the NPV of future cash flows. Potential investments that have an initial cash outflow for investment followed by cash inflows will be less desirable the higher the required rate of return. In this case, a higher required rate of return makes the investment undesirable.

..

465


11-39

(10-15 min.)

1.

NPV @ 10% = 20,000 × 3.7908 = €75,816 - €72,096 = €3,720 NPV @ 12% = 20,000 × 3.6048 = €72,096 - €72,096 = €0 NPV @ 14% = 20,000 × 3.4331 = €68,662 - €72,096 = €(3,434)

2.

The IRR is the interest rate at which NPV = $0; therefore, from requirement 1 we know that IRR = 12%.

3.

The NPV at the company’s cost of capital, 10%, is positive, so the project should be accepted.

4.

The IRR (12%) is greater than the company’s cost of capital (10%), so the project should be accepted. Note that the IRR and NPV models give the same decision.

11-40 (30-45 min.) This problem deals essentially with sensitivity analysis, which asks how the basic forecasted results will be affected by changes in the critical factors (useful life, cash flows) that influence rate of return. 1.

$40,000 ÷ $10,000 = 4 years

2.

NPV= ($10,000 × 3.4331) - $40,000 = $(5,669)

3.

a) NPV = ($10,000 × 2.3216) - $40,000 = ($16,784) b) NPV = ($10,000 × 5.2161) - $40,000 = $12,161

4.

NPV = ($8,000 × 3.4331) - $40,000 = ($12,535)

5.

NPV = ($9,000 × 2.9137) - $40,000 = ($13,777)

11-41

(15-20 min.)

1.

NPV = (BDT1,200,000 × 3.7908) – BDT4,160,000 = BDT4,548,960 – BDT4,160,000 = BDT388,960

2.

(a) NPV = (BDT1,200,000 × 3.1699) - BDT4,160,000 = BDT3803880 – BDT4,160,000 = BDT(356,120) (b) NPV = (BDT1,200,000 × 4.8684) - BDT4,160,000 = BDT5,842,080 – BDT4,160,000 = BDT1,682,080

3.

(a) NPV = (BDT960,000 × 3.7908) - BDT4,160,000 = BDT3,639,168 – BDT4,160,000 = BDT(520,832)

..

466


(b) NPV = (BDT1,440,000 × 3.7908) - BDT4,160,000 = BDT5,458,752 – BDT4,160,000 = BDT1,298,752 4.

(a) NPV = (BDT1,440,000 × 4.8684) - BDT4,160,000 = BDT7,010,496 – BDT4,160,000 = BDT2,850,496 (b) NPV = (BDT960,000 × 3.1699) - BDT4,160,000 = BDT3,043,104 – BDT4,160,000 = BDT(1,116,896)

5.

(Savings × 3.7908) - BDT4,160,000 =0 Savings = BDT4,160,000  3.7908 Savings = BDT1,097,394

11-42

(5-10 min.) Amounts in thousands of dollars. Annual Income Statement Effects: (S) Sales (E) Expenses excluding depreciation (D) Depreciation Total expenses Income before income taxes (T) Income taxes outflow at 36% (I) Net income

750 275 200 475 275 99 176

Total after-tax cash inflow from operations is either S - E - T = 750 - 275 - 99 = 376 or I + D = 176 + 200 = 376 Annual Cash Flow Effects: Cash effect of operations: (S - E) Pretax cash inflow from operations,750 - 275 475 Multiplied by (1 – tax rate) ×.64 After-tax cash inflow from operations Cash effect of depreciation tax deduction: (D) Depreciation 200 Multiplied by tax rate ×.36 Tax savings due to depreciation Total after-tax effect on cash 11-43

72 376

(5-10 min.)

Cash effect of operations: Cash inflow from operations: $1,700,000 - $350,000 Multiplied by (1 –37% tax rate) After-tax inflow from operations

..

304

$1,350,000 ×.63 $850,500

467


Cash effects of depreciation tax deduction: Depreciation Income tax rate 37% Tax Savings due to depreciation Total after-tax effect on cash

$425,000 ×.37 157,250 $1,007,750

Another way to find the after-tax effect on cash is: Cash inflow from operations: $1,700,000 - $350,000 Actual income taxes @ 37% After-tax cash inflow from operations 11-44

$1,350,000 (342,250) $1,007,750

(10 min.)

The month and day on which an asset is acquired does not affect its tax depreciation. MACRS applies the half-year convention to all assets, taking ½ year of depreciation in the year of acquisition and ½ year of depreciation in the final year of the recovery period. 20X8 20X9 1. 3-year property: 33.33% & 44.45% of $55,000 $18,332 $24,448 2. 5-year property: 20% and 32% of $3,500

700

1,120

3. 5-year property: 20% and 32% of $16,000

3,200

5,120

4. 7-year property: 14.29% and 24.49% of $9,500

1,358

2,327

11-45

(10 min.) This problem could be solved by specifying appropriate schedules of tax savings and computing the present values. However, the process would be extremely timeconsuming. The steps outlined in the chapter make the computations quite simple. (a) (b) (c) (d) (e)

$220,000 × .5798 × .33 = $ 42,093 $640,000 × .6810 × .33 = $ 143,827 $ 40,000 × .7059 × .29 = $ 8,188 $950,000 × .7975 × .45 = $ 340,931 $420,000 × .7782 × .25 = $ 81,711

11-46

(10-15 min.) Annual addition to profit = 25% × $14,000 = $3,500.

1.

Payback period is $28,000 ÷ $3,500 = 8 years. It is not a good measure of profitability because it ignores returns beyond the payback period and it does not account for the time value of money.

2.

NPV = $5,114. Reject the proposal because NPV is negative. Computation: NPV = ($3,500 × 6.1446) - $28,000 = $21,506 - $28,000 = $(6,494) ..

468


3.

ARR = (Increase in average cash flow – Increase in depreciation) ÷ Initial investment = ($3,500 - $2,800) ÷ $28,000 = 2.5%

11-47 (10-15 min.) 1. Investment A payback period: $36,000 ÷ $4,000 = 9 years Investment B payback period: $36,000 ÷ $4,500 = 8 years Investment B has the shorter payback period, so it seems more desirable using the payback model. 2. NPV of A: $4,000 × 9.8181 = $39,272 - $36,000 = $3,272 NPV of B: $4,500 × 5.7466 = $25,860 - $36,000 = $(10,140) Investment A has the higher NPV, so it is more desirable. In fact, investment B has a negative NPV, so it would reduce the value of Stallone Company. 3. The payback model does not consider the overall profitability of the two investments. It ignores all cash flows beyond the payback period and gives all cash inflows during the payback period the same value as dollars at time zero, regardless of when received. Although investment B returns its investment in 8 years, it has no additional cash flows. Investment A returns its investment more slowly, but the 12 extra years of $4,000 annual cash flows make it more valuable than investment B. 11-48

(15 min.)

1.

₹700,000 ÷ ₹140,000 = 5 years

2.

₹140,000 × 6.1446 = ₹860,244. The KMC should buy the vehicles because the net present value is a positive ₹860,244 – ₹700,000 = ₹160,244.

3.

ARR = [₹140,000 – (₹700,000 ÷ 10 years)] ÷ ₹700,000 = ₹70,000 ÷ ₹700,000 = 10%

11-49

(30 min.)

1 & 2. See Exhibit 11-49 on the following page for requirements 1 and 2. The footnotes for the exhibit follow: a Be sure to use a nominal discount rate, which includes an element attributable to inflation, and adjust the predicted cash flows for inflationary effects. Each year is 2 3 adjusted for anticipated inflation: $91,000 × 1.05, $91,000 × 1.05 , $91,000 × 1.05 , etc. ..

469


b

The annual savings in income taxes will be unaffected by inflation. Why? Because the income-tax deduction must be based on original cost of the asset in year 0 dollars. Amounts are 35% of (.20 × $290,000), (.32 × $290,000), (.192 × $290,000), (.1152 × $290,000), (.1152 × $290,000), and (.0576 × $290,000).

c

Shortcut using Exhibit 11-7: .5631 × $290,000 × .35 = $57,155, which differs from the $57,158 calculated above only because of rounding error.

d

A common error is to adjust the discount rate as above but not adjust the predicted cash inflows.

3.

The method of Requirement 1 is correct. The required rate of return includes an inflation element, and the cash inflows are adjusted for inflation. In Requirement 2 the required rate of return includes an inflation element, but the cash inflows are not adjusted for inflation. This understates the cash flows, so the net present value is understated. The incorrect method can lead to underinvestment, because desirable investments can be rejected.

..

470


EXHIBIT 11-49 (See footnotes on previous page.) 1. Correct Analysis Cash operating inflows:a Pretax inflow in year-0 dollars Tax effect at 35% After-tax effect

Subtotal Cash effect of depreciation:b Savings in income taxesc

Investment in equipment Net present value 2. Incorrect Analysis Cash operating inflows after taxesd Tax effect of depreciation (same as above) Investment in equipment Net present value

..

At 25 Percent P.V. Present Factor Value

$140,000 49,000 $ 91,000 .8000 .6400 .5120 .4096 .3277

Sketch of Relevant Cash Flows (in dollars) –––––––––––––––––––––––––––––––––––– 0 1 2 3 4 5 6

$ 76,440 64,210 53,936 45,306 38,060 $ 277,952

< ––––––—95,550 <––––––––––––— 100,328 <––––––––––––––––––— 105,344 < ––––––––––––––––––––––––— 110,611 < ––––––––––––––––––––––––––––—— 116,142

16,240 20,787 9,978 4,789 3,832 1,532 57,158 (290,000) $ 45,110

<–––––— 20,300 <––––––––––— 32,480 <–––––––––––––––– 19,488 <––––––––––––––––––––––— 11,693 <––––––––––––––––––––––––––– 11,693 <–––––––––––––––––––––––––––––— 5,846

2.6893

$ 244,726

<––––––—91,000 91,000 91,000 91,000 91,000

1.0000

57,158 (290,000) $ 11,884

(290,000)

.8000 .6400 .5120 .4096 .3277 .2621 1.0000

(290,000)

471


11-50

(30-40 min.) Answers are in Mexican pesos.

1.

After-tax cash operating savings, .6 × 150,000 = 90,000 PV of cash operating savings, 90,000 × 3.1272 Income tax savings from depreciation .4 × (420,000 ÷ 5) = .4 × 84,000 = 33,600 PV = 33,600 × 3.1272 PV of total savings Required outlay at time zero Net present value

281,448 105,074 386,522 (420,000) (33,478)

Note how income taxes have a two-edged effect. They chop the present value of the cash operating savings by 40%, but the depreciation deduction provides income tax savings. 2.

See Exhibit 11-50 on the following page for requirement 2.

3.

The analysis in Requirement 2 is correct. The cash flows and the required rate of return incorporate the 10% rate of inflation. In Requirement 1, the 18% required rate of return includes an inflation element, but the predicted cash flows ignore inflationary effects.

11-51

(30-35 min.)

1. Salaries Overtime Repairs and maintenance Toner, supplies, etc. Total annual cash outflows

Annual Operating Cash Flows Xerox $74,880(a) 3,888(c) 2,700 1,800 $83,268

Brother $58,240(b) -600 4,500 $63,340

Difference $16,640 3,888 2,100 (2,700) $19,928

(a) ($12 × 40 hrs.) × 52 weeks × 3 employees = $480 × 52 × 3 = $74,880 (b) ($14 × 40 hrs.) × 52 weeks × 2 employees = $560 × 52 × 2 = $58,240 (c) ($18 × 6 hrs.) × 12 months × 3 machines = $108 × 12 × 3 = $ 3,888 Initial Cash Flows Purchase of Brother machines Sale of Xerox machines Training and remodeling Total

..

Xerox $ ---$ --

Brother $60,000 -5,250 3,500 $58,250

Difference $60,000 -5,250 3,500 $58,250

472


EXHIBIT 11-50 All numbers are expressed in Mexican pesos. 2. Cash operating savings:*

Total Income tax savings from depreciation not changed by inflation, see 1 Total Required outlay at time zero Net present value

18% PV Factor .8475 .7182 .6086 .5158 .4371

Total Present Value 83,903 78,212 72,904 67,966 63,356 366,341

3.1272

105,074 471,415 (420,000) 51,415

1.0000

Sketch of Relevant Cash Flows 0

1 99,000

2

3

4

5

108,900 119,790 131,769 144,946

33,600

33,600

33,600

33,600

33,600

(420,000)

*Amounts are computed by multiplying (150,000 × .6) = 90,000 by 1.10, 1.10 2, 1.10 3, etc.

..

473


PV

Present value of $1.00 Present value of Discounted Cash Flows Annual Cash Flows at 14% 0 1 2 … 9 10 TOTAL PROJECT APPROACH: Brother: Init. cash outflow 1.0000 $ (58,250) Oper. cash flows 5.2161 (330,388) (63,340) (63,340) … (63,340) (63,340) Total $(388,638) Xerox: Oper. cash flows 5.2161 $(434,334) Difference in favor of replacement $ 45,696 INCREMENTAL APPROACH: Initial investment 1.0000 $(58,250) Annual operating cash savings 5.2161 103,946 Net present value of purchase $ 45,696 2.

(83,268) (83,268)

(83,268) (83,268)

19,928

19,928

19,928

19,928

The Xerox machines should be replaced by the Brother equipment. Net savings = (Present value of expenditures to retain Xerox machines) less (Present value of expenditures to convert to Brother machines) = $434,334 - $388,638 = $45,696

3.

a. b.

..

How flexible is the new machinery? Will it be useful only for the presently intended functions, or can it be easily adapted for other tasks that may arise over the next 5 years? What psychological effects will it have on various interested parties?

474


11-52

(40 min.) Year 1

Year 2

Year 3

(Figures in million dollars) Year 4 Year 5 Total PV 39.93 43.92 27.951 30.744 7.29 6.561 2.187 1.968 0.6830 0.6209 19.091 19.089 95.453 1.494 1.222 9.500

Net savings in costs 30.00 33.00 36.30 After tax net savings 21.00 23.1 25.41 Depreciation 10 9 8.1 Tax on depreciation @ 30% 3 2.7 2.43 PV Factor 0.9091 0.8264 0.7513 PV of after-tax net savings 19.091 19.090 19.091 PV of tax savings on 2.727 2.231 1.826 depreciation Total PV Less. Initial investment NPV The computerized distribution system should be installed. Schedule of costs and depreciation: Year 1 Year 2 Year 3 Savings in costs: Reduction in labor cost 10.00 11.00 12.10 Reduction in space 18.00 19.80 21.78 Reduction in inventory 12.00 13.20 14.52 Total savings 40.00 44.00 48.40 Operation and maintenance cost 10.00 11.00 12.10 Net savings in cost 30.00 33.00 36.30

Year 4

Year 5

13.31 23.96 15.97 53.24 13.31 39.93

14.64 26.35 17.57 58.56 14.64 43.92

Book value at the beginning Less. Depreciation Book value at the end

72.9 7.29 65.61

65.61 6.561 59.049

..

100 10 90

90 9 81

81 8.1 72.9

104.953 100.000 4.953

475


11-53

(30 min.)

The initial purchase cost of the badminton stadium and the operating receipts and disbursements for the first season of ownership are irrelevant to the present decision. The relevant annual costs which Karthikeyan should take into consideration are: Electricity, (300 × 1 KW) × (130 × 5 hrs.) × ₹6.40 per KWH Labor cost, 130 × ₹6,000 Light bulb cost Repairs and maintenance of lighting system, .057 × ₹7,200,000 Total additional operating expenses

₹1,248,000 780,000 120,000 410,400 ₹2,558,400

Annual revenue from night operations: Years 1 and 2: 130 × ₹33,600 Years 3, 4, and 5: 130 × ₹24,000

₹4,368,000 ₹3,120,000

One-time cash flows: Present value of initial investment Salvage value, year 5 Example of Cash Flow Analysis PV Revenue Expenses Net Flow Factor Year 1 ₹4,368,000 – ₹2,558,400 = ₹1,809,600 .9091 Year 2 4,368,000 – 2,558,400 = 1,809,600 .8264 Year 3 3,120,000 – 2,558,400 = 561,600 .7513 Year 4 3,120,000 – 2,558,400 = 561,600 .6830 Year 5 5,920,000 – 2,558,400 = 3,361,600 .6209 Present value of cash flows * Rounded to a whole number.

₹7,200,000 ₹2,800,000 PV of Cash Flows* ₹1,645,107 1,495,453 421,930 383,573 2,087,217 ₹6,033,280

Since the present value of the annual cash flows is ₹1,166,720 less than the initial investment of ₹7,200,000, the proposed lighting system should not be installed. If significant increases in revenue were predictable, the plan might become attractive to Karthikeyan.

..

476


11-54

(20-25 min.) PV Factor

Old machine: Operating cash Outflows Disposal value Present value New machine: Net cash outlay ($114,000- $24,000) Operating cash outflows Disposal value Present value NPV in favor of old machine

Total Present Value

3.00 .40

$(300,000) 3,200 $(296,800)

1.00

$ (90,000)

3.00 .40

(240,000) 1,600 $(328,400)

Sketch of Cash Flows (in thousands) 0 1 2 3 4 5 (100) (100) (100) (100) (100) 8

(90) (80)

(80)

(80)

(80)

(80) 4

$ 31,600

The old machine minimizes the present value of future costs by $31,600.

..

477


11-55 (30-35 min.) This is one of our favorite problems. The heart of the solution extends through the first paragraph of the response to requirement 2. The remainder is amplification.

Revenue Expenses: Miscellaneous Salaries Net cash flow from operations

Analysis of Cash Flows (in thousand KES) Present Proposed Difference KES100,000 KES7,500 * KES50,000 55,000

105,000 KES (5,000)

6,500 KES1,000

KES6,000

Required investment: Equipment KES KES9,500** Termination pay 17,500 Total KES KES27,000 KES27,000 * 10% × KES75,000,000 = KES7,500,000 commission. ** An acceptable alternative would be to show KES1,500,000 and KES11,000,000 respectively. The incremental investment would still be KES9,500,000. 1.

Present value of KES6,000 per year for 10 years at 10% = KES6,000 × 6.000 Required investment Net present value

KES36,000 27,000 KES19,000

The requirements of the problem focus on the incremental approach. The total project approach could view the problem as choosing the alternative that minimizes the net present value of the future costs: Present: Operating cash outflows, KES5,000 × 6.000 Proposed: Operating cash inflows, KES1,000 × 6.000 Termination pay Equipment Total Difference in favor of proposed investment

KES (30,000) KES 6,000 (17,500) (9,500) KES (21,000) KES 9,000

2.

The minimum amount of annual revenue that the Museum would have to receive to justify the investment would be that amount yielding an incremental net present value of zero. As the initial investment is constant, any change in the incremental net present value is due solely to a change in the amount of revenue. Therefore, the maximum drop in the incremental net present value of KES9,000 equals the maximum drop in the present value of the revenue stream. This implies a maximum drop of KES9,000 ÷ 6 = KES1,500 in annual revenue and a minimum amount of annual revenue of KES7,500 – KES1,500 = KES6,000.

..

478


Let X = Revenue at point of indifference, where net present value is zero NPV = PV of (New annual cash flows - Old annual cash flows) – Required investment 0 = 6.000[(X – 6,500) - (-5,000)] - 27,000 0 = 6.000(X – 6,500 + 5,000) - 27,000 0 = 6.000(X – 1,500) - 27,000 0 = 6.000X - 9,000 - 27,000 6.000X = 36,000 X = 6,000 Part 2 demonstrates sensitivity analysis, where the manager may see the potential impact of the possible errors in the revenue forecasts. Such analysis shows how much of a margin of safety is available. In this case, his “best guess” is revenue of KES7,500 (part 1). A sensitivity analysis shows him that a decline of revenue would have to occur from KES7,500 to KES6,000 before the rate of return on the project would decline to the minimum acceptable level. The following alternative approach to solving requirement 2 is longer, but it may be clearer for many students: If 10% is the minimum acceptable rate of return, the minimum acceptable net present value must be zero, using the 10% rate: NPV = PV of future cash flows - Initial investment Let X = Annual cash inflow Then 0 = 6.000(X) – KES27,000 X = KES27,000 ÷ 6.000 = KES4,500 Present value of KES4,500 per year for 10 years at 10% = KES4,500 × 6.000 KES27,000 Required investment 27,000 Net present value KES 0 Many students will stop at this point, giving an answer of KES4,500. But the problem asks for the minimum amount of revenue, as distinguished from the difference in cash flows. The following analysis shows that revenue can fall to KES6,000. Note also that there can be negative cash flows under both alternatives; the alternative with the least negative cash flow is preferable: Difference in Present Proposed Cash Flows Revenue KES100,000 KES6,000 Expenses 105,000 6,500 Net cash flow from operations KES (5,000) KES (500) KES4,500

..

479


11-56

(30-40 min.)

1. Discount Present Factor Value of Sketch of Cash Flows (thousands) at 18%Cash Flows (thousands) 0 1 2 3

4

End of Year A. Continue with common carriers: 192,500 Kg @ ₹43.20

2.6901

₹(22,371)

(8316)

(8316) (8316) (8316)

B. Purchase truck: Cost of truck 1.0000 (4,000) (4,000) Cash operating costs* 2.6901 (42,730) (15,884) (15,884) (15,884) (15,884) Back-haul revenue, 55 trips @ ₹184,000 2.6901 27,224 10,120 10,120 10,120 10,120 Present value of net cash flows ₹(19,506) Difference in favor of truck ₹ 2,865 *192,500 kgs. ÷ 3,500 kg = 55 trips 55 trips × 3,800 km × ₹76.00 per km = ₹15,884,000 2.

The PV of back-haul revenue must fall by ₹2,865,000 before the incremental net present value equals zero. Therefore, the total present value of back-haul revenue would need to be ₹27,224,000 minus ₹2,865,000, or ₹24,359,000. Let X = number of trips (2.6901) × (₹184,000) × (X) = ₹24,359,000 ₹494,978.4X = ₹24,359,000 X = 49.21 = 50 trips Consequently, 50 trips would have to be guaranteed to yield a non-negative present value—49 would be slightly too few. This means that Sirish would need to be assured that nearly every return trip would be fully loaded. Since Smart-Print anticipates a need for up to 100 loads, it is likely this break-even point will be reached.

3.

..

The greatest difficulty is the reliability of the numbers in a world of uncertainty. Although “the numbers” indicate the truck is a favorable alternative, the following other factors could influence the final decision:

480


(a) If the back-haul agreement can be cancelled by Smart-Print at any time, the truck becomes a riskier investment since the back-haul revenue is needed to make the investment produce a return of 18% or more. (b) What is the outlook for other investments over the life of the truck investment? Does purchasing the truck preclude taking advantage of more favorable opportunities during the 4-year life of the truck? (c) Does the management have the required expertise to run the truck operation efficiently? (d) Will the truck give the company better service than common carriers? (e) How certain are the predicted cash flows? Are shipment figures and operating cost predictions considered to be relatively accurate? 11-57

(15 min.)

1.

Straight-line depreciation: Annual depreciation = £25,000 ÷ 5 = £5,000 per year PV of tax savings = £5,000 × 0.40 × 3.6048 = £7,210

2.

MACRS depreciation:

Year Tax Savings 1 .2000 × £25,000 × 0.4 = £2,000 2 .3200 × 25,000 × 0.4 = 3,200 3 .1920 × 25,000 × 0.4 = 1,920 4 .1152 × 25,000 × 0.4 = 1,152 5 .1152 × 25,000 × 0.4 = 1,152 6 .0576 × 25,000 × 0.4 = 576 Total present value of tax savings

PV factor 0.8929 0.7972 0.7118 0.6355 0.5674 0.5066

Present Value £ 1786 2551 1367 732 654 292 £7,382

You can also use Exhibit 11-7: 0.7381 × £25,000 × 0.4 = £7,382 3.

Immediate write-off: £25,000 × .4 = £10,000

4.

Silver Bricks would prefer immediate write-off. Note that the total tax savings are £10,000 under all three methods. However, only the immediate write-off provides the entire savings immediately. Straight-line depreciation delays receipt of the tax savings the longest, and therefore it has the lowest present value.

11-58

(30 min.)

1.

See Exhibit 11-58 on the following page. There is a net advantage in purchasing because the net present value is slightly positive.

..

481


Exhibit 11-58 (all figures in thousand ₩) Total present values are rounded to whole numbers. Present Value Discount Factors, @ 16%

Total Present Values

Recurring operating cash savings ₩36,000 Income taxes, @ 30% (10,800) After-tax operating cash savings ₩ 25,200 2.2459 ₩56,597 Tax savings due to depreciation: 3-year property @ ₩80,000 18,083* Residual value, all subject to tax because book value will be zero ₩ 12,000 Less: 30% income tax on disposal gain (3,600) Net cash inflow ₩ 8,400 0.6407 5,382 Initial required investment (80,000) Net present value of all cash flows ₩ 62 *The tax savings due to MACRS depreciation can be calculated as follows: (1) (2) (3) (4) (5) (6) Income Tax PV Present Sketch of Cash Flows MACRS Deduction Savings, Factors, Values at End of Year Year Percentage ₩80,000×(2) 0.30×(3) @ 16% (4)×(5) 0 1 2 3 4 1 0.3333 ₩26,664 ₩7,999 0.8621 ₩ 6,896 ₩7,999 2 0.4445 35,560 10,668 0.7432 7,928 ₩10,668 3 0.1481 11,848 3,554 0.6407 2,277 ₩3,554 4 0.0741 5,928 1,778 0.5523 982 ₩1,778 ₩18,083 The tax savings due to MACRS depreciation can also be computed using Exhibit 11-7: 0.7535 × ₩80,000 × 0.30 = ₩18,083.

..

482


11-59 (30 min.) Investment Cash operating savings Annual savings Income taxes @ 44% After-tax effect on cash Present value ($3,360 × 6.7101) PV of tax savings from depreciation: Investment × PV factor (Exhibit 11-7) × Tax rate = $35,000 × .8617 × .44 Overhaul required: Total cost Less income tax savings @ 44% Total after-tax effect Present value ($5,040 × .5835) Residual value: Cash received Book value Gain Income tax @ 44% Total after-tax effect Present value ($4,760 × .4632) Net present value of all cash flows

$(35,000) $6,000 2,640 $ 3,360 22,546 13,270* $ 9,000 3,960 $ 5,040 (2,941) $ 8,500 0 $ 8,500 3,740 $ 4,760 2,205 $ 80

The investment is desirable. *The PV of the tax savings from depreciation can also be calculated as follows: Year Tax Savings 1 .3333 × $35,000 × .44 = $5,133 2 .4445 × 35,000 × .44 = 6,845

..

PV factor .9259 .8573

Present Value $ 4,752 5,868

483


3 .1481 × 35,000 × .44 = 2,281 4 .0741 × 35,000 × .44 = 1,141 Total present value of tax savings

.7938 .7350

1,810 839 $13,270

11-60 (45-60 min.) A.

Investment: $665,000 + (20 × $30,000) = $1,265,000

B.

PV of cash inflows from operations: Monthly rental payments = ($760 × 12) + ($880 × 8) = $16,160 Repair and maintenance = .15 × $16,160 = $2,424 Annual before-tax cash inflow = 12 × ($16,160 – $2,424) = $164,832 Annual after-tax cash inflow = 0.62 × $164,832 = $102,196 Present value of inflows @ 10% = $102,196 × 6.1446 = $627,954

C.

PV of tax savings: Annual depreciation = $1,265,000 ÷ 31.5 = $40,159 Annual tax savings = $40,159 × 0.38 = $15,260 PV of tax savings for 10 years = $15,260 × 6.1446 = $93,767

D.

PV of cash effects of disposal: Cash received Time 0 present value ($1,960,000 × .3855)

E.

..

Net present value at time 0: A. Investment

$1,960,000 $755,580

$(1,265,000)

484


B. PV of operating cash inflows C. PV of income tax savings D. PV of cash effect of disposal: PV of net cash received Net present value

627,954 93,767 755,580 $ 212,301

The net present value is positive, so the NPV model indicates that ELF should purchase the office complex.

..

485


11-61 (15-20 min.) Amounts are in thousands of Japanese yen. 1.

Depreciation expense: (¥400,000 - ¥50,000) ÷ 10 = ¥35,000

2.

Net income: Revenues Less expense: Depreciation Other Operating income Less income tax (60%) Net income

¥330,000 ¥ 35,000 165,000

200,000 ¥130,000 78,000 ¥ 52,000

3.

Cash flow: ¥52,000 + ¥35,000 = ¥87,000 per year or ¥330,000 - ¥165,000 - ¥78,000 = ¥87,000

4.

Payback period: ¥400,000 ÷ ¥87,000 = 4.6 years You might note that 4.6 years is a reasonably long payback period for United States companies, and many companies would be inclined to reject such a project. However, in Japan managers tend to take a longer-run point of view, and a 4.6-year payback period is often acceptable.

5.

Accounting rate of return: ¥52,000 ÷ ¥400,000 = 13% or, if average investment is used: (¥400,000 + ¥50,000) ÷ 2 = ¥225,000 average investment; ¥52,000 ÷ ¥225,000 = 23.1%

6.

NPV: Annual cash flows, ¥87,000 × 5.2161 Salvage value, ¥50,000 × 0.2697 Gross present value

..

¥453,801 13,485 ¥467,286

486


Less: Investment Net present value

..

400,000 ¥ 67,286

487


11-62 (50-60 min.) 1. Table of Cash Flows: End Of Year 2013 2014 2015 2016 2017 2018 2019 2020

Operating Cash Inflow $ 0 100,000 220,000 340,000 460,000 470,000 410,000 150,000

Operating Cash Outflow $199,500 100,000 180,000 260,000 320,000 280,000 200,000 120,000

Net After-Tax Operating Cash Flow $(199,500) 0 24,000** 48,000 84,000 114,000 126,000 18,000

Depreciation Tax Shield $0 11,400* 11,400 11,400 11,400 11,400 11,400 11,400

Net Cash Flow $(199,500) 11,400 35,400 59,400 95,400 125,400 137,400 29,400

* ($199,500 ÷ 7) × .4 = $11,400 ** ($220,000 - $180,000) × (1 - .4) = $24,000; etc. Table of Cumulative Cash Flows: End Of Year 2013 2014 2015 2016

..

Cumulative Net Cash Flow $(199,500) (188,100) (152,700) (93,300)

18% PV Factor 1.000 .8475 .7182 .6086

PV of Net Cash Flow $(199,500) 9,662 25,424 36,151

Cumulative PV of Net Cash Flow $(199,500) (189,838) (164,414) (128,263)

488


2017 2018 2019 2020

2,100 127,500 264,900 294,300

.5158 .4371 .3704 .3139

49,207 54,812 50,893 9,229

(79,056) (24,244) 26,649 35,878

2. The payback time is just under four years as shown by the Cumulative Net Cash Flow column. Because the maximum allowable payback period is 3 years, DGI would not produce the game if the company uses the payback method. 3. The NPV is $35,878. The project has an NPV greater than zero at a discount rate of 18%. Therefore, the company would produce the game if it uses the NPV method. 4. The payback model and NPV model lead to different decisions. In general, the NPV method leads to better decisions than the payback model because the payback model doesn’t measure profitability. Therefore, DGI should probably accept the project and produce the game. A final recommendation would also depend on other factors such as     

..

Potential for proprietary position – such as an important patent that provides a market advantage, Potential for collaborations and outside funding, Need to establish competency in a technology, Potential for spin-off products, and Need to round out a profitable product line.

489


11-63

(20-35 min.)

1.

$25,000 × 5.3349 factor ($17,500 + $3,500) × .4665 factor Total present value Less initial investment: $125,500 + $3,500 Net present value (NPV)

2.

$133,373 9,797 $143,170 129,000 $ 14,170

a.

Annual depreciation is ($125,500 – $17,500) ÷ 8 = $13,500 Increase in expected average annual operating income = $25,000 – $13,500 = $11,500 Initial investment is $129,000 Rate of return is $11,500 ÷ $129,000 = 8.9%

b.

Note the rate of return is not twice the 8.9%. Why? Because the investment at the end of eight years is not zero: Investment at end of 8 years: $17,500 + $3,500 = Initial investment Total "Average" investment: $150,000 ÷ 2 = $75,000 Rate of return is $11,500 ÷ $75,000 = 15.3%

3.

..

$ 21,000 129,000 $150,000

The model in requirement 1 would induce a positive decision. However, the 8.9% accounting rate of return based on an initial investment might induce a negative decision because it is less than 10%. An administrator’s reluctance to buy would be understandable if there is no reasonable consistency between the decision model and the performance evaluation model. If the decisions are supposed to be based on DCF models and the performance is evaluated on accrual accounting models, the latter tend to be persuasive.

490


11-64 1.

(20 min.)

Investment = $2,200,000 + $1,480,000 = $3,680,000 Annual cash inflow = 300 skiers × 40 days × $65/skier-day = $780,000 Annual cash outflow = (200 days × $500/day)+($9/skier-day × 300 × 40) = $208,000 PV of cash flows @ 14% = ($780,000 - $208,000) × 6.6231 = $3,788,413 NPV = $3,788,413 - $3,680,000 = $108,413 The new lift will create value of $108,413, so it is a profitable investment.

2.

After-tax cash flows = $572,000 × .6 = $343,200 PV of after-tax cash flows @ 8% = $343,200 × 9.8181 = $3,369,572 PV of tax savings = $3,680,000 × .4 × .7059 (from Exhibit 11-7) = $1,039,085 NPV after-tax = $3,369,572 + $1,039,085 - $3,680,000 = $728,657 The investment in the lift is more profitable on an after-tax basis than on a pretax basis.

3.

Subjective factors that might affect this decision include:  Profits on sales of food, rental of equipment, and other items purchased by the additional skiers.  More satisfied customers because of less crowding on the days that the additional lift does not result in additional skiers being attracted to Deer Valley.  Additional skiers may not be as many as estimated if the weather is poor.

..

491


11-65 (30 min.) Investment CHF (84,000) Net cash operating inflows Annual savings (65,000 – 47,000) CHF18,000 Income taxes @ 30% 5,400 After-tax effect on cash CHF12,600 Present value (CHF12,600 × 5.2161) 65,723 PV of tax savings from depreciation: CHF8,000 × .30 × 5.2161 12,519 Residual value of new machine: Cash received CHF 4,000 Book value 4,000 Loss CHF 0 Income tax savings @ 30% 0 Total after-tax cash inflow effect CHF 4,000 Present value (CHF4,000 × .2697) 1,079 Disposal of old machine: Cash received CHF 9,000 CHF 9,000 Book value 31,000 Loss CHF22,000 Income tax savings @ 30% 6,600 Total after-tax cash inflow effect CHF15,600 Present value (CHF15,600 × 1.000) 15,600 Net present value of all cash flows CHF 10,921 The investment is desirable. 11-66

(40-50 min.)

1. See Exhibit 11-66 on the following page for the solution to requirement. 2. The greatest difficulty is the reliability of the numbers in a world of uncertainty. Although "the numbers" indicate the truck is a favorable alternative, the following other factors could influence the final decision: (a) If the back-haul agreement can be canceled by Retro at any time, the truck becomes a more risky investment since the back-haul revenue is needed to make the investment produce an after-tax return of 20% or more. (b) What is the outlook for other investments over the life of the truck investment? Does purchasing the truck preclude taking advantage of more favorable opportunities during the 5-year life of the truck? (c) Does the management have the required expertise to run the truck operation efficiently? (d) Will the truck give the company better service than common carriers? (e) How certain are the predicted cash flows? Are shipment figures and operating cost predictions considered to be relatively accurate?

..

492


EXHIBIT 11-66 Alternative 1: Continue w/ common carrier 500,000 lbs @ 26¢ $130,000 Inc. tax savings @ 40% (52,000) After-tax (Year 0) $ 78,000 Adjust $78,000 for 10% inflation $78,000 × 1.1; × 1.12; × 1.13; × 1.14; × 1.15 Present value factor @ 20% Present value Alternative 2: Purchase truck Initial cash investment Depreciation deductions: Year % Deduction Tax Savings 1 20 $ 10,000 $4,000 2 32 16,000 6,400 3 19.2 9,600 3,840 4-5 11.52 5,760 2,304 6 5.76 2,880 1,152 Back-haul revenue* 50 trips** @ $2,400 $120,000 Income tax @ 40% (48,000) After-tax $ 72,000 Cash operating costs: 250,000 miles† @ 90¢ $225,000 Income tax savings (90,000) $135,000 Adjust $135,000 for inflation after Year 1 $135,000 × 1.1; × 1.12; × 1.13; × 1.14 Total cash flow Present value factors @ 20% Present value PV difference in favor of purchasing truck *Not subject to inflation due to 5-year agreement.

..

Total Present Value Year 0

Year 1

$(302,679)

(85,800) .8333 (71,497)

After-Tax Cash Flows (in dollars) Year 2 Year 3 Year 4

Year 5

Year 6

(94,380) (103,818) (114,200) (125,620) .6944 .5787 .4823 .4019 (65,537) (60,079) (55,079) (50,487)

(50,000) 4,000 6,400 3,840 2,304

2,304 1,152

72,000

(298,497) $ 4,182

(50,000) 1.0000 (50,000)

72,000

72,000

72,000

72,000

(135,000) (148,500) (163,350) (179,685) (197,654) (59,000) (70,100) (87,510) (105,381) (123,350) 1,152 .8333 .6944 .5787 .4823 .4019 .3349 (49,165) (48,677) (50,642) (50,825) (49,574) 386

**500,000 lbs ÷ 10,000 lbs/trip = 50 trips †50 trips @ 5,000 round-trip miles = 250,000 miles

493


11-67

(20-30 min.)

1 & 2. See Exhibit 11-67 for the solution to requirements 1 and 2. 3.

The correct analysis of inflation can affect decisions. Using a required rate of return that includes an inflation element but failing to adjust cash inflows for inflation will understate the present value, causing possible rejection of desirable projects.

11-68

(25-30 min.) Amounts are in Estonian kroons (EEK).

1.

Annual cash savings (EEK 520,000 × 5) Additional operating expenses Net annual savings

EEK 2,600,000 (1,800,000) EEK 800,000

Investment

EEK 2,800,000

NPV = (EEK 800,000 × 4.9676*) - EEK 2,800,000 = EEK 3,974,080 - EEK 2,800,000 = EEK 1,174,080 *From Table 2, 12% column, 8-year row. The system should be purchased because the NPV is positive.

2.

Pessimistic: Annual savings = EEK 800,000 - EEK 520,000 = EEK 280,000 Economic life = 5 years NPV = (EEK 280,000 × 3.6048) - EEK 2,800,000 =EEK 1,009,344 - EEK 2,800,000 = EEK (1,790,656) Optimistic: Annual savings = EEK 800,000 + EEK 520,000 = EEK 1,320,000 Economic life = 10 years NPV = (EEK 1,320,000 × 5.6502) - EEK 2,800,000 = EEK 7,458,264 - EEK 2,800,000 = EEK 4,658,264 Most likely: NPV = EEK 1,174,080 (from requirement 1) This analysis shows that predictions of savings and economic life can greatly affect the decision. Although the expected NPV is EEK 1,174,080, it is possible that the realized NPV might be as low as a negative EEK 1,790,656. It might be worthwhile to gather more information about the savings and economic life before making the decision.

..

494


EXHIBIT 11-67

Description 1. Per Problem Instructions (But that is an incorrect analysis, which includes an inflation element in the discount rate but does not adjust the predicted cash flows for inflation.) Cash operating savings New machine, investment Net present value

14% PV Factor

Total Present Value

3.4331 1.0000

$ 3,433 (3,500) $ (67)

.8772 .7695 .6750 .5921 .5194

$ 930 865 804 748 695 $ 4,042 (3,500) $ 542

2020

Sketch of Relevant Cash Flows (in dollars) 2021 2022 2023 2024

1,000

1,000

1,000

1,000

2025

1,000

(3,500)

2. Correct Analysis: (Includes an inflation element in both the discount rate and the predicted cash flows.) Cash operating savings:

New machine, investment Net present value

1.0000

1,060* 1,124 1,191 1,263 1,339 (3,500)

*1,000 × 1.06, then 1,000 × (1.06)2, then 1,000 × (1.06)3, etc.

..

495


3.

Investment in new technology often has many effects that are difficult to quantify. A special report in Business Week reported that most companies do not provide a quantitative cost justification for the purchase of computers. However, the article goes on to point out that analyses such as NPV are being increasingly demanded by top management to justify investment in new technology. The company should be concerned with the amount of investment specified. The system can be purchased for EEK 2,800,000, but might additional costs be incurred in implementing the system? Will the quality of design be improved by the new system? Or might the system be incapable of meeting current standards? Maybe most important, the analysis is based on the implementation of CAD only. Is there any chance that the CAM portion will be used? If so, the purchase has more value than shown in the analysis of CAD only.

11-69 (30-40 min.) This case focuses on the appropriate baseline for NPV analysis for an investment in a high technology production system. It highlights the possible loss of competitive position if the company does not undertake the investment. The potential magnitude of errors from omission of some factors in an NPV analysis is shown. 1.

This is a straightforward NPV analysis: End Present Value of @ 12% Year from Table 1 2011* 1.0000 2012 .8929 2013 .7972 2014 .7118 2015 .6355 2016 .5674 2017 .5066 Total * Or beginning of 2012.

Differential Net Cash Flow $(6,000,000) (400,000) 1,400,000 1,400,000 1,400,000 1,400,000 1,400,000

Present Value of Differential Cash Flow $(6,000,000) (357,160) 1,116,080 996,520 889,700 794,360 709,240 $(1,851,260)

The investment in the CIM has a negative NPV of more than $1,850,000. It appears that it would be a mistake to invest.

..

496


2.

An additional advantage of the CIM must be recognized in this analysis. In the absence of investment in the CIM, some of the existing contribution margin will be lost each year. Investment avoids this loss, so the amount of the contribution margin that would have been lost is in essence a savings from investment in the CIM. The current market share of 40% and sales of $12 million implies that each 1% of the market is worth sales of $12,000,000 ÷ 40 = $300,000. Current sales are $12,000,000 and variable costs are $4,000,000 + $2,000,000 = $6,000,000, making the contribution margin percentage 50%. Therefore, for each $1 of lost sales, Nashville Tool loses $.50 in contribution margin. The loss of $300,000 in sales results in a loss of $150,000 in contribution margin. Therefore, the potential lost contribution margin each year is: Year 2012 2013 2014 2015 2016 2017

Lost Market Share 3% 6% 9% 12% 15% 18%

Lost Sales $ 900,000 1,800,000 2,700,000 3,600,000 4,500,000 5,400,000

Lost Contribution Margin $ 450,000 900,000 1,350,000 1,800,000 2,250,000 2,700,000

Combining the savings from variable costs with the savings in contribution margin, the NPV becomes a positive $4,017,325, computed as follows: Present Value Invest. @ 12% from and Cost Table 1 Savings* 1.0000 $(6,000,000) .8929 (400,000) .7972 1,400,000 .7118 1,400,000

Year 2011 2012 2013 2014 2015 2016 2017 Total

.6355 .5674 .5066

1,400,000 1,400,000 1,400,000

Savings in Contribution Margin $ 450,000 900,000 1,350,000

Total Diff. Cash Flow $(6,000,000) 50,000 2,300,000 2,750,000

PV of Diff. Cash Flow $(6,000,000) 44,645 1,833,560 1,957,450

1,800,000 2,250,000 2,700,000

3,200,000 3,650,000 4,100,000

2,033,600 2,071,010 2,077,060 $ 4,017,325

*These cost savings are the same as those in part 1 because even though the variable cost of goods sold declines with the decrease in sales, this only occurs if the CIM investment is not made. If the investment is made, market share remains constant at 40% ($12,000,000 in revenues) every year, and thus the cost savings are also a constant $1,400,000 per year.

The picture has changed radically from that in requirement 1. Avoiding the lost contribution margin has made the CIM a very desirable investment. ..

497


3.

To the Board of Directors: I recommend that Nashville Tool invest in the new CIM system. I have made two net present value analyses, the first one showing a negative NPV of more than $1,850,000 and the second showing a positive NPV of over $4 million. Let me explain why the second analysis is better. The first analysis compares revenues and costs under the CIM to those that would be incurred if operations continue exactly as they did in 2011. However, if we do not invest in CIM, operations will not continue the way they are today. Many of our competitors are investing in technologically sophisticated production systems, and if we do not invest, they will have advantages over us in quality of products, response to design changes desired by our customers, and flexibility of delivery schedules. Investment in the CIM will not only save variable costs of production, it will allow us to maintain our market share. The second analysis uses the correct baseline for comparisons. It compares the costs and revenues with the CIM to those we expect if we do not invest. It includes consideration of the lost sales, and therefore lost contribution margin, that we would experience if our competitors gain a competitive advantage by investing in CIM while we do not. If we do not upgrade to CIM or some similar system in the next six years, we risk losing nearly half our business. This risk is much greater than that of not achieving all the cost savings projected for the CIM. In addition to the items included in my analysis, there are other potential benefits to investing in the CIM. First, it encourages our employees to think about the production process and places where we might eliminate or reduce non-value-added activities. It also introduces technologically sophisticated operations so that future expansion of similar activities may be easier. In summary, cost savings alone do not justify investment in the CIM. But cost savings are not the only advantage of investment. When we add the extra contribution margins from business we will maintain only if we invest in the CIM, plus other qualitative advantages, the investment is certainly desirable.

..

498


11-70

(30-40 min.)

This problem includes a complex analysis of relevant costs in addition to its focus on an investment decision. This solution will first identify the relevant costs in four categories: 1. 2. 3. 4.

Initial investment Current annual quality control costs Annual quality control costs with new process Forgone profits if quality is not improved

Initial investment: Worker training X-ray machine Total investment

$950,000 250,000

Current annual quality control costs: Inspection cost Correction of defects (1,500 × $85) Refunds to customers (500 × $210) Total current quality control costs

$ 30,000 127,500 105,000

Annual quality control costs with new process: Inspection cost ($30,000 + $60,000) Correction of defects (450 × $50) Refunds to customers (50 × $315) Total new quality control costs

$ 90,000 22,500 15,750

$1,200,000

$ 262,500

128,250

Net savings in quality control costs

$ 134,250

Difference in contribution margin if quality is not improved: 2014 2015 2016 2017

$

0 350,000 700,000 1,050,000

(5,000 × $70) (10,000 × $70) (15,000 × $70)

Therefore, the total annual cash flows from the change in the quality control process are: Differences Net Savings in Total Net Cash in Quality Contribution Flow from Control Costs Margin Operations 2014 2015 2016 2017 ..

$134,250 134,250 134,250 134,250

$

0 350,000 700,000 1,050,000

$ 134,250 484,250 834,250 1,184,250 499


The net present value of the investment in the new quality control is positive, so invest: PV of $1 @ 20% Initial investment Annual cash flows

Net present value

..

1.0000 .8333 .6944 .5787 .4823

Total Present Value

Sketch of annual cash flows |–––––––|–––––––|–––––––|–––––––| 0 1 2 3 4

$(1,200,000) 111,871 <––––134,250 336,263 <–––––––––––484,250 482,780 <–––––––––––––––––––834,250 571,164 <––––––––––––––––––––––––1,184,250 $ 302,078

500


11-71

(60-90 min.)

This is a complex problem because it requires comparing three alternatives. It reviews Chapter 6 as well as covering several of the topics of Chapter 11. The following answer uses the total project approach. The total net future cash outflows are shown for each alternative. 1. Alternative A: Continue to manufacture the parts with the current tools. Annual cash outlays Variable cost, $92 × 8,000 $(736,000) Fixed cost, 1/3 × $45 × 8,000 × .6 (72,000) Tax savings, .4 × ($736,000 + $72,000) 323,200 After-tax annual cost $(484,800) Present value, 3.6048 × $484,800 $(1,747,607) PV of remaining tax savings on MACRS: 11.52% × $2,000,000 × .4 × .8929 82,290 5.76% × $2,000,000 × .4 × .7972 36,735 Total present value of costs, Alternative A $(1,628,582) Alternative B: Purchase from outside supplier Annual cash outlays Purchase cost, $110 × 8,000 Tax savings, $880,000 × .4 After-tax annual cost Present value, $528,000 × 3.6048 Sale of old equipment: Sales price Book value [(11.52% + 5.76%) × $2,000,000] Gain Taxes @ 40% Total after-tax effect ($400,000 - $21,760) Total present value of costs, Alternative B

..

$(880,000) 352,000 $(528,000) $(1,903,334) $ 400,000 345,600 $ 54,400 (21,760) 378,240 $(1,525,094)

501


Alternative C: Purchase new tools Investment Annual cash outlays Variable cost, $73 × 8,000 $(584,000) Fixed cost (same as A) (72,000) Tax savings, .4 × ($584,000 + $72,000) 262,400 After-tax annual cost $(393,600) Present value, $393,600 × 3.6048 Tax savings on new equipment* Effect of disposal of new equipment Sales price $ 500,000 Book value 0 Gain $500,000 Taxes @ 40% 200,000 Total after-tax effect $ 300,000 Present value, $300,000 × .5674 Effect of disposal of old equipment (see Alternative B) Total present value of costs, Alternative C

$(1,800,000)

(1,418,849) 579,217

170,220 378,240 $(2,091,172)

* Using the MACRS schedule for tax depreciation, the depreciation rate for each year of a 3year asset's life is shown in Exhibit 11-6: Depreciation Tax Year Rate Savings 1 33.33% .3333 × $1,800,000 × .40 = $239,976 2 44.45% .4445 × 1,800,000 × .40 = 320,040 3 14.81% .1481 × 1,800,000 × .40 = 106,632 4 7.41% .0741 × 1,800,000 × .40 = 53,352 Total present value of tax savings

PV Factor .8929 .7972 .7118 .6355

Present Value $214,275 255,136 75,901 33,905 $579,217

Using Exhibit 11-7, we get .8044 × $1,800,000 × .4 = $579,168, which differs from $579,217 by a $49 rounding error. The alternative with the lowest present value of cost is Alternative B, purchasing from the outside supplier. 2.

Among the major factors are (1) the range of expected volume (both large increases and decreases in volume make the purchase of the parts relatively less desirable), (2) the reliability of the outside supplier, (3) possible changes in material, labor, and overhead prices, (4) the possibility that the outside supplier can raise prices before the end of five years, (5) obsolescence of the products and equipment, and (6) alternate uses of available capacity (alternative uses make Alternative B relatively more desirable).

..

502


11-72 1.

2. 3. 4.

(30 min.) From Note 1, Nike uses the straight-line method for reporting to shareholders. Nike probably uses MACRS, an accelerated depreciation method, for reporting to tax authorities because that will maximize the present value of the tax savings from depreciation. From Note 3, the original cost of Nike’s machinery and equipment is $2,115.0 million. If Nike invests an average of about $400 million a year, the average useful life would be just over 5 years: $2,115.0 ÷ 400 = 5.2875 years. Let CF be the minimum average annual pre-tax cash inflow: $432,000,000 = CF × 3.4331 CF = $125,833,790 a) Payback period = $432,000,000 ÷ $125,833,790 = 3.43 years b) Accounting rate of return: Income = $125,833,790 – ($432,000,000 ÷ 5) = $39,433,790 Average investment = $432,000,000 ÷ 2 = $216,000,000 Accounting rate of return = $39,433,790 ÷ $216,000,000 = 18.3%

11-73 (20-30 min.) For the solution to this Excel Application Exercise, follow the step-bystep instructions provided in the textbook chapter. 1.

NPV = ($16,000 × 3.2743) - $60,000 = $(7,611)

2.

Payback period = $60,000 ÷ 16,000 = 3.75 years

3.

The NPV is negative, so do not invest in the machine.

11-74

(20 min.) The purpose of this exercise is to see how financial analyses and behavioral and ethical issues interact in decision making. We first present the NPV analysis that should form the basis of Rossi’s meeting with Sharma. Then we discuss other items that are likely to surface in the meeting. To gain a 16% rate of return, the net present value at 16% must be positive. For Delhi Chicken Soup, optimistic, expected, and pessimistic present values of predicted cash inflows, assuming cash flows at the end of each year, are (in thousands of dollars): (1)

Year

PV of $1 at 16%

(2) (3) Optimistic Cash Present Flow Value (1) × (2)

Expected Cash Present Flow Value (1) × (4)

(6) (7) Pessimistic Cash Present Flow Value (1) × (6)

1 2

.8621 .7432

800 1,800

600 1,200

400 600

..

690 1,338

(4)

(5)

517 892

345 446 503


3 4 5 Total

.6407 .5523 .4761

2,500 4,000 5,000

1,602 2,209 2,381 8,220

1,500 2,200 2,600

961 1,215 1,238 4,823

500 400 200

320 221 95 1,427

The investment and salvage values do not depend on the optimistic and pessimistic forecasts: Investment (1.0000 × $6,000,000) Salvage value of facilities (.4761 × $1,200,000) Total

$(6,000,000) 571,320 $(5,428,680)

Therefore, net present values are: Optimistic ($8,220,000 - $5,428,680) Expected ($4,823,000 - $5,428,680) Pessimistic ($1,427,000 - $5,428,680)

$ 2,791,320 $ (605,680) $(4,001,680)

If you believe the expected amounts, the product has a negative present value and should not be launched. Sharma might raise some of the following issues supporting the project:     

The required rate of return is less than 16%. The optimistic scenario is more likely than the pessimistic scenario, making the expected cash flows more than those listed. The cash flow predictions for either the optimistic or pessimistic scenarios (or both) are understated. The contribution margin is 58% rather than 50%. The investment is less than $6 million.

For example, he might maintain that the required rate of return for a project of this risk should be 12% instead of 16%. Then the product’s expected net present value would be $5,434,000 - $6,000,000 + (.5674 x $1,200,000) = $114,088: (1)

Year 1 2 3 4 5 Total ..

PV of $1 at 12%

(2) (3) Optimistic Cash Present Flow Value (1) × (2)

Expected Cash Present Flow Value (1) × (4)

(6) (7) Pessimistic Cash Present Flow Value (1) × (6)

.8929 .7972 .7118 .6355 .5674

800 1,800 2,500 4,000 5,000

600 1,200 1,500 2,200 2,600

400 600 500 400 200

714 1,435 1,780 2,542 2,837 9,308

(4)

(5)

536 957 1,068 1,398 1,475 5,434

357 478 356 254 113 1,558 504


Or he might maintain that each expected cash flow should be $200,000 higher, making the net present value $4,823,000 + ($200,000 × 3.2743) - $5,428,680 = $49,180. Or, if the contribution margin were 58% rather than 50%, the net present value would be [$4,823,000 × (58/50)] - $5,428,680 = $166,000. Finally, if the investment is less than $6,000,000 by at least $605,680, the net present value would be positive. Sharma could use some combination of these changes to make the net present value of the product positive. The ethical issues in this exercise can be revealing. If Rossi believes her information is accurate, it would be unethical to produce biased numbers just to satisfy Sharma. Among the ethical requirements for management accountants are to “communicate information fairly and objectively,” “disclose fully all relevant information,” and “prepare concise and clear reports and recommendations after appropriate analyses of relevant and reliable information.” These standards would be violated if Rossi were to change her analysis just to satisfy Sharma. Therefore, Rossi should report numbers that she believes are accurate. This may upset her supervisor, Sharma, and that may create problems for Rossi. Nevertheless, using biased information to justify a manager’s pet projects can create more serious problems. 11-75 (35-50 min.) NOTE TO INSTRUCTOR: This solution is based on the web site as it was in late 2012. Be sure to examine the current web site before assigning this problem, as the information there may have changed. 1.

Carnival Corporation operates 100 cruise ships under the following lines: Carnival Cruise Lines, Holland America Line, Princess Cruises and Seabourn in North America; P&O Cruises, and Cunard Line in the United Kingdom; AIDA in Germany; Costa Cruises in Southern Europe; Iberocruceros in Spain; and P&O Cruises in Australia. At the time this solution was prepared, Carnival planned to add 9 new cruise ships by March 2016, reflecting plans for significant growth in business.

2.

From Carnival’s 2011 Annual Report, its capacity (defined as available berths) has increased each of the last five years: 2007: 2008: 2009: 2010: 2011:

Passengers Carried 7,672,000 8,183,000 8,519,000 9,147,000 9,559,000

Passenger Capacity (# of berths) 158,352 169,040 180,746 191,464 195,872

The passengers carried (capacity used) and capacity increased by about 24% from 2007 to 2011. 3.

Carnival continues to expand its fleet, though some of the new vessels will replace older ships. It has a total of 10 new vessels on order. Three of the ships will be delivered in 2012 and the remaining 7 will be delivered at a rate of 2 or 3 per year.

4.

In 2011, Carnival invested $2.7 billion in property and equipment, and Carnival used about $1.1 billion of cash for financing activities, paying off short-term and long-term debt, paying dividends, and purchasing treasury stock during the year. About $3.8 billion of cash was generated by operating activities.

..

505


CHAPTER 12 COVERAGE OF LEARNING OBJECTIVES

LEARNING OBJECTIVE LO1: Describe the general framework for cost allocation. LO2: Allocate the variable and fixed costs of service departments to other organizational units. LO3: Use the direct and step-down methods to allocate service department costs to user departments. LO4: Allocate costs from producing departments to products or services using the traditional and ABC approaches. LO5: Allocate costs associated with customer actions to customers. LO6: Allocate the central corporate costs of an organization. LO7: Allocate joint costs to products using the physical-units and relativesales-value methods

..

CASES, NIKE 10K, EXCEL, FUNDAMENTAL ADDITIONAL COLLAB. & ASSIGNMENT ASSIGNMENT INTERNET MATERIAL MATERIAL EXERCISES A1, B1, B3 37, 39, 39, 44, 44 A1, B1, B3

5, 7, 25, 27, 28, 29, 37, 38, 38, 39, 39, 44, 44

54, 56

A1, B1, B3

8, 31, 32, 32, 37, 54 40, 40, 45, 45, 46, 46

B3

16, 30, 30, 31, 37, 40, 40, 47, 48, 48

51, 55

A2, B2, B3

19, 33, 33, 37, 41, 41, 42, 43

51, 53, 55

B3

3, 21, 26, 30, 49, 52 49

A3, B3

34, 35, 35, 36, 36, 50

506


CHAPTER 12 Cost Allocation 12-A1 (30-50 min.) The numerical answers for requirements 1 and 2 are in Exhibit 12A1. Most students will favor the direct method because the final allocations are not affected significantly. 3. The allocation types from Exhibit 12-1 are 3 and 4. Each of the allocations to the engineering and cafeteria departments from general factory administration are from one service department to another service department (type 3). Each of the allocations to the three producing departments from the three service departments are type 4 allocations. 12-A2 (40-50 min.) 1. Sales Cost Of Sales Gross Profit Margin Gross Profit Margin Percentage

A $5,000 4,500 $ 500

Product B $6,000 4,800 $1,200

C $25,000 15,000 $10,000

Total $36,000 24,300 $11,700

10.0%

20.0%

40.0%

32.5%

Product C is the most profitable based on gross margin and gross margin percentage. 2. A Customer Type 1 Sales $500 Gross Margin 50 Gross Margin Percentage of Sales Customer Type 2 Sales $4,500 Gross Margin 450 Gross Margin Percentage of Sales

..

Product B

C

Total

$1,000 200

$13,000 5,200

$14,500 5,450 37.6%

$5,000 1,000

$12,000 4,800

$21,500 6,250 29.1%

507


EXHIBIT 12-A1 Total labor hours Employees Engineering hours

General Factory Total Administration Cafeteria 1,296,000 36,000 780 80,000 -

Cost Drivers Method 1, Direct Method Total department overhead before allocation General factory administration Cafeteria Engineering Totals Method 2, Step-Down Method Total department overhead before allocation General factory administration Cafeteria Engineering Totals

Engineering 120,000 60 -

Machining 300,000 120 50,000

Assembly 720,000 540 20,000

Finishing and Painting 120,000 60 10,000

Total Engineering Finishing Labor Hours Employees Hours Machining Assembly & Painting $950,000 (950,000)

$150,000 $1,600,000 - - - - - - - - - - Not Given- - - - - - - - - - - $ 250,000 1 $ 600,000 $100,000 2 (150,000) 25,000 112,500 12,500 3 (1,600,000) 1,000,000 400,000 200,000 $1,275,000 $1,112,500 $312,500

$950,000 (950,000)

$150,000 $1,600,000 - -- - - - - - - Not Given- - - - - - - - - - 26,389 4 87,963 $219,907 $527,778 $ 87,963 5 (176,389) 13,568 27,137 122,115 13,568 6 (1,701,531) 1,063,457 425,383 212,691 $1,310,501 $1,075,276 $314,222

1 300 + 720 + 120 = 1,140; (300÷1,140) × 950,000 = 250,000; (720÷1,140) × 950,000 = 600,000; etc. 2 120 + 540 + 60 = 720; (120÷720) × 150,000 = 25,000; (540÷720) × 150,000 = 112,500; etc. 3 50 + 20 + 10 = 80; 50/80 × 1,600,000 = 1,000,000; 20/80 × 1,600,000 = 400,000; etc. 4 36+120+300+720+120 =1,296 ; (36÷1,296)×950,000 =26,389 ; (120÷1,296)×950,000 =87,963 ; (300÷1,296)×950,000 =219,907 ; etc. 5 60+120+540+60 =780 ; (60÷780)×176,389 =13,568 ; (120÷780)×176,389 =27,137 ; (540÷780)×176,389 =122,115 ; etc. 6 5+20+10 = 80; 50/80 × 1,701,531 = 1,063,457; 20/80 × 1,701,531 = 425,383; etc.

..

508


3.

Cost to Serve Cost to Serve per Visit

$12,000 $12,000 ÷ (4 + 16) = $600

Customer Type 1 Sales Manager Visits Cost to Serve Cost to Serve Percentage of Sales Customer Type 2 Sales Manager Visits Cost to Serve Cost to Serve Percentage of Sales 4.

Customer Type 1 Sales Operating Income Operating Income Percent of Sales Customer Type 2 Sales Operating Income Operating Income Percent of Sales

5. Profitability Measure Gross Margin Gross Margin Percentage of Sales Operating Income Operating Income Percentage of Sales

..

$14,500 4 $2,400 16.6% $21,500 16 $9,600 44.7% $14,500 $3,050 21.0% $21,500 ($3,350) (15.6%) Most Profitable Customer Type 2 1 1 1

509


12-A3

(20-30 min.)

Note that total joint costs are $11 × 1,000,000 + $4 × 1,000,000 = $15,000,000. 1.

A B 2.

Physical units method: Pounds 200,000 800,000 1,000,000

Weighting (200 ÷ 1,000) × $15,000,000 (800 ÷ 1,000) × $15,000,000

Relative sales value method:

Relative Sales Value at Split-off A $30.00 × 200,000 = $ 6,000,000 B $15.00 × 800,000 = 12,000,000 $18,000,000 3.

..

Weighting (6 ÷ 18) × $15,000,000 (12 ÷ 18) × $15,000,000

Allocation of Joint Costs $ 5,000,000 10,000,000 $15,000,000

The sales value of B at the split-off point must be approximated: Sales value of B

A B

Allocation of Joint Costs $ 3,000,000 12,000,000 $15,000,000

= Final sales value - Separable costs = ($18.75 × 800,000) - [$200,000 + ($1 × 800,000)] = $15,000,000 - $1,000,000 = $14,000,000

Relative Sales Value at Split-off $ 6,000,000 14,000,000 $20,000,000

Weighting (6 ÷ 20) × $15,000,000 (14 ÷ 20) × $15,000,000

Allocation of Joint Costs $ 4,500,000 10,500,000 $15,000,000

510


12-B1 (30-40 min.) 1.

Direct method: Personnel

Direct departmental costs before allocation Personnel Administrative Total costs after allocation

$ 70,000 (70,000)

Administrative

Residential

Commercial

$ 100,000

$240,000 28,000 37,500 $305,500

$400,000 42,000 62,500 $504,500

(100,000)

Calculations: 12 + 18 = 30 (12 ÷ 30) × $70,000 = $28,000 (18 ÷ 30) × $70,000 = $42,000 240,000 + 400,000 = 640,000 (240,000 ÷ 640,000) × $100,000 = $37,500 (400,000 ÷ 640,000) × $100,000 = $62,500 2.

Step-down method:

Direct departmental cost before allocation Personnel Administrative Total cost after allocation

Personnel

Administrative

Residential

Commercial

$ 70,000 (70,000)

$ 100,000 10,000 $(110,000)

$240,000 24,000 41,250 $305,250

$400,000 36,000 68,750 $504,750

Calculations: 5 + 12 + 18 = 35 (5 ÷ 35) × $70,000 = $10,000 (12 ÷ 35) × $70,000 = $24,000 (18 ÷ 35) × $70,000 = $36,000 240,000 + 400,000 = 640,000 (240,000 ÷ 640,000) × $110,000 = $41,250 (400,000 ÷ 640,000) × $110,000 = $68,750 3.

(a) Residential: (b) Commercial:

4.

Allocations from the personnel to the administrative departments are type 3 allocations. Allocations from the administrative to the residential and commercial operating departments are type 4 allocations.

..

$305,250 ÷ 24,000 hours = $12.72 per direct-labor hr $504,750 ÷ 9,970,000 sq. ft. = $.05 per square foot

511


12-B2 (40-50 min.) 1. Sales Cost Of Sales Gross Profit Margin Gross Profit Margin Percentage

Alpha $4,000 2,000 $2,000

Product Beta $8,000 2,000 $6,000

Gamma $20,000 14,000 $ 6,000

Total $32,000 18,000 $14,000

50.0%

75.0%

30.0%

43.8%

Beta has the largest gross profit margin percentage and Bets and Gamma have the largest dollar amount of gross profit. 2. Customer Type 1 Sales Gross Margin Gross Margin Percentage of Sales Customer Type 2 Sales Gross Margin Gross Margin Percentage of Sales 3.

Cost to Serve Cost to Serve per Visit

Alpha

Product Beta

Gamma

Total

$2,000 1,000

$5,000 3,750

$1,000 300

$8,000 5,050 63.1%

$2,000 1,000

$3,000 2,250

$19,000 5,700

$24,000 8,950 37.3%

$10,000 $10,000 ÷ (6 + 4) = $1,000

Customer Type 1 Sales Manager Visits Cost to Serve Cost to Serve Percentage of Sales Customer Type 2 Sales Manager Visits Cost to Serve Cost to Serve Percentage of Sales

..

$8,000 6 $6,000 75.0% $24,000 4 $4,000 16.7%

512


4.

Customer Type 1 Sales Operating Income Operating Income Percent of Sales Customer Type 2 Sales Operating Income Operating Income Percent of Sales

$24,000 $4,950 20.6%

Profitability Measure Gross Margin Gross Margin Percentage of Sales Operating Income Operating Income Percentage of Sales

Most Profitable Customer Type 2 1 2 2

$8,000 ($950) (11.9)%

5.

12-B3 (15 min.) The joint costs include the purchase cost of $1,000,000 × $.80 = $800,000 and the processing cost before the split-off point of $.40 × 1,000,000 = $400,000, a total of $1,200,000. 1. Oat flour Oat bran 2. Oat flour Oat bran

Pounds 800,000 200,000 1,000,000 Relative Sales Value at Split-off* $1,200,000 400,000 $1,600,000

Weighting 800/1,000 × $1,200,000 200/1,000 × $1,200,000

Allocation of Joint Costs $ 960,000 240,000 $1,200,000

Allocation of Weighting Joint Costs 1,200/1,600 × $1,200,000 $ 900,000 400/1,600 × $1,200,000 300,000 $1,200,000

*$1.50 × 800,000 and $2.00 × 200,000 3.

Estimated value of oat flour at split-off: Sales value of oat flakes, $2.90 × 800,000 pounds - Processing cost after split-off point, ($.60 × 800,000 pounds) + $240,000

Oat flakes Oat bran

..

Relative Sales Value at Split-off $1,600,000 400,000 $2,000,000

$2,320,000 720,000 $1,600,000

Weighting 1,600/2,000 × $1,200,000 400/2,000 × $1,200,000

Allocation of Joint Costs $ 960,000 240,000 $1,200,000

513


12-1 For most companies, accountants can directly trace less than 60% of operating costs to products, services, and customers. For the rest of a company’s costs, accountants must either apply cost-allocation methods or leave costs unallocated. Most managers prefer to allocate these indirect costs. 12-2 Exhibit 12-1 shows the ten types of cost assignments. 1. 2. 3. 4. 5. 6.

Directly traced costs to departments Indirect costs allocated to departments Service department costs allocated to other service departments Service department costs allocated to producing departments Producing department costs allocated to other producing departments Directly traced costs to producing departments that an organization can also trace directly to products and services 7. Producing department costs that an organization allocates to products or services 8. Directly traced costs to service departments that an organization can also trace directly to customers 9. Service department costs allocated to customers 10. Product/service costs assigned to customers 12-3 Costs should always be allocated on the basis of a certain cost allocation base that causes the cost. Hence, in case such an allocation base is not found, it is advisable to keep the cost unallocated to avoid distortions in determining the appropriate cost of a cost object. 12-4

The preferred guidelines for allocating service department costs are:

a.

Evaluate performance using budgets for each service (staff) department, just as they are used for each production or operating (line) department. When feasible, maintain distinctions between variable-cost pools and fixed-cost pools. Allocate variable- and fixed-cost pools separately. This is sometimes called the dual method of allocation. Note that one service department (such as a computer department) can contain a variable-cost pool and a fixed-cost pool. That is, costs may be pooled within and among departments if desired. Establish part or all of the details regarding cost allocation in advance of rendering the service rather than after the fact.

b.

c.

12-5 The distinction between direct and indirect costs depends on the cost object. A cost such as the salaries of service department personnel are a direct cost when the cost object is the service department. However, when the cost object is outside the service department, such as a production department that uses the services of the service department, the salaries of the service department must be allocated to the production department and hence are indirect.

..

514


12-6 Using budgeted rather than actual cost rates protects the using departments from inefficiencies in the service departments and from intervening price fluctuations. 12-7 In an organization, allocation of fixed costs must be based on the long-run capacity available to the user, rather than the actual usage from period to period because the level of fixed costs is affected by long range planning regarding the expected required overall level of service and not by short-run fluctuations in actual usage. 12-8 There are two alternatives: 1. Identify multiple cost pools, each with its own cost-allocation base. 2. Allocate all costs by the direct or step-down method using the same costallocation base. 12-9 Two methods of allocating service department costs are the direct method and the step-down method. The direct method ignores other service departments when any given service department's costs are allocated. No costs are allocated from one service department to another. The step-down method recognizes that some service departments provide services to other service departments as well as to producing departments. The costs of the first service department are allocated to all other service departments and the producing departments. Then the second service department's costs are allocated to the remaining service departments (i.e., all service departments except those whose costs have already been allocated) and the producing departments. Once a service department's costs have been allocated, no subsequent service department's costs are allocated back to it. This procedure continues until all service department costs have been allocated. 12-10 The direct method of allocating service department costs ignores services that one service department provides to another service department. The method is easier to apply and easier to understand, and often it leads to almost the same cost as more sophisticated methods, especially if there are only small services provided by service departments for other service departments. Companies may decide that the cost of a more sophisticated system is greater than the added accuracy it provides. 12-11 No. Both the direct and step-down methods allocate the same total amount of costs to the producing departments. 12-12 The four steps are as follows: 1. Divide the costs in each producing department into direct costs and indirect costs. 2. Trace the direct costs to the appropriate products or services. 3. Select cost pools and related cost-allocation bases in each production department, and assign all the indirect departmental costs to the appropriate cost pool. 4. Allocate (apply) the costs in each cost pool to the products or services in proportion to their usage of the related cost-allocation base. 12-13 First, managers identify the key activities in the organization, and they collect overhead costs for each activity. Cost drivers are then selected for each activity, and those cost drivers are used to allocate the costs to the products, services, or customers. ..

515


12-14

Some possible activities and cost drivers are: Activity Cost driver Group of machines Machine hours Set-up costs Number of set-ups Quality inspection Units passing inspection point Personnel department Number of employees

12-15 Step 1: Determine the key components of the system. Step 2: Develop the relationships between resources, activities, and cost objects. Step 3: Collect relevant data concerning costs and the physical flow of cost-allocation base units among resources and activities. Step 4: Calculate and interpret the new ABC information. 12-16 One of the primary purposes of any costing system is to provide as accurate cost information as possible subject to the cost-benefit criterion. There is always a trade-off between the accuracy of a system and the costs to implement and maintain it. Generally, as the operations of a company become more complex, the diversity of demands upon resources increases across products (services). In order to accurately track resource costs in such a diverse operating environment, many cost pools are needed for various activities, i.e., an ABC system. 12-17 When the cost objective is customers, allocating customer-related servicedepartment costs to products causes customer-cost distortion because the customer coststo-serve are allocated based on production-related cost-allocation bases and product mix percentages rather than allocation bases with a causal relationship to customer actions. 12-18 Suppose that not only are all of a company’s products profitable (that is, gross profit is positive), its average gross profit margin percentage is 30%. What if the total costs of the distribution and customer service value-chain functions is 35% of sales? In such a case, even without considering unallocated costs associated with R & D, design, and corporate support, the company is operating at a loss. The costs associated with customer actions, costs to serve, can often be either directly traced or allocated to customers. Identifying those customers whose costs to serve are greater than the gross profit they generate will help the company develop a strategy for profit improvement. 12-19 An organization should adopt the following strategies to lower the cost to serve the customers:  Motivate the customers to place large orders.  Ensure that the orders are specific and not ambiguous to reduce order changes.  Reduce pre- and post-sales support.  Ensure regular scheduling.  Ensure standard delivery  Ensure strict compliance with order specification to reduce the chances of returns. ..

516


12-20 If allocations are based on actual rather than forecasted sales, the allocation to a particular unit are affected by the actual sales results in other units. A unit may achieve exactly its expected level of sales but still have more allocated central corporate costs if other units fail to achieve their budgeted level of sales. 12-21 In some cases, it may be quite difficult to find a cause-and-effect relationship between a cost-allocation base and some central corporate support costs such as president’s salary and related costs. The company, in such circumstances, may allocate costs on the basis of revenue so that the department with higher revenues is allocated higher costs and vice versa. The logic is that a department with higher revenue has the ability to absorb higher costs. This is known as the “ability-to-bear” philosophy of cost allocation. 12-22 Joint costs are allocated to products or services for purposes of inventory valuation and income determination. They may also be allocated for cost-reimbursement contracts. 12-23 The physical units method allocates joint costs in proportion to some physical property of the products (e.g., weight or volume) at the split-off point. The relative sales value method allocates joint costs in proportion to the amounts for which the products can be sold at the split-off point. 12-24 By-products, like joint products, are not separately identifiable before the splitoff point. However, by-products have relatively insignificant sales values compared to main products. Only separable costs are applied to by-products; no joint costs are allocated to them. Revenues from by-products, less separable costs, are deducted from the cost of the main product. 12-25 Fixed costs are often allocated separately from variable costs because they are caused by different activities. Fixed costs are affected primarily by long-range decisions about the overall level of service. In contrast, variable costs depend on short-run fluctuations in actual usage.

..

517


12-26 Sales dollars are often a poor basis for allocation of costs because they reflect efficiency of sales effort and variations in pricing margins, neither of which is related to costs. Further, if actual sales is used as the allocation base, changes of sales in one department can affect costs allocated to the other departments. 12-27 One way to allocate national advertising costs to territories is on the basis of expected sales in each territory, computed by some formula combining population, income, appeal, competition, and supply capability. 12-28

(10-15 min.)

1.

Business Fixed costs per month: 30% of $100,000 70% of $100,000 Variable costs @ $200 per hour: 210 hours 400 hours Total costs

2. Fixed costs per month: 210/610 × $100,000 400/610 × $100,000 Variable costs, as before Total costs

Engineering

$30,000 $ 70,000 42,000 $72,000

80,000 $150,000

Business

Engineering

$34,426 42,000 $76,426

$ 65,574 80,000 $145,574

The dean of Business would probably be unhappy. The Business School has operated exactly in accordance with the long-range plan. Nevertheless, Business is bearing an extra $4,426 of fixed costs because of what another consumer is using. The dean would prefer the method in Requirement 1 because it insulates Business from short-run fluctuations in costs caused by the actions of other users.

..

518


12-29 (10-15 min.) 1.

Rate = [$3,000 + ($.05 × 100,000)] ÷ 100,000 = $.08 per copy Cost allocated to City Planning in August = $.08 × 42,000 = $3,360.

2.

Fixed cost pool allocated as a lump sum depending on predicted usage: To City Planning: (36,000 ÷ 100,000) × $3,000 = $1,080 per month Variable cost pool allocated on the basis of actual usage: $.05 × number of copies Cost allocated to City Planning in August: $1,080 + ($.05 × 42,000) = $3,180.

3.

The second method, the one that allocated fixed- and variable-cost pools separately, is preferable. It better recognizes the causes of the costs. The fixed cost depends on the size of the photocopy machine, which is based on predicted usage and is independent of actual usage. Variable costs, in contrast are caused by actual usage.

12-30

(10 - 15 min.)

1. Allocation based on budgeted sales* 2. Allocation based on actual sales**

Kolkata ₹216,000 240,000

Bengaluru ₹360,000 280,000

Delhi ₹144,000 200,000

*₹720,000 × (1,200/4,000); ₹720,000 × (2,000/4,000); ₹720,000 × (800/4,000) ** ₹720,000 × (1,200/3,600); ₹720,000 × (1,400/3,600); ₹720,000 × (1,000/3,600) 3.

The major argument against using actual sales as a cost driver for cost allocation is that a department's allocation depends on the success of other departments. Here, Bellevue is allocated an extra $12,000 because sales in the Richfield store are below budget, even though Bellevue's sales came in right on target. Further, stores with poor sales results probably do not cause reduced central office costs. If anything, a department with poor performance requires more central attention. Also, using budgeted sales reduces surprises; managers know what amount of allocated cost to expect. Often managers are more upset by unexpected changes in allocated amounts than by the size of the allocation itself.

..

519


12-31 (25-30 min.) 1.

See Exhibit 12-31. Calculations for the exhibit follow: 3 + 12 + 18 + 8 = 41 (3  41) × $92,000 = $6,732 (12  41) × $92,000 = $26,927 (18  41) × $92,000 = $40,390 (8  41) × $92,000 = $17,951 $240,000 + $400,000 = $640,000 ($240,000  $640,000) × $170,000 = $63,750 ($400,000  $640,000) × $170,000 = $106,250

2.

See Exhibit 12-31. Calculations for the exhibit follow: 5 + 3 + 12 + 18 + 8 = 46 (5  46) × $92,000 = $10,000 (3  46) × $92,000 = $6,000 (12  46) × $92,000 = $24,000 (18  46) × $92,000 = $36,000 (8  46) × $92,000 = $16,000 $240,000 + $400,000 = $640,000 ($240,000  $640,000) × $180,000 = $67,500 ($400,000  $640,000) × $180,000 = $112,500

..

520


Exhibit 12-31 Direct method:

Direct costs Personnel Administrative Total costs after allocation

Personnel $92,000 (92,000)

Residential Division Admin. Activity 1 Activity 2 Total $170,000 $60,000 $240,000 $300,000 6,732 26,927 33,659 (170,000) 0 63,750 63,750

Commercial Division Activity 3 Activity 4 Activity 5 $400,000 $90,000 $110,000 40,390 0 17,951 106,250 0 0

Total $600,000 58,341 106,250

$

$

$546,640

$127,951

$764,591

0

0

$66,732

$330,677

$397,409

$90,000

Step-down method:

Direct costs Personnel Administrative Total costs after allocation

..

Personnel $92,000 (92,000)

Residential Division Admin. Activity 1 Activity 2 Total $170,000 $60,000 $240,000 $300,000 10,000 6,000 24,000 30,000 (180,000) 0 67,500 67,500

Commercial Division Activity 3 Activity 4 Activity 5 $400,000 $90,000 $110,000 36,000 0 16,000 112,500 0 0

Total $600,000 52,000 112,500

$

$

$548,500

$764,500

0

0

$66,000

$331,50

$397,500

$90,000

$126,000

521


12-32

(15-20 min.)

1.

Direct method: Personnel

Store

Cloth

Garments

Direct department costs before allocation $180,000 $280,000 $2,400,000 $3,200,000 Personnel* (180,000) 80,000 100,000 Store** (280,000) 80,000 200,000 Total cost after allocation $ 0 $ 0 $2,560,000 $3,500,000 *(800  1800) × $180,000; (1,000  1,800) × $180,000 **(40  140) × $280,000; (100  140) × $280,000 2.

Step-down method:

Direct department costs before allocation Personnel* Supply** Total cost after allocation

Personnel

Store

$180,000 (180,000)

$280,000 $2,400,000 $3,200,000 11,250 75,000 93,750 (291250) 83,214 208,036 $ 0 $2,558,214 $3,501,786

$

0

Cloth

Garments

* (120  1,920) × $180,000; (800  1,920) × $180,000; (1,000  1,920) × $180,000 **(40  140) × $291250; (100  140) × $291250 12-33 (30-40 min.) 1. Sales Cost of sales Gross profit margin Units sold Gross profit margin per unit Gross profit margin percentage

Product A £64,000 40,000 £24,000 5,800 £4.14 37.5%

Product B £176,000 140,800 £35,200 8,600 £4.09 20.0%

Product C £560,000 448,000 £112,000 10,900 £10.28 20.0%

Product D £286,000 162,000 £ 124,000 3,200 £38.75 43.4%

Product D is the most profitable with a gross profit margin percentage of 43.4%. 2. – 4. Exhibit 12-33 shows calculations for requirements 2 – 4.

..

522


The most profitable customer type depends on the measure of profitability used. In this case customer type 1 has the greatest operating income percentage (40.5% – 17.3% = 23.2%) as well as the largest pound contribution to operating income (£34,502 – £14,736 = £19766).

..

523


Exhibit 12-33 Customer Type 1 Sales Gross price profit per margin Gross Product unit per unit Units Revenue profit 1 A £11.03 £ 4.14 400 £ 4,412 £ 1656 B 20.47 4.09 200 4,094 818 C 51.38 10.28 100 5,138 1,028 D 89.38 38.75 800 71,504 31,000 Total 1,500 £85,148 34,502 Cost to serve 14,736 Operating income £19,766 Customer gross margin percentage 40.5% Cost to serve percentage 17.3% Customer operating income percentage 23.2% 1

..

Customer Type 2

Units 4,400 2,400 800 1,600 9,200

Gross Revenue profit £ 48,532 £ 18,216 49,128 9,816 41,104 8,224 143,008 62,000 £281,772 98,256 90,386 £ 7870 34.9% 32.1% 2.8%

Customer Type 3

Units 1,000 6,000 10,000 800 17,800

Gross Revenue profit £ 11,030 £ 4,140 122,820 24,540 513,800 102,800 71,504 31,000 £719,154 162,480 174,878 (£12,398) 22.6% 24.3% (1.7%)

£64,000 ÷ 5,800 units; etc. The rounded numbers from the first two columns are used in subsequent calculations.

524


5. The chart below shows customer profitability for the three customer types and suggested strategies for profit improvement.

Grow business with this customer type by focused sales efforts and quantity discounts. Work with customers to lower the cost to serve. Seek internal process improvements to lower those elements of the cost to serve controllable by the company.

Work with customers to change their ordering patterns, focusing more on the more profitable products. Also, these customers may be able to lower the cost to serve. Seek internal process improvements to lower those elements of the cost to serve controllable by the company.

..

525


12-34 (15-20 min.) 1. Solvent A Solvent B 2. Solvent A Solvent B

Gallons 9,000 6,000 15,000

Weighting 9/15 × $100,000 6/15 × $100,000

Relative Sales Value at Split-off* $ 90,000 90,000 $180,000

Allocation of Joint Costs $ 60,000 40,000 $100,000

Weighting 9/18 × $100,000 9/18 × $100,000

Allocation of Joint Costs $ 50,000 50,000 $100,000

* $10 × 9,000 and $15 × 6,000 12-35

(10 min.)

1. Brown flour White flour

2.

Metric Ton 1,200 400 1,600

Relative Sales Value at Split-off* Brown flour $480,000 White flour 120,000 $600,000

Weighting 12/16 × $60,000 4/16 × $60,000

Allocation of Joint Costs $45,000 15,000 $60,000

Weighting 480/600 × $60,000 120/600 × $60,000

Allocation of Joint Costs $48,000 12,000 $60,000

* $400 × 1,200 and $300 × 400

..

526


12-36

(10-15 min.)

1.

None. The entire joint cost is allocated to the main product, i.e., rice.

2.

$4,000. The total inventory cost of the husk is the separable cost, that is, the cost incurred after the split-off point.

3.

Inventory cost of rice: Direct materials (seeds) Labor cost Irrigation, threshing, winnowing & milling cost Total Less: Revenue less separable costs of by-product $(200 x 30 – 4,000) Net cost of rice

..

$800,000 700,000 500,000 $2,000,000

(2,000) $1,998,000

527


12-37 1.

(30-40 min.) To properly classify a cost, it is necessary to specify the cost object. For example, power cost is a direct cost if the cost object is the power department but an indirect cost if the cost objective is the maintenance department, assembly department, or display types.

Type of Cost Assignment per Exhibit 12-1 1. Directly traced cost to departments

2. Indirect costs allocated to departments 3. Service department costs allocated to other service departments 4. Service department costs allocated to producing departments 5. Producing department costs allocated to other producing departments 6. Directly traced costs to departments that an organization can also trace directly to products and services 7. Producing department costs that an organization allocates to products or services 8. Directly traced costs to service departments that an organization can also trace directly to customers 9. Service department costs allocated to customers 10. Product/service costs assigned to customers

Example from Exhibit 12-21 Power cost in power department (power department is the cost object); $90,000 of direct costs of the maintenance department (maintenance department is the cost object); parts and direct labor costs in the assembly department (the cost object is the assembly department). General costs such as occupancy allocated to the maintenance and the assembly departments. Power department costs allocated to the maintenance department. Power costs allocated to the assembly departments; maintenance department costs allocated to the assembly department. Since there is only one producing department, no example exists. Parts and direct labor costs in the assembly department.

All assigned costs of setup and assembly activities, including assembly supervisor salaries, machine depreciation, power, maintenance, and occupancy. In this problem requirement, we assume that Darling does not determine customer costs. In this problem requirement, we assume that Darling does not determine customer costs. In this problem requirement, we assume that Darling does not determine customer costs.

2. The assembly facility uses the step-down method. Power department costs are first allocated to the maintenance service department and the assembly department before allocating the maintenance department costs to the two major activities in the assembly department.

..

528


3. Direct costs Allocated general costs** Allocated power department costs*** Allocated maintenance department costs**** Total

Power General Department Costs $ 60,000* $ 600,000 $(600,000)

Maintenance Department $ 90,000

Setup Activity

Assembly Activity

60,000

$120,000

$420,000

6,000

6,000

48,000

$(156,000)

52,000 $178,000

104,000 $572,000

$(60,000)

* 10 × $600 + 10 × $600 + 80 × $600 ** 10 + 20 + 70 = 100; (10 ÷ 100) × $600,000; etc. *** 10 + 10 + 80 = 100; (10 ÷ 100) × $60,000; etc. **** 2,000 + 4,000 = 6,000; (2,000 ÷ 6,000) × $156,000; etc. 4.

Parts Direct labor Setup activity Assembly activity Total Displays Cost per display

..

Cost Display Display Display per Type X Type Y Type Z Driver Driver Driver Driver Unit Units Cost Units Cost Units Cost $1,053,800 $ 575,000 $239,700 344,000 303,000 123,000 $1,310 20 26,200 60 78,600 120 157,200 203 1,000 203,000 1,800 365,400 1,200 243,600 $1,627,000 $1,322,000 $763,500 100,000 50,000 15,000 $16.27 $26.44 $50.90

529


12-38

(20-25 min.)

1.

Annual costs for 18,000 miles: Fixed Variable (€.40 × 18,000)

€ 5,100 7,200 €12,300

Cost per mile = €12,300 ÷ 18,000 miles = €.68 per mile 2.

Two factors caused the July allocation of €.90 per mile to exceed the average of €.68 per mile: (1) The motor pool’s operating inefficiencies are passed on to the user departments. The cost of 90,000 miles in April should have been [(€5,100 ÷ 12 months) × 70 cars] + (€.40 × 90,000 miles) = €29,750 + €36,000 = €65,750. Therefore, (€81,000 – €65,750) = €15,250 of “unnecessary” cost was assigned to user departments, which is €15,250 ÷ 90,000 miles = €.17 per mile. (2) July was a month of low general usage. In an average month, 105,000 miles are driven (1,500 miles per car), and the fixed cost per mile is (€5,100 ÷ 12 months) ÷ 1,500 miles = €425 ÷ 1,500 miles = €.28 per mile. In July, the €425 fixed cost of each car was spread over only 1286 miles, so the fixed cost per mile was €425 ÷ 1,286 = €.33 per mile. This factor accounts for an extra €.05 per mile.

3.

Undesirable behavioral effects include: (a)

The total actual carpool cost is allocated. The manager is not motivated to control these costs. (b) Allocated costs are affected by car usage in other departments. A department is better off if its car usage happens to fall in a month when other departments have high mileage. (c) Decisions about whether driving another mile is worth its cost are not appropriately made. The municipality incurs only €.40 more expense for an additional mile, but departments are charged more. (d) The cost allocation is affected only by miles driven, not number of cars assigned to a department. A department with two cars each being driven 9,000 miles per year is allocated the same cost as one with one car driven 18,000 miles per year. But each car causes the same average fixed costs, so fixed costs should be allocated on the basis of number of cars rather than miles driven. This may be the reason the chief of tax collection department was continually concerned with his car costs. His department’s cars were driven an average of 2,000 miles per month, but the city’s average was only 1,500 miles. Because both fixed and variable costs are allocated on a per-mile basis, his department’s cars were allocated more fixed costs than the average car in the municipality. If fixed costs were allocated on the basis of number of cars, each car would be charged €425 per month. This becomes ..

530


(€425 ÷ 2,000) = €.21 per mile for the city planner’s cars compared to (€425 ÷ 1,500) = €.28 for the average car in the municipality. 4.

Two basic principles should be applied: (a) Allocate budgeted, not actual, costs. Inefficiencies of the carpool should not be passed on to user departments. (b) Separate costs into fixed and variable cost pools. The fixed costs should be allocated on the basis of number of cars assigned to a department or long-run predicted use of cars. Variable costs are appropriately assigned on a per-miledriven basis. This cost-allocation method illustrates why the chief of tax collection department has a legitimate complaint. In July he paid €.17 per mile extra because of motor pool inefficiency, €.05 per mile extra because other departments had light usage in July, and €.07 per mile extra because fixed costs were charged on a per-mile basis rather than a per-car basis.

12-39 1.

(20-30 min.)

Actual costs Rate per shipment* To North To South

£600,000 + £0.80(600,000) = £1,080,000 £1,080,000 ÷ 600,000 = £1.80 300,000 × £1.80 = £540,000 300,000 × £1.80 = £540,000

*Rate is per shipment 2.

3.

Actual costs £600,000 + £0.80(480,000) = £984,000 Rate per shipment £984,000 ÷ 480,000 = £2.05 To North 180,000 × £2.05 = £369,000 To South 300,000 × £2.05 = £615,000 Note that South’s costs increased from £540,000 to £615,000, or 13.9%, solely because North’s shipments declined. Rate per shipment To North To South

£1,500,000 ÷ 600,000 300,000 × £2.50 300,000 × £2.50

= = =

£2.50 £750,000 £750,000

Such allocation seems unjustified because the operating departments have to bear another department’s cost of inefficiency. Note that the use of a predetermined or budgeted total amount geared to the various levels of activity of the operating departments would eliminate this difficulty. For example, the £1.80 rate of part (1) would be used here despite the excess of actual costs over budgeted costs. ..

531


4.

Basic maximum capacity: 432,000 + 288,000 = 720,000 shipments. Fixed costs: To North, 432/720 × £600,000 To South, 288/720 × £600,000 Variable costs: To North, £.80 × 180,000 To South, £.80 × 300,000 Total costs

North £360,000 -

South £240,000

144,000 £504,000

240,000 £480,000

Note that North’s costs are $504,000 rather than the £369,000 in part (2). This method has the following advantages: a. The use of a predetermined unit rate for variable costs prevents the total charges from being affected by the efficiency of price changes of the service department. b. The use of a predetermined lump-sum for fixed costs prevents the total charges from being affected by the consumption of service or the activity levels of other operating departments or the activity level of the service department. 12-40

(25-30 min.)

There a several ways to organize an analysis that provides product costs. We like to focus first on determining total activity-cost pools and activity cost per driver unit. Schedule a:

Activity center cost pools

Resources Supporting the Setup Center Assembly supervisors Assembly machines Facilities management Power Total assigned cost Cost per driver unit (setup) Resources Supporting the Assembly Activity Center Assembly supervisors Assembly machines Facilities management Power ..

€161,400 ÷ 80

Allocated Cost € 5,400 104,000 40,000 12,000 €161,400 €2,017.50

Allocation Calculation €180,000 × 97% €494,000 × (3,000 ÷ 3,800) €190,000 × (3,000 ÷ 3,800) €108,000 × (160 ÷ 180)

Allocated Cost €174,600 390,000 150,000 96,000

Allocation Calculation €180,000 × 3% €494,000 × (800 ÷ 3,800) €190,000 × (800 ÷ 3,800) €108,000 × (20 ÷ 180)

532


Total assigned cost Cost per driver unit (machine hour)

..

€810,600 ÷ 3,000

€810,600 € 270.20

533


Exhibit 12-40 Contribution to cover other value-chain costs by product Schedule b: Gross profit Cost per Driver unit Activity/Resource (Schedule a) Setup €2,017.50 Assembly € 270.20 Parts Direct labor Total Units Cost per display Selling price Unit gross profit Total gross profit

Window AC

Split AC

Driver Units Cost 40 € 80,700 2,000 540,400 31,000,000 11,600,000 €43,221,100 200,000 € 216.1055 300.0000 € 83.8945 €16,778,900

Driver Units Cost 24 € 48,420 800 216,160 5,160,000 1,500,000 €6,924,580 20,000 € 346.229 500.000 € 153.771 €3,075,420

Inverter AC Driver Units 16 200

Cost € 32,280 54,040 720,000 240,000 €1,046,320 2,000 € 523.16 700.00 € 176.84 € 353,680

The total contribution of these products is €16,778,900 + €3,075,420 + €353,680 = €20,208,000. The total contribution of these products is $387,650 + $180,630 + $122,86 = $691,140.

..

534


12-41 (10-15 min.)

Window AC Split AC Inverter AC Total

Customer Type 1 Gross Profit Units per Unit Sold Gross Profit € 83.8945 150,000 €12,584,175 153.7710 10,000 1,537,710 176.8400 0 0 €14,121,885

Customer Type 2 Units Sold Gross Profit 50,000 €4,194,725 10,000 1,537,710 2,000 353,680 €6,086,115

12-42 (15-20 min.) 1. Sales ($460 × 2,800; $800 × 2,000) Cost of sales: Purchase cost ($70 × 2,800; $120 × 2,000) Indirect cost Gross product margin

Footwear $1,288,000

Equipment $1,600,000

196,000 630,000 1 826,000 $ 462,000

240,000 750,000 2 990,000 $ 610,000

1 $1,380,000 ÷ (18.75 × 2,800 + 31.25 × 2,000) = $12.00 per pound. The allocation to footwear is $12 × 2,800 × 18.75 = $630,000. 2 $12 per pound × 31.25 × 2,000 = $750,000 2.

Gross margin per case: Footwear, $462,000 ÷ 2,800 = $165 Equipment, $610,000 ÷ 2,000 = $305

Specialty Stores Gross Margin, Footwear* $198,000 Gross margin, Equipment ** 122,000 Total gross margin $320,000 *$165 × 1,200 = $198,000; $165 × 1,600 = $264,000 **$305 × 400 = $122,000; $305 × 1,600 = $488,000

Department Stores $264,000 488,000 $752,000

3. The gross margin per case of equipment is much larger so more emphasis should be placed on equipment sales, especially at specialty stores.

..

535


12-43 (25-30 min.) 1. Sales ($460 × 2,800; $800 × 2,000) Cost of sales Purchase cost ($70 × 2,800; $120 × 2,000) Indirect cost Product gross margin

Footwear $1,288,000

Equipment $1,600,000

196,000 378,000 1 574,000 $ 714,000

240,000 450,000 2 690,000 $ 910,000

1 ($1,380,000 - $552,000) ÷ (18.75 × 2,800 + 31.25 × 2,000) = $7.20 per pound. The allocation to footwear is $7.20 × 2,800 × 18.75 = $378,000. 2 $7.20 per pound × 31.25 × 2,000 = $450,000 2. Gross margin per case Cases Product gross margin Customer gross margin Cost to serve Customer profit margin Revenue Gross margin percentage Cost-to-serve percentage Customer profit percentage

Specialty Stores Footwear Equipment $255 1 $455 2 1,200 400 $306,000 $182,000 $488,000 384,000 3 $104,000 $872,000 56.0% 44.0% 11.9%5

Department Stores Footwear Equipment $255 $455 1600 1600 $408,000 $728,000 $1,136,000 168,000 4 $968,000 $2,016,000 56.3% 8.3% 48.0%

1 $714,000 ÷ 2,800 2 $910,000 ÷ 2,000 3 The cost per order = $552,000 ÷ (160 + 70) = $2,400. The allocation to specialty stores is 160 × $2,400 = $384,000. 4 $2,400 × 70 = $168,000 556.0% - 44.0% = 12.0%, which differs from 11.9% due to rounding error.

..

536


Exhibit 12-43

CUSTOMER PROFITABILITY

GROSS PROFIT MARGIN PERCENTAGE

100% 90% 80% Department Stores

70%

Specialty Stores

60% 50% 40% 30% 20% 10% 0% 0%

20%

40%

60%

80%

100%

COST TO SERVE PERCENTAGE

..

537


3. Exhibit 12-43 depicts the profitability of both customer types as a function of product gross margin and the cost to serve. Note that both customers have about the same product profitability based on the mix of products they purchase. However, the cost to serve is dramatically different, resulting in significant differences in overall profitability. Specialty stores order 1,600 ÷ 160 = 10 cases per order compared to 3,200 ÷ 70 ≈ 46 cases per order by department stores. Suggested strategies for profit improvement: 

Department stores are clearly generating most of the profit for TCS. The company should both protect this customer from inroads by competitors through its pricing strategy (discounts) and profile this customer type to see if it is possible to apply actions to specialty stores that would reduce their cost to serve. The cost to serve of specialty stores needs to be reduced. If there is a cause-effect relationship between number of orders and the cost to serve, actions should be taken to increase the order size.

4. A comparison of customer profitability based on the two treatments of the costs to serve is shown in the table below.

Specialty store profit Department store profit Total TCS profit

Treatment of Cost to Serve As Product Cost As Customer Cost (Problem 12-42) (Problem 12-43) $ 320,000 $ 104,000 752,000 968,000 $1,072,000 $1,072,000

The difference in profitability is due to the use of orders rather than pounds purchased to allocate the $552,000 costs of the order-processing and customer-service activities. To the extent that orders is a more plausible and reliable cost driver (cost-allocation base), management should carefully evaluate their customer mix strategy. For example, the table below gives some food for thought. Percent of profit Percent of cases sold Percent of weight shipped (purchased) Percent of orders

Specialty Stores 19.9% 33.3 30.4 69.6

Department Stores 80.1% 66.7 69.6 30.4

The percent of overall TCS profit for specialty stores is significantly lower than each of the non-financial metrics that drive costs.

..

538


12-44

(20-30 min.)

1.

Basic long-run usage: 90 + 60 = 150 chemotherapies per month Total costs incurred: $28,000 + 120 chemotherapies ($220) = $54,400 In-house Department Fixed costs: 90/150 × $28,000 $16,800 60/150 × $28,000 Variable costs: 60 × $220 13,200 60 × $220 Total allocated costs $30,000

2.

Out-Patient Department

$11,200

13,200 $24,400

For budgetary control and motivation purposes, it is best not to allocate the $5,600 efficiency variance ($60,000 minus the $54,400 computed above). For cost recovery purposes, if reimbursement is based on actual costs, it should be allocated.

3.

In-house Department Total costs incurred, $54,400: 60/120 × $54,400 60/120 × $54,400

Out-Patient Department

$27,200 $27,200

The OPD bears $2,800 more costs than in part (1) despite the fact that its volume was exactly in accordance with its long-run average usage. In short, the OPD’s costs have increased solely because of a fellow consumer’s actions, not its own actions. The in-house department’s failure to reach its predicted usage results in shifting $2,800 more fixed costs to the OPD. A behavioral effect of this method would be toward more erratic scheduling (to the extent to which this discretion exists). For instance, if the in-house had a relatively light month, it would be motivated toward not scheduling procedures during the final week and bunching them in the first week of the second month. In this way, its unit costs for the second month would be lowered. 4.

..

Both the in-house and the OPD would be induced to underestimate usage. Of course, if both play the same game, the final fraction borne by each would change little. One way to counteract these tendencies is to exert higher arbitrary 539


cost allocations to both the in-house and OPD if they consistently exceed their predicted usage. Also, first priority on scarce resources can be extended to those consumers who are committed to the higher fractions.

..

540


12-45

(20-30 min.)

1. Building Services Direct department costs before allocation Building services Materials receiving and handling Total costs after allocation

Materials Receiving and Handling

Traditional Non-leather Leather Shoes Shoes

CHF120,000 CHF96,000 CHF544,000 CHF438,400 (120,000) 80,000 40,000 (96,000)

32,000 64,000 CHF656,000 CHF542,400

Calculations: 25,000 + 12,500 = 37,500 (25,000 ÷ 37,500) × CHF120,000 = CHF80,000 (12,500 ÷ 37,500) × CHF120,000 = CHF40,000 No. of components: 10 × 4,000 = 40,000; 16 × 5,000 = 80,000 40,000 + 80,000 = 120,000 (40,000 ÷ 120,000) × CHF96,000 = CHF32,000 (80,000 ÷ 120,000) × CHF96,000 = CHF64,000 2.

Traditional leather shoes: CHF656,000 ÷ 24,000 hours = CHF27.3333 per direct-labor hour Non-leather shoes: CHF542,400 ÷ 80,000 components = CHF6.78 per component

3.

Total cost = Direct materials cost + Manufacturing cost N1: CHF74 + (CHF27.3333 × 2.4) = CHF74 + CHF65.60 = CHF139.60 N2: CHF86 + (CHF27.3333 × 4.8) = CHF86 + CHF131.20 = CHF217.20 F1: CHF63 + (CHF6.78 × 10) = CHF63 + CHF67.80 = CHF130.80 F2: CHF91 + (CHF6.78 × 20) = CHF91 + CHF135.60 = CHF226.60

..

541


12-46

(20-30 min.)

1.

Direct department costs before allocation Building services Materials receiving and handling Total costs after allocation

$

Building Services

Materials Receiving and Traditional Non-leather Handling Leather Shoes Shoes

$120,000 (120,000)

$ 96,000 7,500

$544,000 75,000

$438,400 37,500

0

$(103,500) $ 0

34,500 $653,500

69,000 $544,900

Calculations: 2,500 + 25,000 + 12,500 = 40,000 (2.5 ÷ 40) × $120,000 = $7,500 (25 ÷ 40) × $120,000 = $75,000 (12.5 ÷ 40) × $120,000 = $37,500 No. of components: 10 × 4,000 = 40,000; 16 × 5,000 = 80,000 40,000 + 80,000 = 120,000 (400 ÷ 1,200) × $103,500 = $34,500 (800 ÷ 1,200) × $103,500 = $69,000 2.

Traditional leather shoes: $653,500 ÷ 24,000 hours = $27.229 per direct-labor hour Non-leather shoes: $544,900 ÷ 80,000 components = $6.81 per component

3.

Total cost = Direct materials cost + Manufacturing cost N1: $74 + ($27.229 × 2.4) = $74 + $65.35 = $139.35 N2: $86 + ($27.229 × 4.8) = $86 + $130.70 = $216.70 F1: $63 + ($ 6.81 × 10) = $63 + $ 68.10 = $131.10 F2: $91 + ($ 6.81 × 20) = $91 + $ 136.20 = $227.20

..

542


12-47

(15-25 min.)

1.

See Exhibit 12-47, Part 1.

2. See Exhibit 12-47, Part 2. Only the first column is required. However, the other two columns verify the following discussion. The cost of the model 1 circuit boards decreases from ¥961,600 to ¥886,921, a decrease of ¥74,679. But because the decrease is due to a lower allocation and this is from fixed costs that do not change, the decrease is now allocated to models 2 and 3. The costs of models 2 and 3 increase to absorb the decrease in model 1 cost. So, why would Tokuga’s management want to implement this process improvement? Because the improved efficiencies will free up processing capacity in resources used for these two activities. The freed up capacity can be deployed to meet other needs such as an increase in demand. The total cost (¥6,120,000) of all three models does not change. 12-48

(25 min.)

1.

Inspection cost: $4.00 × 3,200 = Packaging Labor cost: ($26,600 / 380,000) × 64,000 = Shipment cost: $15 × 400 = Total cost

$ 12,800 4,480 6,000 $23,280

2.

Inspection cost: $4.00 × 500 = Labor cost saving: No savings; fixed cost * Shipment cost saving: $15 × 200 = Total cost saving

$ 2,000 0 3,000 $5,000

* Capacity is made available. If there is a profitable use of that capacity (that is, if the opportunity cost is not zero), a savings would result equal to the benefit from the use of the capacity. 3.

Receiving cost per pound: $23,280  64,000 = $0.36375 Estimated cost saved from 10,000 pounds = $0.36375 × 10,000 = $3,637.50

The company would have underestimated the savings by $5,000 – $3,637.50 = $1,362.50, and they may have continued to purchase and stock small-sales-level brands that are actually unprofitable.

..

543


Exhibit 12-47, Part 1 Model 1 Direct materials: Model 1: ¥4,000 × 80 boards ¥320,000 Model 2: ¥6,000 × 160 boards Model 3: ¥8,000 × 300 boards Material handling activity1: Model 1: ¥26 × 20 × 80 41,600 Model 2: ¥26 × 15 × 160 Model 3: ¥26 × 10 × 300 Assembly activity2: Model 1: ¥67 × 40 × 80 214,400 Model 2: ¥67 × 30 × 160 Model 3: ¥67 × 16 × 300 Soldering activity3: Model 1: ¥47 × 60 × 80 225,600 Model 2: ¥47 × 40 × 160 Model 3: ¥47 × 20 × 300 Quality assurance activity4: Model 1: ¥400 × 5 × 80 160,000 Model 2: ¥400 × 3 × 160 Model 3: ¥400 × 2 × 300 Total cost for circuit boards ¥961,600 Cost per circuit board ¥ 12,020 1 ¥182,000 ÷ (80 × 20 + 160 × 15 + 300 × 10) = ¥26 per distinct part 2 ¥857,600 ÷ (80 × 40 + 160 × 30 + 300 × 16) = ¥67 per automatic insertion 3 ¥808,400 ÷ (80 × 60 + 160 × 40 + 300 × 20) = ¥47 per part 4 ¥592,000 ÷ (80 × 5 + 160 × 3 + 300 × 2) = ¥400 per minute

..

Model 2

Model 3

¥960,000 ¥2,400,000 62,400 78,000 321,600 321,600 300,800 282,000 192,000 ¥1,836,800 ¥11,480

240,000 ¥3,321,600 ¥11,072

544


Exhibit 12-47, Part 2 Model 1 Direct materials: Model 1: ¥4,000 × 80 boards ¥320,000 Model 2: ¥6,000 × 160 boards Model 3: ¥8,000 × 300 boards Material handling activity1: Model 1: ¥30.13245 × 8 × 80 19,285 Model 2: ¥30.13245 × 15 × 160 Model 3: ¥30.13245 × 10 × 300 Assembly activity2 Model 1: ¥67 × 40 × 80 214,400 Model 2: ¥67 × 30 × 160 Model 3: ¥67 × 16 × 300 Soldering activity3: Model 1: ¥47 × 60 × 80 225,600 Model 2: ¥47 × 40 × 160 Model 3: ¥47 × 20 × 300 Quality assurance activity4: Model 1: ¥448.48485 × 3 × 80 107,636 Model 2: ¥448.48485 × 3 × 160 Model 3: ¥448.48485 × 2 × 300 Total cost for circuit boards ¥886,921 Cost per circuit board ¥11,087 1 ¥182,000÷ (80 × 8 + 160 × 15 + 300 × 10) = ¥30.13245 per distinct part 2 ¥857,600÷ (80 × 40 + 160 × 30 + 300 × 16) = ¥67 per automatic insertion 3 ¥808,400÷ (60 × 80 + 40 × 160 + 20 × 300) = ¥47 per part 4 ¥592,000÷ ( 3 × 80 + 3 × 160 + 2 × 300) = ¥448.48485 per minute

..

Model 2

Model 3

¥960,000 ¥2,400,000 72,318 90,397 321,600 321,600 300,800 282,000 215,273 ¥1,869,991 ¥ 11,687

269,091 ¥3,363,088 ¥11,210

545


12-49

(20 min.)

1.

Allocations are in millions:

Divisions: Northern Central Southern Total 2.

Actual Revenue

Allocated Costs

€180 330 390 €900

[(180 ÷ 900) × €45] = € 9.0 [(330 ÷ 900) × €45] = 16.5 [(390 ÷ 900) × €45] = 19.5 €45.0

Northern Division’s manager would probably be indifferent. Central’s manager would be pleased, while Southern’s would be displeased. The major weakness of using revenue as a basis for cost allocation is that it often fails to portray underlying cause-and-effect relationships. The major point of this problem is to show how strange results occur when the costs being allocated to a given segment are dependent on the activity of some other segment. The Southern Division may have done the most to reduce the unit cost of central services, but it is being charged with a heavier dose of common costs. Indeed, Central may have received more rather than less attention because of its current competitive troubles. Most of the central costs are discretionary. Pinpointing cause-and-effect relationships is hard. Such costs are usually predetermined by management fiat or by budgeted revenue. Serious consideration should be given to one or more of the following: a. b. c.

..

No allocation, because no convincing allocation base is available. Dividing the services into sub-categories and allocating by the use of several different cost drivers. Using budgeted revenues rather than actual revenues as a cost driver for allocation. Of course, the use of budgeted revenues may induce more “gamesmanship” than is typically encountered during the budgetary process. There is a tendency to “under-budget” whenever a lower cost allocation will result.

546


3.

Allocations are in millions: Budgeted Revenue Divisions: Northern Central Southern Total

Allocated Costs

€180 360 420 €960

[(180 ÷ 960) × €45] = € 8.4375 [(360 ÷ 960) × €45] = 16.8750 [(420 ÷ 960) × €45] = 19.6875 €45.0000

Many managers prefer this method because it portrays causes and effects somewhat better than in requirement (1). That is, at least the overall level of costs tends to be planned. In requirement (1), the allocated costs were each 5% of actual revenue. However, in requirement (3), the allocation is predetermined, and therefore the percentages of actual revenue vary:

Divisions: Northern Central Southern Total

(1) Actual Revenue

(2) Allocated Costs

(3) Percentage (2) ÷ (1)

€180 330 390 €900

€ 8.4375 16.8750 19.6875 €45.000

4.7% 5.1% 5.0%

Note that Central’s budgeted percentage would have been €16.875 ÷ €360 = 4.7%. The resultant deviation of the actual percentage (5.1%) from the budgeted percentage (4.7%) would highlight the effects of Central’s troubles. 4.

..

Many accountants and managers oppose allocating any central costs when no convincing causes and effects can be established in any economically feasible way. The opponents of cost allocation feel that the managers of subunits will have better attitudes and will make better decisions if no allocation occurs.

547


12-50

(20-40 min.)

1.

(a) The allocation of joint costs would be in a 1:5 ratio:

Sales value Joint costs Separable costs Total costs Operating profit

Product A $2,000 $400 700 $1,100 $ 900

Product B $2,000 $2,000 400 $2,400 $ (400)

Total $4,000 $2,400 1,100 $3,500 $ 500

(b) No. Joint costs are not relevant for this decision because you cannot stop incurring that part allocated to one product and still continue to incur only the other part. If the total process is profitable, you should process any product that shows a positive contribution after the split-off point. Although Product B shows a book loss of $400, it has a contribution after the split-off point of $2,000 - $400, or $1,600. 2.

(a) The relative sales value method deducts separable costs to arrive at an imputed sales value at split-off point: A Sales value $2,000 Separable costs 700 Sales value imputed at split-off point $1,300 Allocation of joint cost, 1,300/2,900 and 1,600/2,900, respectively 1,076 Operating profit $ 224

B $2,000 400 $1,600

Total $4,000 1,100 $2,900

1,324 $ 276

2,400 $ 500

(b) No. Product B does have the greater book profit and contribution after the split-off point, but Product A has the greatest contribution per pound, which is the scarce resource in this case. If, for example, the engineer changes the process by 40 pounds, so that we end up with 440 pounds of B and 40 pounds of A, separable costs would become $350 for A and $440 for B, totaling $790 (assuming separable costs are all variable). Sales values would become $1,000 for A and $2,200 for B, and total of $3,200. Total contribution after the split-off would drop from $2,900 to $2,410 and total profit would drop from $500 to $10. Pounds Sales value Separable costs Contribution to joint costs Joint costs Operating profit

..

A 40

B 440

Total 480

$1,000 350 $ 650

$2,200 440 $1,760

$3,200 790 $2,410 2,400 $ 10

548


12-51 (100 – 200 min.) 1. Exhibits 12-51A and 12-51B show the calculation of customer gross margin percentage and customer cost-to-serve percentage for the 4 customer types. Exhibit 1251C shows a plot of customer gross margin percentage versus customer cost-to-serve percentage for the 4 customer types. 2. Suggested strategies for profit improvement for the 4 customer types follow. 

Customer type 1 - Mega stores. These stores have the lowest cost-to-serve. Profitability can be improved by focusing on a better product mix. A quarter of the sales (cases) to these stores are from bulk and singles products – both of which have a negative gross margin. A shift in mix towards more regular and fragile product types would improve profitability.

Customer type 2 – Local small stores. These stores have a product mix that contains a substantial amount (32%) of the negative gross margin products. The same change in sales focus that applies to mega stores can be applied to local small stores. But unlike mega stores, small stores are very costly to serve. From Exhibit 12-51 B, the largest single cost to serve local small stores is truck deliveries. The average number of cases per order (the same as per truck delivery) is 6,000,000 ÷ 80,000 = 75. Compare this to mega stores that average 7,680,000 ÷ 32,000 = 240 cases per order (delivery). This is a significant factor causing the high cost-toserve. For example, suppose that the average order size could be increased from 75,000 to 150,000 cases. If the total annual cases sold is unchanged (6,000,000), a total of 40 orders, a 50% reduction, would be made. An estimate of the cost savings and the impact on the cost-to-serve percentage can be made as follows: Cost per Driver Unit (Exhibit 12-51B) $167.55 27.49 5.83 19.44

Truck delivery Order processing Regular scheduling Expedited scheduling Total cost savings (000) Cost savings as a percent of revenue New cost-to-serve as a percent of revenue

Reduction in Driver Units of 50% 34,000 40,000 36,000 4,000

Cost Savings (000) $5,696.70 1,099.60 209.88 77.76 $7,083.94

24.9% 60.1%

In addition to the above savings, other activities would also be impacted by the reduction in orders such as customer service. So while the total impact of focusing on increasing order size can only be estimated, it is reasonable to expect dramatic cost savings from the current 85% of revenue. ..

549


Other factors that should be investigated include the high level of corporate support and customer service.

..

Customer type 3 – Local large stores. Local large stores generate $68,400 ÷ $136,230 = 50% of DSI’s total revenue and with a net margin of 58% - 47% = 11%. The key to local large store profitability is sales of a large percentage (80%) of regular product. The cost-to-serve percentage is 47%. This could be reduced as for customer type 2 by increasing the order size from the current level of 14,400,000 ÷ 120,000 = 120 cases per order. But a dramatic improvement should not be expected. In general, local large stores are sustaining DSI’s business and their loyalty should be cultivated.

Customer type 4 – Specialty stores. Specialty stores have a low gross margin of 22% coupled with a very large cost-to-serve percent of 106%! Although these stores do not account for a significant portion of DSI’s revenue the company should rationalize their business. Several actions could be suggested. One is to charge a premium for all high-security products. The vast majority of these products are sold to specialty stores with only marginal sales to mega and local small stores. Another action is to adopt a customer loyalty program based on volume of sales. The list price of $7.25 per case would apply to customers with sales volumes less than a specified level. Most of DSI’s customers would qualify for discounts (similar to those currently existing) so prices would not be significantly different. For specialty stores, prices would increase dramatically. This may result in losing specialty-store business so DSI needs to decide is this is a direction they wish to consider.

550


Customer Type

Exhibit 12-51A (Units and dollars are in thousands.)

Product Product mix percentage

Regular 60%

Short 5%

Fragile 5%

Bulk 20%

High Security 5%

Singles 5%

Total 100%

4,608

384

384

1,536

384

384

7,680

Cases sold

Gross Profit Percentage

1

Total Revenue @ 4.75/case

$ 21,888

$ 1,824

$ 1,824

$7,296

$ 1,824

$ 1,824 $36,480

35%

2

Gross Profit per Case Total Gross Profit Product mix percentage Cases sold Total Revenue @ 4.75/case Gross Profit per Case Total Gross Profit Product mix percentage Cases sold

$ 3.28 $ 15,114 50% 3,000 $ 14,250 $ 3.28 $ 9,840 80% 11,520

$ 1.58 $ 607 5% 300 $ 1,425 $ 1.58 $ 474 0% -

$ 2.74 $ 1,052 5% 300 $ 1,425 $ 2.74 $ 822 10% 1,440

$(1.44) $(2,212) 30% 1,800 $ 8,550 $ (1.44) $(2,592) 10% 1,440

$ 0.54 $ (5.30) $ 207 $(2,035) $12,733 8% 2% 100% 480 120 6,000 $ 2,280 $ 570 $28,500 $ 0.54 $ (5.30) $ 259 $ (636) $ 8,167 0% 0% 100% 14,400

29%

Total Revenue @ 4.75/case Gross Profit per Case Total Gross Profit Product mix percentage

$ 54,720 $ 3.28 $ 37,786 10%

$ $ 1.58 $ 20%

$ 6,840 $ 2.74 $ 3,946 0%

$ 6,840 $ (1.44) $(2,074) 0%

$

$ 0.54 $ 70%

58%

Cases sold Total Revenue @ 4.75/case

-

$

120 570

-

$

60 285

$

-

420 $ 1,995

$

Gross Profit per Case

$

3.28

$

1.58

$

2.74

$ (1.44)

$

0.54

$ (5.30)

Total Gross Profit

$

197

$

190

$

-

$

$

227

$

3

4

..

$

-

$ - $68,400 $ (5.30) $ - $39,658 0% 100% -

600 - $ 2,850 - $

22%

613

551


2

Parcel Delivery Deliveries

Truck Delivery Deliveries

Shipping Pallets

Expedited Scheduling Orders

Regular Scheduling Orders

Corporate Support Labor Hours

Order Changes Number of Changes

Customer Service Labor Hours

1

Orders

Customer Type

Activity

Order Processing

Exhibit 12-51B (Units and dollars are in thousands.)

Cost/Driver Unit

$27.49

$43.34

$32.63

$51.66

$5.83

$19.44

$6.60

$167.55

$23.89

Total

Driver Units Cost to Serve

32 $879.68

18.7 $810.46

3.2 $104.42

-

29 $169.07

3 $58.32

416 $2,745.6

25.6 $4,289.28

1.6 $38.22

$9,095.05

Cost Driver

Revenue (See Exhibit 12-51A) Cost-to-Serve Percentage

$36,480.00 24.9%

Driver Units

80

100

8

Cost to Serve

$2,199.2

$4,334

$261.04

Revenue (See Exhibit 12-51A) Cost-to-Serve Percentage

..

20

72

$1,033.2 $419.76

8

640

68

8

$155.52

$4,224

$11,393.4

$191.12

$24,211.24 $28,500.00 85.0%

552


Parcel Delivery Deliveries

Truck Delivery Deliveries

Shipping Pallets

Expedited Scheduling Orders

Regular Scheduling Orders

Corporate Support Labor Hours

Order Changes Number of Changes

Customer Service Labor Hours

3

Orders

Customer Type

Activity

Order Processing

Exhibit 12-51B (continued)

Cost/Driver Unit

$27.49

$43.34

$32.63

$51.66

$5.83

$19.44

$6.60

$167.55

$23.89

Driver Units

120

70

2.4

80

108

12

840

90

6

$78.31

$4,132.8

$629.64

$233.28

$5,544

$15,079.5

$143.34

Cost Driver

Cost to Serve

$3,298.8 $3,033.8

Revenue (See Exhibit 12-51A)

4

Cost to Serve

12

47.0% 30

$329.88 $1,300.2

Revenue (See Exhibit 12-51A) Cost-to-Serve Percentage

..

$32,173.47 $68,400.00

Cost-to-Serve Percentage Driver Units

Total

1.2

0

10

2

60

4.8

2.4

$39.16

-

$58.3

$38.88

$396

$804.24

$57.34

$3,023.99 $2,850.00 106.1%

553


Exhibit 12-51C

CUSTOMER PROFITABILITY

GROSS PROFIT PERCENTAGE

100% 90% 80% 70% 60%

CT3, 47%, 58%

50% 40% 30%

CT1, 24.9%, 34.9%

20%

CT2, 85%, 29% CT4, 106%, 22%

10% 0%

0% 10% 20% 30% 40% 50% 60% 70% 80% 90% 100 110 120 % % % COST-TO-SERVE PERCENTAGE ..

554


12-52 (50-60 min.) 1.

Systems Department Claims First Department Quarter Historical Budget Usage

Hardware and other capacity-related costs Software development Computer-related operations Input/output-related operations 2.

Solution is in Exhibit 12-52.

3.

a.

b.

..

$150,000 141,750 189,000 75,600 $556,350

50% 40 15 75

Claims Department First Quarter Budget $ 75,000 56,700 28,350 56,700 $216,750

The new charging system should improve cost control in the Systems Department (if the rates are valid) because inefficiencies can no longer be passed on to the user departments. Thus, the Systems Department would be forced to watch its costs closely. The recommended system for charging costs to user departments should improve planning and cost control in the user departments. Decisions that affect capacity-related costs will affect the allocation of those costs, while decisions affecting only short-run operating costs will affect the allocation of only the operating costs.

555


EXHIBIT 12-52 Total First Quarter Systems Department Not Costs Allocated Hardware and other capacityrelated costs Software development Computer-related operations Input-output-related operations (1) $150,000 × .25 (2) $150,000 × .50 (3) $150,000 × .20 (4) $150,000 × .05

..

(5) $30 × 450 (6) $30 × 1,800 (7) $30 × 1,600 (8) $30 × 400

$155,000 130,000 187,000 78,000 $550,000

Total

$ 5,000 $150,000 2,500 127,500 3,000 184,000 (1,000) 79,000 $ 9,500 $540,500

(9) $200 × 540 (10) $200 × 194 (11) $200 × 126 (12) $200 × 60

Records

Allocated Department Claims Finance

Outside

$ 37,500 (1) $ 75,000 (2) $ 30,000 (3) $ 7,500 (4) 13,500 (5) 54,000 (6) 48,000 (7) 12,000 (8) 108,000 (9) 38,800 (10) 25,200 (11) 12,000 (12) 15,400 (13) 55,400 (14) 4,100 (15) 4,100 (16) $174,400 $223,200 $107,300 $35,600

(13) $10 × 1,540 (14) $10 × 5,540 (15) $10 × 410 (16) $10 × 410

556


12-53 (15-20 min.) Amounts are in millions.

Region Revenues North America $ 7,578 Western Europe 3,810 Central & Eastern Europe 1,031 Greater China 2,060 Japan 766 Emerging Economies 2,736 Total $17,981

Earnings Before Corporate Expenses $1,750 721 233 777 114 688 $4,284

Allocation of Corporate Expenses $325 163 44 88 33 117 $771

Earnings After Allocation of Corporate Expenses $1,425 558 189 689 81 571 $3,512

Allocating corporate expenses based on revenues is an allocation based on ability-to-bear, not cause-and-effect. Such allocations are not generally useful for decisions. If the allocation had made the earnings after allocation of corporate expenses negative, that still would not indicate that Nike would be better without selling to the region.

12-54 (40-35 min.) For the solution to this Excel Application Exercise, follow the stepby-step instructions provided in the textbook chapter. 1.

Residential = $305,500 Commercial = $504,500

2.

Residential = $305,250

3.

Commercial = $504,750

4. In this case there is little difference between the two methods, so the less costly direct method may be preferred. In general, if there is a difference in the allocations, the step-down method tends to be more accurate.

..

557


12-55

(100 min. or more) The purposes of this exercise are to conduct library research in the current management accounting literature and to gain a better understanding of activity-based costing and activity-based management. Students must find their own article on ABC or ABM, and this will test their skills with library searches. Using electronic search procedures is likely to be a time-saver, but names of journals are given so that someone could just browse the library holdings of one of the journals to find an appropriate article. Textbooks are limited in the space they can devote to stories about actual costaccounting systems. This exercise requires students to deal with real-world issues relating to ABC or ABM. All applications of ABC or ABM are not successful, either because it was not an appropriate techniques where applied or because of mistakes in implementation. Although the literature will be dominated by success stories (companies do not often advertise their failures), by looking at several companies who have implemented ABC or ABM, students should be able to make some of the generalizations called for in requirement 2. By sharing information among group members, students should get a broader perspective on ABC and ABM than they would get from reading a single article.

..

558


12-56 (30-40 min.) NOTE TO INSTRUCTOR: This solution is based on the web site as it was in late 2012. Be sure to examine the current web site before assigning this problem, as the information there may have changed. 1. Sears Holdings consists of Kmart and Sears stores. Sears and Kmart operate over 2,600 stores in the United States and Canada. The number of these companies listed in an area will be specific to the location of the school but in most cases both companies should operate close to the student. 2. In footnote 17 to the financial statement in the 10K report for 2011, the company reports revenue, operating costs and expenses, operating profits (loss), total assets, and capital expenditures for three segments: Kmart, Sears Domestic, and Sears Canada. All three segments had an operating loss in 2011, and the sum of the operating losses was $1,501 million. The operating loss on the 2011 income statement is also $1,501 million. Because these amounts are the same, for financial reporting purposes the company must allocate 100% of its operating expenses to segments – including 100% of the companywide operating expenses and 100% selling and administrative expenses. 3. The percent of selling and administrative costs allocated to segments with different cost-allocation bases is: Cost-Allocation Base Kmart Actual “bases” used 3,371 ÷ 10,664 = 32% Revenue 15,285 ÷ 41,567 = 37% Total assets 4,548 ÷ 21,381 = 21%

Sears Domestic 6,042 ÷ 10,664 = 57% 21,649 ÷ 41,567 = 52% 13,913 ÷ 21,381 = 65%

Sears Canada 1,251 ÷ 10,664 = 12% 4,633 ÷ 41,567 = 11% 2,920 ÷ 21,381 = 14%

Yes, allocations based on revenue and total assets differ from the actual bases used. Sears Holdings uses more than one cost-allocation base, so we have imputed the average allocation rate from the amount of the allocations given. Allocations based on revenue would allocate more selling and administrative costs to Kmart and less to Sears Domestic. Allocations based on total assets would be the opposite – Sears Domestic would be allocated more selling and administrative costs and Kmart would be allocated less. The allocations to Sears Canada would vary slightly, but not as much as those to Kmart and Sears Domestic.

..

559


CHAPTER 13 COVERAGE OF LEARNING OBJECTIVES

LEARNING OBJECTIVE LO1: Compute budgeted factory-overhead rates and apply factory overhead to production. LO2: Determine and use appropriate cost-allocation bases for overhead application to products and services. LO3: Use normalized variable- and fixed-overhead application rates and explain the disposition of overhead variances. LO4: Compare variable- and absorption-costing systems.

FUNDAMENTAL ASSIGN-MENT MATERIAL A1, B2, B4

B4

69, 74, 76

A2, B1, B4

8, 10, 42, 63, 66, 66

A3, A4, B3, B4

13, 35, 43, 43, 44, 44, 45, 45, 55, 56, 56, 57, 57, 58, 59, 64, 65 43, 43, 45, 45, 55, 56, 56, 57, 57, 58, 59, 64, 65 16, 24, 43, 43, 45, 45, 55, 56, 56, 57, 57, 58, 59, 64, 65 48, 49, 54, 54, 60

70, 72, 73

29, 36, 37, 46, 47, 47, 48, 49, 60, 61, 61, 62, 62, 67, 6

71, 71, 72, 73

LO5: Construct an income statement using the variablecosting approach.

B4

LO6: Construct an income statement using the absorption-costing approach. LO7: Distinguish between product-costing and planning-and-control purposes in accounting for variable and fixed costs. LO8: Compute the production-volume variance and show how it should appear in the income statement.

A4, B4

..

ADDITIONAL ASSIGNMENT MATERIAL 1, 33, 34, 38, 39, 39, 40, 40, 50, 51, 52, 53, 53, 60, 63 3, 32, 40, 41, 41, 50, 51, 52, 53, 53, 60

CASES, NIKE 10K, EXCEL, COLLAB. & INTERNET EXERCISES 69, 75, 76

B4

B4

70, 72, 73

70, 72, 73, 77

560


LO9: Reconcile variable- and absorption-costing operating income and explain why a company might prefer to use a variable-costing approach.

..

B4

31, 55, 56, 61, 64

70

561


CHAPTER 13 Accounting for Overhead Costs 13-A1 (15-20 min.) This is a solid basic problem concerning overhead application. 1.

Overhead rate = Budgeted overhead ÷ Appropriate cost driver Department A = $1,936,000 ÷ 352,000 = $5.50 per machine hour Department B = $1,020,600 ÷ 126,000 = $8.10 per direct-labor hour

2.

Department A = $5.50 × 3,520 Department B = $8.10 × 1,260 Total applied overhead

3. Direct material Direct labor Applied factory overhead . Totals

$19,360 10,206 $29,566 Dept. A

Dept. B

Total

$12,500 10,600 19,360 $42,460

$19,529 9,600 10,206 $39,335

$32,029 20,200 29,566 $81,795

Unit cost, $81,795 ÷ 105 4.

$

779.00

Students must be on guard to get their definitions clear. "Overapplied" essentially means that "actual" overhead is less than that absorbed by (applied to) the products worked on during the period. Computations follow:

Actual Applied, 304,000 × $5.50 and 117,000 × $8.10 Underapplied (overapplied)

..

Dept. A

Dept. B

Factory as a Whole

$1,390,000

$1,230,000

$2,620,000

1,672,000 $ (282,000)

947,700 $ 282,300

2,619,700 $ 300

562


13-A2 (15 min.) Note that the direct materials inventory is irrelevant. 1.

Overapplied overhead = $119,000 - $126,000 = $7,000

2.

Adjusted gross profit = $55,000 + $7,000 = $62,000

3.

Proration schedule:

Work in process Finished goods Cost of goods sold Totals

Unadjusted Balances

Proration of Overapplied Overhead

$ 72,000 99,000 505,000 $676,000

72/676 × $7,000 = 99/676 × $7,000 = 505/676 × $7,000 =

Adjusted Balances $ 746 1,025 5,229 $7,000

$ 71,254 97,975 499,771 $669,000

Adjusted gross profit = $55,000 + 5,229 = $60,229 4.

Underapplied overhead = $128,000 - $126,000 = $2,000 Adjusted gross profit = $55,000 - $2,000 = $53,000

13-A3 (15-20 min.) Gross margin and ending direct-materials inventories are irrelevant. 1. & 2. Direct materials used Direct labor Variable manufacturing overhead Fixed manufacturing overhead Total production costs

(1) Variable Costing $ 3,000 4,000 2,500 -$9,500

(2) Absorption Costing $ 3,000 4,000 2,500 5,000 $14,500

Ending inventories are .16 × total production costs*

$ 1,520

$ 2,320

* .16 = (2,500 units produced – 2,100 units sold) ÷ 2,500 3.

The $800 difference in ending inventories is accounted for by the 16% of the $5,000 fixed manufacturing overhead that is lodged in ending inventory under absorption costing. Operating income would be $800 lower under variable costing because all of the fixed manufacturing overhead is released to expense in the current period. (That is why the fixed cost is sometimes called a "period cost" by variable cost proponents; period costs are those that are totally released to expense in the current period rather than being inventoried.) Note also that the difference in operating income is a function of the change in inventory levels. It is not a function of the ending inventories alone. The change and the ending inventory just happen to be the same because the beginning inventory was zero. See the next problem.

..

563


13-A4 (20-30 min.) 1.

HOFFMAN COMPANY Absorption Costing Income Statement For the Year Ended December 31, 20X1 Sales Deduct cost of goods sold: Beginning inventory, 80 @ $12.50* Add: Absorption cost of goods manufactured, 1,220 units @ $12.50 Cost of goods available for sale Ending inventory, 50 @ $12.50 Cost of goods sold -- at standard Production volume variance (unfavorable) Adjusted cost of goods sold Gross margin Selling and administrative expenses ($450 + $300) Operating income

$18,750 $ 1,000 15,250 $16,250 (625) $15,625 875 ** (16,500) 2,250 (750) $ 1,500

*Fixed overhead rate: $5,145 ÷ 1,470 units = $3.50 per unit. Unit production cost: $9 + $3.50 = $12.50 **(1,470 - 1,220) × $3.50 = $875 underapplied 2.

Change in inventory units 80 - 50 = 30 decrease Fixed factory overhead rate is $3.50 Difference in operating income: 30 × $3.50 = $105 less under absorption costing

13-B1

(10-15 min.) Note that the direct materials inventory is irrelevant.

1.

Adjusted cost of goods sold is $276,000 plus $42,000 or $318,000.

2. (in thousands) Work in process Finished goods Cost of goods sold Totals

Unadjusted Balances $161 138 276 $575

Proration of Underapplied Overhead (161 ÷ 575) × $42 = 11.76 (138 ÷ 575) × $42 = 10.08 (276 ÷ 575) × $42 = 20.16 $42.00

Adjusted Balances $ 172.76 148.08 296.16 $617.00

Gross profit would be higher in requirement 2 by $42,000 - $20,160, or $21,840. Adjusted cost of goods sold would be $276,000 + $20,160 = $296,160 in requirement 2 but $276,000 + $42,000 = $318,000 in requirement 1. The lower cost of goods sold in requirement 2 would make gross profit higher. ..

564


13-B2 (15-20 min.) 1.

Overhead rate = Budgeted overhead ÷ Budgeted Cost Driver Level Pharmacy = $216,000 ÷ 80,000 = $2.70 per prescription Medical Records = $246,400 ÷ 44,000 = $5.60 per patient visit

2.

Pharmacy = $2.70 × 5 Medical records = $5.60 × 1 Total applied overhead

$13.50 5.60 $19.10

3.

Students must be on guard to get their definitions clear. "Overapplied" essentially means that "actual" overhead is less than that absorbed by (applied to) the products worked on during the period. Computations follow:

Actual Applied, 82,000 × $2.70 and 65,000 × $5.60 Underapplied (Overapplied) 13-B3 (10 min.) Production costs: Direct materials used Direct labor Variable manufacturing overhead Fixed manufacturing overhead Total Ending inventories, 7/23 of total production costs

..

Pharmacy

Medical Records

Total

$240,000

$305,000

$545,000

221,400 $ 18,600

364,000 $ (59,000)

585,400 $ (40,400)

(1) Absorption Costing

(2) Variable Costing

$ 3,900 4,400 400 3,000 $11,700

$3,900 4,400 400 $8,700

$ 3,561

$2,648

565


13-B4 (30-40 min.) 1.

DESK PC DIVISION Income Statement (Variable Costing) For the Year 20X1 (in thousands of dollars)

Sales (15,000 × $500) Opening inventory, at variable standard cost of $300 Add: Variable cost of goods manufactured Available for sale Deduct: Ending inventory, at variable standard cost of $300 Variable cost of goods sold, at standard Net variances for all variable costs, unfavorable Variable cost of goods sold, at actual Variable selling expenses, at 5% of dollar sales Total variable costs charged against sales Contribution margin Fixed factory overhead Fixed selling and administrative expenses Total fixed expenses Operating income

$7,500 $ 900 4,650 5,550 1,050 $4,500 18 4,518 375 4,893 2,607 1,560* 650 2,210 $ 397

*This can be shown in two lines, $1,500 budget plus $60 variance.

..

566


DESK PC DIVISION Income Statement (Absorption Costing) For the Year 20X1 (in thousands of dollars) Sales Opening inventory, at standard cost of $400 Add: Cost of goods manufactured, at standard Available for sale Deduct: Ending inventory, at standard Cost of goods sold, at standard Net variances for variable manufacturing costs, unfavorable Fixed factory overhead budget variance, unfavorable Production-volume variance, favorable Total variances Cost of goods sold, at actual Gross profit, at "actual" Selling and administrative expenses: Variable Fixed Operating income

$7,500 $1,200 6,200 7,400 1,400 6,000 $18 60 (50)* 28 6,028 1,472 375 650

1,025 $ 447

* Production-volume variance is $100 × (15,000 expected production volume – 15,500 actual production). 2.

The $50,000 difference in operating income is attributable to the 500-unit increase in inventory levels. This means that $50,000 of fixed factory overhead (500 units × fixed rate of $100) was held back in inventory under absorption costing, whereas all fixed overhead was released as expense under variable costing.

13-1 Factory overhead is applied to production through budgeted factory overhead rates. First, the budgeted overhead application rate is calculated by dividing the predicted factory overhead for the budget period with the predicted machine hours for that period. The factory overhead is then applied to a job by multiplying the budgeted overhead application rate with the actual quantity of allocation base used on that job. 13-2 No. In the past, most organizations have used only one cost-allocation base per department. However, the trend is toward using multiple cost-allocation bases. Whether more than one costallocation base is used is a cost/benefit issue. If most overhead costs are caused by a single cost driver, using that one cost driver for cost application is logical. If overhead costs are caused by multiple cost drivers, managers must compare the value of more accurate product costs versus the cost of a complex accounting system that uses multiple cost-allocation bases for overhead application.

..

567


13-3 There is no single cost-allocation base that is appropriate in all circumstances. Therefore, the principle that should be followed is to find a suitable cost-allocation base that best links the cause and effect. 13-4 Cost-allocation bases might include direct labor cost, direct labor hours, direct material cost, total direct cost, machine hours, number of batches, number of engineering hours used, number of change orders, etc. 13-5 The comparison of actual overhead costs to budgeted overhead costs is part of the control process. It tells managers when the actual results differ from what was expected. 13-6 Incurred overhead will differ from applied overhead in much the same way as any estimate will differ from actual experience. Specific causes might be: variations in suppliers' prices; inefficiencies in production (excessive down-time, for example); failure of sales to materialize; failure to meet production quotas; and unexpected increases in fixed overhead (increase in insurance rates, for example). They also can arise because of inaccurate overhead cost predictions. 13-7 No. Using "actual" overhead rates, unit costs will be lower as production volume increases and higher with low volume. The variable overhead rate will be approximately constant; the fixed overhead rate will vary inversely with volume. The two rates together form the total overhead rate. 13-8 In a normal costing system, the cost of the manufactured product is composed of the actual direct material, actual direct labor, and normal applied overhead. The normal applied overhead is the product of the pre-determined overhead application rate and actual quantity of allocation base used. 13-9 The best theoretical method of allocating underapplied or overapplied overhead is to disregard it completely and recompute an actual overhead rate based on actual costs incurred allocated over actual production units. Proration is usually a reasonable approximation to this theoretical ideal. 13-10 The objective of the proration approach of disposition of underapplied or overapplied overhead is to obtain as accurate an application of actual costs as possible. Thus, under this method, overapplied or underapplied overhead is assigned to work-in-progress goods and finished goods and the cost of goods sold in proportion to the magnitude of applied overhead in the ending balances of these accounts. 13-11 Variable costing expenses fixed manufacturing overhead immediately. Absorption costing applies fixed manufacturing overhead to inventory as a product cost until the goods are sold. Then the fixed manufacturing overhead is expensed as cost of goods sold. 13-12 The production-volume variance appears only on an absorption-costing income statement. 13-13 This is because it is believed that variable costing links manufacturing performance more closely with measures of that performance by removing the impact of changing inventory levels from financial results.

..

568


13-14 No. Variable costing means that all variable costs of manufacturing are inventoried. These include direct material, direct labor, and the overhead costs that are incurred in direct proportion to the volume of production, even though these costs may only indirectly affect the production process and thus are categorized as "overhead." 13-15 Fixed manufacturing overhead is considered a noninventoriable or period cost under variable costing but a product cost under absorption costing. 13-16 In any absorption costing income statement, fixed factory overhead appears in two places: as a part of the cost of goods sold and as a production-volume variance. A production-volume variance appears whenever the actual production deviates from the budgeted production used in computing the fixed overhead application rate. 13-17 The tax authorities and those in charge of the rules for financial reporting do not allow use of variable costing. Why? They believe it violates the matching principle. 13-18 Variable costing and cost-volume-profit analysis are both based on separate measurements of fixed and variable costs. Both focus on computation of the contribution margin, the difference between revenue and all variable costs. 13-19 The contribution margin is revenue less variable costs (including both variable manufacturing costs and variable selling and administrative costs). In contrast, gross margin is revenue less manufacturing costs (including both variable manufacturing costs and fixed manufacturing costs). 13-20 Fixed overhead is applied to product via a budgeted unit overhead rate multiplied by an actual cost-allocation base activity level such as machine hours or production units. 13-21 First, the unit product cost in absorption costing includes an allocation of fixed costs, while in variable costing it consists of only variable manufacturing costs. Second, fixed costs appear as a single line in a variable-costing statement, but they are in two places (part of product cost and as a production volume variance) in an absorption-costing statement. Finally, a variable-costing statement separates costs into fixed and variable components, while absorption-costing statements separate them into manufacturing and nonmanufacturing components. 13-22 This statement describes the treatment of fixed costs in an absorption-costing system. Production volume does not affect total fixed costs, but it does affect applied fixed costs, which are proportional to the units of production. 13-23 Yes. Only when actual production volume exactly equals the expected volume is the applied fixed manufacturing overhead equal to that budgeted. Although the exact equality is rare, most of the time the difference will not be great. 13-24 The expected cost-allocation base activity selected to determine the fixed-overhead rate is known as the denominator level.

..

569


13-25 Direct labor is a variable cost. The expected amount (i.e., flexible-budget amount) for a variable cost is the same as the amount allocated (or applied) to the product. There is no conflict between the budgeting and control purpose and the product-costing purpose. Therefore, no variance is caused by production volume differing from an expected volume. 13-26 No. Production-volume variances provide no information about the control of fixed manufacturing costs. Such variances arise solely because the actual production volume differs from the expected volume. 13-27 Yes. The unit fixed cost is inversely proportional to the denominator, expected units of production. 13-28 No. When the number of units sold exceeds the number produced, that is, when inventory decreases, variable-costing income exceeds absorption-costing income. 13-29 An adverse production volume variance results when the actual production is lower than the budgeted production. It may occur due to idle facilities caused by disappointing sales, poor production scheduling, unusual machine breakdowns, shortages of skilled workers, strikes, etc. 13-30 Variable- and absorption-costing incomes differ only when the level of inventory changes. Furthermore, the amount of the difference in income is proportional to the change in inventory. When inventories are small, changes in inventory are also generally small. Therefore, companies without much inventory will report nearly the same operating income with variable costing as with absorption costing. 13-31 No, all variances except the production-volume variance appear in both variable-costing system and absorption-costing system. 13-32 A strong relationship between the factory overhead incurred and the cost-allocation base is the best available indication of a cause-and-effect relationship. That is, the more of the cost-allocation base that is used, the higher the actual overhead incurred. It is important to consider the time period involved. Some overhead costs, equipment for example, have a weak or no relationship to machine hours used in the short run but a strong relationship to providing the capability to operate machines over extended time periods. Such costs are often called capacity costs. 13-33 No. Some service firms trace only direct-labor costs to individual jobs. However, with advances in computer technology and because competition causes a need for better cost information about specific services, jobs, or customers, more service firms are tracing additional costs to jobs. The more costs that are traced to jobs instead of being allocated, the more accurate are the job costs. 13-34 Fixed costs are difficult to deal with because revenue must be enough to cover fixed as well as variable costs before a company makes a profit, but fixed costs do not vary with the volume of production. Suppose that a company views a product cost as the amount that needs to be received in revenue in the long-run to be profitable. Such companies often want to assign fixed costs to the products. Remember that the separation of fixed and variable costs is inherently a short-run phenomenon. In the long run, where capacity can be altered and all commitments can be renegotiated,

..

570


nearly all costs are variable. Thus, knowing the long-run product cost can be important for strategic decisions. However, the long-run product cost may differ from the accounting cost that includes an allocation of fixed costs, because many fixed costs (for example, depreciation) represent historical costs that may differ significantly from the future cost needed to provide the same services. Thus, it makes sense to include fixed costs in a prediction of long-run product costs, but measuring those fixed costs by allocating fixed historical costs to the products may not provide an accurate measure. Yet, it may be better than the alternatives. 13-35 Most pricing and promotion decisions are short-run decisions. They can be reversed if conditions in the marketplace change. Thus, the decisions are unlikely to affect fixed costs unless they increase or decrease demand enough that the volume moves outside the relevant range. The immediate effect on the company’s profits is measured by the contribution margin -- revenue less the variable cost. If the pricing or promotion decision has long-term effects as well, for example provides a level of market penetration that will affect future sales levels, then the short-term impact must be compared to the long-term effects. Separating long-term effects from short-term effects is often useful. We can predict short-term effects from current accounting data, while the long-term effects are often poorly measured by data directly from the accounting system. Allocation of fixed costs to products may give information about long-term effects, but it is usually better to separately estimate these effects. 13-36 A production-volume variance arises when production exceeds or falls short of the volume used to set the fixed overhead rate, often the expected volume. However, unlike the sales-volume variance, the production-volume variance does not directly measure the economic consequences of the production volume. If production falls 10% short of the predicted volume, the production-volume variance merely indicates that 10% of the fixed costs were not applied to the products produced. It does not indicate that the company incurs 10% more fixed costs than planned. A sales-volume variance of $10,000 means that if sales had met the target, the company would have been $10,000 better off. If the production-volume variance was $10,000, it does not mean that production of the additional products would have generated $10,000 of benefit to the company. 13-37 Some companies apply all costs from various stages of the value chain to their products or services. This gives a measure of all of the costs that have to be covered by revenues during the product’s life cycle. It is most useful for strategic decisions – decisions relating to long-run commitments to product lines and facilities and establishing product mix and pricing policies. Such allocations are less useful for tactical decisions – those relating to short-term sales and production effects. There is no single measure of cost that is appropriate for all decisions. Rather, cost measures must be tailored to reflect the decisions for which they are being used. 13-38 (10-15 min.) Total budgeted amount of cost-allocation base = $200,000 ÷ $4 = 50,000 machine hours Total applied overhead = $4 × 62,000 machine hours = $248,000 Underapplied overhead = actual overhead incurred – applied overhead

..

571


= $260,000 - $248,000 = $12,000 Actual costs must be reported in the income statement. So the $12,000 underapplied overhead must be added to cost of goods sold. 13-39 (10-15 min.) Budgeted overhead application rate = Total budgeted factory overhead ÷ Total budgeted amount of cost driver = £252,500 ÷ 25,000 Machine hours = £10.10 per machine hour Applied overhead = Overhead rate × Actual number of driver units Actual number of machine hours = £308,050 ÷ £10.10 = 30,500 Machine hours Actual costs must be reported in the income statement. So, the £308,050 – £288,500 = £19,550 overapplied overhead must be deducted from the cost of goods sold. 13-40

(15-20 min.)

A major lesson of this exercise is the distinction between budgeted, actual, and applied overhead. Case 2 is more challenging, but it forces the student to learn the basic relationships. 1.

2.

..

c.

€323,000 ÷ €212,500 = 152% of direct-labor cost

f.

1.52 × €277,500 = €421,800

g.

€425,000 - €421,800 = €3,200 underapplied

b.

€207,500 ÷ 1.25 = €166,000

f.

€210,000 - €15,000 = €195,000

d.

€195,000 ÷ 1.25 = €156,000

572


13-41

(10-15 min.) (in thousands) Case 1 Case 2

1.

2.

3.

Applied overhead:

100 × $4.10 = 140 × $4.10 =

Overhead incurred: $58 + $32 + $72 + $274 = $86 + $66 + $98 + $414 = Underapplied overhead: $436 – $410 = Underapplied overhead: $664 – $574 =

$410 $574

436 664 26 90

Note the irrelevant items: Sales commissions are selling expenses. Depreciation of finished goods warehouse is also a selling expense because the manufacturing processing has been completed. Cost of goods sold is an overall figure of no use in this problem. Direct-labor cost and direct-material cost are not pertinent either.

..

573


13-42 (10-15 min.) Overhead is overapplied by $456,000 - $416,000 = $40,000. First Way Unadjusted cost of goods sold Deduct: Overapplied overhead Adjusted cost of goods sold

$396,000 40,000 $356,000

Factory department overhead control 40,000 Cost of goods sold Second Way Cost of goods sold Work in process Finished goods Totals

Unadjusted $396,000 231,000 198,000 $825,000

Factory department overhead control Cost of goods sold Work in process Finished goods

40,000

Proration of Overapplied Overhead 396/825 × $40,000 = $19,200 231/825 × 40,000 = 11,200 198/825 × 40,000 = 9,600 $40,000

Adjusted $376,800 219,800 188,400 $785,000

40,000

Cost of Goods Sold with Proration: Unadjusted cost of goods sold Deduct: Overapplied overhead for cost of goods sold Adjusted cost of goods sold

19,200 11,200 9,600 $396,000 19,200 $376,800

Cost of goods sold would be $376,800 - $356,000 (or $40,000 - $19,200) = $20,800 lower (and gross profit higher) under the first way, that is with no proration.

..

574


13-43

(20 min.)

1. This exercise helps students obtain a fundamental look at the essential conceptual differences between the two inventory methods. Amounts are in thousands of euros. Absorption Costing Balance Sheets Income Statements January 1, 2018 Cash 306 Capital Stock 306 December 31, 2018 Year 2018 Cash, 306-112 194 Capital stock 306 None Inventory, +112 112 306 December 31, 2019 Year 2019 Cash, 194+88 282 Capital stock 306 Revenue 88 Inventory, 112-56 56 Retained earn., +32 32 Cost of Gds. Sold 56 338 338 Net income 32 December 31, 2020 Year 2020 Cash, 282+88 370 Capital stock 306 Revenue 88 Inventory, 56-56 0 Retained earn., 32+32 64 Cost of Gds. Sold 56 370 370 Net income 32 2. January 1, 2018 Cash December 31, 2018 Cash, 306-112 Inventory, +74 December 31, 2019 Cash, 194+88 Inventory, 74-37 December 31, 2020 Cash, 282+88 Inventory, 37-37

Variable (Direct) Costing Balance Sheets 306

Capital Stock

Income Statements 306

194 74 268

Capital stock Retained earn., -38

306 - 38 268

282 37 319

Capital stock Retained earn.,-38+51

306 13 319

370 0 370

Capital stock Retained earn., 13+51

306 64 370

Year 2018 Revenue Cost of Gds. Sold Net loss Year 2019 Revenue Cost of Gds. Sold Net income Year 2020 Revenue Cost of Gds. Sold Net income

0 38 -38 88 37 51 88 37 51

Students can discuss how the income statements differ for each year. As inventory levels build (2018), the fixed overhead is charged to expense under variable costing and lodged in inventory under absorption costing, so the income under variable costing is less. But as inventory levels decline, that fixed overhead is released under absorption costing so that the income under absorption costing is less than that under variable costing in such years. ..

575


13-44

(15 min.)

1.

Variable-costing operating income equals absorption-costing operating income whenever the inventory level is unchanged (the beginning inventory equals the ending inventory). No change in the inventory level implies that the units produced equal those sold, as in 2018.

2.

The absorption-costing operating income exceeds the variable-costing operating income when inventory levels increase, because (under absorption costing) some fixed costs are applied to the units in the enlarged inventory. Units produced exceed units sold in 2017 and 2019.

3.

Repeat the idea in part (1), now considering the four-year total operating income, which is the same (¥2,472,000) under both variable and absorption costing. Thus, the beginning inventory in 2017 (0 units) equals the ending inventory in 2020 (0 units), or ¥0.

4.

2020’s variable-costing operating income exceeds the absorption-costing operating income by the amount of fixed costs borne by the decreased inventory (¥206,000). At ¥412.00 per unit, the units sold exceed the units produced by 500.

13-45

(10-15 min.)

1.

Variable manufacturing cost per unit = €248,000 ÷ 31,000 = €8.00 Assume that variable nonmanufacturing costs vary with sales units. Then, variable nonmanufacturing cost per unit = €22,000 ÷ 22,000 = €1.00 Operating income = (22,000 × €19) – (22,000 × €8.00) – (22,000 × €1.00) – €108,000 – €35,600 = €418,000 – (€176,000 + €22,000 + €143,600) = €76,400

2.

a.

(31,000 – 22,000) × [€8.00 + (€108,000 ÷ 30,000)] = 9,000 units × €11.6 unit cost = €104,400

b.

Total costs incurred = €248,000 + €108,000 + €22,000 + €35,600 = €413,600 Operating income = Sales – (Total costs – Costs in inventory) = €418,000 – (€413,600 – €104,400) = €108,800 or:

€76,400 + ( €3.6 × 9,000) = €108,800 From part 1

..

Fixed cost in inventory 576


13-46

(5-10 min.)

This exercise requires sorting the relevant information from the irrelevant. Computing the production-volume variance requires knowledge of the fixed-overhead rate: Fixed-overhead rate = ¥ 24,192,000 ÷ 5,600 units = ¥ 4,320 per unit The actual overhead costs are irrelevant. In addition to the fixed-overhead rate, the only items needed are expected and actual production volume: Production-volume variance = (actual volume - expected volume) × fixed-overhead rate = (5,700 – 5,600) × ¥ 4,320 = 100 × ¥ 4,320 = ¥ 432,000 favorable 13-47

(10-15 min.)

1.

(a) (b) (c)

2.

Fixed costs charged by variable costing Fixed costs charged by absorption costing: In cost of goods sold, £7 × 16,500 Production-volume variance, favorable Total fixed costs Difference in fixed costs

£7 × 15,750 = £110,250 £7 × 2,250 = £15,750 F £27,000 – (£7 × 1,500) = £16,500 £110,250 £115,500 –15,750 99,750 £ 10,500

Because the fixed costs are £10,500 less under absorption costing, the operating income is £10,500 greater under absorption costing than under variable costing.

..

577


13-48

(15 min.) Variances in dollars: Flexible-budget variance

5,000 U

Fixed Variable

800 U 1 4,200 U 2

Production-volume variance

200 U

Fixed Variable

200 U 3 NA

Spending variance

4,400 U

Fixed Variable

800 U 4 3,600 U 5

Efficiency variance

600 U

Fixed Variable

NA 600 U 6

NA = not applicable 1. $12,800 - $12,000 2. $14,400 - $10,200 3. $12,000 - $11,800 4. $12,800 - $12,000 5. $14,400 - $10,800 6. $10,800 - $10,200

..

578


13-49

(15-20 min.) Note that the budget for standard hours allowed for actual output achieved for variable overhead must be $41,000, the same as applied. In contrast, the budget for actual output achieved for fixed overhead must be $70,000, the same as the budget for actual hours of input. Variances are in dollars. The answers follow: Total Overhead Variable Fixed 1. Spending variance 400F 3,100U 3,500F 2. Efficiency variance 4,000U 4,000U NA 3. Production-volume variance 5,200U NA 5,200U 4. Flexible-budget variance 3,600U 7,100U 3,500F 5. Underapplied overhead 8,800U 7,100U 1,700U NA = not applicable These relationships could be presented in the same way as in Exhibit 13-10:

Variable

Cost Incurred: Actual Inputs × Actual Prices 48,100

Predicted Overhead Based on Actual Driver Use × Standard Prices 45,000

Spending, 3,100U

Flexible Budget: Standard Driver Use Allowed for Output Achieved × Standard Prices 41,000

Efficiency, 4,000U

Prod.-Volume, NA

Flexible-budget variance, 7,100U Underapplied overhead, 7,100U

Fixed

Cost Incurred: Actual Inputs × Actual Prices

Predicted Overhead Based on Actual Driver Use × Standard Prices

66,500

70,000

70,000 Efficiency, NA

Flexible-budget variance, 3,500F Underapplied overhead, 1,700U

..

NA

Flexible Budget: Standard Driver Use Allowed for Output Achieved × Standard Prices

Spending, 3,500F

Product Costing: Applied Overhead 41,000

Product Costing: Applied Overhead 64,800

Prod.-volume, 5,200U Prod.-volume 5,200U. ___

579


13-50 (15-20 min.) (Note that final dollar amounts are rounded to the nearest dollar.) 1.

Overhead rate = ($277,800 + $103,200) ÷ 69,450 machine hours = $5.48596 per machine hour

2. 3.

Overhead applied = 70,000 Machine Hours × $5.48596 per machine hour = $384,017 The overhead is overapplied by $384,017 – $381,000 = $3,017. This is because the amount of machine hours used exceeds the budgeted amount by 70,000 – 69,450 = 550 and 550 × $5.48596 = $3,017. When separate overhead application rates are used, the total applied overhead is $375,000 as shown on page 535 of the text. This is $6,000 less than budgeted. The table below compares the departmental application to the factory-wide application.

Budget Applied Over (Under) Applied

Two Department Overhead Rates Machining Assembly Total $277,800 $103,200 $381,000 280,000 95,000 375,000 $ 2,200 $ (8,200) $ (6,000)

Single Factory-Wide Overhead Rate $381,000 384,017 $ 3,017

13-51 (15-20 min.) (Note that final dollar amounts are rounded to the nearest dollar.) 1.

Overhead rate = ($277,800 + $103,200) ÷ $206,400 Direct Labor Dollar = $1.84593 per Direct labor Dollar

2.

Overhead applied = $190,000 Direct Labor Dollars × $1.84593 = $350,727

3. The overhead is underapplied by $381,000 - $350,727 = $30,273. This is because the direct labor cost is less than the budgeted amount by $206,400 – $190,000 = $16,400 and $16,400 × $1.84593 = $30,273. When separate overhead application rates are used, the total applied overhead is $375,000 as shown on page 535 of the text. This is $6,000 less than budgeted. The table below compares the departmental application to the factory-wide application.

Budget Applied Over (Under)

..

Two Department Overhead Rates Machining Assembly Total $277,800 $103,200 $381,000 280,000 95,000 375,000 $ 2,200 $ (8,200) $ (6,000)

Single Factory-Wide Overhead Rate $381,000 350,727 $ (30,273)

580


13-52

(20-35 min.)

1.

$10,000,000 ÷ $5,000,000 = 200% of direct labor

2.

($10,000,000 - $3,000,000) ÷ $5,000,000 = 140% of direct labor ($10,000,000 - $3,000,000) ÷ ($5,000,000 + $3,000,000) = $7,000,000 ÷ $8,000,000 = 87.5% of total direct costs

3.

4.

Direct labor Applied overhead @ 200% Total costs

Engagement Eagledale First Valley $15,000 $15,000 30,000 30,000 $45,000 $45,000

Total direct costs Applied overhead @ 140% of $15,000 Total costs

$25,000 21,000 $46,000

$21,000 21,000 $42,000

Total direct costs Applied overhead @ 87.5% Total costs

$25,000 21,875 $46,875

$21,000 18,375 $39,375

The billings would differ significantly: Engagement Eagledale First Valley

5.

Method 1: Total costs Total billings @ 130%

$45,000 $58,500

$45,000 $58,500

Method 2: Total costs Total billings @ 130%

$46,000 $59,800

$42,000 $54,600

Method 3: Total costs Total billings @ 130%

$46,875 $60,938

$39,375 $51,188

The first method is inferior to the other two because the latter give more accurate measures of how specific jobs cause increases in costs. In general, the more costs that are directly charged to jobs, the more accurate the picture of where the money is really spent. As between the other two methods, the answer depends on what causes the indirect costs to rise. If direct labor is the dominant cause, then the 140% rate is better. If the increases in indirect costs are more closely related to increases in all direct costs, then the 87.5% rate is preferable. Additional studies of how indirect costs behave would be necessary to answer this question.

..

581


13-53 (15 min.) 1.

If the other departments are indeed providing services to the water department, it is certainly appropriate to include the cost of these services in the water department’s budget and to have them paid for by the water customers. Charging administrative overhead is not a ruse; it is a real cost of providing water services. However, it is not clear from the case whether the administrative overhead allocation is accurately measured. It appears that there is only one overhead pool and consequently only one cost-allocation base used for allocation. It is likely that the services are quite varied and a single cost-allocation base may not be appropriate.

2.

It would be useful to identify the activities involved when other departments provide services to the water department. If it is not too expensive, it would be worthwhile to measure each type of service and charge the water department only the cost of those services that are actually used. In essence, it would be good to directly (physically) trace as many costs to the department as possible, charging directly for the services. At a minimum, using multiple cost pools and cost-allocation bases for allocating diverse costs to the water department should be considered.

13-54 (15-25 min.) This problem is intended to highlight the distinction and relation between accounting for control and accounting for product costing. 1.

First six months: (52,000 × ₹400.00) – ₹21,120,000 = ₹320,000, underapplied Last six months: (42,000 × ₹400.00) – ₹19,120,000 = ₹2,320,000, underapplied

2.

Overhead rate: Fixed, ₹28,000,000 ÷ 100,000 DLH ₹280 per DLH Variable, ₹12,000,000 ÷ 100,000 DLH 120 per DLH Overall rate ₹400 (a)

In the first period, direct-labor hours used exceeded half of the year’s total budget (50,000) by 2,000. This makes the fixed overhead overapplied by 2,000 × ₹280 = ₹560,000. The actual variable overhead is ₹21,120,000 – (50% × ₹28,000,000) = ₹7,120,000. The variable overhead applied is 52,000 × ₹120 = ₹6,240,000, so it is underapplied by ₹7,120,000 – ₹6,240,000 = ₹880,000. Therefore, the total overhead is underapplied by ₹880,000 – ₹560,000 = ₹320,000.

(b) In the last period, the direct-labor hours used were less than half the year’s total budget by 8,000. The fixed overhead is thus underapplied by 8,000 × ₹280 = ₹2,240,000. The actual variable overhead is ₹19,120,000 – (50% × ₹28,000,000) = ₹5,120,000. The variable overhead applied is 42,000 × ₹120 = ₹5,040,000, so it is underapplied by ₹5,120,000 – ₹5,040,000 = ₹80,000. Therefore, the total overhead is underapplied by ₹2,240,000 + ₹80,000 = ₹2,320,000.

..

582


13-55 (35-45 min.) This is an excellent problem for presentation in class. It is less satisfactory as a homework assignment because students tend to make the problem harder than it really is. 1.

One way to present the problem in class is to begin with 9 columns on the board or screen. Each column lists the sales and production quantities. Provide six rows for the variablecosting statement and seven rows for the absorption-costing statement, labeled as in the format provided in the problem, there are 18 "income statements" to be completed. Ask a different student to complete each statement. Patterns soon become clear, and students fill in the statements quickly. If students fail to recognize some patterns, you can prod them with discussion of the patterns as the statements are completed. The completed statements follow: Variable costing (in thousands of dollars)

(1) (2) (3) (4) (5) (6) (7) (8) (9) Revenue 300 400 500 400 500 600 500 600 700 Cost of goods sold (120) (160) (200) (160) (200) (240) (200) (240) (280) Contribution margin 180 240 300 240 300 360 300 360 420 Fixed mfg costs (150) (150) (150) (150) (150) (150) (150) (150) (150) Fixed sell. & admin. exp. (30) (30) (30) (30) (30) (30) (30) (30) (30) Operating income 0 60 120 60 120 180 120 180 240 Absorption costing (in thousands of dollars) Revenue Cost of goods sold Gross profit at standard Favorable (Unfavorable) production-volume var. Gross profit at "actual" Selling and admin. exp. Operating income

(1) (2) (3) (4) (5) (6) (7) (8) (9) 300 400 500 400 500 600 500 600 700 (210) (280) (350) (280) (350) (420) (350) (420) (490) 90 120 150 120 150 180 150 180 210 (30) 60 (30) 30

(30) 90 (30) 60

(30) 120 (30) 90

0 120 (30) 90

0 150 (30) 120

0 180 (30) 150

30 180 (30) 150

30 210 (30) 180

30 240 (30) 210

The following points are key to rapid completion of the statements: (a) (b) (c) (d)

Cost of goods sold under variable costing is variable cost ($8) times units sold. Fixed costs are $150,000 and $30,000 on each variable-costing statement. Cost of goods sold under absorption costing is full-absorption cost ($14) times units sold. No separate row for fixed manufacturing cost appears on an absorption-costing statement. Fixed manufacturing costs are included in cost of goods sold and in the production-volume variance. (e) Production-volume variance is production units less expected unit volume times fixed manufacturing cost per unit ($6). (f) Selling and administrative expenses are not inventoried; even on an absorption-costing statement the $30,000 is charged each period.

..

583


Patterns of operating income are discussed in question 2. If the student did not prepare the problem as a homework assignment, but you use it for class discussion, you might list the following information on the board before proceeding to the statements: Sales price = $20 per unit Variable cost = $8 per unit Fixed manufacturing cost = $150,000 per year Fixed-overhead rate = $150,000 ÷ 25,000 units = $6 per unit Full cost = $8 + $6 = $14 Fixed selling and administrative cost = $30,000 per year 2.

(a) Variable-costing income is greater than absorption-costing income when sales exceed production: (3), (6), and (9). Variable-costing income is lower than absorption-costing income when production exceeds sales: (1), (4), and (7). Variable-costing income equals absorption-costing income when production equals sales: (2), (5), and (8). (b) Production-volume variance is unfavorable when expected volume exceeds actual volume: (1), (2), and (3). It is favorable when actual volume exceeds expected volume: (7), (8), and (9). (c) Each additional unit sold adds $20 - $8 = $12 to profit under variable costing and $20 $14 = $6 under absorption costing. For example, compare (1) and (2). Production is 20,000 units in each case, but sales are 5,000 units greater in (2). Operating income is $60,000 greater in (2) than in (1) under variable costing and $30,000 greater under absorption costing. $60,000 ÷ 5,000 = $12 per unit for variable costing, and $30,000 ÷ 5,000 = $6 per unit for absorption costing. (d) Producing an additional unit does not affect operating income under variable costing. Compare, for example, (2) and (4). But under absorption costing, production of one unit increases profit by $6. Again, compare (2) and (4). The only difference is production of 5,000 additional units in (4) and operating income is $30,000 higher: $30,000 ÷ 5,000 = $6 per unit. (e) Variable costing provides a better measure of performance. Why? Because differences in operating income arise from differences in sales, not production, under variable costing.

..

584


13-56

(25-35 min.) Please allow ample time for classroom discussion.

1.

Comments on the Following Statements The accounting for fixed overhead in absorption costing is affected primarily by what expected production volume is selected as the base (the denominator) for applying fixed overhead to the product. In this case, is 6,800,000 liters per year, 13,600,000 liters per year, or some other activity level the most appropriate base? The above possibilities are usually outlined on the board and the students are asked to indicate by vote how many used one version of absorption costing versus another. Incidentally, the discussion tends to move more clearly if variable-costing statements are discussed first, because there is little disagreement regarding the computations under variable costing.

Variable Costing (in thousands of dollars) 2019 2020 Together Sales (and contribution margin) 1,122 1,122 2,244 Fixed costs 980 980 1,960 Net income 142 142 284 Absorption Costing (in thousands of dollars) Option One* Option Two** 2019 2020 Together 2019 2020 Together 1,122 1,122 2,244 1,122 1,122 2,244

Sales Less cost of goods sold: Beginning inventory Cost of goods manufactured Cost of goods available for sale Ending inventory Cost of goods sold – at normal cost Under applied overhead – loss from idle capacity Over applied overhead – gain from over-utilization Other expenses Total charges Net income (loss)

750 750 375

375 375 -

750 750 -

1,500 1,500 750

750 750 -

1,500 1,500 -

375

375

750

750

750

1,500

-

750

750

-

750

750

230 605 517

230 1,355 (233)

460 1,960 284

(750) 230 230 230 1,730 892 (608)

(750) 460 1,960 284

* $750,000 ÷ 13,600,000 liters as “normal capacity” = $.055 per liters ** $750,000 ÷ 6,800,000 liters as “normal capacity” = $.11 per liters

..

585


2.

Break-even point = Fixed expenses ÷ Contribution margin per liter = $980,000 ÷ $.165 = 5,939,394 liters If the company would sell 860,606 fewer liters per year at $.165 each, it would just break even. Most students will say that the break-even point is 5,939,394 liters per year under both absorption and variable costing. The logical question to ask a student who answers 5,939,394 units for absorption costing is: “What profit do you show for 2019 under absorption costing?” If the student mentions negative profit, such as $(230,000), ask: “But you say your breakeven point is 5,939,394 liters. How can you show a loss on 6,800,000 liters sold during 2020?” The answer to the break-even point dilemma is that net income is affected by both sales and production under absorption costing. The variable-costing approach dovetails precisely with the cost-volume-profit analysis that the students learned earlier, but absorption costing does not unless some special assumption is made regarding inventory changes. The latter usually assumes that all production for a given period is sold—that no inventory changes exist.

3.

Absorption costing: Either $375,000 or $750,000 at the end of 2019 and zero at the end of 2020. Variable costing: Zero at all times. This is a major criticism of variable costing and focuses on the issue of the definition of an asset. Supporters of variable costing answer that zero is the correct inventory value because the existence of the inventory does not save any future cost.

4.

Comments should include the following: (a) The central issue is the timing of release of fixed factory overhead to expense. (b) Variable costing dovetails exactly with general break-even analysis, while absorption costing does not. (c) Variable costing rests on a simple basic assumption that is easy to understand, while the results under the same set of facts can differ considerably when absorption costing is applied. (d) Variable cost proponents would inventory the units at zero cost because they believe no costs should be carried forward to the future if they cannot obviate a future cost incurrence, while absorption-costing adherents view potential cost recovery as the criterion for carrying costs as assets.

..

586


13-57

(25-30 min.)

1. Variable Costing (in thousands of dollars) 2019 1,122 238 884 742 142

Sales Variable cost of sales @ $.035 per liter Contribution margin Fixed costs Net income (loss)

2020 1,122 238 884 742 142

Together 2,244 476 1,768 1,484 284

Absorption Costing (in thousands of dollars) Option One* Option Two** 2019 2020 Together 2019 2020 Together 1,122 1,122 2,244 1,122 1,122 2,244

Sales Less cost of goods sold: Beginning inventory Cost of goods manufactured Cost of goods available for sale Ending inventory Cost of goods sold – at normal cost Under applied overhead – loss from idle capacity Over applied overhead – gain from over-utilization Other expenses Total charges Net income (loss)

988 988 494

494 494 -

988 988 -

1,500 1,500 750

750 750 -

1,500 1,500 -

494

494

988

750

750

1,500

-

512

512

-

512

512

230 724 398

230 1,236 (114)

460 1,960 284

(512) 230 230 468 1,492 654 (370)

(512) 460 1,960 284

*Variable cost per unit, $.035 + Fixed costs per unit, ($512,000 ÷ 13,600,000 liters) = $.03765 **Variable cost per unit, $.035 + Fixed costs per unit, ($512,000 ÷ 6,800,000 liters) = $.0753 2. Inventory: December 31, 2019 December 31, 2020

..

Variable Costing $238,000 -0-

Absorption Costing Option One Option Two $494,000 $750,000 -0-0-

587


13-58

(30-35 min.)

1.

Standard Variable Costing TWIN LAKES COMPANY Income Statement For the Year Ended December 31, 20X0 (1)

Sales -- at standard prices (13,600 × $79) $1,074,400 Opening inventory Add variable cost of goods manufactured at standard* 688,000 Variable cost of goods available for sale 688,000 Deduct ending inventory at standard variable cost: 3,600 × $40 144,000 Variable manufacturing cost of goods sold 544,000 Variable selling and administrative costs at budget of $11 per unit sold 149,600 Total variable costs 693,600 Contribution margin at standard 380,800 Fixed factory overhead at budget 146,000 Fixed selling and administrative costs 83,000 Total fixed costs 229,000 Operating income $ 151,800

(2) (1) - (2)

*17,200 × $40 = $688,000 Standard Absorption Costing TWIN LAKES COMPANY Income Statement For the Year Ending December 31, 20X0 Sales -- at standard prices (13,600 × $79) Opening inventory Add cost of goods manufactured at standarda Absorption cost of goods available for sale Deduct ending inventory at standard absorption cost: 3,600 × $50b Absorption cost of goods sold at standard Gross profit at standard Deduct selling and administrative costs: Variable at standard (13,600 × $11) Fixed at budget Total selling and administrative costs Operating income before variances Variances: Production-volume variancec Operating income

$1,074,400 860,000 860,000 180,000 680,000 394,400 149,600 83,000 232,600 161,800 26,000F $ 187,800

a

17,200 × $50 = $860,000. Variable cost of $40 + fixed factory overhead of $10 = $50. c (17,200 - 14,600 expected volume) × $10 = $26,000F. b

..

588


2.

If inventories increase, operating income will be higher under absorption costing: Difference in operating income = Change in inventory units × Fixed overhead rate = (3,600 - 0) × $10 = $36,000

13-59 (40-45 min.) This problem should not be assigned without also assigning problem13-58. 1.

Standard Variable Costing TWIN LAKES COMPANY Income Statement For the Year Ended December 31, 20X1

(1)

Sales -- at standard prices (15,600 × $79) $1,232,400 Opening inventory, at standard variable cost: 3,600 × $40 144,000 Add variable cost of goods manufactured at standard* 544,000 Variable cost of goods available for sale 688,000 Deduct ending inventory at standard variable cost: 1,600 × $40 64,000 Variable manufacturing cost of goods sold 624,000 Variable selling and administrative costs at budget of $11 per unit sold 171,600 (2) Total variable costs 795,600 (1)-(2) Contribution margin at standard 436,800 Fixed factory overhead at budget 146,000 Fixed selling and administrative costs 83,000 Total fixed costs 229,000 Operating income before variances 207,800 Variances: Selling prices (a) 20,000F Variable manufacturing costs (b) 73,000F Variable selling & administrative costs (c) 6,500F Fixed factory overhead (d) 2,500F Total variances 102,000F Operating income $ 309,800 *13,600 × $40 = $544,000. (a) $1,252,400 - (15,600 × $79) (b) $262,000 + $171000 + $38,000 - $544,000 (c) $165,100 - $171,600 (d) $143,500 - $146,000

..

589


Standard Absorption Costing TWIN LAKES COMPANY Income Statement For the Year Ending December 31, 20X1 Sales -- at standard prices (15,600 × $79) Opening inventory -- at standard absorption cost: 3,600 × $50a Add cost of goods manufactured at standardb Absorption cost of goods available for sale Deduct ending inventory at std. absorption cost: 1,600 × $50 a Absorption cost of goods sold at standard Gross profit at standard Deduct selling and administrative costs: Variable at standard (15,600 × $11) Fixed at budget Total selling and administrative costs Operating income before variances Variances: Selling prices Variable manufacturing costs Variable selling & administrative costs Fixed factory overhead: Budget variance Production-volume variancec Total variances Operating income

$1,232,400 180,000 680,000 860,000 80,000 780,000 452,400 171,600 83,000 254,600 197,800 20,000F 73,000F 6,500F 2,500F 10,000U 92,000F $ 289,800

a

Variable cost of $40 + fixed factory overhead of $10 = $50. 13,600 × $50 = $680,000. c (13,600 – 14,600) × $10 = $10,000 U. b

2. If inventories decrease, operating income is lower under absorption costing: Difference in operating income = Change in inventory units × Fixed overhead rate = (3,600 – 1,600) × $10 = $20,000

..

590


13-60

(30-40 min.) This is a straightforward problem that is quite informative for most students.

1.

$60,000 ÷ 7,500 hrs. = $8.00 per hour; $8.00 × 2 hours = $16.00 per unit

2. $50,000 a & b. Budgeted and applied lines are superimposed @ $5.00

10,000 Volume in Standard Hours

3. a. Budget $60,000

b. Applied at $8.00

7,500 10,000 Volume in Standard Hours

..

591


We suggest using graphs in class as you explain the subsequent parts. (A)

(B)*

Cost Incurred: Actual Inputs × Actual Prices

Predicted Overhead Based on Actual Driver Use × Standard Prices

(C) Flexible Budget: Standard Driver Use Allowed for Output Achieved × Standard Prices

(D)

Product Costing: Applied Overhead

4. Variable Overhead

Fixed Overhead

6,000×$5= $31,000 $30,000 $30,000 Flexible-budget variance, 1,000U No variance Under applied overhead, 1,000U Lump-sum 6,000×$8= $60,000 $48,000 (7,500-6,000)×$8= Prod.-volume var., Flexible-budget variance, 2,000U 12,000U_ Under applied overhead, 14,000U

$62,000

5. Variable Overhead

7,800×$5= $37,700 $39,000 $39,000 Flexible-budget variance, 1,300F No variance Over applied overhead, 1,300F

Fixed Overhead

Lump-sum 7,800×$8= $62,000 $60,000 $62,400 (7,800-7,500)×$8= Prod.-volume var., Flexible-budget variance, 2,000U 2,400F Over applied overhead, 400F

*Not given in this problem.

..

592


13-61 (15-20 min.) 1.

Variable manufacturing costs per unit, £24,480,000 ÷ 800,000 Fixed manufacturing costs per unit, £12,000,000 ÷ 800,000 Total manufacturing costs per unit

£30.60 15.00 £45.60

2.

Production-volume variance = (800,000 - 850,000) × £15 = £750,000 F

3.

Revenue (820,000 × £65.85) Cost of goods sold (820,000 × £45.60) Gross margin Production-volume variance Profit

£54,000,000 37,392,000 16,608,000 750,000 £17,358,000

4.

Revenue (820,000 × £65.85) Cost of goods sold (820,000 × £30.60) Contribution margin Fixed costs Profit

£54,000,000 25,092,000 28,908,000 12,000,000 £16,908,000

5.

Neither measure is inherently better. They give different signals about performance. The variable-costing profit is a better measure of the effect of sales on profit. It is not affected by production volumes. The absorption-costing profit is affected by both sales and production volumes. Because production volume exceeded sales volume, the profit is higher under absorption costing. From an incentive point of view, the variable-costing profit has the advantage of not being affected by production decisions. Under absorption costing, a manager can increase profits merely by producing more units, even if they are not sold.

..

593


13-62

(10 min.) Overhead rates: €1,460,000 ÷ 116,800 = €12.50 and €1,460,000 ÷ 146,000 = €10.00.

Cost Incurred: Actual Inputs × Actual Prices 1. Using Practical Capacity

2. Using Expected Activity

3.

Predicted Overhead Based on Actual Driver Use × Standard Prices

Flexible Budget: Standard Driver Use Allowed for Output Achieved × Standard Prices

Product Costing: Applied Overhead

122,000 × €10 €1,514,000 €1,460,000 = €1,220,000 (122,000 - 146,000) × €10.00 = Prod.-volume var. Flexible-budget variance, €54,000 U = €240,000 U Under applied overhead, €294,000 U __ 122,000 × €12.50 = €1,514,000 €1,460,000 €1,525,000 (122,000 - 116,800) × €12.50 = Prod.-volume var. Flexible-budget variance, €54,000 U = €65,000 F Over applied overhead, €11,000F _____

The flexible-budget variance for fixed overhead is the difference between the amount incurred and the budget figure. The budget figure is the same regardless of the actual level of activity and the rate used in applying fixed overhead. Consequently, the flexible-budget variances in parts (1) and (2) would be identical. The production-volume variance is the difference between fixed overhead applied and the budgeted amount. Fixed overhead applied is the product of standard hours allowed and the standard fixed-overhead rate. The difference arises from the different measures of volume used in computing the standard fixed-overhead rate, the rate being €10 per standard hour in part (1) while being €12.50 per standard hour in part (2). The difference in variances would be computed by multiplying the difference in rates by standard hours allowed: (€12.50 - €10) × 122,000 = €305,000. Note how the production-volume variance will never be favorable when based on practical capacity. Also note how different expected volume assumptions can produce radically different production-volume variances. Both accountants and managers should be aware of these possibilities when analyzing fixed overhead variances.

..

594


13-63

(25-30 min.)

1.

Total dollars and machine hours are in thousands:

Year 20X1 20X2 20X3

Base a $36,000 ÷ 2,500 = $14.40 $36,000 ÷ 2,850 = $12.63 $36,000 ÷ 3,000 = $12.00

Base b* $36,000 ÷ 2,637.5 = $13.65 $36,000 ÷ 2,637.5 = $13.65 $36,000 ÷ 2,637.5 = $13.65

Base c $36,000 ÷ 3,000 = $12.00 $36,000 ÷ 3,000 = $12.00 $36,000 ÷ 3,000 = $12.00

*Average volume (in thousands): (2,200 + 2,500 + 2,850 + 3,000) ÷ 4 = 2,637.5. 2.

Method a: This is the most popular method. It keeps the fixed-overhead rate constant over a year and generates no expected production-volume variance for the year in total. Method b: This method keeps the fixed-overhead rate constant over four years, but annual budgets must recognize an expected unfavorable production-volume variance in the first two years and an expected favorable production-volume variance in the last two. Method c: This method consistently generates unfavorable production-volume variances. The variances show the potential to spread fixed costs over a larger output whenever operations are below capacity.

3.

Most students will prefer Method a because the text indicates that it is most popular. However, a case can also be made for either of the other methods. For example, practical capacity has increased in popularity in recent years, principally because the Internal Revenue Service permits its use. For tax purposes, practical capacity permits faster write-off of factory overhead.

13-64

(30 min.)

1.

Because Leeds Tool Company uses absorption costing, the net income is influenced by both sales volume and production volume. Sales volume was increased in the November 30, 20X0 forecast, and at standard gross profit rates this would increase gross margin before taxes by £4,800. However, during the same period production volume was below the January 1, 20X0 forecast, causing an unplanned production-volume variance of £6,000. The productionvolume variance and the increased selling expenses (due to the 10% increase in sales) overshadowed the added gross margin from sales as shown below: Increased sales Increased cost of sales at standard Increased gross margin at standard Less: Increased selling expense Expected increase in earnings Production-volume variance Decrease in earnings

..

£15,600 10,800 £ 4,800 1,120 £ 3,680 6,000 U £(2,320)

595


2.

The basic cause of the lower forecast of profits is low production. If raw materials can be obtained, and if it is reasonable in light of expected future sales, Leeds Tool Company could schedule heavy production which would reduce the unfavorable production-volume variance.

3.

Leeds Tool Company could adopt variable costing. Then fixed manufacturing costs would be treated as period costs and would not be assigned to production. Consequently, earnings would not be affected by production volume but only by sales volume. The following statements are prepared on a variable-costing basis. LEEDS TOOL COMPANY Forecasts of Operating Results

Sales Variable costs: Manufacturing Selling expenses Total variable costs Contribution margin Fixed costs: Manufacturing Administration Total fixed costs Earnings before taxes

Forecasts as of January 1, 20X0 November 30, 20X0 £156,000 £171,600 £78,000 11,200**

£85,800* 12,320 89,200 £ 66,800

£30,000 20,000

98,120 £ 73,480 £30,000 20,000

50,000 £ 16,800

50,000 £ 23,480

*(171,600  156,000) × £78,000 = £85,800 **Because selling expenses increased by 10% when sales increased by 10%, they are variable costs. 4.

..

Variable costing would not be acceptable for financial reporting purposes because generally accepted accounting principles seem to require the allocation of some fixed manufacturing costs to inventory.

596


13-65 (30-40 min.) The unknowns, labeled a through f, are indicated in the following variablecosting and absorption-costing income statements. SCHLOSSER CO. Variable Costing Income Statement Sales, 150,000 units at $20.00 Variable expenses: Beginning inventory, 15,000 at $11.00 Cost of goods manufactured, 145,000* at $11.00 Available for sale Ending inventory, 10,000 at $11.00 Standard variable cost of sales Add variance in variable costs of production Variable manufacturing cost of sales Variable selling and administrative expenses Contribution margin Fixed expenses: Manufacturing Selling and administrative Operating income

$3,000,000 $ 165,000 (a) 1,595,000 $1,760,000 (110,000) $1,650,000 33,000 $1,683,000 450,000

(2,133,000) (b) $ 867,000

$ 165,000 650,000 (c) $

(815,000) 52,000

*Production = sales – decrease in inventory = 150,000 – 5,000 = 145,000 units SCHLOSSER CO. Absorption Costing Income Statement Sales, 150,000 units at $20.00 Cost of sales: Beginning inventory, 15,000 at $12.10* Cost of goods manufactured, 145,000 at $12.10 Available for sale Ending inventory, 10,000 at $12.10 Standard cost of sales Add unfavorable variances: Variable manufacturing costs Prod.-volume variance, 5,000 at $1.10 Gross margin Selling and administrative expenses: Variable, 150,000 at $3.00 Fixed Operating income

$3,000,000 $ 181,500 (d) 1,754,500 $1,936,000 (121,000) $1,815,000 33,000 5,500 $ 450,000 650,000

(1,853,500) (e) $1,146,500 (1,100,000) (f) $ 46,500

*$165,000 fixed overhead ÷ 150,000 units = $1.10; $1.10 + $11.00 = $12.10.

..

597


13-66 1.

(20-30 min.)

(in thousands) a. b. Revenue, 156,000 × $40 $ 6,240 $ 6,240 Standard cost of goods sold, 156,000 × ($18 + $8 + $4) (4,680) (4,680) Gross margin at standard $ 1,560 $ 1,560 Manufacturing variances (168) (134.4)* Operating income $ 1,392 $ 1,425.6

*Total variance = $168,000 U. Proration to sales = (156,000 ÷ 195,000) × $168,000 = $134,400 2.

a. b.

Ending inventory, Method (a): 39,000 units × $30 = $1,170,000 Ending inventory, Method (b): (39,000 units × $30) + (39,000 ÷ 195,000) × $168,000 = $1,170,000 + $33,600 = $1,203,600

Note that Method (b) provides $33,600 more operating income and $33,600 higher inventory because $33,600 of the variances was allocated to inventory rather than to expense. 3.

Supporters of Method (a) claim that variances arise from inefficiencies or efficiencies of the period and therefore should affect the current period’s income statement. They are not necessary costs of production and therefore should not be inventoried. Supporters of Method (b) claim that the income statement gives a better picture of actual costs using Method (b).

..

598


13-67

(35-45 min.)

Cost Incurred: Actual Inputs × Actual Prices 12,000 × $12.50

Predicted Overhead Based on Actual Driver Use × Expected Prices 12,000 × $13.00*

Flexible Budget: Standard Driver Use Allowed for Actual Output Achieved × Standard Prices 10,800 × $13.00*

Product Costing Applied to Product

Direct Labor:

= $150,000 = $156,000 = $140,400 Price variance, Usage variance, 12,000 hrs. × $.50 1,200 hrs. × $13 =$6,000F = $15,600U Flexible-budget variance, $9,600U

$140,400

Variable Overhead:

12,000 × $3.00* 10,800 × $3.00* $37,000* = $36,000 = $32,400 Efficiency variance, Spending variance, 1,200 hrs. × $3.00 $1,000U = $3,600U Flexible-budget variance, $4,600U Under applied overhead, $4,600U

Fixed Overhead:

Lump-sum Lump-sum 10,800 × $3.30** $38,000* $39,600 $39,600 = $35,640 Spending variance, $1,600F No variance Prod.-Vol. Var., Flexible-budget variance, $1,600F* $3,960U Under applied overhead, $2,360U

$32,400

*Given **39,600  (2,000 × 6) = $3.30

..

599


13-68

(35-40 min.)

Cost Incurred: Actual Inputs × Actual Prices Direct Labor

Predicted Overhead Based on Actual Driver Use × Standard Prices

Flexible Budget: Standard Driver Use Allowed for Output Achieved × Standard Prices

Product Costing Applied to Product

1,000 × € 42.5 1,000 × € 44* 900 × € 44* = € 42,500* = € 44,000 = € 39,600 € 39,600 Price variance, Usage variance, 1,000 hrs. × € 1.5 = 100 hrs. × € 1,500F € 44 = € 4,400U Flexible-budget variance, No variance € 2,900U

Variable Overhead

1,000 × € 11 900 × € 11* € 10,400* = € 11,000 = € 9,900 € 9,900 Efficiency variance, Spending variance, 100 hrs. × € 11 € 600F = € 1,100U Flexible-budget variance, € 500U No variance Under applied overhead, € 500U

Fixed overhead:

Lump-sum Lump-sum 900 × € 6** € 6,300* € 6,600 € 6,600 = € 5,400 Spending variance, € 300F No variance Prod.-Vol. Var., Flexible-budget variance, € 300F* € 1,200U Under applied overhead, € 900U

*Given **€ 6,600 ÷ (220 × 5) = € 6

..

600


13-69

(15-20 min.)

1. Activity 1. 2. 3. 4. 5. 6. 7. 8. Total

Factory Overhead Costs Applied 1 × $ 1.20 = $ 1.20 39 × .07 = 2.73 28 × .20 = 5.60 15 × .40 = 6.00 1 × 3.20 = 3.20 8× .60 = 4.80 .15 × 80.00 = 12.00 .05 × 90.00 = 4.50 $40.03

Direct materials Factory overhead applied Total manufacturing product cost

$ 55.00 40.03 $95.03

2.

Direct labor is no longer traced separately via time tickets to individual products. Instead, it becomes part of activity cost pools and is included in each activity's factory overhead application rate. This reduces accounting costs because there is no elaborate tracking of labor.

3.

Managers would primarily favor this multiple overhead rate, activity-based costing system because of more accurate product costing. In this way, managers will have more confidence in their decisions regarding pricing and emphasizing or de-emphasizing various products. The older system may be easier to understand but less believable.

..

601


13-70

(40-60 min.) Note that € is the symbol for the Euro.

1. (a) income:

The division manager would want to build inventory and thereby maximize current

Desired ending inventory, maximum possible December sales Total needs November 30 inventory, 110,000 + 10,000 - 100,000 Production scheduled (b) Sales, 106,000 units at € 400 Less cost of goods sold: Beginning inventory, 10,000 at € 250 Manufacturing costs, 121,000 at € 250 Total standard cost of goods available for sale Ending inventory, 25,000 at € 250 Standard cost of goods sold Less overapplied fixed manufacturing overhead, 1,000 at € 85, favorable Gross margin Other expenses: Variable, 106,000 at € 40 Fixed Operating income

Units 25,000 6,000 31,000 20,000 11,000 € 42,400,000

€ 2,500,000 30,250,000 € 32,750,000 6,250,000 € 26,500,000 85,000 € 4,240,000 10,200,000

26,415,000 15,985,000 14,440,000 € 1,545,000

(c) If December production were 4,000 units instead of 11,000 units, the under applied overhead would be 6,000 units at € 85, or € 510,000. Net income would be € 1,545,000 less the € 595,000 difference in the applied overhead, or € 950,000. The ending inventory would be 18,000 units (20,000 + 4,000 production - 6,000 sales). The following tabulation may be helpful: Cumulative Manufacturing Costs Incurred Applied Variance* December production, 11,000 units: Variable Fixed December production, 4,000 units: Variable Fixed

€ 19,965,000 10,200,000

€ 19,965,000 10,285,000

0 85,000F

€ 18,810,000 10,200,000

€ 18,810,000 9,690,000

0 510,000U

*U = under applied, F = over applied. 2. (a)(b) ..

602


Sales, 106,000 units at € 400 Variable costs: Manufacturing, 106,000 at € 165 Other, 106,000 at € 40 Contribution margin Fixed costs: Manufacturing Other Operating income

€ 42,400,000 € 17,490,000 4,240,000 € 10,200,000 10,200,000

21,730,000 € 20,670,000 20,400,000 € 270,000

Operating income is the same under variable costing regardless of December production schedules, because income is influenced by sales alone rather than by sales and production. 2. (c) December production schedule, units Operating income as shown in requirement (1) Inventory increase for the year: 15,000 units at fixed-overhead rate of € 85 8,000 units at € 85 Operating income as shown in requirement (2) 3.

11,000 € 1,545,000

4,000 € 950,000

1,275,000 € 270,000

680,000 € 270,000

The division manager should set the minimum production schedule of 4,000 units. This will reduce the inventories by 2,000 units. She may be tempted to ask for permission to reduce production even below 4,000 units, because the outlook is for ending inventories far in excess of reasonable sales demands. Note that production scheduling can influence short-run reported operating income under absorption costing, but such scheduling has no effect on operating income under variable costing. Thus, the accounting technique used may influence the manager's decision in the former case but not in the latter. It is undesirable to have the accounting technique in itself influence decisions in a direction that may conflict with overall company goals.

4.

4,000 units should be scheduled in December. This will minimize income for the current year and will therefore minimize current income taxes. Additional income taxes will be paid in the future when the rates will be lower.

..

603


13-71

(20-30 min.)

1.

The fixed overhead variance does not reveal how well fixed overhead costs have been controlled. The standard is not an appropriate basis of comparison. Why? Because the standard accounts for a fixed cost as if it were variable. Note that the volume decreased by 10.5%, from 1,520,000 quintals to 1,360,000 quintals, so the standard fixed overhead decreased by 10.5%, from ₹389,120,000 to ₹348,160,000. But the fixed overhead would not be expected to change. The flexible (control) budget for fixed overhead, based on 2019 costs, is $389,120,000. From a control perspective, there was a ₹389,120,000 – ₹385,920,000 = ₹3,200,000F variance. The standard used by Bhavyesh is the same as the applied amount in a standard-cost system. The difference between the actual amount and this applied amount can be summarized as follows: Actual Fixed Overhead ₹385,920,000

Flexible Budget: Fixed Overhead ₹389,120,000

Flexible-budget variance, ₹3,200,000F

Applied Fixed Overhead ₹348,160,000

Production-volume variance, ₹40,960,000 U

A major part of the total variance is the production-volume variance, which serves a productcosting purpose not a control purpose. 2.

Setting standards based on last year’s costs is not uncommon. Managers must carefully interpret the resulting variances. Such variances do not necessarily measure efficiency, as they do with currently attainable standards. Instead, such variances simply indicate changes in costs. Such information can be useful. However, managers should be alert for any past inefficiency built into the standards. Otherwise, inefficiencies will probably persist over a series of years.

..

604


13-72 (25-35 min.) HOLDEN CORP. Variable Costing Income Statement Sales, 100,000 units at $10.00 Variable expenses: Beginning inventory, Cost of goods manufactured, 130,000 at $6.00* Available for sale Ending inventory, 30,000 at $6.00 Standard variable cost of sales Add variance in variable costs of production Variable manufacturing cost of sales Variable selling and administrative expenses Contribution margin Fixed expenses: Manufacturing Selling and administrative Operating income

$1,000,000 $ 0 780,000 $780,000 (180,000) $600,000 10,000** $610,000 30,000 $ 90,000 170,000

(640,000) $ 360,000 (260,000) $ 100,000

* Variable manufacturing CGS = standard absorption CGS – the fixed factory overhead rate. The standard absorption CGS is $750,000 ÷ 100,000 units sold = $7.50. The fixed factory overhead standard rate = budgeted fixed factory overhead divided by denominator level, which equals $150,000 ÷ 100,000 = $1.50. So variable manufacturing CGS is $7.50 - $1.50 = $6.00. ** Since Holden does not prorate, the $95,000F variance on its absorption income statement must be the net total of all variances under that system. Since the total absorption-cost variances were $95,000 F and fixed factory overhead variances yield a net variance of $105,000F (production-volume variance = $1.50 × (130,000 – 100,000) = $45,000 F and production spending variance = $150,000 - $90,000 = $60,000F), the inference must be that there are $10,000U variances in variable manufacturing costs. See the following analysis of variances: Fixed overhead:

..

Actual Lump-sum Lump-sum Applied $ 90,000 $ 150,000 $ 150,000 $ 195,000 Spending variance, $60,000F No variance Prod.-Vol. Var. Flexible-budget variance, $ 60,000F $ 45,000F Over applied overhead, $ 105,000F

605


13-73 (25-35 min.) First, it is helpful to determine the variable-costing income statement: MOSELEY CORP. Variable Costing Income Statement Sales, 80,000 units at $10.00 Variable expenses: Beginning inventory, $ 0 Cost of goods manufactured, 160,000 at $5.00* 800,000 Available for sale 800,000 Ending inventory, 80,000 at $5.00 (400,000) Variable manufacturing cost of sales $ 400,000 Variable selling and administrative expenses 100,000 Contribution margin Fixed expenses: Manufacturing ($150,000 - $60,000) $ 90,000 Selling and administrative 60,000 Operating income

$ 800,000

(500,000) $ 300,000 (150,000) $ 150,000

* The variable manufacturing CGS is $400,000 ÷ 80,000 units sold = $5.00. Next, analyze the fixed overhead: Fixed overhead:

Applied 160,000 × $ .50* = $ 80,000

Actual Lump-sum Lump-sum $ 90,000 $ 100,000 $ 100,000 Spending variance, $10,00F No variance Prod.-Vol. Var. Flexible-budget variance, $ 10,000F $ 20,000U Under applied overhead, $ 10,000U

* The fixed factory overhead standard rate = budgeted fixed factory overhead divided by denominator level, which equals $100,000/ 200,000 = $.50.

..

606


MOSELEY CORP. Absorption Costing Income Statement Sales, 80,000 units at $10.00 Cost of sales: Beginning inventory, Cost of goods manufactured, 160,000 at $5.50* Available for sale Ending inventory, 80,000 at $5.50 Standard cost of sales Add net unfavorable variances: Variable manufacturing costs Fixed overhead variances, prorated ** Gross margin Selling and administrative expenses: Variable, Fixed Operating income

$ 800,000 $

0 880,000 880,000 (440,000) $ 440,000 0 5,000 $100,000 60,000

(445,000) $ 355,000 (160,000) $ 195,000

* Standard absorption CGS = variable manufacturing CGS + the fixed factory overhead rate. The variable manufacturing CGS is $400,000 ÷ 80,000 units sold = $5.00. So standard absorption CGS is $5.00 + $.50 = $5.50. ** Since Moseley prorates, half of the $10,000U net variances from fixed factory overhead goes to CGS, and the other half goes to ending finished goods (they are evenly split between the two accounts because there is no ending WIP, and each account is worth the same amount, 80,000 × $5.50). Therefore, the adjustment on the income statement is for ½ × $10,000 = $5,000U.

..

607


13-74 (20-25 min.) 1. Umbro makes soccer gear and apparel. A plant that makes soccer gear would have numerous variable- and fixed-cost resources that are indirect costs and are allocated to the various product types. A partial list: Indirect-Cost Resources Variable Cost Fixed Cost Electrical power Plant depreciation Overtime labor not dedicated to a specific product line Equipment depreciation Temporary labor not dedicated to a specific product line Supervision salaries Manufacturing supplies Regular labor wages Fuel for equipment such as forklifts Process and product engineers’ salaries 2. A dedicated production line that makes only soccer shoes would result in several resources being directly traceable to the cost object – soccer shoes. Among these would be line supervisors’ salaries, equipment depreciation, overtime labor, temporary labor, and product/process engineering costs if engineers and the equipment they use are also dedicated to the soccer shoe product line. 13-75 (20-30 min.) For the solution to this Excel Application Exercise, follow the step-by-step instructions provided in the textbook chapter. 1.

A: $5.50/machine hour; B: $8.10/direct labor hour.

2.

To A: $1,672,000; Overapplied by $282,000.

3.

To B: $947,700; Underapplied by $282,300.

13-76 (180 min. or more) The purpose of this exercise is to learn how real companies allocate costs. It involves learning what costs are included in overhead, how they are categorized, whether cost allocations recognize cost-behavior patterns, what cost drivers are used for allocation, and the process by which costs are allocated to final products or services. The requirement for a diagram makes students put what they learn into a coherent package. It is easy to sit and listen to what seems like a very logical explanation but not understand it fully. The diagram of a cost allocation system cannot be done without a thorough understanding of the system. A very useful exercise is to have several groups present their findings to the class. In addition to learning about different cost allocation systems, the students making the presentation will hone their communication skills and those listening will learn a great deal about a variety of companies. We find that several short (approximately 5-minute) presentations can be more effective than a couple of longer ones. Students learn to focus quickly on the most important issues.

..

608


13-77 (30-40 min.) NOTE TO INSTRUCTOR. This solution is based on the web site as it was in late 2012. Be sure to examine the current web site before assigning this problem, as the information there may have changed. For numbers 2 and 3, students may need to access the 10K report because Dell provides very limited financial information in its annual reports. 1.

Answers will vary depending on the computer chosen. In the laptop/notebook computers for home and home office family, there are Inspiron and XPS models. Inspiron notebooks range in price from about $400 to $2,500. Information includes the size of the display, memory, computing speed, weight, and other hardware included such as DVD. Features vary which can cause the price to be different from the base price.

2.

The total revenue for Dell was $62.1 billion for the year ended February 3, 2012. The FY 2012 10-K states the average price for desktop PCs decreased by 2% from FY 2011. Apparently, price competition in the desktop PC market has been keen.

3.

From Dell’s 2012 financial statements, the cost of goods sold (cost of revenue) was $48,260,000,000, selling, general and administrative expenses were $8,524,000,000, and research, development, and engineering expenses were $856,000,000. Depreciation and amortization was $936,000,000. Based on the assumptions given, Dell’s fixed costs are $936 million + $3,000 million = $3,936 million. Therefore, variable costs are (in millions): Cost of net revenue Operating expenses Less: fixed costs Variable costs

$48,260 9,380 (3,936) $53,704

Therefore, variable costs percentage is $53,704,000,000 ÷ 62,071,000,000 = 86.5% and the contribution margin percentage is 13.5%. The breakeven point in dollar sales is: BEP

= Fixed cost ÷ Contribution margin percentage = $3,936,000,000 ÷ .135 = $29,155 million.

Does this seem reasonable? Yes, because Dell’s sold approximately $33 billion above the breakeven point at a contribution margin of $.135 per sales dollar, meaning operating income should be about $4,455 million, just a little larger than the actual $4,431 million.

..

609


CHAPTER 14 COVERAGE OF LEARNING OBJECTIVES

LEARNING OBJECTIVE LO1: Distinguish between job-order costing and process costing. LO2: Prepare summary journal entries for the typical transactions of a jobcosting system. LO3: Use an activity-basedcosting system in a joborder environment. LO4: Show how service organizations use job costing. LO5: Explain the basic ideas underlying process costing and how they differ from job costing. LO6: Compute output in terms of equivalent units. LO7: Compute costs and prepare journal entries for the principal transactions in a process-costing system. LO8: Demonstrate how the presence of beginning inventories affects the computation of unit costs under the weighted-average method. LO9: Understand the concept of transferred-in costs in a process-costing system with sequential processes. LO10: Use backflush costing with a JIT production system. ..

FUNDAMENTAL ASSIGNMENT MATERIAL A1, B1, B4

ADDITIONAL ASSIGNMENT MATERIAL 15, 16, 18, 20, 20, 29, 29

B4

5, 21, 22, 22, 24, 25, 26, 26, 27, 27, 40, 41, 41, 50, 51

B4

28

B4

17, 17, 23, 23, 39, 42, 42

B4

30

A2, B2, B4

7, 11, 31, 32, 32, 33, 33, 36, 36, 44, 44, 45, 45, 46, 46, 47 32, 32, 33, 33, 34, 35, 37, 38, 43, 45, 45, 46, 46, 47

A2, B2, B4

A3, B3, B4

49

B4

13, 48

A4, B4

19

CASES, NIKE 10K, EXCEL, COLLAB. & INTERNET EXERCISES 52, 54, 55

55

53

610


CHAPTER 14 Job-Costing and Process-Costing Systems 14-A1 (15-20 min.) Answers are in thousands. 1.

a. b. c. d.

e.

f. g.

Direct materials inventory Cash

350

Work in process inventory Direct materials inventory

210

Work in process inventory Accrued payroll

150

Factory department overhead control Various accounts (130 + 40 + 70 = 240)

240

Work in process inventory Factory department overhead control (180% × 150)

270

Finished goods inventory Work in process inventory

575

Cost of goods sold Finished goods inventory

415

350 210 150 240

270

575 415

2. Direct Materials Inventory _ a. 350 b. 210 * Bal. 140 b. c. e.

Work in Process Inventory 210 f. 150 270

* Bal.

55

575

Finished Goods Inventory ___ f. 575 g. 415 * Bal. 160 g.

d.

Cost of Goods Sold____ 415 Factory Department Overhead Control _ 240 e.

270

* 12/31/X1 Balance

..

611


14-A2 (10-15 min.) 1.

(Step 2) (Step 1) Equivalent Units Physical Direct Conversion Units Materials 20,000 20,000 20,000 3,000 3,000 1,000 23,000 23,000 21,000

Flow of Production Started and completed Work in process, ending inventory Direct materials added: 3,000 × 1 Conversion costs added: 3,000 × (1/3) Total accounted for Total work done

Total costs to account for (Step 3): Divide by equivalent units (Step 4) Unit costs 2. Application of costs (Step 5): To units completed and transferred to Testing, 20,000 units × $8.60 To units not completed and still in process, Feb. 28, 3,000 units: Direct materials Conversion costs Work in process, Feb. 28 Total costs accounted for

..

Total Costs $187,000 $

8.60

Direct Conversion Materials Cost $73,600 $113,400 23,000 21,000 $ 3.20 $ 5.40

Totals Direct Materials

Details Conversion Costs

$172,000 $ 9,600 5,400 $ 15,000 $187,000

3,000 × $3.20 1,000 × $5.40

612


14-A3 (25-30 min.) (Step 1) Physical Units 10,000 (25%)* 80,000 90,000

Flow of Production Work in process, beg. inv. Started To account for Completed and transferred out during current period Work in process, end. inv. Units accounted for Work done to date

70,000 20,000 (50%)* 90,000

Costs

(Step 3) (Step 4)

(Step 5)

Work in process, beg. inv. Costs added currently Total costs to account for Divisor, equivalent units for work done to date Cost per equivalent unit Application of Costs Completed and transferred (70,000 units) Work in process, ending inv. (20,000 units): Direct materials Conversion costs Total work in process Total costs accounted for

Totals

(Step 2) Equivalent Units Direct Conversion Materials

70,000 20,000

70,000 10,000

90,000

80,000

_Details Direct Conversion Materials Costs

$ 175,500 1,486,500 $1,662,000

$138,000 $ 37,500 852,000 634,500 $990,000 $672,000

$

÷ 90,000 ÷ 80,000 $ 11.00 $ 8.40

19.40

$1,358,000 $ 220,000 84,000 $ 304,000 $1,662,000

20,000($11.00) 10,000($8.40)

*Degree of completion for conversion costs.

..

613


14-A4

(15-20 min.)

1.

Materials inventories Accounts payable

235,000

Conversion costs Accrued payroll Miscellaneous accounts

100,000

Finished goods inventories (10,000 × $33) Materials inventories (10,000 × $22) Conversion costs (10,000 × $11)

330,000

Cost of goods sold (9,970 × $33) Finished goods inventories

329,010

Conversion costs Cost of goods sold To recognize actual conversion costs that were $10,000 less than the amount applied to the products.

10,000

2.

..

235,000 24,000 76,000 220,000 110,000 329,010 10,000

614


14-B1 (20-25 min.) Entries are in thousands of British pounds (£). 1.

a. b. c. d.

e. f. g. h.

Direct materials inventory Accounts payable

74

Work in process inventory Direct materials inventory

52

Work in process inventory Accrued payroll

200

Factory department overhead control Various accounts, such as cash or accounts payable

156

74 52 200

156

Work in process inventory 160 Factory department overhead control

160

Finished goods inventory Work in process inventory

417 417

Cost of goods sold Finished goods inventory

532

Accounts receivable Sales

742

532 742

2. * Bal. a. ** Bal.

Direct Materials Inventory 12 b. 52 74 34

Work in Process Inventory * Bal. 20 f. b. 52 c. 200 e. 160 ** Bal.

15

Finished Goods Inventory_ * Bal. 130 g. 532 f. 417 ** Bal. 15 417

g.

Cost of Goods Sold 532

d.

Factory Department Overhead Control 156 e. 160

* 12/31/11 Balance ** 12/31/12 Balance

..

615


14-B2 (10-15 min.) 1.

Flow of Production Started and completed Work in process, ending inv. Units accounted for Units work done to date

Total costs to account for (Step 3) Divide by equivalent units (Step 4) Unit costs

(Step 2) Equivalent Units (Step 1) Physical Units

Direct Materials

Conversion Costs

287,000 283,000

287,000 70,750*

570,000

357,750

Totals

Direct Materials

Conversion Costs

$3,825,000

$2,394,000

$1,431,000

$8.20

570,000 $4.20

357,750 $ 4.00

Direct Materials

Details Conversion Costs

287,000 283,000 570,000

*283,000 × .25 2. Application of costs (Step 5): To units completed and transferred to Finishing, 287,000 units × $8.20 To units not completed and still in process, end, 283,000 units: Direct materials Conversion costs Work in process, end Total costs accounted for

..

Totals

$2,353,400 $ 1,188,600 283,000 $ 1,471,600 $3,825,000

283,000 × $4.20 70,750 × $4.00

616


14-B3 (25-35 min.) (Step 1) Physical Units 900 (30%)* 9,200 10,100

Flow of Production Work in process, beg. inv. Started To account for Completed and transferred out during current period, 900 + 9,200 – 2,400 Work in process, end. inv. Units accounted for Work done to date

7,700 2,400 (25%)* 10,100

Costs Work in process, beg. inv. Costs added currently (Step 3) Total costs to account for (Step 4) Divisor, equivalent units For work done to date Cost per equivalent unit (Step 5) Application of Costs Completed, (7,700 units) Work in process, ending inventory (2,400 units): Direct materials (2,400) Conversion costs (600) Total work in process Total costs accounted for

Totals

(Step 2) _ Equivalent Units___ Direct Materials Conversion

7,700 2,400

7,700 600

10,100

8,300

_Details __ Direct Conversion Materials Costs

$ 4,600 122,750 $127,350

$ 2,900 82,950 $85,850

$ 1,700 39,800 $41,500

$13.50

÷ 10,100 $8.50

÷ 8,300 $5.00

$103,950 $ 20,400 3,000 $ 23,400 $127,350

2,400($8.50) 600($5.00)

*Degree of completion for conversion costs.

..

617


14-B4 (15 min.) 1.

2.

Materials inventories Accounts payable

23,980

Conversion costs Accrued payroll and miscellaneous accounts

10,490

Cost of goods sold (2,200 × $14.10) Materials inventories (2,200 × $9.40) Conversion costs (2,200 × $4.70)

31,020

23,980

10,490

Cost of goods sold 150 Conversion costs To recognize actual conversion costs that were $150 more than the amount applied to the products.

20,680 10,340 150

14-1 Three purposes of product costing are to satisfy differing demands for (a) inventory valuation and income determination in accordance with generally accepted accounting principles, (b) income tax reporting, and (c) guiding strategic and operational decision-making. 14-2 The distinction between the job cost and the process cost methods centers largely around how product costing is accomplished. Unlike process costing, which deals with broad averages and great masses of like units, the essential feature of the job-cost method is the attempt to apply costs to specific jobs that may consist of either a single physical unit (a custom sofa) or a few like units (a dozen tables) in a distinct batch or job lot. 14-3 The basic record for the accumulation of job costs is the job-cost sheet or job-cost record. Exhibit 14-1 shows a Job-Cost Sheet, and it also shows the related source documents. A file of current job-cost sheets becomes the supporting details for the Work-in-Process Inventory account. 14-4 Source documents include materials requisitions and labor time tickets (time cards). 14-5 The journal entry for applying the factory overhead costs to job-orders is: WIP inventory ………………….XXX Factory department overhead control……………….XXX 14-6 No, the amount of value-chain activity not captured in either job-cost or processcost systems is independent of the type of operating system used.

..

618


14-7 In process costing, at the end of a reporting period, there may be some units partially completed. Since a partially completed unit can never be assigned the same costs as the completed output, the total output should not be stated in terms of physical units; it should be measured in equivalent units, i.e., the number of completed (whole) units that could have been produced from the resources required for the partially completed units. Thus, “equivalent units” facilitates appropriate measurement of the per-unit resources cost. 14-8 Examples of process costing include handling of mail, income tax returns, automobile registrations, and driver’s license examinations. 14-9

Five key steps in process cost accounting are; Step 1: Summarize the flow of physical units Step 2: Calculate output in terms of equivalent units Step 3: Summarize the total costs to account for, which are the total debits in Work in Process (that is, the costs applied to Work in Process) Step 4: Calculate unit costs (cost per equivalent unit) Step 5: Apply costs to units completed and to units in ending work in process

14-10 The first two steps concentrate on what is occurring in physical or engineering terms. The financial impact of the production process is measured in the final three steps. 14-11 (1 × 12,000) + (.6 × 6,000) = 15,600 equivalent units. 14-12 Beginning inventories + Units started = Units transferred out + Ending inventories. 14-13 In a sequential process costing system, transferred-in costs, i.e., costs incurred in a previous department for items that have been received by a subsequent department, are accounted for a direct material that is 100% added at the beginning of the subsequent process with only one exception: the transferred-in costs are kept separate from the other direct material added in the receiving department. 14-14 When actual conversion costs exceed the amount applied, the excess in the conversion cost account is charged directly to cost of goods sold; the treatment is similar to accounting for underapplied overhead. 14-15 In addition to inventory valuation and income determination, managers want accurate job costs as guides to pricing and to allocating effort among particular products, services, or customers. They are also necessary in contracts that reimburse the cost of a product or service. 14-16 The most important point is that product costing is an averaging process. The unit cost used for inventory purposes is the result of taking some accumulated cost and dividing it by some measure of production. The basic distinction between job order costing and process costing is the breadth of the denominator: in job order costing, it is

..

619


small (for example, one painting, 100 advertising circulars, or one special packaging machine); but in process costing, it is large (for example, thousands of pounds, gallons, or board feet). 14-17 Though this may be a common practice in many service firms, it may lead to inaccurate measurement of job cost. The materials and supplies cost may largely vary across jobs without showing any direct relationship with the professional hours spent on them. Hence, in a competitive industry, costs other than professional compensation should also be traced to jobs using computer technology as far as possible. 14-18 The central product costing problem in process costing is how each department should compute the cost of goods transferred out and the cost of goods remaining in the department. 14-19 No, but they are especially appropriate for companies with just-in-time systems. Any company with small inventories might find backflush costing appealing. 14-20 (5 min.) 1. and 4. are service-sector companies; 2. is merchandising; and 3. is manufacturing. 14-21 (10-15 min.) You may wish to use T-accounts. Amounts are in millions of dollars. You can also use the expression: ending balance (of any account) equals the beginning balance plus additions less subtractions or EB = BB + A - S. In this case “Purchased” is “additions” and “Used” is “subtractions.” 1.

7 + 10 - 10 = 7 (BB + A - S = EB)

2.

15 + 7 - 13 = 9 (BB + A - EB = S)

3.

6 + Purchases - 10 = 11.

4.

Beginning inventory + 7 - 6 = 4. Beginning inventory = 3

Purchases = 15

14-22 (10-15 min.) Amounts are in thousands of dollars. 1.

2.

..

Finished goods inventory Work in process inventory Finished goods = 105 + 120 = 225

225 225

Debits: 27 + 141 + 90 + 81 = 339 Credits: 105 + 120 = 225 Balance, July 31 114

620


3.

..

Accounts receivable Sales Sale of Job X16

147

Cost of goods sold Finished goods inventory Cost of Job X16 sold

105

147

105

621


14-23 (10-15 min.) COVID-19 Research Project Biotechnology Department

Reference

Date

Unit Quantity Cost

Direct Materials: Var. medical supplies Various chemicals

Sept. 5 Sept. 7

Direct Labor: Research associates Research assistants

Sept. 5-12 Sept. 7-12

125 hrs. ×₹3,360 205 hrs. ×₹1,440

Sept. 12

₹715,200 × .60

Project overhead applied Total costs

Amount

Summary

₹ 540,000 1,220,000

₹ 1760,000

₹420,000 295,200 715,200 ₹429,120

429,120 ₹2,904,320

14-24 (10 min.) 1.

$8,400 + $4,800 = $13,200

2.

$9,200 + $8,600 = $17,800

3.

$6,700

..

622


14-25 (15 min.) Answers are in thousands of dollars. 1.

a Construction in Process Job Sept. 30 43 51 52 53 200 61 115 62 180 71 118 81 106 719 1

200 + 50

2

b Finished Houses, Sept. 30

115 + 20

c Cost of Houses Sold Sept. 180 170

a Construction in Process Oct. 31

150 2501

1352

2053

4

___ 150 3

154 1545 308

___ 350 180 + 25

4

118 + 36

5

___ 135

Cash

500

Oct 590

500

590

345

Sales To record sale of Job 53 Cost of houses sold Finished houses inventory To record cost of Job 53 sold

___ 605

106 + 48

Sept. Finished houses inventory Construction in process Sept.: 180 + 170 + 150 = 500 Oct.: 250 + 135 + 205 = 590

..

c Cost of Houses Sold Oct.

150

2.

3.

b Finished Houses, Oct. 31

345 250 250

623


14-26 (30 min.) The answers (in millions) are €316, €140, and €200. Step-by-step entries are keyed alphabetically. The sequence depends on where the student prefers to start. You may wish to raise the question of whether the overapplied overhead should be prorated among the affected accounts at the end of the year. __ Direct Materials Bal. 316 (a) 656 (b) 450 Bal.

110

Work in Process Bal. 140 (e) Completed 1940 (a) Dir. Materials 656 (c) Dir. Labor 1,200* (d)Applied overhead 1,920 Bal.

1,976

Finished Goods Bal. 200 (f) 1,950 (f) (e) 1,940

Cost of Goods Sold 1,950

Bal.

190

* €1,920  160% = €1,200

..

624


14-27 (30 min.) The answers (in millions) are $51, $348, and $426. Step-by-step entries are keyed alphabetically. The sequence depends on where the student prefers to start. You may wish to raise the question of whether the underapplied overhead should be prorated among the affected accounts at the end of the year. Note the heavy ending Finished Goods. Direct Materials Bal.* 51 (a) 726 (b) 897 Bal.

222

Work in Process Bal.* 348 (e) Completed (a) Dir. Materials 726 (c) Dir. Labor 348 (d) Applied overhead 522 Bal.

Finished Goods Bal.* 426 (f) 1,776 (e) 1,890 Bal.

1,890

54

Cost of Goods Sold (f) 1,776

540

* Let BB = beginning balance Direct materials: BB + 897 – 726 = 222 BB = 17 Work in process: BB + 726 + 348 + 522 – 1,890 = 54 BB = 348 Finished goods: BB + 1,890 – 1,776 = 540 BB = 426 14-28 (5-10 min.) Case A, $3,600,000 ÷ $2,000,000 = 180% of direct-labor cost Case B, $5 × 450,000 = $2,250,000 Case C, $1,500,000 ÷ 250,000 = $6 per machine hour 14-29 (5 min.) PepsiCo and Reliance are manufacturing companies. Blue Dart and HDFC ERGO are service companies.

..

625


14-30 (5 min.) (1) (2) (3) (4)

The debit to (increase in) the work in process account when transferring a subcomponent from Process A to the assembly process is a transferred-in cost. The direct materials used in process A and assembly are variable-cost resources. Direct labor costs in process A and assembly are directly traced fixed-cost resources. An example of an indirect resource cost is the indirect material and indirect labor used for process A and the assembly process.

14-31 (5 min.) The direct material is the limestone rock that is delivered to the plant. Because crushing and screening the rock can begin immediately, we assume that direct material is always 100% completed. Thus, the equivalent units of direct material is the entire 400 tons. The 320 tons of rock that have been stocked are 100% complete with respect to both direct labor and overhead. The 80 tons of rock that are in process at the end of March are 40% complete. This is 32 equivalent tons (80 tons × .40). Thus, the total work done during March is 400 tons of direct material and 352 (that is, 320 + 32) equivalent tons of direct labor and overhead.

..

626


14-32

(10-15 min.)

1.

Flow of Production Started and completed Work in process, ending inv. Units accounted for

(Step 1) Physical Units (lts.) 1,350,000 498,000 1,848,000

Work done to date

Total costs to account for (Step 3) Divide by equivalent units (Step 4) Unit costs

(Step 2) Equivalent Units (lts.) Direct Conversion Materials 1,350,000 1,350,000 498,000 174,300*

1,848,000

1,524,300

Totals

Direct Materials

Conversion Costs

£12,126,750

£8,316,000

£3,810,750

£7.00

1,848,000 £4.50

1,524,300 £2.50

*498,000 × .35 2. Application of costs (Step 5): To units completed and transferred, 1,350,000 units × £7.00 To units still in process, end, 498,000 units: Direct materials Conversion costs Work in process, end Total costs accounted for

..

Totals

Details

£9,450,000

£2,241,000 435,750 £2,676,750 £12,126,750

498,000×£4.50 174,300 × £2.50

627


14-33

(15-20 min.)

1.

Flow of Production Units started and completed Work in process, end: Materials added: 3,000 × 0.90 Conversion costs: 3,000 × 0.70 Units accounted for Work done to date 2.

Costs to account for (Step 3) Divide by equivalent units (Step 4) Unit costs Application of costs (Step 5): To units completed and transferred, 34,000 × ¥1,328 To units still in process, end, 3,000 units: Direct materials Conversion costs Work in process, end Total costs accounted for

..

(Step 2) Equivalent Units (Step 1) Physical Units 34,000 3,000

Direct Materials Conversion 34,000 34,000 2,700 2,100

37,000 36,700

36,100

Details Total Direct Conversion Costs Materials Costs ¥48,209,600 ¥16,441,600 ¥31,768,000 36,700 36,100 ¥1,328 ¥448 ¥880

¥45,152,000

¥ 1,209,600 1,848,000 ¥ 3,057,600 ¥48,209,600

2,700×¥448 2,100×¥880

628


14-34

(10-15 min.)

1.

Work in process – Assembly Direct materials inventory Materials added to production in February

73,600

Work in process -- Assembly Accrued payroll Direct labor in February

59,000

Work in process -- Assembly Factory overhead Factory overhead applied in February

54,400

Work in process -- Testing Work in process -- Assembly Cost of goods completed and transferred in February from Assembly to Testing

172,000

2.

3.

4.

73,600

59,000

54,400

172,000

The Key T-account would show: 1. Direct materials 2. Direct labor 3. Factory overhead Bal. February 28

..

Work in Process -- Assembly 73,600 4. Transferred out 59,000 to Testing 54,400

172,000

15,000

629


14-35

(10-15 min.)

1.

Work in process -- Assembly Direct materials inventory Materials added to production

2,394,000

Work in process -- Assembly Accrued payroll Direct labor

561,000

Work in process -- Assembly Factory overhead Factory overhead applied

870,000

2.

3.

4.

2,394,000

561,000

Work in process -- Finishing 2,353,400 Work in process -- Assembly Cost of goods completed and transferred from Assembly to Finishing

870,000

2,353,400

The key T-account would show: 1. 2. 3.

Direct materials Direct labor Factory overhead

Balance

..

Work in Process -- Assembly 2,394,000 4. Transferred out 561,000 to Finishing 870,000

2,353,400

1,471,600

630


14-36

(10-15 min.)

Flow of Production Work in process, beg. inventory* Started To account for Completed and transferred out (75,000 – 17,500) Work in process, ending inventory** Units accounted for Work done to date

(Step 1) Physical Units 40,000 35,000 75,000 57,500 17,500 75,000

(Step 2) Equivalent Units Direct Materials Conversion

57,500 4,375a

57,500 2,625a

61,875

60,125

*Degree of completion: materials, 95%; conversion costs, 90% **Degree of completion: materials, 25%; conversion costs, 15% a 0.25 × 17,500 and 0.15 × 17,500 14-37 (5-10 min.) 1. 2. 3.

Work in process, Department A Direct-materials inventory

82,950

Work in process, Department A Various accounts

39,800

Work in process, Department B Work in process, Department A

103,950

82,950 39,800 103,950

14-38

(5-10 min.)

1.

Work in process, Assembly Department Direct materials inventory

852,000

Work in process, Assembly Department Various accounts

634,500

Work in process, Finishing Dept. Work in process, Assembly Dept.

1,358,000

2. 3.

..

852,000 634,500 1,358,000

631


14-39

(10-15 min.)

Dell would most likely use a job-cost system with each order considered a job. Because each order is assembled from a set of common parts, there is a single cost for each part. Most of the parts are purchased, so the cost is the purchase price. If some parts are made, the production cost would be used as the cost of the part. Each order would call for several materials, and each would be added to the order's job-cost sheet. Labor would be incurred in assembly, so the direct-labor cost could be allocated to each order based on the number of hours used for assembly. If assembly is highly automated, it is possible that no labor is considered "direct", and labor becomes one more overhead item. Overhead costs would be allocated based on one or more cost drivers. Possible drivers include direct-labor hours or cost (if direct labor is measured separately), hours in assembly, or number of component parts. For a highly automated process, the latter would be a likely cost driver. Testing and quality control costs might be part of overhead. Alternately, costs of testing final computers could be charged directly to the order (job). If different types of computers require different amounts of testing, this is a logical allocation method. 14-40

(15-25 min.)

1.

Ending inventory = Beginning inventory + Purchases - Usage 90 = 70 + Purchases - 468 Purchases = 488

2.

Total manufacturing costs = Direct + Direct charged to production materials labor 864 = 468 + DL + .8 DL 864 - 468 = 1.8 DL 1.8 DL = 396 DL = 220

3.

4.

..

+

Factory overhead

Cost of goods = Cost of goods + Beginning finished available for sale manufactured goods = 864 + 100 = 964 Cost of goods sold

= Cost of goods available - Ending finished for sale goods = 964 - 120 = 844

632


14-41

(25-35 min.)

1.

Job 515 (€18,000 + €8,000 + €16,000) Job 516 (€24,000 + €10,000 + €20,000) Work-in-process inventory, May 31

2.

The job-cost records indicate an overhead application rate of €16,000 ÷ €8,000 = 200% or €20,000 ÷ €10,000 = 200%.

3.

a.

b.

c.

d.

e.

Bal. Bal.

Work-in-process inventory Direct materials inventory Job 515 of €5,000 + Job 516 of €26,000 Work-in-process inventory Accrued payroll €3,000 + €5,000 + €4,000 Work-in-process inventory Factory department overhead control €12,000 × 200% Finished goods inventory (Job 515) Work-in-process inventory €42,000 + €5,000 + €3,000 + 200% of €3,000 Cost of goods sold Finished goods inventory The €66,000 amount is given.

Direct Materials Inventory 38,000 (a) 31,000 7,000 Accrued Payroll (b)

12,000

Fact. Dept. Overhead Control (c) 24,000

Bal. (e) Bal. ..

€42,000 54,000 €96,000

31,000 31,000 12,000 12,000 24,000 24,000 56,000 56,000 66,000 66,000

Work-in-Process Inventory Bal.* 96,000 (d) 56,000 (a) 31,000 (b) 12,000 (c) 24,000 163,000 Bal. 107,000 Finished Goods Inventory Bal. 36,000 (e) 66,000 (d) 56,000 Bal. 26,000

Cost of Goods Sold 900,000 66,000 966,000

633


*107,000 + 56,000 – (31,000 + 12,000 + 24,000) 4.

Job 516 (€54,000 + €5,000 + 200% of €5,000) Job 517 (€26,000 + €4,000 + 200% of €4,000) Work-in-process inventory, June 30

14-42

(20 min.)

1.

Compensation for non-chargeable time, 0.25 × $6,000,000 Other costs Total overhead Direct labor, 0.75 × $6,000,000 Overhead application rate, (a) ÷ (b)

(a) (b)

2.

Hourly rate: $199,680 ÷ (48 × 40) = $199,680 ÷ 1,920 =

€69,000 38,000 €107,000

$1,500,000 1,875,000 $3,375,000 $4,500,000 75%

$104.00

Many students will forget that “his work there” includes an overhead application: Direct labor, 24 × $104.00 Applied overhead, $2,496.00 × 0.75 Total costs applied

$2,496.00 1,872.00 $4,368.00

We point out that direct-labor time on a job is usually compiled for all classes of accountants and then applied at their different compensation rates. Overhead is usually not applied on the piecemeal basis demonstrated here. Instead, it is applied in one step after all the labor costs of the job have been accumulated.

..

634


14-43

(30-40 min.) Answers to requirements follow the analysis. (Step 1) Physical Units 24 (3/4)* 288 312

Flow of Production Work in process, beg. inv. Started To account for Completed and transferred out Work in process, end. inv. Units accounted for Work done to date

297 15 (1/3)* 312

(Step 2) Equivalent Units Direct Conversion Materials

297 15

297 5

312

302

*Degree of completion for conversion costs. Costs Work in process, beg. inv. Costs added currently (Step 3) Total costs to account for (Step 4) Divisor, equivalent units for work done to date Cost per equivalent unit (Step 5) Application of Costs Completed and transferred (297 tons) Work in process, end. inv. (15 tons): Direct materials Conversion costs Total work in process Total costs accounted for

Totals $ 6,000 75,230 $81,230 $

265

Details Direct Conversion Materials Costs $ 2,880* $ 3,120 34,560* 40,670 $37,440 $43,790 ÷ 312 $ 120

÷ 302 $ 145

$78,705 $ 1,800 725 $ 2,525 $81,230

15×$120 5×$145

* $120 × 24 = $2,880; $120 × 288 = $34,560. 1.

$265 per ton

2.

$2,525

3. This requirement cannot be answered directly from the data using the weighted average process cost method. We must look at the equivalent units of conversion work done in May only: Work done through the end of May 302 tons Work done before May (3/4 × 24 tons) 18 tons Work done in May 284 tons ..

635


Budget for 284 tons: $16,000 + ($80 × 284) = $38,720 Budget - Actual = $38,720 - $40,670 = $1,950 unfavorable During May, conversion costs were $1,950 (or 5%) above budget. 14-44

(15-20 min.)

1.

Biscuits are homogeneous products with low unit cost that must be processed through a sequence of continuous steps (sequential processing). Biscuits are produced continuously rather than to order. As a result, a process-cost system is the most logical cost accounting system to use for product-costing purposes.

2.

Activity-based accounting systems are most beneficial when products and/or processes are characterized by diversity. Diversity can be in the volume of product produced or the degree of complexity in the production process across product lines. Since neither of these forms of diversity characterizes the biscuit industry, it is doubtful that activity-based accounting would pass the cost-benefit test. It may be that some specialty producers of gourmet biscuits may have sufficient diversity to warrant the use of an activity-based accounting system.

3.

Suppose that Britannia produces over 3,000 kilograms of biscuits each hour, 24 hours a day. This translates into more than 26 million kilograms per year. Since at any point in time the work-in-process amounts to no more than one-half an hour (it takes 30 minutes to completely produce the end product), work-inprocess accounts for about 1,500/26,000,000 or 0.006 percent of total annual production. The implication is that work-in-process can be ignored for productcosting purposes due to its immaterial amount.

..

636


14-45

(15-20 min.)

1.

Units started and completed Work in process, end Units accounted for Work done to date

(Step 1) Physical Units 12,800,000 3,200,000 16,000,000

(Step 2) Equivalent Units Materials & Supplies Conversion 12,800,000 12,800,000 3,200,000 3,040,000* 16,000,000 15,840,000

* 3,200,000 × 0.95 2. Total Costs Cost to account for (Step 3) Divide by equivalent units (Step 4) Unit costs 3.

₹849,664,000 ₹53.60

₹64,000,000 ÷ 16,000,000 ₹4.00

₹785,664,000 ÷ 15,840,000 ₹49.60

Ending work in process, 3,200,000 units: Materials and supplies, 3,200,000 × ₹4.00 ₹ 12,800,000 Conversion costs, 3,040,000 × ₹49.60 150,784,000 Cost of 3,200,000 returns not yet completed ₹163,584,000

Check: Cost of completed returns (12,800,000 × ₹53.60) Cost of not-yet-completed returns Total costs accounted for

..

Details Materials Conversion & Supplies Costs

₹686,080,000 163,584,000 ₹849,664,000

637


14-46

(20 min.)

1.

Flow of Production Work in process, beginning Started To account for Completed Work in process, ending Units accounted for Work done to date

(Step 1) Physical Units 0 25,000 25,000 20,000 5,000* 25,000

(Step 2) Equivalent Units Metallic Chemical Component Compound Conversion

20,000 5,000

20,000 0

20,000 2,000

25,000

20,000

22,000

*40% complete Costs Costs to account for (Step 3) Divide by equivalent units (Step 4) Cost per equivalent unit 2. (Step 5) Application of Costs Units completed * Work in process, ending: Material – Metallic component Conversion costs Total work in process, ending Total costs accounted for

Total Costs $365,000

$15.50

Metallic Chemical Conversion Component Compound Costs $225,000 $30,000 $110,000 ÷25,000 $9.00

Totals $310,000 $ 45,000 10,000

÷20,000 $1.50

÷22,000 $5.00

Details

5,000 × $9 2,000 × $5

$55,000 $365,000

*20,000 × $15.50

..

638


14-47

(20-30 min.)

1. Flow of Production Units started and completed Work in process, end Direct materials added: 5,000 × 1.00 Cartons added: none Conversion costs: 5,000 × .95 Units accounted for Work done to date

Step 2 Equivalent Units

(Step 1) Physical Flow 145,000 5,000

Direct Materials 145,000 ---

Cartons 145,000 ---

---

5,000

---

---

--150,000

----150,000

----145,000

4,750 --149,750

Conversion 145,000 ---

Details Costs accounted for (Step 3) Divide by equivalent units (Step 4) Unit costs

Total Costs £3,767,000

Direct Materials £2,250,000

£25.20

÷150,000 £15.00

2. Application of costs (Step 5): To units completed, 145,000 × £25.20 £3,654,000 Work in process, end, 5,000 units: Direct materials 75,000 Conversion costs 38,000 Work in process, end 113,000 Total costs accounted for £3,767,000

..

Conversion Cartons Costs £319,000 £1,198,000 ÷145,000 £2.20

÷149,750 £8.00

5,000×£15.00 4,750×£8.00

639


14-48

(25-30 min.)

Flow of Production Work in process, beg. inv. Started To account for Completed and transferred out during current period Work in process, end. inv. Units accounted for Work done to date

(Step 1) Physical Units

(Step 2) Equivalent Units Transferred- Direct In-Costs Materials Conversion

10,000 (50%)* 50,000 60,000 40,000 20,000 (30%)* 60,000

Costs: Work in process, beg. inv. Costs added currently (Step 3) Total costs to account for $736,000 (Step 4) Divisor, equivalent units for work done to date Cost per equivalent unit $15.00 (Step 5) Application of Costs Completed and transferred 40,000 units × $15.00 $600,000 Work in process, ending inv. (20,000 units): Transferred-in-costs $100,000 Direct materials 0 Conversion costs 36,000 Total work in process $136,000 Total costs accounted for $736,000

40,000 20,000

40,000 0

40,000 6,000

60,000

40,000

46,000

Transferred- Direct In Costs Materials

Conversion Costs

$ 50,000 $ 40,000 250,000 120,000 $300,000 $160,000

$ 30,000 246,000 $276,000

÷ 60,000 $ 5.00

÷ 46,000 $ 6.00

÷ 40,000 $ 4.00

20,000×$5.00 0×$4.00 6,000×$6.00

*Degree of completion for conversion costs.

..

640


14-49

(15-20 min.)

1.

Materials and parts inventory Accounts payable or cash

273,800

Conversion costs Accrued payroll, accounts payable, accumulated depreciation, etc.

89,600

2.

273,800

89,600

Finished goods inventory (12,800 × $28) 358,400 Materials and parts inventory (12,800 × 21) Conversion costs (12,800 × $7)

268,800 89,600

Cost of goods sold Finished goods inventory

358,400

358,400

All costs incurred during April are charged to cost of goods sold in April. This assumes that all altimeters are sold and shipped immediately upon production. Therefore, the balance in Finished Goods Inventory is zero at the end of the month. 3.

Because the balance in the Conversion Costs account must be zero at the end of the month, and because only $89,600 was transferred out of the Conversion Costs account while $90,600 was added to the account, the remaining $1,000 must be transferred to Cost of Goods Sold: Cost of goods sold Conversion costs

..

1,000 1,000

641


14-50 (15-20 min.) Amounts are in dollars. Direct Materials 90,000

Work in Process Beg. Bal. 120,000 To Fin. Gds. 220,000 Dir. Materials 90,000 Dir. Labor 130,000 Applied overhead 221,000* End. Bal.

Finished Goods 70,000 160,000 220,000

341,000

Cost of Goods Sold 160,000

130,000 *141,000 actual + 80,000 overapplied 1. Cost of goods sold comes from the finished goods account: 70,000 + 220,000 - Cost of goods sold = 130,000 Cost of goods sold = 160,000. 2. Let EB = Ending balance in work in process 120,000 + 90,000 + 130,000 + 221,000 – 220,000 = EB EB = 341,000. 3. Budgeted rate (from WIP entries) is 221,000/130,000 = 170% of direct labor costs. 4. Since overhead is overapplied by $80,000, the adjusted CGS = 160,000 – 80,000 = 80,000.

..

642


14-51 (15-20 min.) Amounts are in dollars. Direct Materials 20,000 50,000 50,000 20,000

Finished Goods 100,000 500,000 600,000 200,000

Work in Process Beg. Bal. 300,000 To Fin. Gds. 600,000 Dir. Materials 50,000 Dir. Labor 100,000 Applied overhead 450,000 900,000 End. Bal. 300,000 Cost of Goods Sold 350,000

1.

Direct materials used: 20,000 + 50,000 – Direct materials used = 20,000 Direct materials used = 50,000

2.

Factory overhead applied = (50,000 + 100,000) × 300% = $450,000.

3.

Cost of Goods manufactured is (from the WIP account): Let CGM = cost of goods manufactured 300,000 + 50,000 + 100,000 + 450,000 – CGM = 300,000 CGM = 600,000.

4.

Cost of goods sold before adjustment for over- or under-applied overhead comes from the finished goods account: Unadjusted Cost of goods sold = 100,000 + 600,000 - 200,000 = 500,000. Adjusted Cost of goods sold is given in the problem as 350,000. Since the company does not prorate and adjusted cost of goods sold is 150,000 less than unadjusted cost of goods sold, overhead must be overapplied by 150,000. Therefore: Actual factory overhead = $450,000 - $150,000 = $300,000.

..

643


14-52 (20-25 min.) 1. Contract manufacturers make apparel, footwear, and equipment products for Nike. Almost all these products would be made through process manufacturing so the costing system would be a process-cost system. Nike distribution centers would probably use a job-order system based on retail account orders. NIKE’s three significant distribution centers in the United States for NIKE Brand products are located in Memphis, Tennessee. Other distribution centers include Foothill Ranch, California, Greenland, New Hampshire, and Ontario, California. There are about 20,000 retail accounts (customers) in the United States. 2. The choice of cost-allocation base is a key to accurate costing of products and customers. Managers at the various distribution centers give much thought to the selection of plausible and reliable cost-allocation bases since there are many from which to choose for a specific activity- or cost-resource pool. The suggestions below should serve as a starting point for discussion. The cost of the forklift resource could be allocated to the listed activities using the number of hours the forklift is used for each activity. The ABC system designer would ask a knowledgeable manager for a percentage breakdown of the time the forklift is used for each of the activities. Then these percentages would be multiplied times the total cost of operating the forklift to calculate the allocation. Occupancy costs could be allocated using the square feet of space used by each activity. Variable computer costs could be allocated using number of lines entered on the related source document (purchase order for supplies, sales order from customers, and invoices for payment from customers). 14-53 (30-40 min.) For the solution to this Excel Application Exercise, follow the stepby-step instructions provided in the textbook chapter. 1.

$304,000

2.

$1,358,000

3.

Materials, $11.00; Conversion, $8.40.

14-54

(45 min. or more)

The purpose of this exercise is to make students think about the characteristics of real production processes and how to account for them. Depending on the assumptions students make about the type of production process used in each of these examples, they

..

644


may suggest a different type of accounting system than those listed on the next page. These are just suggestions about what the groups might conclude. a. b. c. d. e. f. g.

Process costing, because there are large volumes of identical product. Process costing. Although each application is unique, it is likely that identifying the differences and trying to account for them is not cost-benefit efficient. Probably job costing. It depends on how many identical couches students think that Ethan Allen makes at one time. If each is unique, or if small batches are produced, a job-costing system is most likely used. Job costing. Major construction projects are generally treated as a single job. Process costing. Refining oil into gasoline is a classic process-costing environment, where there is a single continuous process. Job costing. Each order at FedEx Office’s is unique. The only question is whether it is cost-benefit efficient to determine job costs for each order. Job costing. Each ferry built is a single job, although there may be parts that are produced in a process that allows process costing.

14-55 (30-40 min.) NOTE TO INSTRUCTOR. This solution is based on the Web sites as they were in late 2012. Be sure to examine the current Web sites before assigning this problem, as the information may have changed. 1.

Land’s End is mainly a merchandising firm. The main activity is selling clothing to individuals, although the firm also engages in some corporate sales. Process-costing systems are used to determine the average cost of like products that are produced. Land’s End purchased large quantities of finished products but the costs are readily determinable so averaging is not necessary.

2.

La-Z-Boy is a manufacturing firm. Its main activity is manufacturing furniture. The firm states that it has an extensive line of furniture products – in many types and designs. The decision by La-Z-Boy to use a job-order cost system or a processcosting system (or some hybrid) depends on the nature of the products and production system. If the company produces large enough quantities of a type of furniture using the same processes, a process-cost system might be best. For small quantities of products that are custom manufactured for a specific customer, a joborder costing system might be best.

3.

Tasty Baking Company is a manufacturing firm. The firm makes large volumes of snack cakes in a continuous process. The cakes are moving in a continuous flow through the factory. Because each cake is alike, produced in large volumes with small unit costs, a process-costing system would be ideal.

..

645


CHAPTER 15 COVERAGE OF LEARNING OBJECTIVES

LEARNING OBJECTIVE LO1: Read and interpret the basic financial statements. LO2: Analyze typical business transactions using the balance sheet equation. LO3: Distinguish between the accrual basis of accounting and the cash basis of accounting. LO4: Make adjustments to the accounts under accrual accounting. LO5: Explain the nature of dividends and retained earnings. LO6: Select relevant items from a set of data and assemble them into a balance sheet and an income statement. LO7: Distinguish between the reporting of corporate owner’s equity and the reporting of owner’s equity for partnerships and sole proprietorships. LO8: Explain the role of auditors in financial reporting and how accounting standards are set. LO9: Identify how the measurement conventions of recognition, matching and cost recovery, and stable monetary unit affect financial reporting. L10: Define continuity, relevance, faithful representation, materiality, conservatism, and cost-benefit (Appendix 15A). LO11: Use T-accounts, debits, and credits to record transactions (Appendix 15B). ..

FUNDAMENTAL ASSIGNMENT MATERIAL B3

EXCEL, COLLAB., & INTERNET EXERCISES 54

A3, B3, B3

ADDITIONAL ASSIGNMENT MATERIAL 2, 24, 25, 32, 38, 51 27, 29, 29, 30, 34, 52 34, 37, 38, 39, 46, 46, 51 26, 40

B3

7, 10, 35, 35, 36

B3

13, 31, 31, 33, 33, 48, 48, 49, 49, 51 39, 40, 40, 47, 47, 52 48, 48, 49, 49, 51

A1, A2, B1, B3

A2, B3

53

B3

50, 50

54

B3

15

54

B3

19

B3

28

B3

41, 42, 43, 44, 45

646


CHAPTER 15 Basic Accounting: Concepts, Techniques, and Conventions 15-A1 (20-30 min.) Case 1.

E = 140 - 120 = 20 D = 40 + 20 = 60 C = 25 because there were no additional investments by stockholders A = 80 - 25 - 40 = 15; or 80 - (25 + 40) = 15 B = 90 - 25 - 60 = 5; or 90 - (25 + 60) = 5

Case 2.

K = 20 + 170 = 190 J = 55 + 20 - 5 = 70 H = 10 + 30 = 40 F = 55 + 10 + 90 = 155 G = 275 - 70 - 40 = 165

Case 3.

P = 300 - 270 = 30 Q = 90 + 30 - 110 = 10 N = 85 - 35 = 50 L = 105 + 50 + 90 = 245 M = 95 + 85 + 110 = 290

This problem was designed for an equation-type solution, but some students may find a different approach more helpful in understanding the solution and its steps. Such an approach can be easily developed on the board as follows, using Case 1 as an example: Given: Liabilities Paid-in capital Retained earnings Total (equal to total assets) Revenues Expenses Net earnings

..

Beginning A 25 40 80

Ending Steps: B 1. A = 80 - (40 + 25) = 15 C 2. E = 140 - 120 = 20 D 3. D = 40 + 20 - 0 = 60 90

4. 5.

C = 25 + 0 = 25 B = 90 - (60 + 25) = 5

140 120 E

647


15-A2 (40-55 min.) 1.

See Exhibit 15-A2 on the following page.

2.

MONTHA COMPANY Income Statement For the Month Ended April 30, 20X1 Sales (revenue) Deduct expenses: Cost of goods sold Wages, salaries and commissions Rent, 2,000 + 11,000 Depreciation Total expenses Net income

$110,000 $40,000 49,000 13,000 1,000 103,000 $ 7,000

MONTHA COMPANY Balance Sheet April 30, 20X1 Liabilities and Stockholders' Equity

Assets Cash Accounts receivable Merchandise inventory Prepaid rent Equipment and fixtures, net Total assets 3.

..

$ 74,000 55,000 20,000 4,000 35,000 $188,000

Liabilities: Accounts payable Note payable Total liabilities Stockholders' equity: Paid-in capital Retained earnings Total stockholders' equity Total liabilities and stockholders’ equity

$ 7,000 24,000 $ 31,000 $150,000 7,000 157,000 $188,000

Most businesses tend to have net losses during their start-up phase, so Montha's ability to show a net income for April is good. Many points can be raised, including the problem of maintaining an "optimum" cash balance so that creditors can be paid neither too quickly nor too slowly. See the next solution also.

648


EXHIBIT 15-A2 MONTHA COMPANY Analysis of Transactions for April 20X1 (in thousands of dollars) Assets = Liabilities + Stockholders’ Equity MerPreEquipLiabilities Stockholders' Equity Accounts chandise paid ment and = Note Accounts Paid-in Retained Cash + Receivable +Inventory +Rent + Fixtures = Payable +Payable + Capital + Earnings +150 = +150

Description a. Incorporation b. Purchased merchandise -35 +35 c. Purchased merchandise +25 d1.Sales +35 +75 d2. Cost of inventory sold -40 e. Collections +20 -20 f. Disbursements to trade creditors -18 g. Purchased equipment -12 +36 h. Prepaid rent - 6 +6 i. Rent expense -11 j. Wages, etc. -49 k. Depreciation - 1 l. Rent expense -2 Balances, April 30, 20X1 +74 +55 +20 +4 +35 ____________________________________ 188

..

= = = = =

+25

= = = = = = = =

-18

+110(revenue) - 40(expense)

+24 - 11(expense) - 49(expense) - 1(expense) - 2(expense) +24 + 7 +150 + 7 _________________________ 188

649


15-A3 (5-10 min.) Revenue (cash basis): Cash sales Cash collected from credit customers Total revenue

$35,000 20,000 $55,000

The accrual basis provides a more accurate measure of economic performance. As long as the two recognition criteria are met (earned and realized), the $110,000 measure of revenue on the accrual basis is preferred to the $55,000 measure of revenue on the cash basis. The $110,000 is the more accurate measure of accomplishments for April. 15-B1

(10-15 min.) This is straightforward. Computations are in millions of dollars. A = 6,579 - (1,066+506) = 5,007 B = 9,700 - 613 = 9,087 C = 506 + 613 – 266 = 853 D = 1,066 + 102 = 1,168 E = 5,441 + 1,168 + 853 = 7,462

Instructors may wish to comment about the $102 million additional investments by stockholders. For Nordstrom this was primarily the exercise of stock option, shares issued as part of an employee stock purchase plan, and stock-based compensation. No shares were sold to the general public.

..

650


15-B2

(30-40 min.)

1.

See Exhibit 15-B2 on the following page.

2.

THE VOLVO GROUP Statement of Earnings For the Month Ended January 31, 2012 (in millions) Sales Deduct expenses: Cost of goods sold Selling and administrative expenses Rent and insurance expense Depreciation Total expenses Net earnings

SEK650 SEK 390 110 90 20 610 SEK 40

THE VOLVO GROUP Balance Sheet January 31, 2012 (in millions) Liabilities and Stockholders' Equity

Assets Cash Accounts receivable Inventories Property, plant, and equip., net Other assets Total 15-B3

SEK 30,059 81,632 44,709

Accounts payable Other liabilities Stockholders’ equity

SEK 56,596 211,017 85,721

53,637 143,297 SEK 353,334

Total

SEK 353,334

(5-10 min.) Revenue (cash basis): Cash sales Collections from credit customers Total revenue

SEK 190,000,000 300,000,000 SEK 490,000,000

The accrual basis provides a more accurate measure of economic performance. As long as both recognition criteria are met (earned and realized), the SEK 650 million measure of revenue on the accrual basis is preferred to the SEK 490 million measure of revenue on the cash basis. The SEK 650 million is the more accurate measure of accomplishments for January. ..

651


EXHIBIT 15-B2

THE VOLVO GROUP Analysis of Transactions for January 2012 (in millions of SEK) Assets

Transaction Balances 1/1/12 a1. a2. b. c. d. e. f. g. h. Balances, 1/31/12

..

=

Liabilities + Stockholders’ Equity Property Liabilities Stockholders' Equity Accounts Inven- Plant, & Other = Accounts Other Paid-in Capital & Cash + Receivable + tories +Equip. + Assets = Payable + Liabilities Retained Earnings +30,379 +81,472 +44,599 +53,657 + 143,137 =+56,546 +211,017 +85,681 +190 +460 = +650(increase revenue) -390 = -390(increase expense) +500 = +500 +300 -300 = -250 +250 = -450 = -450 -110 = -110(increase expense) - 90 = - 90(increase expense) - 20 = - 20(increase expense) +30,059 +81,632 +44,709 +53,637 +143,297 =+56,596 +211,017 +85,721 _____________________________________ ________ ________________ +353,334 +353,334

652


15-1 The income statement answers questions about financial performance over a span of time. The balance sheet answers questions about financial status at a point in time. 15-2 The relationship between assets and liabilities can be expressed by means of the following equation: Assets = Liabilities + Owner’s Equity 15-3 The income statement is a link between two balance sheets. The income statement provides details about how stockholders’ equity, specifically retained earnings, changes over a period of time. 15-4 This statement is fallacious because it does not take into consideration withdrawals (dividends) or increases in investment by owners, both of which affect the ownership capital account but not net income. 15-5 Under the accrual basis, companies recognize revenue as it is earned and realized (or realizable) and record expenses in the period when resources are consumed. In contrast, the companies using the cash basis recognize revenue when they collect cash and record most expenses when they pay out cash. 15-6 Adjusting entries differ from routine entries in that they deal with implicit transactions in contrast to the explicit transactions that trigger nearly all the day-to-day routine entries. 15-7 Acquisition leads to an asset that refers to the unexpired costs. When such assets are used, their costs are said to be expired. Thus, assets (unexpired costs) are included in the balance sheet whereas expired costs are shown in the income statement. 15-8 It is preferable to refer to the costs rather than the values of assets such as plants or inventory because the word value has many meanings and is more vague than the word cost. Cost explicitly recognizes that balance sheet amounts are based on the amounts spent on assets, not their current values. Accountants assume that asset costs reflect their value at acquisition, but costs may differ greatly from value in subsequent periods. 15-9 Yes. Depreciation is simply the allocation of the acquisition cost (less estimated residual value) of assets over the periods that benefit from the asset's use. It is not a measure of the changes in market value of an asset. 15-10 Unearned revenue is a liability because the company is obligated to either deliver the goods or services promised against the money received or to refund the money altogether.

..

653


15-11 Accrued wages are payments due to employees for work already done but not yet paid for. It is a liability account on the balance sheet. Accruing an expense means charging the expense on the income statement before paying the obligation in cash. 15-12 Retained earnings is neither an asset nor a preferred claim against cash or any other asset, but it represents a general claim against total assets. In most instances no "pot of gold" exists, as the cash inflow from operations will be largely reinvested in other assets. 15-13 Even if there is large balance in retained earnings, the ability of a company to pay cash dividend largely depends on its cash position and the future needs for cash to pay debts or to purchase additional assets. 15-14 The basic principles of accounting apply to all businesses. Accounting for assets, liabilities, revenues, and expenses does not depend on whether the business is a sole proprietorship, a partnership or a corporation. There are slight differences in accounting for owners’ equity, but the differences are in details and not in general principles. 15-15 Yes. Although financial statements are a precise presentation of the overall position and performance of a firm, they are the result of a complex measurement process that depends on a large number of assumptions and conventions difficult to verify. Further, it is practically impossible to verify every bit of data that goes into the financial statements. 15-16 This statement is not true. The capital markets are increasingly global, and IFRS are used in more than 100 countries, including those in the European Union. In addition, the SEC allows companies headquartered outside the U.S. who trade their shares on U.S. markets to use IFRS for their reports to the SEC. It is probably only a matter of time until the SEC will allow U.S. companies to report using IFRS. 15-17 Accountants create value through the information they supply to decision makers. If users cannot trust the information, it will have little value. Therefore, accountants must be especially careful to adhere to strict ethical guidelines and exhibit the utmost integrity. 15-18 Accountants record revenue when 1) the company has earned it and 2) the revenue is realized or realizable. This is important because it triggers the recognition not only of the revenue, but also of the related expenses (via the matching principle). 15-19 The major limitation of the basic measurement unit in accounting is that it ignores the change in the purchasing power of a currency. As a result, the figures shown in financial statements against various assets and liabilities are hardly additive. This may seriously mislead the decision makers in countries with hyperinflation.

..

654


15-20 The going-concern concept means that a company is expected to use existing resources to fulfill the general purpose of a continuing business rather than sell them in tomorrow's real estate or equipment market. 15-21 Relevance is the capability of information to make a difference to the decision maker. Faithful representation is a quality of information that ensures that it captures the economic substance of the transactions, events, or circumstances it describes. It requires information to be complete, neutral, and free from material errors. 15-22 Economic feasibility sometimes inhibits the adoption of new ways to measure financial performance and position because the apparent benefits may not exceed the obvious costs of gathering and interpreting the information. 15-23 Debits are left-side entries and credits are right-side entries in a double-entry system. The debits must equal the credits for each transaction recorded. 15-24 Because accountants record land at its historical cost and do not depreciate it, the land purchased in 1912 would still be listed at its purchase price. The current market value of that land more than 100 years later is likely to be substantially more than the amount on the books. The equipment purchased in 2012 would be listed at its purchase price less the depreciation taken on it since 2012. This net book value may be more or less than the current market price of the equipment. Even though the book value of the equipment may differ from its market price, the difference between the book value and the market price of the equipment is likely to be much less than the difference between the book value of the land and its market price. 15-25 A marketing manager generally focuses on changes in assets, not the general level of the assets. Marketing decisions deal with generating revenues and the costs of those revenues. Such items are recorded on the income statement. Although the income statement and balance sheet articulate, that is, the income statement explains changes in the retained earnings section of the balance sheet, the most direct measures of marketing performance are in the income statement. 15-26 A principle of good performance measures is that they recognize performance as close as possible to the time of the performance. If the goal of the sales staff is to make sales, then such staff should be evaluated on the amount of sales made. Any delay between the making of the sale and the recognition of sales staff performance weakens the motivational effects of the performance measure. One potential delay is the time between making the sale and the time the company receives payment for the goods or services. Accrual accounting does not wait until the receipt of cash before recognizing a sale, whereas cash accounting does. Thus, accrual accounting generally provides a more relevant sales performance measure than does cash-basis accounting. 15-27 We know that the ending balance in stockholders’ equity will be the beginning balance plus net income, $250,000 + $50,000 = $300,000, and the difference between the

..

655


amount of assets and the amount of liabilities will also be $300,000. But we do not know the exact amount of the assets and the liabilities. 15-28 Relevance means the information makes a difference to the decision maker. If information has no impact on a decision, it is not relevant to that decision. Information that cannot affect a decision has no value to a decision maker. If it has predictive value, users can use the information to help them form their expectations about the future. If it has confirmatory value, it can confirm or change existing expectations. Information that faithfully represents the transactions, events, or circumstances it describes is important because users need to rely on financial statements to convey the information it purports to portray. The information should not leave out important facts, it should be neutral (free from bias), and it should be free from material errors. 15-29

(10-15 min.)

1.

F. Adjustments (adjusting entries) are entries that record implicit transactions such as unpaid wages, prepaid rent, interest owed, and the like.

2.

F. Cash dividends are distributions of cash to stockholders. They reduce retained earnings.

3.

F. Accounts payable should be classified as a liability.

4.

T

5.

T

6.

T

15-30 (5-10 min.) This is a simple exercise showing how the two sides of the balance sheet must always balance. LIABILITIES AND ASSETS STOCKHOLDERS’ EQUITY Cash $ 10,000* Accounts payable $ 10,000 Accounts receivable 15,000 Long-term debt 45,000*** Plant & equipment 75,000 Stockholders’ equity $ 45,000 Total assets $100,000 Total Liab. and stk. equity $100,000** *$100,000 - $15,000 - $75,000 **Equal to total assets, $100,000 ***$100,000 - $10,000 - $45,000

..

656


15-31

(15-20 min.)

The theme of this solution is that retained income is not a pot of cash awaiting distribution to stockholders. 1.

2.

ASSETS Cash

¥6,000

Cash Inventory Total assets

¥3,000 3,000 ¥6,000

LIABILITIES & OWNERS’ EQUITY Paid-in capital ¥6,000 Paid-in capital

¥6,000

Note in both requirements 1 and 2 that the ownership equity is fundamentally a claim against the total assets (in the aggregate). For example, 3/4 of the shareholders do not have a specific claim on cash, and 1/4 of the shareholders do not have a specific claim on inventory. Instead, they have an undivided claim against (or interest in) all of the assets. 3.

Cash

¥8,100

Total assets

¥8,100

Paid-in capital Retained earnings Total liabilities and stockholders’ equity

¥6,000 2,100 ¥8,100

Retained earnings is part of the stockholders’ equity. Even though cash and retained earnings have increased by identical amounts over their balance in part 1, the retained earnings is fundamentally a general interest in total assets (just as paid-in capital is a general interest in total assets). Retained earnings is the net rise in ownership claim attributable to profitable operations. However, the assets themselves should not be confused with the claims against the assets. 4.

Cash Inventory Equipment Total assets

¥1,200* 2,400 4,500 ¥8,100

Paid-in capital Retained earnings Total liabilities and stockholders’ equity

¥6,000 2,100 ¥8,100

*(¥8,100 – ¥2,400 – ¥4,500)

..

657


Transaction 4 should clarify the lack of a specific link between retained earnings (and paid-in capital) and any particular assets discussed in requirement 3. The ownership claims are general, not specific. 5.

Cash Inventory* Equipment

¥ 1,200 4,500 4,500

Total assets

¥10,200

Accounts payable Paid-in capital Retained earnings Total liabilities and stockholders’ equity

¥ 2,100 6,000 2,100 ¥10,200

*(¥2,400 + ¥2,100) The meaning of retained earnings was explained in answer 3. Purchases on “open account” usually create a general liability; that is, the trade creditors usually hold only general claims against the total assets, not specific claims against particular assets (as created by mortgages on buildings). In sum, both the creditors and the owners hold general claims against the assets. Of course, if the corporation is liquidated (all assets converted to cash to be distributed to claimants), the creditors’ general claims must be satisfied before the owners get any amount. Thus, the stockholders are said to have residual claim or residual interest. 15-32

(10-15 min.)

1.

The name of the statement is antiquated. It is usually titled income statement or statement of earnings or sometimes a statement of operations.

2.

The date should not be for a point in time but for an indicated span of time.

3.

Companies do not usually recognize increases in market values when using historical cost accounting.

4.

Dividends are not expenses, and companies do not deduct them when calculating net profit.

5.

The appropriate deduction is the cost of goods (clothes) sold, not cost of goods (clothes) purchased.

6.

The bottom line is more often titled net income or net earnings, although net profit is acceptable.

7.

“Cash received from loan” should not be listed on the income statement.

..

658


8.

Although this is not the major point of the problem, the income statement has apparently omitted some expenses. For example, neither rent nor depreciation is shown; at a minimum, one or the other would ordinarily be included, even if no cash payment occurs during the period.

..

659


15-33 (5-10 min.) This is a simple exercise showing how the income statement and balance sheet are linked by the retained earnings account. The purchase of property and equipment is irrelevant to this exercise because it does not affect the retained earnings. From the income statement, we determine the net income: Total revenues £1,060,000 - Total costs 990,000 Net income £ 70,000 Calculation of ending retained earnings: Beginning retained earnings + Net income - Dividends Ending retained earnings

15-34

£113,560 70,000 24,000 £159,560

(10 min.)

Amazon Fremont Studios A = L + SE A = L + SE Prepaid Unearned Meeting Meeting Sales Sales Cash Expense Expense Cash = Revenue Revenue 1. Dec. payment –100,000 +100,000 = +100,000 = +100,000 2. May meeting –100,000 = –100,000 = –100,000 +100,000

15-35

(10-15 min.)

Basu Enterprises, Property Owner A = L+ SE Prepaid Cash Tax 1. –19,600 +19,600 2. –4,900 3. –4,900 4. –4,900 5. –4,900

..

Tax Expense = = = = =

–4,900 –4,900 –4,900 –4,900

Kolkata Municipal Corp. A = L + SE Unearned Tax Tax Cash Revenue Revenue +19,600 = +19,600 = –4,900 +4,900 = –4,900 +4,900 = –4,900 +4,900 = –4,900 +4,900

660


15-36 (10 min.) 1. At the end of 2012, the company has not yet delivered the magazines, so it cannot recognize any revenue. Thus, there is no impact on the 2012 income statement. The entire 5,000 × €50 = €250,000 will be a liability on the balance sheet called “unearned subscription revenue” or “deferred subscription revenue.” During the first six months of 2013, the company will fulfill half of the subscription obligations, so €125,000 will be deducted from the unearned revenue account and added to the revenue for 2013. 2. The monthly salaries were €240,000 ÷ 12 = €20,000. At the end of 2012, the company will owe €20,000 for salaries earned in December. This will be a liability for “salaries payable” on the balance sheet. The 2012 income statement will include all of the €240,000 salaries earned in 2012, including the €20,000 remaining to be paid in January 2013. 15-37

(15-20 min.)

Dec. 31, 20X0: Jan. 1, 20X0: Change:

Assets - Liabilities = Stockholders' equity B125,000 - B45,000 = B80,000 B 80,000 - B30,000 = B50,000 B 45,000 - B15,000 = B30,000

1. As above, B50,000. This is the easiest computation. 2. Change in stockholders' equity + Cash dividends = Net income B30,000 + B16,000 = B46,000 3. Let X = Cost of goods sold Sales - Cost of goods sold - Operating expenses = Net income B265,000 X B50,000 = B46,000 -X = B46,000 - B265,000 + B50,000 X = B169,000

..

661


15-38

(20-30 min.) 1 $11,000

2 $3,000 6,000 2,000

X Y Z A B Computations: 1: X = $9,000 + 2: X = $9,000 : Y = $11,000 : Z = $5,000 3: X = $6,000 + : Y = $16,000 : Z = $7,000 4: X = $8,000 + 5: X = $3,000 + 6: X = $14,000 : Y = $6,000 : Z = $8,000 + : A = $10,000 : B = $4,500 + 7: X = $8,200 : Y = $9,900 : Z = $3,900 + : A = $100 + : B = $20,000 -

3 $9,000 7,000 3,000

$4,000 $ 6,000 = $5,000 = $3,000 = $10,000 $9,000 = $4,000 = $12,000 $4,500 = $6,000 = $4,000 = $7,000 $2,000 + $12,000 = $4,000 = $6,000 = $ 400 $4,700 = $4,800 =

Case 4 $14,000

$2,000 = $3,000 $6,000 $2,000 $7,000 = $7,000 $3,000 $6,000 = $7,500 $8,000 $2,000 $6,000 = $1,500 $16,500 $4,200 $3,900 $4,200= $4,800 $15,200

5 $7,500

6 $ 8,000 2,000 9,000 4,500 16,500

7 $ 4,200 3,900 100 4,800 15,200

$11,000

$ 9,000 $14,000

$9,000 $5,000 = $4,500

$100

Note: The formula for cost of goods sold is not discussed in the chapter, but it is given in the problem at the bottom of Exhibit 15-8. The following framework may help on cases 6 and 7: Stockholders' equity: Beginning Additional investments Net profit Dividends End

Case 6 ? =

4,500 +5,000 Y = 6,000 - 4,000 = +2,000 -1,500 $10,000

Case 7 $8,200 - 4,000 = 4,200 0 ? = 100 -400 $9,900 - 6,000 = 3,900

In case 6, the $4,500 is the beginning balance, $10,000 - ($5,000 + $2,000 - $1,500) = $4,500. In case 7, the net profit of $100 is ($3,900 + $400) - $4,200 = $100.

..

662


15-39

(45-75 min.)

1.

See Exhibit 15-39 on the following page.

2.

CONNECTIVITY PLUS Statement of Income For the Month Ended April 30, 20X1 Sales Cost of goods sold Gross profit Operating expenses: Rent Depreciation Advertising Wages and salaries Miscellaneous Operating income Interest expense Net income

..

$62,000 31,000 $31,000 $ 500 50 9,000 11,000 2,510

23,060 $ 7,940 40 $ 7,900

663


EXHIBIT 15-39 Assets

= Liabilities + Stockholders’ Equity PreFixtures Liabilities Stockholders' Equity Accounts paid & Equip- Accounts Notes Accr. Accr. Paid-in Retained Trans. Cash + Receivable +Inventory +Rent + ment = Payable +Payable + Wages + Int. + Capital + Earnings a. +36,000 = +36,000 b. -20,000 +40,000 = +20,000 c1. - 1,000 +1,000 = c2.* - 500 = - 500 (E) d1. = + 3,000 - 3,000 (E) d2. - 6,000 = - 6,000 (E) e1. +10,000 +52,000 = +62,000 (R) e2. -31,000 = -31,000 (E) f1. - 4,000 = - 4,000 (E) f2. = +7,000 - 7,000 (E) g. - 2,510 = - 2,510 (E) h. - 1,000 +6,000 = +5,000 i. = +40** 40 (E) j. - 50 = 50 (E) k. - 6,000 = - 6,000 (D) Balance 4/30 + 5,490 +52,000 +9,000 + 500 +5,950 = +23,000 +5,000 +7,000 +40 +36,000 + 1,900 *This and other adjustments could be made at the end of this series of entries. **.096 × $5,000 × 1/12 = $40

..

664


CONNECTIVITY PLUS Balance Sheet April 30, 20X1 Assets

Equities

Cash Accounts receivable Inventory Prepaid rent Fixtures and equipment, net

$ 5,490 52,000 9,000 500 5,950

Total assets

$72,940

Liabilities: Accounts payable $23,000 Notes payable 5,000 Accrued wages and salaries payable 7,000 Accrued interest payable 40 Total Liabilities $35,040 Stockholders' equity: Paid-in capital $36,000 Retained earnings 1,900 37,900 Total equities $72,940

CONNECTIVITY PLUS Changes in Retained Earnings For the Month Ended April 30, 20X1 Retained earnings, April 1, 20X1 Add: Net income for April Total Deduct: Cash dividends Retained earnings, April 30, 20X1 3.

$

0 7,900 $7,900 6,000 $1,900

The picture in this set of financial statements is not unusual for new businesses. Some of the liabilities are very current: accounts payable, $23,000, and accrued wages, $7,000. These are much larger than the $5,490 of cash. Unless much of the accounts receivable can either be collected or discounted (sold to a bank or other lender) the company may be unable to meet its payroll and pay its bills on time. Moreover, the inventory badly needs replenishment if sales are to continue at their current pace. Payment of a $6,000 dividend may not have been wise. Many new businesses can show a respectable net income but simultaneously be at the brink of financial disaster because they are "under-capitalized." That is, there is insufficient long-term investment capital to sustain a smooth growth. Too often, creditors and employees need cash far in advance of when customers provide the cash to the business. This may be such a case, unless customers pay promptly.

..

665


15-40 1.

(30 min.)

ATTORNEY JAMES BROWN Income Statement For the Year Ended December 31, 2020 Cash Basis Accrual Basis Fee revenue $158,000 $194,000 1 Expenses: Rent $ 15,000 $ 12,000 2 Utilities 1,400 1,600 3 Salaries 32,000 34,000 4 Depreciation 28,000 28,000 5 Total expenses $76,400 $75,600 Operating income $81,600 $118,400 1

$158,000 collected – $4,000 unearned + $40,000 receivable $15,000 – $3,000 applicable to the first quarter of 2021 3 $1,400 + $200 owed 4 $32,000 + $2,000 owed 5 $168,000  6 = $28,000 2

The term “cash basis” is ambiguous. A strict interpretation of cash basis would permit deducting the full $168,000 paid for equipment as an expense in 2020. Operating income would be $109,600 – $168,000 paid for equipment = a loss of $58,400. Or depreciation could be ignored, giving a net income of $54,800. But the problem says to charge depreciation on either the cash or accrual basis, so the income statements would appear as above. 2.

The accrual basis provides a better measure of economic performance because it encompasses all assets and liabilities arising from operations rather than their immediate cash effects alone. For example, the $4,000 advance payment has not yet been earned and therefore represents an obligation for James. However, the $40,000 fees billed have been earned and represent a legitimate economic resource of that magnitude (unless their full collectability is in doubt). The government permits the cash basis primarily to ease the cash demands on taxpayers and to reduce the record-keeping tasks of small businesses. In short, if you extend credit to your customers, the government does not feel it equitable to demand tax payment if you have not yet received your cash.

..

666


Remember, therefore, that income measurement may legitimately differ for different purposes. In this case, the cash basis may be the preferable way to measure income for tax purposes. But to measure his own economic performance as an attorney, James would probably prefer the accrual basis. This is a major point—there is nothing inherently evil about having “two sets of books.” 15-41

(5 min.)

1. Dr.

2. Cr.

15-42

(10 min.)

1.

8. 9. 10.

F. Purchases of inventory should be debited to Inventory and credited to Accounts Payable. F. Increases in asset accounts are entered on the left. T F. Decreases in liability accounts should be on the left (or decreases in asset accounts should be on the right). F. The first sentence is correct. However, credit entries always must be on the right. F. Amounts borrowed are debited to Cash and credited to Notes Payable. F. Decreases in assets are shown on the credit side, but decreases in liabilities and stockholders’ equity are shown on the debit side. T F. All credits are on the right. F. Payments on mortgages are credited to cash and debited to Mortgage Payable.

15-43

(10-15 min.)

2. 3. 4. 5. 6. 7.

3. Cr.

4. Cr.

5. Dr.

6. Cr.

7. Cr.

8. Cr.

1 and 2. These accounts will generally have a beginning balance. The balances are omitted in the following T-accounts. Cash

Dues Receivable a. 600

Equipment c. 210 b. 350

Accounts Payable c. 210

a. 600 b. 150 d. 200

..

Accounts Receivable b. 200 d. 200

667


15-44

(20-30 min.) See Exhibit 15-44 on the next page.

15-45

(20-40 min.) See Exhibit 15-45 on the page after next.

..

668


EXHIBIT 15-44 Cash 150 (b) 35 (f) 20 (g) (h) (i) (j) 74

(a) (d1) (e)

Bal. (d1)

Amounts are in thousands of dollars. 35 18 12 6 11 49

Accounts Receivable 75 (e) 20

(b) (c)

Merchandise Inventory 35 (d2) 40 25

(h)

Prepaid Rent 6 (l)

(g)

Equipment and Fixtures 36 (k) 1

Note:

..

2

Note Payable (g)

(f)

Accounts Payable 18 (c) 25

Cost of Goods Sold (d2) 40

(i) (l)

24

Rent Expense 11 2

Paid-in Capital (a)

150

Retained Earnings Net Inc. 7

Sales (d1)

110

Wages, Sal., & Comm. (j) 49

Depreciation Expense (k) 1

*Details of the revenue and expense accounts appear in the income statement. Their net income effect appears in Retained Earnings in the balance sheet. Ending balances should be drawn for each account, but most are not shown here because they can be computed mentally.

669


EXHIBIT 15-45 (a) (e1)

Bal.

Cash 36,000 (b) 20,000 10,000 (c1) 1,000 (d2) 6,000 (f1) 4,000 (g) 2,510 (h) 1,000 (k) 6,000 5,490

Note Payable (h) 5,000

Accr. Interest Payable (i) 40

Accounts Payable (b) 20,000 (d1) 3,000

Paid-in Capital (a) 36,000

Acc. Wages & Sal. Payable (f2) 7,000

Retained Earnings (k) 6,000 Net Inc.7,900

Accounts Receivable (e1) 52,000

(b)

(c1)

Inventory 40,000 (e2) 31,000 Prepaid Rent 1,000 (c2)

(e2)

Cost of Goods Sold 31,000

(c2)

Rent Expense 500

Advertising Expense (d1) 3,000 (d2) 6,000 500

Fixtures and Equipment (h) 6,000 (j) 50

Wages & Sal. Expense (f1) 4,000 (f2) 7,000

Sales (e1) 62,000

(j)

Depreciation Expense 50

(i)

Interest Expense 40

(g)

Miscellaneous Expense 2,510

Note: Ending balances should be drawn for each account, but they are not shown here because they can be computed mental

..

670


15-46

(10-15 min.)

1.

The bank’s assets (cash) and liabilities (deposits) would each increase by £10,000. The mix of assets would change, but total assets and liabilities and owners’ equity would not, assuming that the cash on hand had already been recorded as, say, cash on hand (asset) and capital (owners’ equity). If the latter recording had been made, the deposit would merely represent the transforming of one asset (cash on hand) into another (cash in bank); there would be no effect on liabilities or owners’ equities.

2.

Cash (asset) would increase by £20,000, and liabilities (note payable) would increase by the same amount.

3.

The bank’s total assets and liabilities would be unaffected. The only change would be in the form of assets. Cash would decrease by £450,000, and notes receivable would increase by the same amount.

15-47

(20 min.)

DAIMLER AG Balance Sheet August 28, 2019 (in millions of euros) Assets Liabilities and Stockholders' Equity* Cash € 48,108 Accounts payable € 78,528 Accts. receivable 11,580 Accrued liabilities 25,116 Inventory 79,656a Long-term debt 15,036 Property, net of Other liabilities 51,576 accumulated depreciation 149,184 Total liabilities 170,256 c Other assets 32,604 Common stock €54,216 Retained earnings 85,332 Other Stk. Equity 11,328 Total stockholders' equity 150,876 b Total assets €321,132 Total liab. and stk. equity** €321,132 *This is the heading used in most actual annual reports. **Same amount as total assets. (a) €321,132 – €48108 – €11,580 – €149,184 – €32,604 = €79,656 (b) €54,216 + €85,332 + €11,328 = €150,876 (c) €321,132 – €150,876 = €170,256 or €78,528 + €25,116 + €15,036 + €51,576 = €170,256 Note that total revenues is not a balance sheet account. ..

671


15-48

(20-25 min.)

The following statements follow the general format used by Facebook. Obviously, various alternatives are possible. The cash account is a balance sheet account and thus is irrelevant to this problem. 1.

FACEBOOK INC. Income Statement For the Year Ended December 31, 2018 (in millions) Revenues $ 55,838 Operating costs and expenses (30,925) Operating income 24,913 Net interest income 448 Income before taxes 25,361 Provision for income taxes (3,249) Net income $ 22,112

2.

FACEBOOK INC. Changes in Retained Earnings For the Year Ended December 31, 2018 (in millions ) Balance at beginning of year Net income for the year Dividends paid Balance at end of year

$33,990 22,112 ? $41,981

Facebook paid $14,121 cash dividends in the year ended December 31, 2018: $33,990 + $22,112 – Dividends = $41,981 Dividends = $14,121 3.

Facebook paid $14,121 cash dividend, which is 64% of the net income for the year. It means the management decided that funds generated by operations were better used for internal purposes, possibly for expanding operations and purchasing other companies, than paying out those funds entirely to stockholders as cash dividends.

..

672


15-49

(15-25 min.)

The following is Swift’s income statement. Students may use other acceptable formats. Accounts payable and cash are irrelevant. SWIFT INC. Income Statement Year Ended December 31,2020 (in millions) Net revenue Cost of products and services Gross profit Selling and administrative expenses C$17,048 Research, development, and engineering expenses 1,712 Interest and other expenses 382 Earnings before income taxes Income tax provision Net income

C$124,142 96,520 27,622

19,142 8,480 1,496 C$ 6,984

To determine cash dividend, we need to look at the items affecting Swift’s retained earnings account: SWIFT INC. Changes in Retained Earnings Year Ended December 31, 2020 (in millions) Balance at beginning of year + Net earnings - Dividends to shareholders Balance at end of year

C$49,488 6,984 ? C$56,472

Swift declared no cash dividends in the year ended December 31, 2020: C$49,488 + C$6,984 – cash dividends = C$56,472 Cash dividends = C$56,472 – C$49,488 – C$6,984 = C$0

..

673


15-50

(10 min.)

The sale of the new shares will bring in cash of €100,000 and add €100,000 to the existing €300,000 of capital, but instead of the capital being identified with a particular owner, it will simply be capital stock. Thomson Company Balance Sheet May 31, 2020 Assets Cash Accounts receivable Equipment Total assets 15-51

€160,000 50,000 290,000 €500,000

Liabilities and Owner’s Equity Accounts payable Other liabilities Capital stock (paid-in capital) Total liabilities and Owner’s Equity

€ 30,000 70,000 400,000 €500,000

(15-25 min.)

1. Percentage increase in total assets: ($14,998 ÷ $14,419) – 1 = 4.0% Percentage increase in revenues: ($20,862 ÷ $19,014) – 1 = 9.7% Total revenues increased by 5.7 percentage points more than assets. 2.

Assets $14,998

= Liabilities + Shareholders’ Equity = $5,155 + $9,843

3.

Beginning retained earnings Net income Dividends Other changes in retained earnings Ending retained earnings

4.

Percentage growth in net income, fiscal 2011: ($2,133 ÷ $1,907) – 1 = 11.9% Percentage growth in net income, fiscal 2010: ($1,907 ÷ $1,487) – 1 = 28.2%

$6,095 2,133 (569) (1,858) $5,801

The percentage growth experienced in fiscal 2010 was much higher than that in fiscal 2011.

..

674


15-52 (45-60 min.) For the solution to this Excel Application Exercise, follow the step-by-step instructions provided in the textbook chapter. 1,

$188,000

2. Stockholders’ equity increased by $7,000, the amount of net income for the month. 3. Cash decreased by $76,000. This is not unusual for a young company. Many of the sales were on credit, and the cash has not yet been received. Nevertheless, payments need to be made for purchases of inventory and payment of wages, rent, and other expenses. 15-53

(20 – 30 min.)

The purpose of this game is to help students identify different types of implicit transactions. Usually implicit transactions are harder for students to understand than explicit transactions, and this game makes students identify and classify a large number of implicit transactions. The game has an element of chance because of the roll of the die, and there is competition both within groups and between groups. The game will become more interesting and more challenging when the examples in the text have all been used and students must come up with their own examples. Students with experience in business will have an advantage in the competition, but it is also a good chance for students without such experience to learn from those with it.

..

675


15-54 (15-25 min.) NOTE TO INSTRUCTOR: This solution is based on the web site as it was in late 2012 and is based on the 2011 financial statements. Be sure to examine the current web site before assigning this problem, as the information there may have changed. 1.

McDonalds’ largest asset is property and equipment, comprising about 69% of the company’s assets. Assets such as inventories and prepaid expenses are unexpired costs. Accruals of unrecorded expenses include accrued interest and accrued payroll and other liabilities.

2.

One measure of the size of a company is its total assets. McDonalds’ total assets increased 3%, from $32.0 billion to $33.0 billion. This is shown on the balance sheet.

3.

McDonalds’ total revenues grew about 12.2%, from $24,075 million to $27,006 million. Meanwhile, net income increased from $4,946 million to $5,503 million, also a growth rate of 11.3%. It is a not good sign that revenues grew faster than income, though the difference here is not large.

4.

Each of the basic financial statements includes clues that McDonalds is a corporation. Most obvious is that each statement is labeled “consolidated.” The Income Statement shows information about the earnings and dividends per share and the number of shares outstanding. The balance sheet reports Shareholder’s Equity. The Statement of Shareholders’ Equity shows why the amounts in the various shareholders’ investment accounts on the balance sheet changed.

5.

The “Report of Independent Auditors” indicates that McDonalds’ financial statements comply with GAAP. The quote from the auditor’s report is: “[T]he financial statements . . . present fairly . . . in conformity with U.S. generally accepted accounting principles.” The notes to the financial statements also make many references to U. S. GAAP standards.

..

676


CHAPTER 16 COVERAGE OF LEARNING OBJECTIVES

LEARNING OBJECTIVE LO1: Recognize and define the main types of assets in the balance sheet of a corporation. LO2: Recognize and define the main types of liabilities in the balance sheet of a corporation. LO3: Recognize and define the main elements of the stockholders’ equity section of the balance sheet of a corporation. LO4: Recognize and define the principal elements in the income statement of a corporation. LO5: Recognize and define the elements that cause changes in retained earnings. LO6: Explain the purposes of the cash flow statement and identify activities that affect cash, and classify them as operating, investing, or financing activities. LO7: Assess financing and investing activities using the statement of cash flows. LO8: Use both the direct method and the indirect method to explain cash flows from operating activities. LO9: Explain the role of depreciation in the statement of cash flows. LO10: Describe and assess the effects of the four main methods of accounting for inventories (Appendix 16A). ..

FUNDAMENTAL ASSIGNMENT MATERIAL A1, B1, B3 A1, B1, B3 A1, B1, B3

ADDITIONAL ASSIGNMENT MATERIAL 4, 37, 38, 38, 42, 43, 61, 61, 63, 63, 65, 66, 66 10, 43, 62, 62, 63, 63, 64, 64, 65 43, 44, 63, 63, 65

A1, B3

37, 38, 38, 39, 43, 66, 75

B3

63, 75

A2, B2, B3

19, 45, 45, 46, 47, 48, 51, 54, 55, 67, 68, 68, 75

A2, B2, B3

21, 23, 24, 40, 40, 51, 51, 54, 55 27, 28, 31, 45, 45, 46, 47, 47, 48, 49, 50, 51, 52, 52, 54, 55, 68, 68, 69, 69, 70, 70 37, 53, 53

A2, A3, A4, B2, B3, B3

B3 B3

41, 56, 57, 58, 59, 60, 71, 72, 73, 73, 74

EXCEL, COLLAB., & INTERNET EXERCISES 77, 78, 78 77, 78, 78 77, 78, 78

77, 78, 78

78, 78

76, 78, 78

677


CHAPTER 16 Understanding Corporate Annual Reports: Basic Financial Statements 16-A1

(20-25 min.) REIGLE COMPANY Balance Sheet December 31, 20X0 ASSETS: Current assets: Cash and equivalents Accounts receivable, net Inventories Prepaid expenses Total current assets Noncurrent assets: Property, plant, and equipment, at cost Less: Accumulated depreciation Property, plant, and equipment, net Goodwill, patents, and trademarks Other long-term assets Total noncurrent assets Total assets

$ 60,000 48,000 36,000 15,000 159,000 580,000 170,000 410,000 75,000 110,000 595,000 $754,000

LIABILITIES AND SHAREHOLDERS' EQUITY: Current liabilities: Notes payable $ 51,000 Accounts payable 43,000 Income taxes payable 37,000 Current portion of long-term debt 16,000 Total current liabilities 147,000 Noncurrent liabilities: Long-term debt, excluding current portion 210,000 Deferred income tax liability 44,000 Total noncurrent liabilities 254,000 Total liabilities 401,000 Shareholders' equity: Common stock (50,000 shares @ $.50) 25,000 Additional paid-in capital 124,000* Retained earnings 204,000 Total shareholders' equity 353,000 Total liabilities and shareholders' equity $754,000 *Additional paid-in capital is the same as capital in excess of stated value. To determine the amount of additional paid-in capital, you must begin by computing total liabilities and shareholders' equity = total assets = $754,000.

..

678


Then: Total shareholders' equity

Additional paid-in capital

= $754,000 - current liab. - noncurrent liab. = $754,000 - $147,000 - $254,000 = $353,000 = shareholders' equity - common stock - retained earnings = $353,000 - $25,000 - $204,000 = $124,000

REIGLE COMPANY Income Statement For the Year Ended December 31, 20X0 Revenues Cost of sales Gross profit Selling and administrative expenses Income from operations Other income (expense): Interest expense Interest income Total other income (expense) Income before income taxes Provision for income taxes Net income Earnings per share ($70,000 ÷ 50,000)

..

$790,000 470,000 $320,000 150,000 $170,000 $ (55,000) 15,000 $ (40,000) $130,000 60,000 $ 70,000 $1.40

679


16-A2 (15-20 min.) Although the requirements do not call for it, many students will find it useful to prepare a balance sheet equation (without beginning balances, which are not given). Comparing the entries in the Cash column to those in the Retained Earnings column shows why net income differs from cash provided by operations. This understanding is necessary to interpret (or prepare) the schedule that reconciles net income to net cash provided by operating activities (see 16-A3). RIDGEWOOD ANTIQUES Statement of Cash Flows For the Year Ended December 31, 20X1 (in thousands) Cash flows from operating activities Cash collections from customers Cash payments: To suppliers To employees For other expenses For interest For income taxes Cash disbursed for operating activities Net cash provided by operating activities

$ 1,270 $(725) (180) (100) (11) (35) (1,051) 219

Cash flows from investing activities: Purchase of plant and facilities Cash flows from financing activities: Issued debt Paid dividends Net cash provided by financing activities Net increase in cash Cash, December 31, 20X0 Cash, December 31, 20X1

..

(235) 110 (39)

$

71 55 50 105

680


16-A3

(10-15 min.) RIDGEWOOD ANTIQUES Supporting Schedule to Statement of Cash Flows Reconciliation of Net Income to Net Cash Provided by Operating Activities For the Year Ended December 31, 20X1 (in thousands)

Net income Adjustments to reconcile net income to net cash provided by operating activities: Add: Depreciation, which was included in computing net income but does not affect cash Deduct: Increase in accounts receivable Deduct: Increase in inventory Add: Increase in accounts payable Add: Increase in salaries and wages payable Add: Increase in income taxes payable Net cash provided by operating activities * Sales revenues Less expenses: Cost of goods sold Salaries & wages Depreciation Interest expense Other expenses Income before income taxes Income taxes Net income

..

$361*

48 (330) (50) 175 10 5 $219

[1,600-1,270] [900-850] [900-725] [190-180] [40-35]

$1,600 $850 190 48 11 100

1,199 $401 40 $361

681


16-A4

(10 min.)

1.

Sales Non-depreciation expenses [600,000-80,000] Depreciation Net income Add back depreciation Net cash provided by operating activities

$695,000 (520,000) (80,000) $95,000 80,000 $175,000

2.

Sales Non-depreciation expenses Depreciation Net income (loss) Add back depreciation Net cash provided by operating activities

$ 695,000 (520,000) (240,000) $ (65,000) 240,000 $ 175,000

NOTE: Since depreciation is included in the total operating expenses for this problem, the solution to part 2 assumes the total operating expenses would increase by the $160,000 increase in depreciation (from $600,000 to $760,000) so that cash operating expenses remain constant at $520,000. Notice that the additional depreciation expense did not affect net cash provided by operating activities. The direct method clearly shows this phenomenon: Direct method: Cash collections from customers Operating expenses paid in cash Net cash provided by operating activities

..

$ 695,000 (520,000) $ 175,000

682


16-B1 (15-20 min.) INTEL Balance Sheet January 1, 2012 (in millions) ASSETS Current assets: Cash and cash equivalents Short-term investments Accounts receivable Inventories Other current assets Total current assets Property, plant, and equipment, at cost Accumulated depreciation Long-term investments Goodwill Identified intangible assets Other assets Total assets LIABILITIES AND SHAREHOLDER'S EQUITY Current liabilities: Short-term debt Accounts payable Accrued compensation and benefits Deferred income Other accrued liabilities Accrued advertising liability Total current liabilities Long-term debt Long-term deferred liabilities Other long-term liabilities Common shareholders' equity: Common stock and capital in excess of par value Retained earnings Accumulated other comprehensive income (loss) Total liabilities and shareholders' equity

$ 5,065 5,181 3,650 4,096 7,880 $25,872 $58,073 (34,446)

$

23,627 1,451 9,254 6,267 4,648 $71,119

247 2,956* 2,948 1,929 2,814 1,134 $12,028* 7,084 2,617 3,479

$17,036 29,656 (781) 45,911 $71,119

*Total Current Liabilities = $71,119 - $45,911 - $3,479 - $2,617 - $7,084 = $12,028 Accounts Payable = $12,028 - $1,134 - $2,814 - $1,929 - $2,948 - $247 = $2,956

..

683


16-B2 (25 min.) This is a good exercise in recognizing items that fit in a Statement of Cash Flows and placing them in the proper section of the statement. Three items listed in the problem do not appear in a Statement of Cash Flows: net sales, retained earnings, and total assets. WALGREEN COMPANY Statement of Cash Flows For the Year Ended August 31, 2011 (in millions) Cash flows from operating activities: Net earnings Adjustments to reconcile net earning to net cash provided by operating activities: Depreciation and amortization Deferred income taxes Stock compensation expense Gain on sale of business Other non-cash expenses Changes in current assets and liabilities: Increases in accounts receivable Increases in inventories Increases in other current assets Increases in trade accounts payable Increases in accrued expenses and other liabilities Increases in income taxes payable Increases in other current liabilities Net cash provided by operating activities Cash (Used for) Provided by Investing Activities: Additions to property and equipment Proceeds from sale of assets Business acquisitions, net of cash received Proceeds from sale of business Other cash used by investing activities Net cash used for investment activities Cash (Used for) Provided by Financing Activities: Stock repurchases Repayments on long-term debt Cash dividends paid Proceeds related to employee stock plans Other financing activities Net cash used for financing activities Changes in Cash and Cash Equivalents: Net decrease in cash and cash equivalents Cash and cash equivalents at beginning of year* Cash and cash equivalents at end of year * $1,556 + $324 = $1,880 ..

$ 2,714 1,086 132 135 (434) 53 (243) (592) (24) 384 218 102 112 3,643 (1,213) 79 (630) 442 (203) (1,525) (2,028) (17) (647) 235 15 (2,442)

$

(324) 1,880 1,556

684


16-B3 (10-15 min.) All of the items listed, except provision for income taxes and interest expense, are additions to (or deductions from) net income that are required in computing net cash flow from operating activities. The main problem is to decide whether each one should be added to or deducted from net income. TARGET CORPORATION Supporting Schedule to Statement of Cash Flows Reconciliation of Net Income to Net Cash Provided by Operating Activities For the Year Ended January 28, 2012 (in millions) Net earnings Add non-cash expenses: Depreciation and amortization Bad debt expense Non-cash losses Deferred income taxes Other non-cash charges affecting earnings Deduct increases in non-cash current assets: Accounts receivable Inventory Other assets Add increases in operating current liabilities: Accounts payable Accrued liabilities Deduct decreases in operating current liabilities: Other liabilities Net cash provided by operating activities

$2,929 2,131 154 22 371 90 (187) (322) (107) 232 218 (97) $5,434

The net cash from operating activities exceeds net income by $5,434 - $2,929 = $2,505 million, primarily due to the add back of $2,131 + $154 + $22 + $371 + $90 = $2,768 of depreciation and other non-cash expenses. Note that cost of sales is not an item on the statement of cash flows. 16-1 The operating cycle is the time span during which cash is spent to acquire goods and services that are used to produce the organization's output, which in turn is sold to customers, who in turn pay for their purchases in cash. For some firms, such as large construction companies, this may be much longer than one year. For others, such as grocery stores, it is much shorter than one year. 16-2 Prepaid expenses belong in current assets because if they were not present more cash would be needed in the next year to conduct current operations. ..

685


16-3 Current assets usually include cash and cash equivalents, short-term investments, trade receivables, inventories, and prepaid expenses. 16-4 The amount of depreciation to be charged as an expense depends on three factors as follows: a. The depreciable amount, which is the difference between the total acquisition cost and the estimated residual value. b. The estimated useful life of the asset. c. The depreciation method. 16-5 Depreciation is a method of cost allocation, not valuation. Therefore, it represents the decrease in book value but not the decrease in market value. 16-6 The useful life of depreciable assets is never longer than its physical life. However, it is most heavily influenced by economic obsolescence and technological changes rather than physical wear and tear, so the useful life is generally less than the physical life. 16-7 No. Capitalizing an amount means only that it will not immediately be charged as an expense. It will be charged as an expense over the useful life of the capitalized asset. 16-8 Yes. Goodwill arises only when one company buys another company or part of another company. It is simply the excess of the purchase price over the current value of the separable assets acquired less the liabilities. 16-9 Yes. Working capital is current assets less current liabilities, so it shows how much cash should be available in the next year (or the next operating cycle, if longer) to pay current liabilities that come due in that period. 16-10 The main purpose of including the convertibility option is to increase the appeal of the bond to be issued. Convertibility allows bondholders to participate in a company’s success without the risk of holding it. Investors can convert their bonds into stock if the stock price increases and can opt to be paid out in cash if the stock price decreases. 16-11 Unlike individual proprietors or partners, stockholders' personal assets cannot be confiscated to satisfy the debts of an incorporated entity. 16-12 Stock frequently has a designated par or legal or stated value that is printed on the face of the certificate. For preferred stock (and bonds), par is a basis for computing the amount of dividends (or interest). Par value of common stock has no practical importance. Historically, it was used for establishing the maximum legal liability of the stockholder in case the corporation could not pay its debts. Currently, it is set at a nominal amount (say $1 or even 1¢) in relation to the market value of the stock upon issuance (say $20). ..

686


16-13 Other comprehensive income consists of a few special types of gains and losses that do not appear on the income statement and thus do not become part of retained earnings. They are generally small in amount and are found in the statement of stockholders’ equity. On the balance sheet, other comprehensive income is reported in the stockholders’ equity section. (Companies reporting under international standards will have a separate statement of comprehensive income.) 16-14 Treasury stock is indeed negative stockholders' equity. It is a contraction of or deduction from outstanding capital stock. It is not an asset. 16-15 A multi-step income statement contains subtotals that provide users an easy way to examine specific types of performance, such as gross margin or operating income, that may give insights into the company’s overall performance. 16-16 Retained earnings is increased (decreased) by net income (loss), decreased by dividends paid, and decreased by the repurchase of common shares. 16-17 A cash flows statement aids in predicting future cash flows, evaluating management's generation and use of cash, and determining a company's ability to pay dividends and interest and pay debts when due. It also reveals commitments to assets that may restrict or expand future courses of action. 16-18 Operating activities, investing activities, and financing activities are the three major types of activities summarized in the statement of cash flows. 16-19 The section in the statement of cash flow that lists the cash flow effects of transactions that affect the income statement is known as cash flow from operating activities. For example, cash collection from customers. 16-20 Major investing activities include sales and purchases of property, plant, and equipment, sales and purchases of securities that are not cash equivalents, and making and collecting loans. 16-21 The section in the statement of cash flow that lists the cash flow effects of obtaining cash from creditors and owners, repaying creditors or buying back stock from owners, and paying cash dividends is known as cash flow from operating activities. For example, interest paid on loan. 16-22 Interest paid or received appears in the operating activities section. Some commentators favor showing interest paid as a financing activity and interest received as an investing activity. However, the FASB decided that, because interest income and interest expense are included in income, they should be included in operating activities. Under IFRS companies have the option of including interest paid as a financing item.

..

687


16-23 The principal loan amount will be shown as an inflow under cash flow from financing activities. The interest of $15,200,000 x 8% = $1,216,000 will be shown as an outflow under cash flow from financing activities. 16-24 The purchase of fixed assets worth $2.5 million will be shown as a cash outflow under cash flow from investing activities. 16-25 Non-cash investing and financing activities generally could have been accomplished identically in substance (though not in form) by cash transactions. For example, issuing debt to purchase an asset could have been accomplished by issuing debt for cash and then using the cash to purchase the asset. Companies should not be able to prevent disclosure of such a transaction to readers of the statement of cash flows simply by using a non-cash form of transaction. 16-26 The direct method and indirect method are the two major ways of computing net cash provided by operating activities. The direct method shows cash inflows and outflows directly. The indirect method begins with net income and adds adjustments to get net cash provided by operating activities. 16-27 Depreciation is an allocation of original cost of an asset to expense that does not entail a current outlay of cash; that is, depreciation is a non-cash expense. It is added to net income when using the indirect method only to offset its deduction in computing net income. Thus, it is not a source of cash. 16-28 Companies record purchase of merchandise on an accrual basis, not a cash basis. Because there is often a delay between the purchase recorded on an accrual basis and the actual payments of cash to suppliers, the two amounts will seldom be exactly equal. 16-29 Cash flow from operations does not recognize the investment necessary to replace the fixed resources used in generating the period’s revenues. If a company does not generate enough cash to both carry out its operations and to replace the fixed assets it uses up, it cannot stay in business long. Free cash flow tells us whether the cash generated by operations is enough to support the investment activities of the company. 16-30 Depreciation appears only in the operations section of an indirect-method cash flow statement or in a supporting schedule to the body of the statement of cash flows in a direct-method statement. Depreciation is one of the items that reconciles net income to net cash flow from operating activities. However, it does not appear directly on a directmethod cash flow statement because it does not directly affect cash. 16-31 When a company uses the direct method for preparing the statement of cash flow, users of financial statements often miss important information that relates net income to operating cash flows. This is why a statement of cash flow under the direct method is supplemented by a reconciliation statement to reconcile the net income to operating cash flow.

..

688


16-32 Specific identification recognizes the actual cost paid for the particular physical item sold. First-in, first-out (FIFO) assumes that the items acquired earliest are sold or used up first. Last-in, last-out (LIFO) assumes that the items acquired most recently are sold or used up first. Weighted average assumes that the cost of all items available for sale during the period are divided by the number of items to get an average unit cost. 16-33 FIFO will have the highest net income, because the older (and hence lower cost) items comprise the cost of goods sold, making cost of goods sold lower and net income higher. 16-34 Yes. Purchases under LIFO can affect income immediately, because the latest purchases are regarded as cost of goods sold. 16-35 The statement is true under IFRS. The first part is true under both U.S. GAAP and IFRS -- if replacement cost falls and lower ultimate sales prices are expected, the inventory is written down. But under U. S. GAAP, once written down the inventory is never written up again. The cost to which inventory is written down becomes the "new cost" and is therefore the ceiling for any future valuation of the inventory. 16-36 No. The opposite is true. Tax expense on reports to shareholders has exceeded the actual tax payments. 16-37 Most accounting measures of fixed assets are based on historical cost, not market values. Companies record fixed assets at the cost paid for them, and they spread this cost as depreciation over the years they expect to use the asset. The book value of the asset is the remainder of the cost that has not yet been charged as depreciation expense; it is not intended to be an approximation of the market price of the asset. When asset values increase, U.S. GAAP does not allow a revaluation upward of the asset’s book value. Under IFRS, revaluation is allowed if applied to an entire class of assets. In cases where assets have been revalued, book values would be closer to the market values of the assets. 16-38 Under both U.S. GAAP and IFRS, research costs are always expensed. But IFRS allows capitalization of development costs subject to the fulfilment of conditions for initial recognition. In the present case, the drug is yet to pass through multiple trials and hence cannot be considered to have met the recognition criteria of development cost. Thus, under IFRS, it is still a research phase cost and hence to be expensed. In the U.S. GAAP the cost is always expensed. 16-39 The gross margin on an income statement might be an appropriate measure for assessing the success of a sales department. Sales managers are generally responsible for the price charged for goods or services (and therefore the margin received for them) and the volume of sales. Both of these factors are reflected in the gross profit (or gross margin). To separate the effects of volume and profit margin, managers might look at the gross margin percentage as a measure of margin and total sales revenues as a measure of volume. It would also be good to examine contribution margins when making short-run decisions. ..

689


16-40 A statement of cash flow has three sections: cash flow from operating activities, cash flow from investing activities, and cash flow from financing activities. In the present case, the sale of old machineries and purchase of new ones will appear in the cash flow from investing activities section. While the sale of old machineries will be shown as an inflow, the purchase of new ones will be shown as an outflow. Again, the sale of investments will also appear in the cash flow from investing activities section as an inflow. On the other hand, the amount of loan raised from financial institutions may be found in the cash flow from financing activities as an inflow while the interest paid on loan may be found in the same section as an outflow. 16-41 If the purchasing officer wishes to maximize her performance evaluation by reporting the largest possible gross margin, she will not buy the oil at $70 per barrel if the company uses LIFO. Why? Because the $70 spent for the most recent purchase of oil becomes part of cost of goods sold under LIFO, replacing oil charges at $50 per barrel or less. This would reduce the gross margin. Under FIFO, the purchase decision would not affect current year’s gross margin. Therefore, the purchasing officer would not have any special incentive to either purchase or avoid purchasing the oil. With an incentive not to purchase under LIFO and no particular incentive under FIFO, she is more likely to purchase the oil if the company uses FIFO than if it uses LIFO.

..

690


16-42 (10-15 min.) The purpose of this problem is to stress the limitations of the use of historical costs, particularly where there are significant amounts of property, plant, and equipment. Kr stands for the kroner, the Norwegian measure of currency. The balance sheet values do not come close to the Kr30 million current market value of the land and building. Consequently, in terms of current values before expansion and modernization, stockholders' equity is understated (in millions): Market value of land and building Net book value: Land Building Excess of market value over net book value

Kr30.0 Kr4.0 1.3

5.3 Kr24.7

As conventionally prepared after the expansion and modernization, the balance sheet would be (in millions): Assets Liabilities and Stockholders’s Equity Cash Kr 3.0 Liabilities: Land 4.0 Mortgage payable Kr18.0 Building at cost Kr26.0 Accumulated depreciation 6.7 Stockholders' equity 8.3 Net book value 19.3 Total liabilities and Total assets Kr 26.3 stockholders' equity Kr26.3 The balance sheet would be unusually deceiving. The mortgage would appear to be exceedingly high in relation to the book value of the assets. The historical cost and resulting stockholders' equity have little meaning.

..

691


16-43 (25-30 min.) This problem is similar to 16-A1, but it is more difficult because it includes items not shown in exhibits 16-1 and 16-5 and terminology is varied slightly. HOKKAIDO COMPANY Balance Sheet May 31, 20X1 (in millions) ASSETS: Current assets: Cash and equivalents Receivables Inventories Other current assets Total current assets Noncurrent assets: Fixed assets, net Capital construction fund Intangible assets Long-term investments Total noncurrent assets Total assets

¥ 45,000 22,000 29,000 6,000 102,000 217,000 28,000 21,000 15,000* 281,000 ¥383,000

LIABILITIES AND STOCKHOLDERS' EQUITY: Current liabilities: Accounts payable ¥ 24,000 Accrued expenses payable 20,000 Other current liabilities 9,000 Total current liabilities 53,000 Noncurrent liabilities: Mortgage bonds payable 84,000 Debentures 77,000 Deferred income tax liability 12,000 Total noncurrent liabilities 173,000 Total liabilities 226,000 Stockholders' equity: Redeemable preferred stock 15,000 Common stock, at par 5,000 Paid-in capital in excess of par 102,000 Retained income 48,000 Less: Treasury stock (13,000) Total stockholders' equity 157,000 Total liabilities and stockholders' equity ¥383,000 *To compute the amount for long-term investments, recognize that total assets must be ¥383,000 (equal to total liabilities and stockholders' equity). Then:

..

692


Total noncurrent assets = Total assets - Total current assets = ¥383,000 - ¥102,000 = ¥281,000 Long-term investments = Noncurrent assets - Fixed assets, net - Capital construction fund – Intangible assets = ¥281,000 - ¥217,000 - ¥28,000 - ¥21,000 = ¥15,000

HOKKAIDO COMPANY Income Statement For the Year Ended May 31, 20X1 (in millions except net income per share) Net sales Cost of goods sold Gross margin Operating expenses: Administrative and general expenses Research and development expenses Selling and distribution expenses Total operating expenses Operating income Other income (expenses), net Income before income taxes Income taxes Net income Net income per share*

¥422,000 (195,000) 227,000 (65,000) (42,000) (41,000) (148,000) 79,000 (12,000) 67,000 (51,000) ¥ 16,000 ¥320,000

*¥16,000,000,000 ÷ 50,000 = ¥320,000

..

693


16-44 (5-10 min) Sydney Company’s retained earnings were increased by the net income in 20X1 and decreased by the dividends paid. The other comprehensive income does not affect retained earnings, but it does affect total stockholders’ equity. Retained earnings, January 1, 20X1 + Net income - Cash dividends Retained earnings, December 31, 20X1

$56,000 21,000 8,000 $69,000

Total stockholders’ equity, January 1, 20X1 + Increase in retained earnings ($21,000 - $8,000) + Other comprehensive income Total stockholders’ equity, December 31, 20X1

$75,000 13,000 4,000 $92,000

16-45

(5 min.)

The split between cash and credit sales is irrelevant for purposes of this problem. Sales Less increase in accounts receivable Cash received from customers 16-46

£300,000 (5,500) £294,500

(5 min.)

Cost of goods sold £200,000 Add increase in inventory (£44,500 – £32,000) 12,500 Deduct increase in accounts payable (£20,500 – £6,500) (14,000) Cash paid to suppliers £198,500 16-47

(5-10 min.)

Wages and salary expense Cash paid to employees Increase in accrued wages and salaries payable

£77,500 68,000 £ 9,500

Beginning balance, accrued wages and salaries payable Increase in accrued wages and salaries payable Ending balance, accrued wages and salaries payable

£ 4,000 9,500 £ 13,500

..

694


16-48

(5-10 min.) HOOGENDOORN AND ASSOCIATES Statement of Cash Flows from Operating Activities For the Year Ended December 31, 20X0

Collections from customers (€510,000 – €17,000) Cash expenses (€280,000 - €50,000) Net cash provided by operating activities 16-49

€493,000 230,000 €263,000

(5-10 min.) HOOGENDOORN AND ASSOCIATES Reconciliation of Net Income to Net Cash Provided by Operating Activities For the Year Ended December 31, 20X0.

Net income Add depreciation, which was deducted in computing net income but does not affect cash Deduct increase in accounts receivable Net cash provided by operating activities 16-50

€230,000 50,000 (17,000) €263,000

(10 min.) HALIFAX COMPANY Reconciliation of Net Loss to Net Cash Provided by Operating Activities For the Year Ended December 31, 20X2

Net loss Add depreciation Add decrease in accounts receivable Deduct increase in inventory Add increase in accounts payable Add increase in wages and salaries payable Net cash provided by operating activities

..

$(25,000) 19,000 4,000 (3,000) 18,000 5,000 $ 18,000

695


16-51

(15-25 min.) ROTICCIO AGRO PRODUCTS Statement of Cash Flows For the Year Ended December 31, 2020 (in thousands) Cash flows from operating activities: Cash collections from customers Cash payments: To suppliers To employees For other operating expenses For interest For income taxes Cash disbursed for operating activities Net cash provided by operating activities Cash flows from investing activities: Purchase of warehouse Proceeds from sale of equipment Net cash used in investing activities Cash flows from financing activities: Issued common stock Retired long-term debt Dividends paid Net cash used in financing activities Net decrease in cash Cash, January 1, 2020 Cash, December 31, 2020

CHF5,780 CHF(4,260) (610) (210) (52) (214) (5,346) 434 (1,080) 74 (1,006) 56 (42) (178) (164) (736) 760* CHF 24

*CHF736 + CHF24

..

696


16-52

(10-20 min.) ROTICCIO AGRO PRODUCTS Supporting Schedule to Statement of Cash Flows Reconciliation of Net Income to Net Cash Provided by Operating Activities For the Year Ended December 31, 2020 (in thousands)

Net income CHF478 Adjustments to reconcile net income to net cash provided by operating activities Add: Depreciation 302 Deduct: Increase in accounts receivable (6,006 – 5,780) (226) Deduct: Increase in inventory (112) Add: Increase in accounts payable (4,192 + 112 – 4,260) 44 Deduct: Decrease in salaries and wages payable (48) Deduct: Decrease in income taxes payable (214 – 210) (4) Net cash provided by operating activities CHF434 16-53

(10 min.) TEXAS IRON AND STEEL WORKS (in millions)

1.

Income Statement: Sales Non-depreciation expenses ($338 – $34) Depreciation Net income

$406 $304 54

Reconciliation of net income to net cash provided by operating activities: Net income Add noncash expenses: Depreciation Deduct net increase in noncash operating working capital Net cash provided by operating activities

358 $ 48

$ 48 54 (30) $ 72

2.

An increase in depreciation does not affect the net cash flow from operating activities. The $20 million increase in depreciation decreases the net income by $20 million and increases the add-back by $20 million. The net effect is zero. We add depreciation to the net income merely to offset its deduction when computing the net income, not because it provides cash.

..

697


16-54 1.

(20-30 min.) O’TOOLE COMPANY Statement of Cash Flows For the Year Ended December 31, 20X2 (in millions)

Cash flows from operating activities: Net income Adjustments to reconcile net income to net cash provided by operating activities: Depreciation Increase in receivables Increase in inventories Increase in current liabilities Net cash provided by operating activities

20 (35) (50) 77 $ 72

Cash flows from investing activities: Purchase of fixed assets

(195)*

Cash flows from financing activities: Issuance of long-term debt Payment of dividends Cash provided by financing activities Net decrease in cash Cash balance, December 31, 20X1 Cash balance, December 31, 20X2

$ 60

$120* (10) 110 $ (13) 25 $ 12

* This assumes that the debt was issued for cash and the cash used to buy the fixed assets. If the debt were issued directly to the seller of the fixed assets, the cash outflow for purchase of fixed assets would be $75 million, there would be no cash from issuance of long-term debt, and a supporting schedule would have an investment and financing activity of $120 million for acquiring the fixed assets. 2.

Dear Mr. O’Toole: Severe shortages of cash commonly accompany rapid corporate growth. Profitable operations usually produce heavy supplies of cash. But the insatiable demand for cash to expand receivables, inventories, and fixed assets may deplete the cash on hand despite profitable operations. This is why so many so-called growth companies usually pay little or no dividends. Note also that the ratio of current assets to current liabilities is 4.4 to 1 on December 31, 20X1 but only 1.8 to 1 on December 31, 20X2. It appears that the need for cash to support increases in receivables and inventory has come primarily from increases in current liabilities.

..

698


16-55 1.

(30-40 min.) SOUTH AFRICAN IMPORTS COMPANY Statement of Cash Flows For the Year Ended December 31, 20X1 (in millions)

Cash flows from operating activities: Cash collections from customers (R315-R27) Cash payments: To suppliers (R188+R22-$14) For general expenses (R51+R1) For taxes (R10-R1) Total cash payments for operating activities Net cash provided by operating activities

R288 R(196) (52) (9)

Cash flows from investing activities: Acquisition of plant assets Proceeds from sale of plant assets Net cash used for investing activities Cash flows from financing activities: Issuance of long-term debt Payment of cash dividends Net cash provided by financing activities Net decrease in cash Cash balance, December 31, 20X0 Cash balance, December 31, 20X1 2.

..

(103) 6 (97) 50 (4) 46 (20) 25 R 5

Reconciliation of Net Income to Net Cash Provided by Operating Activities

Net income Adjustments to reconcile net income to net cash provided by operating activities: Depreciation Increase in accounts receivable Increase in inventory Increase in prepaid general expenses Increase in accounts payable for merchandise Increase in accrued taxes payable Net cash provided by operating activities 3.

(257) 31

R 26 40 (27) (22) (1) 14 1 R 31

Botha’s stress may be reduced but not eliminated. The statement of cash flows shows why cash has fallen by R20 million. Operating activities provided R31 million, and financing activities provided an additional R46 million, a total of R77 million. However, R97 million was needed for the net acquisition of the warehouse.

699


Severe crunches on cash commonly accompany quick corporate growth. There may be large net income and working capital provided by operations, but heavy demands for cash to expand receivables, inventories, and plant assets diminish the cash on hand despite profitable operations. Hence, most "growth" companies pay skimpy or no dividends. 16-56

(15 min.)

1.

a. FIFO Method: Inventory shows: Costs:

600 tons on hand. 300 tons @ $69.00 250 tons @ $66.00 50 tons @ $64.00 July 31 inventory valuation

b. LIFO Method: Inventory shows: Costs:

600 tons on hand. 500 tons @ $62.00 100 tons @ $64.00 July 31 inventory valuation

$20,700 16,500 3,200 $40,400

$31,000 6,400 $37,400

T accounts (not required) are: Inventory (FIFO) 31,000 To cost of goods sold 64,000 16,500 20,700 132,200 40,400

Balance Purchases: Available Balance

Inventory (LIFO) 31,000 To cost of goods sold 64,000 16,500 20,700 132,200 37,400

Balance Purchases: Available Balance 2. Revenue Cost of goods sold Gross profit

..

FIFO $105,000 91,800 $ 13,200

91,800

94,800

LIFO $105,000 94,800 $ 10,200

700


16-57

(5-10 min.)

The inventory would be written down from $200,000 to $170,000 on March 31, 20X1. The new $170,000 valuation is "what's left" of the original $200,000 cost. In other words, the $170,000 is the unexpired cost and may be thought of as the new cost of the inventory for future accounting purposes under U.S. GAAP. Even though subsequent replacement values exceed the $170,000 cost, write-ups above "cost" are not acceptable accounting practice. The valuation remains at $170,000 until it is written down to $150,000 on the following December 31, 20X1. 16-58

(5-10 min.)

The inventory would be written down from $200,000 to $170,000 on March 31, 20X1. On June 30 it would be written back up to $180,000 under IFRS rules. On September 30 it would be written up to $200,000, the original invoice price. It cannot be valued at more than the original invoice price. On December 31 it would be written down to $150,000. 16-59

(20 min.) Amounts are in dollars.

1. Sales Cost of goods sold: Inventory, December 31, 20X1 Purchases Cost of goods available for sale Inventory, December 31, 20X2 Cost of goods sold Gross margin or gross profit

2.

..

*15,000 @ $12 = 20,000 @ $14 = 4,000 @ $16 =

$180,000 280,000 64,000 $524,000

**34,000 @ $16 = 5,000 @ $14 =

$544,000 70,000 $614,000

Units 30,000

LIFO $710,000

FIFO $710,000

15,000 54,000 69,000 39,000 30,000

180,000 180,000 824,000 824,000 1,004,000 1,004,000 524,000* 614,000** 480,000 390,000 $ 230,000 $ 320,000

Gross margin is higher under FIFO. However, cash will be higher under LIFO by .40 × ($320,000 - $230,000) = .45 × $90,000 = $40,500.

701


16-60

(20-30 min.)

1. and 2.

Sales, 28,000 @ $17 Deduct cost of goods sold: Inventory, December 31, 20X0 20,000 @ $10 Purchases: 30,000 @ $12 and $8, respectively Cost of goods available for sale Deduct: Inventory, December 31, 20X1, 22,000 bags: 22,000 @ $12 or 20,000 @ $10 + 2,000 @ $12 or 22,000 @ $8 or 20,000 @ $10 + 2,000 @ $8 Cost of goods sold Gross margin

Requirement 1 (1) (2) FIFO LIFO

Requirement 2 (3) (4) FIFO LIFO

$476,000

$476,000

$476,000

$476,000

200,000

200,000

200,000

200,000

360,000 560,000

360,000 560,000

240,000 440,000

240,000 440,000

264,000 224,000 176,000 296,000 $180,000

336,000 $140,000

264,000 $212,000

216,000 224,000 $252,000

3a.

LIFO results in more cash by the difference in income tax effects. LIFO results in a lower cash outflow of .35 × ($180,000 - $140,000) = $14,000.

3b.

FIFO results in more cash when inventory prices are falling. Why? Because income tax cash outflow would be more under LIFO by .35 x ($252,000 $212,000) = $14,000.

..

702


16-61

(10-15 min.)

1.

(a)

The operating income was lower by the $3,292 million because the R&D must be charged to expense.

(b)

The operating income would be $7,202 + $3,292 = $10,494 million.

(c)

There would be no balance-sheet asset if the R&D were expensed. However, if patents were purchased instead of Pfizer itself conducting the R&D (as in requirement (b)), assets would be higher by $3,292 million, the cost of the patents. Only $2,292 million of the R&D would be charged as expense in 2020. The $1,000 million of development costs would be capitalized and amortized over the life of the products.

(d)

2.

$30 million ÷ 4 = $7.5 million

3.

$16 billion – $3 billion = $13 billion of goodwill The goodwill would remain on Walmart’s books unless the management determines that its value has fallen below the amount initially recorded. At that time, it would be written down (or written off entirely if its value had fallen to zero).

..

703


16-62

(15-20 min.)

1.

Sales and either cash or accounts receivable (or a combination of both) would increase by £500 million. Warranty expense and liability for warranties would rise by 0.032 × £500 million = £16 million. Liability for warranties would decrease by £15 million and cash would decrease by £15 million. There is a net increase of £1 million in the liability for warranties.

2.

Cash and the liability account called Deposits on Cans would be increased by RM.06 × 2,500,000 = RM150,000. In turn, both accounts would be decreased by RM125,000.

3.

Cash and the liability account called Deposits would be increased by S$50,000 on April 1. On June 30, Interest Expense and Deposits would be increased by 3/12 × 0.025 × S$50,000 = S$312.5. On July 1, Cash and Deposits would be decreased by S$50,000 + S$312.5 = S$50312.5.

4.

(a)

Cash and the liability, Unearned Sales Revenue, would be increased by £240,000 on December 31 for the ticket sales.

(b)

On January 31, Unearned Sales Revenue would be decreased by £240,000 ÷ 4 = £60,000. On the income statement, Sales would be increased by £60,000.

16-63

(15 min.) Amounts are in millions.

1. 2020 accounts receivable was: ₩20,329,100 – ₩343,200 + ₩60,500 = ₩20,046,400 million. This could also be calculated as gross receivables of ₩20,906,600 – ₩343,200 = ₩20,563,400 less estimated uncollectible accounts of ₩577,500 – ₩60,500 = ₩517,000, or ₩20,563,400 – ₩517,000 = ₩20,046,400 million. 2. 2020 liabilities for warranties were ₩2,691,700 – ₩2,918,300 + ₩2,922,700 = ₩2,696,100 million. 3. 2020 retained earnings were: ₩35,964,500 + ₩7,781,400 – ₩917,400 – ₩4,035,900 = ₩38,792,600 million. ..

704


16-64 (10-15 min.) The following income statement and balance sheet contain the most common subtotals, which are underlined: GOOGLE INC. Income Statement for the Three Months Ended March 31, 2020 (in millions) Revenue Cost of revenue Gross margin Operating expenses: Research and development Sales and marketing General and administrative Total operating expenses Operating income Other income (expense) Income before income taxes Provision for income taxes Net income

$34,814 7,904 26,910 5,034 6,828 2,300 14,162 12,748 (22) 12,726 2,510 $10,216

GOOGLE INC. Balance Sheet as of March 31, 2020 (in millions) Assets Current assets: Cash and cash equivalents $ 12,776 Short-term investments 106,282 Total cash and cash equivalents, and short-term investments 119,058 Accounts receivable, net of allowance for doubtful accounts of $644 21,922 Inventories 2,824 Deferred income taxes 4,700 Other current assets 5,216 Total current assets 153,720 Long-term assets: Property and equipment, net of accumulated depreciation of $21,904 16,450 Equity and other investments 18,136 Goodwill 39,396 ..

705


Intangible assets Other long-term assets Total long-term assets Total assets Liabilities and stockholders’ equity Current liabilities: Accounts payable Accrued compensation Income taxes Short-term unearned revenue Securities lending payable Other current liabilities Total current liabilities Long-term liabilities: Long-term debt Long-term unearned revenue Deferred income taxes Other long-term liabilities Total long-term liabilities Total liabilities Commitments and contingencies Stockholders’ equity: Common stock and paid-in capital—shares authorized 48,000; outstanding 16,800 Retained earnings Total stockholders’ equity Total liabilities and stockholders’ equity

..

5,512 2,806 82,300 $236,020

$ 7,580 6,544 1,916 27,858 2,420 6,022 52,340 23,876 2,524 2,912 17,050 46,362 98,702

130,546 6,772 137,318 $236,020

706


16-65

(10-15 min.) A framework for the balance sheet equation that is popular in Europe and other parts of the world is: non-current assets – working capital – non-current liabilities = stockholders’ equity In contrast, the U.S. framework is: assets = liabilities + stockholders’ equity

. Thus, under the alternative framework, the bottom line of the left-side (or top) of the balance sheet is net assets (assets – liabilities), which equals stockholders’ equity. Further, the accounts are presented in different orders with different subtotals. Consider first the assets: ► Non-current assets are listed first, whereas in the U. S., current assets would be first. ► All liabilities are shown as negative numbers because they are to be deducted from total assets to get the bottom line of net assets. ► Current assets are listed from least liquid to most liquid, the reverse of the order in most U.S. statements. ► Current liabilities are subtracted from current assets to give a subtotal for net current liabilities (would be net current assets if current assets exceeded current liabilities). ► The statement next deducts both net current liabilities (working capital) and long-term liabilities from total assets to give a subtotal for total net assets, which will equal the subtotal for stockholders’ equity. ► The equity section contains two categories of “reserves” that do not appear in U.S. statements. ► Called up share capital and share premium account refer to paid-in capital. Although J Sainsbury’s financial statements contain most of the same basic information that U. S. statements contain, the format is quite different. 16-66 (10-20 min.) This problem can be the basis for a discussion of the strengths and weaknesses of accounting theory. 1.

There would be a “gain from insurance on cargo sinking” recognized on the income statement: Insurance payments received Book value of cargo Gain from insurance on cargo sunk

€7,200,000 1,250,000 €5,950,000

The total recorded value of the assets would increase by €5,950,000, the amount of the gain. The fleet of cargoes would be essentially the same as before the ..

707


accident, except that a cargo with a book value of €7.2 million has replaced a similar cargo with a book value of only €1.25 million. 2.

Accounting for casualties is very controversial. It gets to the heart of the question of what is income and what is capital. Does the €7.2 million represent a return of capital or a payment of both capital and income? The traditional accounting model ignores changes in general purchasing power and intervening changes in specific prices while an asset is held. When an asset is disposed of, the gain or loss is measured in nominal euros (almost always without regard to the intended use of the proceeds). Many theorists and practitioners define the income of a going concern to be a function of whether the proceeds will be reinvested in the same types of assets. These individuals maintain that no gain is realized on the cargo getting sunk, because the €7.2 million is really a return of capital (where capital is thought of in physical terms as cargoes, inventories, etc.). Thus, the “gain” would not be shown in the income statement. Instead, it would appear as a special balance sheet item called Revaluation Equity, or some similar title.

16-67

(5-10 min.)

a. b. c. d.

Investing Financing Operating * Operating *

e. f. g.

Financing Investing Investing

h. i. j.

Financing Operating * Financing

* These items appear in the operating section of the statement of cash flows only if the indirect method is used. Therefore, Chevron must use the indirect method. 16-68

(10-15 min.)

1.

The only line for interest on the statement of cash flows will be under operating activities: Cash payments for interest (€31,037,500)

2.

The increase of €34,797,000 – €31,037,500 = €3,759,500 in interest payable would be added to the net income in computing net cash provided by operating activities. Why? Because the interest expense of €34,797,000 was deducted in computing net income, but the cash payment of €31,037,500, €3,759,500 less, should be deducted in computing cash flow.

..

708


16-69

(30 min.)

1.

EXCLUSIVE FURNITURES Statement of Cash Flows from Operating Activities (Indirect Method) For the Year Ended December 31, 2020 (in millions)

Net income Adjustments to reconcile net income to net cash provided by operating activities: Depreciation and amortization Other noncash expenses Increase in receivables Increase in merchandise inventories Increase in payables Increase in other operating liabilities, net Net cash provided by operating activities

2.

MXN30,840

17,100 5,380 (1,620) (12,840) 16,080 9,020 MXN63,960

EXCLUSIVE FURNITURES Statement of Cash Flows from Operating Activities (Direct Method) For the Year Ended December 31, 2020 (in millions MXN)

Cash collections from customers (1,778,300 – 1,620) Cash payments to suppliers (1,554,780 + 12,840 – 16,080) Cash payment for operating expenses (174,740 – 17,100 – 5,380 – 9,020) Cash payments for income taxes Cash paid for other expenses, net Net cash provided by operating activities

..

MXN1,776,680 (1,551,540) (143,240) (16,820) (1,120) MXN 63,960

709


16-70 1.

(25-35 min.) NATURE FRESH INC. Cash Flows from Operating Activities For the Year Ended December 31, 2020 (in millions)

Cash collections from customers (32,631 - 294) Cash payments: To suppliers of goods (19,776 + 411 – 162) For selling, general, and administrative expenses (9,108 – 1,113 – 351 – 18 – 579) For interest For income taxes (1,308 + 36) Cash disbursed for operating activities Net cash provided by operating activities 16-71

€32,337 €(20,025) (7,047) (390) (1,344) (28,806) €3,531

(30-40 min.) Amounts are in millions.

1. Revenue (150 @ $15 + 160 @ $15) Deduct cost of revenue: Inventory, December 31, 2011, 100 @ $8 Purchases (200 @ $10 + 140 @ $13) Cost of goods available for sale Deduct: Inventory, June 30, 2012, 130 units: 130 @ $13 or 100 @ $8 + 30 @ $10 or 130 @ ($4,620 ÷ 440) or 130 @ 10.50 or 80 @ $8 + 50 @ $10 Cost of sales of products Gross margin

FIFO $4,650

Specific Weighted IdentifiLIFO Average cation $4,650 $4,650 $4,650

800 3,820 4,620

800 3,820 4,620

800 3,820 4,620

800 3,820 4,620

1,690 1,100 1,365 2,930 $1,720

3,520 $ 1,130

3,255 $1,395

1,140 3,480 $1,170

2a.

Income before income taxes will be lower under LIFO: $1,720 - $1,130 = $590. The income tax will be lower by .40 × $590 = $236.

2b.

Income before income taxes will be lower under weighted average: $1,720 $1,395 = $325. The income tax will be lower by .40 × $325 = $130.

..

710


16-72

(20-30 min.) This problem explores the effects of LIFO layers.

There would be no effect on gross margin, income taxes, or net income under FIFO. The balance sheet would show a higher inventory by $900. A detailed income statement would show both purchases and ending inventory as higher by $900, so the net effect on cost of goods sold would be zero. LIFO would show a lower gross margin, $830, as compared with $1,130, a decrease of $300. Hence, the impact of the late purchase would be a savings of income taxes of 40% of $300 = $120. For details, see the accompanying tabulation.

Net sales of products, as before Deduct cost of sales of products: Inventory, December 31, 2011, 100 @ $8 Purchases, 340 units, as before, and 400 units Available for sale Ending inventory: First layer ,100 @ $8 $800 Second layer, 30 @ $10 300 First layer, 100 @ $8 Second layer, 90 @ $10 Cost of sales of products Gross margin *340 units, as before $3,820 60 units @ $15 900 $4,720

Without Late Purchase $4,650

With Late Purchase $4,650

$ 800 3,820

$ 800 4,720* $5,520

$4,620 1,100 $800 900

1,700

3,520 $ 1,130

3,820 $ 830

Although purchases are $900 higher than before, the new LIFO ending inventory is only $1,700 - $1,100 = $600 higher. The cost of sales is $3,820 - $3,520 = $300 higher. To see this another way, compare the ending inventories: Late purchase added to cost of goods available for sale: 60 @ $15 Deduct 60-unit increase in ending inventory: Second layer is 90 - 30 = 60 units higher @ $10 Cost of sales is higher by 60 @ ($15 - $10)

..

$900 600 $300

711


16-73

(15 min.)

1.

The inventory would have decreased by ₹64 million, from ₹848 million to ₹784 million, under LIFO compared to ₹24 million, and from ₹848 million to ₹824 million, under FIFO. Therefore, the cost of merchandise sold would have been ₹40 million higher, and the operating income would have been ₹40 million lower under LIFO. Cost of merchandise sold = ₹3,688 million + ₹40 million = ₹3,728 million Operating income = ₹536 million – ₹40 million = ₹496 million, or 7.5% less An alternate way to see this is to compute cost of goods sold under FIFO and LIFO (in millions): FIFO LIFO Beginning inventory ₹848 ₹848 Purchases 3,664 3,664 Goods available for sale 4,512 4,512 Ending inventory 824 784 Cost of goods sold ₹3,688 ₹3,728

2.

At a tax rate of 40%, the ₹40 million reduction in income would result in a tax savings of ₹40 million × 40% = ₹16 million.

3.

The prices were rising during fiscal 2019. The most recent prices must be higher than the beginning prices because the ending inventory under FIFO (which contains the most recent prices) is greater than the ending inventory under LIFO (which contains older layers of inventory). Alternatively, the cost of merchandise sold under LIFO (which contains the most recent prices) is higher than the cost of merchandise sold under FIFO (which includes older prices). Note that the decrease in inventory value during fiscal 2019 was due to a decrease in the physical inventory, not a drop in prices.

..

712


16-74

(20 min.)

The inventory method determines how costs will be divided between ending inventory and cost of goods sold. Under the FIFO method, inventory would have increased by $26 million (that is, $291 million - $265 million) more than it did under LIFO (in millions): LIFO FIFO 2011 $1,609 $1,609 + $168 = $1,777 2010 1,344 1,344 + 142 = 1,486 Increase in inventory $ 265 $ 291 Therefore, cost of goods sold would have been $26 million lower under the FIFO method. Pretax income for 2011 would have been $26 million higher: $2,428 million + $26 million = $2,454 million. Total inventory under FIFO would have exceeded that under LIFO by $168 million. Therefore, cumulative operating income would have been $168 million higher under FIFO.

..

713


16-75 (25-30 min.) 1.

Nike’s principal business activity is “the design, development and worldwide marketing of high quality footwear, apparel, equipment, and accessory products.” About 42% of its sales are in the United States.

2.

Nike’s gross margin increased from $8,800 million in fiscal 2010 to $9,509 million in fiscal 2011, an 8% increase. However, gross margin as a percentage of revenue dropped from ($8,800 ÷ $19,014) = 46.3% in 2010 to ($9,509 ÷ $20,862) = 45.6% in 2011.

3.

Nike’s working capital grew from $10,959 - $3,958 = $7,001 in 2010 to $11,297 $3,364 = $7,933 in 2011.

4.

Nike generated $1,812 million from its operating activities, but it used $1,021 million for investing activities and $1,972 million for financing activities. A little less than half of the funds used for investing activities were for additions to property, plant, and equipment and other assets, and a little over half was a net increase in short-term investments. The major financing uses of funds were to repurchase common stock and to pay dividends. A minor effect was the $57 million increase in cash from exchange rate changes, a topic beyond the scope of this text.

5.

Nike declared dividends on common stock of $1.20 per share or $569 million. This information is found on the statement of shareholders’ equity, and the dividends per share are also found on the income statement. Nike paid $555 million in cash for dividends as reported on the statement of cash flows.

16-76 (30-40 min.) For the solution to this Excel Application Exercise, follow the step-by-step instructions provided in the textbook chapter. 1 & 2. FIFO always assigns the most recent prices to the ending inventory. Therefore, regardless of whether prices are rising or falling, FIFO has inventory values that most closely reflect current replacement costs. 3. LIFO has the highest cost of goods sold only when prices are rising; when prices are falling, FIFO has a higher cost of goods sold. 16-77

(30 min.)

The purpose of this exercise is to learn which accounts belong to the income statement and which to the balance sheet. Doing the exercise in teams of two persons each allows each student to recall income statement and balance sheet accounts himself or herself and also to react to accounts listed by someone else. Discussion of those accounts for which there is disagreement should generate consideration of what criteria make an account an income statement account or a balance sheet account. It also forces consideration of the labels put on accounts and how well they identify the nature of the account.

..

714


16-78 (15-25 min.) This solution is based on the website as it was in late 2020. The financial statements are for the year ended Mach 31, 2020. Be sure to examine the current website before assigning this problem, as the information there may have changed. 1. Tata Steel calls its income statement a “Statement of Profit and Loss.” The company uses a multiple step format with subheadings including “Total Income,” “Expenses,” “Profit before Exceptional Items and Tax,” “Exceptional Items,” “Profit before Tax,” “Tax Expense,” “Profit for the Year,” “Other Comprehensive Income,” “Total Comprehensive Income (Loss) or the Year,” and “Earnings per Share.” The net income of the company for the financial year 2019–2020 was ₹67,438 million. The net income for the current year is lower than that of the previous year by ₹37,894 million. 2. The largest current asset of the company was inventories. The largest current liability of the company was trade payables. The balance in “Property, Plant, and Equipment” account was ₹663,923.5 million. The original cost was ₹834,439 million and accumulated depreciation and impairment was ₹170,515.5 million. 3. The largest component of Tata Steel’s stockholders’ equity was “other equity.” It represents 95.5% of total stakeholders’ equity. This implies strong profitability in the past because the retained earnings are the summation of past profits less dividends. 4. The cash flow from operating activities was ₹134,536.6 million, which is more than the net income (₹67,438 million) of the company. This was mainly for two reasons: (a) depreciation and amortization expense, which is a non-cash operating expense, and (b) finance cost, which is added back to net income to calculate cash flow from operating activities. The cash and cash equivalent increased by ₹4487.9 million. 5. The depreciation and amortization expense were ₹39,201.2 million and purchase of capital assets (other than investments) was ₹47,492.8. The cost of acquisition of fixed assets is larger. This shows that Tata Steel is still growing.

..

715


CHAPTER 17 COVERAGE OF LEARNING OBJECTIVES

LEARNING OBJECTIVE LO1: Contrast accounting for investments using the equity method and the market-value method. LO2: Explain the basic ideas and methods used to prepare consolidated financial statements. LO3: Describe how goodwill arises and how to account for it. LO4: Use financial statement analysis to evaluate an organization’s performance. LO5: Explain and use a variety of popular financial ratios. LO6: Identify the major implications that efficient stock markets have for accounting.

..

EXCEL, FUNDAMENTAL ADDITIONAL COLLAB., & ASSIGNMENT ASSIGNMENT INTERNET MATERIAL MATERIAL EXERCISES A1, B1, B3, B5 4, 25, 25, 29, 36, 29, 39, 41, 51 39, 41

A2, A3, B2, B5

7, 26, 30, 32, 34, 34, 39, 40, 51 35, 36, 39, 40, 42, 43, 48, 49

A4, B5

12, 31, 32, 33, 44, 45

B5

14, 27, 37, 38, 47

A5, B4, B5, B5

27, 37, 38, 46, 47

B5

23, 24, 28, 28

45

37, 49, 50,51

716


CHAPTER 17 Understanding and Analyzing Consolidated Financial Statements 17-A1

(15-20 min.) Answers are in millions of dollars.

1.

a. Acquisition b. Net income of Mobile Media c. Dividends from Mobile Media Effects for year

Equity Method Assets = Liab. + Stk. Eq. InvestLiabilStock. Cash ments ities Equity -75 +75 = +12 = +12 +7 - 7 = -68 +80 = +12

The journal entries that would accompany this table are: a. Investment in Mobile Media Cash

75

b. Investment in Mobile Media Investment revenue*

12

c. Cash Investment in Mobile Media

7

75 12 7

* More frequently called Equity in Earnings of Affiliates Under the equity method, Star Electronics recognizes income as Mobile Media earns it rather than when Star Electronics receives dividends. Cash dividends do not affect net income; they increase cash and decrease the investment balance. In a sense, the dividend is a partial liquidation of the investor's "claim" against the investee. The receipt of a dividend is similar to the collection of an account receivable. The revenue from a sale of merchandise on account is recognized when the receivable is created; to include the collection also as revenue would be double-counting. Similarly, it would be double-counting to include the $7 million of dividends as income after the $12 million of income is already recognized as it is earned. 2.

a. Acquisition b. Dividends from Mobile Media c. Increase in market value Effects for year ..

Market-Value Method Assets = Liab. + Stk. Eq. InvestLiabilStock. Cash ments ities Equity -75 +75 = +7 = +7 (Revenue) +6 = + 6 (Other comprehensive income) -68 +81 = +13 717


The journal entries that would accompany this table are: a. Investment in Mobile Media Cash

75

b. Cash Dividend revenue**

7

c. Investment in Mobile Media Unrealized gain on available-for-sale securities

6

75 7 6

** Frequently called "dividend income" 17-A2 (25-35 min.) A common mistake is to think that the $80 million is additional money flowing into the Neptune Company rather than into the pockets of the Neptune shareholders as individuals. Amounts are in millions. 1.

Assets Investment in Neptune +

Poseidon’s accounts, Jan. 1: Before acquisition Acquisition of Neptune +80 Neptune’s accounts, Jan. 1 Totals before eliminating entries 80 Intercompany eliminations -80 Consolidated, Jan. 1 0 2. Sales Expenses Operating income Pro-rata share (100%) of unconsolidated subsidiary net income Net income

..

Cash and Other Assets

350 - 80 100 370 +

370

= Liab.+ Stockholders' Equity Accounts StockPayable, holders' = etc. + Equity

= = = = = =

Poseidon $330 245 $ 85

Neptune $110 90 $ 20

20 $105

$ 20

110

+

240

20 130

+

130

+

80 320 - 80 240

Consolidated $440 335 $105

718


3.

Poseidon’s parent-company-only income statement would show its own sales and expenses plus its pro-rata share of Neptune's net income, as the equity method requires. Reflect on the changes in Poseidon’s balance sheet equation (in millions): Assets = Liab.+Stockholders' Equity InvestCash ment and Accounts Stockin Other Payable, holders' Neptune + Assets = etc. + Equity

Poseidon’s accounts: Beginning of the year 80 Operating income Share of Neptune's income +20 End of year 100 Neptune's accounts: Beginning of the year Net income End of the year Totals before eliminating entries 100 Intercompany eliminations -100 Consolidated, end of year 0

+

270 + 85

= =

+

355

= =

+

100 + 20 120 475

+

475

= = = = = =

110

+

110

+

20

+

20 130

+ +

130

+

240 + 85 + 20 345 80 + 20 100 445 -100 345

Neptune’s balance sheet accounts would have increased by $20 million. At this point, review to see that consolidated statements are the summation of the individual accounts of two or more separate legal entities. These statements are prepared periodically via worksheets. A consolidated entity does not have a separate continuous set of books like its legal entities. Moreover, a consolidated income statement is merely the summation of the revenue and expenses of the separate legal entities being consolidated after the elimination of double-counting. 4.

..

Consolidated accounts would be unaffected. Neptune’s cash and stockholders' equity would decline by $10 million. Poseidon’s investment in Neptune would decline by $10 million, but Poseidon’s cash would rise by $10 million.

719


17-A3 (30-45 min.) A common error is to think that the $64 million is additional money flowing into the Neptune Company rather than into the pockets of the Neptune shareholders. Amounts are in millions. 1.

Assets Investment in Neptune

Poseidon’s accounts, Jan. 1: Before acquisition Acquisition of Neptune Neptune’s accounts, Jan. 1 Totals before eliminating entries Intercompany eliminations Consolidated, Jan. 1

=

Cash and Other + Assets

+64 64 + -64 0 +

Liab.+Stockholders' Equity

Accounts NonconPayable, trolling = etc. + Interest +

Stockholders' Equity

350 - 64

= =

110

+

240

100 386

= = = =

20 130

+ +

80 360 - 80 240

386

130

+16 + 16

+

2.

The same basic procedures are followed by Poseidon and Neptune regardless of whether Neptune is 100% owned or 80% owned. However, the presence of a noncontrolling interest changes the consolidated statements slightly. The income statements would include: Poseidon Neptune Consolidated Sales $330 $110 $440 Expenses 245 90 335 Operating income $ 85 $ 20 $105 Pro-rata share (80%) of unconsolidated subsidiary net income 16 Net income $101 $ 20 Noncontrolling interest (20%) in consolidated subsidiaries' net income 4 Net income to consolidated entity $101

..

720


3.

Assets Investment in Neptune

Poseidon’s accounts: Beginning of year 64 Operating income Share of Neptune's income + 16 End of year 80 Neptune’s accounts: Beginning of year Net income End of year Totals before eliminating entries 80 Intercompany eliminations -80 Consolidated, end of year 0 a

=

Cash and Other + Assets

Liab.+Stockholders' Equity

Accounts NonconPayable, trolling = etc. + Interest +

+

286a = + 85 =

+

371

= =

100 + 20 120 + 491 ____ + 491

= = = = = =

110

+

110

+

20

+

20 130 + + 130 +

+ +

20b 20

+

Stockholders' Equity 240 + 85 + 16 341 80 + 20 100 441 - 100 341

350 beginning of year - 64 for acquisition = 286 16 beginning of year + (.20 × 20) = 16 + 4 = 20

b

4.

Consolidated accounts would be affected because the noncontrolling interest's claim would be partially liquidated in the amount of 20% of $10 million, or $2 million. Neptune’s cash would decline by $10 million, Poseidon’s investment in Neptune would decline by .80 × $10 million = $8 million, but Poseidon’s cash would rise by $8 million. See following balance sheet equations: Assets InvestCash ment and in Other Neptune + Assets

End of year balances: Poseidon’s accounts Effect of Neptune dividend Balance

80

+

- 8 72

+

=

Accounts NonconPayable, trolling = etc. + Interest +

371 = +

Liab.+Stockholders' Equity

8 = 379 =

Stockholders' Equity

110

+

341

110

+

341

20

+

20 130

+ +

100 - 10 90 431 - 90 341

Neptune’s accounts (from 3): Effect of Neptune dividend Balance Totals before eliminating entries Intercompany eliminations Balance ..

72 - 72 0

+ +

120 = - 10 = 110 = 489 = = 489 =

+ 130 +

18 18

721


17-A4 (25-35 min.) 1.

Assets InvestCash ment and in Good- Other Neptune + will + Assets

Poseidon’s accounts, Jan. 1: Before acquisition 350 Acquisition of 100% of Neptune +105 - 105 Neptune’s accounts, Jan. 1 100 Totals before eliminating entries 105 + 345 Intercompany eliminations - 105 +25 ___ Consolidated, Jan. 1 0 + 25* + 345

= Liab.+Stockholders' Equity

=

Accounts Payable, etc.

+

Stockholders' Equity

=

110

+

240

20 130

+ +

80 320 - 80 240

= = = = =

130

* The $25 million "goodwill" would appear in the consolidated balance sheet as a separate intangible asset account. It often is shown as the final item in a listing of assets. It remains on the books until its value is impaired. 2.

a.

If the book values of the Neptune’s individual assets are not equal to their fair values, the usual procedures are: (1) Neptune continues as a going concern and keeps its accounts on the same basis as before. (2) Poseidon records its investment at its acquisition cost (the agreed purchase price). (3) For consolidated reporting purposes, the excess of the acquisition cost over the book values of Neptune is identified with the individual assets, item by item. (In effect, they are revalued at the current market prices prevailing when Poseidon acquired Neptune.) Any remaining excess that cannot be identified is labeled as purchased goodwill.

The balance sheet accounts immediately after acquisition would be the same as in Requirement 1, except that goodwill would be $13 million instead of $25 million (that is, $27 million - $15 million = $12 million less), and other assets would be higher by $12 million. The $12 million would appear in the consolidated balance sheet as an integral part of the "other assets." That is, Neptune’s equipment would be shown at $12 million higher in the consolidated balance sheet than the carrying amount on Neptune’s books. Similarly, the depreciation expense on the consolidated income statement would be higher. For instance, if the equipment had four years of useful life remaining, the straightline depreciation would be $12 million ÷ 4 = $3 million higher per year. As in

..

722


b.

the preceding tabulation, the $13 million "goodwill" would appear in the consolidated balance sheet as a separate intangible asset account. Consolidated income would be lower by the amount of depreciation on the additional individual assets: Extra annual depreciation, $12,000,000 ÷ 4 years = $3,000,000 The assigning of a "basket purchase price" to the various assets can have a dramatic effect on income. Every dollar assigned to individual assets rather than goodwill will become an expense sometime, but dollars assigned to goodwill remain indefinitely on the books until the value of the goodwill becomes impaired.

17-A5 (10-15 min.) 1.

(a) (b) (c)

$500 million × 12% = $60 million $60 million ÷ 6% = $1,000 million $1,000 million ÷ $500 million = 2.0 times; or 12% ÷ 6% = 2.0 times

2.

(a) (b) (c)

¥200 million ÷ 4 = ¥50 million ¥20 million ÷ ¥200 million = 10% ¥20 million ÷ ¥50 million = 40%; or 10% × 4 = 40%

17-B1 (15 min.) The year-end balance in Investment in Chang is $57 million under the equity method, and $39 million under the market-value method. 1.

Assets Cash

Equity Method: 1. Acquisition 2. Net income of Chang 3. Dividends from Chang Effects for year Market-Value Method: 1. Acquisition 2. Dividends from Chang 3. Adjustment to market value Effects for year

..

-50

+ Investments

= Liab.+Stockholders' Equity Stockholders' = Liabilities + Equity

+50 +10

= =

+10

+ 3 -47

- 3 +57

= =

+10

-50 +3

+50

= =

+3 (revenue)

= =

-11 (loss) - 8

-47

- 11 +39

723


Journal entries (not required): Equity Method 1. Investment in Chang Cash

50 50

2. Investment in Chang Investment revenue*

10

3. Cash Investment in Chang

3

10 3

* More frequently called Equity in Earnings of Affiliates Market-Value Method 1. Investment in Chang Cash

50 50

2. Cash Dividend revenue**

3

3. Loss on trading securities Investment in Chang

11

3 11

** Frequently called "dividend income" Google would be required to use the equity method because its ownership of 33% is between 20% and 50%.

..

724


17-B2 (25-40 min.) Amounts are in millions of dollars. This is a real acquisition, although the numbers have been rounded off for simplicity. A common mistake is to think that the $400 million is additional money flowing into Bayliner rather than into the pockets of Bayliner shareholders as individuals. 1.

Assets Investment in Bayliner

Brunswick's accounts, Jan. 1: Before acquisition Acquisition of Bayliner Bayliner's accounts, Jan. 1: Totals before eliminating entries Intercompany eliminations Consolidated, Jan. 1 2. Sales Expenses Operating income Pro-rata share (100%) of unconsolidated subsidiary net income Income of parent company

..

=

1,400 - 400 +

600 1,600

+

1,600

+

+400 400 -400 0

= Liab.+Stockholders' Equity Cash and Other Assets

Brunswick $1,800 1,400 $ 400

Accounts Payable, etc.

+

Stockholders' Equity

= =

800

+

600

= = = =

200 1,000

+ +

1,000

+

400 1,000 -400 600

Bayliner $500 350 $150

Consolidated $2,300 1,750 $ 550

150 $ 550

725


3.

Brunswick's parent-company-only income statement would show its own sales and expenses plus its pro-rata share of Bayliner's net income (as the equity method requires). Reflect on the changes in Brunswick's balance sheet equation (in millions of dollars): Assets Investment in Bayliner

Brunswick's accounts: Beginning of year Operating income Share of Bayliner's income End of year Bayliner's accounts: Beginning of year Net income End of year Totals before eliminating entries Intercompany eliminations Consolidated, end of year

+

400 + +150 550 +

550 + -550 0 +

= Liab.+Stockholders' Equity Cash and Other Assets

=

1,000 +400

= =

1,400

= =

600 +150 750 2,150 2,150

= = = = = =

Accounts Payable, etc.

+

800

+

800

+

200

+

200 1,000

+ +

1,000

+

Stockholders' Equity 600 +400 +150 1,150 400 +150 550 1,700 -550 1,150

4.

The important point to see is that the consolidated accounts would be unaffected. Bayliner's cash and stockholders' equity would decline by $12 million. Brunswick's investment in Bayliner would decline by $12 million, but Brunswick's cash would rise by $12 million.

..

726


17-B3 (15-20 min.) 1.

Under the equity method Berkshire Hathaway will recognize 13.0% of American Express's net income: 13.0% × $4,935,000,000 = $641,550,000.

2.

The balance is increased by Berkshire Hathaway's share of American Express's net income ($641,550,000 from requirement 1) and decreased by the cash dividends received from American Express (13.0% × $855,000,000 = $111,150,000): $6,200,000,000 + $641,550,000 - $111,150,000 = $6,730,400,000. Using a T account might clarify the calculation: Investment in American Express Company Beginning bal. 6,200,000,000 Dividends received from Equity in American American Express Express’s net income 641,550,000 Ending balance

3.

111,150,000

6,730,400,000

(a)

Under the market-value method the dividends received from American Express would be recognized as income by Berkshire Hathaway: 13.0% × $855,000,000 = $111,150,000

(b)

The account balance would be adjusted to market value, $7,151 million.

(c)

The $747 million increase would be added to other comprehensive income, which Berkshire Hathaway shows in its statement of stockholders’ equity.

4. Berkshire Hathaway is obliged to follow the generally accepted accounting principles for investments:

..

(a)

Investments that represent more than a 50% ownership interest must be consolidated. A subsidiary is a corporation controlled by another corporation. The usual condition for control is ownership of a majority (more than 50%) of the outstanding voting stock. In parent-company-only statements, the equity method is used.

(b)

The equity method is generally used for a 20% through 50% interest because such a level of ownership creates the presumption that the owner has the ability to exert significant influence. However, consolidated statements are not reported.

(c)

All other investments in marketable equity securities must be accounted for using the market-value method.

727


17-B4 (10-20 min.) Amounts are in millions. 1. a. Operating return on sales = operating income ÷ sales Operating income = sales × operating return on sales Operating income = £1,501.3 × 13.741% = £ 206.3 b. Oper. return on ave. total assets = oper. income ÷ ave. total assets Ave. total assets = operating income ÷ oper. return on ave. total assets Ave. total assets = £206.3 ÷ 16.610% = £1,242.0 c. Return on shareholders’ equity = net income ÷ ave. shareholders’ equity Ave. shareholders’ equity = net income ÷ return on shareholders’ equity Ave. shareholders’ equity = £206.3 ÷ 30.855% = £668.6 2. Shareholders are most concerned with return on shareholders’ equity. It shows the amount of net income generated by the shareholders’ claims. Return on sales can vary greatly by industry, and it does not necessarily show how profitable a company is to its investors. 17-B5

(20 min.)

1. 2. 3. 4. 5. 6. 7. 8. 9.

4,309  2,128 = 2.02 4,667  2,755 = 169.4% 5,274 14,549 = 36.25% 833  14,549 = 5.7% 833  [(1/2) × (2,755 + 4,080)] = 24.37% (833 - 0)  529 = $1.57 18.93  1.57 = 12.1 0.45  18.93 = 2.4% 0.45  1.57 = 28.7%

17-1 Trading securities are investments “held for current resale,” typically investments that management intends to sell shortly. Available-for-sale securities are all other investments in marketable securities – those that management does not intend to sell in the near future. 17-2 No. It is true that increased in the market value of available-for-sale securities is added to other comprehensive income. However, other comprehensive income is a separate account in stockholders’ equity, not a part of net income. Items that end up in other comprehensive income (loss) are not added (subtracted) in computing net income – they are carried directly to stockholders’ equity. They may appear after net income at the bottom of the income statement (in fact, under IFRS they must appear on the income statement), but they are separate from net income. On the statement of stockholders’ equity, net income affects retained earnings and other comprehensive income affects accumulated other comprehensive income.

..

728


17-3 Under the equity method, investments are carried in the balance sheet at original cost plus the investor's share of accumulated retained income since acquisition (that is, original cost plus accumulated net income less dividends received). 17-4 While accounting for investments, a company must follow the market value method if it invests in less than 20% of the common stock of another company. 17-5 The equity method is usually appropriate for long-term investments when the investor has an ownership interest between 20% and 50%, because the owner would usually have the ability to exert significant influence over the investee. 17-6 A parent-subsidiary relationship exists when one corporation (the parent) exerts control over another. This generally occurs when one corporation owns more than 50% of the outstanding voting shares of another corporation (the subsidiary). 17-7 There are many benefits. Subsidiaries often help to limit risk, save income taxes, help conform to government regulations, and allow the parent to do business in a foreign country. 17-8 No, consolidations are more complex than simply adding together the separate accounts. After adding together the separate statements, intercompany eliminations must be undertaken to avoid double-counting. 17-9 No. The parent-company-only and consolidated statement will show the same net income. Only the way it is presented will differ. 17-10 If the parent owns less than 100% of the subsidiary stock, then outsiders to the consolidated group own the remainder. The account Noncontrolling Interest in Subsidiaries is a measure of the outside shareholders’ interest. Note that this noncontrolling interest is an interest in the subsidiary, not in the parent company or the consolidated company. 17-11 Goodwill is measured by the excess of purchase price over the fair-value, not the book value, of the net assets (assets less liabilities) acquired. 17-12 The acquirer company must assign the excess of the acquisition cost over the fair value of assets less liabilities to goodwill. 17-13 Goodwill is not systematically reduced. It is only reduced when management deems that its value has been impaired. 17-14 Pro forma statements show hypothetical (“as if”) amounts. Formal financial statements report historical results following U.S. GAAP for U.S. companies and either U.S. GAAP or IFRS for non-U.S. companies filing with the SEC.

..

729


17-15 It is difficult to compare financial statements of firms that differ in size. Using component percentages (or common-size statements) allows direct comparison of percentages across companies that differ in size. 17-16 MD&A is management’s discussion and analysis of an organizations financial statements. It is useful to investors because it describes what management posits as the reason for many of the changes in the financial statements in the last year. 17-17 Three types of comparisons are: 1) time-series comparisons, 2) comparisons with benchmarks, and 3) cross-sectional comparisons. 17-18 Two short-term ratios are the current ratio and the quick ratio. Five profitability ratios are gross profit rate or percentage, return on sales, return on stockholders’ equity, earnings per share, and price-earnings ratio. 17-19 Pre-tax operating rate of return on total assets = operating income percentage on sales × total asset turnover. 17-20 Ratios are mechanical because their computation requires following a set rule. They are incomplete because they give only a hint as to their importance or relevance; they must be used in conjunction with further information. 17-21 No. An efficient capital market is one in which market prices "fully reflect" all information publicly available at a given time. Therefore, searching for "underpriced" securities using public information is fruitless. 17-22 Sources of information include dividend announcements, industry statistics, trade publications, and national and global economic indicators. 17-23 The quote assumes that the market applies a fixed price-earnings ratio to income, regardless of the accounting methods used to calculate net income. There is much evidence that this is not so. If drug development costs are already disclosed, it is highly unlikely that requiring them to be expensed would have affected Sun Pharma’s share price. 17-24 There is much evidence showing that the stock market is not likely to be “fooled” by accounting changes that mechanically increase reported income. Only an accounting change that discloses new information will affect stock prices.

..

730


17-25 A purchase of about 20% of another company places the purchase at the borderline between the market-value or equity methods. More than 20% and the equity method should be used; less than 20% and Novartis should use the market-value method. The initial recorded value will be the same under either method, but the accounting subsequent to the acquisition will differ. Under the equity method, changes in market value are ignored. The book value of the investment will increase only by Novartis’s share of the company’s profits less any dividends, which is likely to result in only a very small increase in the reported asset value. In contrast, the market-value method would record the asset at its market value, which Novartis expects to increase significantly. Thus, if Novartis’s expectations are met, the market-value method would eventually result in much larger asset value recorded in Novartis’s investment account on its balance sheet. To achieve this, Novartis must buy less than 20% of the company. 17-26 The amount that P pays above the book value of the net assets of S consists of two parts. The first part is an adjustment of the book values of S’s net assets to market values. This amount becomes part of the depreciation of S’s assets in the consolidated statements, although it is not charged on S’s books. The second part is the amount that P paid in excess of the market value of the net assets. This additional amount is goodwill. It was initially recorded as an asset. Because some of the goodwill was written off, management must have determined that the value of the goodwill asset had declined since the purchase. 17-27 If a company reduces its inventories (a component of current assets), and everything else remains unchanged, its current ratio will decrease. Because holding inventory can be expensive, both for the capital invested in them and for costs of handling and storing the inventories, it is often good to reduce inventories. However, many analysts think that companies benefit from a higher current ratio. This seems to generate a conflict. In general, analysts will look at the current ratio differently for companies that use a just-in-time inventory system. They will expect such companies to have a lower current ratio and thus will not downgrade their estimate of the liquidity of such companies. An old rule of thumb was that most companies should have a current ratio of about 2.0 to have sufficient liquidity. However, recently that standard has been lowered, especially for companies using just-in-time inventory methods. 17-28 If markets are efficient, investors cannot make abnormal returns by investing on the basis of publicly available information. Unless you have access to new information or have some special ability to obtain and process information, your best investment strategy is probably to seek a diversified portfolio that, across time, will return about the market-average return.

..

731


17-29 (15 min.) The year-end balance in Investment in Beta is €84 million under the equity method, and €96 million under the market-value method: (in € millions) Assets = L iab. + Stockholders’ Equity Cash + Investments = Liabilities + Stk. Equity Equity Method: 1. Acquisition -80 +80 = 2. Net income of Beta + 8 = + 8 3. Dividends from Beta + 4 – 4 = Effects for year – 76 +84 = + 8 Market-Value Method: 1. Acquisition – 80 +80 = 2. Net income of Beta No entry and no effect. 3. Dividends from Beta + 4 = + 4 4. Increase in market value of Beta + 16 = + 16 Effects for year – 76 +96 = +20 If this were a trading security, the €16 million increase in market price would be included in the income. If it were an available-for-sale security, the €16 million increase would be added directly to the other comprehensive income account in stockholders’ equity.

..

732


17-30 (35-50 min.) The formal statements are not presented here because the following tabulations are easier to understand (in thousands of dollars): 1. Ace Mining’s net income would have been $450,000 - $200,000 = $250,000. 2. (In thousands) Sales (other income is reclassified below) Expenses Operating income Ace Mining’s share of Alberta' net income Net income *5,500 - 200 = 5,300

Ace Mining 5,300* 5,050 250 200 450

Assets InvestCash ment and in Other Alberta + Assets Ace Mining’s accounts: Beginning of year Investment in Alberta Operating income Share of Alberta’ income End of year Alberta’ accounts: Beginning of year Net income End of year Totals before eliminating entries Intercompany eliminations Consolidated, end of year

+300 +200 500 +

500 + -500 0 +

3. Sales (other income is reclassified below) Expenses Operating income Ace Mining’s share (60%) of Alberta' net income Net income Noncontrolling interest (40%) in Alberta’ net income Net income to consolidated entity

Alberta 1,100 900 200 200

Consolidated 6,400 5,950 450

= Liab.+Stockholders' Equity

=

1,000 - 300 + 250

= = =

Accounts Payable, etc.

+

Stockholders' Equity

400

+

600 + 250

= 950 550 200 750 1,700

= = = = = =

1,700

Ace Mining 5,300* 5,050 250 120 370

400

+

250

+

250 650

+ +

650

+

+ 200 1,050 300 + 200 500 1,550 - 500 1,050

Alberta Consolidated 1,100 6,400 900 5,950 200 450 200 80 370

*5,420 - 120 = 5,300 ..

733


The consolidated balance sheet would be as follows: Assets = Liab.+Stockholders' Equity InvestCash ment and Accounts Noncon- Stockin Other Payable, trolling holders' Alberta + Assets = etc. Interest + Equity Ace Mining’s accounts: Beginning of year 1,000 Investment in Alberta + 180 + - 180 Operating income + 250 Share of Alberta’ income + 120 End of year 300 + 1,070 Alberta’ accounts: Beginning of year 550 Net income 200 End of year 750 Totals before eliminating entries 300 + 1,820 Intercompany eliminations - 300 Consolidated, end of year 0 + 1,820

= = =

400 +

600 250

=

+120 970

400 + = = = = = =

250 + 250 + 650 + 650 +

+200* 200 +

300 200 500 1,470 - 500 970

* Beginning noncontrolling interest (.40 × $300 = $120) plus noncontrolling interest in net income (.40 × $200 = $80) 17-31

(10 min.)

The $50,000 "goodwill" would appear as a separate intangible asset account in the consolidated balance sheet. As long as the goodwill is not impaired, the consolidated balance sheet at year-end would continue to show goodwill at $50,000, and consolidated net income would not be affected.

..

734


17-32

(20-25 min.) Amounts are in millions of dollars.

1.

Cash Cloud's accounts: Before acquisition 850 + Acquisition of Tron -300 Tron's accounts 80 + Totals before eliminating entries 630 + Intercompany eliminations Consolidated 630 +

Assets Plant InvenAssets, tories Net 350

+

= Investment in Tron

390

= =

+300

Stockholders' Equity Common Stock, Retained etc. Income 620

+

970

=

120

+

90

=

70

+

60

420

+

450

+

300

=

740

+ 1,060

+

+ 90 540* +

-300 0

= =

-120 620

- 90 970

420

+

*The $90 million would appear as an integral part of the plant assets because they would be carried at $90 million higher in the consolidated balance sheet than the carrying amount on Tron's books. Therefore, plant assets would appear on the consolidated balance sheet as $390 + ($60 + $90) = $540. 2.

The Tron plant assets would be carried in the consolidated balance sheet at $100 million instead of $150 million, and the $50 million difference between the $300 purchase price and the $250 fair value of net assets acquired ($80 + $70 + $100) would be recorded as goodwill, a separate intangible asset.

3.

Cash would be $40 million less, and a goodwill account of $40 million would be created. The balance in Cloud's Investment in Tron account would be $340 million instead of $300 million. Of the $340 million, $150 million would pertain to plant assets, and $40 million to goodwill in the consolidated statement.

..

735


17-33

(10-15 min.)

Many students will fail to see that depreciation must be adjusted. The assumption here is that a 20% rate is appropriate for the year in question. Why? Because Tron’s depreciation is $12 million; therefore, the rate must be $12 ÷ $60 = 20% of Tron’s cost basis. The fair value of net assets acquired is Cash = $80, Inventories = $70, and Plant Assets = $150 (with a book value of $60) for a total fair value of $300. Thus, goodwill is $340 $300 = $40. The $90 excess of fair value of plant assets over book value will result in increased depreciation. 1. The computation (in millions) follows: Consolidated Net income before adjustments, $105 + $35 $140 Adjustments: Depreciation, 20% of $90 (18) Net income $122 2.

..

Goodwill write-off is $20 million. Net income would be $122 - $20 = $102 million.

736


17-34

(30 min.)

1.

The combined company would have the following balance sheet accounts immediately after the acquisition (in millions of rupees): Cash and receivables 2,400 + 1,760 = 4,160 Inventories 9,600 + 240 + 6,400 = 16,240 Plant assets, net 12,000 + 7,600 = 19,600 Total assets 40,000 Current liabilities Common stock Retained income Total equities

2.

4,000 + 1,600 = 5,600 8,000 + 14,400 = 22,400 = 12,000 40,000

Net income for 2019 Net income for 2020: ₹1,600 + [₹1,280 – (0.25 × ₹6,400)]

₹1,520 million ₹1,280 million

If the ₹6,400 million were assigned to goodwill and no goodwill was written off, net income for 2020 would be: 1,600 + 1,280 = ₹2,880 million The chairman of Gemini Pictures may prefer to assign as much as possible to goodwill because Gemini Pictures could generate net income almost “on demand” by the timing of rentals or sales of its films. This avoids revelation of the cost of the library of films acquired. The immense size of this impact on income is vividly demonstrated by the fact that the net income could jump to ₹2,880 million in just one year. This could take place because Gemini Pictures would carry the film library at zero rather than the ₹6,400 million actual cost. The point of this example is to stress that, depending on earnings-per-share objectives, there will be a general pressure by the management toward assigning as much of the total purchase price as possible to goodwill rather than to other assets.

..

737


17-35

(50-65 min.)

This is a worthwhile problem because it provides an overall view of relationships. On balance sheets the noncontrolling interest in subsidiaries is a subpart of the stockholders' equity section. On income statements, the noncontrolling interest in consolidated subsidiaries’ net income is deducted as if it were an expense of the consolidated entity. CYPRESS TOOL & DIE COMPANY Consolidated Income Statement For the Year Ended December 31, 20X1 (in millions of dollars) Net sales and other operating revenue Cost of goods sold and operating expenses, exclusive of depreciation and amortization Depreciation and amortization Total operating expenses Operating income before share of unconsolidated net income Equity in earnings of affiliated companies Total income before interest expense and income taxes Interest expense Income before income taxes Income tax expense Income before noncontrolling interest Noncontrolling interest in consolidated subsidiaries’ net inc. Net consolidated income to Cypress Tool and Die Company Preferred dividends ($3.50 × 2,000,000) Net income to Cypress Tool and Die Company common stockholders Earnings per share of common stock: On shares outstanding (11,000,000 shares)

$900 660 20 680 220 20 240 25 215 90 125 20 105* 7 $ 98 $8.91**

* This is the total figure in dollars that the accountant traditionally labels net income. It is reported accordingly in the financial press. ** This is the figure most widely quoted by the investment community.

..

738


CYPRESS TOOL & DIE COMPANY Consolidated Balance Sheet As of December 31, 20X1 (in millions of dollars) Current assets: Cash Short-term investments at market value Accounts receivables, net Inventories at average cost Total current assets Investments in affiliated companies Property, plant, and equipment, net Other assets: Goodwill Total assets Current liabilities: Accounts payable Accrued income taxes payable Total current liabilities Long-term liabilities: First mortgage bonds, 10% interest, due Dec. 31, 20X9 Subordinated debentures, 11% interest, due Dec. 31, 20X8 Total long-term liabilities Total liabilities Stockholders' equity: Preferred stock, 2,000,000 shares, $50 par* Common stock, 11,000,000 shares, $1 par Paid-in capital in excess of par Retained earnings Total stockholders' equity attributable to Cypress stockholders Noncontrolling interest in subsidiaries Total stockholders’ equity Total liabilities and stockholders' equity

$ 20 45 175 340 580 100 125 95 $900 $210 20 230 80 100 180 410 100 11 101 188 400 90 490 $900

* Dividend rate is $3.50 per share.

..

739


17-36

(15-20 min.)

1.

Because Quantum Electronics accounts for its 19% investment in Conn Transport using the market-value method and classifies the securities as available-for-sale securities, only the amount of dividends paid by Conn are part of the income of Quantum Electronics. Changes in the market value of Conn are entered directly into Quantum’s stockholders’ equity. Thus, regardless of what happens to the market value of Conn, Quantum will include only Conn’s dividends in income. By increasing the Conn dividends, Quantum will increase its net income. This illustrates the principle that the FASB uses to govern whether the equity method should be used. The requirement is that an investment be accounted for under the equity method whenever the investee firm has “significant influence.” The usual indicator of significant influence is percentage ownership, where ownership of 20% or more creates the presumption of influence. However, the presumption can be overcome by other evidence of significant influence. The fact that Quantum was able to influence dividend policy so dramatically suggests that Quantum has significant influence over the investee, Conn, and therefore should account for the investment in Conn using the equity method. Note that under the equity method, the extra dividend will not be included in Quantum’s income.

2.

At least two ethical issues arise. First is the investment by Quantum in Conn. If the decision was made by Salvador Cruz based only on his friendship with Bud Conn, and if it was not in the best interests of the shareholders of Quantum, Cruz was not appropriately carrying out his duties as an officer of Quantum. Presently this may not be of much concern because the investment appears to have turned out to be profitable to the Quantum shareholders. Nevertheless, if the decision had been based on predicted personal rather than corporate benefits, it was not appropriate. Second is the influence of Cruz on Conn’s dividend policy. Not only does this manipulation of Conn’s policy violate the intent of the accounting principles, it may not be in the best interests of Conn’ other shareholders (i.e., those shareholders other than Quantum). If Conn pays out $4 million in dividends and then borrows to meet its capital needs, future profitability of Conn may be diminished. The personal obligation of Conn to Cruz should not influence the corporate decisions. On the other hand, Conn and Quantum may be essentially forming an implicit strategic alliance. When one company needs special help, the other is willing to provide it. Companies in Japan have had such alliances for years, and they have worked well. Often it is hard to judge the ethical implications of actions without being able to assess intent. Still, this situation at least possesses the appearance of possible ethical violations.

..

740


17-37 (10-15 min.) 1. Gross margin percentage: 2020: £1,300 ÷ £3,200 = 40.6% 2019: £1,160 ÷ £3,000 = 38.7% Return on sales: 2020: £520 ÷ £3,200 = 16.3% 2019: £444 ÷ £3,000 = 14.8% Return on stockholders’ equity: 2020: £520 ÷ [(£4,160 + £2,960) ÷ 2] = 14.6% 2019: £444 ÷ [(£2,960 + £2,960) ÷ 2] = 15.0% 2.

Both the gross margin percentage and the return on sales increased in 2020, which is generally good. However, the most important return to stockholders is the return on stockholders’ equity, which fell from 15% to 14.6%. If this is indicative of a long-run situation, it is not good for the stockholders. However, it may just be a temporary situation because of the inability to immediately put to work the additional funds raised in 2020.

17-38

(40-50 min.) Amounts are in millions of euros.

1.

(a)

Rate of return on sales: (Sales: 20X1: 380+620 = 1,000; 20X2: 520+980 = 1,500) 20X1: 60 ÷ 1,000 = 6.0% 20X2: 90 ÷ 1,500 = 6.0%

(b)

Rate of return on stockholders' equity: 20X1: 60 ÷ ½[(205 + 10) + (205 + 55)] = 25.3% 20X2: 90 ÷ ½[(205 + 55) + (205 + 110)] = 31.3%

(c)

Current ratio: 20X1: ( 5 + 70 + 85) ÷ 55 = 2.9 to 1 20X2: (20 + 85 + 120) ÷ 65 = 3.5 to 1

(d)

Ratio of total debt to stockholders' equity: 20X1: (65 + 55) ÷ (205 + 55) = 46.2% 20X2: (85 + 65) ÷ (205 + 110) = 47.6%

..

741


(e)

Ratio of current debt to stockholders' equity: 20X1: 55 ÷ (205 + 55) = 21.2% 20X2: 65 ÷ (205 + 110) = 20.6%

(f)

Gross profit rate: 20X1: 380 ÷ 1,000 = 38% 20X2: 520 ÷ 1,500 = 34.7%

(g)

Average collection period for accounts receivable: 20X1: [(1/2) × (40 + 70) × 365] ÷ 1,000 = 20.1 days 20X2: [(1/2) × (70 + 85) × 365] ÷ 1,500 = 18.9 days

(h)

Price-earnings ratio (Earnings per share are 60 ÷ 10 = 6 for 20X1 and 90 ÷ 10 = 9 for 20X2): 20X1: 90 ÷ 6 = 15 20X2: 117 ÷ 9 = 13

(i)

Dividend-payout percentage (Dividends per share are 15 ÷ 10 = 1.50 for 20X1 and 35 ÷ 10 = 3.50 for 20X2): 20X1: 1.50 ÷ 6 = 25.0% 20X2: 3.50 ÷ 9 = 38.9%

(j)

Dividend yield: 20X1: 1.50 ÷ 90 = 1.7% 20X2: 3.50 ÷ 117 = 3.0%

2.

(a) (b) (c)

Yes, a, b No, f No, a

(d) (e) (f)

Yes, b No, g Yes, i, j

(g) (h) (i)

No, d, e Yes, h No, c

(j) (k)

No, j Yes, g

3.

The company has grown rapidly and profitably (ratio b). Sales have tripled; earnings have nearly quadrupled; dividends have increased by a factor of 7; and total assets have increased 60%. Moreover, the large increase in retained income indicates that the expansion has been financed largely by internally generated funds. The expansion has been accompanied by increased liquidity of current assets (ratio c). The stock is priced reasonable low with respect to the level of earnings (h), and the dividend policy seems reasonable (i and j).

..

742


17-39

(10 min.)

The first two items indicate that there are noncontrolling (minority) shareholders in the subsidiaries, whose individual sales, assets, and other detailed accounts have been added together in Unilever’s consolidated statements. The net income attributable to noncontrolling interests appears in the income statement; the other account appears as part of stockholders’ equity on the balance sheet. The first item indicates the portion of Unilever’s income from consolidated subsidiaries that belongs to noncontrolling shareholders, that is, shareholders of the stock of the subsidiary that Unilever doesn’t own. The second item represents the equity of consolidated subsidiaries that Unilever does not own. The last two items summarize Unilever’s investments in affiliated (or associated) companies. Investments in affiliates appear under long-term assets in the balance sheet, and share of net profit of affiliates generally appears under “other income” in the income statement. The presence of affiliates indicates that Unilever owns between 20% to 50% of the shares of another company or companies—investments that are accounted for by the equity method. Investments in affiliates represent the carrying value of the investments and equity in earnings of affiliates represents Unilever’s share of the earnings of those affiliated companies. 17-40

(5-10 min.)

1. 2. 3. 4 5. 6.

Part of stockholders’ equity on the balance sheet. A component of income statement generally appears in other income. Non-current liability on the balance sheet. Part of current assets in the balance sheet. Part of non-current liabilities. Deducted in the income statement while calculating operating profit.

..

743


17-41

(15 min.)

1.

Siemens AG received €13,529 million in dividends from their equity affiliates in 2019. The balance sheet equation and T-account summaries are shown below (in millions of euros): Equity Method Investments Beg. Bal. 47,061 Income +3,064 Dividends -X Add. Investment + 0 End. Bal. 63,654

Equity Method Investments Beg. Bal. 47,061 Income 3,064 Dividends Add. Inv. 0 End. Bal. 63,654

X

€47,061 million + €3,064 million – X + €0 = €63,654 million X = €13,529 million 2.

Siemens recognized €3,064 million of income from the affiliated companies. Therefore, without the affiliates, pretax income would have been €12,596 – €3,064 = €9,532 million. The income from affiliates was more than 24% of Siemen’s pretax income.

3.

The income recognized would remain at €3,064 million. The amount of dividends paid by investees does not affect the parent’s income.

17-42

(20 min.) Amounts are in billions.

1.

¥ 465. All reported income would be attributable to Toyota’s stockholders.

2.

Let X = subsidiaries’ net income .10 × X = ¥57 X = ¥570

3.

¥215 ÷ ¥642 = 33.5%. The maximum ownership is 50%. If Toyota owned more than 50% of the company it would become a consolidated subsidiary.

..

744


17-43

(15 min.) Amounts are in millions.

1.

$14,443 million; the GE income would not change. Instead of consolidating all the accounts of GE and GECS, under the equity method GE would show the entire $6,637 income of GECS on one line, but the same amount of net income would be shown.

2.

$14,443 million - $6,637 million = $7,806 million. The income of GECS represents 46% of GE’s total net income.

3.

The consolidated statement allows a comprehensive look at the financial results of the entire entity owned by the shareholders of General Electric. For example, it would combine the amount of goods and services sold to customers of GE and GECS. The unconsolidated statement does not show all of these revenues. Similarly, the unconsolidated statement does not show all the individual expenses because those of GECS are offset against its revenues when the single line, earnings of nonconsolidated affiliates, is included in the GE unconsolidated statement. In contrast, the unconsolidated statements avoid combining assets, liabilities, revenues, and expenses for totally different types of operations (e.g., jet engine production and lending money). Some critics of consolidation maintain that adding the accounts of such unlike operations is like adding apples and oranges - it may make numerical sense but it does not make economic sense.

17-44

(10-15 min.) Amounts are in billions.

1.

Goodwill = $53.4 – ($29.7 + $10.0 – $21.2) = $34.9. This entire amount will stay on P&G’s balance sheet until the value of the goodwill has decreased (or been impaired).

2.

Goodwill would have been $34.9 + $29.7 = $64.6, which would remain on the books until it is impaired. In contrast, P&G would amortize the $29.7 of identifiable intangible assets over 10 years, resulting in amortization of $29.7 ÷ 10 = $2.97 per year. Thus, operating income will be less by $2.97 if P&G treats the $29.7 as specific intangible assets.

3.

A manager might prefer to have this as goodwill and avoid an amortization charge in future years. This will serve to increase the reported income in these years. The manager might especially favor treatment as good will if he or she has a bonus based on operating income or net income. Offsetting this is the need to write off some or all of the $34.9 if the goodwill should become impaired.

..

745


17-45

(15-20 min.)

1.

Goodwill would equal to the purchase price less the fair value of the net assets (tangible assets plus intangible assets less liabilities): Goodwill = $10 – ($0 + $8.4 – $1.9) = $3.5 billion

2.

Consolidated pretax income is Newmont’s pretax income (which is 2019 pretax income plus (minus) Goldcorp’s pretax income (loss) less the amortization of the intangible assets, which is $8.4 ÷ 10 = $0.84 billion. Therefore, consolidated pretax income = $2.88 – $0.515 – $0.84 = $1.525 billion.

3.

There would be no amortization of the intangible assets. Assuming no impairment of goodwill, the consolidated net income would be: $2.88 – $0.515 = $2.365 billion.

17-46

(10-20 min.) Dollar amounts are in thousands.

1.

Average assets: ($1,690,057 + $1,666,218) ÷ 2 = $1,678,137.5 Net income percentage of average assets: $24,355 ÷ $1,678,137.5 = 1.4513%

2.

Total revenues: $24,355 ÷ .0115427 = $2,109,992

3.

Average stockholders' equity: $24,355 ÷ .0350805 = $694,260

4.

Asset turnover:

..

(a)

$2,109,992 ÷ $1,678,137.5 = 1.26

(b)

1.4513% ÷ 1.15427% = 1.26

746


17-47

(15-20 min.) Monetary amounts are in billions of yen. HONDA MOTOR COMPANY Income Statement For the Year Ended March 31, 2011

Net sales Cost of sales Gross profit Selling and administrative expenses Research and Development Operating income Other income (expense): Interest income Interest expense Equity in income of affiliates Other, net Income before income taxes Income tax expense Net income * Adds to 8.7% due to rounding error. 2.

3.

..

Amount ¥8,937 6,497 2,440 1,383 487 570

Percentage 100.0% 72.7 27.3 15.5 5.4 6.4

23 (9) 140 46 770 207 ¥ 563

0.3 (.1) 1.6 0.5 8.6* 2.3 6.3%

a.

Current ratio = Current assets ÷ Current liabilities = ¥4,690 ÷ ¥3,568 = 1.31

b.

Total debt to equity = Total liabilities ÷ Stockholders' equity = (¥3,568 + ¥3,420) ÷ ¥4,583 = 152.5%

c.

Gross profit rate = Gross profit ÷ Sales = ¥2,440 ÷ ¥8,937 = 27.3% Note that the gross profit rate is shown in the common-size income statement.

d.

Return on stockholders' equity = Net income ÷ Avg. stockholders' equity = ¥563 ÷ 1/2(¥4,583 + ¥4,456) = 12.5%

e.

Price-earnings ratio = Market price per share ÷ Earnings per share = ¥2,207 ÷ ¥296 = 7.5

f.

Dividend-payout ratio = Dividends per share ÷ Earnings per share = ¥54 ÷ ¥296 = 18.2%

These ratios themselves are difficult to interpret without 1) time-series comparisons, 2) cross-sectional comparisons, and 3) comparisons with benchmarks. Thus, it would be helpful to know 1) Honda’s ratios for the past few years to aid the identification of trends, 2) average industry ratios for comparison, and 3) general benchmarks for Japanese rather than U.S. firms. 747


17-48 (15-25 min.) 1.

The fact that Nike’s financial statements are “consolidated” means that Nike combines the statements of the parent company with those of 50% to 100% owned subsidiaries. Because Nike shows no “minority [noncontrolling] interest” on either the income statement or balance sheet, it must own 100% of all of its subsidiaries (or any noncontrolling interest is not material).

2.

Sales among consolidated entities must be eliminated when forming consolidated statements. Thus, the sales from Converse to the Nike retail stores must be eliminated. That means that, after adding together the accounts of Nike and its subsidiaries, sales must be reduced by $10 million and cost of sales by $6 million in the income statement, reducing operating income by $4 million. On the balance sheet, since income is $4 million less, retained earnings will also be $4 million less. In addition, the $10 million initially shown on the balance sheet for Nike’s retail stores’ inventory must be reduced to Converse’s purchase price of $6 million, a $4 million decrease.

3.

Following are the interpretations of the changes in each of the ten ratios: a. Current ratio – The decline is generally bad, reflecting less liquidity. However, it might reflect better inventory control, possibly by using just-intime methods, which could be good. b. Average collection period – A slight improvement. c. Debt to equity – The increase shows an increasing use of debt, which increases risk but may also mean that Nike is making better use of leverage. d. Gross profit rate – The decrease, although not large, is bad news. It reflects decreasing margins on sales. e. Return on sales – The increase is positive, showing that control of expenses other than cost of sales must have been sufficient to offset the decrease in gross margin. f. Return on stockholders’ equity – The increase is positive and more than 1 percentage point. g. Earnings per share – The 14% increase in EPS is indeed good news. The earnings attributable to each share of stock are significantly higher in 2011 than in 2010. h. Price-earnings – The slight increase in the P-E ratio shows that investors have revised upward their predicted growth rate for Nike. This is generally a good sign. However, it should also be compared with the change in the average P-E ratio for all stocks; it might be caused at least partly by overall stock market movements, not just by expectations for Nike. i. Dividend yield – The dividend yield is slightly less in 2011. It means that investors are receiving a slightly smaller percentage of their investment in the form of dividends in 2011 than in 2010. j. Dividend payout – The decreasing dividend payout ratio can have mixed interpretations. Although the change is extremely small, a decrease in payout means the shareholders get less assured return. However, it can also mean that Nike has greater investment opportunities for the cash it is generating.

..

748


17-49 (25-35 min.) For the solution to this Excel Application Exercise, follow the step-by-step instructions provided in the textbook chapter. 1.

Gap’s current ratio of 2.02 is almost exactly equal to the 2 to 1 rule of thumb.

2. Gap’s debt to equity ratio shows that debt is 169.4% of equity. The rule of thumb is that this ratio should be no more than 100%, so Gap has more debt compared to equity than the rule of thumb deems desirable. 3. The dividend payout ratio is 28.7%. This means that Gap paid to shareholders only slight more than one quarter of the amount of its net income. Apparently the Gap board of directors believes that Gap has good opportunities to invest capital and thus elects to retain most of the income rather than pay it in dividends. 17-50

(60 min. or more)

The purpose of this exercise is two-fold. The first is to establish familiarity with four basic ratios. The second is to deduce why these ratios might vary from company to company. Computing the ratios will cause students to find and read a company’s annual financial statements. They will become familiar with some aspects of that particular company as well as learning where in the financial statements to find the needed information. Students’ reasoning skills will be developed when they come together as a group and try to determine reasons for differences in companies’ ratios. Some of the conclusions they may draw are:

..

a.

Earnings per share depend on the number of shares issued compared to the value of the company. If one share costs more than $100,000, as for Berkshire Hathaway class A shares, earnings per share will be much higher than for a share that costs $10. Earnings per share is also a good measure of the economic success of a company over the last year. If earnings per share decline from one year to the next, it is likely that economic results were not favorable.

b.

The price-earnings ratio depends primarily on the growth prospects for a company’s earnings. The greater the expected earnings growth, the greater the P-E ratio.

c., d.

Dividend yield is a measure of the amount of dividends a shareholder can expect per dollar invested. Dividend payout ratio is the percentage of income paid out in dividends. These two ratios are highly related. Companies that have excellent internal investment opportunities for cash (often growing companies) will generally pay out a low percentage of their income and have a low dividend yield.

749


17-51 (30-40 min.) NOTE TO INSTRUCTOR: This solution is based on the web site as it was in 2012 when the 2011 annual report was the latest one available. Be sure to examine the current web site before assigning this problem, as the information there may have changed. 1.

General Electric labels its financial statements “General Electric Company and consolidated affiliates.” Therefore, the company must consolidate some subsidiaries into its statements. On the income statement there is an account called “Net earnings attributable to noncontrolling interests,” so there must be some subsidiaries that are not 100% owned.

2.

GE calls its balance sheet a “Statement of Financial Position.” GE does not have subtotals for current assets and current liabilities, although the current items are listed first under both assets and liabilities. Noncontrolling interest is $1,696 million. This means that of the equity in the consolidated subsidiaries, GE owns all but $1,696 million, which is the amount of the equity in the subsidiaries held by outside shareholders.

3.

Yes. The income statement shows cash dividends of $0.61 per share, out of net income of $1.23 per share, for a payout ratio of $.61 ÷ $1.23 = 49.6%.

4.

GE’s return on stockholders’ equity for the last two years is: 2011: $13,120 ÷ (1/2 × ($116,438 + $118,936)) = 11.1% 2010: $11,344 ÷ (1/2 × ($118,936 + $117,291*)) = 9.6% *From 2010 annual report. All other numbers are in the 2011 annual report. Note that we used net earnings attributable to GE common shareholders and total GE shareholders’ equity in these calculations. The return increased in 2011 from what it was in 2010, which is good for investors. The increase shows GE’s slow recovery from the major recession on 2008-09, although the return remains below what it was before the recession.

..

750


Chapter 1 Managerial Accounting, the Business Organization, and Professional Ethics LEARNING OBJECTIVES: When your students have finished studying this chapter, they should be able to: 1. 2. 3. 4. 5. 6. 7.

Explain why accounting is essential for decision makers and managers. Describe the major users and uses of accounting information. Explain the role of budgets and performance reports in planning and control. Describe the cost-benefit and behavioral issues involved in designing an accounting system. Discuss the role accountants play in the company’s value-chain functions. Identify current trends in management accounting. Explain why ethics and standards of ethical conduct are important to accountants.

Copyright ©2023 Pearson Education, Ltd.

1


CHAPTER 1:

ASSIGNMENTS

CRITICAL THINKING EXERCISES 28. Finance and Management Accounting 29. Accounting’s Position in the Organization: Controller and Treasurer 30. Marketing and Management Accounting 31. Production and Management Accounting EXERCISES 32. Management Accounting and Financial Accounting 33. Planning and Control, Management by Exception 34. Line Versus Staff and Value Chain Responsibility 35. Microsoft’s Value Chain 36. Objectives of Management Accounting 37. Cost-Benefit of the Ethical Environment 38. Early Warning Signs of Ethical Conduct PROBLEMS 39. Management and Financial Accounting 40. Use of Accounting Information in Hospitals 41. Costs and Benefits 42. Importance of Accounting 43. Changes in Accounting Systems 44. Value Chain 45. Role of Controller 46. The Accountant’s Role in an Organization 47. Ethics and Accounting Personnel 48. Ethical Issues 49. Hundred Best Corporate Citizens CASES 50. 51. 52.

Line and Staff Authority Professional Ethics and Toxic Waste Information in Nike’s 10k Report

EXCEL APPLICATON EXERCISE 53. Budgets and Performance Evaluation COLLABORATIVE LEARNING EXERCISE 54. The Future Management Accountant INTERNET EXERCISE 55. Institute of Management Accountants (www.ifac.org)

Copyright ©2023 Pearson Education, Ltd.

2


CHAPTER 1: I.

OUTLINE

Management Accounting and Your Career A.

{L. O. 1}

Certified Management Accountant Financial accounting has long provided auditing positions that are typically staffed by Certified Public Accountants (CPAs) in the United States and chartered accountants (CAs) in many other nations. The certified management accountant (CMA) designation is the management accountant’s counterpart to the CPA. There are three key professional qualifications providing routes into accounting: ACA (Institute of Chartered Accountants), ACCA (Association of Chartered Certified Accountants), and CIMA (Chartered Institute of Management Accountants). Each of these qualifications is globally recognized. Each course can take 3 to 5 years.

B.

Training for Top Management Positions In addition to preparing you for a position in an accounting department, studying accounting, and working as a management accountant, can prepare you for the very highest levels of management, such as CEO. Users of Accounting Information In general, users of accounting information fall into two general categories. 1. Internal managers who use information for day-to-day operating decisions and for long-range strategic decisions, and 2. External parties, such as investors and government authorities, who use the information for making decisions about the company. The internal managers make use of Management Accounting information whereas the external parties make use of Financial Accounting information. See EXHIBIT 1-1 for the major distinctions between these two types of accounting information.

II.

Roles of Accounting Information A.

{L. O. 2}

Accounting System—a formal mechanism for gathering, organizing, and communicating information about an organization’s activities. A good accounting system helps an organization achieve its goals and objectives by helping to answer three types of questions. 1. 2.

Scorecard Questions (Am I doing well or poorly?)—accumulation and classification of data, Attention-Directing Questions (Which problems should I look into?)— focuses on operating problems and opportunities, and

Copyright ©2023 Pearson Education, Ltd.

3


3.

III.

Problem-Solving Questions (Of the several ways of doing the job, which one is best?)—quantifies the likely results of possible courses of action for long-range planning.

Management by Exception A.

{L. O. 3}

The Nature of Planning and Controlling The management process is a series of activities in a cycle of planning and control with Decision Making—the purposeful choice from among a set of alternative courses of action designed to achieve some objective, as the core. Planning—the setting of objectives and outlining how they will be attained. Controlling—the implementation of plans and using feedback to attain objectives. Planning determines action, action generates feedback, and feedback influences further planning and possible corrective actions. EXHIBIT 1-2 shows that the accounting system formalizes plans by expressing them as budgets. A budget is a quantitative expression of a plan of action. Budgets are the chief devices for compelling and disciplining management planning. Performance Reports—provide feedback by comparing results with plans and by highlighting Variances (i.e., deviations from plans). The accounting system records, measures, and classifies actions in order to produce performance reports. See EXHIBIT 1-3 for an example of a performance report for a hypothetical store, the Mayfair Starbucks. Management by Exception—concentrating on areas that need attention and ignoring areas that appear to be running smoothly. Managers use performance reports to investigate exceptions (i.e., items for which actual amounts differ significantly from budgeted amounts). Operations are then brought into conformity with plans, or the plans are revised.

B.

Influences on Accounting Systems An accounting system is a formal mechanism for gathering, organizing, and communicating information about an organization’s activities. In the European Union and more than 100 countries worldwide, companies must comply with International Financial Reporting Standards (IFRS). In the United States (and select other countries), reports for external users are bound by generally accepted accounting principles (GAAP) and legal requirements. The auditor must make a judgment as to whether the financial statements of the company comply with the approved accounting standard and that they are a true and fair view of the situation. The auditor’s role is to express an opinion on the financial statements; it is the responsibility of the company directors to prepare and present the financial statements and to set in place any internal controls, policies to protect

Copyright ©2023 Pearson Education, Ltd.

4


and make the most efficient use of an organization’s assets. Because government agencies have legal power to order into evidence any internal document that they deem necessary, internal accounting systems may be affected by government regulation. An example is the way universities and defense contractors must allocate their costs to government contracts. As of January 2021, all U.K. companies are required to use U.K.-adopted international accounting standards (IAS) as opposed to those adopted by the EU. Management Audit—a review performed by internal auditors of profit-seeking organizations and governmental agencies to determine whether the policies and procedures specified by top management have been implemented. Under the Companies Act 2016 in Malaysia, it is a requirement of all public listed companies, private limited companies, and branch offices in Malaysia to appoint an approved auditor to audit the accounts of the company. IV.

Cost-Benefit and Behavioral Considerations

{L. O. 4}

Cost-Benefit Balance—weighing estimated costs against probable benefits. This is the primary consideration in choosing among accounting systems and methods. The value of a system must be seen as exceeding its cost. Behavioral Implications—the accounting system’s effects on the behavior (decisions) of managers. A system that managers believe in and trust will be used more in making decisions than one they distrust.

V.

Planning and Control for Product Life Cycles and the Value Chain

{L. O. 5}

Product Life Cycle—the various stages through which a product passes: from conception and development; introduction into the market; maturation; and, finally, withdrawal from the market. In the planning process, managers must determine revenues and costs over the entire life cycle. Accounting also needs to track actual costs and revenues throughout the life cycle. Periodic comparisons between planned costs and revenues and actual costs and revenues allow managers to assess the current profitability of a product, determine its current life-cycle stage, and make any needed changes in strategy. See EXHIBIT 1-4 for a typical product life cycle. The Value Chain—set of business functions that add value to the products or services of an organization. These functions, not of equal importance, include research and development, design of products or services, production, marketing, distribution, and customer service. See EXHIBIT 1-5 for a depiction of business functions’ value chain. VI.

Accounting’s Position in the Organization

Copyright ©2023 Pearson Education, Ltd.

5


A.

Line and Staff Authority Line Authority—authority extended downward over subordinates. Staff Authority—authority to advise but not command. It may be exerted downward, laterally, or upward. See EXHIBIT 1-6 for how a traditional manufacturing company divides responsibilities between line and staff managers. Line Departments are those that are central to the mission of the organization, whereas Staff Departments lend support and service to the line department. See EXHIBIT 1-6 for a partial organizational chart of a manufacturing company. Although controllers have a staff role, they are generally empowered by the firm’s president to approve, install, and oversee the organization’s accounting system to assure uniform accounting and reporting methods.

B.

Controller and Treasurer Functions The chief financial officer (CFO), the top executive who deals with all finance and accounting issues in an organization, oversees the accounting function in most organizations. The controller (or comptroller in a government organization) is the top accounting officer in an organization. This executive, like virtually everyone in an accounting function, fills a staff role, whereas sales and production executives and their subordinates fill line roles. The duties of Treasurer include provision of capital, investor relations, short-term financing, banking and custody, credits and collections, investments, and risk management (insurance).

VI.

Adaptation to Change {L. O. 6} As decisions change, demands for information change. Accountants must adapt their systems to the changes in management practices and technology. Four major trends are influencing management accounting today: 1. 2. 3. 4. A.

Shift from a manufacturing to a service-based economy Increased global competition Advances in technology Changes in business process management

Service Sector The characteristics of service organizations include the following: 1. Labor is a major component of costs 2. Output is usually difficult to measure 3. Service organizations cannot store their major inputs and outputs

B.

Global Competition

Copyright ©2023 Pearson Education, Ltd.

6


There has been a shift in the balance of economic power in the world due to countries lowering tariffs and duties, and a trend toward deregulation. C.

Advances in Technology The dominant influence on management accounting over the past two decades has been technological change, affecting both the production and the use of accounting information. One of the more rapidly growing uses of technology is electronic commerce or e-commerce. A major effect of technology on accounting systems has been the growing use of enterprise resource planning systems (ERP), which refers to conducting business online.

D.

Changes in Business Process Management Some companies implement sweeping changes in operations through business process reengineering, the fundamental rethinking and radical redesign of business processes to improve performance in areas such as cost, quality, service, and speed. Companies can use computer-aided design (CAD) to design products that can be manufactured efficiently and computer-aided manufacturing (CAM) to produce a smoother, more efficient flow of production with fewer delays. The impact of both of these is to reduce processing time. Computer-Integrated Manufacturing (CIM) Systems—systems that use CAD and CAM together with robots and computer-controlled machines. Just-in-time (JIT) philosophy—the elimination of waste by (1) reducing the time that a product spends in the production process and (2) eliminating the time that products spend on activities that do not add value (e.g., inspection and waiting time). While it was a dominant mode of inventory keeping for manufacturers, it was crucial for suppliers to be totally reliable. The COVID-19 pandemic was probably the biggest factor in exposing the limitations of JIT. With businesses and consumers finding new ways to operate and interact in the post-pandemic era, suppliers and transport networks that were used to JIT are unable to cope with the sudden demand. The digital transformation of supply chain management means that businesses can reduce risks and manage supply chains better. Another step in gaining efficiency is lean manufacturing, which applies continuous process improvements to eliminate waste from the entire enterprise. Total Quality Management (TQM) initiatives minimizes costs by maximizing quality. It focuses on continuous improvement in quality and has come to represent a focus on satisfying one’s customers. The focus on quality has shifted to Six Sigma, a disciplined, data-driven approach to

Copyright ©2023 Pearson Education, Ltd.

7


a continuous process improvement effort designed to reduce costs by improving quality and ensuring that internal processes are running as efficiently as possible. E.

Implications of Process Changes for the Study of Management Accounting To adapt to changes, the student must understand why techniques are being used, not just how they are used. Students should develop their understanding of underlying concepts and principles, not just memorize rules and techniques.

VII.

Ethical Conduct for Professional Accountants A.

{L. O. 7}

Standards of Ethical Conduct Ethical standards require CPAs and CMAs to adhere to codes of conduct regarding integrity, objectivity, professional competence and due care, confidentiality and professional behavior. See Exhibit 1-7 for the International Code of Ethics for Professional Accountants, a code of conduct developed by the International Ethics Standards Board for Accountants (IESBA) of the International Federation of Accountants (IFAC). Professional accounting organizations have procedures for reviewing alleged behavior that is not consistent with the standards. The standards are shown as responsibilities regarding competence, confidentiality, integrity, and objectivity. A code of conduct is a document specifying the ethical standards of an organization. It is the centerpiece of most ethics programs.

B. Ethical Dilemmas Ethical dilemmas exist when managers must choose an alternative and there are (1) significant value conflicts among differing interests, (2) several alternatives are justifiable, and (3) there are significant consequences for stakeholders in the situation. C.

Resolution of Ethical Conflicts When faced with ethical issues, you should follow your organization’s established policies on the resolution of such ethical conflict.

Copyright ©2023 Pearson Education, Ltd.

8


CHAPTER 1:

Quiz/Demonstration Exercises

Learning Objective 1 1.

The CMAs must pass an exam that includes _____. a. financial control b. financial decision making c. financial planning d. all of the above

2.

Which designation is the internal accountant’s counterpart to the CPA? a. CA b. CMA c. CFP d. none of the above

Learning Objective 2 3.

The major internal users of accounting information are _____. a. managers who use information for day-to-day operating decisions b. investors for investment decisions c. government authorities d. none of the above

4.

In the European Union external reporting must comply with _____. a. GAAP b. IFRS c. IRS d. FASB

Learning Objective 3 5.

Which of the following should be considered in the selection of an accounting system? a. b. c. d.

behavioral effects of the system on managers costs of buying and operating the system improved decision-making power resulting from the system all of the above

Learning Objective 4 6.

Concentrating on areas that need attention and ignoring areas that appear to be running smoothly is called _____. a. variance analysis b. management by exception

Copyright ©2023 Pearson Education, Ltd.

9


c. d.

management smoothing budget analysis

7.

Which of the following is not an example of a special report? a. cash flow report b. customer survey c. competitor analysis d. advertising impact analysis

8.

The various stages through which a product passes are called: a. planning cycle b. control period c. budget cycle d. product life cycle

Learning Objective 5 9.

The focus on customers occurs in which functions of the value chain? a. research and development b. production c. marketing d. distribution e. all of the above

10.

The value-chain is a set of business functions that _____. a. are of equal importance b. are non-value added functions c. are preferable functions d. are value-added functions e. are only related to distribution

11.

The functions of planning for control, evaluating and consulting, and governmental reporting are typically assumed within organizations by _____. a. the company treasurer b. the company controller c. the company vice president of marketing d. external auditors

12.

The treasurer function includes _____. a. tax administration b. evaluating and consulting c. investor relations d. economic appraisal

Learning Objective 6

Copyright ©2023 Pearson Education, Ltd.

10


13.

Trends that are causing changes in management accounting today include: a. advances in technology b. increased global competition c. a shift from a manufacturing to a service-based economy d. A and B e. A, B, and C

14.

A step in changes in business process management to gain efficiency by continuous process improvement to eliminate waste from the entire enterprise is called _____. a. just in time b. lean manufacturing c. waste management d. none of the above

Learning Objective 7 15.

Ethical obligations of management accountants are governed by the IMA Statement of Ethical Professional Practice, which outlines responsibilities regarding _____. a. incompetence, full disclosure of all information, moral decay, and partisanship b. assisting in maximizing profits regardless of the means necessary c. competence, confidentiality, integrity, and objectivity d. none of these

16.

In the IMA’s Statement of Ethical Professional Practice, which of the following is not an example of competence? a. provide decision support information and recommendations that are accurate b. avoid actual or apparent conflicts of interest c. maintain an appropriate level of professional expertise d. perform professional duties in accordance with relevant laws

Copyright ©2023 Pearson Education, Ltd.

11


CHAPTER 1: 1. 2. 3. 4. 3. 4. 5. 6. 7. 8. 9. 12. 13. 14. 15. 16.

Solutions to Quiz/Demonstration Exercises

[c] [d] [a] [b] [d] [b] [a] [d] [e] [d] [b] [b] [e] [b] [c] [b]

Copyright ©2023 Pearson Education, Ltd.

12


Chapter 2 Introduction to Cost Behavior and Cost-Volume Relationships LEARNING OBJECTIVES: When your students have finished studying this chapter, they should be able to: 1. 2. 3. 4. 5. 6. 7. 8. 9.

Explain how cost drivers affect cost behavior. Show how changes in cost-driver levels affect variable and fixed costs. Explain step- and mixed-cost behavior. Create a cost-volume-profit (CVP) graph and understand the assumptions behind it. Calculate break-even sales volume expressed in terms of total sales revenue earned and total units produced at break-even. Calculate the sales volume needed to reach a target profit in terms of units produced and in sales revenues earned. Differentiate between contribution margin and gross margin. Explain the effects of sales mix on profits (Appendix 2A). Compute cost-volume-profit (CVP) relationships on an after-tax basis (Appendix 2B).

Copyright ©2023, Pearson Education, Ltd

1


CHAPTER 2:

ASSIGNMENTS

CRITICAL THINKING EXERCISES 24. Marketing Function of Value-Chain and Cost Behavior 25. Production Function of Value-Chain and Cost Behavior 26. Tenneco Automotive's Value Chain EXERCISES 27. Identifying Cost Drivers 28. Basic Review Exercises 29. Variable- and Fixed-Cost Behavior 30. Variable- and Fixed-Cost Behavior 31. Basic Review Exercises 32. Basic Cost-Volume Graph 33. Basic Cost-Volume Graph 34. Basic Cost-Volume Graphs 35. Basic Cost-Volume Graphs 36. Hospital Costs and Pricing 37. Cost-Volume-Profit at a Hospital 38. Motel Rentals 39. Variable Cost to Break Even 40. Sales-Mix Analysis 41. Income Taxes 42. Income Taxes and Cost-Volume-Profit Analysis PROBLEMS 43. Joe’s Pub, Cost-Volume-Profit Analysis in a Small Business 44. Kroger Grocery Chain, Variable and Fixed Costs 45. Fixed Costs and Relevant Range 46. Comparing Contribution Margin Percentages 47. Movie Manager 48. Promotion of Rock Concert 49. Cost Reduction Program at Boeing 50. Basic Relationships, Restaurant 51. Changing Fixed Costs to Variable Costs at Blockbuster Video 52. CVP and Financial Statements for a Mega-Brand Company 53. Bingo and Leverage 54. Leverage at eBay 55. Adding a Product 56. Government Organization 57. Gross Margin and Contribution Margin 58. Choosing Equipment for Different Volumes 59. Sales Compensation, Variable/Fixed Costs, and Ethics 60. Sales-Mix Analysis 61. Hospital Patient Mix 62. Income Taxes on Hotels

Copyright ©2023, Pearson Education, Ltd

2


63. CASES 64. 65. 66. 67. 68.

Tax Effects, Multiple Choice

Hospital Costs CVP in a Modern Manufacturing Environment Multiproduct Break-Even in a Restaurant Effects of Changes in Costs, Including Tax Effects Operating Leverage

EXCEL APPLICATION EXERCISE 69. CVP and Break-Even COLLABORATIVE LEARNING EXERCISE 70. CVP for a Small Business INTERNET EXERCISE 71.

Cost Behavior at Southwest Airlines (http://www.southwest.com)

Copyright ©2023, Pearson Education, Ltd

3


CHAPTER 2: I.

OUTLINE

Identifying Activities, Resources, Costs, and Cost Drivers {L. O. 1} Cost Drivers - Output measures of resources and activities. EXHIBIT 2-1 shows the traditional and activity-based views of cost behavior. To apply the activity-based view, we must identify the resources used by each activity and the cost driver for each resource. Organizations can have many cost drivers. In this chapter, volume-based cost drivers are used to examine cost behavior. See EXHIBIT 2-2 for examples of costs and potential cost drivers for value-chain functions.

II.

Variable-Cost and Fixed-Cost Behavior

{L. O. 2}

The terms variable and fixed are used to describe costs based on how a cost behaves with respect to changes in a particular cost driver. Variable Cost - a cost that changes in direct proportion to changes in the cost driver level (i.e., costs per unit do not change, total costs do change). Examples include the costs of materials, merchandise, parts, supplies, commissions, and many types of labor. Fixed Cost - a cost that is not immediately affected by changes in the cost driver level (i.e., costs per unit do change, total costs do not change within the relevant range). Examples include real estate taxes, real estate insurance, many executive salaries, and space rentals. EXHIBIT 2-3 summarizes the relationship between fixed and variable costs. See EXHIBIT 2-4 for the relationship between the receiving activity and the costs of the fuel and equipment resources. See EXHIBIT 2-5 for the total cost lines for total fuel and equipment lease costs. IIII.

Cost Behavior: Further Considerations A.

Complicating Factors for Fixed and Variable Costs Relevant Range - the limits (i.e., time period and/or activity) of cost-driver activity within which a specific relationship between costs and the cost driver is valid. See EXHIBIT 2-6 for a graph of fixed cost behavior within a relevant range.

B.

Step- and Mixed-Cost Behavior Patterns

{L. O. 3}

Step Cost is a cost that changes abruptly at different intervals of activity because the resources and their costs come in indivisible chunks. See PANEL A of EXHIBIT 27 for an illustration of a decision where a step cost could be treated as a fixed cost. See PANEL B of EXHIBIT 2-7 for an illustration of a decision where a step cost could be treated as a variable cost. Mixed Cost is a cost that contains elements of both fixed- and variable-cost behavior.

Copyright ©2023, Pearson Education, Ltd

4


C.

Effect of Time Horizon and Magnitude on Cost Behavior Cost behavior often depends on the time frame affected by a decision and on the magnitude of the change in cost-driver activity. For long time spans or large changes in activity level, more costs behave as variable. For short time spans or small changes in activity level, more costs behave as fixed. The

IV.

Cost-Volume-Profit Analysis Cost-Volume-Profit (CVP) Analysis—the study of the effects of output volume on revenue (sales), expenses (costs), and net income (net profit). The major simplifying assumption is to classify costs as either variable or fixed with respect to the volume of output activity. A CVP scenario follows. A.

CVP Scenario

B.

Graphing the CVP Relationship

{L. O. 4}

The BEP can also be found by graphing the cost and revenue relationships. The process takes the following steps. Step 1:

Step 2:

Step 3: Step 4:

Step 5:

Draw the axes. The horizontal axis = sales volume, and the vertical axis = cost and revenue amounts expressed in terms of currency units Plot revenue. Select a convenient value at the upper end of the relevant range for sales volume. Then draw a line between the origin and the point. Plot the fixed costs. It should be a horizontal line intersecting the vertical axis at the level of fixed costs. Plot fixed costs plus variable costs. Determine the variable portion of cost at the volume of a convenient level of activity. Add this amount to fixed costs and plot the point. Then draw a line from the intersection of the vertical axis to this point. This line represents total costs, and the difference between the fixed cost line and this new line represents the variable costs. Locate the break-even point. The break-even point is where the total cost line crosses the sales revenue line. See EXHIBIT 2-8 for an illustration of a CVP graph. Break-Even Point is the level of sales at which revenue equals total cost and net income is zero.

Almost all break-even graphs show revenue and cost lines extending back to the vertical axis. This approach misleads because the relationships depicted are only valid

Copyright ©2023, Pearson Education, Ltd

5


within the relevant range. CVP analysis is based on a set of important assumptions, which include the following: 1. 2. 3. 4.

5.

C.

Costs can be classified into variable and fixed categories. Efficiency and productivity remain unchanged. The behavior of revenues and costs is accurate and is linear over the relevant range. Sales Mix (i.e., the relative proportions or combinations of quantities of products that constitute total sales) is constant. [See APPENDIX 2A for more on sales mixes.] The difference in inventory levels at the beginning and end of a period is insignificant.

Computing the Break-Even Point 1.

{L. O. 5}

General Equation Approach. The basic income statement equation used for CVP analysis is (unit sales price × number of units sold) − (unit variable cost × number of units sold) − fixed expenses = net income

2.

Contribution-Margin Method. Contribution Margin (CM) Per Unit - the sales price per unit minus the variable cost per unit. The BEP is reached when total contribution margin equals total fixed costs. Dividing total fixed costs by the CM per unit gives the BEP in number of units. To compute the sales revenue needed to break even using the equation technique, the variable costs must be expressed as a percentage of sales revenue, which is called the Variable-Cost Ratio or Percentage. Then, letting S = the sales revenue to break even, solve for S in the equation: S - (variable-cost ratio x S) - fixed expenses = 0 CM Percentage or Ratio - the portion of every sales dollar/euro/pound etc. that contributes to covering fixed costs and, hopefully, provides for profit (divide total contribution margin by total sales). Dividing total fixed costs by the CM ratio (total contribution margin / total sales) yields the sales revenues required to break even. The use of the CM ratio is necessary when a firm produces more than one product.

2.

Relationship Between the Two Techniques. Both approaches result in the following short-cut formulas: break-even volume (units) = (fixed cost)/(CM per unit) break-even volume (revenue) = (fixed cost)/(CM ratio)

Copyright ©2023, Pearson Education, Ltd

6


D.

E.

Effects of Changes in Fixed Expenses or Contribution Margin 1.

Changes in Fixed costs Increases (decreases) in fixed costs increase (decrease) the BEP.

2.

Changes in Unit Contribution Margin Increases (decreases) in the CM per unit decrease (increase) the BEP.

Target Net Profit and an Incremental Approach

{L. O. 6}

CVP analysis can be used to determine the target sales, in units and revenues, needed to earn a target profit. Using either the contribution margin or equation techniques results in the following shortcut equations. target sales volume in units = fixed cost + target net income CM per unit target sales volume in sales revenue= fixed cost + target net income CM ratio The Incremental Approach (i.e., the change in total results under a new condition in comparison with some given or known condition) can be used. Divide the target net income by the CM per unit and add the result to the unit BEP to get the target sales volume in units. Likewise, divide the target net income by the CM ratio and add the result to the BEP sales revenue to get the target sales volume in sales revenue. F.

Multiple Changes in Key Factors Multiple factor changes can be demonstrated by constructing income statements reflecting the changes and comparing before change and after change results. Also, an incremental approach can be used to isolate just the effects of the changes and eliminate irrelevant and potentially confusing data.

G.

Nonprofit Application Nonprofit organizations, such as government agencies, can use the principles of CVP analysis to determine how many individuals they can serve with limited budgets and to assess the impact of changes in the level of funding and/or costs on their ability to provide services. See EXHIBIT 2-9 for a graphical presentation of the nonprofit analysis.

H.

CVP Analysis and Computer-Based Spreadsheets Numerous combinations of fixed costs, selling prices, variable costs, and target income levels can be analyzed quickly using these computerized spreadsheets. See EXHIBIT 2-10 for an example of spreadsheet analysis.

Copyright ©2023, Pearson Education, Ltd

7


IV.

Additional Uses of CVP Analysis A.

Margin of Safety The margin of safety shows how far sales can fall below the planned level of sales before losses occur. It compares the level of planned sales with the break-even point: margin of safety = planned unit sales – break-even unit sales A small margin of safety generally indicates greater risk exposure than a larger margin of safety.

B.

Operating Leverage Operating Leverage - the firm’s ratio of fixed and variable costs. In highly leveraged firms, (i.e., those with high fixed costs and low variable costs) small changes in sales volume will result in large changes in net income. Less leveraged firms show smaller changes in net income with changes in sales volume. Above the BEP, net income increases faster for highly leveraged firms. However, below the BEP, losses mount more rapidly. See EXHIBIT 2-9 for a graph comparing high versus low operating leverage.

C.

Best Cost Structure Companies try to find their most desirable combination of fixed- and variable-cost factors. Some choose to increase their CM ratios and fixed costs by automating, while others may choose to lower their fixed costs and lower their CM ratios by putting their sales force on commissions rather than paying salaries. When the CM percentage of sales is low, large increases in volume are necessary before significant improvements in net profits are possible. As sales exceed the BEP, a high CM percentage increases profits faster than a low CM percentage.

VI.

Contribution Margin and Gross Margin

{L. O. 7}

Gross Margin (or Gross Profit) - the excess of sales over the Cost of Goods Sold (i.e., cost of the acquired or manufactured merchandise to be sold). Contribution Margin is the excess of sales over all variable costs. See EXHIBIT 2-12 which shows costs divided on two different dimensions: gross margin and contribution margin. V.

Appendix 2A: Sales-Mix Analysis

{L. O. 8}

Sales Mix - the relative proportions or combinations of quantities of products that comprise total sales. If the proportions of the mix change, the CVP relationships may also change.

Copyright ©2023, Pearson Education, Ltd

8


Generally, selling a higher (lower) proportion of high CM products than anticipated results in higher (lower) net income. Factors other than CM per unit of product (e.g., CM per unit of time) can be useful in making sales mix decisions (see Chapter 5 for further explanation). VI.

Appendix 2B: Impact of Income Taxes

{L. O. 9}

The target sales equation can be rewritten as target sales - variable expenses - fixed expenses = target after-tax income / (1 - tax rate) Letting N = the number of units of sales necessary to achieve the desired after-tax income and substituting values for the selling price per unit, variable expenses per unit, fixed expenses, target after-tax income, and the tax rate into the equation, N can be solved. Alternatively, the following shortcut formula may be used: change in net income = (change in volume in units) x (CM per unit) x (1 - tax rate) Each unit beyond the BEP adds to after-tax net profit at the unit CM multiplied by (1 - income tax rate). When incorporating income taxes in CVP analysis, the BEP does not change because the BEP is the point of zero profits. Therefore, there are no taxes on losses or zero profits.

Copyright ©2023, Pearson Education, Ltd

9


CHAPTER 2:

Quiz/Demonstration Exercises

Learning Objective 1 1.

Cost drivers _____. a. b. c. d.

2.

can be volume based affect the total level of costs incurred by companies are activities that cause costs to be incurred all of these

Production is one of the value-chain functions. Which one of the following is not an example of a cost driver for production costs? a. labor hours b. number of people supervised c. sales revenue d. machine hours

Learning Objective 2 3.

Which of the following will remain constant if the level of cost-driver activity increases within the relevant range? a. variable cost per unit b. total variable costs c. total fixed costs d. total costs e. A and C. f. B, C, and D

4.

The limits of cost-driver activity within which a specific relationship between costs and the cost driver is valid is called _____. a. variable range b. total range c. relevant range d. valid range

Learning Objective 3 Items 5 and 6 are based on the following data: Sustainable Games produces and sells the high quality pool tables produced in accordance with environmentally and socially sustainable materials and processes. The company expects the following sales and expenses in 2021 for its tables: Sales (1,000 tables @ $400 per table)

$ 400,000

Copyright ©2023, Pearson Education, Ltd

10


Variable expenses Fixed expenses

200,000 120,000

5.

How many tables must be sold for Sustainable Games to break even? a. 200 b. 400 c. 600 d. 800

6.

What is the breakeven level of sales revenue? a. $60,000 b. $120,000 c. $150,000 d. $240,000

Learning Objective 4 7.

Which of the following is not an assumption of cost-volume-profit analysis? a. The behavior of revenues and expenses is accurately portrayed and is linear over the relevant range. b. Costs can be classified into variable and fixed categories. c. Sales mix will be constant. d. Efficiency and productivity will both increase. e. The inventory level at the end of the period will be insignificantly different from that at the beginning.

8.

Increase in contribution margin per unit _____. a. increases break-even point b. decreases break-even point c. does not change break-even point d. means there is a change in fixed cost per unit

Learning Objective 5 Items 9 and 10 are based on the following data (ignore income taxes): Bamboozle Ltd. manufactures and sells widgets. A projected income statement for the expected sales volume of 100,000 widgets is as follows: Sales Variable expenses Contribution margin Fixed expenses Before-tax profit

£7,500,000 3,000,000 £4,500,000 2,500,000 £2,000,000

9.

How many widgets would need to be sold to earn a pre-tax profit of £2,900,000? a. 110,000 d. 140,000 b. 120,000 e. 100,000 c. 130,000 f. none of the above

10.

What amount of sales revenue would be required to achieve a pre-tax profit of £3,500,000? a. £7,000,000 b. £7,500,000 c. £9,500,000 d. £10,000,000

Copyright ©2023, Pearson Education, Ltd

11


e. some other amount Learning Objective 6 11.

The difference between sales and cost of goods sold is commonly called _____. a. contribution margin b. operating income c. gross margin d. excess sales

12.

If variable selling costs increase, then contribution margin (assuming all else constant) must _____. a. stay the same b. increase c. decrease d. need more information

Learning Objective 7 13.

TwinCo produces and sells two products. Product A sells for $8 and has variable expenses of $3. Product B sells for $18 and has variable expenses of $10. It predicts sales of 20,000 units of A and 10,000 units of B. Fixed expenses are $100,000 per month. Assume that TwinCo hits its sales goal for February of $600,000, and exceeds its expected before-tax profit of $70,000. What has happened? a. TwinCo sold 40,000 units of product A and no product B. b. TwinCo sold more of both products A and B than expected. c. TwinCo sold more of product A and less of product B than expected. d. TwinCo sold more of product B and less of product A than expected.

14.

Breakeven in units for a multi-product firm is calculated as fixed costs divided by _____. a. the sum of the contribution margin percentages for each product b. the weighted average contribution margin of all the products c. the sum of the individual product contribution margins d. it is not possible to calculate breakeven in units for a multi-product firm

Learning Objective 8 15.

Refer to the data provided for Bamboozle Ltd. in problems 9 and 10. Now assume that the company is subject to a 40% tax rate. How many widgets must it sell to achieve an aftertax income of £1,500,000? a. 111,111 b. 333,333 c. 66,666 d. 142,857

16.

After-tax profit equals before-tax profit _____. a. multiplied by the tax rate b. multiplied by 1 minus the tax rate

Copyright ©2023, Pearson Education, Ltd

12


c. d.

divided by the tax rate divided by 1 minus the tax rate

Copyright ©2023, Pearson Education, Ltd

13


CHAPTER 2: 1. [d] 2. [c] 3. [e] 4. [c] 5. [c]

6. [d]

7. [d] 8. [b] 9. [b]

10. [d]

11. [c] 12. [c] 13. [c] 14. [b] 15. [a]

Solutions to Quiz/Demonstration Exercises

The CM per unit must be computed. In this case, it is $200 ($400,000 $200,000)/1000 tables. Dividing the $120,000 fixed expenses by the $200 per unit CM gives 600 sets. Either multiplying the unit BEP by the unit selling price or by dividing the fixed expenses by the CM ratio. Using the first method, 600 tables multiplied by a price of $400 per table gives $240,000 of sales to break even. With the second method, $120,000 of fixed expenses divided by .50 ($200,000 CM/$400,000 Sales) also yields $240,000 to break even.

Add the before-tax desired profit to the fixed expenses and divide the result by the CM per unit. In this case, $2,900,000 + $2,500,000 = $5,400,000 / ($4,500,000 / 100,000 cases) gives 120,000 cases. Divide the sum of the target before-tax income and the fixed expenses by the CM percentage. In this case that is $6,000,000 [$3,500,000 + $2,500,000] divided by .60 [$4,500,000/$7,500,000] = $10,000,000.

The CM ratios for the two products are 62.5% for A and 44.4% for B. When the sales mix shifts to products with higher CM ratios, profits increase. To solve this problem it is necessary to convert the after-tax income desired to the pretax income necessary. Dividing £1,500,000 by .60 (1 - tax rate) gives £2,500,000 in pre-tax income required. Adding this to the £2,500,000 in fixed costs yields a required contribution margin of £5,000,000. Using the data provided for 100,000 widgets, the selling price per widget is £75.00 and the variable costs per widget are £30.00. This gives a CM per unit of £45.00, which can be divided into the £5,000,000 total contribution margin to give 111,111 widgets needed to sell to get £1,500,000 after-tax profit.

16.[b]

Copyright ©2023, Pearson Education, Ltd

14


Chapter 3 Measurement of Cost Behavior LEARNING OBJECTIVES: When your students have finished studying this chapter, they should be able to: 1. 2. 3. 4.

Explain management influences on cost behavior. Measure and mathematically express cost functions and use them to predict costs. Describe the importance of activity analysis for measuring cost functions. Measure cost behavior using the engineering analysis, account analysis, high-low, visual-fit, and least-squares regression methods.

Copyright ©2023 Pearson Education, Ltd

30


CHAPTER 3:

ASSIGNMENTS

CRITICAL THINKING EXERCISES 26. Mixed Costs and the Sales Force 27. Committed and Discretionary Fixed Costs in Manufacturing 28. Cost Functions and Decision Making 29. Statistical Analysis and Cost Functions EXERCISES 30 Step Costs 31. Mixed Costs 32. Various Cost-Behavior Patterns 33. Plotting Data 34. Cost Function for Expedia 35. Predicting Costs 36. Identifying Discretionary and Committed Fixed Costs 37. Cost Effects of Technology 38. Mixed Cost, Choosing Cost Drivers, High-Low and Visual-Fit Methods 39. Account Analysis 40. Linear Cost Functions 41. High-Low Method 42. Economic Plausibility of Regression Analysis Results PROBLEMS 43. Controlling Risk, Capacity Decisions, Technology Decisions 44. Step Costs 45. Government Service Cost Analysis 46. Cost Analysis at US Airways 47. Separation of Drug Testing Laboratory Mixed Costs into Variable and Fixed Components 48. School Cost Behavior 49. Activity Analysis 50. High-Low, Regression Analysis 51. Interpretation of Regression Analysis 52. Regression Analysis 53. Choice of Cost Driver 54. Use of Cost Functions for Pricing 55. Review of Chapters 2 and 3 CASES 56. 57. 58. 59.

Government Health Cost Behavior Activity Analysis Identifying Relevant Data Nike 10-K Problem: Step- and Mixed Cost Drivers

EXCEL APPLICATION EXERCISE Copyright ©2023 Pearson Education, Ltd

31


60.

Fixed and Variable Cost Data

COLLABORATIVE LEARNING EXERCISE 61. Cost-Behavior Examples INTERNET EXERCISE 62. Cost Behavior at Southwest Airlines (http://www.southwest.com)

Copyright ©2023 Pearson Education, Ltd

32


CHAPTER 3: I.

OUTLINE

Cost Drivers and Cost Behavior Accountants and managers assume that cost behavior is linear over some relevant range of activities or change in cost drivers. Linear-Cost Behavior - graphed with a straight line when a cost changes proportionately with changes in a single cost driver. Volume of a product produced or service provided is the primary driver for some costs. Other costs are more affected by activities not directly related to volume and often have multiple cost drivers. These costs are not easily identified with or traced to units of output. In practice, many organizations use a single cost driver to describe each cost even though many have multiple causes. Careful use of linear-cost behavior with a single cost driver often provides cost estimates that are accurate enough for most decisions, though each cost may have a different cost driver. The use of linear cost behavior may be justified on costbenefit grounds. See EXHIBIT 3-1 for a graph of linear-cost behavior, the relevant range, and an activity or resource cost driver level.

Copyright ©2023 Pearson Education, Ltd

33


II.

Management Influence on Cost Behavior

{L. O. 1}

Managers can influence cost behavior through their decisions about such factors as product or service attributes, capacity, technology, and policies to create incentives to control costs. A.

Product and Service Decisions and the Value Chain - product mix, design, performance, quality, features, distribution, and so on influence costs (i.e., the value chain).

B.

Capacity Decisions. Strategic decisions about the scale and scope of an organization's activities result in fixed levels of capacity costs. Capacity Costs are the fixed costs of being able to achieve a desired level of production or service. Companies, such as Volkswagen, must be careful in controlling the level of capacity costs when they have long-term variation in demand.

C.

Committed Fixed Costs – usually associated with investments in non-current assets such as property, plant, and equipment. These are typically large, indivisible amounts that the organization is obligated to incur or usually would not consider avoiding (e.g., mortgage or lease payments, interest payments on long-term debt, property taxes, insurance, and salaries of key personnel).

D.

Discretionary Fixed Costs - no obvious relationship to levels of output activity but are determined as part of the periodic planning process. Management decides that certain levels of these costs should be incurred to meet the organization's goals (e.g., advertising and promotion costs, public relations, research and development costs, charitable donations, employee training programs, and purchased management consulting services). Discretionary fixed costs can be easily altered but become fixed until the next planning period.

E.

Technology Decisions (e.g., labor-intensive versus robotic manufacturing or traditional banking services versus online banking) position a company to meet current goals and to respond to changes in the environment and affect the costs of products and services.

F.

Cost-Control Incentives - created by management to have employees control costs. Managers use their knowledge of cost behavior to set expectations, and employees may receive compensation or other rewards that are tied to meeting these expectations while maintaining quality and service.

Copyright ©2023 Pearson Education, Ltd

34


III.

Cost Functions

{L. O. 2}

Cost Measurement (or measuring cost behavior) - the first step in estimating or predicting costs as a function of appropriate cost drivers. The second step is to use these cost measures to estimate future costs at expected levels of the cost-driver activity. Measuring costs without obvious links to cost drivers presents some difficulty. Assumed relationships between costs and cost drivers are often used. A.

Form of Cost Functions. Cost Function - Algebraic equations that describe the relationship between a cost and its cost driver(s). A typical cost function equation is: Y = F + VX

where:

Y = Total cost F = Fixed cost V = Variable cost X = Cost-driver activity

When this mixed cost function is graphed, F is the intercept of the vertical axis and V is the slope of the cost function. Sometimes two or more cost drivers are used. B.

Developing Cost Functions

C.

Two principles should be applied to obtain accurate and useful cost functions: plausibility (i.e., believable) and reliability (conformity between a cost function’s estimate of costs at actual levels of activity and actually observed costs). Choice of Cost Drivers: Activity Analysis {L. O. 3} Activity Analysis - identifies appropriate cost drivers and their effects on the costs of making a product or providing a service. The final product or service may have a number of cost drivers because a number of separate activities may be involved. Cost Prediction - applies cost measures to expected future activity levels to forecast future costs. Activity analysis is especially important for measuring and predicting costs for which cost drivers are not obvious. For many years, most organizations used only one cost driver: the amount of direct labor. However, previously "hidden" activities greatly influence cost behavior. Activities related to the complexity of performing tasks affect costs more directly than labor usage or other volume-related activity measures.

IV.

Methods of Measuring Cost Functions A.

{L. O. 4}

Engineering Analysis - measures cost behavior according to what costs should be, not by what costs have been. It entails a systematic review of materials, supplies, labor, support services, and facilities needed for products and services. This can be used for existing products or for new products similar to what has been produced before. Disadvantages are that it is extremely costly and not timely. Copyright ©2023 Pearson Education, Ltd

35


B.

Account Analysis - selects a volume-related cost driver and classifies each account from the accounting records as a variable or fixed cost. The cost analyst then looks at each cost account balance and estimates either the variable cost per unit of cost driver activity or the periodic fixed cost.

C.

High-Low, Visual Fit, and Least-Squares Methods These methods rely on the use of past cost data to predict costs. These methods may not be particularly useful in predicting costs for changing organizations. If these methods are used, the cost analyst must be careful that the historical data that is from a past environment is not obsolete. Historical data may hide past inefficiencies that could be reduced if they are identified. 1.

High-Low Method - makes use of the costs and activity levels for the high and low activity levels in a set of data (unless one of these levels is viewed as an outlier). The difference in costs for the two activity levels is divided by the difference in activity levels to determine the variable cost per unit of activity. Then, either the high activity level and the cost at that activity level or the low activity level and the cost at that activity are used to solve for the fixed cost. Due to its reliance on just two data points, this method is rarely used in practice (see EXHIBIT 3-3).

2.

Visual-Fit Method - more reliable than the high-low method because all the available data are used. In the visual fit method, the cost analyst visually fits a straight line through a plot of all of the available data. The line is extended back until it intersects the vertical axis of the graph. The analyst measures where the line intersects the cost axis to estimate the fixed cost. An activity level is selected and the total mixed cost at that activity level is used to determine the variable cost per unit of activity by subtracting fixed cost from the total mixed cost amount and dividing by the activity level. The subjectivity in placing the line and in estimating the fixed and variable costs are disadvantages of this method and is rarely used in practice (see EXHIBIT 3-).

3.

Least-Squares Regression Method – (or simply regression analysis) uses statistics to fit a cost function to all the data. Using one cost driver requires simple regression, while using more than one cost driver requires the use of multiple regression. Regression analysis usually measures cost behavior more reliably than other cost measurement methods. In addition, regression analysis yields important statistical information about the reliability of cost estimates so analysts can assess confidence in the cost measures and select the best cost driver. Coefficient of Determination (R2) - measures how much of the fluctuation of a cost is explained by changes in the cost driver (see EXHIBIT 3-5).

Copyright ©2023 Pearson Education, Ltd

36


V.

Appendix 3: Use and Interpretation of Least-Squares Regression This appendix provides an example of the use of computer spreadsheet regression commands to perform simple linear regression. Plotting the data and the possible elimination of outliers are discussed. A data set with two possible cost drivers is presented (see EXHIBIT 3-6) and the output from the regression commands is presented. The R2 values of two regression equations are compared to see which regression equation best fits the data. Caution is required because the assumptions of regression analysis should be examined to ensure that the data comply so that useful results can be obtained. Regression Assumptions – (1) Linearity within the relevant range; (2) Constant variance of residuals; (3) Independence of residuals; and (4) Normality of residuals.

Copyright ©2023 Pearson Education, Ltd

37


CHAPTER 3:

Quiz/Demonstration Exercises

Learning Objective 1 1.

Managers influence cost behavior through their _____. a. technology decisions b. product and service decisions c. capacity decisions d. all of these e. only A and B

2.

One of the following costs is an example of a discretionary fixed cost. a. investment in production equipment b. investment in the factory c. electricity costs d. research and development costs e. raw material purchases

Learning Objective 2 3.

In the cost function equation Y = F + VX, V represents the _____. a. total cost at the X level of activity b. fixed cost at the Y level of activity c. variable cost per unit of activity X d. variable cost at the F level of activity

4.

In the cost function equation Y = F + VX, F represents the _____. a. slope b. intercept c. dependent variable d. independent variable

Learning Objective 3 5.

Which of the following organizations may use activity analysis? a. Robert Bosch GmbH b. Booking.com c. PwC (an international auditing, tax, and consulting firm) d. only A and B e. only C f. A, B, and C

6.

Activity analysis identifies cost drivers for cost functions. The cost functions should _____. a. predict costs b. be plausible c. be reliable Copyright ©2023 Pearson Education, Ltd

38


d. e. f. g.

have benefits that outweigh the costs A, B, C, and D only A and D only A, C, and D

Learning Objective 4 7.

A school cafeteria incurred the following costs for September 20x7: Monthly Cost September 20x7 Amount Manager's salary Hourly workers' wages and benefits Food Equipment depreciation Supplies Total cafeteria costs

$ 3,000 18,000 15,000 2,000 3,000 $41,000

The cafeteria served 12,500 meals to students and staff during the month. Using an account analysis to classify costs, the cost function for the cafeteria is _____ per meal. a. c.

$5,000 + $2.88 b.$8,000 + $1.62 $33,000 + $1.60 d.$20,000 + $3.52

Use the following information for questions 10 through 12. The restaurant and bar at Hirsch Hotel has recorded the following costs and number of meals served from January through September of 20x7: Month

Cafeteria Costs

January February March April May June July August September

€31.800 34.400 36.900 38.300 38.600 34.700 36.300 43.000 41.200

Meals Served 8.800 9.000 9.800 10.200 10.500 9.200 9.700 12.000 11.500

8.

Using the high-low method of cost estimation, the variable cost per meal served is _____. a. €2,666 b. €2,76 c. €3,50 d. €3,54

9.

The estimate of the fixed costs of running the cafeteria department using the high-low method is _____. a. €630 b. €1,000 c. €10,733 d. €10,928 Copyright ©2023 Pearson Education, Ltd

39


10.

The cost function derived using the high-low method, which can be used to estimate the costs of running the cafeteria is _____. a. €630 + €3,54X b. €10,733 + €2,666X c. €1,000 + €3,50X d. €10,928 + €2,76X e. €10,733 + €2,76

11.

Hampton Plc. used regression analysis to predict the annual cost of indirect materials. The results were as follows: Regression Output: Constant Std Err of Y Est R Squared Number of Observations X Coefficient(s) Std Err of Coef.

a. b. c. d.

£21,890 4,560 0.7832 22 11.75 2.1876

What is the linear cost function? Y = £20,100 + £4.60X Y = £21300 + £2.1876X Y = £21,890 +11.75X Y = £ 4,560 + £5.15X

Copyright ©2023 Pearson Education, Ltd

40


CHAPTER 3:

Solutions to Quiz/Demonstration Exercises

1. [d] 2. [d] 3. [c] 4. [b] 5. [f] 6. [e] 7. [a] In order to answer this problem, the costs must first be classified as fixed or variable in relation to the cost driver. In this case, the supervisor's salary ($3,000 per month) and the equipment depreciation ($2,000 per month) are fixed, while the remaining costs ($36,000) vary with the number of meals served. Dividing the variable costs by the number of meals served gives $2.88 per meal, and the department's cost function is $5,000 + $2.88 per meal. 8. [c] The data for January and August are used since 8,800 is the low level of activity for the data set and 12,000 is the high level of meals served. Subtracting the cost for the low activity level from the cost at the high activity level gives $11,200 ($43,000 - $31,800). Dividing this by the difference in the activity levels of 3,200 meals (12,000 - 8,800) gives a rate of $3.50 per meal. 9. [b] Using the variable cost per meal found above of $3.50 and the low level of activity of 8,800 meals, the variable cost at that activity level is $30,800. Since the total cost of operating the cafeteria when 8,800 meals were served is $31,800, $1,000 is the estimate of fixed costs ($31,800 - $30,800). The variable cost per meal could also be multiplied by the highactivity level 12,000 to get $42,000 which can be deducted from the total cost at that level of $43,000 to also arrive at the estimate of fixed costs of $1,000. 10. [c] This answer is constructed using the elements found in answering questions 10 and 11. 11. [c]

Copyright ©2023 Pearson Education, Ltd

41


Chapter 4 Cost Management Systems and Activity-Based Costing LEARNING OBJECTIVES: When your students have finished studying this chapter, they should be able to: 1. 2. 3. 4. 5. 6. 7. 8. 9.

Describe the purposes of cost management systems. Explain the relationship among cost, cost object, cost accumulation, and cost assignment. Distinguish between direct and indirect costs. Explain the major reasons for allocating costs. Identify the main types of manufacturing costs: direct materials, direct labor, and indirect production costs. Explain how the financial statements of merchandisers and manufacturers differ because of the types of goods they sell. Understand the main differences between traditional and activity-based costing (ABC) systems and why ABC systems provide value to managers. Use activity-based management (ABM) to make strategic and operational control decisions. Describe the steps in designing an activity-based costing system (APPENDIX 4).

Copyright ©2023 Pearson Education, Ltd.

43


CHAPTER 4:

ASSIGNMENTS

CRITICAL THINKING EXERCISES 31. Marketing and Capacity Planning 32. ABC and ABM Compared 33. ABC for Product Costing and Operational Control 34. ABC and Cost Management Systems 35. ABC and Benchmarking EXERCISES 36. Classification of Manufacturing Costs 37. Confirm Your Understanding of the Classification of Manufacturing Costs 38. Variable Costs and Fixed Costs; Manufacturing and Other Costs 39. Direct, Indirect, and Unallocated Costs 40. Cost Allocation in ABC 41. Activity-Based Costing 42. Two-Stage Activity-Based Costing—Stage One 43. Two-Stage Activity-Based Costing, Banking, Benchmarking 44. Direct, Indirect, and Unallocated Costs PROBLEMS 45. Cost Accumulation and Allocation 46. Hospital Allocation Base 47. Traditional and ABC Cost Accounting, Activity-Based Management 48. Activity-Based Costing and Product Line Profitability 49. Activity-Based Costing and Activity-Based Management, Automotive Supplier 50. Research in Activity-Based Costing or Activity-Based Management 51. Review of Chapters 2, 3, and 4 52. Review of Chapters 2, 3, and 4 53. Review of Chapters 2, 3, and 4 CASES 54. 55. 56. 57. 58.

Multiple Allocation Bases Traditional Versus ABC Systems ABC and Customer Profitability in Financial Services Identifying Activities, Resources, and Cost Drivers in Manufacturing Nike 10k Problem: Nike’s Cost Accounting System

EXCEL APPLICATION EXERCISE 59. Traditional Costing Versus Activity-Based Costing COLLABORATIVE LEARNING EXERCISE 60. Internet Research, ABC, and ABM INTERNET EXERCISE

Copyright ©2023 Pearson Education, Ltd.

44


61.

Vermont Teddy Bear Factory (http://www.vermontteddybear.com)

Copyright ©2023 Pearson Education, Ltd.

45


CHAPTER 4: I.

OUTLINE

Cost Management Systems

{L. O. 1}

A collection of tools and techniques that identifies how management’s decisions affect costs. The primary purposes are 1) aggregate measures of inventory value and cost of goods manufactured for external reporting to investors, creditors, and other external stakeholders; 2) cost information for strategic management decisions; and 3) cost information for operational control. A.

Cost Accounting Systems

{L. O. 2}

All kinds of organizations need some form of cost accounting, that part of the cost management system that measures costs for the purposes of management decisionmaking and financial reporting. The cost accounting system typically involves two processes: (1) Cost Accumulation—collecting costs by some “natural” classification such as materials or labor, and (2) Cost Allocation/Assignment—attaching costs to one or more cost objectives. (See EXHIBIT 4-1) B.

Cost Objectives Cost—a sacrifice or giving up of resources for a particular purpose. Cost Object (or Cost Objective)—anything for which managers want a separate measurement of costs (e.g., a product, a department, a sales region, a program, or something else for which decisions are made).

II.

Cost Terms Used for Strategic Decision Making and Operational Control Purposes {L. O. 3} A.

Direct Costs, Indirect Costs, and Cost Allocation

Direct Costs—identified specifically and exclusively with a given cost objective in an economically feasible way (tracing). Indirect Costs—not identified specifically and exclusively with a given cost objective in an economically feasible way. Cost allocation—to assign indirect costs to cost objects. A cost-allocation base is some measure of input or output that determines the amount of cost to be allocated to a particular cost object. Managers prefer to classify many costs as direct whenever it is “economically feasible” because it gives them greater confidence in their costs of products and services (i.e., less subjectivity). A particular cost can be direct for one cost objective but indirect for others. B.

Purposes of Cost Allocation

{L. O. 4}

Four purposes of cost allocation: (1) To predict the economic effects of strategic and

Copyright ©2023 Pearson Education, Ltd.

46


operational control decisions; (2) To provide desired motivation and to give feedback for performance evaluation; (3) To compute income and asset valuations for financial reporting; and (4) To justify costs or obtain reimbursement. Allocating fixed costs usually creates the greatest problems. C.

Methods of Cost Allocation Allocating indirect costs is a 5-step process: 1.

2. 3. 4. 5.

D.

Accumulate indirect costs for a period into one or more cost pools. A cost pool is a group of individual costs that a company allocates to cost objects using a single cost-allocation base. Select an allocation base for each cost pool, preferably a cost driver, that is, a measure that causes the costs in the cost pool. Measure the units of the cost-allocation base for each cost object (e.g., a product) and compute the total units for all cost objects (e.g., all products). Determine the percentage of total cost-allocation base units used for each cost object. Multiply the percentage in step 4 by the total costs in the cost pool to determine the cost allocated to each cost object.

Unallocated Costs Unallocated Costs are too difficult to establish a cause-and-effect relationship (see EXHIBIT 4-2). Unallocated costs are costs that an accounting system records but does not allocate to any cost object (e.g., research and development, process design, legal expenses, accounting, information services, and executive salaries). (See EXHIBIT 4-3.)

III.

Cost Terms Used for External Reporting Purposes

A.

{L. O. 5}

Categories of Manufacturing Costs Direct-Materials Costs—the acquisition costs of all materials that are physically identified as a part of the manufactured goods and that may be traced to the manufactured goods in an economically feasible way. Direct-Labor Costs—the wages of all labor that can be traced specifically and exclusively to the manufactured goods in an economically feasible way. Indirect Production Costs (Indirect Manufacturing Costs, Factory Overhead, Factory Burden, or Manufacturing Overhead)—include all costs other than direct material or direct labor that are associated with the manufacturing process (e.g., power, supplies, indirect labor, supervisory salaries, property taxes, rent, insurance, and depreciation).

Copyright ©2023 Pearson Education, Ltd.

47


B.

Product Costs and Period Costs Product Costs—costs (e.g., direct materials, direct labor, and factory overhead) initially identified with goods produced or purchased for resale (i.e., inventory) and become expenses (i.e., cost of goods sold) only when the inventory is sold. Period Costs—costs (e.g., selling and general administration expenses) that are deducted as expenses during the current period without going through the inventory stage. EXHIBIT 4-4 shows the flow of costs for both merchandising companies and manufacturing companies. Note that manufacturing companies have three categories of inventory whereas only one is present for merchandisers. Direct-Materials Inventory—materials on hand and awaiting use in the production process. Work-In-Process Inventory—goods undergoing the production process but not yet fully completed. Costs include appropriate amounts of the three major manufacturing costs (i.e., direct material, direct labor, and factory overhead). Finished-Goods Inventory—goods fully completed but not yet sold.

C.

Balance Sheet and Income Statement Presentation of Costs

{L. O. 6}

Typically, manufacturing and merchandising companies treat selling and administrative expenses in the same manner, but the detail of COGS differs. Manufacturers show the manufacturing cost of goods produced and then sold, which is composed of the three major cost categories: direct materials, direct labor, and factory overhead (see EXHIBIT 4-4). Merchandisers simply show the purchased cost of items, including freight, rather than the cost of goods manufactured and sold. IV.

Traditional and Activity-Based Cost Accounting Systems

{L. O. 7}

In the past, almost all companies used traditional costing systems—those that do not accumulate or report costs of activities or processes. This works well for fairly simple production and operating systems. The business environment has become more complex. This has led to the most significant improvement in cost accounting system design—activitybased costing (ABC). Many ABC teams find it useful to develop a process map—a schematic diagram with symbols that captures the interrelationships between cost objects, activities, and resources (see EXHIBIT 4-5). A.

An Illustration of Traditional (see EXHIBIT 4-6) and ABC Systems Activity-Based Accounting (ABA) or Activity-Based Costing (ABC) systems first

Copyright ©2023 Pearson Education, Ltd.

48


accumulate overhead costs for each of the activities of an organization, and then assign the costs of activities to the products, services, or other cost objects that caused that activity. Cost Drivers are identified for each activity to establish a cause-effect relationship between an activity and a cost object. Traditional systems often use only one cost driver and do not attempt to identify, accumulate, or report costs by activities performed. There are many variations in the design of ABC systems. In a two-stage ABC system (see EXHIBIT 4-7), there are two stages of allocation to get from the original resource cost to the final product or service cost. The first stage allocates costs to activity-cost pools. A cost pool is a group of individual costs that is allocated to cost objectives using a single cost driver. The second stage allocates activity costs to the products or services. The first-stage cost drivers are usually percentages. V.

Activity-Based Management: A Cost Management System Tool

{L. O. 8}

Activity-based management (ABM) is using the output of an activity-based cost accounting system to aid strategic decision-making and to improve operational control of an organization. One of the most useful applications of ABM is distinguishing between value-added cost (the necessary cost of an activity that a company cannot eliminate without affecting a product’s value to the customer), and nonvalue-added cost (unnecessary costs that a company tries to minimize and eliminate without affecting a product’s value to the customer). Examples of non-value-added costs are handling and storing inventories, and changing the setup of production-line operations to produce a different model of the product. Another ABC-related technique that has gained popularity is benchmarking (the continuous process of comparing products, services, and activities to the best industry standards. A.

Benefits of Activity-Based Costing and Activity-Based Management Many organizations in the manufacturing industry and service sector are adopting ABC systems because: 1.

2.

3. 4.

5.

Fierce competitive pressure has resulted in shrinking profit margins. Companies often do not have confidence in the accuracy of the margins for individual products or services. Greater diversity in the types of products and services, as well as customer classes, results in greater business operating complexity. The consumption of a company’s shared resources also varies substantially across products and customers. New production techniques have increased the proportion of indirect costs. The rapid pace of technological change has shortened product life cycles. Companies do not have time to make price or cost adjustments once they discover costing errors. The costs associated with bad decisions that result from inaccurate cost determinations are substantial (e.g., lost bids and hidden losses from

Copyright ©2023 Pearson Education, Ltd.

49


6.

VI.

undercosted products). Computer technology has reduced the costs of developing and operating ABC systems.

Appendix 4: Detailed Illustration of Traditional and Activity-Based Cost Accounting Systems The chapter presents an illustration of the use of activity-based costing to provide more accurate costing for the Billing Department of AT&T’s smaller customer care centers, with two major customer classes (i.e., account inquiry and bill-printing services). EXHIBIT 4-8 shows the traditional costing system that has been in use. All billing department costs are totaled and then divided by the total number of customer inquiries (residential + commercial) to arrive at a cost per inquiry. This amount is then multiplied by the number of inquiries from each type of customer and divided by the number of customers of each type to give the cost/customer. The amounts derived are $4.58 per residential customer and $6.88 per commercial customer. Not being satisfied with the accuracy of the amounts produced under their traditional costing system, AT&T developed an activity-based costing system. A.

Design of an Activity-Based Cost Accounting System 1. 2. 3. 4.

{L. O. 9}

Determine the key components (project scope, cost objectives, key activities, resources, and related cost drivers). Determine the relationships among cost objects, activities, and resources (see EXHIBIT 4-9). Collect relevant data concerning costs and the physical flow of cost-driver units among resources and activities (see EXHIBITS 4-10 and 4-11). Calculate/interpret the new activity-based cost information (see EXHIBITS 4-12 and 4-13).

The detailed application of these steps for AT&T is provided in the text. Of particular interest are the key results of the study—the actual determination of the activity-based costs. In the key results, the residential customer’s cost is $3.98 compared to $4.58 with the traditional costing system, and the commercial customer’s cost is $10.50 compared to $6.88 derived under the traditional system. This result is common when high-volume cost objects with simple processes are overcosted when only one volume-based cost driver is used. B.

Strategic Decisions, Operational Cost Control, and ABM The conclusion of the AT&T chapter problem is summarized.

Copyright ©2023 Pearson Education, Ltd.

50


CHAPTER 4:

Quiz/Demonstration Exercises

Learning Objective 1 1.

The primary purposes of a cost-management system do not include _____. a. operational control b. inventory valuation c. strategic management decisions d. cost allocation

2.

The cost accounting system typically involves two processes—cost allocation/assignment and _____. a. cost accumulation b. optimal cost mix c. cost estimation d. cost identification

Learning Objective 2 3.

The tracing or reassigning of costs to one or more cost objectives is referred to as _____. a. mixed-up costing b. historical costing c. cost allocation d. cost making

4.

A sacrifice or giving up of resources of particular purpose is called _____. a. cost objective b. allocation base c. objective d. cost

Learning Objective 3 Items 5 and 6 are based on the following data: Anton, Inc., incurred the following costs in 2010 in producing video games:

Machine operators Factory janitors Factory supervisor

5.

Wages $350,000 20,000 50,000

Computer chips Casings Labels Glue

Materials used $300,000 200,000 100,000 25,000

Anton’s 2010 product indirect labor was _____. a. $ 70,000 b. $ 20,000

Copyright ©2023 Pearson Education, Ltd.

51


c. d. 6.

$ 350,000 $ 370,000

Mario’s product direct materials costs for 2010 were _____. a. $ 625,000 b. $ 300,000 c. $ 600,000 d. $ 25,000

Learning Objective 6 7.

Which of the following can be inventory costs? a. raw materials b. indirect labor c. direct labor d. factory overhead e. A, B, C, and D f. only A, B, and C

8.

Reed Company had the following partial balance sheet for the year ended 20x7. Cash Accounts receivable Merchandise inventories Plant and equipment

$ 400,000 50,000 5,773,420 1,000,000

Reed is an example of a _____. a. service company b. merchandise company c. wholesale organization d. none of these Learning Objective 7 9.

The cost accounting system that uses a single cost pool for all indirect production costs is called _____. a. activity-based costing b. single level costing c. traditional costing d. all of the above

10.

A cost pool is a group of costs that is allocated to cost objectives using _____. a. only on cost driver b. the number of products made c. the number of products sold d. at least two cost drivers

Copyright ©2023 Pearson Education, Ltd.

52


e.

no more than five cost drivers

Learning Objective 9 11.

The fourth step in the design and implementation of an activity-based-costing system is _____. a. determine cost objective b. collect relevant data concerning costs c. calculate and interpret the new activity-based information d. develop a process-based map that represents the flow of activities

12.

Activity-based costing usually can _____. a. decrease the number of cost drivers b. decrease the amount of money spent on a system c. turn indirect costs into direct costs d. reduce the accuracy of the cost allocations

Copyright ©2023 Pearson Education, Ltd.

53


CHAPTER 4: 1. [d] 2. [a] 3. [c] 4. [d] 5. [a]

6. [c]

7. [e] 8. [b]

Solutions to Quiz/Demonstration Exercises

If the cost objective is the product, then direct labor costs are those traceable to the product. Under most accounting systems, only the machine operators costs would be traceable to the product and the janitors and supervisors wages would be classified as indirect. Whereas all of the materials costs would be direct to the plant, only the computer chip, casing, and label costs would be direct to the product. The glue would be an allocated cost to the product. Manufacturing companies typically have three categories of inventory whereas merchandisers have only one category listed in their balance sheets. The income statements also differ (e.g., a cost of goods manufactured and sold versus a cost of purchased goods sold).

9. [c] 10. [a] 11. [c] 12. [d]

Copyright ©2023 Pearson Education, Ltd.

54


Chapter 5 Relevant Information for Decision Making with a Focus on Pricing Decisions LEARNING OBJECTIVES: When your students have finished studying this chapter, they should be able to: 1. 2. 3. 4. 5. 6. 7. 8.

Discriminate between relevant and irrelevant information for making decisions. Apply the decision process to make business decisions. Construct absorption and contribution-margin income statements, and identify their relevance for decision making. Decide to accept or reject a special order using the contribution-margin approach. Explain why pricing decisions depend on the characteristics of the market. Identify the factors that influence pricing decisions in practice. Compute a sales price by various methods, and compare the advantages and disadvantages of these methods. Use target costing to decide whether to add a new product.

Copyright ©2023 Pearson Education, Ltd.

67


CHAPTER 5:

ASSIGNMENTS

CRITICAL THINKING EXERCISES 23. Fixed Costs and the Sales Function 24. Income Statements and Sales Managers 25. The Economics of the Pricing Decision 26. Pricing Decisions, Ethics, and the Law 27. Target Costing and the Value Chain EXERCISES 28. Pinpointing Relevant Costs 29. Information and Decisions 30. Identification of Relevant Costs 31. Straightforward Absorption Statement 32. Straightforward Contribution Income Statement 33. Straightforward Absorption and Contribution Statement 34. Absorption Statement 35. Contribution Income Statement 36. Special-Order Decision 37. Unit Costs and Total Costs 38. Advertising Expenditure and Nonprofit Organizations 39. Variety of Costs Terms 40. Acceptance of Low Bid 41. Pricing by Auto Dealer 42. Pricing to Maximize Contribution 43. Target Selling Prices 44. Competitive Bids 45. Target Costing 46. Target Costing PROBLEMS 47. Pricing, Ethics, and the Law 48. Analysis with Contribution Income Statement 49. Pricing and Contribution-Margin Technique 50. Cost Analysis and Pricing 51. Pricing of Education 52. Profit from Direct Selling vs. Royalty from Pay per View 53. Use of Passenger Jets 54. Effects of Volume on Operating Income 55. Pricing for a Guided Tour in Darjeeling Himalayan Railway 56. Pricing of a Special Order 57. Pricing and Confusing Variable and Fixed Costs 58. Demand Analysis 59. Target Costing 60. Target Costing and ABM 61. Target Costing Over Product Life Cycle

Copyright ©2023 Pearson Education, Ltd.

68


CASES 62. 63.

Use of Capacity Nike 10k Problem – Special Order

EXCEL APPLICATION EXERCISE 64. Determining Whether to Accept a Special Order COLLABORATIVE LEARNING EXERCISE 65. Understanding Pricing Decisions INTERNET EXERCISE 66. Marketing Decisions at Colgate-Palmolive (www.colgatepalmolive.com)

Copyright ©2023 Pearson Education, Ltd.

69


CHAPTER 5: I.

OUTLINE

The Concept of Relevance

{L. O. 1}

Whether information is relevant depends on the decision to be made. Accountants assist managers in making decisions by collecting and reporting relevant information. A.

What is Relevance? Relevant Information—the predicted future costs and revenues that will differ among alternatives. Although past data may be helpful in predicting future costs and revenues, past data is irrelevant in making future decisions. The authors provide two examples of the need for deciding relevant costs (i.e., which gas station to select to fill your tank and get the car lubricated, and the choice of materials for making a line of food containers). The expected future costs that differ between alternatives are the relevant costs to be used in selecting from the competing alternatives.

B.

A Decision Model

{L. O. 2}

See EXHIBIT 5-1 for an illustration of the decision process and the role of information in the process. Historical data from the accounting system is combined with other data from outside the accounting system to formulate predictions. The predictions are then input into a decision model (i.e., any method used for making a choice) in order for a decision to be made, implemented, and evaluated. C.

Accuracy and Relevance Accountants often are forced to choose between more relevance and more accuracy. Precise but irrelevant information is worthless for decision-making. Imprecise but relevant information can be quite useful. Qualitative (subjective) information can be as important, or more important, than quantitative (financial) information.

D.

The Relevance of Alternative Income Statements

{L. O. 3}

EXHIBIT 5-2 and EXHIBIT 5-3 provide schedules of factory overhead, and selling and administrative expenses for Cordell Company. E.

Absorption Approach (see EXHIBIT 5-4 for Absorption Income Statement). Absorption Costing—all factory overhead (both variable and fixed) is treated as product cost that becomes an expense in the form of manufacturing cost of goods sold only as sales occur. GAAP accounting requires the use of absorption costing. Gross Profit or Gross Margin is the difference between sales and fully absorbed cost of goods sold. Selling and Administrative expenses are treated as period expenses and

Copyright ©2023 Pearson Education, Ltd.

70


are deducted from gross profit to arrive at operating income. Costs are classified according to the three major functions (i.e., manufacturing, selling, and administrative) of management under absorption costing. F.

Contribution Approach (see EXHIBIT 5-5 for contribution income statement). Contribution Approach—all variable costs (both manufacturing and selling and administrative) are deducted from sales to result in Contribution Margin. Fixed expenses are deducted from the contribution margin to arrive at Operating Income. Costs are classified according to behavior (i.e., variable or fixed) under the contribution approach to income determination.

G.

Comparing Contribution and Absorption Approaches The contribution-margin approach separates costs based on the primary distinction of cost behavior pattern. It deducts variable costs from sales to compute a contribution margin and then deducts fixed costs to measure profit. The absorption approach separates costs based on the primary distinction of manufacturing versus nonmanufacturing costs to measure profit. It deducts manufacturing costs from sales to compute a gross margin and then deducts nonmanufacturing costs to measure profit.

II.

Pricing Special Sales Order A.

Illustrative Example See EXHIBIT 5-5 for the contribution income statement of the Cordell Company. They examine whether a special order offered at $26 per unit received near year-end from a mail-order house should be accepted. The order would (1) not affect regular business, (2) not raise antitrust issues regarding price discrimination, (3) not affect total fixed costs, (4) not require additional variable selling and administrative expenses, and (5) use some otherwise idle manufacturing capacity.

B.

Correct Analysis—Focus on Relevant Information and Cost Behavior {L. O. 4} See EXHIBIT 5-6 for the correct analysis using the contribution format income statement. The only relevant items are the increased revenues ($2,600,000) and increased costs ($2,400,000) associated with the special order. Although they may be included in the analysis, fixed costs that do not differ between alternatives are irrelevant.

C.

Incorrect Analysis—Misuse of Unit Cost Misinterpreting fixed unit costs may cause a manager to decide that a special order results in a bad decision due to a mistreatment of the fixed

Copyright ©2023 Pearson Education, Ltd.

71


manufacturing costs (i.e., treating them as variable). D.

Confusion of Variable and Fixed Costs The $6 of fixed manufacturing cost per unit included in the $30 per unit manufacturing cost used in the incorrect analysis creates the flaw in the analysis. Total fixed costs will remain at constant regardless of whether different amounts of units are produced as long as both levels of activity are within the relevant range (see EXHIBIT 5-6).

E.

Activity-Based Costing, Special Orders, and Relevant Costs Businesses that have identified all their significant cost drivers can predict the effects of special orders more accurately. The Cordell Company example is extended to indicate that, of the $24,000,000 of variable manufacturing costs expected to produce 1,000 units of product, $21,000,000 varies directly with units produced and $3,000,000 varies with the number of setups. The profitability of the special order for 100,000 units would now depend on the number of setups necessary for its production. Two levels are examined with one showing that the order would be profitable and the other showing that it would decrease profits.

III.

Basic Principles for Pricing Decisions

{L. O. 5}

In addition to pricing special orders, managers make the following pricing decisions: (1) Setting the price of a new or refined product; (2) Setting the price of products sold under private labels; (3) Responding to a new price of a competitor; and (4) Pricing bids in both sealed and open bidding situations. A.

The Concept of Pricing In perfect competition, all competing firms sell the same type of product at the same price. A firm can sell as much of a product as it can produce. The marginal cost (see EXHIBIT 5-7) is the additional cost resulting from producing and selling one additional unit. With a fixed set of production facilities, the marginal cost often decreases as production increases up to a point because of efficiencies created by larger volumes. Marginal revenue is the additional revenue resulting from the sale of an additional unit. In perfect competition, the marginal revenue curve is a horizontal line equal to the price per unit at all volumes of sales. As long as the marginal cost is less than the marginal revenue (price), additional production and sales are profitable. The profit-maximizing volume is the quantity at which marginal cost equals price. In imperfect competition, the price a firm charges for a unit will influence the quantity of units it sells (see EXHIBIT 5-8). At some point, the firm must reduce prices to generate additional sales.

Copyright ©2023 Pearson Education, Ltd.

72


To estimate marginal revenue, managers must predict the price elasticity – the effect of price changes on sales volume (see EXHIBIT 5-9). If small price increases cause large volume declines, demand is highly elastic. If prices have little or no effect on volume, demand is highly inelastic. Marginal cost is essentially the variable cost. Accountants assume that variable cost is constant within a relevant range of volume, whereas marginal cost may change with each unit produced. Within large ranges of production volume, however, changes in marginal cost are often small. B.

Pricing and Accounting Accountants seldom compute marginal revenue curves and marginal cost curves. Instead, they use estimates based on judgment to predict effects of additional production and sales on profits. In addition, they examine selected volumes, not the whole range of possible volumes.

IV.

General Influences on Pricing in Practice A.

{L. O. 6}

Legal Requirements Predatory Pricing (illegal)—a company establishes prices so low that competitors are driven out of the market so that the predatory price has no significant competition and the company can then raise prices dramatically. Pricing below average variable cost has been viewed by U.S. courts as predatory pricing. Discriminatory Pricing—charging different prices to different customers for the same product or service. It is illegal unless it reflects a differential incurred in providing the good or service. Both predatory pricing and discriminatory pricing charges can be defended by a company that cites its costs as a basis for its prices.

B.

Competitors’ Actions Competitors usually react to price changes of their rivals. Knowledge of their rival’s capacity, technology, and operating policies help managers predict competitors’ reactions to a company’s prices. Tinkering with prices is based on the price setter’s expectations of competitors’ reactions and of the overall effects on total industry demand for the good or service in question.

C.

Customer Demands If customers believe a price is too high, they may turn to other resources for the product or service, substitute a different product, or decide to produce the item themselves.

V.

Cost-Plus Pricing

Copyright ©2023 Pearson Education, Ltd.

73


A.

What is Cost-Plus Pricing? In some industries, such as agricultural commodities, costs have little or no effect on the setting of prices. In others, such as the automobile industry, managers use costs as a base in cost-plus pricing. The markup (i.e., the amount by which price exceeds cost) is originally set to provide a target return on investment, but must be flexible in order to meet market demands (e.g., defense contracting). Ultimately, the market sets prices. In the short run, the minimum price to be quoted, subject to consideration of longrun effects, should be equal to the costs that may be avoided by not landing the order— often all variable costs of producing, selling, and distributing the good or service. In the long run, the price must be set high enough to cover all costs, including fixed costs.

B.

Cost Bases for Cost-Plus Pricing

{L. O. 7}

Cost plus is often the basis for target prices. The size of the “plus” depends on target (desired) operating income. Target prices can be based on a host of different markups based on a host of different definitions of cost. These bases include variable manufacturing costs, total variable costs, full absorption manufacturing costs, and full costs. Thus, there are many ways to arrive at the same target price. See EXHIBIT 510 for an illustration. Note that full cost (or fully allocated cost) is the total of all manufacturing costs plus the total of all selling and administrative costs. Because managers’ performance evaluations and bonuses are frequently based on absorption costing income, markups based on full absorption costs are prevalent. C.

Advantages of Contribution Margin Approach in Cost-Plus Pricing Prices based on variable costs represent a contribution approach to pricing. Full absorption costing fails to highlight different cost behavior patterns. The contribution approach is sensitive to cost-volume-profit relationships which makes it easier for managers to prepare schedules at different volume levels. See EXHIBIT 5-11 for examples of using the contribution approach and full costing approach for analyzing the effects of volume changes on operating income. A normal or target-pricing formula can be developed as easily using variable costs as full absorption or full costs. The contribution approach offers insight into the shortrun versus long-run effects of cutting prices on special orders. The manager can consider whether the increase in operating income (contribution margin) generated from a special order outweighs potential reductions in long-run profitability due to expectations of lower prices by customers. If a company is using full absorption costing, a manager must conduct a special study in order to make the special order decision.

Copyright ©2023 Pearson Education, Ltd.

74


D.

Advantages of Absorption-Cost Approaches in Cost-Plus Pricing Absorption or full costs are far more widely used in practice than is the contribution approach. The following are some of the reasons offered: 1.

2.

3.

4.

5. 6. 7.

E.

In the long run, all costs must be covered to stay in business. Sooner or later fixed costs do indeed fluctuate as volume changes. Therefore, it is wise to assume that all costs are variable (even if some are fixed in the short run). Computing target prices based on cost-plus may indicate what competitors might charge, especially if they have approximately the same level of efficiency as you and also aim at recovering all costs in the long run. Absorption-cost formula pricing meets the cost-benefit test. It is too expensive to conduct individual cost-volume tests for the many products (sometimes thousands) that a company offers. There is much uncertainty about the shape of the demand curves and the correct price-output decisions. Absorption-cost or full-cost pricing copes with this uncertainty by not encouraging managers to take too much marginal business. Absorption-cost pricing tends to promote price stability and planning is more dependable. Absorption-cost pricing provides the most defensible basis for justifying prices to all interested parties, including government antitrust investigators. Absorption-cost pricing provides convenient reference (target) points to simplify hundreds of thousands of pricing decisions.

Using Multiple Approaches No single method of pricing is always the best. Most companies have gathered costs using some form of full-manufacturing-cost system because this is what is required for financial reporting. Managers are reluctant to focus just on variable costs when their bonuses are based on income shown in published financial statements that must use absorption costing.

F.

Formats for Pricing

See EXHIBIT 5-12 for an illustration of a quote sheet to be used in pricing. A minimum price based on variable costs and a maximum price based on what the company thinks it can obtain are shown. Construction and service industries (e.g., auto repair) compile separate categories of costs of (1) direct materials, parts, and supplies and (2) direct labor. Different markup rates are used for each category to assure the recovery of direct costs, overhead costs, and to provide for profits. For decision-making purposes, it may be more beneficial to pinpoint costs first, before adding markups, than to have a variety of markups already embedded in the “costs” used as guides for setting selling prices.

Copyright ©2023 Pearson Education, Ltd.

75


VI.

Target Costing

{L. O. 8}

The focus is on marketing and the revenue side of the profit equation. A.

Target Costing and New Product Development Target Costing—a tool for making cost a key focus throughout the life of a product (i.e., the desired profit margin is subtracted from the market price to determine the target cost). The emphasis is on proactive, up-front planning throughout every activity of the new product development process. It is most effective at reducing costs during the product design phase, when the vast majority of costs are committed. Market research guides the whole product development process by supplying information on customer-required product functions. There is a strong emphasis on understanding customer needs (see EXHIBIT 5-13). Value engineering is a cost-reduction technique, in primarily the design stage, which uses information about all value chain functions to satisfy customer needs while reducing costs. Kaizen costing is continuous improvement during manufacturing.

B.

Illustration of Target Costing Consider the target-costing system used by ITT Automotives to set their pricing approach for customers such as Mercedes-Benz.

C.

Target Costing and Cost-Plus Pricing Compared A comparison is shown in relation to an automotive part bid. As global competition has increased, companies are more limited in influencing market prices. Cost management is the key to profitability.

Copyright ©2023 Pearson Education, Ltd.

76


CHAPTER 5:

Quiz/Demonstration Exercises

Learning Objective 1 1.

In making managerial decisions, relevant information involves _____ costs that _____ between alternatives. a. future; differ b. future; do not differ c. past; do not differ d. past; differ

2.

For a revenue to be irrelevant to a particular decision, the revenue must _____. a. differ between the alternatives being considered b. be a future revenue c. be a past revenue d. A and B e. A and C f. only C

Learning Objective 2 3.

The role of historical data from the accounting system in making managerial decisions is _____. a. to serve directly as inputs in decision models b. to assist in making predictions that are inputs to a decision model c. to assist in making predictions about other information needed for making decisions d. none of the above

4.

What is the last portion of the decision process? a. Implementation and evaluation b. prediction method c. feedback d. decision model

Learning Objective 3 Items 5 and 6 are based on the following data: In 20x1, its first year of operations, Knot, Inc., manufactured 110,000 units of its single product, ties. Variable manufacturing costs were $6 per unit of product. Fixed manufacturing costs were $110,000 and are based on the production volume of 110,000 units. Knot sold 100,000 ties during the year at an average selling price of $10. Variable selling costs were 50¢ per tie and fixed selling and administrative costs were $80,000. 5.

Knot’s operating income using the absorption approach for 20x1 is _____.

Copyright ©2023 Pearson Education, Ltd.

77


a. b. c. d. 6.

$0 $ 300,000 $ 160,000 $ 170,000

Horn Gren’s operating income using the contribution approach for 20x1 is _____. a. $0 b. $ 160,000 c. $ 170,000 d. $ 300,000

Learning Objective 4 Use the following information for questions 7 and 8. Tire Ready Tire Co. sells tires. A partial income statement for a typical month is given below. Sales (10,000 tires) $ 100,000 Costs: Direct Materials $ 20,000 Direct Labor 16,000 Overhead (50% variable) 20,000 56,000 Gross Profit $ 44,000 A local car dealer has offered to buy 500 tires for an upcoming promotion to launch the new line of sports cars he will carry. Although the normal selling price is $10 per tire, the dealer has offered $8 each, citing the large volume of the order as the reason for cutting the price. There is no change in fixed costs. 7.

If Ready Tire Co. accepts this order, the effect on the company’s income, assuming regular sales are unaffected, is a _____. a. $1,000 decrease b. $2,200 increase c. $2,300 decrease d. $1,700 increase

8.

The fixed overhead of $2 per tire _____. a. is irrelevant in making the decision because the fixed costs per unit are unaffected b. is irrelevant in making the decision because the total fixed costs are unaffected c. will increase to above $1 per tire if the order is accepted d. will increase to above $1 per tire if the order is not accepted

Learning Objective 5 9.

In _____ competition, all competing firms will sell the same type of product at the same price. a. perfect b. fair

Copyright ©2023 Pearson Education, Ltd.

78


c. d. 10.

imperfect unfair

In perfect competition, the _____ curve is a horizontal line equal to the price per unit at all volumes of sales. a. marginal revenue b. step revenue c. incremental cost d. marginal cost

Learning Objective 6 11.

_____ influence pricing decisions. a. Costs b. Customer demands c. Competitors’ actions d. All of the above

12.

Which of the following legal requirements influences pricing in practice? a. competitive pricing b. predatory pricing c. nondiscriminatory pricing d. markup pricing

Learning Objective 7 13.

Popular markup formulas for pricing do not include a percentage of _____. a. variable manufacturing costs b. total variable costs c. fixed costs d. full costs

Learning Objective 8 14.

The majority of costs are committed in which stage of the value chain? a. research and development b. design c. production d. customer service

15.

A factor not usually included in determining the feasibility of earning the desired target profit margin is _____. a. interest rates b. competitor pricing c. inflation rates d. depreciation

Copyright ©2023 Pearson Education, Ltd.

79


CHAPTER 5: 1. [a] 2. [c] 3. [b] 4. [c] 5. [d]

Solutions to Quiz/Demonstration Exercises

Income under Absorption Approach Sales (100,000 * $10) COGS ((100,000 * ($6 + $1.00))

$1,000,000 (700,000)

Gross Profit 300,000 Selling and Administrative Expenses (100,000 * 50¢) + $80,000 130,000 Operating income $170,000 6. [b]

Income under Contribution Approach Sales (100,000 * $10) Variable costs: Manufacturing (100,000 * $6) Selling (100,000 * 50¢) Contribution Margin Fixed Expenses: Manufacturing Selling and Administrative

$1,000,000 $600,000 50,000

$350,000 $110,000 80,000

Operating Income 7. [d]

8. [b]

(650,000)

(190,000) $160,000

The solution to this problem requires determining the variable cost per tire and comparing those to the offer price. In this case, the variable costs are $20,000 for direct materials, $16,000 for direct labor, and $10,000 for variable overhead (1/2 of $20,000) for a total of $46,000. For 10,000 tires, this is a variable cost per tire of $4.60 ($46,000/10,000). Because the offer is for $8 per tire and variable cost is $4.60, then contribution margin would be $3.40 per tire. The profits would increase $1,700 (500 tires times $3.40). Total fixed costs should be used in comparing the alternatives. If the totals do not differ, fixed costs are irrelevant for the decision at hand.

9. [a] 10. [d] 11. [d] 12. [b] 13. [c] 14. [b] 15. [d]

Copyright ©2023 Pearson Education, Ltd.

80


Chapter 6 Relevant Information and Decision Making with a Focus on Operational Decisions LEARNING OBJECTIVES: When your students have finished studying this chapter, they should be able to: 1. 2. 3. 4. 5. 6. 7. 8.

Use a differential analysis to examine income effects across alternatives and show that an opportunity-cost analysis yields identical results. Decide whether to make or buy certain parts or products. Choose whether to add or delete a product line using relevant information. Compute the optimal product mix when production is constrained by a scarce resource. Decide whether to process a joint product beyond the split-off point. Decide whether to keep or replace equipment. Identify irrelevant and misspecified costs. Discuss how performance measures can affect decision making.

Copyright ©2023 Pearson Education, Ltd.

75


CHAPTER 6:

ASSIGNMENTS

CRITICAL THINKING EXERCISES 24. Measurement of Opportunity Cost 25. Outsourcing Decisions 26. Unitized Costs 27. Historical Costs and Inventory Decisions EXERCISES 28. Opportunity Costs 29. Opportunity Cost of Home Ownership 30. Opportunity Cost at Nantucket Nectars 31. Hospitality Opportunity Cost 32. Make or Buy 33. Make or Buy at Nantucket Nectars 34. Make or Buy and the Use of Idle Facilities at Nantucket Nectars 35. Profit per Unit of Space 36. Deletion of Product Line 37. Sell or Process Further 38. Joint Products, Multiple Choice 39. Obsolete Inventory 40. Replacement of Old Equipment 41. Unit Costs 42. Relevant Investment 43. Weak Division 44. Opportunity Cost PROBLEMS 45. Hotel Rooms and Opportunity Costs 46. Extension of Preceding Problem 47. Make or Buy 48. Relevant-Cost Analysis 49. Hotel Pricing and Use of Capacity 50. Special Air Fares 51. Choice of Products 52. Analysis of Unit Costs 53. Use of Available Facilities 54. Joint Costs and Incremental Analysis 55. Joint Products: Sell or Process Further 56. Relevant Cost 57. New Machine 58. Conceptual Approach 59. Book Value of Old Truck 60. Decision and Performance Models 61. Review of Relevant Costs 62. Make or Buy

Copyright ©2023 Pearson Education, Ltd.

76


63. 64. CASES 65. 66. 67. 68.

Make or Buy, Opportunity Costs, and Ethics Irrelevance of Past Costs at Starbucks

Make or Buy Make or Buy Make or Buy Nike 10k Problem: Make or Buy

EXCEL APPLICATION EXERCISE 69. Identifying Relevant Revenue, Costs, and Income Effects COLLABORATIVE LEARNING EXERCISE 70. Outsourcing INTERNET EXERCISE 71. Green Mountain Coffee Company (http://www.greenmountaincoffee.com)

Copyright ©2023 Pearson Education, Ltd.

77


CHAPTER 6: I.

OUTLINE

Analyzing Relevant Information: Focusing on Future and Differential Attributes O. 1} A.

{L.

Opportunity, Outlay, and Differential Costs and Analysis Opportunity Cost—maximum available contribution to profit forgone (i.e., rejected) by using limited resources for a particular purpose. Opportunity cost applies to a resource that a company already owns or that it has already committed to purchase. Outlay Cost—requires a future cash disbursement and is the typical cost recorded by accountants. Presenting revenues, costs, and income of two alternatives and the differences in those revenues, costs, and incomes is referred to as differential analysis. Differential Costs (Revenue) (or Incremental Costs)—the difference in costs (revenue) between two alternatives.

II.

Make-or-Buy Decisions A.

{L. O. 2}

Basic Make or Buy and Idle Facilities Manufacturers have to decide whether to make or buy parts and subassemblies that go into their final products. Some companies make all their own parts and subassemblies in order to assure product quality, whereas others buy most of theirs to protect mutually advantageous long-run relationships with suppliers. When idle facilities exist, quantitative factors bearing on the decision typically include the direct material, direct labor, variable overhead, fixed overhead that may be avoided if the part is purchased, and the purchase cost of buying the parts. A make-or-buy decision for Nantucket Nectars Company is presented. The key to make-or-buy decisions is identifying the additional costs for making (or the costs avoided by buying) a part or component. Using activity analysis can help in identifying these costs.

B.

Make or Buy and the Use of Facilities The alternative uses for the facilities that would be occupied if the part were made should be considered. The previous example is expanded to include the possible renting out of the space used to make the parts and the possibility of producing other products in that space. In all cases, companies should relate these decisions to the long-run policies for the use of capacity.

III.

Deletion or Addition of Products, Services, or Departments

Copyright ©2023 Pearson Education, Ltd.

{L. O. 3}

78


A.

Avoidable and Unavoidable Costs Avoidable Costs—will not continue if an ongoing operation is changed or deleted. These costs are relevant in making the decision. Unavoidable Costs—will continue even if an operation is halted. These are not relevant because they will not differ between alternatives. Unavoidable costs may include Common Costs—costs of facilities and services that are shared by users (e.g., building depreciation, heating, air conditioning, and general management expenses). The same principles regarding relevance applied to special orders apply to decisions concerning adding or deleting products or departments. The example provided in this section is whether to drop the grocery line from the offerings of a discount department store that has three major departments: groceries, general merchandise, and drugs. Fixed expenses are divided into two categories, avoidable and unavoidable. In the initial analysis, the grocery line is kept because it provides a contribution over its avoidable fixed costs of $50,000. After analyzing whether to keep or drop the grocery line, the analysis is extended to consider replacing groceries with expanded general merchandise. When this alternative is considered, however, the proper decision is to expand general merchandise because this option contributes $80,000 rather than $50,000 toward covering common and other unavoidable costs.

IV.

Optimal Use of Limited Resources: Product Mix Decisions

{L. O. 4}

If a plant that makes more than one product is being operated at capacity, the orders to accept are those that make the biggest total profit contribution per unit of the limiting factor. Limiting Factor or Scarce Resource—an item that restricts or constrains the production or sale of a product or service. For example, in retail sales, the limiting resource is often floor space. Thus, retail stores must either focus on products using less space or using the space for shorter periods of time (i.e., greater Inventory Turnover—number of times the average inventory is sold per year). Do not emphasize those products that give the largest contribution per sales dollar or per unit of product. See EXHIBIT 6-2 for the effect of turnover on profit. V.

Joint Product Costs: Sell or Process Further Decisions

{L. O. 5}

Joint Products—two manufactured products have relatively significant sales values, and are not separately identifiable as individual products until their split-off point (e.g., chemicals, lumber, flour, and the products of petroleum refining and meatpacking). Split-Off Point— time in manufacturing where the joint products become individually identifiable. Separable Costs—any costs beyond the split-off point due to not being part of the joint process and can be exclusively identified with individual products. Joint Costs—costs of manufacturing joint products before the split-off point. A.

Sell or Process Further

Copyright ©2023 Pearson Education, Ltd.

79


Decisions on whether to sell joint products at the split-off point or to process some or all products further are frequently made by managers. The essence of the decision whether or not to process further is to compare the difference between incremental revenues and costs with the opportunity cost of selling the product at the split-off point. See EXHIBIT 6-3 for an example of a sell or process further analysis. See EXHIBIT 6-4 for another presentation of the sell or process further for the entire firm. The joint costs are included in the analysis along with the revenues generated from the sale of the other joint product. These items are not differential revenues or costs, and thus they do not affect the decision. It is important to recognize that the joint costs do not play a role in determining whether or not to process a joint product further. VI.

Keeping or Replacing Equipment

{L. O. 6}

Depreciation—purchased equipment cost is spread over (or charged to) the future periods in which the equipment is expected to be used (i.e., a periodic cost). Book Value (or net book value)—the original cost less accumulated depreciation, which is the summation of depreciation charged to past periods. Sunk Cost (historical or past cost)—a cost that has already been incurred and is irrelevant in the decision-making process. In making equipment replacement decisions, the book value of old equipment is a sunk cost and should therefore not be considered. The only relevant costs in making this decision are the expected future costs (e.g., the disposal value of old equipment and the cost of new equipment). The gain or loss on disposal and the book value of the old equipment are irrelevant. However, tax consequences of these items should be included in a keep-or-replace decision. See EXHIBITS 6-5 and 6-6 for an analysis of a keep-or-replace decision. VII.

Identify Irrelevant or Misspecified Costs

{L. O. 7}

In addition to past costs, future costs (whether fixed or variable), that will be the same under all feasible solutions, are irrelevant in making managerial decisions. Salaries of top managers and obsolete inventories are given as examples of future costs that will not differ in most decisions and are, therefore, irrelevant for those decisions. There are pitfalls of using unit costs, not total costs, for decisions. The effect that the expected volume of activity may have on a decision to replace an existing piece of equipment. If the expected volume level is at the level used by an equipment sales representative in claiming that a cost reduction is possible, then no problem exists with the analysis. On the other hand, if the expected volume level is much lower than that used by the sales representative in making his claim, a different result may occur. VIII. Conflicts Between Decision Making and Performance Evaluation

{L. O. 8}

To motivate people to make optimal decisions, performance evaluation methods should be consistent with the decision analysis. The equipment replacement decision is an example of performance evaluation, based on annual income, resulting in a bad decision. Because income

Copyright ©2023 Pearson Education, Ltd.

80


in Year 1 would be less if the replacement were made, managers may choose to retain the existing equipment. The savings in Years 2, 3, and 4 are, therefore, not realized. Failing to replace the existing equipment also allows the manager to hide the potential “loss on disposal” as depreciation expense over the remaining useful life of the old equipment. Ideally, performance would be evaluated against predictions made when making decisions. However, with the complexity of modern organizations and the innumerable decisions being made, this is not possible.

Copyright ©2023 Pearson Education, Ltd.

81


CHAPTER 6:

Quiz/Demonstration Exercises

Learning Objective 1 1.

If you can work for the year and make $25,000, but you decide to go to college, then the $25,000 is a(n) _____. a. sunk cost b. outlay cost c. misplaced cost d. opportunity cost

2.

A cost that requires a cash disbursement sooner or later is referred to as a(n) _____ cost. a. opportunity b. outlay c. immediate d. differential

3.

A piece of equipment purchased last year is an example of a(n) _____. a. opportunity cost b. relevant cost c. sunk cost d. differential cost

Learning Objective 2 4.

Who Corporation produces a part that is used in the manufacture of one of its products. The costs associated with the production of 10,000 units of this part are as follows: Direct materials $45,000 Direct labor 65,000 Variable overhead 30,000 Fixed overhead 70,000 Total Cost $210,000 Of the fixed overhead costs, $30,000 is avoidable. When Company has offered to sell 10,000 units of the same part to Who Corporation for $18 per unit. Assuming there is no other use for the facilities, Who should _____. a. make the part, as this would save $3 per unit b. make the part, as this would save $1 per unit c. buy the part, as this would save the company $30,000 d. buy the part, as this would save the company $3 per unit

5.

Qualitative factor(s) that should be considered when evaluating a make-or-buy decision is (are) _____. a. the quality of the outside supplier’s product

Copyright ©2023 Pearson Education, Ltd.

82


b. c. d.

can the outside supplier provide the needed quantities can the outside supplier provide the product when it is needed all of the above

Learning Objective 3 Use the following information in answering questions 6 and 7. Buy Best is an electronic store having three operating departments. An income statement for the most recent month of operations appears below. Computers TVs

Radios Total

Sales $55,000 $44,000 $11,000 $110,000 Variable Costs 33,000 17,600 5,500 56,100 Contribution Margin 22,000 26,400 5,500 53,900 Fixed Costs Direct, avoidable (5,000) (4,000) (4,000) (13,000) Common, allocated based on sales dollars (10,000) (8,000) (2,000) (20,000) Profit (Loss) $ 7,000 $14,400 ($ 500) $ 20,900 6.

If Buy Best were to drop the toy line and make no other changes to its operations, income for the month would be _____. a. $ 20,400 b. $ 18,400 c. $ 19,400 d. $ 21,400

7.

The space currently being used by the radio department could be converted to a phone department. If this were done, sales of the phones are expected to be $22,000 with variable costs of $8,800 and avoidable direct fixed costs of $3,000. Assuming no effects on the computers and TV departments, income for the month would be _____. a. $ 28,200 b. $ 29,600 c. $ 28,700 d. some other amount

Learning Objective 4 8.

When a multiproduct plant is being operated at capacity, the products that should be emphasized are those that provide the highest contribution margin _____. a. ratio b. per sales dollar c. per unit of limited resource d. per unit of product

Copyright ©2023 Pearson Education, Ltd.

83


7.

Which of the following is not a scarce resource of a company or firm? a. customers b. floor space c. time d. laborers

Learning Objective 5 10.

PB Company drills for oil, and delivers it to refining companies. Currently it is selling its crude oil at $20/barrel. PB has been asked to refine the oil for commercial use. For a standard of refined oil, PB will be paid $30/barrel. PB estimates that the additional labor and refining cost involved in further processing of a barrel of oil is$6/barrel. PB Company should _____. a. continue selling crude oil at $20/barrel b. sell the super crude oil to another company for $21/gallon c. sell refined oil for an increased profit of $4/barrel d. do none of these

11.

A joint product should be processed beyond split-off if additional _____ from further processing exceeds _____. a. revenue; joint costs b. revenue; allocated joint costs c. revenue; additional costs of further processing d. revenue; allocated joint costs and additional costs of further processing

Learning Objective 6 12.

The book value of old equipment is irrelevant in replacement decisions because _____. a. it is a sunk cost b. it will be capitalized if the equipment is kept c. it represents a future cost that will differ between the options of replacing or keeping the equipment d. it represents a future cost that will not differ between the options of replacing or keeping the equipment.

13.

In analyzing whether to replace or keep existing equipment, the cost of the new equipment _____. a. is irrelevant because it is a historical cost b. is relevant because equipment is always relevant c. represents a future cost that will differ between the options of replacing or keeping the equipment d. both A and B

Learning Objective 7 14.

In general, a decision maker should be wary of _____. a. unit variable costs

Copyright ©2023 Pearson Education, Ltd.

84


b. c. d.

unit fixed costs unit sales price none of the above

Copyright ©2023 Pearson Education, Ltd.

85


CHAPTER 6: 1. [d] 2. [b] 3. [c] 4. [b]

Solutions to Quiz/Demonstration Exercises

The difference is $1 per unit ($10,000/10,000 units) in favor of making. It is easiest to see the difference in a total cost analysis: Make Buy Cost of purchasing Direct materials cost Direct labor cost Variable overhead Fixed overhead Total cost

5. [d] 6. [c] 7. [b]

$

45,000 65,000 30,000 70,000 $ 210,000

$180,000 0 0 0 40,000 $220,000

Income statements for answering 6 and 7 appear below. Income statements for answering 6 and 7 appear below.

Sales Variable costs Contribution margin Fixed costs: Direct Common Profit (loss)

Keep Toys $11,000 5,500 5,500

Drop Toys $99,000 50,600 48,400

Add Jewelry $121,000 59,400 61,600

(4,000) (9,000) (12,000) ( 2,000) (20,000) (20,000) $ (500) $19,400 $ 29,600

The reduction in the contribution margin exceeds the reduction in the direct fixed costs and the produce line should be kept if not considering adding the jewelry. For #7, the jewelry department provides an $8,700 higher segment margin than the toy line so it should replace produce. 8. [c] 9. [a] 10. [c]

The additional revenues generated of $10/barrel are greater than the further processing costs of $6/barrel.

11. [c] 12. [a] 13. [c] 14. [b]

Copyright ©2023 Pearson Education, Ltd.

86


Chapter 7 Introduction to Budgets and the Master Budget LEARNING OBJECTIVES: When your students have finished studying this chapter, they should be able to: 1. 2. 3. 4. 5. 6. 7. 8. 9.

Explain how budgets facilitate planning and coordination. Anticipate possible human relations problems caused by budgets. Explain potentially dysfunctional incentives in the budget process. Explain the difficulties of sales forecasting. Explain the major features and advantages of a master budget. Follow the principal steps in preparing a master budget. Prepare the operating budget and the supporting schedules. Prepare the financial budget. Use a spreadsheet to develop a budget (Appendix 7)

Copyright ©2023 Pearson Education, Ltd.

86


CHAPTER 7:

ASSIGNMENTS

CRITICAL THINKING EXERCISES 22. Budgets as Limitations on Spending 23. Sales Personnel and Budgeting 24. Master Budgets for Research and Development 25. Production Budgets and Performance Evaluation EXERCISES 26. Fill In the Blanks 27. Cash Budgeting 28. Purchases and Cost of Goods Sold 29. Purchases and Sales Budgets 30. Sales Budget 31. Sales Budget 32. Cash Collection Budget 33. Purchases Budget 34. Purchases Budget 35. Cash Budget PROBLEMS 36. Cash Budget 37. Cash Budget 38. Budget at Indian Hotel Company 39. Activity-Based Budgeting 40. Budgeting, Behavior, and Ethics 41. Spreadsheets and Sensitivity Analysis of Income Statement 42. Spreadsheets and Sensitivity Analysis of Operating Expenses CASES 43. 44. 45. 46.

Comprehensive Cash Budgeting (EXHIBITS 7-14, 7-15, 7-16) Cash Budgeting for a Hospital Comprehensive Budgeting for a University Nike 10k Problem: Budgeting Assumptions at Nike

EXCEL APPLICATION EXERCISE 47. Preparing a Cash Budget to Assist Long-Range Planning COLLABORATIVE LEARNING EXERCISE 48. Personal Budgeting INTERNET EXERCISE 49. Carnival Corporation (http://www.carnivalcorp.com)

Copyright ©2023 Pearson Education, Ltd.

87


CHAPTER 7: I.

OUTLINE

Budgets and the Organization

{L. O. 1}

Budget—a condensed business plan for the forthcoming year (or less). A budget is used in attracting funds from investors and banks and by managers to guide them in allocating resources, maintaining control, and measuring and rewarding progress. The most important functions of budgets are for planning, performance evaluation, and communication. Budgets provide a comprehensive financial overview of planned company operations. Budgets highlight potential problems and opportunities early, allowing managers to take steps to avoid the problems or use the opportunities wisely. Budgets are used for performance evaluation. Managers use budgets as a benchmark – a measure of expected or desired performance—against which they compare actual performance. Finally, budgets provide an important two-way communication channel. A.

Advantages of Budgets 1. 2. 3. 4.

It provides an opportunity for managers to reevaluate existing activities and evaluate possible new activities. It compels managers to think ahead by formalizing their responsibilities for planning. It aids managers in communicating objectives to units and coordinating actions across the organization. It provides benchmarks to evaluate subsequent performance.

Formalization of Planning Forces managers to think ahead—to anticipate and prepare for changing conditions. Planning is an explicit management responsibility. Managers will set goals and objectives and establish policies to aid in their achievement. The objectives are the destination points, and budgets are the road maps guiding us to those destinations. Evaluation of Activities Budgeting typically uses the current activities of the organization as a starting point for planning, but how managers use this starting point varies. At one extreme, in some organizations the budget process automatically assumes the activities for the new budget period will be the same as the activities for the previous period. At the other extreme, some organizations use a zero-base budget, which starts with the assumption that current activities will not automatically be continued. Communication and Coordination Budgets tell employees what is expected of them. A good budget process communicates both from the top down and from the bottom up. Top management makes clear the goals and objectives of the organization in its budgetary directives. Employees and lower-level managers then inform higher-level managers how they plan to achieve the goals and objectives. Budgets also help managers coordinate objectives. The budgetary process forces

Copyright ©2023 Pearson Education, Ltd.

88


managers to visualize the relationship of their department’s activities to those of other departments and the company as a whole. Performance Evaluation Budgeted goals and performance are generally a better basis for judging actual results than is past performance. The major drawback of using historical results for judging current performance is that inefficiencies may be concealed in the past performance. Changes in economic conditions, technology, personnel, competition, etc., also limit the usefulness of comparisons with the past. B.

Potential Problems in Implementing Budgets

{L. O. 2}

Budget Participation and Acceptance of the Budget In order to benefit an organization, budgets need the support of all the firm’s employees. Top management’s support of the budget can influence that of lower-level managers. Because budgets are often used to compare with actual results in evaluating subordinates, subordinates may regard them as straightjackets that are unduly restrictive. Accountants and higher-level managers need to show how budgets can help each manager and employee achieve better results. In addition, problems may arise if the employees and managers are rewarded on dimensions other than meeting budgets. Participative Budgeting—because the effectiveness of any budget depends on whether the affected managers and employees understand and accept the budget, some companies involve the affected personnel. C.

Incentives to Lie and Cheat

{L. O. 3}

Lying can arise if the budget process creates incentives for managers to bias the information that goes into their budgets. Managers may want to increase the resources allocated to their department (e.g., space, equipment, and personnel) in order to reach output targets and receive higher rewards. If organizations use budgets as a target for performance evaluations, managers may create budgetary slack or budget padding—overstate their budgeted costs or understate their budgeted revenues to create a budgeted profit level that is easier to achieve (see EXHIBIT 71). Budgetary slack helps buffer managers from budget cuts imposed by higher-level management and provides protection against cost increases or revenue shortfalls due to unforeseen events. Lying and cheating create cynicism about the budget process and a culture of unethical behavior in the organization. D.

Difficulties of Obtaining Accurate Sales Forecasts

{L. O. 4}

The sales budget is the foundation of the entire master budget. The accuracy of the estimated purchases budgets, production schedules, and costs depends on the detail and accuracy (in dollars, units, and mix) of the budgeted sales. Sales Forecast (i.e., a prediction of sales under a given set of conditions) is used to prepare the Sales Budget (i.e., the result of decisions to create the conditions that will generate a desired level of sales). A firm may have sales forecasted for various levels of advertising. Once a decision has been made regarding the level of advertising

Copyright ©2023 Pearson Education, Ltd.

89


expenditures, the sales budget is determined. Sales forecasts are usually prepared under the direction of the top sales executive. Important factors considered by a forecaster include: 1. 2. 3. 4. 5. 6. 7. 8.

Past patterns of sales Estimates made by the sales force General economic conditions Competitors’ actions Changes in the firm’s prices Changes in product mix Market research studies Advertising and sales promotion plans

Sales forecasting usually combines various techniques. In addition to the opinions of sales staff, statistical analysis of correlations between sales and economic indicators (prepared by economists and members of the market research staff) and opinions of line management provide valuable help. Ultimately, the sales budget is the responsibility of line management. III.

Types of Budgets

{L. O. 5}

The planning horizon for budgeting may vary from one day to many years. Strategic Plan—the most forward-looking budget, which sets the overall goals and objectives of the organization. Long-Range Planning—forecasted financial statements for 5- or 10-year periods. Long-range planning includes decisions about the addition or deletion of product lines, design and location of new plants, acquisition of buildings and equipment, and other long-term commitments. Capital Budgets—detail the planned expenditures for facilities, equipment, new products, and other long-term investments in coordination with long-range plans. A budget is a formal, quantitative expression of management plans. Master Budget (Pro Forma Statements)—summarizes the planned activities of all subunits of an organization (e.g., sales, production, distribution, and finance). It quantifies targets for sales, cost-driver activity, purchases, production, net income, and cash position, and any other objective that management specifies. It is a periodic business plan that includes a coordinated set of detailed operating schedules and financial statements. It includes forecasts of sales, expenses, cash receipts and disbursements, and balance sheets. Managers may also prepare daily or weekly task-oriented budgets that help them carry out their particular functions and meet operating and financial goals. Continuous Budgets (Rolling Budgets)—common form of master budgets that add a month in the future as the month just ended is dropped. This type of budget forces managers to think specifically about the forthcoming 12 months and thus maintain a stable-planning horizon. While a new month is added to a continuous budget, the other 11 months can also be updated.

Copyright ©2023 Pearson Education, Ltd.

90


A.

Components of a Master Budget The usual master budget for a nonmanufacturing company has the following components: 1. a. b. c. d. e. 2.

Operating Budget (profit plan) Sales budget (and other cost-driver budgets as necessary) Purchases budget Cost of goods sold budget Operating expenses budget Budgeted income statement

Financial budget a. Capital budget b. Cash budget c. Budgeted balance sheet

See EXHIBIT 7-2 for a condensed diagram of the relationships among the various parts of the master budget for a nonmanufacturing company. Manufacturing companies must prepare ending inventory budgets and budgets for labor, materials, and factory overhead in addition to the budgets indicated for nonmanufacturing organizations. The two major parts of the master budget are the operating budget and the financial budget. Operating Budget—focuses on the income statement and its supporting schedules. It is sometimes called a Profit Plan, although it may show a budgeted loss or may be used to simply budget expenses in an agency with no revenues. Financial Budget—focuses on the effects that the operating budget and other plans (e.g., capital budgets and repayments of debt) will have on cash. IV.

Preparing the Master Budget A.

The Cooking Hut Background information on the company used to illustrate the preparation of the master budget, Cooking Hut Company (CHC), is provided. The actual March Sales of $40,000 and expected sales for the next five months are provided. See EXHIBIT 7-3 for a balance sheet at the beginning of the budget period. The details regarding the composition of sales as to whether they are cash or credit is given, as is the collection pattern for the credit sales. The company’s inventory policy and the payment pattern for purchases, wages and commissions and various operating expenses are supplied. In addition, the desire of the company to maintain a minimum cash balance at the end of each month is expressed. Borrowings and repayments of loans are said to be in multiples of $1,000. Loans are made at the beginning of a month and repayments occur at the end of a month with interest computed using a 12% annual rate.

Copyright ©2023 Pearson Education, Ltd.

91


B.

Steps in Preparing the Master Budget

{L. O. 6}

The principal steps in preparing the master budget are a. Supporting Budgets and Schedules 1. Using the data given, prepare the following budgets and schedules for each of the months of the planning horizon: Schedule a. Sales budget Schedule b. Cash collections from customers Schedule c. Purchases and cost-of-goods-sold budget Schedule d. Cash disbursements for purchases Schedule e. Operating expense budget Schedule f. Cash disbursements for operating expenses b. Operating Budget 2. Using the supporting budgets and schedules, prepare a budgeted income statement for the 3 months ending June 30, 20X1. c. Financial Budget 3. Prepare the following budgets and forecasted financial statements: a. Capital budget b. Cash budget, including details of borrowings, repayments, and interest for each month of the planning horizon c. Budgeted balance sheet as of June 30, 20X1 1.

Step 1: Preparing Basic Data {L. O. 7} Using the data given in the description of the problem, the following schedules are prepared: a. Sales Budget (Schedule a) This schedule shows, by month, the expected credit, cash, and total sales. Total sales for the budget period are shown on the budgeted income statement (EXHIBIT 74). b. Cash Collections from Customers (Schedule b) This schedule indicates the sources for the cash collections as the current month’s cash sales and the collection of the prior month’s credit sales. Total cash collections for each month are the sum of these two amounts. The total of these disbursements is used in helping to construct the cash budget (EXHIBIT 7-5). c. Purchases (Schedule c) The budgeted purchases are found using the following equation:

Budgeted purchases

=

desired ending + cost of goods sold – inventory

beginning inventory

The cost of goods sold is derived from the sales budget by multiplying an appropriate percentage by the budgeted sales. The total amount of this expense appears in the budgeted income

Copyright ©2023 Pearson Education, Ltd.

92


statement (EXHIBIT 7-4). d. Disbursements for Purchases (Schedule d) The monthly payment for purchases is based on the dollar purchases derived in Schedule c and the company’s payment pattern for its merchandise purchases. CHC pays for half of its purchases in the month of purchase and the other half in the month following purchase. Therefore, payments for purchases include half of the current month’s purchases and half of the prior month’s purchases. The total of these disbursements is used in helping to construct the cash budget (EXHIBIT 7-4). e. Operating Expense Budget (Schedule e) This schedule details the amounts of wages, commissions, miscellaneous, rent, insurance and depreciation expenses for each month. Some of these vary with activity, whereas others are usually fixed each period. Totals for the budget period for these items are included on the budgeted income statement (EXHIBIT 7-4) f. Disbursements for Operating Expenses (Schedule f) The payments for these expenses include half of last month’s wages and commissions, half of the current month’s wages and commissions, and the current period’s rent and miscellaneous expenses. The total of these disbursements is used in helping to construct the cash budget (EXHIBIT 7-5). 2.

Step 2: Preparing the Operating Budget Amounts from the sales, purchases, and operating expenses schedules are used in helping to construct the budgeted income statement (EXHIBIT 7-4). Interest expense, determined in the cash budget (EXHIBIT 7-5), also is needed to complete the budgeted income statement.

3.

Step 3: Preparation of Financial Budget

{L. O. 8}

The second part of the master budget is the financial budget, which consists of the capital budget, cash budget, and ending balance sheet. This chapter focuses on the cash budget and the budgeted balance sheet. a.

The capital budget is discussed in Chapter 11.

b.

Cash Budget (EXHIBIT 7-5) The cash budget is constructed using three major sections. First, expected cash receipts (Schedule b) are added to the beginning cash balance to get the total cash available before financing. Then, cash disbursements for purchases (Schedule d), operating expenses

Copyright ©2023 Pearson Education, Ltd.

93


(Schedule f), and capital acquisitions or other cash expenses are added to the minimum cash balance desired to get the total cash needed. An excess or deficiency of cash is computed as the difference between the cash available and the total cash needed in order to determine whether the company needs to borrow funds or if prior borrowings and interest may be paid off. The last section of the cash budget details the financing activity and indicates the ending cash balance. c.

Budgeted Balance Sheet (EXHIBIT 7-6) The final step in preparing the master budget is to construct the budgeted balance sheet that projects each balance sheet item in accordance with the business plan expressed in the previous schedules. The first draft is rarely the final draft. As it is reworked, the budgeting process becomes an integral part of the management process itself—budgeting is planning and communicating.

C.

Activity-Based Master Budgets Functional Budgets—budgeting process that focuses on preparing budgets for various functions, such as production, selling, and administrative support. Activity-Based Budgets—budgets that focus on the budgeted cost of activities required to produce and sell products and services.

V.

Budgets as Financial Planning Models Financial Planning Models—mathematical models, used by most companies, of the master budget that can react to any set of assumptions about sales, costs, product mix, etc. Dow Chemical’s model that uses 140 separate, constantly revised cost inputs that are based on several different cost drivers is used as an example. The models give managers answers to “what-if” questions concerning deviations from sales targets, changes in input prices, and others.

VI.

Appendix 7: Use of Spreadsheet Models for Sensitivity Analysis

{L. O. 9}

This appendix shows how budgets can be prepared using spreadsheet computer software (see EXHIBIT 7-9, EXHIBIT 7-10, EXHIBIT 7-11, and EXHIBIT 7-12 for spreadsheet examples). It is important to have a data section where input values are located and to use cell formulas to construct the budgets. Then, one can easily alter the input data to instantly see the impact on the budget. Sensitivity Analysis (or what-if analysis)—the systematic varying of budget data input to determine the effects of each change on the budget.

Copyright ©2023 Pearson Education, Ltd.

94


CHAPTER 7:

Quiz/Demonstration Exercises

Learning Objective 1 1.

Which of the following is not an advantage of budgets? a. provide one-way communication channel b. compel managers to think ahead by formalizing their responsibilities for planning c. aid managers in communicating objectives to units d. provide benchmarks to evaluate subsequent performance

Learning Objective 2 2.

In order for budgets to be accepted by all the affected employees, the following process should be used: _____ budgeting. a. autocratic b. participative c. authoritarian d. socialist

Learning Objective 3 3.

The term used when managers overstate their budgeted costs or understate their budget revenues to create a budgeted profit level that is easier to achieve is _____. a. budgetary slack b. hypothetical budgeting c. budgetary tightening d. adverse budgeting

Learning Objective 4 4.

Which of the following budgets is considered the foundation of the entire master budget? a. purchases budget b. operating expense budget c. capital budget d. sales budget

Learning Objective 5 5.

_____ sets the overall goals and objectives of the organization. a. capital budgeting b. long-range planning c. strategic planning d. master budgeting

6.

The _____ summarizes the planned activities of all subunits of an organization – sales, production, distribution, and finance – and is the periodic business plan that includes a

Copyright ©2023 Pearson Education, Ltd.

95


coordinated set of detailed operating schedules and financial statements. a. sales budget b. master budget c. strategic plan d. long-range plan 7.

The _____ includes the sales budget, purchases budget, cost of goods sold budget, operating expenses budget, and budgeted income statement, whereas the _____ includes the capital budget, cash budget, and the budgeted balance sheet. a. operating budget; financial budget b. financial budget, operating budget c. strategic plan; financial budget d. strategic plan; operating budget

8.

_____ financial statements are another term for forecasted financial statements. a. long-range b. estimated c. future-oriented d. pro forma

Learning Objective 6 9.

The second step in the budgeting process is the preparation of the _____ budget. a. production b. sales c. cash collection from customers d. operating expense

10.

Which of the following is usually prepared before the disbursements for operating expenses budget? a. operating expense budget b. cash budget c. production budget d. budgeted balance sheet

Learning Objective 7 11.

Dow Corporation sells a product for $50. Budgeted sales for the first quarter of 20x1 are as follows: January $1,000,000 February 1,200,000 March 1,300,000 The company collects 60% in the month of sale, 20% in the following month, and 10% two months after the sale. Ten percent of all sales are uncollectible and are written off.

Copyright ©2023 Pearson Education, Ltd.

96


Budgeted cash receipts for March are _____. a. $ 780,000 b. $ 1,020,000 c. $ 1,120,000 d. $ 1,220,000 12.

Projected sales for Peck, Inc., for next year and beginning and ending inventory data: Sales 100,000 units Beginning Inventory 120,000 units Targeted Ending. Inv. 300,000 units The selling price is $10 per unit. Each unit requires 5 pounds of material, which costs $2 per pound. The beginning inventory of raw materials is 10,000 pounds. The company wants to have 9,000 pounds of material in inventory at the end of the year. Budgeted sales would be _____. a. $ 1,100,000 b. $ 960,000 c. $ 1,200,000 d. $ 1,000,000

Use the following information for questions 13 through 15. Projected sales for Module Company for the next month and beginning and ending inventory data are as follows: Sales Beginning inventory Targeted ending inventory

80,000 units 6,000 units 14,000 units

The selling price is $40 per unit. Each unit requires 8 pounds of material, which costs $2 per pound. The beginning inventory of raw material is 30,000 pounds. The company wants to have 40,000 pounds of material in inventory at the end of the month. 13.

Budgeted sales would be _____. a. $ 2,880,000 b. $ 3,520,000 c. $ 1,600,000 d. $ 1,920,000

14.

According to the production budget, how many units should be produced? a. 90,000 units b. 70,000 units c. 88,000 units d. 80,000 units

15.

Pounds of material to be purchased would be _____. a. 352,000 lbs.

Copyright ©2023 Pearson Education, Ltd.

97


b. c. d.

362,000 lbs. 392,000 lbs. 342,000 lbs.

Learning Objective 8 16.

Which of the following is not a part of the financial budget? a. production budget b. capital budget c. budgeted balance sheet d. budgeted income statement

17.

In constructing the cash budget, the finance section does not include _____. a. borrowing b. collections from customers c. repayments d. interest payments

Copyright ©2023 Pearson Education, Ltd.

98


CHAPTER 7: Solutions to Quiz/Demonstration Exercises

1. [a] 2. [b] 3. [a] 4. [d] 5. [c] 6. [b] 7. [a] 8. [d] 9. [c] 10. [a] 11. [c] The $1,120,000 expected cash receipts include $780,000, which represents 60% of March’s sales and $240,000 that is 20% of February’s purchases and $100,000 that is 10% of January sales. 12. [d] 100,000 units x $10 sales price per unit 13. [c] Budgeted sales are 80,000 units x $20/unit selling price, which is $1,600,000 in sales. 14. [c] The units to be produced are found by adding the units expected to be sold to the targeted ending inventory and then deducting the beginning inventory as shown below: Sales 80,000 units + Targeted ending inventory 14,000 units = Total needed 94,000 units - Beginning inventory 6,000 units = Production needed 88,000 units 15. [b] The number of pounds of material to be purchased is based on the production needs, target ending inventory, and the inventory of material already on hand. The computation appears below. Materials needed for production

88,000 units x 4 lbs./unit 352,000 lbs. + Desired ending inventory 40,000 lbs. = Total material needs 392,000 lbs. - Beginning inventory of material on hand 30,000 lbs. = Pounds of material to be purchased 362,000 lbs. 16. [a] 17. [b]

Copyright ©2023 Pearson Education, Ltd.

99


Chapter 8 Flexible Budgets and Variance Analysis LEARNING OBJECTIVES: When your students have finished studying this chapter, they should be able to: 1. 2. 3. 4. 5. 6. 7. 8.

Identify variances and label them favorable or unfavorable. Distinguish between flexible budgets and static budgets. Use flexible-budget formulas to construct a flexible budget. Compute and interpret static-budget variances, flexible-budget variances, and sales activity budgets. Understand how the setting of standards affects the computation and interpretation of variances. Compute and interpret price and quantity variances for materials and labor. Compute variable overhead spending and efficiency variances. Compute the fixed-overhead spending variance.

Copyright ©2023 Pearson Education, Ltd.

1


CHAPTER 8:

ASSIGNMENTS

CRITICAL THINKING EXERCISES 19. Interpretation of Favorable and Unfavorable Variances 20. Marketing Responsibility for Sales-Activity Variances 21. Production Responsibility for Flexible-Budget Variances 22. Responsibility of Purchasing Manager 23. Variable Overhead Efficiency Variance EXERCISES 24. Flexible Budget 25. Basic Flexible Budget 26. Flexible Budget 27. Basic Flexible Budget 28. Activity-Level Variances 29. Direct-Material Variances 30. Labor Variances 31. Quantity Variances 32. Labor and Material Variances 33. Material and Labor Variances PROBLEMS 34. Hotel Parking Service 35. Flexible and Static Budgets 36. Summary Explanation 37. Explanation of Variance in Income 38. Activity and Flexible-Budget Variances at KFC 39. Summary of App Based Taxi Aggregator’s Performance 40. Hospital Costs and Explanation of Variances 41. Flexible Budgeting 42. Activity-Based Flexible Budget 43. Straightforward Variance Analysis 44. Variance Analysis 45. Similarity of Direct-Labor and Variable-Overhead Variances 46. Material, Labor, and Overhead Variances 47. Automation and Direct Labor as Overhead 48. Standard Material Allowances 49. Role of Defective Units and Nonproductive Time in Setting Standards 50. Review Problem 51. Review Problem on Standards and Flexible Budgets; Answers Are Provided CASES 52. 53. 54.

Activity and Flexible-Budget Variances Activity-Based Costing and Flexible Budgeting Analyzing Performance

Copyright ©2023 Pearson Education, Ltd.

2


55. 56.

Complete Variance Analysis Nike 10k Problem: Performance Standards

EXCEL APPLICATION EXERCISE 57. Flexible-Budget and Sales-Activity Variances COLLABORATIVE LEARNING EXERCISE 58. Setting Standards INTERNET EXERCISE 59. Flexible Budgets at Hershey Food Corporation (www.hersheyland.com)

Copyright ©2023 Pearson Education, Ltd.

3


CHAPTER 8: I.

OUTLINE

Using Budgets and Variances to Evaluate Results A.

{L. O. 1}

Favorable and Unfavorable Variances Favorable profit variance occurs when actual profit exceeds budgeted profit. Unfavorable profit variance occurs when actual profit falls below budgeted profit. Favorable cost variance occurs when actual costs are less than budgeted costs. Favorable (F) versus Unfavorable (U) Variances Profits Actual > Expected F Actual < Expected U

Revenues F U

Costs U F

Unfavorable Cost Variance occurs when actual costs exceed budgeted costs. Static Budget Variances (i.e., variances from master budget amounts) may not be very useful in helping management assess whether costs are being controlled adequately when the actual activity level differs considerably from the static budget activity level. B.

Static Budgets Versus Flexible Budgets

{L. 0. 2}

A static budget is prepared for only one level of activity (for example, volume of sales activity). Differences between actual results and the static budget are staticbudget variances. Actual results could be compared with the original plan, even though a different activity level was reached than was used in constructing the static budget. See EXHIBIT 8-1 for an illustration of this in the performance report. A performance report is a generic term that usually means a comparison of actual results with some budget. Variances shown on the performance report direct management’s attention to significant deviations from expected results, allowing management by exception. Flexible Budget (or Variable Budget)—a budget that adjusts for changes in sales volume and other cost-driver activities. It is identical to the static budget in format, but it can be prepared for any levels of activity. For performance evaluation, the flexible budget would be prepared for the actual levels of activity achieved. Differences between actual results and a flexible budget are flexible-budget variances. In contrast, the static budget is kept fixed at only the originally planned levels of activity. C.

Flexible-Budget Formulas

{L. O. 3}

Copyright ©2023 Pearson Education, Ltd.

4


The cost functions or formulas that were discussed in Chapters 2 and 3 are used in constructing flexible budgets within the relevant range of activity. See EXHIBIT 82 for an illustration of the use of a budget formula for Beg Pakaian to create budgets for 7,000, 8,000, and 9,000 units of sales. The fixed costs/expenses are the same, in total, at each volume level. The variable costs/expenses increase by the budgeted amount for each unit increase in the activity level. Cost drivers other than units sold or produced must be considered in creating flexible budgets. See EXHIBIT 8-3 for a graph of flexible budget of costs. D.

Activity-Based Flexible Budgets Organizations are increasingly adopting activity-based costing (ABC) systems that have multiple cost drivers. Activity-Based Flexible Budget—based on budgeted costs for each activity center and related cost driver. ABC systems focus on activities as the primary cost objects. Costs of activity centers are then assigned to final cost objects such as products or customer classes using cost drivers. Companies that use ABC systems develop a flexible budget for each activity center. See EXHIBIT 8-4 for an illustration of Beg Pakaian’s activity-based flexible budget.

E.

Static-Budget Variances and Flexible-budget Variances {L. O. 4} There are two types of reasons why actual performance might not have conformed to the master budget. First, sales and cost-driver activity may be different than that forecasted (Activity-Level Variances). Second, revenues or variable costs per unit of activity and fixed costs per period may not be as expected (Flexible-Budget Variances). See EXHIBIT 8-6 for an illustration of these two types of variances. The sum of the flexible-budget variances and the activity-level variances is the master budget variance.

F.

Integrating Static-Budget, Flexible-Budget, and Sales-Activity Variances Managers use comparisons between actual results, master budgets, and flexible budgets to evaluate organizational performance. In evaluating performance, it is useful to distinguish Effectiveness (i.e., the degree to which a goal, objective, or target is met) and Efficiency (i.e., the degree to which inputs are used in relation to a given level of outputs). Effectiveness may be measured by determining whether the master budget goal has been met. Efficiency can be measured by comparing actual results to the flexible budget.

III.

Revenue and Cost Variances Sales-Activity Variances These variances measure how effective managers have been in meeting the planned level of sales. The final three columns in EXHIBIT 8-6 for Beg Pakaian show the sales-activity variances. All unit prices and variable costs are held constant in constructing the master budget and the flexible budget. The differences between the amounts are due to the level of sales activity.

Copyright ©2023 Pearson Education, Ltd.

5


The sales-activity variance indicates to managers the effect of not selling the budgeted sales level. Marketing managers are typically in the best position to explain why actual sales activities differed from plans. Flexible-Budget Variances Flexible-budget variances measure the efficiency of operations at the actual level of activity. The differences between columns 1 and 3 in EXHIBIT 8-6 for Beg Pakaian are flexible-budget variances. The total flexiblebudget variance is the difference between the actual income achieved and the flexible budget income for the achieved activity level. The total flexible-budget variance arises from sales prices received, and the variable and fixed costs incurred. Flexible-budget variances may serve as the basis for periodic performance evaluation. Operations managers are in the best position to explain these variances. The variances should not be used to fix blame. Managers being evaluated may resort to cheating to beat the system. See EXHIBIT 8-7 for an expanded, line-by-line computation of flexible-budget variances for Beg Pakaian. The variances should be interpreted as signals that actual operations have not occurred exactly as anticipated when the flexible-budget formulas were set, rather than as being good or bad. Any cost differing significantly (materially) from the flexible budget must be explained. IIIII. The Role of Standards in Determining Variances A.

{L. O. 5}

Setting Standards Expected Cost—the cost that is most likely to be attained. Standard Cost—a carefully developed cost per unit that should be obtained. Standard Cost Systems—value products according to only standard costs and are used for inventory valuation purposes. For planning and control purposes, expected future costs and expected future activity levels are used to set budgets and prepare performance reports. The standard costs from the standard cost system are not necessarily used because they may differ from the expected future costs. Companies use different cost systems for inventory valuation, product costing for decisionmaking, and for performance evaluation. What standard of expected performance should be used? Perfection Standards (or Ideal Standards)—expressions of the most efficient performance possible under the best conceivable conditions, using existing specifications and equipment. No provision is made for spoilage, waste, machine breakdowns, and so on. These standards are not frequently used because of the adverse effect on employee motivation resulting from their use. The unfavorable variances resulting from the use of these standards indicate the improvement that is possible through continuous improvement efforts. Currently Attainable Standards—levels of performance that can be achieved by realistic levels of effort. They are set just tightly enough so that employees regard

Copyright ©2023 Pearson Education, Ltd.

6


their attainment as highly probable if normal diligence and effort are exercised. These standards allow for normal defectives, waste, spoilage, and nonproductive time. Variances from these standards should be random and negligible. Another interpretation is that they are set tightly and employees regard their fulfillment as possible, though unlikely. They can only be achieved under very efficient operations. With this interpretation, variances tend to be unfavorable while employees view them as being tough but reasonable goals. Advantages are that they can be used for financial budgeting, inventory valuation, and departmental performance evaluation. They also have a desirable motivational impact on employees. IVV. Finding Explanations for Variances A.

Trade-offs Among Variances Because the operations of organizations are linked, the level of performance in one area of operations will affect performance in other areas. Paying higher than standard costs for materials results in an unfavorable materials price variance. However, if the higher price is due to a better-than-standard quality of material being purchased, less scrap and rework than normal may be possible, resulting in a favorable materials usage variance. The labels “favorable” and “unfavorable” are attention directors, not problem solvers. Faulty expectations may be the cause of variances rather than the execution of plans by managers. The validity of expectations must be questioned whenever variances exist.

B.

When to Investigate Variances If the variance is a result of random fluctuations, investigation is not needed. Managers expect a range of “normal” variances: this range may be based on economic (how large a dollar amount) or statistical (number of standard deviations from the expected mean) criteria. A typical investigation rule of thumb is to investigate all variances exceeding a certain dollar amount or percentage of expected cost, whichever is lower. The goal is to investigate those variances for which corrective action creates savings larger than the cost of investigation (i.e., benefits are greater than the costs).

C.

Comparisons with Prior Period’s Results Some organizations compare the most recent budget period’s actual results with last year’s results for the same period or last month’s results rather than use the flexible budget’s benchmarks. Unless the activities undertaken in the current period are nearly the same as those for the year ago period or prior month, this comparison does not reveal much meaningful information.

V.

More Detailed Analysis of Flexible-Budget Variances

Copyright ©2023 Pearson Education, Ltd.

7


Materials, labor, and overhead variances may be subdivided into price, usage, and spending components. If direct-labor costs are small in relation to total costs, they may be treated as overhead. Therefore, separate labor variances are not computed. A.

Variances for Direct Material and Direct Labor Variances from material and labor standards are found by comparing the flexible budget at the actual output level with the actual costs for these items. The flexiblebudget amounts are those that would have been spent for the actual output with expected efficiency. They are computed as follows: Flexible budget = Units of actual output achieved × Standard input allowed per unit of output achieved × standard price per unit of input

B.

Computing Price and Quantity Variances

{L. O. 6}

Flexible-budget variances measure the relative efficiency of achieving the actual output. The price and usage variances subdivide the flexible-budget variance. Price Variance—the difference between actual input prices and standard input prices multiplied by the actual quantity of inputs used. Quantity Variance—the difference between the quantity of inputs used and the quantity of inputs that should have been used to achieve the actual quantity of output. Rate Variance—the difference between actual labor rates and standard labor rates multiplied by the actual quantity of labor used. The variances should be separated into those that are subject to a manager’s direct influence and those that are not. Prices are typically less controllable than usage factors. The variances, once computed, should be used to raise questions, provide clues, and direct attention rather than to explain why budgeted operating income was not achieved. The effects of trade-offs between prices and usage should be analyzed. Was the purchase of substandard, lower-price materials a good idea? The objective is to hold either price or usage constant so that the effect of the other can be isolated (see EXHIBITS 8-9 and 8-10). Direct-Material Price Variance = (actual price – standard price) x actual quantity Direct-Labor Price (Rate) Variance = (actual price – standard price) x actual quantity Direct-Materials Quantity Variance = (actual quantity used – standard quantity) x standard price Direct-Labor Quantity (Efficiency or Usage) Variance = (actual quantity used – standard quantity) x standard price

Copyright ©2023 Pearson Education, Ltd.

8


C.

Summary of Materials and Labor Variances Exhibit 8-10 presents the analysis of direct material and direct labor in a format that deserves close study.

D.

Interpretation of Price and Quantity Variances If the actual price is less than standard or the actual quantity used is less than the standard quantity allowed, the variance is favorable. The opposite relationships imply unfavorable variances. See EXHIBIT 8-10 for a graphical representation of the variances. When production does not equal sales, the sales-activity variance is the difference between the static budget and the flexible budget for the number of units sold. In contrast, the flexible-budget cost variances compare actual costs with flexiblebudgeted costs for the number of units produced. Therefore, two flexible budgets must be prepared. When the number of units of raw materials differs from the amount used in production, the price variance should be computed based on the actual amount purchased. The usage variance should still be based on the actual usage of materials as compared to the quantity allowed for the production level achieved.

V.

Overhead Variances A.

Variable Overhead Variances

{L. O. 7}

The flexible-budget variance for variable overhead is subdivided into the variableoverhead efficiency variance and the variable overhead spending variance, computed as follows: Variable-Overhead Efficiency Variance = (actual quantity of – standard quantity) x cost-driver unit of cost driver allowed

standard variable-overhead rate per cost-driver

Variable-Overhead Spending Variance = actual variable overhead – (standard variable overhead rate per unit of cost-driver

x

actual cost-driver) activity used

The efficiency variance is controlled by regulating the cost-driver activity and the spending variance through the price paid for variable-overhead items. See EXHIBIT 8-11 for illustrations of the general approach for subdividing the

Copyright ©2023 Pearson Education, Ltd.

9


flexible-budget variances into the direct-materials price and usage variances, directlabor price and usage variances, and variable-overhead spending and usage variances. Actual costs are in the left-most column (A). A flexible budget based on actual inputs with expected prices is the center column (B). Finally, the right-most column contains a flexible-budget amount based on expected inputs for the actual outputs achieved at the expected prices (C). Differences between (A) and (B) are due to prices and differences between (B) and (C) are due to usage. B.

Fixed Overhead Variances

{L. O. 8}

The flexible budget in Column B based on actual inputs and the flexible budget in Column C based on standard inputs allowed are always the same. Fixed overhead is not expected to vary with the level of output (nor with the level of inputs). The entire fixed overhead flexible-budget variance shown in Exhibit 8-11 arises due to the difference between columns A and B because there is, by definition, no difference between columns B and C. This difference between the actual fixed overhead cost in column A and the budgeted cost in column B is the fixed overhead spending variance.

Copyright ©2023 Pearson Education, Ltd.

10


CHAPTER 8:

Quiz/Demonstration Exercises

Learning Objective 1 1.

_____ budgets provide expected revenues and costs for several levels of activity. a. flexible b. continuous c. master d. static

2.

_____ are budgets for a single activity level. a. flexible budgets b. master budgets c. both A and B d. static budget

Learning Objective 2 3.

The Tiger Company has the following budgeted costs for the production of its only product, exercise machines: Variable manufacturing costs $ 200.00 per unit Selling expenses $ 30.00 per unit Administrative $ 20.00 per unit Fixed manufacturing costs $ 300,000 per month Fixed selling and admin. costs $ 150,000 per month What are High Tech’s expected costs for 10,000 units of product to be produced and sold in March? a. $ 2,300,000 b. $ 2,950,000 c. $ 450,000 d. $ 2,500,000

4.

The flexible budget is based on the same assumptions of revenue and cost behavior (within the relevant range), as is the _____. a. master budget b. static budget c. neither A nor B d. both A and B

Learning Objective 3 5.

Which of the following is descriptive of an activity-based flexible budget? a.

based on actual costs for each activity center and related cost driver

Copyright ©2023 Pearson Education, Ltd.

11


b. c. d. 6.

based on budgeted costs for each activity center and related cost driver is limited to no more than ten activity centers A and C

The key differences between the traditional flexible budget and the activity-based flexible budget are _____. a. b. c. d.

the traditional should be used when a significant portion of the costs vary with cost drivers other than units of production traditional flexible budgeting is dramatically increasing in popularity some manufacturing costs that are fixed with respect to unit are variable with respect to cost drivers, and other than units, used for an activity-based flexible budget the larger the company, the more likely the activity-based flexible budget will not be used

Learning Objective 4 7.

When revenues or variable costs per unit of activity and fixed costs per period may not be as expected, this is called _____. a. flexible-level variance b. activity-budget variance c. static-budget variance d. master-budget variance

8.

The flexible budget is prepared using the _____. a. estimated levels of activity of the closest competitor b. historical levels of activity c. most conservative levels of activity d. actual levels of activity

Learning Objective 5 9.

_____variances measure how effective managers have been in meeting the planned level of sales. a. continuous-budget b. flexible-budget c. sales-activity d. master-budget

10.

_____ variances measure the efficiency of operations at the actual level of activity. a. b. c. d.

zero-based budget master-budget flexible-budget sales-activity

Copyright ©2023 Pearson Education, Ltd.

12


Learning Objective 6 Use the following information for questions 11 through 14. The Victor Company has developed the following standards for one of their products. Direct materials: 10 pounds x $4 per pound Direct labor: 5 hours x $10 per hour Variable overhead: 5 hours x $2 per hour The following activity occurred during the month of July: Materials purchased: 1,000,000 pounds at $4.10 per pound Material used: 800,000 pounds Units Produced: 20,000 units Direct labor: 110,000 hours at $7.50 per hour Actual variable OH: $250,000 The company records the materials price variance at the time of purchase. 11.

The materials price variance is _____. a. $80,000 favorable b. $80,000 unfavorable c. $100,000 unfavorable d. $100,000 favorable

12.

The labor usage variance is _____. a. $70,000 favorable b. $70,000 unfavorable c. $100,000 unfavorable d. $100,000 favorable

Learning Objective 7 13.

The variable overhead spending variance is _____. a. $50,000 unfavorable b. $50,000 favorable c. $100,000 favorable d. $100,000 unfavorable

14.

The variable overhead efficiency variance is _____. a. $20,000 unfavorable b. $20,000 favorable c. $10,000 favorable d. $10,000 unfavorable

Copyright ©2023 Pearson Education, Ltd.

13


CHAPTER 8:

Solutions to Quiz/Demonstration Exercises

1. [a] 2. [c] 3. [b]

From the information provided, Tiger’s flexible-budget cost formula is $450,000 + $250.00 X, where X is the number of units produced. Inserting 10,000 for X gives $450,000 + ($250.00 x 10,000) which equals $2,950,000.

4. [d] 5. [b] 6. [c] 7. [a] 8. [d] 9. [c] 10. [c] 11. [c] As stated, the company determines the price variance based on the material purchased. Therefore, the price variance is ($4.10 – $4.00) x 1,000,000 pounds, which is $100,000. The variance is unfavorable because the actual price paid ($4.10) exceeds the standard price ($4.00). 12. [c] The labor usage variance is found by multiplying the standard labor rate ($10) by the difference between the actual hours worked (110,000) and the number of hours that should have been taken to produce 20,000 units of 100,000 = 20,000 x 5 hrs./unit. The resulting variance is $100,000 ((110,000-100,000) x $10), which is unfavorable because more hours were worked than should have been for the production level achieved. 13. [a] The spending variance is the difference between the flexible-budget variance and the actual variable-overhead costs. In this case, the flexible budget for variable overhead would be $200,000 (20,000 x 5 hours) x unit x $2/hr. This yields a flexible budget variance of $50,000 unfavorable because actual variable-overhead costs are $250,000. 14. [a] The efficiency variance is the difference between the actual quantity of the cost-driver activity and the standard quantity allowed, which is then multiplied by the standard rate. The actual quantity of 110,000 hours is more than the standard allowed of 100,000 hours. The 10,000 hours difference is multiplied by the standard rate of $2 to arrive at a $20,000 unfavorable variance.

Copyright ©2023 Pearson Education, Ltd.

14


Chapter 9 Management Control Systems and Responsibility Accounting LEARNING OBJECTIVES: When your students have finished studying this chapter, they should be able to: 1. 2. 3. 4. 5. 6. 7. 1.

Describe the relationship of management control systems to organizational goals. Explain the importance of evaluating performance and describe how it impacts motivation, goal congruence, and employee effort. Develop performance measures and use them to monitor the achievements of an organization. Use responsibility accounting to define an organizational subunit as a cost center, a profit center, or an investment center. Prepare segment income statements for evaluating profit and investment centers using the contribution margin and controllable-cost concepts. Measure performance against nonfinancial performance measures such as quality, cycle time, and productivity. Use a balanced scorecard to integrate both financial and nonfinancial measures of performance. Describe the difficulties of management control in service and nonprofit organizations.

Copyright ©2023 Pearson Education, Ltd.

1


CHAPTER 9:

ASSIGNMENTS

CRITICAL THINKING EXERCISES 28. 29. 30. 31. 32.

Management Control Systems and Innovation Municipal Responsibility Accounting Control Systems and Customer Service Function of the Value Chain Control Systems and the Production Function of the Value Chain Key Performance Indicators

EXERCISES 33. Responsibility for Stable Employment Policy 34. Salesclerk’s Compensation Plan 35. Common Measures on a Balanced Scorecard 36. Goals and Objectives at Bharti Net 37. Performance Evaluation 38. Simple Controllable Costs 39. Quality Theories Compared 40. Quality Control Chart (EXHIBIT 9-12) 41. Cycle-Time Reporting PROBLEMS 42. Multiple Goals and Profitability 43. Responsibility Accounting, Profit Centers, and Contribution Approach 44. Incentives in Planned Economies 45. Balanced Scorecard 46. Quality Cost Report 47. Six Sigma, Mean, and Variance (EXHIBIT 9-13) 48. Productivity 49. Productivity Measurement CASES 50. 51. 52. 53.

Trade-offs Among Objectives Six Sigma Review of Chapters 1–9 Nike 10k Problem: Strategy at Nike

EXCEL APPLICATION EXERCISE 54. Wages for New Salary-Plus-Bonus Plan COLLABORATIVE LEARNING EXERCISE 55. Goals, Objectives, and Performance Measures INTERNET EXERCISE 56. Management Control System at Procter & Gamble (http://www.pg.com)

Copyright ©2023 Pearson Education, Ltd.

2


CHAPTER 9: I.

OUTLINE

Management Control Systems and Organizational Goals

{L. O. 1}

The foundation for control is the planning process. The outcome of planning provides the basis for control. Management Control System (MCS)—a logical integration of management accounting tools to gather and report data and to evaluate performance. See EXHIBIT 9-1 for the components of an MCS. The purposes of an MCS are: 1. 2. 3. 4. A.

clearly define and communicate the organization’s goals. ensure that every manager and employee understands the specific actions required of him/her to achieve organizational goals. communicate results of actions across the organization. motivate managers and employees to achieve the organization’s goals.

Management Control Systems and Organizational Goals A well-designed MCS aids and coordinates the process of making decisions and motivates individuals throughout the organization to work toward the same goals. It also coordinates forecasting revenue- and cost-driver levels, budgeting, measuring and evaluating performance. See EXHIBIT 9-2 for a description of the process of setting goals, objectives, and performance measures. Overall company goals, objectives, and performance measures are set by top management, not changed often, and reviewed on a periodic basis (usually once a year). Goals answer the question, “What do we want to achieve?” However, goals without measures do not motivate managers. Targets for goals are specific quantified levels of the measures. Goals and measures are often too vague to provide guidance. Therefore, critical processes and critical success factors are used. Critical Processes—series of related activities that directly affect the achievement of organizational goals (e.g., the goal “exceed guest expectations” would have “produce and deliver services” as a critical process). Critical (Key) Success Factors—actions that must be done well to drive the organization toward its goals (e.g., timeliness is a critical success factor for the “produce and deliver services” process and is measured by check-in time, check-out time, and response time to guest requests). Managers often face trade-off decisions with measures.

II.

Designing Management Control Systems and Organizational Goals {L. O. 2} A.

Motivating Employees To achieve maximum benefits at minimum costs, an MCS must foster goal congruence and managerial effort. Goal Congruence—individuals and groups aiming at the same organizational goals. It occurs when employees, working in their

Copyright ©2023 Pearson Education, Ltd.

3


perceived best interest, make decisions that meet the overall goals of the organization. Managerial Effort—exertion toward a goal or objective. Effort means not only working faster, but also working better (i.e., more efficient and effective). Incentives must be incorporated in the MCS to encourage goal-congruent behavior and managerial and employee effort. Performance evaluation along with bonuses tied to the achievement of objectives may help in this area. Motivation—aiming for some selected goal together with the resulting drive (effort) that creates action toward that goal. B.

Developing Performance Measures

{L. O. 3}

Both financial and nonfinancial measures of performance are important in achieving an organization’s objectives. Common to good performance measures are that they will have the following characteristics: 1. Reflect key actions and activities that relate to the goals of the organization 2. Affected by actions of managers and employees 3. Readily understood by employees 4. Reasonably objective and easily measured 4. Used consistently and regularly in evaluating and rewarding managers and employees 5. Balance long-term and short-term concerns Consideration of nonfinancial measures of performance can improve operational control. These nonfinancial measures may be timelier, and more easily understood and closely affected by employees at lower levels of the organization, where the product is made or services are rendered. Activities are now stressed that drive revenues and costs, instead of explaining the financial measures after the activity has occurred. The effects of nonfinancial measures of performance typically are not seen in the financial measures until considerable ground is lost. Financial measures are lagging indicators that arrive too late to help prevent problems and ensure the organization’s health. C.

Monitoring and Reporting Results A key driver of enterprise performance is the culture within the company that fosters continual learning and growth at all levels of management. Improvement in business processes must take place across all parts of the value chain. The performancereporting system, if effective, aligns results with managers’ goals and objectives, provides guidance to managers, communicates goals and their level of attainment throughout the organization, and enables organizations to anticipate and respond to change in a timely manner (see EXHIBIT 9-3).

D.

Weighing of Costs and Benefits Designers of an MCS must also weigh the costs and benefits of various alternatives. These are often difficult to measure, and both may become apparent only after

Copyright ©2023 Pearson Education, Ltd.

4


experimentation or use. III.

Controllability and Measurement of Financial Performance MCS often distinguish between controllable and uncontrollable events and between controllable and uncontrollable costs. Usually, responsibility center managers are in the best position to explain their center’s results even if the managers had little influence over them. Uncontrollable Cost—cannot be affected by the management of a responsibility center within a given time span. Controllable Costs—influenced by a manager’s decision and actions. Costs that are completely uncontrollable tell nothing about a manager’s decisions and actions because, by definition, nothing the manager does will affect the costs. Uncontrollable costs should be ignored in evaluating a manager’s performance, whereas controllable costs should be used. Activity-based costing is helping companies to identify controllable costs. A.

Identifying Responsibility Centers

{L. O. 4}

Responsibility Center—a set of activities assigned to a manager, a group of managers, or a group of employees. An effective MCS gives each lower-level manager responsibility for a group of activities and objectives and then reports on: 1. 2. 3.

the results of activities the manager's influence on those results effects of uncontrollable events

Responsibility Accounting—identifies what parts of the organization have primary responsibility for each objective, develops performance measures and targets to achieve, and designs reports of these measures by organization subunit, or responsibility center. Responsibility centers usually are classified according to their financial responsibility. 1.

Cost Centers, Profit, and Investment Centers Cost Center—a responsibility center in which a manager is accountable for costs only (e.g., accounting department). Its financial responsibilities are to control and report costs only. Profit Center—responsibility for controlling costs (or expenses) as well as revenues (e.g., marketing department). Nonprofit organizations whose goal is to break even are also considered profit centers because they have responsibilities for both revenues and costs. Investment Center—success is measured not only by its income but also by relating that income to its invested capital, as in a ratio of income to the value of the capital employed. This term is not widely used in practice. Instead, these

Copyright ©2023 Pearson Education, Ltd.

5


responsibility centers are typically referred to simply as profit centers. B.

Contribution Margin Many organizations combine the contribution approach to measuring income with responsibility accounting (i.e., report by cost behavior as well as by degrees of controllability). Line a of EXHIBIT 9-5 gives the contribution margin for each of the various segments of a retail grocery store. The relevant segments of the company’s organization chart are shown in EXHIBIT 9-4.

C.

Contribution Controllable by Segment Managers

{L. O. 5}

Line b in EXHIBIT 9-5 gives the contribution controllable by segment managers. Managers of the various segments may have control over certain advertising, sales promotion, salespersons' salaries, management consulting, training and supervision costs that are deducted from the segment contribution margin to yield the contribution controllable by segment managers. When service department costs are allocated, charges are made for division headquarters, or store depreciation or lease costs are determined. No easy answers exist regarding if, and how much of, these costs are controllable by segment managers. D.

Contribution by Segments Fixed expenses (e.g., depreciation, property taxes, insurance, and perhaps the segment manager's salary) are not under the control of the segment manager. These costs are deducted from the contribution controllable by segment managers to give the contribution by segments. Line c in EXHIBIT 9-5 shows segment contributions for a retail grocery store, which approximate the financial performance of the segments, as distinguished from the financial performance of its manager, which is measured in line b.

E.

Unallocated Costs Central corporate costs (e.g., top management and some corporate-level services, such as legal and taxation) are frequently not allocated to segments unless a persuasive cause and effect, or activity-based justification for allocation of these costs.

IV.

Measurement of Nonfinancial Performance A.

{L. O. 6}

Control of Quality Quality Control—the effort to ensure that products and services perform to customer requirements. In the traditional approach to maintaining the desired level of quality in the United States, companies inspected completed products, and rejected or reworked those that failed inspection. Due to the expense of inspection, only a sample of products was tested. The production process was judged to be in control as long as the

Copyright ©2023 Pearson Education, Ltd.

6


number of defective products did not exceed an acceptable quality level. This meant that some defective products could still make their way to customers. Due to competitive pressures, and seeing the success of Japanese products, U.S. companies have learned that the traditional approach is extremely costly. The resources consumed in making and detecting defective parts are a waste, and considerable rework may be necessary to correct the defects. It is also very costly to repair products in use by customers or to win back a dissatisfied customer. Quality Cost Report—displays the financial impact in quality. See EXHIBIT 9-7 for Eastside Manufacturing Company’s quality cost report. There are four categories of quality costs: 1.

2. 3. 4.

Prevention—costs incurred to prevent the production of defective products or deliver substandard services, including engineering analyses to improve product design for better manufacturing, improvements in production processes, increased quality of material inputs, and programs to train personnel Appraisal—costs incurred to identify defective products or services including inspection and testing Internal Failure—costs of defective components, and final products or services that are scrapped or reworked External Failure—costs caused by delivery of defective products or services to customers, such as field repairs, returns, and warranty expenses The costs stated in reports typically understate the true quality costs because lost sales are not included due to measurement difficulty.

In recent years, more companies are taking the total quality management (TQM) approach to quality control. Total Quality Management—concentrates on the prevention of defects and on customer satisfaction and is the application of quality principles to all the organization’s endeavors to satisfy customers. The TQM approach assumes that the cost of quality is minimized when a firm achieves high quality levels. TQM delegates the responsibilities for many management functions to employees. For it to be successful, employees must be very well trained in the process, the product or service, and the use of quality-control information. In TQM, employees are trained to prepare, interpret, and act on quality-control charts, like that shown in EXHIBIT 9-8. Quality-Control Chart—statistical plot of measures of various product dimensions or attributes. The plot helps to detect process deviations before the process generates defects and identifies excess variation in product dimensions or attributes that should be addressed by process or design engineers. The most recent trend in quality control is six sigma, an analytical method aimed at achieving near-perfect results on a production line. Literally, six sigma requires fewer

Copyright ©2023 Pearson Education, Ltd.

7


than 3.4 defects per million. The focus is on measuring how many defects a company has in its process because once it measures the defects; it can take steps to eliminate them. B.

Control of Cycle Time Cycle Time (or Throughput Time)—the time taken to complete a product or service, or any of the components of a product or service. It is a summary measure of manufacturing efficiency and effectiveness, and is an important cost driver. The longer a product or service is in process the more costs are consumed. Lowering cycle time requires smooth-running processes and high quality, and also creates increased flexibility and quicker reactions to customer needs. As cycle time is decreased, quality problems become apparent throughout the process and must be solved if quality is to improve. Decreasing cycle time also results in bringing products or services more quickly to customers, a service customers’ value. The use of bar coding can be used to measure cycle times. Reports for cycle times, such as that in EXHIBIT 9-10, can be prepared to alert managers' attentions to cycle-time problem areas.

C.

Control of Productivity Productivity—a measure of outputs divided by inputs. The fewer inputs needed to produce a given output, the more productive the organization. Companies differ as to which measures of input are most important depending on whether they are laborintensive, machine-intensive, or material-intensive organizations. Examples of different types of productivity ratios are given in EXHIBIT 9-11.

D.

Choice of Productivity Measures The productivity measures companies choose to manage depend on the behaviors desired. Sometimes myopic attention to one or a few measures comes at the expense of others, resulting in the endangerment of the long-run profitability of a firm. Rather than specify productivity goals, managers may concentrate their control on the more fundamental activities of quality and service. Then the productivity measures can be used to monitor the actual benefits of improvements in these activities.

E.

Productivity Measures Over Time Be careful in comparing productivity measures over time because process changes or inflation can make them misleading.

F.

The Balanced Scorecard

{L. O. 7}

Balanced Scorecard—performance measurement and reporting system that strikes a balance between financial and operating measures, links performance to rewards, and gives explicit recognition to the diversity of stakeholder interests. Line managers can understand the numbers presented due to the use of nonfinancial measures. The

Copyright ©2023 Pearson Education, Ltd.

8


balanced scorecard focuses on performance measures from across the spectrum of the organization. This enhances the learning process because managers are made aware of the results of actions and how these actions are linked to the organizational goals. The classic balanced scorecard includes key performance indicators—measures that drive the organization to meet its goals—grouped into four categories (see EXHIBIT 9-11): 1. 2. 3. 4. IV.

Financial Customers Internal Processes Employee Growth and Learning

Management Control Systems in Service, Government, and Nonprofit Organizations {L. O. 8} Service, government, and nonprofit organizations have more difficulty implementing management control systems than do manufacturing firms because their outputs are more difficult to measure. Nonprofit and governmental agencies have the additional problem of not having a financial “bottom line” as an objective. Also, many people in nonprofit organizations seek their positions for reasons other than monetary rewards (e.g., the desire to help improve conditions in underdeveloped countries). There are six reasons why control systems will probably not be as highly developed in nonprofit organizations as they are in profit-seeking firms. 1. 2. 3.

4. 5. 6. VI.

Organizational goals are less clear and multiple, which requires difficult tradeoffs. Professionals dominate and have been less receptive to the installation and improvement of a formal control system. Measurements are more difficult because there is no profit measure and there are many discretionary fixed costs, which makes the relationships of inputs to outputs difficult to specify and measure. There is less competitive pressure to improve management control systems. The role of budgeting is often more a matter of playing bargaining games with sources of funding than it is rigorous planning. Motivations and incentives of individuals differ from those in for-profit organizations.

Future of Management Control Systems Certain management control principles that can guide the redesign of systems to meet new management needs follow: 1. 2.

3.

Always expect that individuals will be pulled in the direction of their own self-interest. Design incentives so those individuals who pursue their own self-interests are also achieving the organization's objectives. When multiple objectives are present, multiple incentives are appropriate. Evaluate actual performance based on expected or planned performance, revised, if possible, for actual output achieved.

Copyright ©2023 Pearson Education, Ltd.

9


4. 5. 6.

7.

Consider nonfinancial performance to be an important determinant of long-term success. Array performance measures across the entire value chain of the company. Periodically review the success of the management control system. Are objectives being met? Does meeting the objectives mean that subgoals and goals are being met too? Do individuals have, understand, and use the management control information effectively? Learn from management control successes (and failures) of competitors around the world. Despite cultural differences, human behavior is remarkably similar. Successful applications of new technology and management controls may be observed in the performance of others.

Copyright ©2023 Pearson Education, Ltd.

10


CHAPTER 9:

Quiz/Demonstration Exercises

Learning Objective 1 1.

A management control system is distinguished from a purely accounting system by _____. a. its focus on motivation and evaluation of performance consistent with the organization's goals b. its focus on organizational goals and objectives c. its focus on internal management decision making d. all of the above

2.

Which of the following affects all components of a management control system? a. progress measurement b. feedback c. learning d. A and C e. A, B, and C

Learning Objective 2 3.

In order to encourage goal-congruent behavior and to motivate managerial effort, a management control system must include _____. a. ultimatums from top management regarding the accomplishment of short-run profitability without regard to long-run consequences b. organizational goals that are not rewarded by the performance evaluation and incentive structure of the firm c. a performance evaluation and incentive structure inconsistent with the organization's goals d. none of these

4.

A management control system must foster _____. a. goal congruence b. managerial effort c. individuality d. B and C e. A and B

Learning Objective 3 5.

Good performance measures will not be _____. a. concerned with only long-term goals b. easily measured c. used to evaluate managers d. readily understood

Copyright ©2023 Pearson Education, Ltd.

11


6.

Which of the following is a financial measure? a. number of defects b. required return on investments c. amount of wasted materials by employee d. number of customer complaints

Learning Objective 4 7.

Which of the following responsibility centers does not have accountability for revenues? a. investment centers b. profit centers c. cost centers d. none of the above

8.

Which of the following is an example of a cost center? a. a division b. an accounting department c. a plant d. a subsidiary

Learning Objective 5 9.

In a segment income statement prepared using the contribution margin format, the key measure that is used to evaluate managers of the segments is _____. a. contribution controllable by segment managers b. contribution by segments c. unallocated costs d. contribution margin

10.

An example of an uncontrollable cost to a raw materials supervisor is _____. a. quality costs b. information system support cost c. ordering errors d. shipping costs

Learning Objective 6 11.

A quality cost report details the various costs of maintaining and selling quality products to customers and includes which of the following four categories? a. promotion, detention, suspension, and retention b. recision, derision, provision, and sedition c. elevation, degradation, normalization, and interpretation d. prevention, appraisal, internal failure, and external failure

12.

An example of an external failure cost is _____. a. a decrease in the quality of material inputs

Copyright ©2023 Pearson Education, Ltd.

12


b. c. d. 13.

scrap costs warranty costs inspections costs

For a company to improve its productivity, it must _____. a. use more inputs to produce fewer outputs b. produce fewer outputs for the level of inputs used c. use more inputs to produce the same level of output d. produce more outputs for the level of inputs used

Learning Objective 7 14.

The classic balanced scorecard’s key performance indicators include _____. a. financial b. employee growth and learning c. external processes d. A, B, and C e. A and B

15.

Which of the following examples does not measure financial performance? a. inventory turnover b. working capital c. market share d. all of the above

Learning Objective 8 16.

Service and nonprofit organizations have more difficulty in implementing management control systems than do manufacturing firms because _____. a. their outputs are more difficult to measure b. they have no goals c. they are concerned with the satisfaction of their customers d. their managers are less sophisticated and do not understand the purpose and use of management control systems

17.

Service and nonprofit organizations differ from profit-seeking organizations because in service and nonprofit organizations, _____. a. organizational goals and objectives are more easily determined b. less competitive pressure is exerted from other nonprofit organizations c. discretionary costs are usually small d. budgeting is less of a bargaining game

Copyright ©2023 Pearson Education, Ltd.

13


CHAPTER 9:

Solutions to Quiz/Demonstration Exercises

1. [d] 2. [d] 3. [d] 4. [e] 5. [a] 6. [b] 7. [c] 8. [b] 9. [a] 10. [b] 11. [d] 12. [c] 13. [d] 14. [e] 15. [c] 16. [a] 17. [b]

Copyright ©2023 Pearson Education, Ltd.

14


Chapter 10 Management Control in Decentralized Organizations LEARNING OBJECTIVES: When your students have finished studying this chapter, they should be able to: 1. 2. 3. 4. 5. 6. 7. 8. 9.

Define decentralization and identify its expected benefits and costs. Distinguish between responsibility centers and decentralization. Explain how the linking of rewards to responsibility-center performance metrics affects incentives and risk. Compute return on investment (ROI), economic profit, and economic value added (EVA). Compare the incentives created by income, ROI, and economic profit (or EVA) performance measures. Define transfer prices and identify their purpose. State the general rule for transfer pricing and use it to assess alternative transfer prices based on total costs, variable costs, or market prices. Identify the factors affecting multinational transfer prices. Explain how controllability and management by objectives (MBO) aid the implementation of management control systems.

Copyright ©2023 Pearson Education, Ltd.

1


CHAPTER 10:

ASSIGNMENTS

CRITICAL THINKING EXERCISES 23. Decentralization 24. Comparing Financial Measures of Performance 25. Performance Metrics and Ethics 26. Transfer Pricing and Organizational Behavior EXERCISES 27. Simple ROI Calculations 28. Simple ROI Calculation 29. Simple ROI and Economic Profit Calculations 30. EVA 31. Comparison of Asset and Equity Bases 32. Finding Unknowns 33. Gross Versus Net Asset Value 34. Variable Cost as a Transfer Price 35. Maximum and Minimum Transfer Price 36. Multinational Transfer Prices PROBLEMS 37. Agency Theory 38. Margins and Turnover 39. ROI by Business Segment 40. EVA versus Economic Profit 41. EVA 42. EVA 43. Evaluation of Divisional Performance 44. Use of Gross or Net Book Value of Fixed Assets 45. Role of Economic Value and Replacement Value 46. Profit Centers and Transfer Pricing in an Automobile Dealership 47. Transfer Pricing 48. Transfer-Pricing Concession 49. Transfer Prices and Idle Capacity 50. Transfer-Pricing Principles 51. Negotiated Transfer Prices 52. Transfer Prices and Minority Interests 53. Multinational Transfer Prices 54. Review of Major Points in This Chapter CASES 55. 56. 57.

Profit Centers and Central Services Management by Objectives Nike 10k Problem: ROI and Economic Profit

EXCEL APPLICATION EXERCISE

Copyright ©2023 Pearson Education, Ltd.

2


58.

Return on Investment and Economic Profit

COLLABORATIVE LEARNING EXERCISE 59. ROI INTERNET EXERCISE 60. Decentralization at Marriott International (http://www.marriott.com)

Copyright ©2023 Pearson Education, Ltd.

3


CHAPTER 10: I.

OUTLINE

Centralization Versus Decentralization

{L. O. 1}

Decentralization—the delegation of decision-making authority to managers throughout an organization. The lower in the organization that this freedom exists, the greater the degree of decentralization. Decentralization is a matter of degree along a continuum. Decentralization is not right for every firm. Centralization is the process by which decision-making is concentrated with a particular location or group. A.

Costs and Benefits Benefits of decentralization include: 1. Better decisions made by lower-level managers due to their having the best information regarding local conditions, 2. the acquisition of decision-making ability and other management skills that help lower-level managers move up in an organization, and 3. better motivation and higher status of lower-level managers. Costs include: 1. Making decisions that are not in the organization’s best interests, 2. duplication of services, 3. higher costs of accumulating and processing information, and 4. the waste of time due to dickering with other organizational units about goods or services one unit provides to another.

B.

Middle Ground Cost-benefit considerations usually require that some management decisions are highly decentralized and others centralized. Decentralization is most successful when an organization’s segments are relatively independent of one another (i.e., decisions of a manager in one segment will not affect the fortunes of another segment). If much internal buying and selling or buying and selling from the same suppliers and outside markets take place, segments are candidates for heavier centralization.

C.

Responsibility Centers and Decentralization

{L. O. 2}

Do not confuse profit centers (accountability for revenue and expenses) with decentralization (freedom to make decisions). Some profit center managers have considerable freedom in making decisions, whereas others need top-management approval for most decisions. Deciding on whether to use a cost center or a profit center hinges on whether goal congruence and managerial effort are enhanced, not on the degree of decentralization present. II.

Performance Metrics and Management Control

Copyright ©2023 Pearson Education, Ltd.

{L. O. 3}

4


A.

Agency Theory, Performance, and Rewards Managers tend to focus their efforts in areas where performance is measured and where their performance affects rewards. The more objective the measures of performance, the more likely the manager will provide effort. The choice of rewards clearly belongs within an overall system of management control. They may be monetary, as in pay raises and bonuses, or they can be nonmonetary, such as promotions, praise, self-satisfaction, elaborate offices, and private dining rooms. The design of a reward system is mainly the concern of top management.

III.

Measures of Segment Performance A.

{L. O. 4}

Income Measures Measures of income are readily available from the financial reporting system at any level of the organization for which a company can identify revenues and expenses, such as a subsidiary, a division, or a business unit.

B.

Return on Investment Return on Investment (ROI)—income (or profit) divided by the investment required to obtain that income or profit. This is a better test of profitability than is examining the income generated by different business segments. Given the same risks, for any given amount of resources required, the investor wants the maximum income. ROI is a useful common denominator and can be compared with rates inside and outside the organization, and with opportunities in other projects and industries.

C.

ROI as the Product of Return on Sales and Investment Turnover The calculation of ROI is as follows: ROI

= =

income/revenue × revenue/invested capital return on sales × capital turnover

An improvement in either the Income Percentage Of Revenue (or Return on Sales)—income divided by revenue, or Capital Turnover—revenue divided by invested capital, without changing the other will improve ROI. Increasing turnover is one of the advantages of implementing the JIT philosophy. D.

Measuring Investment

E.

Definitions of Investment Possible definitions of investment in this example are as follows: 1. Stockholders’ equity: This definition considers only the investment by the

Copyright ©2023 Pearson Education, Ltd.

5


stockholders. 2. Stockholders’ equity and long-term liabilities. This definition encompasses not only the investment by stockholders but also the investment by debt investors. 3. Stockholders’ equity, long-term liabilities, and current liabilities: This definition encompasses all sources of financing for the firm. Because of the accounting identity, this is also equal to total assets. F.

Valuation of Assets When firms use ROA measures, two additional issues arise related to the valuation of total assets. First, companies could value assets contained in the investment base at either gross book value or at net book value. Second, they should value assets at either historical cost or some version of current cost. Some possible definitions of invested capital are: 1. Total assets 2. Total assets employed (excludes vacant land and construction in process) 3. Total assets less current liabilities 4. Stockholders’ equity Averages for the period are typically used. For divisional manager performance evaluation, any three of the asset bases are recommended rather than stockholders’ equity. Total assets are the best base if the manager’s mission is to put all assets to their best use regardless of their financing. If extra assets must be carried that are not currently productive upon the direction of top management, then total assets employed is the best measure. If the manager has direct control over short-term credit and bank loans, then total assets less current liabilities is best. Managers will focus attention on reducing those assets and increasing those liabilities that are included in their base in order for their ROI or residual income to look better. Most companies use all assets in invested capital when computing ROI and residual income, and about half deduct some portion of current liabilities. Assets can be valued using gross book value (original cost), net book value (original cost less accumulated depreciation), and historical cost, or some version of current value. Practice is overwhelmingly in favor of net book value based on historical cost. Historical cost is used due to cost-benefit, objectivity, predictive ability, and risk reasons. However, for non-routine decisions, special studies should be undertaken to gather any current values that are relevant. Plant and Equipment: Gross or Net? Net Book Value (or book value)—original cost less accumulated depreciation. Most companies use net book value when calculating their investment. Gross Book Value—original cost of an asset before deducting accumulated depreciation. Some companies use gross book value because it facilitates comparisons between years and between plants or divisions. An example showing the impact of using net book value or gross book value on ROI is presented that shows improving results in later years

Copyright ©2023 Pearson Education, Ltd.

6


using net book value and a steady ROI if gross book value is used. The effect on motivation should be considered when selecting between net and gross book value. Managers evaluated using gross book value will tend to replace assets sooner than those firms using net book value. G.

Incentives from ROI Although evaluation based on ROI causes manager to consider both income and investment in their decisions, it still may not align the incentives for the manager with the goals of the firm.

IV.

Economic Profit or Economic Value Added (EVA) Some companies favor emphasizing an absolute amount of income rather than a percentage return. Economic Profit (Residual Income)—net income less “imputed” interest. Cost of Capital (“Imputed” interest)—what the firm must pay to acquire more capital, whether or not it actually has to acquire more capital to take on this project. The cost of capital is the minimum acceptable rate of return for investments in a project or division. The “imputed” interest rate used should reflect the risk of the investment. Economic Value Added (EVA)—net operating income less the after-tax weighted-average cost of capital multiplied by the sum of long-term liabilities and stockholders’ equity. EVA is a variant of residual income. EVA may increase shareholder value due to better allocating, managing, and redeploying scarce capital resources. See EXHIBIT 10-2.

V.

Incentives From Income, ROI, or Economic Profit

{L. O. 5}

The use of ROI may cause some divisions to forgo investments that are desirable from a company-wide viewpoint because the investments lower the division’s ROI. The use of economic profit alleviates this problem, because all investments returning more than the company’s cost of capital is acceptable by all divisions because residual income increases. Thus, the use of economic profit (EVA) promotes goal congruence and leads to better decisions than ROI. Yet, most companies use ROI because it is easier for managers to understand, it facilitates comparison across divisions of different sizes, and when combined with appropriate growth and profit targets, dysfunctional motivations can be minimized. See EXHIBIT 10-5. VII.

Transfer Pricing

{L. O. 6}

When segments interact greatly, there is an increased possibility that what is best for one segment hurts another segment badly enough to have a negative effect on the entire organization. Such a situation may occur when one segment provides products or services to another segment and charges that segment a transfer price. Transfer Prices—amounts charged by one segment of an organization for a product or service that it supplies to another segment of the same organization. It is revenue to the segment providing the product or

Copyright ©2023 Pearson Education, Ltd.

7


service, and it is a cost to the acquiring department. A.

Purposes of Transfer Pricing Transfer prices exist within organizations to communicate data that will lead to goalcongruent decisions. They should help guide managers to make the best possible decisions regarding whether to buy or sell products and services inside or outside the total organization. They are also used to help in evaluating segment performance and, thus, motivate both the selling manager and buying manager toward goal-congruent decisions. Finally, multinational companies use transfer pricing to minimize their worldwide taxes, duties, and tariffs.

B.

A General Rule for Transfer Pricing

{L. O. 7}

The general rule for transfer pricing is: Transfer Price

=

Outlay Cost

+

Opportunity Cost

Outlay cost is the additional amount the selling segment must pay to produce and transfer a product or service to another segment. It is often a variable cost for producing the item transferred. Opportunity cost is the maximum contribution to profit that the selling segment forgoes by transferring the item internally. See EXHIBIT 104. C.

Market-Based Transfer Prices If a competitive market exists for the product or service being transferred internally, the use of market price as a transfer price will generally lead to the desired goal congruence and managerial effort. Some adjustments to market prices can be made for things like selling and delivery expenses that are not necessary if an internal transfer takes place. However, market prices are not always available for items transferred internally.

D.

Transfers at Cost When market prices are not available, are inapplicable, or are impossible to determine, versions of “cost-plus-a-profit” are often used in order to provide a “fair” or “equitable” substitute for regular market prices. Approximately half of the major companies in the world transfer items at cost. However, there are several possible definitions of cost including actual variable cost, standard variable cost, actual full cost, and standard full cost. There may be a problem with the use of cost as a transfer price. A cost’s behavior pattern may be disguised because the cost drivers affecting the acquiring division are typically units and those for the producing division may be more complicated. Problems with the use of actual cost are that the buying division is hindered in planning because actual cost cannot be

Copyright ©2023 Pearson Education, Ltd.

8


known in advance, and the supplying division has no incentive to control its costs because any inefficiencies are simply passed on to the buying division. The use of budgeted or standard costs is preferred. Cost-based transfer prices lead to dysfunctional decisions—decisions in conflict with the company’s goals. E.

Negotiated Transfer Prices Companies committed to segment autonomy often allow their managers to negotiate transfer prices. Cost and market prices may be considered in the negotiations, but no policy requires this. Supporters claim that because these managers have the best knowledge of what a company will gain or lose by producing and transferring the product or service, negotiation allows managers to make optimal decisions. Critics focus on the time wasted in negotiations.

F.

Multinational Transfer Pricing

{L. O. 8}

Whereas domestic companies focus on goal congruence and motivation in establishing their transfer-pricing policies, multinational companies use transfer prices to try to minimize worldwide income taxes, import duties, and tariffs. Divisions in relatively low-tax countries attempt to set high transfer prices for products being sold to divisions in relatively high-tax divisions, and vice versa. Recognizing this tendency to set transfer prices in an attempt to minimize taxes, tax authorities set restrictions on allowable transfer prices. U.S. multinationals must follow an Internal Revenue code rule specifying that transfers be priced at “arm’slength” market values, or at the values that would be used if the divisions were independent companies. Even with this rule, companies have some latitude in deciding an appropriate “arm’s-length” price. VIII. Keys to Successful Management Control Systems

A.

{L. O. 9}

Focus on Controllability Managers should be evaluated based on their controllable performance (in many cases some controllable contribution in relation to controllable investment). Decisions such as increasing or decreasing investment in a division are based on the economic viability of the division, not the performance of its manager.

B.

Management by Objectives and Setting Expectations Management by Objectives (MBO)—the joint formulation by a manager and his or her superior of a set of goals and of plans for achieving the goals for a forthcoming period. The plans often take the form of a responsibility accounting budget (along with supplementary goals such as levels of management training and safety that may not be incorporated into the accounting budget). The manager’s performance is then

Copyright ©2023 Pearson Education, Ltd.

9


evaluated in relation to these agreed-upon objectives. Use of MBO allows managers to accept assignments to less profitable segments with less reluctance because the manager will be evaluated on budgeted results that are negotiated with a superior and not on absolute profitability. C.

Budgets, Performance Targets, and Ethics Either an ROI or a residual income system can promote goal congruence and managerial effort if top management gets everybody to focus on what is currently attainable in the forthcoming budget period.

Copyright ©2023 Pearson Education, Ltd.

10


CHAPTER 10:

Quiz/Demonstration Exercises

Learning Objective 1 1.

Advantages of decentralization include all of the following except _____. a. reduced service duplication b. better motivation and higher status of lower-level managers c. lower manager motivation d. time taken to negotiate between units

2.

Advantages of decentralization include all of the following except _____. a. divisional management is able to react to changing market conditions more rapidly than top management b. divisional management is a source of personnel for promotion to top management positions c. decentralization permits divisional management to concentrate on firm-wide problems and long range planning d. decentralization can motivate divisional managers

Learning Objective 2 3.

Profit centers can _____. a. be used in both centralized and decentralized operations b. never be used by organizations regardless of their degree of centralization in decision making c. only be used if an organization is decentralized in structure d. only be used if an organization is centralized in structure

4.

The main criticism of profit centers is _____. a. a profit center is difficult to identify b. managers are given profit responsibility without authority c. a profit center is too costly to identify d. A and B e. A and C

Learning Objective 3 5.

According to agency theory, employment contracts will not trade off _____. a. incentive b. risk c. timeliness d. cost of measuring performance

6.

An example of an item that is not a reward for managers is _____. a. a promotion

Copyright ©2023 Pearson Education, Ltd.

11


b. c. d.

praise a larger office staff reduction

Learning Objective 4 Use the following information for questions 7 and 8. Wilson Corporation has three divisions: A, B, and C. The expected earnings and amount of investment in these divisions for the coming year are: Division A B C

Investment $ 1,000,000 1,500,000 2,500,000

Earnings $ 100,000 400,000 500,000

A proposed investment that costs $500,000 and has projected earnings of $100,000 has been offered to each of the three divisions. All divisions can make this investment and the company’s cost of capital is 10%. 7.

If division managers are being evaluated based on return on investment, _____ will invest in the project. a. the manager of division B b. the manager of division A c. the manager of division C d. all three managers

8.

If division managers are being evaluated based on residual income, _____ will invest in the project. a. the manager of division A b. the manager of division B c. the manager of division C d. all three managers

Learning Objective 5 9.

An investment base used in calculating ROI or economic profit that facilitates comparisons between years and between plants and facilities is _____ value. a. net current market b. gross current market c. net book d. gross book

10.

The possible reasons that historical cost may be preferred to current market value when valuing assets for measuring performance is _____. a. lower costs of accumulating data

Copyright ©2023 Pearson Education, Ltd.

12


b. c. d. e.

lower risk more objective A, B and C A and C

Learning Objective 6 11.

When one segment or division of a company provides goods or services to another segment or division within the same company, a _____ price is charged which assists the segment managers in deciding whether the transfer should take place. a. competitive b. transfer c. minimum d. segment

12.

The objective of a transfer pricing system should be to _____. a. maximize the transfer price b. maintain goal congruence between the divisions and the entire firm c. minimize the transfer price d. B and C e. none of the above

Learning Objective 7 13.

The use of cost-based transfer prices has the advantage of _____. a. disguising the true cost-behavior pattern of the good or service being transferred b. reducing inefficiency on the part of the supplying division c. being easy to determine using existing accounting records d. leading to goal congruence and managerial effort more than the use of marketbased prices

14.

Sometimes market prices cannot be used in a transfer-pricing situation. Which of the following is not a reason that market prices cannot be used? a. may be very costly to gather the information b. may be impossible to determine c. may be nonexistent d. may be inapplicable in the transfer pricing situation

Learning Objective 8 15.

Factors influencing transfer pricing in multinational companies include _____. a. complying with legal restrictions placed on the setting of transfer prices such as the use of “arm’s-length” values required in the U.S. b. restrictions on sales tax to foreign owners imposed by foreign governments c. maximizing worldwide taxes and duties

Copyright ©2023 Pearson Education, Ltd.

13


. 16.

all of the above

As compared with national companies, multinational companies are _____. a. more likely to use transfer prices b. less likely to use transfer prices c. equally likely to use transfer prices d. likely to only use transfer prices in times of economic crisis

Copyright ©2023 Pearson Education, Ltd.

14


CHAPTER 10: 1. [b] 2. [c] 3. [a] 4. [b] 5. [c] 6. [d] 7. [b]

Solutions to Quiz/Demonstration Exercises

Before considering investing in the proposed project, the ROIs of the three divisions are expected to be: A—10.00% = $100,000 expected earnings/$1,000,000 investment B—26.67% = $400,000 expected earnings/$1,500,000 investment C—20.00% = $500,000 expected earnings/$2,500,000 investment The resulting ROIs for each division were they to make the investment are: A—13.33% = $200,000 expected earnings / $1,500,000 investment B—23.81% = $500,000 expected earnings / $2,100,000 investment C—19.35% = $600,000 expected earnings / $3,100,000 investment Therefore, only division R’s manager would be compelled to make the investment because it would increase his division’s ROI.

8. [d]

Before considering the investment in the proposed project, the economic profit of each division is: A—$0 = $100,000 – (10% x $1,000,000) B—$250,000 = $400,000 – (10% x $1,500,000) C—$250,000 = $500,000 – (10% x $2,500,000) The residual incomes for each division were they to invest in the project: A—$50,000 = $200,000 – (10% x $1,500,000) B—$290,000 = $500,000 – (10% x $2,100,000) C—$290,000 = $600,000 – (10% x $3,100,000) Because the economic profit would increase for all three divisions, each would invest in the project, which is the desired action from a company-wide perspective.

9. [d] 10. [d] 11. [b] 12. [b] 13. [c] 14. [a] 15. [a] 16. [a]

Copyright ©2023 Pearson Education, Ltd.

15


Chapter 11 Capital Budgeting LEARNING OBJECTIVES: When your students have finished studying this chapter, they should be able to: 1. 2. 3. 4. 5. 6. 7. 8. 9.

Describe capital-budgeting decisions and use the net-present-value (NPV) method to make such decisions. Use sensitivity analysis to evaluate the effect of changes in predictions on investment decisions. Calculate the NPV difference between two projects using both the total project and differential approaches. Identify relevant cash flows for NPV analyses. Compute the after-tax net present values of projects. Explain the after-tax effect on cash received from the disposal of assets. Use the payback model and the accounting rate-of-return model and compare them with the NPV model. Reconcile the conflict between using an NPV model for making decisions and using accounting income for evaluating the related performance. Compute the impact of inflation on a capital-budgeting project (Appendix 11).

Copyright ©2023 Pearson Education, Ltd.

160


CHAPTER 11:

ASSIGNMENTS

CRITICAL THINKING EXERCISES 25. Investment in R&D 26. Business Valuation and NPV 27. Replacement of Production Facilities 28. Capital Budgeting, Taxes, and Ethics EXERCISES 29. Exercise in Compound Interest 30. Exercise in Compound Interest 31. Exercise in Compound Interest 32. Basic Relationships in Interest Tables 33. PV and Moratorium 34. Simple NPV 35. NPV Relationships 36. New Truck 37. Present Values of Cash Inflows 38. Effect on Required Rate 39. NPV and IRR 40. Sensitivity Analysis 41. NPV and Sensitivity Analysis 42. Depreciation, Income Taxes, Cash Flows 43. After-Tax Effect on Cash 44. MACRS Depreciation 45. Present Value of MACRS Depreciation 46. NPV, ARR, and Payback 47. Weakness of the Payback Model 48. Comparison of Capital-Budgeting Techniques 49. Inflation and Capital Budgeting 50. Sensitivity of Capital Budgeting to Inflation PROBLEMS 51. Replacement of Office Equipment 52. Installation of a Computerized Distribution System 53. Discounted Cash Flow, Uneven Revenue Stream, Relevant Costs 54. Investment in Machine 55. Replacement Decision 56. Minimization of Transportation Costs Without Income Taxes 57. Straight-Line Depreciation, MACRS Depreciation, and Immediate WriteOff 58. MACRS, Residual Value 59. Purchase of Equipment, MACRS 60. MACRS and Non-residential Real Estate 61. PV of After-Tax Cash Flows, Payback, and ARR 62. Investment Justification Analysis and Graphs

Copyright ©2023 Pearson Education, Ltd.

161


63. 64. 65. 66. 67. CASES 68. 69. 70. 71. 72.

Fixed and Current Assets; Evaluation of Performance Investment Before and After Taxes After-Tax NPV Minimization of Transportation Costs After Taxes, Inflation Inflation and Nonprofit Institution

Investment in CAD/CAM Investment in Technology Investment in Quality Make or Buy and Replacement of Equipment Nike 10k: Nike Capital Budgeting with NPV

EXCEL APPLICATION EXERCISE 73. Net Present Value and Payback Period for a Purchase Decision COLLABORATIVE LEARNING EXERCISE 74. Capital Budgeting, Sensitivity Analysis, and Ethics INTERNET EXERCISE 75. Capital Budgeting at Carnival Corporation (http://www.carnivalcorp.com)

Copyright ©2023 Pearson Education, Ltd.

162


CHAPTER 11: I.

OUTLINE

Capital Budgeting for Programs or Projects Capital Budgeting—long-term planning for making and financing investments that affect financial results over more than just the next year. This chapter concentrates on the planning and controlling of programs or projects that affect more than one year’s financial results. The investments required for programs or projects are often called capital outlays. Accountants are information specialists (i.e., gather and interpret information). Capital budgeting has three phases: 1. 2. 3.

II.

Identifying potential investments, choosing which investments to make, and follow-up monitoring of the investments.

Discounted-Cash-Flow Models Discounted-Cash-Flow (DCF) Models—focus on a project’s cash inflows and outflows while taking into account the time value of money. DCF models are used by the majority of the large industrial firms in the United States and are the best measures of the financial effects of an investment. A.

Major Aspects of DCF DCF models focus on expected cash inflows and outflows rather than on net income and are based on the theory of compound interest. A brief summary of the tables and formulas used is included in APPENDIX B at the end of the book.

B.

Net Present Value (NPV) Net-Present-Value (NPV) Method—a DCF approach to capital budgeting that computes the present values of all expected future cash flows using a minimum desired rate of return. The Required Rate of Return, Hurdle Rate, Discount Rate, or Cost of Capital—the minimum desired rate of return. The manager selects it based upon the project’s risk level. If the sum of the present values of all the expected cash flows is positive, the project is desirable and vice versa. A zero NPV leaves the decision maker indifferent between accepting and rejecting the project (i.e., break even).

C.

Applying the NPV Method

{L. O. 1}

See EXHIBIT 11-1 for an example of the three steps in applying the NPV method: 1. 2.

Prepare a diagram of relevant expected cash inflows and outflows (right-hand side of EXHIBIT 11-1). Find the present value of each expected cash inflow or outflow (see

Copyright ©2023 Pearson Education, Ltd.

163


3. D.

APPENDIX B). Sum the individual present values.

Choice of the Correct Table Students should be shown the relationship between and proper use of the tables appearing in APPENDIX B. TABLE 1 should be used in discounting single amounts, whereas TABLE 2 is used for a series (i.e., annuities) of equal amounts. The factors in TABLE 2 are simply summations of the factors from TABLE 1.

E.

Effect of Required Rate The required rate of return can have a large effect on NPVs. The higher the required rate of return, the lower the present value of each future cash inflow and the lower the NPV of the project. Investments that are desirable at one rate of interest may be undesirable at a higher rate of interest.

F.

Assumptions of the NPV Model First, we act as if the predicted cash inflows and outflows will occur at the times specified. Second, we assume perfect capital markets (i.e., borrow or lend money at the same interest rate—minimum desired rate of return for the NPV model). The use of DCF models also passes the cost-benefit test. Depreciation and NPV We are concerned with cash flows, not revenues and expenses. Depreciation is an expense that does not require a current cash outlay. The entire cost of an asset is typically a lump-sum outflow of cash at time zero. Deducting depreciation from operating cash flows would be a double counting of a cost that has already been considered a lump-sum outflow.

G.

Review of Decision Rules Net-Present-Value (NPV) Model—Comparison by expressing all amounts in today’s monetary units at time zero. 1. 2.

H.

Calculate the NPV using the minimum desired rate of return as the discount rate. If the NPV is positive, accept the project, and vice versa.

Internal Rate of Return (IRR) Model This model determines the interest rate at which the NPV equals zero. If IRR > minimum desired rate of return, then NVP > 0, accept project. If IRR < minimum desired rate of return, then NVP < 0, reject project.

Copyright ©2023 Pearson Education, Ltd.

164


I.

Real Options This model is a capital-budgeting model that recognized the value of contingent investments—that is, investments that a company can adjust as is learns more about their potential for success.

III.

Sensitivity Analysis and Risk Assessment in DCF Models

{L. O. 2}

Sensitivity Analysis—shows the financial consequences that would occur if actual cash inflows differed from those expected. This approach of incorporating risk in capital-budgeting decisions answers “what-if” questions concerning the values of NPV when the cash flows, useful life, or required rate of return are changed. The two major types of sensitivity analysis are: (1) comparing the optimistic, pessimistic, and most likely predictions and (2) determining the amount of deviation from expected values before a decision is changed. Sensitivity analysis shows how risky a project might be by showing how sensitive it is to change. IV.

The NPV Comparison of Two Projects A.

{L. O. 3}

Total Project Versus Differential Approach Total Project Approach—compares two or more alternatives by computing the total impact on cash flows of each alternative and then computing NPVs for each alternative. The project with the largest NPV of total cash flows is preferred. Differential Approach—two alternatives are compared by computing the differences in cash flows between two alternatives and then computing the NPV of those differences. The differential approach is limited to comparing two projects at a time, whereas the total project approach can be used for more than two alternatives. Cash inflows are positive and cash outflows are negative in an analysis. See EXHIBIT 112 for an illustration of the two methods for a keep or replace decision.

V.

Relevant Cash Flows B.

{L. O. 4}

Predicting Relevant Cash Flows Typical items to be included in an NPV analysis are: 1. Initial cash inflows and outflows—Included here are outflows for the purchase and installation of equipment and other items required by the new project, and either inflows or outflows from disposal of any items that are replaced such as their salvage value or costs of dismantling and discarding. 2. Investments in Working Capital—These are typically included as an outlay at time zero and are assumed to be recouped at the end of the project’s life. The difference between the initial outlay for working capital (mostly receivables and inventories) and the present value of its recovery is the present value of the cost of using working capital in the project. 3. Cash inflows and outflows at termination—The disposal value at the date

Copyright ©2023 Pearson Education, Ltd.

165


4.

B.

of termination of a project is an increase in the cash inflow in the year of disposal. Operating cash flows during the life of the project—The major purpose of most investments is to affect revenue or costs (or both). Only overhead costs that differ between alternatives should be included. Depreciation and book values should be ignored. A reduction in a cash outflow (cash savings) is treated the same as a cash inflow.

Cash Flows for Investments in Technology In comparing the cash flows predicted for a computer-integrated manufacturing system (CIM) with the status quo, the expected cash flows for the status quo manufacturing system should be adjusted for probable decreases in market share, and subsequently revenues, if others in the industry are making CIM investments. In addition, unanticipated cost savings such as the flexibility to make product mix changes easily and the ability to implement design changes quickly and cheaply should be considered. Finally, the difficult-to-predict revenue and cost effects of a CIM investment can be incorporated in the investment decision subjectively.

V.

Income Tax Effects

{L. O. 5}

Income taxes require cash disbursements. They can affect the amount and timing of cash flows. One of their roles in capital budgeting is to narrow the cash differences between projects. Cash savings from operations (i.e., inflows) cause an increase in taxable income, which creates partially offsetting increases in tax outflows. For example, a 40% income tax rate would reduce a $1 million cash operating savings to $600,000 because $400,000 of the $1 million would be paid in taxes. Marginal Income Tax Rate—the tax rate paid on additional pretax income. In capital budgeting, the company’s relevant tax rate is applied to additional cash inflows generated by a proposed project. A.

Effects of Depreciation Deductions One item that often differs between tax reporting and public reporting is depreciation. For public reporting purposes, depreciation spreads the cost of an asset over its useful life. Accelerated Depreciation—which charges a larger proportion of an asset’s cost to the earlier years and less to later years for tax purposes. See EXHIBIT 11-3 for an illustration of the interrelationships of income before taxes, income taxes, depreciation, and cash flows. The total after-tax effect on cash from an investment can be determined several ways. In one calculation, expenses other than depreciation are added to income taxes, which are then deducted from sales to obtain the after-tax cash. In another calculation, depreciation is added back to net income to arrive at after-tax cash. In a third method of computation, after-tax income, excluding the depreciation effects, is computed. Then, the tax savings generated because of the presence of depreciation are added to arrive at the total after-tax effect on cash.

Copyright ©2023 Pearson Education, Ltd.

166


Recovery Period—period over which an asset is depreciated for tax purposes. Depreciation of fixed assets creates future tax deductions. The present value (PV) of the deductions depends on their specific yearly effects on future income tax payments. Therefore, the PV is influenced by the recovery period, the depreciation method used, the tax rates, and the discount rate. EXHIBIT 11-4 shows a method of analyzing data for capital budgeting, assuming straight-line depreciation. This method separates the cash effects of operations with the cash effects of depreciation. B.

Timing of Depreciation Tax Deductions and Cash Flow Effects The after-tax cash flows from revenues and expenses other than depreciation are found by multiplying the pretax amounts by (1 – the tax rate). In contrast, the after-tax effects of noncash expenses (e.g., depreciation) are computed by multiplying the tax deduction by the tax rate itself. Note that this is treated as a cash inflow because it is a decrease in the tax payment. The total cash effect of a noncash expense is only the tax-savings effect. Two assumptions are that tax payments occur concurrently with related pretax cash flows and that the companies for which analysis is desired are profitable. The first assumption is realistic because companies pay estimated taxes throughout the year. The second assumption is necessary so that the tax benefits can be realized.

C.

Modified Accelerated Cost Recovery System (MACRS) Accelerated Depreciation Income tax laws allow the use of accelerated depreciation methods. Accelerated depreciation is any pattern of depreciation that writes off depreciable assets more quickly than does ordinary straight-line depreciation. The effect of using an accelerated depreciation method on a capital budgeting analysis is to increase the project’s NPV. This is the result of the tax savings occurring earlier in the life of the project and the fact that early-period cash flows are discounted less than late-period ones. See EXHIBIT 11-5 for examples of assets in the eight MACRS classes. See EXHIBIT 11-6 for MACRS depreciation schedules for recovery periods of three, five, seven, and ten years.

D.

Present Value of MACRS Depreciation Tax Deduction In capital-budgeting decisions, it is often useful to know the PV of the tax savings from depreciation. See EXHIBIT 11-7 for the PVs for $1 to be depreciated over MACRS schedules for 3-, 5-, 7-, and 10-year recovery periods. The PV of tax savings can be found in three steps: 1.

Find the factor from EXHIBIT 11-7 for the appropriate recovery period and

Copyright ©2023 Pearson Education, Ltd.

167


the required return. 2. Multiply the factor by the tax rate to find the tax savings per dollar of investment. 3. Multiply the result by the amount of the investment to find the total tax savings. F.

Tax Effects of Gains or Losses on Disposal at Termination

{L. O. 6}

The disposal of equipment for cash can also affect income taxes. Gains are taxed, reducing the net cash inflow from the sale of assets by the amount of the tax on the gain. Losses create tax savings, which can be added to the cash proceeds from the sale to obtain the net cash inflow from disposing of an asset. Although losses do result in tax savings and gains result in additional taxes, a company still has more net cash inflow from disposing of their assets when gains are realized. Second tax effect occurs when a company disposes of assets before the end of it recovery period. In addition to taxable gains or losses, disposal eliminates future tax depreciation. The roles of depreciation and book value are widely misunderstood when performing an analysis of the replacement of equipment. The following points summarize the role of depreciation for this type of decision (see EXHIBIT 11-8). 1. Initial investment: The amount paid for (and hence the depreciation on) old equipment is irrelevant, except for its effect on tax cash flows. However, the amount paid for new equipment is relevant because it is an expected future cost that will not be incurred if replacement is rejected. 2. Do not double-count: The investment in equipment is a one-time outlay at time zero. It should not be double-counted as an outlay in the form of depreciation. 3. Relation to income tax cash flows: The relevant item is the income tax effect, not the book value or the depreciation. The book value and depreciation are useful in predicting future income tax disbursements. VII.

Other Models for Analyzing Long-Range Decisions A.

{L. O. 7}

Payback Model Payback Time (or Payback Period)—measure of time it will take to recoup, in the form of cash inflows from operations, the initial outlay. In formula form: payback time =

initial investment /

equal annual cash inflows from operations

A weakness is that it does not measure profitability (i.e., the cash flows beyond the payback period) and it ignores the time value of money. A strength is that it provides a rough estimate of risk, especially in decisions involving areas of rapid technological

Copyright ©2023 Pearson Education, Ltd.

168


change. When uneven cash flows are present, the formula given above cannot be employed. Instead, a cumulative approach must be used. Cash flows are accumulated until an amount equaling the initial investment is obtained. Prorating cash flows in the last year of the payback is sometimes required. B.

Accounting Rate-of-Return Model Accounting Rate-of-Return (ARR) Model (or the accrual accounting rate-of-return model, the unadjusted rate-of-return model, the book-value model)—a non-DCF capital-budgeting model expressed as the increase in expected average annual operating income divided by the initial required investment. ARR =increase in expected average / initial required annual operating income investment It shows the effect of an investment on an organization’s financial statements. The ARR may also be computed using the average investment in the asset, in which case the rate doubles. Major weaknesses of the ARR model are that it ignores the time value of money and the timing of cash flows.

VIII. Performance Evaluation A.

{L. O. 8}

Potential Conflict Managers are frequently frustrated if they are instructed to use a DCF model for making decisions and are evaluated later by a non-DCF model, such as the typical accounting rate-of-return model, which is based on accounting income instead of cash flows. An illustration is provided which shows that the ARR can be low in the early years of an asset’s life (due to a high book value) and higher in later years. Unfortunately, managers expecting to move to new positions within the next few years will be reluctant to make equipment replacement decisions because they will not be around when the higher ARR figures occur. They are also reluctant to replace due to heavy book losses on replaced equipment (see discussion in CHAPTER 6).

B.

Reconciliation of Conflict Resolving the conflict between the use of DCF models for decision-making and nonDCF measures of performance can be accomplished using non-DCF models for decision making. However, the DCF models are superior analytical models. Another approach would be to use DCF models for both decision making and performance evaluation. Post-audits—a follow-up evaluation of capital-budgeting decisions. The purposes of post-audits include: 1. 2.

Seeing that investment expenditures are proceeding on time and within budget. Comparing actual cash flows with those originally predicted in order to

Copyright ©2023 Pearson Education, Ltd.

169


3. 4.

motivate careful and honest prediction. Providing information for improving future predictions of cash flows. Evaluating the continuation of the project.

Post-audits are costly to perform because financial information is usually collected, summarized, and reported for products, departments, divisions, and territories, not for projects. However, most large companies use post-audits to some degree. X.

APPENDIX 11: Capital Budgeting and Inflation

{L. O. 9}

In addition to taxes, capital-budgeting decision makers should consider the effects of inflation on their cash flow predictions. If significant inflation is expected over the life of a project, it should be specifically and consistently analyzed in a capital-budgeting model. A.

Watch for Consistency Adjustments for inflation in both the minimum desired rate of return and in the cashflow predictions should be included in an analysis. Nominal (or market) interest rates—quoted market interest rate that includes an inflation element. If used in a capital budgeting analysis, the cash flows should be adjusted for inflation. See EXHIBIT 11-9 for the correct and incorrect ways to analyze an investment when inflation is present. Note that the recommended course of action differs based on the two different NPVs that are calculated. If a real rate of interest were used, the cash flows would not need to be adjusted.

B.

Role of Depreciation Note that the cash savings from depreciation shown in EXHIBIT 11-9 are not affected by inflation because the amount of depreciation allowed by tax laws is based on an initial outlay at time zero. Critics claim that not allowing inflation adjustments for capital assets in computing annual depreciation discourages investment, whereas defenders of the existing U.S. tax laws assert that capital investment is encouraged in many other ways, such as the use of accelerated depreciation method.

Copyright ©2023 Pearson Education, Ltd.

170


CHAPTER 11:

Quiz/Demonstration Exercises

Learning Objective 1 1.

Accountants are usually involved in which phase(s) of capital budgeting? a. human resource decisions b. reviewing mechanical specifications c. identifying potential investments d. B and C e. A, B, and C

Use the following information for questions 2 and 3. Taylor Company is considering the purchase of some labor-saving equipment for its packaging department. The equipment is expected to result in labor cost savings of $40,000 per year for the expected five-year life of the equipment. The cost of the equipment is $110,000 and the desired rate of return is 6%. 2.

The NPV of the investment for Taylor Company is _____. a. $ 79,558 b. $ 27,434 c. $ 147,434 d. $ 45,000

Learning Objective 2 3.

By how much could the annual labor savings of the equipment described in the illustration above decrease for the project to be minimally acceptable? a. none at all, it is just barely acceptable now b. none at all, it is below the acceptable point already c. just over 18% of the present $30,000 per year labor savings d. just over 15% of the present $30,000 per year labor savings

4.

Sensitivity analysis allows a manager to answer “what-if” questions about changes in _____. a. useful life b. cash flows c. risk d. B and C e. A, B, and C

Learning Objective 3 Use the following information for questions 5 and 6. Turner Corporation is considering the replacement of some equipment by a more efficient,

Copyright ©2023 Pearson Education, Ltd.

171


technologically advanced model. The new equipment costs $100,000, but the vendor has agreed to provide a trade-in on the existing equipment of $25,000. The present equipment has a remaining useful life of four years and the new equipment would be retired at the end of its fourth year of service. Given the expected level of future operations, the existing generating equipment’s operating costs are predicted to run $40,000 per year. The new equipment is expected to result in operating costs of $20,000 per year. The current equipment would have a $10,000 salvage value at the end of its useful life, whereas the proposed equipment’s salvage value is estimated to be $20,000. Turner’s minimum desired rate of return on investments is 10%. 5.

In using the total project approach, the NPV of replacing the existing equipment is _____. a. ($163,398) b. ($158,345) c. ($151,665) d. ($149,788)

6.

The NPV difference between the two projects using the differential approach is _____. a. $1,927 in favor of replacing the present equipment b. $5,035 in favor of keeping the generating equipment c. $11,733 in favor of replacing the generating equipment d. $6,680 in keeping the present equipment

Learning Objective 4 7.

When analyzing relevant cash flows for NPV, which of the following should be considered? a. initial cash inflows and outflows at time zero b. investments in receivables and inventories c. fixed overhead d. A and B e. None of above

8.

Which of these cash flows happens over a period of time? a. disposal value b. operating cash flows c. initial cash inflows d. none of the above

Learning Objective 5 Use the following information for questions 9 through 13. National Manufacturing Company is considering buying some new equipment that would allow for increased sales of its product. The incremental impact of the proposed $400,000 investment is shown below using straight-line depreciation and an expected useful life of four years for the equipment. The company has a minimum desired rate of return of 14%.

Copyright ©2023 Pearson Education, Ltd.

172


Revenues Nondepreciation expenses Depreciation Total expenses Taxable income Income tax (40%) Net Income

$500,000 $300,000 100,000 $400,000 $100,000 40,000 $ 60,000

9.

The annual cash inflows expected from the project are _____. a. $ 100,000 b. $ 40,000 c. $ 120,000 d. $ 140,000

10.

The present value of the tax savings from straight-line depreciation is _____. a. $ 100,000 b. $ 116,548 c. $ 81,584 d. $ 174,822

11.

The NPV of the investment using straight-line depreciation is _____. a. ($ 280,000) b. $ 66,192 c. $ 349,644 d. $ 116,548

12.

If the investment was allowed to depreciate over a three-year recovery period and the doubledeclining-balance method of depreciation was used, the present value of the tax savings from depreciation would be _____. a. $ 58,274 b. $ 66,464 c. $ 132,928 d. $ 280,000

13.

Using the DDB depreciation and a three-year recovery period to compute the tax savings from depreciation results in _____. a. a higher NPV, and is still positive to make the investment desirable b. a lower NPV that would suggest that the investment should be made c. an even lower NPV of the investment than resulted using the straight-line method d. the same NPV as was computed when using straight-line depreciation

Learning Objective 6 Use the following information for questions 14 and 15.

Copyright ©2023 Pearson Education, Ltd.

173


Ultra Corporation is considering the replacement of a piece of equipment that it bought three years ago for $150,000. At the time of purchase, the equipment was expected to have a useful life of five years. Ultra, whose tax rate is 40%, uses straight-line depreciation. 14.

If Ultra is able to sell the equipment for $70,000, the net cash flows from the sale are _____. a. $ 10,000 b. $ 60,000 c. $ 66,000 d. $ 56,000

15.

If Ultra is able to sell the equipment for $25,000, the net cash flows from the sale are _____. a. $ 35,000 b. $ 39,000 c. $ 60,000 d. $ 90,000

16.

Depreciation affects capital budgeting decisions by _____. a. creating tax savings in the amount of the annual depreciation b. reducing cash flows provided by projects c. increasing cash flows by a dollar for each dollar of depreciation claimed d. creating tax savings in the amount of the tax rate multiplied by the depreciation claimed

Learning Objective 7 17.

The payback model and the accounting rate-of-return model are _____. a. impossible to apply, and therefore are never used b. alternative models that may be used in making capital-budgeting decisions that ignore the time value of money c. two excellent DCF models d. never used because they are just too simple

18.

A possible solution to the problem of reconciling the conflict between making decisions using a DCF model and using accounting income for measuring the resulting performance is to _____. a. make decisions with accounting-based decision models such as the accounting rate-of-return in the first place b. conduct post audits to compare actual with predicted cash flows for the DCF models rather than basing evaluations of projects on accounting measures c. do either A or B d. neither A or B

Learning Objective 8

Copyright ©2023 Pearson Education, Ltd.

174


19.

DCF models may influence managers to make decisions that only benefit the _____. a. short-run time period b. long-run time period c. neither A and B d. both A nor B

20.

The purposes of a post-audit do not include _____. a. seeing that investment decisions are proceeding on time b. providing information for improving prior predictions of cash flows c. comparing actual cash flows with predicted cash flows d. evaluating the continuation of the project

Learning Objective 9 21.

If a market interest rate, which includes an inflation component, is used by a company as its minimum required rate of return in a net present value analysis, the _____. a. expected cash flows from operations must be adjusted for inflation for the analysis to be consistent b. expected cash flows from operations must not be adjusted for inflation for the analysis to be consistent c. project will automatically be unacceptable d. expected tax savings from depreciation must be adjusted for inflation for the analysis to be consistent

22.

If a real interest rate, which does not include an inflation component, is used by a company as its minimum required rate of return in a net present value analysis, the _____. a. project will automatically be unacceptable b. expected cash flows from operations must be adjusted for inflation in order for the analysis to be consistent c. expected cash flows from depreciation must be adjusted for inflation in order for the analysis to be consistent d. expected tax savings from operations must not be adjusted for inflation in order for the analysis to be consistent

Copyright ©2023 Pearson Education, Ltd.

175


CHAPTER 11: 1. [a] 2. [b]

3. [c]

4. [d] 5. [b]

Solutions to Quiz/Demonstration Exercises

The NPV is found by subtracting the initial investment required from the present value of the future cash inflows. In this case, $45,000 annual cash inflow x 4.2124 (the interest factor from Table 2 for 5 years at 6%) = $147,434. $189,558 – $110,000 = $79,558. The level of annual cash inflows that makes the packaging equipment investment minimally acceptable is $28,487.32. This can be found by dividing the required present value of the cash inflows to make the project minimally acceptable (NPV = $0) of $120,000 by the Table 2 annuity present-value factor of 4.2124. The reduction of $6,512.68 is just over 18% of the initial $35,000 annual savings. The computations for each alternative using the total project approach are:

Item KEEP Operating Costs Salvage Value NPV

PV factor

P Value Period 0 Period 1 Period 2 Period 3 Period 4

3.1699 ($158,495) .6830 $ 6,830 ($151,665)

($50,000) ($50,000) ($50,000) ($50,000) $10,000

Item PV factor P Value Period 0 Period 1 Period 2 Period 3 Period 4 REPLACE Net Initial Outlay 1.0000 ($100,000) ($100,000) Operating Costs 3.1699 ($ 63,398) ($20,000) ($20,000) ($20,000) ($20,000) Salvage Value .6830 $ 13,660 $20,000 NPV ($149,738) 6. [c]

Using the differential approach, from the KEEP perspective the NPV analysis is:

Item PV factor P Value Period 0 Period 1 Period 2 Period 3 Period 4 Net Initial Outlay 1.0000 $100,000 $100,000 Operating Costs 3.1699 ($ 95,097) ($30,000) ($30,000) ($30,000) ($30,000) Salvage Value .6830 ($ 6,830) ($10,000) NPV ($ 1,927) 7. [e] 8. [a] 9. [d]

10. [c]

The easiest way to compute the cash flows generated by the investment is to add the depreciation back to the net income. In this case, $100,000 + $60,000 = $160,000. Alternatively, one could add the after-tax cash flows from operations to the tax savings resulting from the depreciation expense. Here, $120,000 [($500,000-300,000) x (1.40)] + $40,000 [$100,000 x .40] also gives $160,000 in total after-tax cash flows. The annual tax savings from depreciation are $40,000 [$100,000 x .40]. The present value of the savings is computed by multiplying the $40,000 by the present value

Copyright ©2023 Pearson Education, Ltd.

176


11. [d]

factor for an annuity for four years at 14% of 2.9137. The result is $116,548. The computations are shown below. The after-tax cash flows from operations are $120,000 per year [($500,000 – $300,000) x (1 – .40)], whereas the annual tax savings from depreciation are given above.

Item After-tax operating cash flow increase Depreciation tax savings Initial investment Net Present Value

PV factor

PV

Period 0 Period 1 Period 2 Period 3 Period 4

2.9137 349,644 120,000 120,000 120,000 120,000 2.9137 116,548 40,000 40,000 40,000 40,000 1.0000 ($400,000) ($400,000) $ 66,192

12. [d]

The savings from depreciation using DDB and a three-year life can be found by multiplying the depreciation expense by the tax rate and then multiplying the resulting amount by present value factor: Year 1 $400,000 x 2/3 = 266,667 x .40 = $106,667 x .8772 = $ 93,568 Year 2 $133,333 x 2/3 = 88,889 x .40 = $ 35,556 x .7695 = $ 27,360 Year 3 $ 44,444 – 0 = 44,444 x .40 = $ 17,778 x .6750 = $ 12,000 Total $ 132,928

13. [b]

Using the data from questions 11 and 12 above, the NPV would be $82,572, which makes the investment even more acceptable. The $82,572 can be found by adding the difference in the tax savings from depreciation, $16,380 [$132,928 – $116,548], to the NPV found using straight-line depreciation, $66,192.

14. [d]

The $70,000 from the sale must be reduced by the $4,000 tax on the gain from selling the equipment. The book value was $60,000 [$150,000 – (3 x $30,000/yr.)] resulting in a gain of $10,000. $10,000 x .40 = $4,000 of tax on the gain. The $25,000 from the sale must be increased by the tax savings created by the loss from selling the equipment. The loss of $35,000 [$60,000 book value – $25,000 selling price] results in a $14,000 [$35,000 x .40] tax savings, creating total cash flows of $39,000.

15. [a]

16. [a] 17. [a] 18. [c] 19. [a] 20. [c] 21. [b] 22. [d]

Copyright ©2023 Pearson Education, Ltd.

177


Chapter 12 Cost Allocation LEARNING OBJECTIVES: When your students have finished studying this chapter, they should be able to: 1. 2. 3. 4. 5. 6. 7.

Describe the general framework for cost allocation. Allocate the variable and fixed costs of service departments to other organizational units. Use the direct and step-down methods to allocate service department costs to user departments. Allocate costs from producing departments to products or services using the traditional and ABC approaches. Allocate costs associated with customer actions to customers. Allocate the central corporate costs of an organization. Allocate joint costs to products using the physical-units and relative-sales-value methods.

Copyright ©203 Pearson Education, Ltd.

181


CHAPTER 12:

ASSIGNMENTS

CRITICAL THINKING EXERCISES 25. Allocation and Cost Behavior 26. Allocation and the Sales Function 27. Allocation and Marketing EXERCISES 28. Allocation of Computer Costs 29. Fixed- and Variable-Cost Pools 30. Sales-Based Allocations 31. Direct and Step-Down Allocations, Activity-Based Allocation 32. Direct and Step-Down Allocations 33. Customer Profitability: Strategy 34. Joint Costs 35. Joint Costs 36. By-Product Costing PROBLEMS 37. General Framework of Allocation, Service Departments, ABC, Customer Profitability, and Process Maps (EXHIBIT 12-21) 38. Allocation of Automobile Costs 39. Allocation of Costs 40. Service Department Allocation and ABC, Product Costing 41. Service Department Allocation and ABC; Customer Profitability 42. Customer Profitability at a Distributor (EXHIBIT 12-22) 43. Customer Profitability and Allocation of Costs to Service 44. Medical Equipment 45. Direct Method for Service Department Allocation 46. Step-Down Method for Service Department Allocation 47. ABC Allocations; Process Map; What-if Analysis (EXHIBIT 12-23) 48. Activity-Based Allocations 49. Allocation of Central Costs 50. Joint Costs and Decisions CASES 51. 52. 53.

Customer Profitability (EXHIBITs 12-24, -25, -26) Allocation of Data Processing Costs (EXHIBIT 12-27) Nike 10-K Problem: Allocation of Corporate Expenses

EXCEL APPLICATION EXERCISE 54 Allocating Costs Using Direct and Step-Down Methods COLLABORATIVE LEARNING EXERCISE 55. Library Research on ABC and Customer Profitability

Copyright ©203 Pearson Education, Ltd.

182


INTERNET EXERCISE 56. Cost Allocation at Transformco (https://transformco.com/)

Copyright ©203 Pearson Education, Ltd.

183


CHAPTER 12: I.

OUTLINE

A General Framework for Cost Allocation

{L. O. 1}

Cost allocation is fundamentally a problem of linking (1) some cost or groups of costs with (2) one or more cost objectives (e.g., products, departments, and divisions). Ideally, cost allocation should assign each cost to the cost objective that caused it. Linking of costs with cost objectives is accomplished by selecting cost drivers (i.e., activities that cause costs). CostAllocation Base—a cost driver when it is used for allocating costs. Cost Pool—a group of individual costs that is allocated to cost objectives using a single cost driver. Several terms are used to describe the process of assigning costs to cost objectives. Terms frequently used include allocate, apply, absorb, reallocate, trace, assign, distribute, redistribute, load, burden, apportion, and reapportion. See EXHIBIT 12-1. There are two types of departments: 1) producing departments, where employees work directly on the organization’s products or services, and 2) service departments (e.g., personnel department and facility management department), which exist only to support other departments or customers. Some service department activities (e.g., order processing and customer service) support customers rather than the production process. II.

Allocation of Service Department Costs A.

{L. O. 2}

General Guidelines The preferred guidelines for allocating service departments are:

B.

1.

Allocate variable- and fixed-cost pools separately (sometimes called the dual method of allocation). Note that one service department (e.g., a computer department) can contain multiple cost pools if more than one cost driver causes the department's cost. At a minimum, there should be a variable-cost pool and a fixed-cost pool.

2.

Establish the details regarding cost allocation in advance of rendering the service rather than after the fact. This approach establishes the “rules of the game” so that all departments can plan appropriately.

3.

Evaluate performance using budgets for each service (staff) department, just as they are used for each production or operating (line) department. The performance of a service department is evaluated by comparing actual costs with a budget, regardless of how the costs are later allocated. From the budget, variable-cost pools and fixed-cost pools can be identified for use in allocation.

Variable-Cost and Fixed-Cost Pools Variable costs should be allocated as follows: budgeted unit rate x actual hours of cost driver

Copyright ©203 Pearson Education, Ltd.

184


The use of budgeted cost rates rather than actual cost rates for allocating variable costs of service departments protects the using departments from intervening price fluctuations and inefficiencies in the service departments. When an organization allocates actual total service department costs, it holds user department managers responsible for costs beyond their control and provides less incentive for service departments to be efficient. Fixed-Cost Pool The cost driver for the fixed-cost pool is the amount of capacity required when the service department was instituted. Therefore, fixed costs should be allocated as follows: budgeted percent of capacity × total budgeted fixed costs available for use The predetermined lump-sum approach is based on the long-run capacity available to the user, regardless of actual usage from month to month. The level of fixed costs is affected by long-range planning regarding the overall level of service and the relative expected usage, not by short-run fluctuations in service levels and relative actual usage. A major strength is that a user department's allocation is not affected by the actual usage of other user departments. C.

Budgeted Cost-Allocation Rates If fixed costs are allocated based on long-range plans, there is a natural tendency on the part of consumers to underestimate their planned usage and thus obtain a smaller fraction of the cost allocation. Top management can counteract these tendencies by monitoring predictions, and by following up and using feedback to keep future predictions more honest. In addition, rewards may be given for accurate predictions and penalties set (e.g., through higher charges) for usage above that predicted. (see EXHIBIT 12-2)

D.

Allocating Fixed Costs Based on Capacity Available Allocation will be based on the long-run capacity available to the user regardless of the actual usage.

E.

Allocating Service Department Costs to Producing Departments

{L. O. 3}

Service departments often support other service departments in addition to producing departments. See EXHIBIT 12-3 for relevant data in regard to an example, which is used to demonstrate two popular methods for allocating service department costs: the direct method and the step-down method. 1.

Direct Method—ignores other service departments when any given service department’s costs are allocated to the revenue-producing (operating) departments. The costs of operating the service departments are allocated

Copyright ©203 Pearson Education, Ltd.

185


directly to operating departments with no intermediate allocations for the services provided to other service departments. 2.

Step-Down Method—recognizes that some service departments support the activities in other service departments as well as those in production departments. A sequence of allocations is chosen, usually by starting with the service department that renders the greatest service (as measured by costs) to the greatest number of other service departments. The last service department in the sequence is the one that renders the least service to the least number of other service departments. Once a department’s costs are allocated to other departments, no subsequent service department costs are allocated back to it. See EXHIBIT 12-4 for an illustration of the application of the step-down allocation method for the text example.

3.

Comparison of the Methods See EXHIBIT 12-5 for a comparison of the costs ultimately allocated to the producing departments. The method of allocation can greatly affect the amounts distributed to different producing departments. If significant differences are not generated, companies typically use the direct method due to its simplicity. A third method, the Reciprocal Method, provides the most theoretical accuracy because it fully realizes reciprocal services by service departments to each other. With this method, simultaneous equations and linear algebra are used to solve for the impact of mutually interacting services. Due to the difficulty managers have in understanding the application of this method, it is rarely used in practice. This method is described in a footnote in the text.

4.

Costs Not Related to Cost Drivers The examples used in the text thus far have assumed that the costs in a given service department were caused by a single cost driver. The costs were then allocated using this single cost driver. If some costs in the service department are not related to a single cost driver, three alternative methods of cost allocation should be considered. a.

b.

Identify additional cost drivers. Divide the costs in the service department into two or more cost pools and use a different cost driver to allocate the costs in each pool. Divide the service department costs into two cost pools, one with costs that vary in proportion to the cost driver (variable costs), and one with costs not affected by the cost driver (fixed costs). Allocate the former using the direct or step-down method, but do not allocate the latter. Costs not allocated are period costs for the organization and are not regarded as a cost of a particular production department.

Copyright ©203 Pearson Education, Ltd.

186


c.

Allocate all costs by the direct or step-down method using a single cost driver. This assumption implicitly assumes that, in the long run, the cost driver causes all of the service department’s costs, even if a short-term causal relationship is not easily identifiable.

Copyright ©203 Pearson Education, Ltd.

187


IV.

Allocation of Costs to Product or Service Cost Objects

{L. O. 4}

So far the chapter has discussed allocations of costs to departments or segments of an organization. Cost allocation is often carried one step further, to the outputs (e.g., products, parts, services) of these departments. Cost Application (or cost attribution)—the allocation of total departmental costs to the revenue-producing products or services. A.

A Traditional Approach (see EXHIBITs 12-6, 12-7, and 12-8) 1.

2. 3.

4.

V.

Divide the costs in each producing department, including both the direct department costs and all the costs allocated to it, into two categories: 1) the direct costs that you can physically trace to the final cost objectives, and 2) the remainder, the indirect costs. Assign the direct costs to the appropriate products, services, or customers. Select one or more cost pools and related cost drivers in each production department, and assign all the indirect departmental costs to the appropriate cost pool. Allocate (apply) the total costs accumulated in Step 1 to products or services that are the outputs of the operating departments, using the cost drivers specified in Step 3. If only one cost driver is used, two cost pools should be maintained: one for variable costs and one for fixed costs. Variable costs should be allocated based on actual cost-driver activity. Fixed costs should either remain unallocated or be allocated based on budgeted cost-driver activity.

An ABC Approach In the past, companies used direct-labor hours to apply the costs of departments to units of product. However, direct-labor hours are not a very good measure of the cause of costs in modern, highly automated departments. As a result, companies are implementing activitybased costing (ABC) to develop measures that better reflect the consumption of resources and related costs in their environment by accumulating costs into key activities. Both direct-labor hours and machine hours are volume measures. If many costs are caused by non-volume-based cost drivers, ABC should be considered. As Chapter 4 states, ABC is a system that first accumulates the costs of each activity of an organization and then applies the costs of activities to the products, services, or other cost objects using appropriate cost drivers. The ABC system takes one large overhead cost pool and breaks it down into several pools, each associated with a key activity. The goal of activity-based costing is to trace the costs to products or services instead of arbitrarily allocating them. Although it is relatively easy to trace direct material and labor to products using physical measures, advocates of ABC maintain that, by using appropriate cost drivers, many manufacturing overhead costs can also be physically traced to products or services.

Copyright ©203 Pearson Education, Ltd.

188


Illustration of Activity-Based Costing Approach in Manufacturing The text provides an illustration of an ABC system for a manufacturer using the four-step procedure that was introduced in Chapter 4. a. b.

c. d.

V.

First, the costing objective is to determine the costs of custom and standard displays for H.L. Display Company. Next, the interrelationships among activities, cost objectives, and resources were determined based on interviews with key personnel, and a process-based map representing the flow of activities, resources, and their interrelationship was developed. See EXHIBIT 12-9 for the process-based map. Using the process map as a guide, the accountants then collected the required cost and operational data via further interviews. Finally, EXHIBITs 12-10 and 12-11 presents the key results of the activity-based costing study.

Allocation of Costs to Customer Cost Objects to Determine Customer Profitability {L. O. 5} Customer profitability depends on more than the gross margin on the products or services they buy. It also depends on the costs incurred to fulfill customer orders and then provide other customer services such as order changes, returns, and expedited scheduling or delivery (see EXHIBIT 12-12). The following list is a profile of low and high cost-to-serve customers. Low Cost To Serve Large order quantity Few order changes Little pre- and post-sales support Regular scheduling Standard delivery Few returns A.

High Cost To Serve Small order quantity Many order changes Large amounts of pre- and post-sales support Expedited scheduling Special delivery requirements Frequent returns

Measuring and Managing Customer Profitability If service-department costs are assigned to producing departments and then to products, the allocation to products would be based on production-related output measures that may have little relationship to the cause of customer-service costs. An example in the text (Cedar City Distributors) is given to demonstrate this important concept (see EXHIBITS 12-13, 12-14, 12-15, 12-16, 12-17, and 12-18).

VI.

Allocation of Central Corporate Support Costs

{L. O. 6}

Whenever possible, the preferred cost driver for central services is usage, either actual or estimated. For some central services (e.g., data processing, advertising, and operations research), usage appears to be a reasonable basis to allocate costs. For others (e.g., public relations, top corporate management overhead, a real estate department, and a corporate

Copyright ©203 Pearson Education, Ltd.

189


planning department), usage seems an inappropriate base. For these types of costs, companies frequently use revenues as the cost driver, which represents an “ability to bear” philosophy rather than portraying any cause-and-effect relationship. A.

Use of Budgeted Sales for Allocation If the costs of central services are to be allocated based on sales, even though the costs do not vary in proportion to sales, the use of budgeted sales is preferable to the use of actual sales. At least this method means that the short-run costs of a given consuming department will not be affected by the fortunes of other consuming departments.

VII.

Allocation of Joint Costs and By-Product Costs A.

{L. O. 7}

Joint Costs Sometimes inputs are added to the production process before individual products are separately identifiable (i.e., before the split-off point). The costs of these inputs (e.g., materials, labor, and overhead costs) are called joint costs. Although allocations of these costs to the products, which emerge from the joint process, should not affect decisions regarding whether to process the products further, allocations are routinely made for inventory valuation and income determination purposes. Two conventional ways of allocating joint costs to products are widely used: physical units and relative sales values. They allocate the joint costs to the joint products in proportion to their number of physical units or sales dollars generated by the joint products. A twist on the relative-sales-value method is necessary when a joint product cannot be sold at the split-off point. Therefore, the sales value is approximated using: sales value at split-off = final sales value - separable costs

B.

By-Product Costs By-Product—a product that, like a joint product, is not individually identifiable until manufacturing reaches a split-off point. By-products differ from joint products because they have relatively insignificant total sales values in comparison with other products emerging at split-off (e.g., glycerin from soap making and mill ends of cloth and carpets). If an item is accounted for as a by-product, only separable costs are allocated to it. All joint costs are allocated to the main products. Any revenues from by-products, less their separable costs, are deducted from the cost of the main products.

Copyright ©203 Pearson Education, Ltd.

190


CHAPTER 12:

Quiz/Demonstration Exercises

Learning Objective 1 1.

Major purposes for allocating costs are _____. a. to predict the economic effects of planning and control decisions b. to obtain desired motivation c. to compute income and asset valuations d. to justify costs or obtain reimbursement e. all of these

2.

Which of the following purposes of allocation relate to planning and control? a. obtain desired motivation and compute income and asset valuations b. predict economic consequences and justify costs c. obtain economic consequences and justify costs d. compute income valuations and obtain reimbursement

Learning Objective 2 Use the following information for questions 3 and 4. The city of Mars leases a photocopy machine, which it uses in its Copy Services Department for $2,500 per month plus 4¢ per copy made. In addition to the lease costs, operating costs for toner, paper, operator salaries, and so on are variable at 7¢/copy. All departments of the city combined estimated that they would make 70,000 copies per month. The Recreation Department estimated that they would make 10,000 copies per month on average. In May, the Recreation Department made 12,000 copies and the total number of copies made by Copy Services for the month was 58,000. 3.

Following the guidelines of allocating variable and fixed costs of service departments separately, the variable costs of the Copy Services Department that should be allocated to the Recreation Department in June are _____. a. $1,320 b. $840 c. $1,130 d. $480 e. some other amount

4.

Following the guidelines of allocating variable and fixed costs of service departments separately, the fixed costs of the Copy Services Department that should be allocated to the Recreation Department in June are _____. a. $0 b. $200 c. $2,500 d. $357

Learning Objective 3 Use the following information for questions 5 and 6. The Francis Corporation operates two service and two producing departments in its production of go carts. The budgeted direct costs and other pertinent data for an upcoming month follow.

Copyright ©203 Pearson Education, Ltd.

191


Service Departments Maintenance Personnel

Production Departments Tooling Assembly

Direct costs $144,000 $80,000 Machine hours # of employees 20 16

$280,000 $320,000 30,000 20,000 60 100

Personnel costs are allocated based on the number of employees, and maintenance costs are allocated based on machine hours. 5.

The amount of maintenance costs allocated to the Assembly Department using the direct method of cost allocation would be _____. a. $32,000 b. $48,000 c. $86,400 d. $57,600

6.

The amount of maintenance costs (to the nearest dollar) allocated to the Tooling Department using the step-down method would be _____. a. $91,733 b. $54,000 c. $86,400 d. $8,888

Learning Objective 4 7.

The traditional approach to allocation of costs to the final cost objects focuses on _____. a. accumulating costs within producing departments and then allocating producing departments costs; and finally to products, services, or customers b. accumulating costs within departments and then allocating departmental costs to departments, and finally to products, services, or customers c. accumulating costs by products, services, or customers and deriving a cost per unit d. none of the above

Learning Objective 5 8.

Which of the following is a profile of a high cost-to-serve customer? a. large order quantity b. standard delivery c. frequent returns d. regular scheduling e. none of the above Use the following information for questions 9 and 10. Bally Inc. has four categories of overhead. The four categories and expected overhead costs for each category for next year are: Inspection Maintenance Materials Handling

Copyright ©203 Pearson Education, Ltd.

$ 30,000 60,000 9,000

192


Setups

8,000

Currently, overhead is applied using a predetermined overhead rate based upon budgeted direct labor hours, and 20,000 direct labor hours are budgeted for next year. The company has been asked to submit a bid for a proposed job. The company bases its bids on full manufacturing costs. Estimates for the proposed job are as follows: Direct materials Direct labor (400 hours) Number of material moves Number of setups

10 5

$ 2,000 4,000

Number of inspections 2 Number of machine hours

40

In the past, full manufacturing cost has been calculated by allocating overhead using a volume-based cost driver, direct labor hours. Expected activity for the four activity-based cost drivers that would be used is: Machine hours Setups

5,000 200

Material moves Quality inspections

600 1,000

9.

If direct labor hours were used as the cost driver, the total cost of the proposed job would be _____. a. $5,140 b. $6,000 c. $10,280 d. $8,140

10.

If the activity-based cost drivers were used to assign overhead, the total cost of the proposed job would be _____. a. $890 b. $8,140 c. $6,890 d. $10,280

Learning Objective 6 11.

A method of allocating central costs of an organization to divisions, which clearly fails to demonstrate a cause-and-effect relationship, is to _____. a. allocate based on the actual usage of the service b. allocate on the basis of sales dollars c. allocate based on the estimated usage of the service d. do none of these

12.

Which of the following is an example of a central service? a. public relations b. legal services c. accounting d. advertising e. all of the above

Learning Objective 7

Copyright ©203 Pearson Education, Ltd.

193


Use the following for questions 13 and 14. ABC Co., produces two products through a single manufacturing process. Each batch of product results in 400 pounds of product X and 600 pounds of product Y. The process requires materials, labor, and manufacturing overhead costing $50,000 per batch. X sells for $30 per pound, whereas Y sells for $20 per pound. 13.

Using the physical units method of allocating joint production costs would result in an allocation to product X of: _____. a. $0 b. $30,000 c. $20,000 d. $50,000

14.

Using the relative sales value approach of allocating joint production costs would result in an allocation to product X of _____. a. $10,000 b. $30,000 c. $25,000 d.$40,000

Copyright ©203 Pearson Education, Ltd.

194


CHAPTER 12: 1. [e] 2. [c] 3. [a]

4. [d] 5. [d] 6. [a]

7. [b] 8. [e] 9. [d]

10. [c]

Solutions to Quiz/Demonstration Exercises

The allocation of variable costs: the variable portion of the lease of 4¢ per copy and the Copying Services variable operating costs of 7¢ per copy. The actual number of copies made (12,000) is multiplied by the variable cost of 11¢ per copy to give $1,320 allocated.

The $144,000 of maintenance cost is allocated based on machine hours. Assembly uses 40% [20,000 of 50,000 total machine hours] resulting in a $57,600 allocation. With the step-down method, Personnel costs are allocated first with $8,888.88 [$80,000 x (20/(20 + 60 + 100))] allocated to the Maintenance department. Then, of the $152,888.88 now in Maintenance, $91,733.33 [$152,888.88 x (30,000 / (30,000 + 20,000))] would be allocated to the Fabrication Department.

The total cost consists of direct material ($2,000), direct labor ($4,000), and applied overhead. The overhead rate is $5.35 per labor hour [($30,000 + $60,000 + $9,000 + $8,000)/20,000 direct labor hours]. Applying the $5.35 rate to 400 direct labor hours for the job gives $2,140 of overhead applied to this job. Adding this to the $2,000 direct materials and $4,000 direct labor gives $8,140 total cost. Rates are $12/machine hour for maintenance [$60,000/5,000], $15/move for materials handling [$9,000/600], $40/setup [$8,000/200], and $30/inspection [$30,000/1,000]: Maintenance (40 machine hours @ $12) $480 Materials handling (10 moves @ $15) 150 Setups (5 setups @ $40) 200 Inspections (2 @ $30) 60 Total overhead costs applied $890 Adding this to the $6,000 of materials and labor costs gives $6,890.

11. [b] 12. [e] 13. [c] 14. [c]

Based on physical units, X would be allocated 40% [400 pounds / (400 pounds + 600 pounds)] of the $50,000 of joint processing costs, or $20,000. Each product can be sold for $12,000. Product X has 400 pounds at $30 per pound, and product Y has 600 pounds at $20 per pound. Thus, the total sales value of the two products is $12,000, and each product would be allocated $25,000 [50% x $50,000 joint production costs].

Copyright ©203 Pearson Education, Ltd.

195


Chapter 13 Accounting for Overhead Costs LEARNING OBJECTIVES: When your students have finished studying this chapter, they should be able to: 1. 2. 3. 4. 5. 6. 7. 8. 9.

Compute budgeted factory-overhead rates and apply factory overhead to production. Determine and use appropriate cost-allocation bases for overhead application to products and services. Use normalized variable- and fixed-overhead application rates and explain the disposition of overhead variances. Compare variable- and absorption-costing systems. Construct an income statement using the variable-costing approach. Construct an income statement using the absorption-costing approach. Distinguish between product-costing and planning-and-control purposes in accounting for variable and fixed costs. Compute the production-volume variance and show how it should appear in the income statement. Reconcile variable- and absorption-costing operating income and explain why a company might prefer to use a variable-costing approach.

Copyright ©2023 Pearson Education, Ltd.

200


CHAPTER 13:

ASSIGNMENTS

CRITICAL THINKING EXERCISES 32. Relationship Between Cost-Allocation Bases and Factory Overhead 33. Cost Application in Service Firms 34. Accounting for Fixed Costs 35. Marketing Decisions and Absorption Costing 36. Evaluating Production Using the Production-Volume Variance 37. Absorption Costing and the Value Chain EXERCISES 38. Discovery of Unknowns 39. Discovery of Unknowns 40. Relationship Among Overhead Items 41. Underapplied and Overapplied Overhead 42. Disposition of Year-End Underapplied Overhead 43. Simple Comparison of Variable and Absorption Costing 44. Comparison Over Four Years 45. Variable and Absorption Costing 46. Comparison of Production-Volume Variance 47. Reconciliation of Variable-Costing and Absorption-Costing Operating Income 48. Overhead Variances 49. Variances PROBLEMS 50. Choice of Cost-Allocation Base at Enriquez Machine Parts Company 51. Choice of Cost-Allocation Base at Enriquez Machine Parts Company 52. Choice of Cost-Allocation Base in Accounting Firm 53. Allocated Costs and Public Services 54. Overhead Accounting for Control and for Product Costing 55. Comparison of Variable Costing and Absorption Costing 56. All-Fixed Costs 57. Semifixed Costs 58. Absorption and Variable Costing 59. Absorption and Variable Costing 60. Fundamentals of Overhead Variances 61. Production-Volume Variance at Mobility Furnitures 62. Fixed Overhead and Practical Capacity 63. Selection of Expected Volume 64. Analysis of Operating Results 65. Standard Absorption and Standard Variable Costing 66. Disposition of Variances 67. Straightforward Problem on Standard Cost System 68. Straightforward Problem on Standard Cost System

Copyright ©2023 Pearson Education, Ltd.

201


CASES 69. 70. 71. 72. 73. 74.

Multiple Overhead Rates and Activity-Based Costing Inventory Measures, Production Scheduling, and Evaluating Divisional Performance Performance Evaluation Converting an Income Statement from Absorption Costing to Variable Costing Converting an Income Statement from Variable Costing to Absorption Costing Nike 10K Problem—Overhead Costs at Umbro

EXCEL APPLICATION EXERCISE 75. Computing Budgeted Factory Overhead COLLABORATIVE LEARNING EXERCISE 76. Accounting for Overhead INTERNET EXERCISE 77. Dell (http://www.dell.com)

Copyright ©2023 Pearson Education, Ltd.

202


CHAPTER 13: I.

OUTLINE

Accounting for Factory Overhead A.

{L. O. 1}

How to Apply Factory Overhead to Products Few companies wait until the actual factory overhead is finally known before computing the costs of products. Instead, they compute a budgeted (predetermined) overhead rate at the beginning of a fiscal year and use it to apply overhead costs as products are manufactured. This allows managers to have a close approximation of the costs of producing products continuously, not just at the end of the year.

B.

Budgeted Overhead Application Rates The following steps summarize how to account for factory overhead:

II.

1.

Select one or more cost drivers to serve as a base for applying overhead costs (e.g., direct-labor hours, direct-labor costs, machine hours, and production setups). The cost driver should be an activity that is the common denominator for systematically relating a cost or group of costs, such as machinery cost, set-up, and energy with products. The cost driver(s) should be the best measure of the cause-and-effect relationships between overhead costs and production volume.

2.

Prepare a factory-overhead budget for the planning period (ordinarily a year). The two key items are (1) budgeted overhead and (2) budgeted volume of the cost driver. There will be a set of budgeted OH costs and associated budgeted cost-driver level for each overhead. There may be just one set in businesses with simple production systems.

3.

Compute Budgeted Factory-Overhead Rate(s)—divide the budgeted total overhead for each cost pool by the budgeted cost driver level.

4.

Obtain actual cost-driver data (e.g., machine hours) as jobs are produced.

5.

Apply the budgeted overhead to the jobs by multiplying the budgeted rate(s) times the actual cost-driver data.

6.

At the end of the year, account for any differences between the amount of overhead actually incurred and overhead applied to products.

Illustration of Overhead Application Using the Enriquez illustration, the process of applying overhead to products is demonstrated. The budgeted overhead rate, which is used to apply the overhead, is computed as follows:

Copyright ©2023 Pearson Education, Ltd.

203


budgeted overhead application rate = total budgeted factory overhead / total budgeted amount of cost driver The total overhead applied is the result of multiplying actual cost driver by the budgeted overhead rate. A.

Choice of Cost-Allocation Bases

{L. O. 2}

Factory overhead is a conglomeration of manufacturing costs that, unlike direct materials or direct labor, cannot conveniently be applied on an individual job basis. Overhead is an integral part of a product’s total cost. Therefore, it is applied indirectly to products using a cost driver that is common to all jobs worked on and is the best available index of the product’s relative use of, and benefits from, the overhead items (i.e., a strong cause-and-effect relationship). Direct-labor costs or direct-labor hours have been a commonly used base for allocating overhead. In many cases, 80% of total overhead cost can be accounted for with just a few drivers (20% of the drivers identified). III.

Problems of Overhead Application A.

{L. O. 3}

Normalized Overhead Rates The Enriquez illustration used in the chapter has demonstrated the normal costing approach because the annual average overhead rate is used consistently throughout the year for product costing, without altering it from day to day and from month to month. The resultant “normal” product costs include an average or normalized chunk of overhead. As actual overhead costs are incurred by department, they are charged to the departments. On a weekly or monthly basis, these actual costs are then compared with budgeted costs to obtain budget variances for performance evaluation. This control process is distinct from the product-costing process of applying overhead to specific jobs. During the year and at year-end, the actual overhead amount incurred will rarely equal the amount applied. The most common and important contributor to these variances is operating at a different volume level than that used as a denominator in calculating the budgeted overhead rate. Other causes include: poor forecasting, inefficient use of overhead items, price changes in individual overhead items, erratic behavior of individual overhead items (e.g., repairs made only during slack time), and calendar variations (e.g., 20 workdays in one month, 22 in the next). All these peculiarities are mingled in an annual overhead pool. An annual rate is budgeted and used regardless of the month-to-month peculiarities of specific overhead costs. Such an approach is more defensible than applying the actual overhead for each month. This is due to the normal product cost being more useful for decisions and inventory-costing purposes than an “actual” product cost that is distorted by month-to-month fluctuations in

Copyright ©2023 Pearson Education, Ltd.

204


production volume and by the erratic behavior of many overhead costs. In an actual-costing system, overhead would not be applied as jobs are worked on. Only after all overhead costs for the year are known can actual costs be applied to the jobs worked on during the year in an actual costing system. However, increased accuracy would be obtained at the serious sacrifice of timeliness in using costs for measuring operating efficiency, determining selling prices, and producing interim financial statements. It is possible to use a normal-costing system plus year-end adjustments to produce final results that closely approximate the results under actual costing. The manufacturing costs applied to products under the two systems are:

Direct Materials Direct Labor Manufacturing overhead B.

ACTUAL COSTING

NORMAL COSTING

Actual Actual Actual

Actual Actual Budgeted rates

Disposition of Underapplied or Overapplied Overhead When budgeted rates are used, the difference between incurred and applied overhead is typically allowed to accumulate during the year. Overapplied Overhead—the difference of the amount applied to products exceeding the amount incurred by the departments. Underapplied Overhead—the difference of the amount applied to products being less than incurred. At year-end, the difference is disposed of either through a write-off or proration. 1.

Immediate Write-off. With this method (which is more commonly used), underapplied overhead is added to the cost of goods sold to reduce current income, and overapplied overhead is subtracted from cost of goods sold to increase current income. Most of the goods worked on have been sold, and a more elaborate method of disposition is not worth the extra trouble. Also, the extra overhead costs represented by underapplied overhead do not qualify as part of ending inventory costs because they do not represent assets. They should be written off because they largely represent inefficiency or the underutilization of available facilities in the current period.

2.

Proration Among Inventories. Prorate—assign in proportion to the size of the ending account balance. This method prorates underapplied overhead among three accounts (i.e., WIP, FG, and COGS). If the objective is to obtain as accurate a cost allocation as possible, all the overhead costs of the individual jobs worked on should be recomputed using the actual, rather than the budgeted, rates. Prorating should be performed on the basis of the ending balances in each of three accounts. In practice, prorating is done only when

Copyright ©2023 Pearson Education, Ltd.

205


inventory valuations would be materially affected. C.

The Use of Variable and Fixed Application Rates Some companies distinguish between variable overhead and fixed overhead for product costing, as well as for control purposes. When this is the case, two rates may be developed: 1. budgeted variable-overhead application rate = budgeted total variable overhead / budgeted machine hours 2. budgeted fixed-overhead application rate = budgeted total fixed overhead / budgeted machine hours

IV.

Variable Versus Absorption Costing

{L. O. 4}

Two major methods of product costing are compared in this chapter: variable costing (the contribution approach) and absorption costing (the functional, full-costing, or traditional approach). These methods differ in only one conceptual respect: the accounting for fixed manufacturing overhead costs. A.

Accounting for Fixed Manufacturing Overhead Variable costing (or direct costing) signifies that fixed factory overhead is not inventoried. In contrast, absorption costing indicates that inventory values include fixed factory overhead. Variable costing regards fixed manufacturing overhead (fixed factory overhead) as an expired cost to be immediately charged against sales—not as an unexpired cost to be held back as inventory and charged against sales later as a part of cost of goods sold. Absorption costing is more widely used than variable costing. However, the growing use of the contribution approach in performance measurement and cost analysis has led to increasing use of variable costing for internal reporting purposes. However, absorption costing must be used for reports to shareholders and tax authorities. See EXHIBIT 13-2 for the comparison of flow of costs.

B.

Facts for Illustration An example using the following standard cost information is presented for Desk PC, a division of Dell Computer, which produces desktop computers: Direct material:

Copyright ©2023 Pearson Education, Ltd.

$205

206


Direct labor: Variable manufacturing overhead: Standard variable costs per ring

75 20 $300

Fixed manufacturing overhead is budgeted at $1,500,000 for the budgeted production of 15,000 units per year. The sales price is $500 per unit. For simplicity, it is assumed that computers produced is the cost driver for variable manufacturing overhead. Also assumed is that both budgeted and actual selling price and administrative expenses are a yearly $650,000 fixed cost plus a variable sales commission of 5% of dollar sales. Actual production quantities are: 20X7 In units (computers): Opening inventory Production Sales Ending inventory

-------17,000 14,000 3,000

20X8 3,000 14,000 16,000 1,000

There are no variances from the standard variable manufacturing costs, and fixed manufacturing overhead incurred is exactly $1,500,000 per year. Using this information, income statements for both years are created and a reconciliation of the incomes is made. C.

Variable-Costing (VC) Method

{L. O. 5}

See EXHIBIT 13-3 for income statements using the variable-costing approach that are prepared based on the facts above. D.

Absorption-Costing (AC) Method

{L. O. 6}

See EXHIBIT 13-4 for the income statements using the AC method. If the inventory level does not change, the two methods result in the same net income. Companies with little inventory (e.g., in a JIT environment) generally experience only insignificant changes in inventory. As can be seen when comparing these income statements with those prepared using variable costing, there are three main differences. 1.

The unit product cost differs.

2.

Fixed factory overhead does not appear as a separate line in an AC income statement. Instead, it is included in two places. One place is as part of the cost of goods sold, and the other is as a production-volume variance. ProductionVolume Variance—actual production deviates from the expected volume of production used in computing the fixed overhead rate: production-volume variance =

Copyright ©2023 Pearson Education, Ltd.

207


(actual volume - expected volume) x fixed-overhead rate Where the fixed-overhead rate is: budgeted fixed manufacturing overhead / expected volume of production 3.

V.

Finally, the format for an AC income statement separates costs into the major categories of manufacturing and nonmanufacturing. In contrast, a VC income statement separates costs into the major categories of fixed and variable. In the AC method, revenue less manufacturing cost (both fixed and variable) is gross profit or gross margin. In a VC statement, revenue less all variable costs (both manufacturing and nonmanufacturing) is the contribution margin.

Fixed Overhead and Absorption Costs of Product {L. O. 7} All three differences between variable- and absorption-costing formats arise solely because variable costing treats manufacturing overhead differently from absorption costing. In this and subsequent sections, the accounting for factory overhead in an absorption-costing system is explored. A.

Variable and Fixed Unit Costs Graphs of the budgeted and fixed overhead are provided in the text. For variableoverhead costs, the budgeted overhead is the same as what is applied. For fixedoverhead costs, the applied costs may differ from the lump-sum budgeted costs. This is due to a volume difference between what is actually produced and that which was used in setting the fixed-overhead rate. The applied cost depends on volume: fixed cost applied

=

actual volume x fixed-overhead rate

When the volume level actually achieved equals that used in setting the fixedoverhead rate, the applied cost equals the budgeted amount. When actual volume differs from expected volume, the costs used for budgeting and control differ from those used for product costing. For budgeting and control purposes, managers use the actual cost behavior pattern for fixed costs. In contrast, as the graphs indicate, the absorption product-costing approach treats these fixed costs as though they had a variable-cost behavior pattern. The difference between applied and budgeted fixed overhead is the production-volume variance. B.

Nature of Production-Volume Variance

{L. O. 8}

The Production-Volume (or Volume) Variance can be calculated as follows: production-volume = applied fixed - budgeted fixed variance overhead overhead

Copyright ©2023 Pearson Education, Ltd.

208


A production-volume variance arises when the actual production volume achieved does not coincide with the expected volume of production used as a denominator for computing the fixed-overhead rate for product-costing purposes: 1.

When expected production volume and actual production volume are identical, there is no production-volume variance.

2.

When actual volume is less than expected volume, the production-volume variance is unfavorable because the usage of facilities is less than expected and fixed overhead is underapplied. Unfavorable production-volume variances increase the manufacturing costs shown on the income statement. When less is applied to inventory, which is later expensed as part of cost of goods sold than is incurred, the extra amount incurred is expensed on the current period’s income statement.

3.

When actual volume exceeds expected volume, the production-volume variance is favorable because use of facilities is better than expected, and fixed overhead is overapplied. When fixed overhead costs are overapplied, current period expenses must be reduced on the income statement to reflect the favorable variance.

Most companies consider production-volume variances beyond immediate control. Idleness (due to disappointing total sales, poor production scheduling, unusual machine breakdowns, shortages of skilled workers, strikes, storms, etc.) are all possible causes of an unfavorable production-volume variance. With variable overhead, there is no production-volume variance. The concept of production-volume variance arises for fixed overhead because of the conflict between accounting for control (by flexible budgets) and accounting for product costing (by application rates). Remember that fixed costs come in big chunks and are related to the provision of big chunks of production or sales capability, not to the production or sale of a single unit of product. C.

Reconciliation of Variable Costing and Absorption Costing {L. O. 9} See EXHIBIT 13-5 for a reconciliation of the difference in operating incomes under the VC and AC approaches. It can be quickly computed by multiplying the fixedoverhead product-costing rate by the change in the total units in the beginning and ending inventories. The difference also equals the difference in the total amount of fixed manufacturing overhead charged as an expense during a given year. Whenever production exceeds sales, inventory increases and AC income > VC income. Whenever sales exceed production, inventory decreases and VC income > AC

Copyright ©2023 Pearson Education, Ltd.

209


income. D.

Why Use Variable Costing? Many companies use VC income for internal purposes because AC income is affected by production volume and VC income is not. If a manager’s operating performance is heavily based on AC operating income, s/he may be tempted to produce unneeded units to either reduce an unfavorable or increase a favorable production-volume variance. The effect of either of these is an increase in operating income. No such temptation exists under the VC method. Sales-oriented companies may prefer VC because the sales level primarily affects income. In contrast, a production-oriented company (a company that can easily sell all the units it produces) might prefer AC because additional production increases the operating income with AC but not VC.

VI.

Effect of Other Variances Variances, other than production-volume, may be present if a company is using a normal- or standard-costing system. These other variances affect income for both the AC and VC methods. A.

Flexible-Budget Variances See EXHIBIT 13-6 for the relationship between the fixed-overhead flexible-budget variance and the production-volume variance. The flexible budget variance is the difference between the actual fixed-overhead costs and the budgeted fixed-overhead costs. Other than the production-volume variances, several flexible-budget variances may be computed. They measure components of the differences between actual amounts and the flexible-budget amounts for the output achieved. Flexible budgets are primarily designed to assist planning and control rather than product costing. The production-volume variance is not a flexible-budget variance and is designed to aid product costing. See EXHIBIT 13-7 for an illustration of an unfavorable flexible-budget variance having a negative impact on income. When variances are favorable, they increase operating income.

VII.

Appendix 13: Comparisons of Production-Volume Variance with Other Variances The only new variance introduced in this chapter is the production-volume variance, which arises because fixed-overhead accounting must serve two masters: the control-budget purpose and the product-costing purpose. See EXHIBITS 13-11 and 13-12 for displays of a framework for both the flexible-budget variances and the production-volume variance explained in the current chapter. Underapplied or overapplied overhead is always the difference between the actual overhead incurred and the overhead applied.

Copyright ©2023 Pearson Education, Ltd.

210


CHAPTER 13:

Quiz/Demonstration Exercises

Learning Objective 1 1.

Lion Corp. computes its budgeted factory-overhead rate in the machining department based on the number of machine hours estimated for the year. In 2011, 30,000 machine hours were expected for the production of the company’s products using $150,000 budgeted overhead. The budgeted overhead rate for machining was _____. a. $6.00/m.h. b. $5.00/m.h. c. $7.00/m.h. d. $3.00/m.h.

2.

Determine the overhead application rate in the manufacturing department if the overhead is $90,000 and the total direct labor is $30,000, assuming that direct-labor cost is used as a base. If a particular job required $200 of direct-labor cost, the amount of assembly department overhead that would be applied would be _____. a. $375 b. $200 c. $600 d. $400

Learning Objective 2 3.

In regard to all of the identified cost drivers for overhead, 20% of the identified cost drivers cause what percentage of total overhead cost? a. 65% b. 50% c. 95% d. 80% e. none of the above

4.

No matter what cost drivers are chosen, the applied overhead rates are usually changed _____. a. yearly b. monthly c. weekly d. quarterly e. daily

Learning Objective 3 5.

Normalized overhead rates are used in order to _____. a. indicate the actual cost of producing a unit of product each month b. provide an exact allocation of the actual overhead costs to products c. confuse and perplex students taking a course in management accounting d. assist managers in making decisions because they are more representative of the longrun costs of producing a product because they are not affected by volume fluctuations or the erratic behavior of many overhead costs.

6.

The principal difficulty with normal costing is that _____. a. the unit cost information is not received on a timely basis

Copyright ©2023 Pearson Education, Ltd.

211


b. c. d.

it can result in fluctuating per-unit overhead costs there is no difficulty associated with using normal costing estimated overhead and estimated activity are likely to differ from actual overhead and actual costs resulting in underapplied or overapplied overhead.

Learning Objective 4 7.

Variable costing can be used for _____. a. internal reporting b. external reporting c. neither A and B d. both A nor B

Use the following information for questions 8, 9, 11, and 12. Mouse Corp. began operations on January 1, 20X1. The company sells a single product for $60 per unit. During 20X1, 20,000 units were produced and 15,000 were sold. No work-inprocess inventories were present on December 31, 20X1. Costs during 20X1 were as follows:

Direct materials . . . . . Direct labor . . . . . . . Manufacturing overhead Selling and administrative expenses

FIXED COSTS

VARIABLE COSTS

-0-0$44,000

$ 5.00 per unit produced $12.00 per unit produced $ 4.00 per unit produced

$30,000

$ 2.00 per unit sold

The company computes their fixed manufacturing overhead rate for absorption-costing purposes using an expected production level of 22,000 units. The actual fixed overhead for the year was equal to the budgeted amount. Learning Objective 5 8.

The income under the variable-costing approach for Mouse Corp. for the year 20X1 would be _____. a. $491,000 b. $481,000 c. $477,000 d. $487,000

Learning Objective 6 9.

The income under the absorption-costing approach for Mouse Corp. for the year 20X1 would be _____. a. $491,000 b. $481,000 c. $487,000 d. $477,000

Learning Objective 7

Copyright ©2023 Pearson Education, Ltd.

212


10.

The production-volume variance occurs when using _____. a. the variable-costing approach because sales exceed the production for the period b. the variable-costing approach because production exceeds the sales for the period d. the absorption-costing approach because the production level differs from that used in setting the fixed-overhead rate that is used in applying fixed overhead to production d. the absorption-costing approach because production exceeds sales

Learning Objective 8 11.

Mouse Corp. has a production-volume variance for 20X1 of _____. a. $ 4,000 unfavorable b. $14,000 favorable c. $14,000 unfavorable d. $ 4,000 favorable

12.

The production-volume variance computed using the absorption-costing approach should appear on the income statement _____. a. as a reduction in income regardless of where it appears on the statement b. as an adjustment to actual gross profit from standard gross profit c. as neither A nor B d. as either A or B

Learning Objective 9 13.

A company might choose to use a variable-costing approach when determining income _____. a. because changes in sales will not affect the income computed b. because changes in production will not affect the income computed c. because they can sell all they can produce and are a production company d. to satisfy external reporting requirements

14.

A sales-oriented company may prefer which of the following product costing methods in order to provide a better signal about performance? a. absorption costing b. variable costing c. both A and B d. neither A nor B

Copyright ©2023 Pearson Education, Ltd.

213


CHAPTER 13:

Solutions to Quiz/Demonstration Exercises

1. [b]

The rate is found by dividing the budgeted overhead for the machining department by the budgeted level of machine hours. In this case, $150,000/30,000 machine hours = $5.00/m.h.

2. [c]

The rate is 300% = $90,000/$30,000. Applying the rate to $200 of direct-labor costs gives $600.

3. [d] 4. [a] 5. [d] 6. [d] 7. [a] 8. [a]

The variable-costing income statement appears below: Mouse Corp. Income Statement Variable Costing Format Sales Variable cost of sales Direct materials Direct labor Var. man. OH ($4) Selling and adm. Contribution margin Fixed costs Manufacturing overhead Selling and administrative Net income

($60) ($5) ($12) ($2)

$900,000 $75,000 180,000 60,000 30,000

$44,000 30,000

345,000 $555,000

74,000 $481,000

9. [a] The absorption-costing income statement appears below: Mouse Corp. Income Statement Absorption Costing Format Sales ($60) Standard cost of goods sold: Direct materials ($5) $75,000 Direct labor ($12) 180,000 Var. man. OH ($4) 60,000 Fixed man. OH ($2) 30,000 Standard gross profit Fixed OH production-volume variance (2,000 x $2) Actual gross profit

Copyright ©2023 Pearson Education, Ltd.

$900,000

345,000 $555,000 4,000 $551,000

214


Selling and adm. expenses Fixed Variable Net income

($2)

$30,000 30,000

60,000 $491,000

The $10,000 difference in incomes is due to the 5,000-unit increase in inventory during the year [$2 of fixed OH x 5,000 units = $10,000]. 10. [c] 11. [a]

The applied fixed manufacturing overhead would be $40,000 [20,000 x $2.00/unit fixed-overhead rate]. The $2.00/unit rate is found by dividing the $44,000 budgeted fixed overhead by the estimated 22,000 units of production.

12. [c] 13. [b] 14. [b]

Copyright ©2023 Pearson Education, Ltd.

215


Chapter 14 Job Costing and Process-Costing Systems LEARNING OBJECTIVES: When your students have finished studying this chapter, they should be able to: 1. 2. 3. 4. 5. 6. 7. 8. 9. 10.

Distinguish between job-order costing and process costing. Prepare summary journal entries for the typical transactions of a job-order costing system. Use an ABC system in a job-order environment. Show how service organizations use job-order costing. Explain the basic ideas underlying process costing and how they differ from job-order costing. Compute output in terms of equivalent units. Compute costs and prepare journal entries for the principal transactions in a process-costing system. Demonstrate how the presence of beginning inventories affects the computation of unit costs under the weighted-average method. Understand the concept of transferred-in costs in a process-costing system with sequential process. Use backflush costing with a JIT production system.

Copyright ©2023 Pearson Education, Ltd.

209


CHAPTER 14:

ASSIGNMENTS

CRITICAL THINKING EXERCISES 15. Purpose of Accumulating Job Costs 16. Job-Order Compared to Process Costing 17. Cost Allocation in Service Firms 18. Purpose of Product Costing in a Process Production Environment 19. Process Costing in a JIT Environment EXERCISES 20. Job Costing in Business Sectors 21. Direct Materials 22. Use of WIP Inventory Account 23. Job-Cost Record 24. Analysis of Job-Cost Data 25. Analysis of Job-Cost Data 26. Discovery of Unknowns 27. Discovery of Unknowns 28. Relationships Among Overhead Items 29. Processing Costing in Business Sectors 30. Process Map and Process Costing 31. Equivalent Units 32. Basic Process Costing 33. Uneven Flow 34. Journal Entries 35. Journal Entries 36. Compute Equivalent Units 37. Journal Entries 38. Journal Entries PROBLEMS 39. Job Costing at Dell Computer 40. Relationships of Manufacturing Costs 41. Relationship of Subsidiary and General Ledgers, Journal Entries 42. Job Costing in an Accounting Service Provider Firm 43. Weighted Average Process Costing at Nally & Gibson 44. Process and ABC 45. Nonprofit Basic Process Costing 46. Two Materials, Basic Process Costing 47. Materials and Cartons in Basic Process Costing 48. Weighted Average Process Costing with Transferred-in-Costs 49. Backflush Costing 50. Review of Chapters 13 and 14 51. Review of Chapters 13 and 14 52. Nike 10-K Problem: ABC and Distribution Centers

Copyright ©2023 Pearson Education, Ltd.

210


EXCEL APPLICATION EXERCISE 53. Value of Units Produced COLLABORATIVE LEARNING EXERCISE 54. Job and Process Costing INTERNET EXERCISE 55. Process Costing at a Variety of Companies (http://www.landsend.com; http://www.lazboy.com; http://www.tastykake.com)

Copyright ©2023 Pearson Education, Ltd.

211


CHAPTER 14: I.

OUTLINE

Distinction Between Job-Order Costing and Process Costing

{L. O. 1}

Two extremes of product costing are job-order costing and process costing. Job-Order Costing (or Job Costing)—allocates costs to products that are readily identified by individual units or batches (e.g., construction, printing, aircraft, furniture, special-purpose machinery, and any manufacture of tailor-made or unique goods). Each unit or batch receives varying degrees of attention and skill. Process costing—averages costs over large numbers of nearly identical products (e.g., chemicals, oil, textiles, plastics, paints, flour, canneries, rubber, lumber, food processing, glass, mining, cement, and meat packing). These industries involve mass production of like units, which usually pass in continuous fashion through a series of uniform production steps called operations or processes. Job-order costing applies costs to specific jobs, which may consist of either a single physical unit (such as a custom sofa) or a few like units (such as a dozen tables) in a distinct batch or job lot. In contrast, process costing deals with great masses of like units and broad averages of unit costs. Product costing involves an averaging process. For inventory purposes, the unit cost is the result of taking some accumulated cost that has been allocated to departments and dividing it by some measure of production. The difference between the two methods is in the size of the denominator. For job-order costing it is small, and for process costing the denominator is large. Many companies use hybrid-costing systems, which are blends of ideas from both job costing and process costing. II.

Illustration of Job Costing A.

Basic Records of Enriquez Machine Parts Company Job-Cost Record (also called Job-Cost Sheet or Job Order)—where all costs for a particular product, service, or batch of products are recorded on the job-cost record (see EXHIBIT 14-1). A file of job-cost records for partially completed jobs provides supporting details for the Work-in-Process (WIP) Inventory account. A file of completed job-cost records comprises the Finished-Goods (FG) Inventory account. The job-cost record summarizes information contained on source documents. Materials Requisitions—records of materials used in particular jobs. Labor Time Tickets (or Time Cards)—record the time a particular direct laborer spends on each job. With current technology (e.g., online data entry, bar coding, and optical scanning), job-cost records and source documents are likely to be computer files, not paper records. As each job begins, a job-cost record is prepared. As units are worked on, entries are made on the job-cost record. Three classes of costs are accumulated on the job-cost

Copyright ©2023 Pearson Education, Ltd.

212


record as units pass through the departments: material requisitions are the source of direct-materials costs, time tickets provide direct-labor costs, and budgeted overhead rates (i.e., a separate rate for each overhead cost pool) are used to apply factory overhead to products. (The computation of these budgeted rates will be described later in this chapter.) Relevant information is provided for illustrating the use of a job-order costing system for Enriquez Machine Parts Company (see EXHIBIT 14-2). B.

Applying Direct Materials and Direct Labor Costs {L. O. 2}

C.

Applying Factory Overhead Costs

D.

Finished Goods, Sales, and Cost of Goods Sold See EXHIBIT 14-2 for explanations of the journal entries required to record the transactions of Enriquez. These transactions illustrate how the various manufacturing costs are being accumulated in WIP, transferred to FG, and eventually reflected in Cost of Goods Sold (COGS).

III.

Activity-Based Costing/Management in a Job-Costing Environment

{L. O. 3}

Activity-based costing usually increases costing accuracy because it focuses on the cause– effect relationship between work performed (activities) and the consumption of resources (costs). A.

Illustration of ABC in a Job Order Environment See EXHIBIT 14-3 for Dell Computer Corporation’s value chain and ABC system. Dell adopted activity-based costing because of the aggressive cost reduction targets set by top management, and the need to understand product-line profitability. By using the value chain perspective, Dell targeted the non-value-added costs for reduction (e.g., preparation activity in the production facility). Marketing, distribution, and customer service are estimated during the budgeting process and included in the markup used to price a job.

IV.

Job Costing In Service And Nonprofit Organizations

{L. O. 4}

In nonprofit organizations, the “product” is usually not called a “job order”. It may be called a program or class of service. A “program” is an identifiable group of activities that frequently produces outputs in the form of services rather than goods (e.g., a safety program, an education program, and a family counseling program). Costs or revenues may be traced to individual hospital patients, individual social welfare cases, and individual university research projects. However, departments often work simultaneously on many programs, so the “job-order” costing challenge is to “apply” the various department costs to the various programs. Only then can managers make wiser decisions regarding the allocation of limited resources among

Copyright ©2023 Pearson Education, Ltd.

213


competing programs. In service industries (e.g., repairing, consulting, legal, and/or accounting services), each customer order is a different job with a special account or order number. A.

Budgets and Control of Engagements An example is provided illustrating the use of cost data for controlling an auditing engagement for an accounting firm. If a fixed audit fee has been quoted, the profitability of an engagement depends on whether the audit can be accomplished within the budgeted time.

B.

Accuracy of Costs of Engagements Managers of service firms (e.g., auditing and consulting firms) frequently use either the budgeted or “actual” costs of engagements as guides to pricing, and allocate effort among particular services or customers. Hence, the accuracy of costs of various engagements may affect decisions.

V.

Process Costing Basics All product costing uses averaging to determine costs per unit of production. Process-costing systems apply costs to like products that are usually mass-produced in continuous fashion through a series of production processes. These processes usually occur in separate departments, although a single department sometimes contains more than one process. A.

Process Costing Compared with Job Costing {L. O. 5} See EXHIBIT 14-5 for the major differences between job-order costing and process costing. Process costing requires several work-in-process (WIP) accounts, one for each process (or department). As goods move from process to process, their costs are transferred accordingly. See EXHIBIT 14-6 for process costing at Nally & Gibson (a limestone quarry company). Process manufacturing systems vary in design. The processes required for manufacture can be sequential as shown in PANEL B of EXHIBIT 14-5 and in EXHIBIT 14-6, or they can simultaneously produce subcomponents of a final product that are later assembled. Process costing does not distinguish among individual units of product. Instead, accumulated costs for a period are divided by quantities produced during that period to get broad, average unit costs. Process costing can also be applied to nonmanufacturing activities (e.g., the costs of a post office sorting department divided by the number of items sorted). The central product-costing problem is how each department should compute the cost of goods transferred out and the cost of goods remaining in each department. If an identical amount of work was done on each unit transferred and on each unit in ending

Copyright ©2023 Pearson Education, Ltd.

214


inventory, total costs are simply divided by total units. However, if the units in the inventory are each partially completed, the product-costing system must distinguish between the fully completed units transferred out and the partially completed units not yet transferred. VI.

Application of Process Costing An illustration is provided of cost and production data for a company that makes wooden marionettes in two processes: forming and finishing. When all units of product are fully complete at the end of a period, calculation of unit costs is simple. The costs incurred are simply divided by the number of units produced to get the unit costs. However, when some units are only partially complete at the end of a period, a five-step procedure must be used. Step 1: Step 2: Step 3: Step 4: Step 5:

VII.

Summarize the flow of physical units. Calculate output in terms of equivalent units. Summarize the accounted-for total costs, which are the total debits in work in process (i.e., the costs applied to work in process). Calculate unit costs. Apply costs to units completed and to units in the ending work in process.

Physical Units and Equivalent Units (Steps 1 and 2)

{L. O. 6}

The first column in EXHIBIT 14-7 provides a summarization of the flow of physical units. Units started and completed are added to the units remaining in ending workin-process to give the units accounted for. The second column of EXHIBIT 14-7 provides the equivalent units of production for both materials and conversion costs (i.e., all manufacturing costs other than direct materials). Equivalent Units—the number of completed units that could have been produced from the inputs applied (e.g., four units, each one-half completed, represents two equivalent units). Computation of equivalent units requires estimates of how much of a given resource was applied to units in process. Estimating the degree of completion for materials is usually easier than estimating those for conversion costs. The degree of completion for conversion costs depends on what proportion of the total effort needed to complete one unit or one batch has been devoted to units still in process. In industries where no exact estimate is possible or vast quantities in process prohibit costly physical estimates (e.g., textiles), all work in process in every department is assumed to be onethird, one-half, or two-thirds complete. In other cases, continuous processing entails little change of work-in-process levels from month to month. Consequently, in such cases, work in process is safely ignored, and monthly production costs are assigned solely to units completed and transferred out. VIII. Calculation of Product Costs (Steps 3 to 5)

{L. O. 7}

Copyright ©2023 Pearson Education, Ltd.

215


See EXHIBIT 14-8 for a production-cost report, which shows Steps 3 to 5 of process costing. Step 3 summarizes the total costs to account for (i.e., the total costs in, or debits to, Work in Process). Step 4 obtains unit costs by dividing the two categories of total costs by the appropriate measures of equivalent units. Step 5 then uses these unit costs to apply costs to finished and work-in-process (WIP) products. Journal entries to reflect the costs incurred in the process are recorded by debiting the WIP account for a particular department and crediting the appropriate source (i.e., Direct-materials inventory, Accrued payroll, and Factory overhead). When goods are transferred to a subsequent department, the receiving department is charged for the costs of the goods transferred, and the sending department’s WIP account is credited. IX.

Effects of Beginning Inventories: Weighted-Average Method

{L. O. 8}

The goods completed and transferred out come from two sources: units started during the period, and from the partially completed units in beginning inventory. Some decision is required regarding how to account for the costs in the beginning inventory. Two common approaches are used: the weighted-average method and the first-in, first-out method. Weighted-Average (WA) Process-Costing Method—adds the cost of (1) all work done in the current period to (2) the work done in the preceding period on the current period’s beginning inventory of WIP. This total is divided by the equivalent units of work done to date, whether that work was done in the current period or previously. The term weighted-average is used to describe this method primarily because the unit costs used for applying costs to products are affected by the total cost incurred to date, regardless of whether those costs were incurred during or before the current period. See EXHIBIT 14-9 for the production-cost report using the weighted-average method. The costs of the beginning inventory are combined with the current period’s costs to get the total costs to account for (Step 3) and to compute the unit costs for materials and conversion costs (Step 4). The unit costs (average costs) are then used to apply costs to units transferred out and the partially completed ones remaining in ending inventory (Step 5). X.

Transferred-In Costs

{L. O. 9}

Many companies that use process costing have sequential production processes. Processes, other than the first, have units of product transferred in. Accompanying the units are Transferred-In Costs—costs incurred in a previous department for items that have been received by a subsequent department. They are similar to additional direct-materials costs, but should not be called direct-material cost in a subsequent department. Accounting for transferred-in costs is similar to accounting for direct materials, with one exception: Transferred-in costs are kept separate from the direct materials added in the department. Production-cost reports in later departments include three columns of costs instead of two: transferred-in costs, direct-material costs, and conversion costs. The total unit cost is the sum of all three types of unit costs. See EXHIBIT 14-10.

Copyright ©2023 Pearson Education, Ltd.

216


XI.

Process Costing in a JIT System: Backflush Costing

{L. O. 10}

Tracking costs through various stages of inventory (i.e., raw material, work-in-process for each process or department, and finished goods inventory) makes accounting systems complex. Without inventories, all costs would be simply charged directly to cost of goods sold (COGS). Backflush Costing—used by organizations using JIT production systems with very small inventories to avoid having to trace costs through all inventory accounts. With backflush costing, costs are applied to products only when production is complete. A.

Principles of Backflush Costing Only two categories of costs exist in backflush costing: materials and conversion costs. Actual costs are entered into a materials inventory account, and actual labor and overhead costs are entered into a conversion costs account. Costs are transferred from these two temporary accounts directly into finished-goods inventories (FG). Some backflush systems even eliminate the FG inventory accounts and transfer costs directly to COGS. Backflush systems rely on the assumption that, as completion of production follows so soon after the application of conversion activities, balances in the conversion costs accounts always should remain near zero. Costs are transferred out almost immediately after being initially recorded.

B.

Example of Backflush Costing The three steps to apply backflush costing are: 1. 2.

3a.

3b.

Record actual materials and conversion costs. Apply costs to completed units. When production is complete, costs from materials inventory and conversion-costs accounts are transferred directly to FG based on the number of units completed and a standard cost of each unit. Record cost of goods sold during the period. The standard cost of the items sold is transferred from FG to COGS. If completed units are delivered immediately to customers, so that FG inventories are negligible, steps 2 and 3 can be combined. If actual costs do not equal the standard amounts transferred from directmaterials and conversion costs, any differences are written off to COGS.

Copyright ©2023 Pearson Education, Ltd.

217


CHAPTER 14:

Quiz/Demonstration Exercises

Learning Objective 1 1.

_____ allocates costs to products that are readily identified by individual units or batches, each of which receives varying degrees of attention and skill. a. Process costing b. Job-order costing c. Integrated costing d. Nonspecific costing

2.

_____ averages costs over large numbers of nearly identical products and is most often found in such industries as chemicals, oil, textiles, plastics, paints, flour, canneries, rubber, lumber, food processing, glass, mining, cement, and meatpacking. a. Nonspecific costing b. Integrated costing c. Job-order costing d. Process costing

Learning Objective 2 Use the following information for questions 3 and 4 A summary of pertinent transactions for the year 20X1 for Hogan Company appears below. No inventories of materials, work-in-process, or finished goods were present at the beginning of the year.

MACHINING 1. Direct materials purchased on account 2. Direct materials requisitioned for manufacturing 3. Direct-labor costs incurred 4a. Factory overhead incurred 4b. Factory overhead applied 5. Cost of goods completed and transferred to finished-goods inventory 6a. Sales on account 6b. Cost of goods sold

3.

ASSEMBLY

TOTAL $1,200,000

$450,000 50,000 150,000 210,000

$500,000 30,000 75,000 90,000

950,000 80,000 225,000 300,000

--

--

--

--

800,000 1,450,000 600,000

The incidence of labor costs to manufacture the company’s products would be recorded as _____. a. Direct Labor Expense 80,000; Direct Labor Payable 80,000

Copyright ©2023 Pearson Education, Ltd.

218


b. c. d. 4.

WIP Inventory 80,000; Accrued Labor Payable 80,000 Direct Labor Payable 80,000; WIP Inventory 80,000 Accrued Labor Payable 80,000; WIP Inventory 80,000

The entry to recognize the manufacturing overhead applied to the product would be _____. a. Finished-Goods Inventory 300,000; WIP Inventory 300,000 b. Cost of Goods Sold 300,000; Factory Dept Overhead Control 300,000 c. Finished-Goods Inventory 300,000; Overhead Applied 300,000 d. WIP Inventory 300,000; Factory Dept Overhead Control 300,000

Learning Objective 3 5.

Which of the following is a reason to adopt activity-based costing in a job-order environment? a. aggressive cost reduction targets set by top management b. understanding product-line profitability c. both A and B d. small amounts of overhead e. none of the above

6.

Based on the value chain, which function costs are allocated? a. customer service b. product design c. distribution d. production e. all of the above

Learning Objective 4 7.

Price Waterhouse Deloitte is a national CPA firm. Using their typical method for budgeting costs, they have determined the following bidding budget for an audit engagement: Direct professional labor $30,000 Travel costs 4,000 Applied overhead @ 200% of direct professional labor 60,000 Total costs $94,000 In recent months, they have been losing many bids to another national firm, KPYE. The few bids that Price Waterhouse Deloitte has been getting have either barely broken even or created losses for the firm. One lost potential client indicated that KPYE’s bids were much more detailed and contained a much lower overhead assignment than the one submitted by Price Waterhouse Deloitte. It is likely that KPYE _____. a. has lower operating costs than the other firms in the industry and can therefore price all their jobs below the competition b. will be out of business in the near future because their prices for services are so low c. is using an activity-based costing approach to costing their jobs, which traces more direct costs to jobs and leaves fewer costs classified as overhead to be allocated

Copyright ©2023 Pearson Education, Ltd.

219


d.

is engaged in illegal practices because they are not costing jobs with the customary 200% of direct professional labor rate

8.

Which of the following costs usually is not directly traceable to an audit engagement? a. travel to and from the client b. labor incurred at the client’s site c. meals and entertainment with the client d. office space rent for CPA firm Learning Objective 5 9.

Process-costing systems differ from job-order costing systems in that the former _____. a. are far more complex and expensive to maintain b. do not require equivalent unit calculations for computing unit costs c. typically divide accumulated costs by a larger number of units of product for product costing purposes d. require fewer work-in-process accounts

10.

All of the following statements are true with respect to process costing except _____. a. unit costs are determined by processing department and added together to determine total unit costs b. identical products are produced on a continuous basis c. costs are accumulated by individual jobs d. the cost of production report provides the detail for the work-in-process accounts

Learning Objective 6 11.

Mark Inc. manufactures a prescription drug using two processes: Blending and Encapsulating. During a month in which they had no beginning inventory, materials for 200,000 units were placed into production in the Blending Department. At the end of the month, 120,000 units had been transferred to the Encapsulating Department with the remaining 80,000 units 25% complete for conversion costs. The 80,000 units contained all of their material ingredients. The number of equivalent units for material and conversion costs for the month in a processcosting system would be Material:_____ ; Conversion Costs: _____. a. 200,000; 120,000 b. 200,000; 140,000 c. 120,000; 120,000 d. 120,000; 140,000

12.

Equivalent production expresses all activity of the period in terms of _____. a. direct-labor hours b. partially completed units c. units of output d. fully completed units

Learning Objective 7

Copyright ©2023 Pearson Education, Ltd.

220


Use the following information for questions 13 through 17. The Mantle Company produces miniature baseballs that are used for promotions at professional baseball games. The balls are produced in two processes: Forming and Finishing. The following information represents the activity of the Forming department: Units started Units completed Units of ending inventory Cost of materials added Conversion costs incurred

25,000 15,000 10,000 (50% complete for conversion) $50,000 $40,000

All materials are placed into production at the beginning of the process in the forming department. For questions 13 through 16, assume that Mantle had no beginning inventories in the Forming Department. 13.

The entry to record the cost of materials placed in production is _____. a. WIP Forming $50,000 Raw-Materials Inventory $50,000 b.

Raw-Materials Inventory Accounts Payable or Cash

c.

WIP—Finishing Raw-Materials Inventory

$50,000

WIP—Finishing WIP—Forming

$50,000

d.

$50,000 $50,000

$50,000

$50,000

14.

The unit costs in the Forming Department are _____ for materials and _____ for conversion costs. a. $2.50; $1.60 b. $2.50; $2.00 c. $2.00; $2.20 d. $2.00; $1.60

15.

The cost of the 10,000 units remaining in ending inventory would be _____. a. $40,000 b. $30,000 c. $20,000 d. $36,000

Learning Objective 9 16.

The entry to record the costs transferred from the Forming Department to the Finishing Department would be _____. a. WIP—Finishing: $30,000; WIP—Forming: $30,000 b. WIP—Finishing: $30,000; WIP—Forming: $30,000 c. WIP—Forming: $60,000; WIP—Finishing: $60,000

Copyright ©2023 Pearson Education, Ltd.

221


d.

WIP—Forming: $60,000; WIP—Finishing: $60,000

Learning Objective 8 For question 17, assume the Forming Department had the following information related to their beginning inventory: Units Materials costs Conversion costs

4,000 (60% complete for conversion costs) $ 3,400 $ 3,750

17.

In computing unit costs using the weighted-average method, the cost for materials that would be divided by equivalent units of materials would be _____. a. $ 32,900 b. $ 50,000 c. $ 53,750 d. $ 53,400

18.

The cost per equivalent unit using the weighted-average method is calculated as: a. costs added during the period / equivalent units b. total costs to account for / equivalent units c. total costs to account for / number of partially completed units d. costs added during the period / number of partially completed units

Learning Objective 10 19.

Stanford Inc., uses backflush costing. During a recent month, $50,000 of materials were purchased and $70,000 of conversion costs were incurred for production. Materials are acquired just in time for use in production orders. If goods are shipped to customers upon completion of production, _____. a. materials and work-in-process accounts must be used to record production costs b. a finished-goods inventory account must be used to record the costs of finished goods, which are then transferred to cost of goods sold c. cost of goods sold could be charged directly with all production costs d. inventories will be so large that several work-in-process accounts must be used

20.

Any remaining conversion-cost balance at the end of an accounting period (per a backflush system) is charged to _____. a. cost of goods sold b. the finished-goods account c. the work-in-process account d. the materials inventory account

Copyright ©2023 Pearson Education, Ltd.

222


CHAPTER 14: 1. [b] 2. [d] 3. [b] 4. [d] 5. [c] 6. [e] 7. [c] 8. [d] 9. [c] 10. [c] 11. [b]

Solutions to Quiz/Demonstration Exercises

Because all of the ingredients are present, 200,000 equivalent units of materials were produced. Conversion-cost equivalent units are composed of 120,000 completed units and 80,000 partially completed ones. The 80,000 units, which are 25% complete, equal 20,000 equivalent units of production. 120,000 + 20,000 = 140,000 equivalent units of production.

12. [d] 13. [a] 14. [c] 15. [b] 16. [b] (The following production-cost report is used for questions 13–17) Mantle Company Production-Cost Report Forming Department

TOTAL COSTS Costs to account for Divided by equivalent units Unit costs

$90,000 $4.00

Details Direct Conversion Materials Costs $50,000 25,000 $2.00

$40,000 20,000 $2.00

Application of costs to units not completed and still in process: Direct materials Conversion costs Work in process To units transferred to the Finishing Department Total costs accounted for 17. [d]

$ 20,000 10,000 $ 30,000

10,000 * ($2.00) 5,000 * ($2.00)

$60,000 $90,000

15,000 * ($4.00)

With the weighted-average method, the current period’s costs are blended together with the prior period’s costs when computing unit costs. Here, $50,000 + $3,400 =

Copyright ©2023 Pearson Education, Ltd.

223


$53,400. 18. [b] 19. [c] 20. [a]

Copyright ©2023 Pearson Education, Ltd.

224


Chapter 15 Basic Accounting: Concepts, Techniques, and Conventions LEARNING OBJECTIVES: When your students have finished studying this chapter, they should be able to: 1. Read and interpret basic financial statements. 2. Analyze typical business transactions using the balance sheet equation. 3. Distinguish between the accrual basis of accounting and the cash basis of accounting, 4. Make adjustments to the accounts under accrual accounting. 5. Explain the nature of dividends and retained earnings. 6. Select relevant items from a set of data and assemble them into a balance sheet and an income statement. 7. Distinguish between the reporting of corporate owners’ equity and the reporting of owners’ equity for partnerships and sole proprietorships. 8. Explain the role of auditors in financial reporting and how accounting standards are set. 9. Identify how the measurement principles of recognition, matching and cost recovery, and stable monetary unit affect financial reporting. 10. Define continuity, relevance, faithful representation, materiality, conservatism, and cost– benefit (Appendix 15A). 11. Use T-accounts, debits, and credits to record transactions (Appendix 15B).

Copyright ©2023 Pearson Education, Ltd.

228


12. CHAPTER 15:

ASSIGNMENTS

CRITICAL THINKING EXERCISES 24. Accounting Valuation of Fixed Assets 25. Marketing, the Income Statement, and the Balance Sheet 26. Revenue Recognition and Evaluation of Sales Staff 27. Relationship Between the Balance Sheet and the Income Statement 28. Concepts of Relevance and Faithful Representation GENERAL EXERCISES and PROBLEMS 29. True or False 30. Simple Balance Sheet 31. Nature of Retained Earnings 32. Income Statement 33. Income Statement, Balance Sheet, and Dividends 34. Customer and Hotel 35. Property Owner and Municipality 36. Adjustments 37. Find Unknowns 38. Balance Sheet Equation: Solving for Unknowns 39. Fundamental Transaction Analysis and Preparation of Statements 40. Measurement of Income for Tax and Other Purposes 41. Debits and Credits 42. True or False 43. Use of T-Accounts 44. Use of T-Accounts 45. Use of T-Accounts UNDERSTANDING PUBLISHED FINANCIAL REPORTS 46. Balance Sheet Effects 47. Preparation of Balance Sheet for Daimler AG 48. Net Income and Retained Earnings 49. Earnings Statement, Retained Earnings 50. Sole Proprietorship and Corporation 51. Nike 10-K Problem: Interpreting the Income Statement and Balance Sheet EXCEL APPLICATION EXERCISE 52. Monthly Transactions Using the Balance Sheet Equation COLLABORATIVE LEARNING EXERCISE 53. Implicit Transactions INTERNET EXERCISE 54. McDonald’s Financial Statements (https://corporate.mcdonalds.com)

Copyright ©2023 Pearson Education, Ltd.

229


CHAPTER 15: I.

OUTLINE

The Need for Accounting Accounting is the language of business. Managers, investors, and other interested groups usually want the answers to two important questions about an organization: How well did the organization perform for a given period? Where does the organization stand at a given point? Accountants answer these two questions with two major financial statements: an income statement and a balance sheet. To obtain these statements, accountants record an organization’s transactions. Transaction—any event that affects the financial position of an organization and requires recording. Through the years, many concepts, conventions, and rules have been developed regarding what events are to be recorded as accounting transactions and how their financial impact is measured.

II.

Financial Statements: Balance Sheet and Income Statement {L. O. 1} Financial statements are summarized reports of accounting transactions. They can apply to any point in time and to any span of time. Balance Sheet (more accurately called Statement of Financial Position or Statement of Financial Condition)—a snapshot of financial status at an instant of time. The balance sheet equation is assets = equities The equities side of this equation is often divided into two parts: assets = liabilities + owners’ equity Liabilities—the entity’s economic obligations to nonowners. Owners’ Equity—the excess of the assets over the liabilities. For Corporations (i.e., a business organized as a separate legal entity and owned by its stockholders), owners’ equity is called Stockholders’ Equity. Stockholders’ equity is a result of Paid-in Capital (i.e., the ownership claim arising from funds paid-in by the owners) plus Retained Earnings or Retained Income (the ownership claim arising from the investment of previous profits). The balance sheet equation for corporations becomes: assets

= =

liabilities + stockholders’ equity liabilities + (paid-in capital + retained earnings)

{L. O. 2} See EXHIBIT 15-1 and EXHIBIT 15-2 for illustrations of the effects of transactions on the balance sheet equation. Accounts Receivable—amounts due from customers on open account. Accounts Payable—amounts owed on open accounts.

Copyright ©2023 Pearson Education, Ltd.

230


A.

Revenue and Expenses Revenues—increases in ownership claims arising from the delivery of goods or services. To be recognized (i.e., formally recorded in the accounting records as revenue during the current period), revenue (1) must be earned by fully rendering goods or services to customers, and (2) must be realized (i.e., the seller must be reasonably assured that the resources promised in exchange for the goods or services will be received). Expenses—decreases in ownership claims arising from delivery of goods or services, or using up assets. Profits (or Earnings of Income)—the excess of revenues over expenses. Account—each item in a financial statement. Increases in revenues are increases in stockholders’ equity and increases in expenses decrease stockholders’ equity.

B.

Relationship Between Balance Sheet and Income Statement Income Statement—measures the operating performance of the corporation by matching its accomplishments (i.e., revenues or sales) with its efforts (i.e., expenses such as cost of goods sold). The balance sheet shows the financial position at an instant of time, but the income statement measures performance for a span of time, whether it be a month, a quarter, or longer. Thus, the income statement is the major link between balance sheets.

C.

The Analytical Power of the Balance Sheet Equation The balance sheet equation can highlight the link between the income statement and the balance sheet. The relationships between income statement items and those for the balance sheet can be seen in transforming the balance sheet equation as shown below: 1. assets (A) = liabilities (L) + stockholders’ equity (SE) 2. A = L + paid-in capital + retained earnings 3. A = L + paid-in capital + revenue - expenses Revenue and expense accounts are subdivisions of stockholders’ equity (i.e., temporary stockholders’ equity accounts). For every transaction, the balance sheet equation is always kept in balance.

III.

Accrual Basis and Cash Basis

{L. O. 3}

Accrual Basis—recognizes the impact of transactions on the financial statements in the periods when revenues and expenses occur instead of when cash is received or disbursed.

Copyright ©2023 Pearson Education, Ltd.

231


Revenues are recorded as they are earned and expenses recorded as they are incurred, not necessarily when cash changes hands. Cash Basis—revenue and expense recognition would depend solely on the timing of various cash receipts and disbursements. The major deficiency of the cash basis is that it is incomplete. It fails to match efforts with accomplishments (expenses and revenues) in a manner that properly measures economic performance and financial position. It omits key assets (such as accounts receivable and prepaid rent) and key liabilities (such as accounts payable) from the balance sheets. A.

Nonprofit Organizations The examples in this chapter focus on profit-seeking organizations (for-profit), but nonprofit organizations, such as government agencies and charitable organizations, also use balance sheets and income statements. For many years, most nonprofit organizations used cash-basis rather than accrual accounting, but as these organizations are increasingly using accrual accounting.

IV.

Adjustments to the Accounts Adjustments—recording of implicit transactions, in contrast to the explicit transactions that trigger nearly all day-to-day routine entries, at the end of each reporting period in order to measure income for accrual accounting. They refine the accountant’s accuracy and provide a more complete and significant measure of efforts, accomplishments, and financial position. The four categories of transactions (i.e., economic events that should be recorded by an accountant) for which adjustments are necessary are outlined below. Source Documents—explicit evidence of any transactions that occur in the entity’s operation (e.g., sales slips and purchase invoices).

V.

Adjustment Type I: Expiration of Unexpired Costs

{L. O. 4}

Assets frequently expire because of the passage of time. Assets may be viewed as bundles of economic services awaiting future use or expiration. Assets, other than cash and receivables, may be thought of as prepaid or stored costs that are carried forward to future periods rather than immediately charged against revenue. Expenses are used-up assets. Unexpired Cost— any asset that ordinarily becomes an expense in future periods (e.g., inventory and prepaid rent). A.

Timing of Asset Expiration Some assets expire immediately (e.g., advertising services and miscellaneous supplies). When this is the case, the usual accounting for them is to expense them instantaneously rather than recording them first as assets and then expensing them. The time at which assets expire can be debatable. For example, research and development costs must be written off as an expense under current generally accepted accounting principles. However, they may have some future economic Copyright ©2023 Pearson Education, Ltd.

232


benefit. B.

Depreciation Long-lived assets (e.g., equipment and buildings) have their cost recognized as an expense through the periodic recording of depreciation. Accountants usually (1) predict the length of the useful life, (2) predict the ultimate residual value, and (3) allocate the cost of the equipment to the years of its useful life in some systematic way. Residual Value—the predicted sales value of a long-lived asset at the end of its useful life.

VI.

Adjustment Type II: Recognition (Earning) of Unearned Revenues Unearned Revenues (or Deferred Revenues)—liabilities that are present due to the collection of payment prior to the rendering of services or sending products. As the services are rendered or goods delivered, the revenue must be recognized.

VII.

Adjustment Type III: Accrual of Unrecorded Expenses Accrue—to accumulate a receivable or payable during a given period even though no explicit transaction occurs (e.g., the accruals of wages of employees for partial payroll periods and the interest on borrowed money before the interest payment date). Usually, adjusting entries are made to bring these expenses and the resulting liabilities up to date.

VIII. Adjustment Type IV: Accrual of Unrecorded Revenues The final type of adjustment, the realization of revenues that have been earned but not recorded as such in the accounts, is the mirror image of the accrual of unrecorded expenses (e.g., interest receivable and interest revenue may need to be recorded at the end of an accounting period). See EXHIBIT 15-3 for a summarization of the four major types of adjustments needed to implement the accrual basis of accounting. IX.

Dividends and Retained Income

{L. O. 5}

Dividends—distributions of assets to stockholders that reduce retained income. A.

Dividends Are Not Expenses They are not expenses like rent and wages. They should not be deducted from revenues because they are not directly related to the generation of sales or the conduct of operations. The ability to pay dividends is fundamentally caused by profitable operations. Retained income increases as profits accumulate, and decreases as dividends occur. Retained income is frequently the largest stockholders’ equity account.

Copyright ©2023 Pearson Education, Ltd.

233


B.

Nature of Retained Earnings Although retained income is a result of profitable operations, it is not a pot of cash awaiting distribution to stockholders. Its value is distributed among the many assets held by the firm, not just cash. Dividends are distributions of assets that reduce ownership claims. The cash assets that are disbursed typically arose from profitable operations. Thus, dividends or withdrawals are often spoken of as “distributions of profits” or “distributions of retained earnings.” Often, dividends are erroneously spoken of as being “paid out of retained income.” In reality, cash dividends are distributions of assets that liquidate a portion of the ownership claim, which is made possible by profitable operations.

X.

Preparing Financial Statements

{L. O. 6}

Using Stora Exor’s information in EXHIBIT 15-2, financial statements are constructed. See EXHIBIT 15-4 for the balance sheet, EXHIBIT 15-5 for the income statement, and EXHIBIT 15-6 for the Statement of Retained Earnings. The income statement is a “multiple-step” statement because it includes a subtotal for gross profit. Gross profit (or gross margin) is the excess of sales over the cost of the inventory that was sold. A “singlestep” statement would list all the expenses, including cost of goods sold, and deduct the total from sales. XI.

Sole Proprietorships and Partnerships

{L. O. 7}

The basic accounting concepts that underlie the owners’ equity for the corporate form of business also apply to Sole Proprietorships and Partnerships. In proprietorships and partnerships, however, distinctions are rarely made between paid-in capital and retained income. In contrast to corporations, sole proprietorships and partnerships are not legally required to account separately for paid-in capital (i.e., proceeds from issuances of capital stock) and for retained income. Instead, they typically accumulate a single amount for each owner’s original investment, subsequent investments, share of net income, and withdrawals. XII.

Generally Accepted Accounting Principles

{L. O. 8}

Generally Accepted Accounting Principles (GAAP)—the conventions, rules, and procedures that together make up accepted accounting practice at any given time. They become generally accepted by agreement. Such agreement is not solely influenced by formal logical analysis. Experience, custom, usage, and practical necessity contribute to the set of principles. Accordingly, it might be better to call them conventions, because principles suggest that they are the product of airtight logic. A.

Auditor’s Independent Opinion The financial statements of publicly held corporations and many other corporations are subject to an independent audit that forms the basis for a professional accounting

Copyright ©2023 Pearson Education, Ltd.

234


firm’s opinion. An accounting firm must conduct an audit before it can render the opinion. Audit— an “examination” or in-depth inspection that is made in accordance with generally accepted auditing standards. An audit has been developed primarily by the American Institute of Certified Public Accountants (AICPA), which is the leading organization of auditors. An audit includes tests of the accounting records, internal control systems, and other procedures as deemed necessary. After auditing a company, an accountant issues an independent opinion—the accountant’s testimony that management’s financial statements are in conformity with generally accepted accounting principles. B.

Accounting Standard Setters TEACHING TIP: Internet sites—SEC, IASB, and FASB: http://www.fasb.org http://www.sec.gov

http://www.iasb.org

American GAAP is largely the work of the Financial Accounting Standards Board (FASB). The rest of the world is governed by the International Accounting Standards Board (IASB). The FASB and IASB continue to work together to improve comparability and consistency in global financial reporting. Securities and Exchange Commission (SEC)—a government agency established by federal law, has the ultimate responsibility for specifying GAAP for U.S. companies whose stock is held by the public. However, the SEC has informally delegated much rule-making power to the FASB. Congress can overrule both the SEC and the FASB, and the SEC can overrule the FASB. Undermining of the FASB occurs rarely. However, corporations and other interested parties exert pressure on all three tiers, if they think an impending pronouncement is “wrong”. The setting of accounting principles is complex and involves heavy interactions among the affected parties: public regulators (Congress and SEC), private regulators (FASB), companies, the public accounting profession, representatives of investors, and other interested groups. The Sarbanes-Oxley Act of 2002 dictates more regulatory control of both accounting and auditing standards. XIII. Three Measurement Conventions A.

{L. O. 9}

Recognition In general, revenue is recognized when the goods or services in question are delivered to customers. This was discussed earlier in the section entitled Revenues and Expenses.

Copyright ©2023 Pearson Education, Ltd.

235


B.

Matching and Cost Recovery Matching—the relating of accomplishments or revenues (as measured by the selling prices of goods and services delivered) and efforts or expenses (as measured by the cost of goods and services used) to a particular period for which income measurement is desired. Matching is a short description of the accrual basis for measuring income. Cost Recovery Concept—assets (e.g., inventories, prepayments, and equipment) are carried forward as assets because their costs are expected to be recovered in the form of cash inflows (or reduced cash outflows) in future periods.

C.

Stable Monetary Unit The dollar is the monetary unit used for measuring assets and equities. It is the common denominator for quantifying the effects of a variety of transactions. While companies in the United States, Canada, Australia, and New Zealand use the dollar as the monetary unit, other countries use the franc, pound, mark, yen, or some other monetary unit. The dollars used in recording transactions, which result in the financial statements, do not incorporate any changes in its purchasing power. Several countries, including Brazil and Argentina, routinely adjust their accounting numbers for the effects of inflation. The most troublesome aspect is how to interpret the results of these adjusted numbers.

XIV. Appendix 15A: Additional Accounting Concepts A.

{L. O. 10}

The Continuity or Going Concern Convention Continuity (or Going Concern Convention)—the assumption that an organization will continue to exist and operate. Existing resources (e.g., plant assets) will be used to fulfill the general purposes of a continuing entity rather than sold in tomorrow’s real estate or equipment markets. It also implies that existing liabilities will be paid at maturity in an orderly manner.

B.

Relevance and Faithful Representation Relevance and faithful representation are the two main qualities to make accounting information useful. Relevance is whether the information makes a difference to the decision maker. Two characteristic of relevance are predictive value and confirmatory value. Accountants also want information to exhibit faithful representation which requires information to be complete, neutral, and free of material errors. There are four characteristics that enhance relevance and faithful representation. They are comparability which requires consistency, verifiability, timeliness, and

Copyright ©2023 Pearson Education, Ltd.

236


understandability. C.

Materiality Materiality—the accounting convention that justifies the omission of insignificant information if its omission or misstatement would not mislead a user of financial statements. Many outlays that should theoretically be recorded as assets are immediately written off as expenses because of their lack of significance (e.g., a rule that requires the immediate write-off to expense of all outlays under a specified minimum of, say $100, regardless of the useful life of the asset acquired).

D.

The Conservatism Convention (or The Prudence Convention) Conservatism Convention—selecting the method of measurement that yields the gloomiest immediate results (e.g., lower-of-cost-or-market and historical cost).

E.

Cost-Benefit Cost-Benefit Criterion—as an accounting system is changed, its potential benefits should exceed its additional costs. Often the benefits are difficult to measure.

XV.

Appendix 15B: Using Ledger Accounts A.

{L. O. 11}

The Account Ledger Accounts—must be used to keep track of how the multitudes of transactions affect each particular asset, liability, revenue, expense, and so forth. T-Accounts— simplified versions of ledger accounts that take the form of the capital letter T. Double-Entry System—a method of record keeping in which at least two accounts are affected by each transaction.

B.

General Ledger General Ledger—a collection of the group of accounts that supports the items shown in the major financial statements. It is usually supported by various subsidiary ledgers, which provide details for accounts in the general ledger. See EXHIBIT 15-7 for the general ledger of King Hardware Co.

C.

Debits and Credits Debit—the left side of an account. Credit—the right side of an account. Debits increase assets and expense accounts, and decrease liability, owners’ equity accounts, and revenues. Credits decrease asset and expense accounts and increase liability, owners’ equity, and revenue accounts.

Copyright ©2023 Pearson Education, Ltd.

237


CHAPTER 15:

Quiz/Demonstration Exercises

Learning Objective 1 1.

Which of the following is a form of the balance sheet equation? a. Assets = Liabilities + (Paid in Capital + Retained Income) b. Assets +Liabilities = Owner’s Equity c. Assets + Owners’ Equity = Liabilities d. Assets - Paid-in Capital = Liabilities - Retained Income

2.

Which of the following financial statements is a “snapshot” of a company’s financial status at an instant of time? a. income statement b. balance sheet c. statement of retained earnings d. cash flow statement e. both B and D

Learning Objective 2 3.

The effect of purchasing inventory using trade credit on the balance sheet equation is to _____. a. decrease assets and increase liabilities b. increase owners’ equity and decrease liabilities c. increase assets and increase owners’ equity d. increase assets and increase liabilities

4.

The effect of selling an item on credit on the balance sheet is to _____. a. decrease assets and increase liabilities b. increase owner’s equity and decrease liabilities c. increase assets and increase liabilities d. increase assets and increase owner’s equity

Learning Objective 3 5.

When revenues are recognized as earned and expenses are recognized as incurred, a company is using _____. a. the cash basis of accounting b. the accrual basis of accounting c. both of these d. neither of these

6.

Which basis of accounting violates the matching principle? a. cash basis of accounting b. accrual basis of accounting c. both A and B

Copyright ©2022 Pearson Education, Ltd.

238


d.

neither A nor B

Learning Objective 4 7.

What is considered to be a bundle of economic services awaiting future use? a. liabilities b. owners’ equity c. assets d. expenses e. revenue

8.

After the bundle of economic services is used, where does the expired cost appear? a. income statement b. statement of retained earnings c. balance sheet d. all the above correct

Learning Objective 5 9.

_____ are distributions of assets that reduce ownership claims. a. assets b. expenses c. dividends d. retained earnings

10.

Retained income is represented in the company’s balance sheet by _____. a. the balance in the inventory account b. the balance in the plant and equipment account c. all of the assets less the liabilities and paid-in capital d. the balance in the cash account

Learning Objective 6 11.

Cost of goods sold appears on which financial statement? a. balance sheet b. income statement c. retained earnings d. all of the above

12.

Unearned revenue appears on which financial statement? a. balance sheet b. income statement c. statement of retained earnings d. none of the above

Learning Objective 7

Copyright ©2022 Pearson Education, Ltd.

239


13.

Owners’ equity in sole proprietorships and partnerships differs from that for corporations in that _____. a. they do not separate retained income from the owner’s paid-in capital accounts b. they separate retained income from paid-in capital whereas corporations have only one ownership equity account c. they reflect the sum of assets and liabilities rather than the difference d. none of these

14.

Which entity may have two or more owners? a. partnership b. sole proprietorship c. both A and B d. neither A nor B

Learning Objective 9 15.

The practice of accounting for revenue when goods or services are delivered to customers is referred to as _____. a. revenue recognition b. the conservatism convention c. cost recovery d. the stable monetary unit assumption

16.

The practice of relating of accomplishments or revenues and efforts or expenses to a particular period for which the measurement of income is desired is called _____. a. the going concern assumption b. the conservatism convention c. revenue recognition d. the matching principle

17.

The practice of carrying forward assets such as inventories, prepayments, and equipment to be recovered in cash inflows (or reduced cash outflows) in future periods is known as _____. a. the conservatism convention b. the matching principle c. cost recovery d. revenue recognition

18.

The principle that uses the dollar as an unchanging yardstick for measuring transactions is referred to as _____. a. the conservatism convention b. the stable monetary unit assumption c. the going concern assumption d. revenue recognition

Copyright ©2022 Pearson Education, Ltd.

240


CHAPTER 15:

Solutions to Quiz/Demonstration Exercises

1. [c] 2. [b] 3. [d] 4. [c] 5. [b] 6. [a] 7. [c] 8. [d] 9. [c] 10. [c] 11. [b] 12. [a] 13. [a] 14. [a] 15. [c] 16. [d] 17. [c] 18. [b]

Copyright ©2022 Pearson Education, Ltd.

241


Chapter 16 Understanding Corporate Annual Reports: Basic Financial Statements LEARNING OBJECTIVES: When your students have finished studying this chapter, they should be able to: 1. 2. 3. 4. 5. 6. 7. 8. 9. 10.

Recognize and define the main types of assets in the balance sheet of a corporation. Recognize and define the main types of liabilities in the balance sheet of a corporation. Recognize and define the main elements of the stockholders’ equity section of the balance sheet of a corporation. Recognize and define the principle elements in the income statement of a corporation. Recognize and define the elements that cause changes in stockholders’ equity accounts. Explain the purpose of the cash flow statement and identify activities that affect cash, and classify them as operating, investing, or financing activities. Assess financing and investing activities using the statement of cash flows. Use both the direct method and the indirect method to explain cash flows from operating activities. Explain the role of depreciation in the statement of cash flows. Describe and assess the effects of the four major methods of accounting for inventories (Appendix 16A).

Copyright ©2023 Pearson Education, Ltd.

245


CHAPTER 16:

ASSIGNMENTS

CRITICAL THINKING EXERCISES 37. Production Facilities and Depreciation 38. R&D and the Recognition of Intangible Assets 39. Using the Income Statement to Evaluate Sales Success 40. Capital Investment and the Statement of Cash Flows 41. Purchasing Operations and LIFO versus FIFO GENERAL EXERCISES and PROBLEMS 42. Meaning of Book Value 43. Balance Sheet and Income Statement 44. Simple Changes in Retained Earnings and Total Stockholders’ Equity 45. Cash Received from Customers 46. Cash Paid to Suppliers 47. Cash Paid to Employees 48. Simple Cash Flows from Operating Activities 49. Net Income and Cash Flow 50. Net Loss and Cash Flows from Operating Activities 51. Preparation of a Statement of Cash Flows 52. Reconciliation of Net Income and Net Cash Provided by Operating Activities 53. Depreciation and Cash Flows 54. Cash Flows, Indirect Method 55. Preparation of Statement of Cash Flows 56. LIFO and FIFO 57. Lower of Cost or Market—U.S. GAAP 58. Lower of Cost or Market—IFRS 59. LIFO, FIFO, and Cash Effects 60. LIFO, FIFO, Prices Rising and Falling UNDERSTANDING PUBLISHED FINANCIAL REPORTS 61. Various Intangible Assets 62. Various Liabilities 63. Exercises in Assets, Liabilities, and Stockholders’ Equity 64. Classified Income Statement and Balance Sheet 65. Classified Balance Sheet—Britain 66. Gain on a Ship That Sank 67. Identification of Operating, Investing, and Financing Activities 68. Interest Expense 69. Indirect and Direct Cash Flows from Operations 70. Statement of Cash Flows, Direct and Indirect Methods 71. Comparison of Inventory Methods 72. Effects of Late Purchases 73. LIFO and FIFO at Big Bazaar 74. Effect of LIFO

Copyright ©2023 Pearson Education, Ltd.

246


75.

Nike 10-K Problem: Using Financial Statements

EXCEL APPLICATION EXERCISE 76. Analyzing Differences Between Inventory Valuation Methods COLLABORATIVE LEARNING EXERCISE 77. Income Statement and Balance Sheet Accounts INTERNET EXERCISE 78. Tata Steel Limited’s Financial Statements (https://tatasteel.com)

Copyright ©2023 Pearson Education, Ltd.

247


CHAPTER 16: I.

OUTLINE

Classified Balance Sheet

{L. O. 1}

See EXHIBIT 16-1 for the classified balance sheets of Nike, Inc. Assets and liabilities are classified into five main sections: current assets, noncurrent assets, current liabilities, noncurrent liabilities, and shareholders’ equity. These classifications of balance sheet items are described below. A.

Current Assets Current assets include cash and all other assets that are reasonably expected to be converted to cash, sold, or consumed during a normal operating cycle. Operating Cycle—the time span during which cash is spent to acquire goods and services that are used to produce the organization’s output, and then sold to customers, and finally the customers pay for their purchases with cash. As shown in EXHIBIT 16-1, current assets fall into several broad categories (e.g., cash and cash equivalents, accounts receivable, inventories, prepaid expenses, and other assets). Cash consists of bank deposits in checking accounts plus money on hand. Cash Equivalents—short-term investments that can easily be converted into cash with little delay (e.g., money market funds and treasury bills are good examples of cash equivalents). These securities are usually shown at cost or market price, whichever is lower. Accounts receivable is the total amount owed to the company by its customers. Because some customers ultimately will not pay their bill, an allowance or provision for doubtful accounts reduces the total. Inventories consist of merchandise, finished products of manufacturers, goods in process of being manufactured, and raw materials. Inventories are stated at their cost or market price, whichever is lower. Cost of manufactured products normally is composed of raw materials plus the cost of converting it into a finished product (i.e., direct labor and manufacturing overhead). Prepaid expenses are short-term prepayments or advance payments to suppliers (e.g., prepayment of rent and insurance premiums for coverage over the coming operating cycle). They are usually unimportant in relation to other assets. Other current assets are miscellaneous current assets that do not fit into the listed categories (e.g., notes receivable and short-term investments that are not cash equivalents).

B.

Noncurrent Assets: Property, Plant, and Equipment

Copyright ©2023 Pearson Education, Ltd.

248


Property, plant, and equipment are sometimes called Fixed Assets or plant assets. Tangible Assets—physical items that a person can see and touch. Details about these items are frequently found in the footnotes to the financial statements (see EXHIBIT 16-2). Land usually is accounted for as a separate item and is carried at its historical cost. Buildings and machinery and equipment are initially recorded at cost: the invoice amount + freight and installation - cash discounts. The major difficulties of measurement center on the choice of depreciation method (i.e., the allocation of original cost over the periods or products that benefit from the use of the assets). The amount to be depreciated is the difference between the original cost and the asset’s estimated residual value (i.e., the residual value is the amount expected to be received when selling the asset at the end of its economic life). There are three general methods of depreciation typically used: straight-line (see EXHIBIT 16-3), accelerated, and units of production. Leasehold improvements are investments made by a lessee (tenant) in items (e.g., painting, decorating, fixtures, and air-conditioning equipment) that cannot be moved from the premises when the lease expires. The cost of leasehold improvements is written off in the same manner as depreciation; however, it is called amortization. Construction in process is shown separately from other assets because the assets are not yet ready for use. It represents assets that will be part of buildings and equipment when completed. Natural resources (e.g., mineral deposits) are typically grouped with plant assets. Their original cost is written off in the form of depletion as the resources are used. Long-term investments are also noncurrent assets (e.g., long-term holdings of securities of other firms). Accounting for intercorporate investments is discussed in detail in CHAPTER 17. C.

Operating Lease Right-of-Use Assets

D.

Intangible Assets

Intangible Assets—a class of long-lived assets that are not physical in nature. They are rights to expected future benefits deriving from their acquisition and continued possession (e.g., goodwill, franchises, patents, and copyrights). Goodwill—the excess of the cost of an acquired company over the sum of the market values of its identifiable individual assets less the liabilities. Only the goodwill arising from an actual acquisition should be shown as an asset on the purchaser’s records. Goodwill must be amortized over its expected useful life or 40 years, whichever is less, in the United States. The shortest amortization period is not specified, but a lump-sum write-off on acquisition is forbidden for U.S. firms.

Copyright ©2023 Pearson Education, Ltd.

249


D.

Liabilities 1.

{L. O. 2}

Current Liabilities Current Liabilities—an organization’s debts that fall due within the coming year or within the normal operating cycle if longer than a year. Notes payable are short-term debt backed by formal promissory notes held by a bank or business creditors. Accounts payable are amounts owed to suppliers who extend credit for purchases on open account. Accrued liabilities or accrued expenses payable are recognized for wages, salaries, interest, and similar items. The accountant recognizes expenses as they are incurred, regardless of when they are paid for in cash. Income taxes payable is a special accrued expense of enough magnitude to warrant a separate classification. The current portion of long-term debt shows the payments due within the next year on bonds and other long-term debt. Unearned or deferred revenues are recorded when cash is received before the goods or a firm delivers services. Working Capital—current assets less current liabilities.

2.

Noncurrent Liabilities Noncurrent or Long-Term Liabilities—an organization’s debts that fall due beyond 1 year (e.g., deferred income taxes and long-term debt). See EXHIBIT 16-4 for Long-Term Debt Footnote. Long-term debt may be secured or unsecured. Secured debt provides debt holders with first claim on specified assets (e.g., mortgage bonds). If the company is unable to meet its regular obligations on the bonds, the specified assets may be sold and the proceeds used to pay off the firm’s obligations to its bondholders. Secured debt holders have first claim. Unsecured debt consists of Debentures—formal certificates of indebtedness that are accompanied by a promise to pay interest at a specified rate (e.g., bonds, notes, or loans). Unsecured debt holders are general creditors who have a general claim against total assets rather than a specific claim against particular assets. Holders of Subordinated bonds or debentures are junior to the other creditors in exercising claims against assets. Liquidation—converting assets to cash and using the cash to pay off outside claims. To increase the appeal of their bonds, many corporations issue debt that is convertible into common stock.

E.

Stockholders’ Equity

{L. O. 3}

The final element of the balance sheet is stockholders’ equity or owners’ equity or capital or net worth, the total residual interest in the business. The main elements of

Copyright ©2023 Pearson Education, Ltd.

250


stockholders’ equity arise from two main sources: (1) contributed or paid-in capital, and (2) retained income. Paid-in capital typically comes from owners who invest in the business in exchange for stock certificates, which are issued as evidence of stockholder rights. Capital stock can be divided into two major classes: common stock and preferred stock. Common Stock—all companies have common stock. Common stock has no predetermined rate of dividends, and is the last to obtain a share in the assets when the corporation is dissolved. Common shares usually have voting power in the management of the corporation. It is the riskiest class of ownership because it pays no guaranteed return, but may provide the best overall return because there is no limit to the stockholder’s potential participation in earnings. Preferred Stock—issued by about 40% of major U.S. firms. It typically has some priority over other shares regarding dividends or the distribution of assets on liquidation. Preferred shareholders typically do not have voting privileges. Par Value (or Legal Value, Stated Value)—value that is printed on the face of the stock certificate. It is generally illegal for a corporation to sell an original issue of its common stock below par. Shareholders have limited liability. Limited Liability— creditors cannot seek payment from shareholders as individuals if the corporation itself cannot pay its debts. Capital in excess of stated value is the excess received over the stated, par, or legal value of the shares issued. Common shares are usually issued at a price above par. Retained earnings (or retained income) are the increase in stockholders’ equity caused by profitable operations. A foreign currency translation adjustment appears on the balance sheet for companies with foreign operations. It arises from changes in the exchange rate between the dollar and foreign currencies. Treasury Stock—a corporation’s own stock that has been issued and subsequently repurchased by the company, and is being held for a specific purpose. Repurchase is a decrease in ownership claims. It should appear on the balance sheet as a deduction from stockholders’ equity. II.

Income Statement A.

{L. O. 4}

Operating Performance An income statement can take one of two major forms: single step or multiple step. A single-step statement merely lists all expenses without drawing subtotals, whereas a multiple-step statement contains one or more subtotals. Subtotals highlight significant relationships such as when cost of goods sold is deducted from revenues to give gross

Copyright ©2023 Pearson Education, Ltd.

251


margin. The two most common subtotals: gross profit and income from operations (also called operating income). Depreciation expense, selling expenses, and administrative expenses are often grouped as “operating expenses” and deducted from gross profit to obtain operating income, which is also called operating profit. B.

Financial Management Operating management focuses on the major day-to-day activities that generate sales revenue. Financial management focuses on where to get cash and how to use it. Because financial rather than operating decisions affect interest income and expense, they often appear as separate items after operating income. The terms income, earnings, and profits are often used as synonyms. Net Income—the popular “bottom line” (i.e., the residual after deducting all expenses including income taxes. The term net is seldom used for any subtotals that precede the calculation of net income. Net income typically appears after provision for income taxes. Income statements conclude with disclosure of Earnings Per Share (EPS). EPS—net income divided by the average number of common shares outstanding during the year (see EXHIBIT 16-5).

III.

Statement of Changes in Stockholder’s Equity

{L. O. 5}

Statement of Changes in Stockholder’s Equity (or Statement of Retained Earnings)—a financial statement that analyzes changes in the retained earnings over a period of time. Net income increases retained earnings, whereas losses and dividends reduce retained earnings (see EXHIBIT 16-6). IV.

Statement of Cash Flows The statement has the following purposes: 1.

It shows the relationship of net income to changes in cash balances. Cash balances can decline despite positive net income and vice versa.

2.

It reports past cash flows as an aid to: a. b. c.

Predicting future cash flows Evaluating management’s generation and use of cash Determining a company’s ability to pay interest and dividends, and to pay debts when they are due

3.

It reveals commitments to assets that may restrict or expand future courses of action.

A.

Basic Concepts

Copyright ©2023 Pearson Education, Ltd.

252


Statement of Cash Flows—reports the cash receipts and cash payments of an organization during a particular period. The statement of cash flows along with the income statement and statement of retained earnings show why balance sheet items have changed by providing information about operating, investing, and financing activities. The statement of cash flows usually explains where cash came from during a period and where it was spent. B.

Typical Activities Affecting Cash

{L. O. 6}

The fundamental approach to the statement of cash flows is: (1) list the activities that increased cash and those that decreased cash, and (2) place each cash inflow and outflow into one of three categories according to the type of activity that caused it: operating activities, investing activities, and financing activities. Cash flows from operating activities are generally the effects of transactions that affect the income statement (e.g., collections from customers and cash payments to suppliers). Investing activities include (1) lending and collecting on loans and (2) acquiring and selling long-term assets. Financing activities include obtaining resources from creditors and owners and providing creditors with returns of and owners a return on their investments (i.e., payment of dividends). The FASB has decided to include the receipt and payment of interest and the receipt of dividends as operating activities because they affect the computation of income. C.

Focus of a Statement of Cash Flows

{L. O. 7}

See EXHIBIT 16-7 for Balmer Company’s summary transactions for the year, and EXHIBIT 16-8, EXHIBIT 16-9, and EXHIBIT 16-10 for Balmer Company’s condensed income statement and balance sheets, and statement of cash flows. It shows the changes in cash resulting from operating, investing, and financing activities. The change is then added to the beginning cash balance to yield the ending cash balance. 1.

Cash Flows from Financing Activities This section shows cash flows to and from providers of capital (e.g., increase or decrease in long-term debt, issue stock, repurchase stock, and pay dividends).

2.

Cash Flows from Investing Activities This section shows cash flows from the purchase or sale of property, plant, equipment, and other long-lived assets.

3.

Noncash Investing and Financing Activities For example, purchased fixed assets by issuing common stock. Cash did not change hands.

Copyright ©2023 Pearson Education, Ltd.

253


V.

Cash Flow from Operating Activities Many accountants build the statement of cash flows from the changes in balance sheet items, a few additional facts, and their familiarity with the typical causes of changes in cash. For instance, cash collected from sales is usually found by taking sales computed under the accrual basis from the income statement and adjusting it for the changes in the accounts receivable balance. Examples of other operating activity effects on cash are also provided: the purchase of inventory, payments of accounts payable, payments to suppliers, payments to employees, and payments for interest and income taxes. See EXHIBIT 16-11 for the effects of transactions on cash flows from operating activities.

VI.

Interpreting the Cash Flow A.

{L. O. 8}

DIRECT METHOD The direct method consists of listing cash receipts (inflows) and cash disbursements (outflows). The easiest way to construct the statement of cash flows from operations using the direct method is to examine the Cash column of the balance sheet equation (see EXHIBIT 16-7).

B.

INDIRECT METHOD Indirect Method—reconciles net income to the net cash provided by operating activities. It also shows the link between the income statement and the statement of cash flows. The reconciliation begins with net income. Then additions and deductions are made for items that affect net income and net cash flow differently (see EXHIBIT 16-12). The most common additions or deductions from net income on the reconciliation are: • • • • •

Add decreases (or deduct increases) in accounts receivable Add decreases (or deduct increases) in inventories Add decreases (or deduct increases) in accounts payable Add decreases (or deduct increases) in wages and salaries payable Add decreases (or deduct increases) in unearned revenue

The general rules for reconciling these items are: 1. 2. 3. 4.

Deduct increases in noncash current assets Add decreases in noncash current assets Add increases in current liabilities Deduct decreases in current liabilities

A final step is to reconcile for amounts that are included in net income but represent investing or financing activities (in contrast to operating activities).

Copyright ©2023 Pearson Education, Ltd.

254


Examples include: • • C.

Add loss (or deduct gain) from sale of fixed assets Add loss (or deduct gain) on extinguishment of debt

Reconciliation Schedule under Direct and Indirect Methods The FASB requires all statements of cash flows to use either the direct or the indirect method. In addition, a reconciliation schedule must be included showing the reconciliation of cash flows and net income (see EXHIBIT 16-13).

D.

Role of Depreciation

{L. O. 9}

Depreciation does not entail a current outlay of cash and is added back to net income in the indirect method in order to obtain cash provided by operations. However, it should not mistakenly be thought of as a source of cash. E.

Statement of Cash Flows for Nike, Inc. See EXHIBIT 16-14 for Nike, Inc.’s statement of cash flows. It uses the indirect method to report the cash flows from operating activities. The effects of deferred taxes, the issuance of shares in conjunction with an executive stock option compensation plan, and changes in the exchange rates on foreign currencies on cash flows are provided on this statement along with the other items mentioned earlier in the chapter.

VII.

Appendix 16A: Accounting for Inventory—Four Major Inventory Methods 10}

{L. O.

A company’s inventory method affects its income statement, as well as its balance sheet. Each period, accountants must divide the costs of merchandise acquired between cost of goods sold, an expense, and cost of items remaining in ending inventory, an asset. Various inventory methods accomplish this division. If unit prices and costs did not fluctuate, all inventory methods would show identical results (see EXHIBITS 16-18, 16-19, and 16-20). A.

Specific Identification Specific Identification recognizes the actual cost paid for the specific physical item sold. Gross profit depends on which can the company sells. Physical observation and/or the labeling of items in stock with codes are used.

B.

First-In, First-Out (FIFO) Method FIFO assumes that a company sells or uses up first the stock acquired earliest. Assuming inflation, FIFO tends to provide inventory valuations that closely

Copyright ©2023 Pearson Education, Ltd.

255


approximate the actual market value of the inventory at the balance sheet date. Additionally, FIFO leads to a higher gross profit because COGS consists of older, lower costs. C.

Last-In, First-Out (LIFO) Method LIFO assumes that a company sells or uses up first the stock acquired most recently, and treats the most recent costs as COGS. LIFO leads to lower ending inventory valuation, and lower gross profit (which helps lower its tax liability). Income can dramatically increase when a company reduces its inventories. Under LIFO, inventory consists of LIFO layers (or LIFO increments), which are separately identifiable additions to inventory. All units in a particular LIFO layer have the same cost. As a company grows, the LIFO layers tend to pile on one another. Therefore, many LIFO companies show inventories that have very old layers.

D.

Weighted-Average Cost The weighted-average cost method assigns the same unit cost to each unit available for sale. The unit cost is the cost of all units available for sale divided by the number of units available. This method produces a gross profit between FIFO and LIFO.

E.

Inventory Methods and Physical Flow of Inventories Some companies choose the inventory method based upon the physical flow of units. However, it is not required to use the physical flow. Companies can use any one of the four inventory methods, although companies reporting under IFRS cannot use LIFO.

F.

Lower-of-Cost-or-Market (LCM) Method Regardless of the inventory method used, accountants MUST decrease the inventory value if the inventory’s market price (current replacement cost or its equivalent under GAAP and net realizable value under IFRS) drops below its acquisition cost. Companies CANNOT increase inventory values when prices rise (i.e., the conservatism principle).

VIII.

Appendix 16B: Shareholder Reporting, Income Tax Reporting, and Deferred Taxes There are major differences in how to account for transactions between U. S. GAAP and U. S. tax laws (e.g., amortization and depreciation). When revenues and expenses on the income statement differ from the statement to the tax authorities, deferred taxes can arise. Most often, deferred taxes arise when tax expenses exceed book expenses (see EXHIBIT 16-21).

Copyright ©2023 Pearson Education, Ltd.

256


CHAPTER 16:

Quiz/Demonstration Exercises

Learning Objective 1 1.

Cash, accounts receivable, inventories, and prepaid expenses _____. a. b. c. d.

2.

are liabilities on the balance sheet are current assets on the balance sheet are classifications of stockholders’ equity would be considered long-term liabilities

Goodwill, franchises, patents, trademarks, and copyrights are examples of _____. a. b. c. d.

current assets current liabilities intangible assets long-term liabilities

Learning Objective 2 3.

Accounts payable, accrued liabilities, and income taxes payable are examples of _____. a. b. c. d.

4.

current liabilities fixed assets stockholders’ equity long-term liabilities

Bonds that are convertible may be converted into _____. a. b. c. d.

debt fixed assets other bonds stock

Learning Objective 3 5.

Accounts in the stockholders’ equity section of the balance sheet may include _____. a. b. c. d.

6.

cash, accounts receivable, inventories, prepaid expenses, and other current assets accounts payable, accrued liabilities, and income taxes payable machinery, equipment, buildings, land, and leasehold improvements common stock, preferred stock, paid-in-capital in excess of par; treasury stock; and retained earnings

Treasury stock is not used for which of the following reasons?

Copyright ©2023 Pearson Education, Ltd.

256


a. b. c. d.

employee stock purchase plan to reduce the stock’s market value executive’s bonus to acquire another company

Learning Objective 4 7.

The item on the income statement found by deducting cost of sales and other operating expenses from sales is _____. a. b. c. d.

8.

income from operations gross profit net income earnings per share

The item, required at the very bottom of the income statement, is _____. a. b. c. d.

gross profit income from operations earnings per share net income

Learning Objective 5 9.

What reduces retained earnings? a. b. c. d.

10.

net loss net income capital contributions trading one asset for another

What increases retained earnings? a. b. c. d.

net loss net income capital contributions dividend payouts

Learning Objective 6 11.

In a statement of cash flows, the payment to suppliers would be classified as a(n) _____ activity. a. b. c.

investing financing liquid

Copyright ©2023 Pearson Education, Ltd.

257


d. 12.

The purchase of property, plant, and equipment would appear as a(n) _____ activity on a statement of cash flows. a. b. c. d.

13.

operating

financing investing operating liquid

Issuing equity securities would appear as a(n) _____ activity on a statement of cash flows. a. b. c. d.

financing investing operating liquid

Learning Objective 7 14.

Cash collections from customers are computed by _____. a. b. c. d.

15.

sales revenue + increase in accounts receivable sales revenue - increase in accounts receivable sales revenue – decrease in accounts receivable sales revenue + decrease in accounts payable

Charizard Inc.’s cost of goods sold was $72,000. During the year, inventory increased by $7,000 and accounts payable decreased by $11,000. How much was paid to suppliers during the year? a. b. c. d.

$ 90,000 $ 11,000 $107,000 $ 68,000

Learning Objective 8 16.

Under the indirect method, losses on sales of noncurrent assets are _____. a. b. c. d.

17.

deducted from net income to compute cash provided by operating activities added to net income to compute cash provided by operating activities shown under financing activities as a deduction shown under financing activities as an addition

Which of the following items would be added to net income to compute cash provided by (used in) operations under the indirect method?

Copyright ©2023 Pearson Education, Ltd.

258


a. b. c. d.

increase in merchandise inventory decrease in accounts payable increase in accrued liabilities increase in cash dividends payable

Learning Objective 9 18.

Depreciation is added back to net income under the indirect method of computing the changes in cash flows because _____. a. b. c. d.

19.

it is an expense that was deducted in computing net income that did not require a current outlay of cash it represents a source of cash to the firm the useful life and residual value are estimated none of the above

Which of the following is treated like depreciation under the indirect method of computing the changes in cash flows? a. b. c. d.

depletion amortization neither A nor B both A and B

Copyright ©2023 Pearson Education, Ltd.

259


CHAPTER 16:

Solutions to Quiz/Demonstration Exercises

1. [b] 2. [c] 3. [a] 4. [d] 5. [d] 6. [b] 7. [a] 8. [c] 9. [a] 10. [b] 11. [d] 12. [b] 13. [a] 14. [b] 15. [a] 16. [b] 17. [c] 18. [a] 19. {d]

Copyright ©2023 Pearson Education, Ltd.

260


Chapter 17 Understanding and Analyzing Consolidated Financial Statements LEARNING OBJECTIVES: When your students have finished studying this chapter, they should be able to: 1. 2. 3. 4. 5. 6.

Contrast accounting for investments using the equity method and the market–value method. Explain the basic ideas and methods used to prepare consolidated financial statements. Describe how goodwill arises and how to account for it. Use financial statements analysis to evaluate an organization’s performance Explain and use a variety of popular financial ratios. Identify the major implications that efficient stock markets have for accounting.

Copyright ©2023 Pearson Education, Ltd.

271


CHAPTER 17:

ASSIGNMENTS

CRITICAL THINKING EXERCISES 25. Market Method, Equity Method, and Total Assets 26. Depreciation in Consolidated Financial Statements 27. Just-in-Time (JIT) Inventory and Current Ratio 28. Market Efficiency GENERAL EXERCISES and PROBLEMS 29. Equity Method 30 Consolidated Financial Statements 31. Determination of Goodwill 32. Purchased Goodwill 33. Amortization and Depreciation 34. Allocating Total Purchase Price to Assets 35. Preparation of Consolidated Financial Statements 36. Intercorporate Investments and Ethics 37. Profitability Ratios 38. Financial Ratio UNDERSTANDING PUBLISHED FINANCIAL REPORTS 39. Meaning of Account Descriptions 40. Classification on Balance Sheet 41. Effects of Transactions Under the Equity Method 42. Noncontrolling Interests 43. General Electric and GECS 44. Goodwill 45. Accounting for Goodwill 46. Income Ratios and Asset Turnover 47. Financial Ratios 48. Nike 10-K Problem: Using Consolidated Financial Statements EXCEL APPLICATION EXERCISE 49. Calculating Financial Ratios COLLABORATIVE LEARNING EXERCISE 50. Financial Ratios INTERNET EXERCISE 51. General Electric’s Annual Report (http://www.ge.com)

Copyright ©2023 Pearson Education, Ltd.

272


CHAPTER 17:

OUTLINE

Part One: Intercorporate Investments Including Consolidations Firms often invest in the equity securities of other companies. The investor may be simply investing excess cash, or he may be seeking some degree of control over the investee. There are three methods of accounting for intercorporate investments: the equity and market methods and consolidation. An investor that holds less than 20% of another company is assumed to be a passive investor—it cannot significantly influence the decisions of the investee—and it uses the market method. Investors with between 20% and 50% interest use the equity method. At this level of ownership, the investor has the ability to exert significant influence on the investee. Firms with an interest in excess of 50% must use the consolidation approach. I.

II.

Market Value and Equity Methods

{L. O. 1}

A.

Market-Value Method—records the initial investment on the balance sheet at fair market value (FMV). Such investments are often called marketable securities in the financial statements. Trading Securities—investments that the company buys only with the intent to resell them shortly. Available-forSale Securities—investments that the company does not intend to sell in the near future. Changes in market value of trading securities are reported as gains (increase in FMV) or losses (decrease in FMV), whereas available-for-sale securities have their unrealized gains or losses shown in a separate valuation allowance account in the stockholders’ equity section of the balance sheet. (See EXHIBIT 17-1)

B.

Equity Method—accounts for the investment at the acquisition cost adjusted for the investor’s share of dividends and earnings or losses of the investee after the date of investment. Investors increase the carrying amount of the investment by their share of investee’s earnings and reduce the carrying amount by dividends received from the investee and by their share in investee’s losses.

Consolidated Financial Statements

{L. O. 2}

Parent Company—the company owning more than 50% of the other business’s stock. Subsidiary—the company whose stock is owned by the other business. Although parent and subsidiary companies typically are separate legal entities, in many regards they function as one unit. Consolidated Financial Statements—financial statements that combine the financial statements of the parent company with those of various subsidiaries. A.

The Acquisition

Copyright ©2023 Pearson Education, Ltd.

273


When a parent acquires a subsidiary, the evidence of interest is recorded as Investment in Subsidiary. When the consolidated statements are prepared, they cannot show both the evidence of interest and the underlying assets and liabilities of the subsidiary. To avoid such double-counting, the reciprocal evidence of ownership present is eliminated in two places: (1) the Investment in Subsidiary on the parent company’s books and (2) the Stockholders’ Equity on the subsidiary company’s books. The entries necessary to accomplish this are called eliminating entries. III.

Recognizing Income After Acquisition Long-term investments in equity securities (e.g., the investments in a subsidiary) are carried in the investor’s balance sheet by the equity method. The income generated by a subsidiary is recognized by the parent company as an increase in an account titled Investment in Subsidiary. A.

Noncontrolling Interests When a parent holds less than 100% of the stock of a subsidiary, a consolidated balance sheet includes an account on the equities side called Noncontrolling Interests in Subsidiaries, or simply Noncontrolling Interests—the account that shows the outside stockholders’ interest, as opposed to the parent’s interest, in a subsidiary corporation.

B.

Perspective on Consolidated Statements See EXHIBIT 17-2 and EXHIBIT 17-3 for an illustration of how investments in subsidiaries are presented in companies’ annual reports. The headings of the statements indicate that they are consolidated statements. On balance sheets, the minority interest typically appears just above the stockholders’ equity section. On income statements, the minority interest in the net income is deducted as if it were an expense of the consolidated entity after all the other expenses are listed. Investments in Affiliates (or Investments in Associates)—listed as an asset on the balance sheet and reflect the purchase cost and interests in income or loss of investees. The FASB requires all subsidiaries to be consolidated. The major reason for forcing consolidation is to provide a more complete picture of the economic entity. See EXHIBIT 17-4 for a summary of the accounting for different levels of investment in subsidiaries.

C.

Accounting for Goodwill

{L. O. 3}

In CHAPTER 16, goodwill is defined as the excess of cost over fair value of net identifiable assets of businesses acquired. The purchase price of a subsidiary often exceeds its book value. In fact, it frequently exceeds the sum of the fair market values of the identifiable individual assets less the liabilities. When the amount paid exceeds the book values, the assets will be valued at their fair market values for the consolidated statements. Any amounts paid above the fair market values of the individual assets are carried as goodwill in the consolidated financial statements.

Copyright ©2023 Pearson Education, Ltd.

274


A purchaser may be willing to pay more than the current values of the individual assets received because the acquired company is able to generate abnormally high earnings. The causes of this excess earnings power may be traced to personalities, skills, locations, operating methods, and so forth. “Goodwill” is originally generated internally. For example, a happy combination of advertising, research, management talent, and timing may give a particular company a dominant market position for which another company is willing to pay dearly. This ability to command a premium price for the total business is goodwill. The selling company will never record goodwill. Therefore, the only goodwill recognized as an asset is that identified when one company is purchased by another. Part Two: Analysis of Financial Statements {L. O. 4} Careful analysis of financial statements can help decision makers evaluate an organization’s past performance and predict its future performance. Financial statements of Microsoft Corporation in EXHIBIT 17-5 and EXHIBIT 17-6 are used to focus on financial statement analysis. Investors analyze financial statements in order to decide whether to buy, sell, or hold common stock. Managers and the financial community (e.g., bank officers and stockholders) use them as clues to help evaluate the operating and financial outlook for an organization. Budgets or pro forma statements, carefully formulated expressions of predicted results including a schedule of the amounts and timings of cash repayments, are helpful to creditors. They want assurances of being paid in full and on time. IV.

Component Percentages Component Percentages—analysis and presentation of financial statements in percentage form to aid comparability, and is frequently used when comparing companies that differ in size (see EXHIBIT 17-7). The resulting statements are called Common-Size Statements. Income statement percentages are usually based on sales = 100%. Comparing the gross margin rate or the net income percentage with those of other firms in the industry or with prior years may be useful. Better yet, a comparison of these percentages (along with those for other items on the income statement) with what was budgeted for the current year may help in diagnosing what changes created better or worse results. Balance sheet percentages are usually based on total assets = 100%. One can see the shifts in the composition of assets between current and long term. In addition, one can see shifts in the equities side of the balance sheet between current liabilities, noncurrent liabilities, and stockholders’ equity.

V.

Use of Ratios

{L. O. 5}

Copyright ©2023 Pearson Education, Ltd.

275


See EXHIBIT 17-8 for how typical ratios are computed from financial statements. Many more ratios could be computed. For example, Standard & Poor’s Corporation sells a COMPUSTAT service. Via computer, COMPUSTAT can provide financial and statistical information for thousands of companies. The information includes 175 financial statement items on an annual basis and 100 items on a quarterly basis, plus limited footnote information. The SEC makes annual financial statements available online in its Edgar database (http://www.sec.gov/edgar.shtml). The ratios shown in EXHIBIT 17-8 are as follows: TYPICAL NAME OF RATIO

NUMERATOR

DENOMINATOR

Short-Term Ratios: Current ratio

Current assets

Current liabilities

Avg. collection period in days

Avg. A/R x 365

Sales on account

Debt-to-Equity Ratios: Current debt to equity Current liabilities

Stockholders’ equity

Total debt to equity

Total liabilities

Stockholders’ equity

Gross profit rate or percentage

Gross profit or gross margin

Sales

Return on sales

Net income

Sales

Return on stockholders’ equity

Net income

Average stockholders’ equity

Earnings per share

Net income less dividends on P/S

Avg. common shares outstanding

Profitability Ratios:

Price earnings

Market price per Earnings per share share of common stock

Dividend Ratios: Dividend yield

Dividend payout

Dividends per common share

Market price per common share

Dividends per

Copyright ©2023 Pearson Education, Ltd.

Earnings per share

276


common share

A.

Comparisons

{L. O. 5}

Evaluation of a financial ratio requires a comparison. There are three main types of comparisons: (1) Time Series Comparisons—with a company’s own historical ratios (e.g., for 5 to 10 years); (2) Benchmark Comparisons—general rules of thumb (e.g., there is trouble if a company’s current debt is at least 80% of its tangible net worth); and (3) Cross-Sectional Comparisons—ratios of other companies or with industry averages from Dun and Bradstreet. Comparisons for the Microsoft Company data across years, against benchmarks, and to the industry are presented. B.

Discussion of Specific Ratios The current ratio is a widely used statistic. Other things being equal, the higher the current ratio, the more assurance the creditor has about being paid in full and on time. The average collection period in days is another important short-term ratio. An increase in this ratio might indicate increasing acceptance of poor credit risks or less energetic collection efforts. Both creditors and shareholders watch the debt-to-equity ratios to judge the degree of risk of insolvency and stability of profits. Companies with heavy debt in relation to ownership capital are in greater danger of suffering net losses or even bankruptcy when business conditions sour, revenues and many expenses decline, but interest expenses and maturity dates do not change. Investors find profitability ratios especially helpful. The gross profit rate and return on sales are both measures of operating success. Shareholders view the return on their invested capital as more important. The return on equity provides a measure of overall accomplishment. The final four ratios in EXHIBIT 17-8 are based on earnings and dividends. The first, earnings per share of common stock (EPS), is the most popular of all ratios. This is the only ratio that is required as part of the body of the financial statements of publicly held companies in the United States. The EPS must be presented on the face of the income statement. The calculation of EPS can be more complicated than is indicated in EXHIBIT 17-8 depending on the capital structure of the firm and the presence of common-stock equivalents. The price earnings, dividend yield, and dividend payout ratios are especially useful to investors in the common stock of the company.

Copyright ©2023 Pearson Education, Ltd.

277


C.

Operating Performance Ratios Businesspeople often look at invested capital’s rate of return as an important measure of overall accomplishment: rate of return on investment = income / invested capital The measurement of operating performance (i.e., how profitably assets are employed) should not be influenced by the management’s financial decisions (i.e., how assets are obtained). Operating performance is best measured by pretax operating rate of return on average total assets: pretax operating rate = operating income of return on average total assets average total assets The right-hand side of the equation above consists of two important ratios: operating inc. = operating income avg. total assets sales

x

sales avg. tot. assets

The right-hand side terms in the equation above are often called the operating income percentage on sales and total asset turnover (i.e., the two basic factors in profit making). An improvement in either will, by itself, increase the rate of return on total assets. If ratios are used to evaluate operating performance, they should exclude extraordinary items. Such items are not expected to recur, and therefore they should not be included in measures of normal performance. VI.

Efficient Markets and Investor Decisions

{L. O. 6}

Much research in accounting and finance has concentrated on whether the stock markets are “efficient”. Efficient Capital Market—market prices “fully reflect” all information available to the public. Therefore, searching for “underpriced” securities in such a market would be fruitless, unless an investor has information that is not generally available. If the real-world markets are indeed efficient, a relatively inactive portfolio approach would be an appropriate investment strategy for most investors. The hallmarks of the approach are risk control, high diversification, and low turnover of securities. The role of accounting information would mainly be in identifying the different degrees of risk among various stocks so that investors can maintain desired levels of risk and diversification. Many ratios are used simultaneously rather than one at a time for such predictions. Research showed that accounting reports are only one source of information. In the aggregate, companies that choose the least-conservative accounting policies do not fool the market. In sum, the market as a whole generally sees through any attempts by companies to gain favor through the choice of accounting policies that tend to boost immediate income.

Copyright ©2023 Pearson Education, Ltd.

278


Some alternative sources of financial information are the following: company press releases, trade association publications, brokerage house analyses, and government economic reports. If accounting reports are to be useful, they must have some advantage over alternative sources in disclosing new information. Financial statement information may be more directly related to the item of interest, more reliable, lower in cost, and/or more timely than alternative sources.

Copyright ©2023 Pearson Education, Ltd.

279


CHAPTER 17:

Quiz/Demonstration Exercises

Learning Objective 1 1.

The Colts Corporation has acquired a 10% interest in the shares of Giants Corporation. Colts reported income for 20X1 of $25 million and issued dividends of $10 million during the year. Becaise Colts use the available for sale method of accounting for their investment in Giants, for 20X1 the value of their income will _____. a. increase by $25 million, the amount of Giants earnings b. increase by $1 million, the dividends paid by Giants to Colts c. decrease by $10 million, the amount Giants paid out in dividends d. increase by $1.5 million, Colts’ share in the earnings of Giants reduced by their share of the dividends paid out

2.

The Raiders Corporation owns 40% of the outstanding shares of the Warriors Corporation. During 20X1, Warriors reported income of $25 million and paid dividends of $10 million to shareholders. Raiders use the equity method to account for their investment in Warriors. Accordingly, the value of their Warriors investment during 20X1 will increase by _____. a. $14 million, the sum of the dividends received by Raiders and their share in the earnings of Warriors b. $6 million, Raiders’ share in earnings of Warriors reduced by the dividends received c. $4 million, the dividends paid by Warriors to Raiders d. $10 million, Raiders’ share in the earnings of Warriors

Learning Objective 2 3.

When a company purchases more than 50% of the stock of another business, the two companies’ financial statements must be presented _____. a. separately in the same annual report with the nature of the ownership interest fully disclosed b. in two separate sets of financial statements that cannot appear in the same annual report c. in the form of consolidated financial statements after eliminating entries are recorded to avoid the double counting of assets and equity d. together regardless of whether the ownership interest continues

4.

Taylor Company owns 90% of the outstanding shares of Turner Company. If Turner Company reports earnings for the year of $20 million, the consolidated financial statements will show a _____ million increase in the _____. a. $20; consolidated shareholders’ equity b. $2; noncontrolling interest in Turner c. $20; consolidated net assets d. $20; noncontrolling interest in Turner

Learning Objective 3

Copyright ©2023 Pearson Education, Ltd.

280


5.

Goodwill is recognized for accounting purposes _____. a. when a business is purchased for a price that exceeds the fair market value of its assets less liabilities b. every year as long as the IRS does not object c. when the value of a business exceeds its historical cost book value d. when a business is purchased for a price that exceeds the book value of its assets less liabilities

6.

Goodwill may be caused by _____. a. excellent general management skills b. potential efficiency by rearrangement c. dominant market position d. all of the above e. none of the above

Learning Objective 5 Use the comparative balance sheets and income statements below for the Louise Company in answering questions 7 through 10: Louise Company Comparative Balance Sheets December 31, 20X1 and 20X2 20X1

20X2

Assets Cash Accounts Receivable (net) Inventory Property, Plant, and Equipment (net) Total Assets

$ 61,100 $ 27,200 72,500 142,700 122,600 107,800 577,700 507,500 $833,900 $785,200

Liabilities and Stockholders’ Equity Accounts Payable $104,700 $ 72,300 Notes Payable within one year 50,000 50,000 Bonds Payable 200,000 210,000 Common Stock—$10 par value 300,000 300,000 Retained Earnings 179,200 152,900 Total Liabilities and Stockholders Equity $833,900 $785,200 Louise Company Comparative Income Statements For the Years Ended 12/31/X1 and 12/31/X2

Copyright ©2023 Pearson Education, Ltd.

281


Sales Cost of Goods Sold Gross Profit Operating Expenses Selling Expenses Administrative Expenses Interest Expense Income Tax Expense Total Operating Expenses Net Income 7.

20X2

20X1

$ 800,400 454,100 $ 346,300

$ 900,000 396,200 $ 503,800

$ 130,100 $ 104,600 40,300 115,500 25,000 20,000 14,000 35,000 $ 209,400 $ 275,100 $ 136,900 $ 71,300

The Louise Company’s current ratio for 20X2 was _____. a. 1.66 b. 5.39 c. 1.23

d.

1.00

8.

Louise Company’s total debt to equity ratio has _____. a. decreased from 0.74 to 0.73 from 20X1 to 20X2 b. increase from 0.73 to 0.27 from 20X1 to 20X2 c. decrease from 0.32 to 0.74 from 20X1 to 20X2 d. increased from 0.27 to 0.32 from 20X1 to 20X2

9.

Louise Company’s gross profit rate for 20X1 was _____. a. 20.58% b. 55.98% c. 46.65%

d. 61.92%

In 20X2, Louise Company’s return on sales was _____. a. 2.39% b. 4.61% c. 23,27%

d. 17.10%

10.

Learning Objective 6 11.

An efficient capital market _____. a. creates an opportunity for investors to spot “underpriced” securities using publicly available information b. is one in which market prices “fully reflect” all information to the public c. has no bearing on accounting statements and procedures d. explains why some individuals are able to “beat the market” through the use of publicly available information

12.

If markets are truly efficient, then the proper portfolio includes which of the following characteristics? a. high diversification b. risk control c. low turnover of securities d. all of the above e. none of the above

Copyright ©2023 Pearson Education, Ltd.

282


CHAPTER 17: 1. [b] 2. [b] 3. [c] 4. [b] 5. [a]

6. [d] 7. [a]

8. [b]

9. [b] 10. [d]

Solutions to Quiz/Demonstration Exercises

The amount of dividends received will increase the dividend income account. With the equity method, the investor recognizes his share in earnings, but reduces the investment by the amount of dividends received. Consolidated financial statements must be issued when ownership exceeds 50%. The noncontrolling interest will be increased by its share in the earnings of the subsidiary. Goodwill is recorded for the amount by which the purchase cost exceeds the fair market value of the net assets obtained. First, the assets are written up to their fair market values. Then any excess is recorded as goodwill, which is amortized over a period not exceeding 40 years. The current ratio is found by dividing the current assets by the current liabilities. Here current assets are $256,200 [$61,100 + $72,500 + $122,600] and current liabilities are $154,700 [$104,700 + $50,000]. Therefore, the current ratio is 1.66 [$256,200/$154,700]. In 20X1 the total debt-to-equity ratio was 0.73, which was computed as [($72,300 + $50,000 + $210,000) / ($300,000 + $152,900)]. In 20X2, the total debt-to-equity ratio has increased to 0.74 [($104,700 + $50,000 + $200,000) / ($300,000 + $179,200)]. The gross profit rate is found by dividing the gross profit by sales. For 20X1, that would be $503,800 / $900,000 = 55.98%. The return on sales is found by dividing the net income by sales. For 20X2, that was $136,900 / $800,400 = 17.10%.

11. [b] 12. [d]

Copyright ©2023 Pearson Education, Ltd.

283


Turn static files into dynamic content formats.

Create a flipbook
Issuu converts static files into: digital portfolios, online yearbooks, online catalogs, digital photo albums and more. Sign up and create your flipbook.